Download as pdf or txt
Download as pdf or txt
You are on page 1of 325

SOLVING MATHEMATICAL

PROBLEMS

1000 Solved Problems for Master’s Entrances

By Tanujit Chakraborty

1
Solving Mathematical Problems

This page is intentionally left blank.

2
Solving Mathematical Problems

Preface

“This book is useful for all the Masters Entrances conducted by Indian Statistical Institute. Readers
are also suggested to go through the websites of Ctanujit Classes (www.ctanujit.in and
www.ctanujit.org) for other online resources like topic wise lecture notes and previous years
solved papers. Since 2013, I am involved with teaching for different entrance exams at ISI. I tried
to cover several previous year’s papers of Mathematics and Statistics portions asked in ISI
MSTAT, MMATH, MSQE, MSQMS, MTECH & PGDBA Entrance exams. If you have any
queries you can mail me at tanujitisi@gmail.com .”

With Best wishes,

Tanujit Chakraborty

Founder of Ctanujit Classes.

Research Scholar,

Indian Statistical Institute, Kolkata.

Blog: www.ctanujit.org

3
Solving Mathematical Problems

Content of the Book:

1. 15 Objective Problem Sets with Solutions (Total Solved Problems : 450) for all ISI
MS/MTech/PG Entrances.
2. 100 Topic-wise Objective Solved Problems (Total Solved Problems : 100) for all ISI
Entrances.
3. 10 Subjective Problem Sets with Solutions (Total Solved Problems : 100) for all ISI
MS/MTech/PG Entrances.
4. 100 Subjective Problems with Solutions (Total Solved Problems : 100) on Probability &
Statistics (mainly for MSTAT).
5. 15 Subjective Model Papers with Solutions (Total Solved Problems : 150) for ISI
MSQE/MSQMS/MTECH Exams.
6. 100 Subjective Problems with Solutions (Total Solved Problems : 100) on Mathematics
(mainly for MSQE/MSQMS/MTech).

4
Solving Mathematical Problems

ISI OBJECTIVE SAMPLE PAPER 625]=249 .


WITH SOLUTIONS Thus, 1000! ends with 249 zeros.

2. The product of the first 100 positive


SET – 1
integers ends with

(a) 21 zeros (b) 22 zeros (c) 23


There will be 30 questions in MMA Paper. zeros (d) 24 zeros.
For each question, exactly one of the four
Ans:-
choices is correct. You get four marks for
each correct answer, one mark for each
unanswered question, and zero mark for
each incorrect answer. 51 ⟶ 20 (d) 24 zeros .

52 ⟶ 4
1. How many zeros are at the end of
1000! ? Alternatively, put p=5,n=100,thus from
(a) 240 (b) 248 above theorem we have
(c) 249 (d) [100/5]+[100/25]=24 zeros as
None the answer.

Ans:- (c) The number of two’s is enough


to match each 5 to get a 10. 3. Let P (x) be a polynomial of degree 11
𝟏
such that P (x) = 𝒙+𝟏 𝒇𝒐𝒓 𝒙 = 𝟎 (𝟏)𝟏𝟏.
So,

51 ⟶ 200 Then P (12) = ?


𝟏
52 ⟶ 40 ∴Thus, 1000! (a) 0 (b) 1 (c) 𝟏𝟑 (d) none of
Ends with 249 zeros these

53 ⟶ 8 Ans:- (a) P (x)= 𝑥+1


1

54 ⟶ 1
⇒ (x+1)[P (x)]-1 = c (x-0)(x-1)….(x-11)
[Theorem: (de Polinac's formula)
Putting x= -1, 0- 1= c (-1)(-2)….(-12)
Statement: Let p be a prime and e be the
largest exponent of p such that pe 1
⇒ c = - 12!
divides n! , then e=∑ [n/pi ], where i is
running from 1 to infinity.] 1
∴ [P (x)](x+1)-1= - 12!(x-0)(x-1)….(x-11)
So, 1
[1000/5]+[1000/25]+[1000/125]+[1000/ ⇒ P (12) 13-1 = - 12! 12 .11. ….2.1

5
Solving Mathematical Problems

⇒ P (12) 13-1 = -1 Log 0< a < 1, 0< b <1 ∴ log 𝑎 and log 𝑏 are
both negative.
⇒P (12) = 0.
𝑦 𝑎 𝑦
Also 𝑥 > 1 and 𝑏 > 1 . Thus log ( 𝑥 ) and
4. Let s= {(𝒙𝟏 , 𝒙𝟐 , 𝒙𝟑 )| 0≤ 𝒙𝒊 ≤
𝑎
𝟗 𝒂𝒏𝒅 𝒙𝟏 + 𝒙𝟐 + 𝒙𝟑 is divisible by 3}. log ( 𝑏) are both positive. Finally E turns

Then the number of elements in s is out to be a negative value. So, E can’t take
the value ‘2’.
(a) 334 (b) 333 (c) 327 (d)
336 6. Let S be the set of all 3- digits
numbers. Such that
Ans:- (a) with each (𝑥1 , 𝑥2 , 𝑥3 ) identify a (i) The digits in each number are
three digit code, where reading zeros are all from the set {1, 2, 3, …., 9}
allowed. We have a bijection between s and (ii) Exactly one digit in each
the set of all non-negative integers less than number is even
or equal to 999 divisible by 3. The no. of
numbers between 1 and 999, inclusive, The sum of all number in S is
999
divisible by 3 is ( ) = 333 (a) 96100 (b) 133200 (c) 66600
3
(d) 99800
Also, ‘0’ is divisible by 3. Hence, the
number of elements in s is = 333 + 1= 334. Ans:- (b) The sum of the digits in unit place
of all the numbers in s will be same as the
5. Let x and y be positive real number sum in tens or hundreds place. The only
with x< y. Also 0 < b< a < 1. even digit can have any of the three
𝒚 𝒙
positions,
Define E =𝐥𝐨𝐠 𝒂 (𝒙) + 𝐥𝐨𝐠 𝒃 (𝒚). Then E
i.e. 3𝑐1 ways.
can’t take the value
And the digit itself has 4 choices (2, 4, 6 or
(a) -2 (b) -1 (c) -√𝟐 (d) 2
8). The other two digits can be filled in 5× 4
𝑦
𝑦 𝑥 log
𝑥
= 20 ways.
Ans :- (d) E =log 𝑎 (𝑥 ) + log 𝑏 (𝑦) = −
log𝑎
log
𝑦 Then the number of numbers in S = 240.
𝑥
log𝑏
Number of numbers containing the even
= digits in units place = 4 × 5 × 4 = 80
𝑦 1 1
log ( 𝑥 ) {log − log } = The other 160 numbers have digits 1, 3, 5, 7
𝑎 𝑏
𝑦 log −log or 9 in unit place, with each digit appearing
log ( 𝑥 ) {(log 𝑏)(log𝑎 )}
𝑎 𝑏
160
= 32 times. Sum in units place = 32 (1+
= 5

log( )
𝑏
log( )
𝑎 3+ 5+ 7+ 9) + 20 (2+ 4+ 6+ 8)
𝑦 𝑎 𝑦 𝑏
log ( 𝑥 ) . (log = - log ( 𝑥 ) . (log
𝑎 )(log𝑏 ) 𝑎 )(log𝑏 )

6
Solving Mathematical Problems

= 32.52 + 20 × 2 ×
4×5
= 32× 25 + 20 × M is a non –real matrix.
2
20 = 1200 𝟏+𝒊√𝟑
𝟐𝟎𝟎𝟖
9. The value of ( ) is
𝟐
∴ The sum of all numbers= 1200 (1+ 10
+102 ) = 1200× 111 = 133200. (a)
𝟏+𝒊√𝟑
(b)
𝟏−𝒊√𝟑
(c)
𝟐 𝟐
𝒙 −𝟏−𝒊√𝟑 −𝟏+𝒊√𝟑
7. Let y = 𝒙𝟐 +𝟏 , Then 𝒚𝟒 (𝟏)is equals (d)
𝟐 𝟐
(a) 4 (b) -3 (c) 3 (d) -4
1+𝑖√3 −1+𝑖√3
Ans:- (c) A = ( ), 𝐴2 = , 𝐴4 =
Ans:- (b) Simply differentiating would be 2 2
−1−𝑖√3
tedious, = −𝐴
2

So we take advantage of ‘i’ the square root −1−𝑖√3


∴ 𝐴2008 = (𝐴4 )502 = 𝐴4 = .
of ‘-1’ 2

𝑥 1 1 1 10. Let f(x) be the function f(x)=


y = 𝑥 2 +1 = {(𝑥−𝑖) + (𝑥+𝑖)
}
2 𝒙𝑷
𝒊𝒇 𝒙 > 0
{(𝒔𝒊𝒏𝒙)𝒒
𝑑4 𝑦 1 4! 4!
= { + } 𝟎 𝒊𝒇 𝒙 = 𝟎
𝑑𝑥 4 2 (𝑥−𝑖)5 (𝑥−𝑖)5

𝑑𝑛 1 (−1)𝑛 𝑛! Then f(x) is continuous at x= 0 if


Note that, 𝑑𝑥 𝑛 {𝑥+𝑎} = (𝑥+𝑎)𝑛+1
(a) p > q (b) p > 0 (c)
4 (𝑥) 4! 1! 1!
So, 𝑦 = { + } Then q>0 (d) p < q
2 (𝑥−𝑖)5 (𝑥−𝑖)5
𝑥𝑃
𝑦 4 (1)
= 12
1! 1!
{(𝑥−𝑖)5 + (𝑥−𝑖)5 }
1−𝑖
= 12 {(−2𝑖)3 + Ans:- (b) |f(x)-f(0)|= |(𝑠𝑖𝑛𝑥)𝑞 − 0| ≤ |𝑥 𝑃 | < ∊
1−𝑖 1−𝑖 1−𝑖 1 1 1
} = 12 { 8𝑖 + } = 12 (-8 − 8) = -3.
(2𝑖)3 8𝑖 Whenever |x-0| < ∊𝑝 = 𝛿 if p > 0.
8. A real 2× 2 matrix. M such that So, f(x)is continuous for p > 0 at x= 0.
−𝟏 𝟎
𝑴𝟐 = ( ) 𝟏
𝟎 −𝟏−∊ 11. The limit 𝐥𝐢𝐦 𝐥𝐨𝐠(𝟏 − 𝒏𝟐 )𝒏 equals
(a) exists for all ∊ > 0 𝒙→∞
𝟏
(b) does not exist for any ∊> 0 (a) 𝒆 −𝟏
(b) 𝒆−𝟐
(c) 𝒆−𝟐 (d) 1
(c) exists for same ∊> 0
(d) none of the above 1
Ans:- (d) L= (1 − 𝑛2 )𝑛
Ans:- (b) since 𝑀2 is an diagonal matrix, so 1
𝑖 0  𝑙𝑜𝑔𝐿 = 𝑛𝑙𝑜𝑔(1 − 𝑛2 )
M= [ ], 1 1
0 √1−∊  lim 𝑙𝑜𝑔𝐿 = lim [−𝑛{𝑛2 + 2𝑛4 +
𝑥→∞ 𝑥→∞
So, M is not a real matrix, for any values of ⋯ ∞}] = 0

∴L = 𝑒 0 = 1.

7
Solving Mathematical Problems

12. The minimum value of the function (2.24)52 − (2.23)52 252 (2452 −2352 )
Ans:- (b) R= (4.24)26 + (4.23)26 = 426 (2426 + 2326 )
f(x, y)= 𝟒𝒙𝟐 + 𝟗𝒚𝟐 − 𝟏𝟐𝒙 − 𝟏𝟐𝒚 + 𝟏𝟒
252 (2426 + 2326 )(2426 − 2326 )
is = 252 . 2426 + 2326
(a) 1 (b) 3 (c) 14
(d) none = 2426 − 2326
< 2426
2 2
Ans:- (a) f(x, y) = 4𝑥 + 9𝑦 − 12𝑥 −
12𝑦 + 14 Also, R= 2426 − 2326 = (1 + 23)26 − 2326

= (4𝑥 2 − 12𝑥 + 9)+( 9𝑦 2 − = 2326 + 26𝑐1 . 2325 + 26𝑐2 . 2324 + … + 1 −


12𝑦 + 4)+1 2326

= (2𝑥 − 3)2 + (3𝑦 − 2)2 + = 26. 2325 +26𝑐2 . 2324 + … + 1 >26. 2325
1 ≥1 > 23. 2325 = 2326

So, minimum value of f(x, y) is 1. ∴ 2326 < R < 2426

13. From a group of 20 persons, 16. A function f is said to be odd if f (-x)=


belonging to an association, A -f (x) ∀ 𝒙. Which of the following is not
president, a secretary and there odd?
members are to be elected for the (a) f (x+ y)= f(x)+ f(y) ∀ 𝒙, 𝒚
𝒙
executive committee. The number of 𝒙𝒆 ⁄𝟐
(b) f (x)= 𝟏+ 𝒆𝒙
ways this can be done is
(a) 30000 (b) 310080 (c) f (x) = x - [x]
(c) 300080 (d) none (d) f (x) = 𝒙𝟐 𝐬𝐢𝐧 𝒙 + 𝒙𝟑 𝐜𝐨𝐬 𝒙

Ans:- (b) 20𝑐1 × 19𝑐1 × 18𝑐3 𝑜𝑟


20!
= Ans:- (c) f (x+ y)= f(x)+ f(y) ∀ 𝑥, 𝑦
1!1!3!15!
310080 Let x = y = 0
𝐜𝐨𝐬 𝒙−𝐬𝐞𝐜 𝒙
14. The 𝐥𝐢𝐦 is ⇒ f (0) = f (0) + f (0)
𝒙⟶𝟎 𝒙𝟐 (𝟏+𝒙)
(a) -1 (b) 1 (c) 0 ∴ f (0)= 0
(d) does not exist
Replacing y with –x , we have
cos 𝑥−sec 𝑥 − sin2 𝑥
Ans:- (a) lim = lim
𝑥⟶0 𝑥 2 (1+𝑥) 𝑥⟶0 cos 𝑥 (𝑥 2 )(𝑥+1) f (x- x) = f(x) + f (-x)
1 sin 𝑥 2 1
= - lim ( ) . = -1. 1. 1 = - 1. ⇒ f (0) = f(x) + f (-x)
𝑥⟶0 cos 𝑥 𝑥 (𝑥+1)

𝟒𝟖𝟓𝟐 − 𝟒𝟔𝟓𝟐 ⇒ f(x) + f (-x) = 0


15. Let R = 𝟗𝟔𝟐𝟔 + 𝟗𝟐𝟐𝟔 . Then R satisfies
(a) R < 1 (b) 𝟐𝟑𝟐𝟔 < R < 𝟐𝟒𝟐𝟔 ⇒ f (-x) = -f(x)
(c) 1 < R < 𝟐𝟑𝟐𝟔 (d) R >
Thus f is odd.
𝟐𝟔
𝟐𝟒

8
Solving Mathematical Problems
𝑥 4
𝑥𝑒 ⁄2 ∴ m = 65.
Again for f (x) = 1+ 𝑒 𝑥

𝑥 4 524
𝑥
(−𝑥)(𝑒 − ⁄2 ) (−𝑥)(𝑒 − ⁄2 ) .𝑒 𝑥 𝑥
𝑥𝑒 ⁄2 ∴ a= −8 − =− .
f(-x)= = = - 1+ 𝑒 𝑥 = 65 65
1+ 𝑒 −𝑥 1+ 𝑒 𝑥
-f (x) 18. In a special version of chess, a rook
moves either horizontally or vertically
∴ f is odd. on the chess board. The number of
f (x) = x- [x] is not odd. ways to place 8 rooks of different
colors on a 8×8 chess board such that
Counter example:- no rook lies on the path of the other
rook at the start of the game is
f (-2.3) = -2.3 – [-2.3] =-2.3 – (-3) = 3- 2.3 =
(a) 8× ⎿𝟖 (b)⎿8 × ⎿𝟖 (c)
0.7 𝟖 𝟖 64
𝟐 × ⎿𝟖 (d) 𝟐 × ( 8 )
f (2.3) = 2.3 – [2.3] = 2.3 -2 =0.3
Ans:- The first rook can be placed in any
∴ f(2.3) ≠ f(-2.3) row in 8 ways & in any column in 8 ways.
So, it has 82 ways to be disposed off. Since
Thus f is not odd
no other rook can be placed in the path of
f (x) = 𝑥 2 sin 𝑥 + 𝑥 3 cos 𝑥 the first rook, a second rook can be placed in
72 ways for there now remains only 7 rows
f(-x) = -𝑥 2 sin 𝑥 − 𝑥 3 cos 𝑥 = -f(x) and 7 columns. Counting in this manner, the
number of ways = 82 . 72 . 62 … 12 = (8!)2
∴ f is odd here.
𝟏 𝟏
17. Consider the polynomial 𝒙𝟓 + 𝒂𝒙𝟒 + 19. The value of ∫𝟎 ∫𝟎 {𝑴𝒊𝒏 (𝒙, 𝒚) −
𝒃𝒙𝟑 + 𝒄𝒙𝟐 + 𝒅𝒙 + 𝟒. If (1+2i) and (3- 𝒙𝒚}𝒅𝒙𝒅𝒚 is
2i) are two roots of this polynomial (a) ½ (b) 1/3
then the value of a is (c) 1/6 (d) 1/12
(a) -524/65 (b) 524/65 1 1
(c) -1/65 (d) 1/65 Ans:- (d) ∫0 ∫0 {𝑀𝑖𝑛 (𝑥, 𝑦) − 𝑥𝑦}𝑑𝑥𝑑𝑦

1 𝑦 1 𝑥
Ans:- (a) The polynomial has 5 roots. Since = ∫0 ∫0 𝑥𝑑𝑥𝑑𝑦 + ∫0 ∫0 𝑦𝑑𝑥𝑑𝑦
complex root occur in pairs, so there is one
1
real root taking it as m. =3

So, m, 1+2i, 1-2i, 3+2i, 3-2i are the five 1 1 1


And ∫0 ∫0 𝑥𝑦 𝑑𝑥𝑑𝑦 =
roots. 4

1 1 1
Sum of the roots= − 1 = 8 + 𝑚.
𝑎 ∴I=3−4= 12

Product of the roots= (1+4)(9+4)m= 65


4
m=65

9
Solving Mathematical Problems

20. Given that ∑ 𝒂𝒏 converges (𝒂𝒏 > 0); 22. If f(x)= x+ sinx, then find
𝟐𝝅
Then ∑ 𝒂𝒏 𝟑 𝐬𝐢𝐧 𝒏 𝟐
. ∫𝝅 (𝒇−𝟏 (𝒙) + 𝒔𝒊𝒏𝒙)𝒅𝒙
𝝅𝟐
(a) Converges (b) Diverges (a) 2 (b) 3
(b) Doesn’t exist (d) None (c) 6 (d) 9
Ans:- (a) Since ∑ 𝑎𝑛 converges, we have
Ans:- (b) Let x= f(t) ⇒ dx= f′(t)dt
lim 𝑛 . 𝑎𝑛 converges.
𝑛⟶∞
2𝜋 2𝜋
⇒ ∫𝜋 𝑓 −1 (𝑥)𝑑𝑥 = ∫𝜋 𝑡 f ′ (t)dt =
i.e. | 𝑛 . 𝑎𝑛 | ≤ 1 𝑓𝑜𝑟 𝑛 ≥ 𝑀 (say) 2𝜋
(t [f(t)])2π π − ∫𝜋 𝑓(t)dt = (4𝜋 2 − 𝜋 2 ) −
⇒ 𝑛 . 𝑎𝑛 < 1 [∵𝑎𝑛 > 0] 2𝜋
∫𝜋 𝑓(t)dt
1
⇒ 𝑎𝑛 < 2𝜋
𝑛 I= ∫𝜋 (𝑓 −1 (𝑥) +
2𝜋 2𝜋
∴ 𝑎𝑛 3 < 𝑛 3
1 𝑠𝑖𝑛𝑥)𝑑𝑥 = ∫𝜋 𝑓 −1 (𝑥)𝑑𝑥 + ∫𝜋 𝑠𝑖𝑛𝑥𝑑𝑥

2𝜋 2𝜋
1
⇒ 𝑎𝑛 3 sin 𝑛 ≤ 𝑛3 sin 𝑛 ≤ 𝑛3
1 = 3𝜋 2 − ∫𝜋 𝑓(t)dt + ∫𝜋 𝑠𝑖𝑛𝑥𝑑𝑥

2𝜋
⇒∑ 𝑎𝑛 3 sin 𝑛 ≤ ∑ 𝑛3
1 = 3𝜋 2 − ∫𝜋 (𝑓(𝑥) − 𝑠𝑖𝑛𝑥)𝑑𝑥

2𝜋 1
∵ RHS converges so LHS will also = 3𝜋 2 − ∫𝜋 𝑥𝑑𝑥 = 3𝜋 2 − 2 (4𝜋 2 − 𝜋 2 )
converge. 3
= 𝜋2
2
21. The differential equation of all the
ellipses centered at the origin is 2
⇒𝜋2 𝐼 = 3.
(a) 𝒚𝟐 + 𝒙(𝒚′)𝟐 − 𝒚𝒚′ = 𝟎
(b) x y y″+ 𝒙(𝒚′)𝟐 − 𝒚𝒚′ = 𝟎 23. Let P= (a, b), Q= (c, d) and 0 < a < b <
c < d, L≡(a, 0), M≡(c, 0), R lies on x-axis
(c) y y″+ 𝒙(𝒚′)𝟐 − 𝒙𝒚′ = 𝟎 such that PR + RQ is minimum, then R
(d) none divides LM
𝑥2 𝑦2
Ans:- (d) + 𝑏2 = 1, after differentiating (a) Internally in the ratio a: b
𝑎2
w.r.t x, we get (b) internally in the ratio b: c

2𝑥 2𝑦𝑦′ 𝑦𝑦′ 𝑥 (c) internally in the ratio b: d


⤇ 𝑎2 + =0⤇ = − 𝑎2
𝑏2 𝑏2 (d) internally in the ratio d: b
(𝑦′)2 𝑦(𝑦″) 1
⤇ + = − 𝑎2 Ans:- (c) Let R = (𝛼, 0). PR+RQ is least
𝑏2 𝑏2

𝑏2 ⟹ PQR should be the path of light


⤇(𝑦′)2 + 𝑦(𝑦″)2 = − 𝑎2 .
⟹ 𝛥 PRL and QRM are similar

10
Solving Mathematical Problems
𝐿𝑅 𝑃𝐿 𝛼−𝑎 𝑏 Ans:- (d)
⟹ 𝑅𝑀 = 𝑄𝑀 ⟹ =𝑑
𝑐−𝛼
𝑘
(𝑥 +𝑥𝑘 ) 𝑥1 +𝑥𝑘 𝑎+𝑚+𝑎 𝑚𝑘
2 1
⟹ 𝛼d- 𝛼d= bc –𝛼b X= = =
𝑘 2 2
𝑎𝑑+𝑏𝑐
⟹ 𝛼= 𝑜𝑟, 𝑥 = 𝑎 +
(𝑘+1)𝑚
𝑏+𝑑
2

⟹ R divides LM internally in the ratio b : d or, 2(x-a)= (k+1)m ……………..(1)


𝑏
(as 𝑑 > 0)
Similarly,
24. A point (1, 1) undergoes reflection in
2(y-b)= (k+ 1)n ……………….(2)
the x-axis and then the co-ordinate axes
𝝅
are roated through an angle of 𝟒 in We have to eliminate k
anticlockwise direction. The final position
From (1) and (2)
of the point in the new co-ordinate system
𝑥−𝑎 𝑚
is- =
𝑦−𝑏 𝑛

(a) (0, √𝟐) (b) (0, −√𝟐) or, (x- a)n = (y -b)m
(c) −√𝟐, 𝟎 (d) none of these
26. An unbiased die with faces marked 1,
Ans:- . (b) Image of (1, 1) in the x-axis is 2, 3, 4, 5 and 6 is rolled four times. Out of
(1, -1). If (x, y) be the co-ordinates of any four face values obtained the probability
point and (x’ , y’) be its new co-ordinates, that the minimum face value is not less
then x’ = x cos 𝜃+ y sin 𝜃, than 2 and the maximum face value is not
greater than 5 is-
y’= y cos𝜃 – x sin 𝜃, where 𝜃 is the angle
through which the axes have been roated. 𝟏𝟔 𝟏
(a) 𝟖𝟏 (b) 𝟖𝟏
𝜋 𝟖𝟎 𝟔𝟓
Here 𝜃= 4 , x= 1, y= -1 (c) 𝟖𝟏 (d) 𝟖𝟏

∴ x’= 0, y’= -√2 Ans. (a)

25. If a, 𝒙𝟏 , 𝒙𝟐 , … , 𝒙𝒌 and b, 𝒚𝟏 , 𝒚𝟐 , … , 𝒚𝒌 For minimum face value not to be less than


from two A.P. with common difference m 2 and maximum face value not to be greater
and n respectively, then the locus of point than 5, a number out of 2, 3, 4, 5 must occur
∑𝒌𝒊=𝟏 𝒙𝟏 ∑𝒌𝒊=𝟏 𝒚𝟏 in each toss.
(x, y) where x= 𝒊𝒔 𝒂𝒏𝒅 𝒚 = is
𝒌 𝒌
Probability of occurrence of 2, 3, 4, 5 in one
(a) (x-a)m= (y-b)n 4 2
toss = 6 = 3
(b) (x-m) a= (y-n) b
2 4 14
(c)(x-n)a = (y-m)b ∴ Required probability = (3) = 81
(d) (x-a) n-(y-b) m

11
Solving Mathematical Problems

27. The probability of India winning test 𝝅 𝟐𝝅


(a) (b)
𝟑 𝟑
match against west Indies is ½ . Assuming 𝟐𝝅 𝝅
independence from match to match, the (c) (d) 𝟐
√𝟑
probability that in a 5 match series ∞ ∞ 2 +𝑥𝑦+𝑦 2 )
India’s second win occurs at the third Ans:- (c) ∫−∞ ∫−∞ 𝑒 −(𝑥 𝑑𝑥𝑑𝑦
test, is ∞ ∞ 𝑦 2 3 2
=∫−∞ ∫−∞ 𝑒 −{(𝑥−2 ) + 𝑦 )}
4 𝑑𝑥𝑑𝑦
𝟐 𝟏 𝟏 𝟏
(a) 𝟑 (b) 𝟐 (c) 𝟒 (d) 𝟖
𝑦 2 3 2
∞ ∞
= ∫−∞{∫−∞ 𝑒 −(𝑥−2 ) 𝑑𝑥 } 𝑒 −4𝑦 𝑑𝑦
Ans. (c)
∞ 3 2 ∞
2√𝜋 2
Let 𝐸𝑟 denotes the probability that india = ∫−∞ √𝜋 𝑒 −4𝑦 𝑑𝑦 = ∫−∞ 𝑒 −𝑢 𝑑𝑢
√3
wine the rth match. Required probability [let
√3
𝑦 = 𝑢]
2
= P(𝐸1 )P(𝐸2 ’)P(𝐸3 )+P(𝐸1 ′)P(𝐸2 )P(𝐸3 ) =
1 1 1 1 1 1 1 1 1 𝜋
(1 − 2) 2 + (1 − 2) . 2 . 2 = 8 . 8 = 4 = 2√ 3 × √𝜋
2

2𝜋
28. The remainder on dividing 𝟏𝟐𝟑𝟒𝟓𝟔𝟕 + = .
√3
𝟖𝟗𝟏𝟎𝟏𝟏 by 12 is
30. The value of
(a) 1 (b) 7 𝟐 𝟐 𝟐 𝟐 𝒙𝟏 +𝒙𝟐 +𝒙𝟑 −𝒙𝟒
∫𝟏 ∫𝟏 ∫𝟏 ∫𝟏 𝒅𝒙𝟏 𝒅𝒙𝟐 𝒅𝒙𝟑 𝒅𝒙𝟒 is
(c) 9 (d) none 𝒙𝟏 +𝒙𝟐 +𝒙𝟑 +𝒙𝟒

Ans:- ( c) 1234≡1 (mod 3)⇒ 1234567 ≡ (a) ½ (b) 1/3


1 (𝑚𝑜𝑑 3)𝑎𝑛𝑑 89 ≡ −1(𝑚𝑜𝑑3) (c) ¼ (d) 1

⇒ 891011 ≡ −1 (𝑚𝑜𝑑 3) Ans:- (a)


2 2 2 2
∴ 1234567 + 891011 ≡ 0 (𝑚𝑜𝑑 3) 𝑥𝑖 𝑑𝑥1 𝑑𝑥2 𝑑𝑥3 𝑑𝑥4
∫ ∫ ∫ ∫
1 1 1 1 𝑥1 + 𝑥2 + 𝑥3 + 𝑥4
Here 1234 is even, so 1234567 ≡ 1 2 2 2 2
𝑥1 + 𝑥2 + 𝑥3 + 𝑥4
0(𝑚𝑜𝑑 4)𝑎𝑛𝑑 89 ≡ 1 (𝑚𝑜𝑑 4) = 𝑎𝑠 ∫ ∫ ∫ ∫ 𝑑𝑥1 𝑑𝑥2 𝑑𝑥3 𝑑𝑥4
4 1 1 1 1 𝑥1 + 𝑥2 + 𝑥3 + 𝑥4
⇒ 891011 ≡ 1 (𝑚𝑜𝑑 4) = 1.
3 1 1
Thus 1234567 + 891011 ≡ 1 (𝑚𝑜𝑑 4) ∴I=4−4=2.

Hence it is 9 (mod 12)


∞ 𝟐
29. Given that ∫−∞ 𝒆−𝒙 𝒅𝒙 = √𝝅, then the
value of
∞ ∞ 𝟐 +𝒙𝒚+𝒚𝟐 )
∫−∞ ∫−∞ 𝒆−(𝒙 𝒅𝒙𝒅𝒚 , 𝒘𝒉𝒆𝒓𝒆 𝑫 =
𝟐 𝟐 𝟐
{(𝒙, 𝒚) ∊ ℝ ; 𝒙 + 𝒚 ≤ 𝟏} is

12
Solving Mathematical Problems

ISI OBJECTIVE SAMPLE PAPER two quantities is constant, then their product
WITH SOLUTIONS is maximum when the quantities are equal.
𝜆
SET – 2 ∴ a+ b = 𝜆 ⇒a = b = 2

𝜆+1 𝜆+2 2 2
∴ 𝐸2 = 1 + 𝜆2
=( ) ⇒ E = 1+ 𝜆,
𝜆
4
There will be 30 questions in MMA Paper. which is the required result.
For each question, exactly one of the four
choices is correct. You get four marks for 1 1
Alternative: (c) √(1 + 𝑎) (1 + 𝑏) will
each correct answer, one mark for each
unanswered question, and zero mark for minimum when a and b will take the
each incorrect answer. maximum value.

a+b = 𝜆, then the max. Value of a and b is


1. 𝒂𝟏 = 𝒂𝟐 = 𝟏, 𝒂𝟑 = −𝟏, 𝒂𝒏 = 𝜆
a= b=2,
𝒂𝒏−𝟏 . 𝒂𝒏−𝟑 . The value of 𝒂𝟏𝟗𝟔𝟒 is
(a) 1 (b) -1
1 1
(c) 0 (d) none Putting these, we get, √(1 + 𝑎) (1 + 𝑏)

Ans:- (b) 𝑎1 = 𝑎2 = 1, 𝑎3 = 𝑎4 = 𝑎5 = 2 2 2
min = √(1 + 𝜆) (1 + 𝜆) = 1+ 𝜆.
−1, 𝑎6 = 1, 𝑎7 = −1

+1, +1, −1, −1, −1, 1, −1 ,



1, 1, −1, −1, −1, 1, −1, …………
⏟ 3. The number of pairs of integers (m, n)
satisfying 𝒎𝟐 + 𝒎𝒏 + 𝒏𝟐 = 1 is
Since 1964= (7×280)+4= 7×280 +4. Thus (a) 8 (b) 6
we have 𝑎1964 = -1. (c) 4 (d) 2

2. If a, b are positive real variables Ans:- (b) Consider 𝑚2 + 𝑚𝑛 + 𝑛2 − 1


whose sum is a constant 𝜆, then the
The equation is symmetric in m and n, we
𝟏 𝟏
minimum value of √(𝟏 + 𝒂) (𝟏 + 𝒃) make the substitution
is u = m + n and v= m- n
𝟏 𝟐
(a) 𝜆 - 𝝀 (b) 𝜆 + 𝝀
𝟐 So that 𝑢2 + 𝑣 2 = 2 (𝑚2 + 𝑛2 ), 𝑢2 −
(c) 1+ 𝝀 (d) none
𝑣 2 = 4𝑚𝑛
1 1 1 𝑎+𝑏+1
Ans:- (c) 𝐸 2 = 1 + 𝑎 + + 𝑎𝑏 = + Multiplying the given equation by 4, we
𝑏 𝑎𝑏
𝜆+1 have
1= 1+ , it will be minimum when ab
𝑎𝑏
is maximum. Now we know that if sum of 4𝑚2 + 4𝑚𝑛 + 4𝑛2 = 4

⇒ 4 (𝑚2 + 𝑛2 ) + 4mn = 4
13
Solving Mathematical Problems
𝑛 𝑛
⇒ 2 (𝑢2 + 𝑣 2 ) + 𝑢2 − 𝑣 2 = 4 𝐹𝑛 − 2 = 22 + 1 − 2 = 22 − 1 =
𝑛−1 2 𝑛−1 𝑛−1
2
⇒3𝑢 + 𝑣 = 4 2 (22 ) − 1 = (22 + 1)( 22 − 1)
𝑛−1 𝑛−2
Set 𝑢2 = 𝑥, 𝑣 2 = 𝑦 with x, y ≥ 0, then we = (22 + 1) (22 −
2 𝑛−2 2𝑚 𝑚
get 3x+ y= 4 1)( 2 + 1) =( 2 + 1) (22 − 1)
𝑚−𝑛−1
(22 + 1)
The ordered pairs (x, y) satisfying the above
equation in integers are (0, 4) and (1, 1). = 𝜆 𝐹𝑚 ; Now, 𝐹𝑛 − λ 𝐹𝑚 =2
We have, Let d | 𝐹𝑛 and d |𝐹𝑚 then d|2. Then d= 1 or 2.
But 𝐹𝑚 & 𝐹𝑛 are both odd, hence gcd = 1.
𝑢2 = 0 and 𝑢2 = 1 & 𝑣 2 = 4, and 𝑣 2 = 1
6. The number of real roots of the
I.e. u= 0, v= 2; u=0, v= -2;
𝒙 𝒙𝟐 𝒙𝟑 𝒙𝟕
equation 1+ 𝟏 + + +⋯+ =0
𝟐 𝟑 𝟕
u=1, v= 1; u =1, v= -1;
(without factorial) is
u =-1, v =1; u = -1, v= -1;
(a) 7 (b) 5
Giving 6 ordered pair solutions (m, n) viz (1,
(c) 3 (d) 1
-1), (-1, 1), (1, 0), (0, 1), (0, -1), (-1, 0)
Ans:- (d) let f has a minimum at x = 𝑥0 ,
4. The sum of the digits of the number
where then f′(𝑥0 ) = 0
𝟏𝟎𝟎𝟏𝟑 − 𝟐𝟔, written in decimal
notation is 𝑥 𝑥2 𝑥3 𝑥6
f (x)= 1+ 1 + + +⋯+ ;
(a) 227 (b) 218 2 3 6

(c)228 (d) 219 ⇒ 1+ 𝑥0 + 𝑥0 2 + 𝑥0 3 + 𝑥0 4 + 𝑥0 5 = 0


Ans:- (a) 1026 − 26 = ⏟
100 … . .0 − 26 𝑥0 6 −1
⇒ =0
𝑥0 −1
=⏟
999 … .974
(𝑥0 3 −1)(𝑥0 3 +1)
⇒ =0
𝑥0 −1
with 24 9’s
⇒ (𝑥0 2 + 𝑥0 + 1)( 𝑥0 2 − 𝑥0 + 1)( 𝑥0 +
∴ The sum of the digits = 24 × 9 + 7 + 4 =
227. 1)= 0

5. The great common divisor (gcd) of Which has a real root 𝑥0 = -1


𝟐𝟐 𝟐𝟐𝟐
𝟐𝟐 + 𝟏 𝒂𝒏𝒅 𝟐𝟐 + 𝟏 is 1
But, f (-1)= 1-1 + (2 − 3)+ (4 − 5) +6 > 0
1 1 1 1
𝟐𝟐
(a) 1 (b) 𝟐𝟐 +𝟏
𝟐 𝟏𝟏 The f (x)> 0 and hence f has no real zeros.
(c) 𝟐 −𝟏 (d)
𝟐𝟏
𝟐𝟐 −𝟏 Now let, g (x)= 1+ 1 +
𝑥 𝑥2
+
𝑥3
+ ⋯+
𝑥7
2 3 7
2𝑛
Ans:- (a) let 𝐹𝑛 = 2 + 1 , with m > n
14
Solving Mathematical Problems

An odd degree polynomial has at least one For divisibility by 9, the sum of the digits A
real root. + 7 +3 +8+ 9 +b should be divisible by 9.

If our polynomial g has more than one zero, i.e. a+ 7+ 3+ 8+ 9+ 6≡ 0 (mod 9)


say 𝑥1 , 𝑥2
⇒ a+ 6 ≡ 0 (mod 9)
Then by Role’s theorem in (𝑥1 , 𝑥2 ) we have
‘𝑥3 ’ such that g′ (𝑥3 ) = 0 ⇒ a ≡ -6(mod 9)

⇒ a ≡ 3 (mod 9)
⇒ 1+ 𝑥3 + 𝑥3 2 + ⋯ + 𝑥3 6 = 0

But this has no real zeros. Hence the given ∴ a= 3 only. Hence a+ b= 9
polynomial has exactly one real zero. 9. 3 balls are distributed to 3 boxes at
7. Number of roots between –𝜋 and 𝜋 of random. Number of way in which we
𝟐 set at most 1 box empty is (a) 20
the equation 𝐬𝐢𝐧 𝒙. 𝒙= 1 is
𝟑 (b) 6 (c) 24 (d) none
(a) 1 (b) 2
(c) 3 (d) 4 Ans:- (c) zero box empty + 1 box empty

Ans:- (d) sin 𝑥 =


3 = 3 balls in 3 boxes + {3C1 × 3 balls in 2
2𝑥 boxes}
3
Now, draw the curve of y= sin 𝑥 and y = 2𝑥 = 3! + 3× (3)2 = 24.
or xy = 3⁄2

∴ there are 4 real roots. (Draw the graph


yourself)
10. The value of 𝐌𝐚𝐱 𝑰 (𝒂), 𝒘𝒉𝒆𝒓𝒆 𝑰(𝒂) =
8. The number 𝒂𝟕𝟑𝟖𝟗𝒃 , a, b are digits, is 𝒂+𝟏
𝒂

divisible by 72 , Then a+ b equals ∫𝒂−𝟏 𝒆−|𝒙| 𝒅𝒙 is


(a) 10 (b) 9 (a) ea (b) 2- 𝒆𝒂−𝟏 − 𝒆𝒂+𝟏
(c) 11 (d) 12 (c) 𝒆𝒂−𝟏 − 𝒆𝒂+𝟏 (d) none
𝑎
Ans:- (b) 72 = 8× 9, and 8 and 9 are co Ans:- (b) 𝐼 (𝑎) = ∫𝑎−1 𝑒 −|𝑥| 𝑑𝑥 +
prime. As the number 𝑎7389𝑏 is divisible by 𝑎+1 −|𝑥|
∫𝑎 𝑒 𝑑𝑥
72, it is divisible by 9 and 8 both. For
divisibility by 8, the last three digits must be 𝑎 0
Let 0< a< 1 , then ∫𝑎−1 𝑒 −|𝑥| 𝑑𝑥 = ∫𝑎−1 𝑒 𝑥 𝑑𝑥
divisible by 8. 𝑎−1 −𝑥
+∫0 𝑒 𝑑𝑥
i.e. 800+ 90+b |8.
= 1- 𝑒 𝑎−1 − (𝑒 −𝑎 − 1)
so (b+2)| 8 ; ∴ b = 6
𝑎+1 −|𝑥|
And, ∫𝑎 𝑒 𝑑𝑥 = 𝑒 −𝑎 − 𝑒 −𝑎−1

15
Solving Mathematical Problems

∴ I (a) = 2 - (𝑒 𝑎−1 + 𝑒 −𝑎−1 ) 13. If 𝒖𝒏 =


𝟏 𝟏
+ 𝟐(𝒏−𝟏) + 𝟑 (𝒏−𝟐) +
𝟏
𝟏.𝒏
𝑑 𝟏
∴ 𝑑𝑥 𝐼 (𝑎) = 0 … . . + (𝒏−𝟏) ; Then 𝐥𝐢𝐦 𝒖𝒏 equals
𝒏⟶∞
(a) 0 (b) 1
⇒ 𝑒 𝑎−1 = 𝑒 −𝑎+1 (c) ∞ (d) π
⇒ a= 0 1 1 1
Ans:- (a) 𝑢𝑛 = [(1 + 𝑛) + (2 +
(𝑛+1)
𝑎
Also, -1 < a < 0 ; ∫𝑎−1 𝑒 𝑥 𝑑𝑥 = 𝑒 𝑎 − 𝑒 𝑎−1 1 1 1 1
) + (3 + 𝑛−2) + ⋯ + (𝑛 + 1)]
𝑛−1
𝑎+1 −|𝑥| 0 𝑎+1 −𝑥
And ∫𝑎 𝑒 𝑑𝑥 = ∫𝑎 𝑒 𝑥 𝑑𝑥 + ∫0 𝑒 𝑑𝑥 1 1 1
= (𝑛+1) 2 (1 + + ⋯ + 𝑛)
2
= (1- 𝑒 𝑎 ) – (𝑒 𝑎+1 − 1) 1 1 1
1+ + +⋯+ 𝑛
2 3 𝑛
𝑎 𝑎+1 ∴ lim 𝑢𝑛 = 2 lim . lim =
= 2 -𝑒 − 𝑒 𝑛⟶∞ 𝑛⟶∞ 𝑛 𝑛⟶∞ 𝑛+1
2. 0.1= 0.
∴ I (a) = 2- 𝑒 𝑎−1 − 𝑒 𝑎+1
𝟏 𝟏
14. If x+ (𝒙) = -1, The value of 𝒙𝟗𝟗 + (𝒙𝟗𝟗 )
∴ I(a) is maximum at a = 0.
is
𝟏
11. The value of ∏∞
𝒏=𝟐(𝟏 − 𝒏𝟐 ) is (a) 1 (b) 2
(c) 0 (d) none
(a) 1 ( b) 0 (c) ½
(d) none 1
Ans:- (b) If 𝑎𝑛 = 𝑥 𝑛 + 𝑥 𝑛
1
Ans:- (c) ∏∞ ∞
𝑛=2 (1 − 𝑛2 ) =∏𝑛=2 (1 + Then, 𝑎𝑛+1 = 𝑎𝑛 . 𝑎1 − 𝑎𝑛−1 𝑓𝑜𝑟 𝑛 ≥ 1
1 1 𝑛+1 𝑛−1
) (1 − 𝑛) = ∏∞
𝑛=2 𝑛 .
𝑛 𝑛 𝑎0 = 2 , 𝑎2 = −𝑎1 − 𝑎0 = −1, 𝑎3 = 2, 𝑎4
2+1 2−1 3+1 3−1 = −1, 𝑎5 = 2, 𝑎1 = −1 ; 𝑎𝑛
=( . )( . )……… =
2 2 3 3 = −1, 𝑎𝑛−1 = −1.
3 1 4 2 1
. . . ……. = 2 .
2 2 3 3
𝑠𝑜 , 𝑎99 = 2
12. 𝒄𝒐𝒔𝟖 𝜽 − 𝒔𝒊𝒏𝟖 𝜽= 1. Number of roots 1
are there in between [0, 2𝜋] is or, ∴𝑎𝑛+1 = 𝑥 99 + 𝑥 99
(a) 1 (b) 2 (c)
1 1 1
3 (d) 0 = (𝑥 98 + ) (𝑥 + ) − (𝑥 97
+ )
𝑥 98 𝑥 𝑥 97
Ans:- (c) Note that , 𝑐𝑜𝑠 8 𝜃 = 1 + 𝑠𝑖𝑛8 𝜃 is = - 𝑎𝑛 − 𝑎𝑛−1 = +1 + 1 = 2
possible only if,
8 8
15. Consider the equation of the form𝒙𝟐 +
𝑐𝑜𝑠 𝜃 = 1 𝑎𝑛𝑑 𝑠𝑖𝑛 𝜃 =0
𝒃𝒙 + 𝒄 = 𝟎. The number of such,
∴ 𝜃= 0, 𝜋, 2𝜋 equations that have real roots and
have coefficients b and c in the set {1,
Hence 3 roots are three between [0, 2𝜋] 2, 3, 4, 5,6} , (b may be equal to c) is

16
Solving Mathematical Problems

(a) 16 (b) 19 f(0)= 1 ⇒ b= 1


(c) 21 (d) none
f(1)= 3+ a
2
Ans:- (b) Let 𝑥 + 𝑏𝑥 + 𝑐 = 0 has real
f(x) ≥f(1) ⇒ f′(1)=0
roots, then 𝑏 2 − 4𝑐 ≥ 0, and also , s= {1, 2,
3, 4, 5, 6}. ⇒ 4+ a= 0
Now 𝑠1 = {4, 8, 12, 16, 20, 24}= set of ⇒ a = -4
possible values of 4c.
∴ f(x)= 2𝑥 2 + 4𝑥 + 1
Thus the number of equations will be same
as the number of pairs of elements (𝑎1 , 𝑎2 ), ∴ f(2)= 1.
𝟏−𝟑𝑷 𝟏+𝟒𝑷 𝟏+𝑷
𝑎1 ∊ s, 𝑎2 ∊𝑠1 such that 18. Let , , are the
𝟐 𝟑 𝟔

𝑎1 2 − 4𝑎2 ≥ 0, i.e. 1+ 2+ 4+ 6+6 = 19 probabilities of 3 mutually exclusive


and exhaustive events, then the set of
16. If f: R ⟶R, satisfies f(x +y)= all values of P is
f(x)+f(y)∀ 𝒙, 𝒚 ∊ ℝ and f(1)=7, (a) [-1/4, 1/3] (b) (0, 1)
then∑𝒏𝒓=𝟏 𝒇(𝒓) is (c) (0, ∞) (d) none
𝟕 (𝒏+𝟏)
(a) (b) 7n (n+1) 1−3𝑃 1+4𝑃 1+𝑃
𝟐 Ans:- (a) ≥ 0, ≥ 0, ≥ 0 and
𝟕 𝒏(𝒏+𝟏) 2 3 6
(c) (d)none
𝟐
1 − 3𝑃 1 + 4𝑃 1 + 𝑃
Ans:- (c) putting x= 1, y=0, then f(1)= + + =1
2 3 6
f(1)+f(0) 1 1 1 1
⇒− 4 ≤ 𝑃 ≤ 3 ⇒ 𝑃 ∊ [− 4 , 3]
⇒f(0)=0, ⇒ f(1)=7
𝝅 𝒙
19. If √𝟓𝒙 − 𝒙𝟐 − 𝟔 + 𝟐 ∫𝟎 𝒅𝒕 >
Again , putting x=1, y=1, then f(2)= 2f(1)=
𝝅
14, similarly, 𝑥 ∫𝟎 𝐬𝐢𝐧𝟐 𝒕𝒅𝒕, then x ∊
(a) (2, 3) (b) (-∞, 𝟐) ∪ (𝟑, ∞)
f(3)=21 and so on.
(c) (5/2 , 3) (d) none
7 𝑛(𝑛+1)
∑𝑛𝑟=1 𝑓(𝑟)= 7 {1+ 2+ 3+…..+ n}= . 𝜋𝑥 1 𝜋
2 Ans:- (a) √5𝑥 − 𝑥 2 − 6 + > 𝑥 {2 ∫0 (1 −
2

17. Let f(0)= 1, 𝐥𝐭 𝒇″ (𝒙) = 𝟒 𝒂𝒏𝒅 𝒇(𝒙) ≥ 𝑐𝑜𝑠2𝑡)𝑑𝑡}


𝒙→∞
𝒇(𝟏). Let f(x) is polynomial ∀ x∊ℝ. 𝜋𝑥 1
⤇ √5𝑥 − 𝑥 2 − 6 + > 𝑥 {2 (𝑡 −
2
The value of f(2) is 1
(a) 4 (b) 0 sin 2𝑡) 𝜋0}
2
(c) 1 (d) none 𝜋𝑥 𝜋𝑥
⤇√5𝑥 − 𝑥 2 − 6 + >
2 2
Ans:- (c) 𝑓 ″ (𝑥) = 4 = 𝑐𝑜𝑛𝑠𝑡𝑎𝑛𝑡
2
⤇ √5𝑥 − 𝑥 2 − 6 > 0
⇒ f(x)= 2𝑥 + 𝑎𝑥 + 𝑏
17
Solving Mathematical Problems

⤇𝑥 2 − 5𝑥 + 6 < 0, 22. Let f(x)=


𝒙|𝒙|; 𝒙 ≤ −𝟏
⤇ (x-2)(x-3)< 0, i.e. , x ∊ (2, 3). {[𝒙 + 𝟏] + [𝟏 − 𝒙]; −𝟏 < 𝑥 < 1
−𝒙 |𝒙|; 𝒙 ≥𝟏
20. If f(x)= (𝟒 + 𝒙)𝒏 , n ∊ N and 𝒇𝒓 (𝟎)
represents the 𝒓𝒕𝒉 derivative of f(x) at 𝟐
Then the value of ∫−𝟐 𝒇(𝒙)𝒅𝒙 is
𝒇𝒓 (𝟎)
x= o, then the value of ∑∞
𝒓=𝟎 =
𝒓! 𝟖 𝟕
(a) 𝟐𝒏 (b) 𝒆𝒏 (a) - 𝟑 (b) - 𝟑
(c) 𝟓𝒏 (d) none (c) 𝟑
𝟕
(d) none
Ans:- (c) f′(x)= n (4 + 𝑥)𝑛−1
−𝑥 2 , 𝑥 ≤ −1
1 , −1<𝑥 <0
f″(x)= n (n-1) (4 + 𝑥)𝑛−2
Ans:- (a) f(x) = 2 , 𝑥=0
𝑓 𝑟 (𝑥)= n (n-1)….. (n- r+1) . (4 + 𝑥)𝑛−𝑟 , r 1 , 0<𝑥<1
{ 𝑥2 , 𝑥 ≥1
≤𝑛
2
𝑛! ∴ f(x) is an even function, i.e. ∫−2 𝑓(𝑥)𝑑𝑥 =
𝑓 𝑟 (0)= (𝑛−𝑟)! . 4𝑛−𝑟 , r ≤ 𝑛
2
2 ∫0 𝑓(𝑥)𝑑𝑥
= 0, r >𝑛
1 2
= 2 {∫0 𝑓(𝑥)𝑑𝑥 + ∫1 𝑓(𝑥)𝑑𝑥 }
𝑓 𝑟 (0)
∴∑∞
𝑟=0 = ∑𝑛𝑟=0(𝑛𝑟). 4𝑛−𝑟 = (1 + 4)𝑛 =
𝑟!
𝑥3 2 8
5𝑛 . = 2 (1 −
3
)1 = − .
3

21. The two lines r⃗ = a⃗ + 𝜆(b⃗ +c⃗) and r⃗ 23. Area bounded by y = g(x), x-axis and
= b⃗ + 𝜇(c⃗ +a⃗) intersects at a point, the lines x= - 2,
where 𝜆 and 𝜇 are scalars, then
Where g (x)=
(a) a⃗, b⃗ and c⃗ are non-coplanar 𝐦𝐚𝐱{ 𝒇(𝒕): −𝟐 ≤ 𝒕 ≤ 𝒙}, 𝒘𝒉𝒆𝒓𝒆 − 𝟐 ≤ 𝒙 < 0;
{
(b) |a⃗| = |b⃗| = |c⃗| 𝐦𝐢𝐧{ 𝒇(𝒕): 𝟎 ≤ 𝒕 ≤ 𝒙}, 𝒘𝒉𝒆𝒓𝒆 𝟎 ≤ 𝒙 ≤ 𝟑

(c) a⃗.c⃗ = b⃗.c⃗ And f(x)= 𝒙𝟐 − |𝒙|, is equal to


(d) 𝜆(b⃗×c⃗) + 𝜇(c⃗×a⃗)=c⃗
𝟏𝟏𝟑 𝟏𝟏𝟏
(a) (b)
𝟐𝟒 𝟐𝟒
Ans. (c) 𝟏𝟏𝟕
(c) (d) none
𝟐𝟒
The two lines intersect
Ans:- (a) g(x)=
∴ a⃗ + 𝜆(b⃗ × 𝑐⃗) = 𝑏⃗⃗ + 𝜇(𝑐⃗ × 𝑎⃗) 2 ; −2 ≤ 𝑥 < 0
2 1
Taking dot product with c⃗ on both sides, we {𝑥 − 𝑥 ; 0≤𝑥≤2
1 1
get −4 ; <𝑥≤3
2

a⃗. c⃗ = b⃗ .c⃗

18
Solving Mathematical Problems
1 h− 2 𝜋 1
0 ∴ h+ 2 = 𝑐𝑜𝑠 4 = ,
∴ Required area = ∫−2 2𝑑𝑥 + ∫02(𝑥 − √2
3 1 113
𝑥 2 )𝑑𝑥 + ∫1 (4) 𝑑𝑥 = 𝑢𝑛𝑖𝑡 2 2(√2+1)
2 24 ⤇ h= = 2(3 + 2√2) .
(√2−1)
24. Total number of positive integral
26. Tangents and normal drawn to
values of n such that the equations
parabola at A (𝒂𝒕𝟐 , 𝟐𝒂𝒕), 𝒕 ≠ 𝟎 meet
𝐜𝐨𝐬 −𝟏 𝒙 + (𝐬𝐢𝐧−𝟏 𝒚)𝟐 =
𝒏𝝅𝟐 𝝅𝟐
the X- axis at point B and D,
𝒂𝒏𝒅 (𝐬𝐢𝐧−𝟏 𝒚)𝟐 − 𝐜𝐨𝐬 −𝟏 𝒙 = 𝟏𝟔 respectively. If the rectangle ABCD is
𝟒
are constant, is equal to (a) y-2a= 0 (b) y+ 2a= 0
(a) 1 (b) 2 (c) x-2a= 0 (d) none
(c) 3 (d) none
Ans:- (c) Evolution of tangent & normal at
Ans:- (a) Here 2 (sin−1 𝑦)2 =
4𝑛+1
𝜋2 A are 𝑦 𝑡 = 𝑋 + 𝑎𝑡 2 , 𝑦 = −𝑡𝑥 + 2𝑎𝑡 + 𝑎𝑡 3 .
16

4𝑛+1 𝜋2 ∴ B = (-𝑎𝑡 2 , 0) and D= (2a+ 𝑎𝑡 2 , 0)


⤇0≤ 𝜋2 ≤ ,
16 4
Suppose ABCD is rectangle,
1 7
⤇ -4 ≤ 𝑛 ≤ 4.
Then midpoints of BD and AC will be
4𝑛−1 coincident,
Also. 2(cos −1 𝑥)= 𝜋2
16
∴ h+𝑎𝑡 2 = 2𝑎 + 𝑎𝑡 2 − 𝑎𝑡 2 𝑎𝑛𝑑 𝑘 +
4𝑛−1 2
⤇0≤ 𝜋 ≤ 𝜋, 2𝑎𝑡 = 0
16

1 8 i.e. h= 2a, k= -2at.


⤇4 ≤ 𝑛 ≤ 𝜋 + 1.
Hence, the locus is X= 2a, i.e. X-2a=0.
Hence, the least positive integral value of n
is 1. 𝟏
27. The series ∑∞
𝒌=𝟐 (𝒌(𝒌−𝟏)) converges to
25. Radius of bigger circle touching the (a) -1 (b) 1 (c)
circle 𝒙𝟐 + 𝒚𝟐 − 𝟒𝒙 − 𝟒𝒚 + 𝟒 = 𝟎 and 0 (d) does not
both the converges
co-ordinate axis is
𝑛 1 1
(a) 3+2√𝟐 (b) 2(3+2√𝟐) Ans:- (b) 𝑠𝑛 = ∑∞
𝑘=2 𝑘(𝑘−1) = ∑𝑘=2((𝑘−1) −
1 1
(c) 3- 2√𝟐 (d) none ) = (1 − 𝑛)
𝑘

Ans:- (b) Let (h, h) be the centre of the 1


required circle. ∴ lim 𝑠𝑛 = lim (1 − 𝑛) = 1.
𝑛→∞
𝑛→∞
𝜋
∴∠COD= ∠CBE= 4 , CB= h+ 2 AND BD=
h- 2.

19
Solving Mathematical Problems
𝟑𝒙−𝟏
28. The limit 𝐥𝐢𝐦 ( 𝟑𝒙+𝟏)𝟒𝒙 equaqls ISI OBJECTIVE SAMPLE PAPER
𝒙→∞
(a) 1 (b) 0 (c) WITH SOLUTIONS
𝟖 𝟒

𝒆 𝟑 (d) 𝒆 𝟗
SET – 3
1 1
1− − 8
𝑒 3 4
Ans:- (c) lim {( 3𝑥
1 ) 𝑥 }4 = ( 1 ) = 𝑒 −3
𝑥→∞ 1+ 𝑒3
3𝑥
There will be 30 questions in MMA Paper.
𝑘 𝑥 𝑘
[since lim (1 + ) = 𝑒 ]. For each question, exactly one of the four
𝑥→∞ 𝑥
choices is correct. You get four marks for
each correct answer, one mark for each
unanswered question, and zero mark for
𝟏 𝒏 𝒏 𝒏 each incorrect answer.
29. 𝐥𝐢𝐦 𝒏 (𝒏+𝟏 + 𝒏+𝟐 + ⋯ + 𝟐𝒏) equals
𝒙→∞

(a) ∞ (b) 0 (c) 1. Number of solutions are possible in


𝐥𝐨𝐠 𝒆 𝟐 (d) 1 0≤ 𝒙 ≤ 𝟗𝟗 for the equation
1 1 1 1
Ans:- (c) lim 𝑛 ( + + ⋯+ ) 𝟑−𝒙
𝑥→∞ 1+
1
1+
2
1+
𝑛
𝑛
|𝟑 − 𝟑𝒙 | + |𝟏 − 𝟑𝒙 | = 𝟏 − 𝟑𝒙 − is
𝑛 𝑛 𝟒

1 1
= ∫0 𝑑𝑥 = [log(1 + 𝑥)] 10 = log 𝑒 2. (a) 1 (b) 0 (c) 2
1+𝑥
(d) none

Ans:- (b) LHS= |3 − 3𝑥 | + |3𝑥 − 1| ≥


30. Let k be an integer greater than 1. |(3 − 3𝑥 ) + (3𝑥 − 1)| ≥ 2
𝒏 𝒏
Then 𝐥𝐢𝐦 [𝒏+𝟏 + 𝒏+𝟐 + ⋯ ] is 3−𝑥
𝒏→∞ But RHS= 1 - (3𝑥 + )
4
(a) 𝐥𝐨𝐠 𝒆 𝒌 (b) (k-1) 𝐥𝐨𝐠 𝒆 𝒌 𝑥 𝑥
𝑥 − 𝑥 −
(c) 0 (d) ∞ 3 2 3 2
=1- {(3 +
2 ) - 2. 3 .
2 }
2 2
1 𝑘−1 𝑑𝑥
Ans:- (a) lim [∑𝑛(𝑘−1)
𝑟=1 ] = ∫0 = 𝑥 −
3 2 2
𝑥
𝑛→∞ 𝑛+𝑟 1+𝑥
= 2 - (3 +
2 ) <2
log(1 + 𝑥)] 𝑘−1
0
= log 𝑒 𝑘 . 2

∴ given equation has no solution for any


real x.

2. If f(x) = 𝐥𝐨𝐠 𝒆 (𝟔 − |𝒙𝟐 + 𝒙 − 𝟔|), then


domain of f(x) has how many integral
values of x?
(a) 5 (b) 4
(c) infinite (d) none
of these

20
Solving Mathematical Problems

Ans:- (b) f(x) is defined only when 6 − ∴x<0


|𝑥 2 + 𝑥 − 6| > 0
(ii) If 0 ≤ 𝑥 ≤ 1, then |1- x- 1|+x ≤
i.e. |𝑥 2 + 𝑥 − 6| < 6 2
⇒ |x|+x ≤ 2
⇒ -6 < 𝑥 2 + 𝑥 − 6 < 6 ⇒ 2x ≤ 2
⇒ 𝑥 2 + 𝑥 > 0 and 𝑥 2 + 𝑥 − 12 < 0 ⇒ x ≤ 1, ∴0 ≤ 𝑥 < 1
(iii) If 1 ≤ 𝑥 < 2, then |x-1 -1|+x ≤ 2
⇒ x (x+1)>0 and (x+4)(x-3)< 0 ⇒ |x-2|+x ≤ 2
⇒ 2- x +x ≤ 2
⇒ (x< -1 or x > 0) and (-4 < x < 3)
⇒ 2 ≤ 2 (true)
⇒ x ∊ (-4, -1) ∪(0, 3) ⇒ x= -3, -2, 1, 2 as (iv) If x ≥ 2, then |x-1 -1|+x ≤ 2
integral values. ⇒ x-2 +x ≤ 2
⇒x ≤2
3. The sum of the real solution of 2|𝒙|𝟐 + ∴ x=2 [∵ x≥ 2]
51= |1+ 20x| is ∴ Required solution set is
(a) 5 (b) 0 (−∞, 2]
(c) 24 (d) none of
these 𝟏
5. If domain of f(x)= √|𝒙−𝟏|+[𝒙] be (a, b),
Ans:- (d) 2𝑥 2 + 51 = ±(1 + 20𝑥)
then ([ .] denotes greatest integer
2 2
⇒ 𝑥 − 10𝑥 + 25 = 0 𝑜𝑟 𝑥 + 10𝑥 + 26 = function)
0 (a) a= 1, b=∞ (b) a= -∞, b= 0
(c) a= -∞, b= 1 (d) none of
⇒ (𝑥 − 5)2 = 0 𝑜𝑟(𝑥 + 5)2 + 1 = these
0(impossible)
Ans:- (c) we must have, |x-1|> [x]…….(1)
⇒ x= 5, 5
∴ x-1 < [x]≤ x, i.e. [x]> x – 1……….(2)
∴ Sum of the real solution = 5+ 5= 10.
∴ on combining (1) and (2), we have |x -1|>
4. The solution set of ||x- 1|-1| + x ≤ 𝟐 is x -1
(a) (−∞, 𝟐] (b) [0, 1)
(c) [0, 2) (d) [1, 2) This is true only if x-1< 0, i.e. if x < 1, i.e. if
x ∊ (-∞, 1)
Ans:- (a) (i) If x < 0, then |1- x- 1|+x ≤ 2
∴𝐷𝑓 = (−∞, 1) ⇒ a = −∞, b= 1
⇒ |x|+x ≤ 2

⇒ -x +x ≤ 2

⇒ 0 ≤ 2 (true)

21
Solving Mathematical Problems

6. If there are 4 distinct solutions of ||x - 𝒅𝒚 𝟏


8. If 𝒅𝒙 = 𝒇(𝒙) + ∫𝟎 𝒇(𝒙)𝒅𝒙 then the
2012| + 𝐥𝐨𝐠 𝟐 𝒂| = 3, then a ∊
𝟏
equation of the curve y=f(x) passing
(a) (−∞, −𝟔) (b) (−∞, 𝟖) through (0, 1) is
𝟏 𝟐𝒆𝒙 −𝒆+𝟏
(c) (−∞, − 𝟖) (d) none of (a) f(x)= (b) f(x)=
𝟑−𝒆
these 𝟑𝒆𝒙 −𝟐𝒆+𝟏 𝒆𝒙 −𝟐𝒆+𝟏
(c) f(x)=
𝟐(𝒙−𝒆) 𝒆+𝟏
Ans:- (b) we have | x -2012|+log 2 𝑎 = ±3 (d) none of these

⇒ | x -2012| = -log 2 𝑎+ 3, -log 2 𝑎 − 3 Ans:- (a) f″(x)= f′(x)

∴ If there are 4 distinct solutions of the f″(x)


⇒ =1
f′(x)
above equation, then we must have
On integrating f′(x) = c 𝑒 𝑥
-log 2 𝑎+ 3> 0 and -log 2 𝑎 − 3 > 0
Which gives f(x)= c 𝑒 𝑥 +D
i.e. log 2 𝑎 < 3 and log 2 𝑎 < -3 ⇒ a < 2−3
1 But f(0) = 1 ⇒ c+ D = 1
∴ a ∊ (−∞, 8)
∴ f(x)= c 𝑒 𝑥 + 1 − 𝑐
7. The number of value of k for which
the equation 𝒙𝟑 − 𝟑𝒙 + 𝒌 = 𝟎 has two So, f′(x)= c 𝑒 𝑥 putting it in f′(x)=
1
distinct roots lying in the interval (0, f(x)+ ∫0 𝑓(𝑥)𝑑𝑥
1) are
1
(a) 3 (b) 2 (c) infinitely many ⇒ c 𝑒 𝑥 = c 𝑒 𝑥 + 1 − 𝑐 + ∫0 (c 𝑒 𝑥 + 1 −
(d) no value of k satisfies the 𝑐)𝑑𝑥
requirement
2
⇒ c = 3−𝑒
Ans:- (d) Let there be a value of k for which
𝑥 3 − 3𝑥 + 𝑘 = 0 has two distinct roots 2𝑒 𝑥 −𝑒+1
So, f(x)=
between 0 and 1. Let, a, b are two distinct 3−𝑒

roots of 𝑥 3 − 3𝑥 + 𝑘 = 0 lying between 0 9. A staircase has 10 steps, a person can


and 1 such that a < b go up the steps one at a time, or any
combination of 1’s and 2’s . The
Let f (a)= f(b)= 0. Since between any two
number of ways in which the person
roots of a polynomial f(x) there exist at least
can go up the stairs is
one roots of its derivative f′(x).
(a) 89 (b) 144
Therefore, f′(x) = 3𝑥 3 − 3 has at least one (c) 132 (d) 211
root between a and b
Ans:- (a)
But f′(x) =0 has two roots equal to ± 1
which don’t lie between 0 and 1 for any
value of k.

22
Solving Mathematical Problems

x+ 2y= 10, where x is the number of times = 3× 27867 + 4 × 64563


he takes single steps, and y is the number
times he takes two steps = 3(28 − 1)867 + 4 (63 + 1)563

Case Total no. of ways = 3[multiple of 7- 1]+ 4[ multiple of 7+ 1]

1 X=0, y=5 5! /5!=1 = multiple of 7+ 1

2 X=2, y= 4 6! /2! 4! =15 ∴ Hence remainder is 1.

∞ 𝟏
3 X=4, y=3 7! /3!4!= 35 11. The value of ∑∞ ∞
𝒊=𝟎 ∑𝒋=𝟎 ∑𝒌=𝟎 𝟑𝒊 𝟑𝒋 𝟑𝒌
is
(𝒊 ≠𝒋≠𝒌)
4 X=6, y=2 8! /2! 6!= 28 (a) 80/207 (b) 81/208
(c) 1/208 (d) none
5 X=8, y=1 9! /8! = 9
Ans:- (b) Let us first of all find the sum
6 X=10, y=0 10! /10! =1
without any restriction i, j, k.

1 1 3 27
∑∞ ∞ ∞
𝑖=0 ∑𝑗=0 ∑𝑘=0 = (∑∞
𝑖=0 3𝑖 ) =
3𝑖 3𝑗 3𝑘 8
∴ P= 89
For the requirement sum we have to remove
the cases when i= j= k or when any two of
10. The remainder when 𝟏𝟔𝟗𝟎𝟐𝟔𝟎𝟖 + them are equal and not equal to other
𝟐𝟔𝟎𝟖𝟏𝟔𝟗𝟎 is divided by 7 is variable (say, i= j≠ 𝑘).
(a) 1 (b) 2 (c) Case –I:- when i= j= k
3 (d) none
1
In this case ∑∞ ∞ ∞
𝑖=0 ∑𝑗=0 ∑𝑘=0 3𝑖 3𝑗 3𝑘 =
Ans:- (a) 1690= 7× 241 + 3;
1 27
2608= 7 ×372 +4 ∑∞
𝑖=0 = 16
3𝑖

Let s =16902608 + 26081690 Case – II:- i= j≠ 𝑘

= (7 × 241 + 3)2608 + (7 × 372 + In this case, ∑∞ ∞ ∞ 1


𝑖=0 ∑𝑗=0 ∑𝑘=0 3𝑖 3𝑗 3𝑘 =
1690
4) 1 1
(∑∞ ∞
𝑖=0 32𝑖 ) (∑𝑘=0 3𝑘 )
= a number multiple of 7+32608 +
1 3 1
41690 = ∑∞
𝑖=0 32𝑖 (2 − 3𝑖 )

Let s′= 32608 + 41690 3 9


= 2 . 8 − 26 =
27 135
8.26
Clearly remainder in s and s′ will be same 27 27 135
when divided by 7. Hence required sum = 8 − 26 − (8.26) . 3=
81
3×867 3×563
s′= 3× 3 +4×4 208

23
Solving Mathematical Problems

12. The solution of the differential ⇒ 𝑥 2 − 𝑥(3 + 𝑎) + 2𝑎(3 − 𝑎) ≥ 0 ∀ 𝑥 ∊


𝒅𝒚 𝑅
equation f(x) + 𝒇′ (𝒙)𝒚 = 𝟏 is given
𝒅𝒙
by f(x) = ⇒(𝑎 + 3)2 − 8𝑎(3 − 𝑎) ≤ 0 ⇒ (𝑎 −
𝒙+𝒄
(a) yx + c (b) 𝒚 1)2 ≤ 0 ⇒ 𝑎 = 1 which is true ∀ 𝑥 ∊ 𝑅
(c) yc (d) none

Ans:- (b) f(x)dy +f′(x)ydx = dx


15. If A is skew–symmetric matrix, then
i.e. d (f(x), y) = d (x) B = (I- A) (𝑰 + 𝑨)−𝟏 is (where I is the
identity matrix of the same order as
Integrating we get, y. f(x) = x+ c A)
𝑥+𝑐 (a) idempotent matrix (b)
or, f(x) = 𝑦 symmetric matrix (c)orthogonal
𝒙 matrix (d) none
13. If ∫𝟎 𝒇(𝒙)𝒔𝒊𝒏𝒕𝒅𝒕 = constant, 0 < x <
2𝜋 and f(𝜋)= 2 Then find the value of Ans:- (c) B= (I- A) (𝐼 + 𝐴)−1
𝝅
f(𝟐 ) ⇒ 𝐵 𝑇 = (𝐼 + 𝐴𝑇 )−1 (𝐼 + 𝐴𝑇 ) = (𝐼 −
(a) 2 (b) 4 (c) 6 𝐴)−1(I+ A)
(d) 8
𝐵 𝐵 𝑡 = I as (I- A) (I+ A)= (I+ A) (I-A)
Ans:- (b) Differentiable both sides, we get

f′(x) (1- cosx)+ f(x)sinx= 0


𝟏
16. If f(x)= max (𝝅 𝐜𝐨𝐬 −𝟏 (𝒄𝒐𝒔𝝅𝒙), {𝒙})
f′(x) 𝑠𝑖𝑛𝑥
⇒ ∫ f(x) 𝑑𝑥 = ∫ 1−𝑐𝑜𝑠𝑥 𝑑𝑥 and g (x) min
𝟏
𝑥 (𝝅 𝐜𝐨𝐬 −𝟏 (𝒄𝒐𝒔𝝅𝒙) , {𝒙}) (where { .}
⇒ ln(f (x)| = -2ln sin2 + 𝑙𝑛𝑐
represents fractional part of x). Then
𝑐 𝜋 𝟐
⇒ f (x)= 𝑥 ⇒ f(𝜋)=2 ⇒ c=2: f(2 )=4 find the value of ∫𝟏 𝒇(𝒙)𝒅𝒙/
(sin )2
2 𝟐
∫𝟏 𝒈(𝒙)𝒅𝒙 is
(a) 1 (b) 3 (c) 5
(d) 7
14. For a ∊ R if |x+ a-3| + |x- 2a|= |2x –a -
3| is three for all x ∊R, then exhaustive Ans:- (b)
set of a is
2 3 2 1
(a) a ∊ [-4, 4] (b) a ∊ [-3, 2] ⇒∫1 𝑓(𝑥)𝑑𝑥 = 𝑎𝑛𝑑 ∫1 𝑔(𝑥)𝑑𝑥 =
4 4
(c) a ∊ {-2, 2} (d) a ∊ {1}
⇒ Ratio = 3
Ans:- (d) |x|+ |y| = |x+y|

⇒ xy ≥ 0, therefore (x- (3- a)) (x- 2a)≥


0∀𝑥 ∊𝑅

24
Solving Mathematical Problems

17. If sin (sinx +cosx)= cos (cosx- sinx) 𝟓𝒙


19. If function f(x) = cos(nx)×sin( 𝒏 ),
𝝅
and largest possible value of sinx is 𝒌, satisfies f(x+ 3𝜋)= f(x), then find the
then the value of k is number of integral value of n
(a) 2 (b) 3
(c) 4 (d) none (a) 8 (b) 9 (c) 10
(d) 11
Ans:- (c) sin (sinx +cosx)= cos (cosx- sinx)
Ans:- (a) f(x+ 𝜆) = f(x)
𝜋
cos (cosx- sinx) = cos (2 − (𝑠𝑖𝑛𝑥 + 𝑐𝑜𝑠𝑥))
5(x+ λ)
⇒ cosn(x+ 𝜆) sin( )=
𝜋 𝑛
∴𝑐𝑜𝑠𝑥 − 𝑠𝑖𝑛𝑥 = 2𝑛𝜋 ± ( 2 − (𝑠𝑖𝑛𝑥 + 5𝑥
cos(𝑛𝑥) sin( 𝑛 )
𝑐𝑜𝑠𝑥))
5𝜆
Taking + ve sign At x = 0, cos(n𝜆)sin ( 𝑛 )= 0

𝜋 𝜋
𝑐𝑜𝑠𝑥 − 𝑠𝑖𝑛𝑥 = 2𝑛𝜋 + 2 − 𝑠𝑖𝑛𝑥 − 𝑐𝑜𝑠𝑥 If cos(n𝜆)= 0, n𝜆= r𝜋+ 2 , r ∊ I

𝜋 𝜋 𝜋
𝑐𝑜𝑠𝑥= 𝑛𝜋 + 4 , for n= 0, 𝑐𝑜𝑠𝑥 = , which n (3𝜋)= r𝜋+ 2 (∵𝜆= 3𝜋)
4
is the only possible value
(3n- r)= ½ [not possible]
√16− 𝜋2
⇒ 𝑠𝑖𝑛𝑥 = ………………..(i) 5𝜆
∴ cosn𝜆 ≠ 0 ∴ sin ( 𝑛 )= 0 ⇒
5𝜆
= 𝑃𝜋(𝑃 ∊
4
𝑛
15
Taking –ve sign 𝐼) ⇒ 𝑛 = 𝑃
𝜋
𝑠𝑖𝑛𝑥 = ……………..(ii) For P= ±1, ±3, ±5, ±15
4

From (i) & (ii) , we get


𝜋
as the largest n=±15, ±5, ±3, ±1
4
value. Hence k= 4. 20. Let a, b, c be any real numbers such
that 𝒂𝟐 + 𝒃𝟐 + 𝒄𝟐 = 𝟏 then the
18. The number of solution(s) of the
𝟔𝟒 quantity
equation 𝒛𝟐 − 𝒛 − |𝒛|𝟐 − = 𝟎 is / ab +bc+ ca satisfies the conditions
|𝒛|𝟓
are (a) ab+ bc+ ac = constant
(a) 0 (b) 1 (c) 2 (b) - ½ ≤ 𝒂𝒃 + 𝒃𝒄 + 𝒄𝒂 ≤ 𝟏
(d) 3 (c) - ¼ ≤ 𝒂𝒃 + 𝒃𝒄 + 𝒄𝒂 ≤ 𝟏
𝟏
Ans:- (b) z = 2 is the only solution. (d) -1 ≤ 𝒂𝒃 + 𝒃𝒄 + 𝒄𝒂 ≤ 𝟐

So there is only one solution of the given Ans:- (a) (𝑎 + 𝑏 + 𝑐)2 ≥ 0


equation.
⇒ 𝑎2 + 𝑏 2 + 𝑐 2 ≥ −2(𝑎𝑏 + 𝑏𝑐 + 𝑐𝑎)

25
Solving Mathematical Problems
1
⇒ 2 ≥ −(𝑎𝑏 + 𝑏𝑐 + 𝑐𝑎) ⇒ (𝑎𝑏 + 𝑏𝑐 + ⇒(4a+ 1)+ (4b+ 1)+ (4c +1) = 7
1
𝑐𝑎) ≥ − 2 Applying c-s inequality:- 𝑎1 = √4𝑎 + 1,
𝑎2 = √4𝑏 + 1, 𝑎3 = √4𝑐 + 1 & 𝑏𝑖 =1
21. The maximum value of xyz for +ve x,
y, z subject to condition that xy + yz+ 2
∴ (∑3𝑖=1 𝑎𝑖 .1) ≤ (∑3𝑖=1 𝑎𝑖 2 )(∑3𝑖=1 1) ;
zx= 12 is where 𝑎𝑖 =𝑎1 , 𝑎2 , 𝑎3
(a) 9 (b) 6 (c) 8
(d) none ⇒ (√4𝑎 + 1 + √4𝑏 + 1 + √4𝑐 + 1)2 ≤
xy + yz+ zx 1⁄
(4𝑎 + 1 + 4𝑏 + 1 + 4𝑐 + 1) × (1 + 1 + 1)
Ans:- (c) ≥ (xy . yz. zx) 3
3
= 3 × 7 = 21
⇒ (xyz)≤ 8
24. If f(x)is a polynomial function
22. Let a, b, c are 3 positive real numbers 𝟏 𝟏
satisfying f(x)f(𝒙)= f(x)+ f(𝒙) and
such that a+ b+ c= 2, then the value of
𝒂 𝒃 𝒄 f(3)=28 then f(4) is
. . is always (a) 28 (b) 65
𝟏−𝒂 𝟏−𝒃 𝟏−𝒄
(a) > 8 (b) < 8 (c) 78 (d) none
(c) 8 (d) none
Ans:- (c) The given functional equation is
Ans:- (a) Let 1- a= x, 1- b= y, 1- c = z satisfied by f(x)= ± 𝑥 𝑛 + 1

3- (a+ b+ c) = x+ y+ z = 1(∵ a+ b+ c= 2) f(3)= +33 + 1 = 28


1−𝑥 1−𝑦 1−𝑧
Now, . . Hence, n= 3
𝑥 𝑦 𝑧

𝑦+𝑧 𝑧+𝑥 𝑦+𝑥 So, f(4)= 43 + 1 = 65.


= . .
𝑥 𝑦 𝑧
25. If 2x+ 4y= 1, then prove that the
𝑦+𝑧 𝑧+𝑥 𝑦+𝑥 8
=( )( )( ) . 𝑥𝑦𝑧 > quantity 𝒙𝟐 + 𝒚𝟐 is always greater
2 2 2
8 than equal to
√𝑦𝑧 √𝑧𝑥 √𝑧𝑦 . 𝑥𝑦𝑧 (By AM> GM
(a) 1/20 (b) 5/64
inequality) (c) 1 (d) none
𝑎 𝑏 𝑐
⇒ 1−𝑎 . 1−𝑏 . 1−𝑐 > 8 Ans:- (a) Maximize 𝑥 2 + 𝑦 2 subject to 2x+
4y -1=0 by
23. Let a+ b +c = 1 then the value of the
quantity is always √𝟒𝒂 + 𝟏 + Method of Lagrange multiplier⟶

√𝟒𝒃 + 𝟏 + √𝟒𝒄 + 𝟏 F= 𝑥 2 + 𝑦 2 + 𝜆(2𝑥 + 4𝑦 − 1)


(a) equals 21 (b) ≤ 21
𝜕𝐹 𝜕𝐹
(c) > 21 (d) none = 2𝑥 + 2𝜆 = 0 ; 𝜕𝑦 = 2𝑦 + 2𝜆 = 0
𝜕𝑥

Ans:- (b) 4a+ 4b+ 4c = 4 ∴ x= -𝜆 ∴ y= -2𝜆

26
Solving Mathematical Problems
1 1 𝟏 𝟐
2x+ 4y = 1 𝑥𝑚𝑎𝑥 = + 10 , 𝑦𝑚𝑎𝑥 = 5 Hence the value of f(𝟏𝟗𝟗𝟕) + 𝒇 (𝟏𝟗𝟗𝟕) +
𝟏𝟗𝟗𝟔
−1 ⋯ + 𝒇 (𝟏𝟗𝟗𝟕) is
⇒ 𝜆= 10 ;

1 4 5 1 (a) 998 (b) 1996


∴ 𝑥2 + 𝑦2 ≥ + 100 = = 20. (c) 1997 (d) none
100 100

26. If a, b, c are positive real numbers ∋ 41−𝑥 4/4𝑥


Ans:- (a) f(1- x)= = = f(x)+
4−𝑥+1 +2 4/4𝑥 +2
a+ b+ c= 1. The value of 𝒂𝟐 + 𝒃𝟐 + 𝒄𝟐
is always f(1- x)= 1.
(a) ½ (b) 1/3 1 2 3 998
Now, putting x = 1997 , 1997 , 1997 , … . , 1997
(c) ¼ (d) none
1 2 1996
Ans:- (b) Using C-S inequality, So, f(1997) + 𝑓 (1997) + ⋯ + 𝑓(1997)
2 2 2
(∑3𝑖=1 𝑥𝑖 𝑦𝑖 ) ≤ (∑ 𝑥𝑖 )(∑ 𝑦𝑖 ) Taking = (1
⏟ + 1 + ⋯ + 1)
𝑦𝑖 = 1 & xi = a, b, c.
998 terms
⇒(𝑎 + 𝑏 + 𝑐)2 ≤ (𝑎2 + 𝑏 2 + 𝑐 2 ). 3
= 998
2 2 2 1
⇒𝑎 +𝑏 +𝑐 ≥3
29. If gcd (a, b)=1, then gcd (a+b, a-b) is
27. If a, b, c , x are real numbers such that (a) a or b (b) 1 or 2
𝒙𝒃+(𝟏−𝒙)𝒄 𝒙𝒄+(𝟏−𝒙)𝒂 (c) 1 or 3 (d) none
abc≠ 𝟎 𝒂𝒏𝒅 = =
𝒂 𝒃
𝒙𝒂+(𝟏−𝒙)𝒃 Ans:- (b) let d = gcd (a+ b, a- b)then
𝒄
d | (a+ b) and d |(a -b).
Then prove that a+ b+ c equals to
∴ d | (a +b+ a -b) , ⇒ d| 2a and
(a) 1 (b) 2
(c) 0 (d) none ∴ d | (a +b- a +b) , ⇒ d| 2b
𝑥𝑏+(1−𝑥)𝑐 𝑥𝑐+(1−𝑥)𝑎
Ans:- (c) = = Thus d |(2a, 2b), ⇒ d|2(a,b)
𝑎 𝑏
𝑥𝑎+(1−𝑥)𝑏
=1 Hence d= 1 or 2, because gcd(a, b)= 1
𝑐

𝑎−𝑐 𝑏−𝑎 𝑐−𝑏 30. The number of solution (positive


∴ x= 𝑏−𝑐 , 𝑥 = ,𝑥 =
𝑐−𝑎 𝑎−𝑏 integers) of the equation 3x+ 5y = 1008
The only solution of these are: a= b= c or a+ is
b+ c= 0. (a) 61 (b) 67
(c)79 (d) none
28. If f : R ⟶R is given by f(x)=
𝟒𝒙 Ans:- (b) x, y ∊ ℕ, then 3 |5y ⇒ 3|y, y = 3k
∀ 𝒙 ∈ 𝑹, check f(x)+f(1-x)= 1.
𝟒𝒙 +𝟐 ∀𝑘 ∊ℕ

27
Solving Mathematical Problems

Thus 3x + 15k = 1008 𝝅 𝐬𝐢𝐧(𝟐𝒏−𝟏)


2. if 𝒂𝒏= ∫𝟎 dx, then
𝒔𝒊𝒏𝒙
⇒ x + 5k= 336 𝒂𝟏, 𝒂𝟐, 𝒂𝟑, ……..are in

⇒ 5k ≤ 335 (a) A.P. and H.P.


(b) A.P. and G.P. but not in H.P.

(c) G.P. and H.P.


ISI OBJECTIVE SAMPLE PAPER (d) A.P., G.P. and H.P.
WITH SOLUTIONS
Ans. (b)
SET – 4 𝑎𝑛+1 − 𝑎𝑛
𝜋
sin(2𝑛 + 1)𝑥 − sin(2𝑛 − 1)𝑥
=∫ 𝑑𝑥
0 sin 𝑥
𝜋
There will be 30 questions in MMA Paper. 2 cos 2𝑛𝑥. 𝑠𝑖𝑛𝑥 2 sin 2𝑛𝑥 𝜋
= ∫ 𝑑𝑥 = [ ]
For each question, exactly one of the four 0 𝑠𝑖𝑛𝑥 2𝑛 0
choices is correct. You get four marks for =0
each correct answer, one mark for each
unanswered question, and zero mark for ∴𝑎𝑛+1 = 𝑎𝑛 ⟹ 𝑎1 = 𝑎2 = 𝑎3 = ⋯
each incorrect answer.
Also 𝑎1 = 𝜋 ≠ 0

Hence 𝑎1 , 𝑎2 , … 𝑎𝑛 are in A.P. and G.P. but


1. If 𝑺𝒏 denotes the sum of first n terms of
not in H.P. (Equal numbers cannot be in
an A.P. whose
H.P)
(a) P∑𝒏𝒓=𝟏 𝒓 (b) n∑𝒏𝒑=𝟏 𝒑
𝒑
(c) a∑𝒓=𝟏 𝒓 (d) none of
these 3. If a, b, c are proper fractions and are in
H.P. and x=∑∞ 𝒏 ∞ 𝒏
𝒏=𝟏 𝒂 , y=∑𝒏=𝟏 𝒃 ,
Ans. (d) z=∑∞ 𝒏
𝒏=𝟏 𝒄 ,
𝑛𝑥
𝑆𝑛𝑥 [2𝑎+(𝑛𝑥−1)𝑑] 𝑛[2𝑎−1]+𝑛𝑥𝑑
= 2
𝑥 = then x, y, z are in
𝑆𝑥 [2𝑎+(𝑥−1)𝑑] (2𝑎−𝑑)+𝑥𝑑
2
(a) A.P. (b) G.P. (c)
𝑆𝑛𝑥
For to be independent of x H.P. (d) none of these
𝑆𝑥

2a - d= 0 Ans. (c)
𝑎 𝑥
∴ 2a= d X = 1−𝑎 ⟹ 𝑎 = 1−𝑥
𝑝 𝑦 𝑧
Now, 𝑆𝑝 = 2 [2𝑎 + (𝑝 − 1)𝑑] = 𝑝2 𝑎 Similarly, b = 1−𝑦 , 𝑐 = 1−𝑧

Now, a, b, c are in H.P.

28
Solving Mathematical Problems
1+𝑥 1+𝑦 1+𝑧
⟹ , , are in A.P. (a) 𝒂𝟐 +b+c=0
𝑥 𝑦 𝑧
(b) 𝒂𝟐 +b-c=0
1 1 1
⟹𝑥 , 𝑦 , 𝑧 are in A.P.
(c) 𝒂𝟐 +b-2c=0
(d) 𝒂𝟐 +b-9c=0
⟹ x, y, z are in H.P.
Ans. (b)
4. If a, b, c be the 𝒑𝒕𝒉 ,𝒒𝒕𝒉 and 𝒓𝒕𝒉 terms
respectively of an A.P. and G.P. both, A = (6)𝑛 = 6 6 6 … 6(𝑛 𝑑𝑖𝑔𝑖𝑡𝑠) = 6 × 1 +
then the product of the roots of equation 6 × 10 + 6 × 102 + ⋯ 6 × 10𝑛−1
𝒂𝒃 𝒃𝒄 𝒄𝒂 𝒙𝟐 – 𝒂𝒃𝒄𝒙 + 𝒂𝒄 𝒃𝒂 𝒄𝒃 =0 is equal to
6 2
= 9 (10𝑛 − 1) = 3 (10𝑛 − 1)
(a) -1 (b) 1 (c) 2
(d) (b-c)(c-a)(a-b) 8 4
b= (10𝑛 − 1), 𝑐 = (102𝑛 − 1)
9 9
Ans. (b) 4 8
Now 𝑎2 + 𝑏= 9 (10𝑛 − 1)2 + 9 (10𝑛 −
a= x+ (p- 1)d, b= x+ (q-1)d, c= x+ (r-1)d 4
1) = (10𝑛 − 1)2 (10𝑛 − 1 + 2) =
9
a=𝑚𝑛𝑝−1 , 𝑏 = 𝑚𝑛𝑞−1 , 𝑐 = 𝑚𝑛𝑟−1 4
(102𝑛
− 1) = 𝑐
9
∴ Product of roots =
7. Let a=1 1 1.....1(55 digits),
(𝑚𝑛𝑝−1 )(𝑟−𝑞)𝑑 , (𝑚𝑛𝑞−1 )(𝑝−𝑟)𝑑 , (𝑚𝑛𝑟−1 )(𝑞−𝑝)𝑑 =
0 0
b=1+10+𝟏𝟎𝟐 +…𝟏𝟎𝟒 ,
𝑚 . 𝑛 = 1.
c=1+𝟏𝟎𝟓 +𝟏𝟎𝟏𝟎 +𝟏𝟎𝟏𝟓 +…+𝟏𝟎𝟓𝟎 , then
5. If a, b, c, be the 𝒑𝒕𝒉 ,𝒒𝒕𝒉 and 𝒓𝒕𝒉 terms
(a) a=b+c (b) a=bc
respectively of a G.P. then the equation-
(c) b=ac (d) c=ab
𝒂𝒒 𝒃𝒓 𝒄𝒑 𝒙𝟐 + pqrx + 𝒂𝒓 𝒃𝒏 𝒄𝒒 = 0 has Ans. (b)
(a) both roots zero 1055
(b) at least one root zero a= 1+ 10+ 102 + ⋯ + 1054 = =
10−1
(c) no root zero 1055 −1 105 −1
. = 𝑏𝑐
105 −1 10−1
(d) both roots unity
𝒋
8. If ∑𝒏𝒓=𝟏 𝒕𝒓 = ∑𝒏𝒌=𝟏 ∑𝒌𝒋=𝟏 ∑𝒊=𝟏 𝟐, then
Ans. (c)
𝟏
∑𝒏𝒓=𝟏 =
𝑟−𝑞 𝑝−𝑟 𝑞−𝑝 𝒕𝒓
Product of roots= 𝑎 𝑏 𝑐 =1≠0
𝒏+𝟏 𝒏
⟹ no root is equal to zero. (a) (b) 𝒏+𝟏
𝒏
𝒏−𝟏 𝒏
(c) (d) 𝒏−𝟏
𝒏

6. If (𝒓)𝒏 denotes the number rrr….. (n Ans. (b)


digits), where r=1, 2, 3,…9 and a=(𝟔)𝒏 ,
b=(𝟖)𝒏 , c=(𝟒)𝟐𝒏 , then

29
Solving Mathematical Problems

∑𝑛𝑘=1 ∑ℎ𝑗=1 ∑𝑗𝑖=1 2 = ∑𝑛𝑘=1 ∑𝑘𝑗=1 2𝑗= Given 𝑎1 + 𝑎3 + 𝑎5 + ⋯ + 𝑎2𝑛−1 = 𝑎4 +


𝑘(𝑘+1) 𝑎6 + 𝑎8 + ⋯ + 𝑎2𝑛+2
2∑𝑛𝑘=1 ∑𝑘𝑗=1 𝑗 = 2 ∑𝑛𝑘=1 = ∑𝑛𝑘=1 𝑘 2 +
2
∑𝑛𝑘=1 𝑘 = 𝑟 3 (𝑎1 + 𝑎3 + 𝑎5 + ⋯ + 𝑎2𝑛−1 )

=
𝑛(𝑛+1)(2𝑛+1)
+
𝑛(𝑛+1)
=
𝑛(𝑛+1)(𝑛+2) ⟹𝑟 3 = 1 ⟹ 𝑟 = 1, 𝜔, 𝜔2
6 2 3

𝑛(𝑛+1)(𝑛+2)
11. If 𝒙𝟐 - x + a - 3= 0 has at least one
∴ 𝑆𝑛 = negative value of x, then complete set of
3

𝑟(𝑟+1)(𝑟+2)
values of ‘a’ is
⟹ 𝑡𝑟 = 𝑆𝑟 − 𝑆𝑟−1 = −
3
(𝑟−1)𝑟(𝑟+1) (a) (−∞, 𝟏) (b) (−∞, 𝟐)
= 𝑟(𝑟 + 1)
3 (c) (−∞, 𝟑) (d) none
1 1 1 1 1
= = 𝑟 − 𝑟+1 ∴ ∑𝑛𝑟=1 𝑡 = 1 − Ans. (c) 𝑥 2 - x +a - 3= 0 has at least one
𝑡𝑟 𝑟(𝑟+1) 𝑟
1 𝑛 negative root and for real roots,
= 𝑛+1
𝑛+1
1- 4(a - 3) ≥ 0
𝟏 𝟏
9. If a=∑∞
𝒓=𝟏 𝒓𝟒 , then ∑∞
𝒓=𝟏 (𝟐𝒓−𝟏)𝟒 =
13
⇒a≤ 4
𝟏𝟔 𝒂
(a) 𝟏𝟓a (b) 𝟐 13
𝟏𝟓 𝟏𝟒 ⇒ a ∊ (−∞, 4 )
(c) 𝟏𝟔a (d) 𝟏𝟓a
Now, both root will be non-negative of D ≥
Ans. (c) 0, ⇒ 𝑎 − 3 ≥ 0 ⇒ 𝑎 ≥ 3
1 1 1 1
∑∞
𝑟=1 = 14 + 34 + 54 + ⋯ + 𝑡𝑜 ∞ 13
∴ a ∊ (3, 4 )
(2𝑟−1)4

1 1 1 1 1 13 13
= (14 + 24 + 34 + ⋯ + 𝑡𝑜 ∞) − (24 + 44 + ∴ a ∊ (−∞, 4 )∪ a ∊ (3, 4 )
1
+ ⋯ + 𝑡𝑜 ∞)
64 ∴(−∞, 3)
1 1 1 1
= a-24 (14 + 24 + 34 + ⋯ + 𝑡𝑜 ∞) = 𝑎 − 12. Let 𝛼, 𝛽 are the roots of the equation
𝑎 15
= 16 𝑎 𝒙𝟐 +ax +b=0, then maximum value of the
16 𝜶−𝜷 𝟐
expression - (𝒙𝟐 +ax +b) - ( ) will be
𝟐
10. If 𝒂𝟏, 𝒂𝟐, 𝒂𝟑 ,…are in G.P. having
𝟏
common ratio r such that (a) 𝟒 (𝒂𝟐 − 𝟒𝒃) (b) 0
∑𝒏𝒌=𝟏 𝒂𝟐𝒌−𝟏 =∑𝒏𝒌=𝟏 𝒂𝟐𝒌+𝟐 ≠ 𝟎, then number (c) 1 (d) none
of possible values of r is
Ans. (b) let z= - (𝑥 2 -ax +b)
(a) 1 (b) 2
(c) 3 (d) none of these

Ans. (c)
30
Solving Mathematical Problems

𝐷 𝑎2 −4𝑏 4𝑏−𝑎2
Now, 𝑧𝑚𝑎𝑥. = − 4𝑎 = − = =
4 4
𝛼−𝛽 2
+( ) 15. The sum of real roots of the equation
2
𝒙𝟐 − 𝟐𝟐𝟎𝟎𝟕 . 𝒙 + |𝒙 − 𝟐𝟐𝟎𝟎𝟔 | + 𝟐(𝟐𝟒𝟎𝟏𝟏 −
∴Thus the maximum value of the given 𝟏) = 𝟎 is
equation is 0.
(a) 𝟐𝟐𝟎𝟎𝟔 (b) 𝟐𝟐𝟎𝟎𝟕
(c) 𝟐𝟐𝟎𝟎𝟔 +𝟐𝟐𝟎𝟎𝟕 (d) none
13. Let P (x) = 𝒙𝟐 +bx +c, where b and c Ans. (b) ∴ (𝑥 − 22006 )2 + |𝑥 − 22006 | −
are integers and P(x) is a factor of 2=0
both 𝒙𝟒 + 𝟔𝒙𝟐 + 𝟐𝟓 𝒂𝒏𝒅 𝟑𝒙𝟒 + 𝟒𝒙𝟐 + ⇒ |𝑥 − 22006 |2 + |𝑥 − 22006 | − 2=0 ⇒ x=
𝟐𝟖𝒙 + 𝟓, then P(1) is 22006 +1, 22006 -1.
(a) 4 (b) 8 ∴The sum of real roots are =22007
(c) 24 (d) none
16. Consider an expression 𝒙𝟐 + 𝒚𝟐 +
4 2
Ans. (a) ∴ P(x) is a factor of 3 (𝑥 + 6𝑥 + 𝟐𝒙 + 𝒚= constant. If for two constants 𝛼,
25)- (3𝑥 4 + 4𝑥 2 + 28𝑥 + 5)= 14(𝑥 2 − 𝛽, the conditions x> 𝛼 and x > 𝛽 imply the
2𝑥 + 5) same limits for the value of y, then 𝛼 +𝛽 is
∴ P(x) = 𝑥 2 − 2𝑥 + 5 (a) -2 (b) -4
⇒ P(1)= 4. (c) 1 (d) none

Ans. (a) 𝑥 2 + 𝑦 2 + 2𝑥 + 𝑦 = 𝑘
14. The value of a for which (𝒂𝟐 − 𝟏)𝒙𝟐 +
𝟐(𝒂 − 𝟏)𝒙 + 𝟐 > 0 ∀ 𝑥 are 1 5
⇒ (𝑥 + 1)2 + (𝑦 + 2)2 = 𝑘 + 4
(a) a≥ 𝟏 (b) a≤ 𝟏
5 1
(c) a > - 3 (d) none ⇒ x= -1±√(𝑘 + 4)2 − (𝑦 + 2)2
Ans. (d) we know, 𝑃𝑥 2 + 𝑞𝑥 + 𝑐 > 0 if P >
Now, the two values of x corresponds to 𝛼
0, and 𝑞 2 − 4𝑃𝑐 < 0,
and 𝛽 as y takes the same limits of values.
∴ (𝑎2 − 1)𝑥 2 + 2(𝑎 − 1)𝑥 + 2 > 0 ∀ 𝑥
Hence 𝛼 +𝛽= -2.
2 2 2
Now, 𝑎 − 1 > 0 and 4 (𝑎 − 1) − 8(𝑎 −
𝒂𝟒 +𝒃𝟒 𝒃𝟒 +𝒄𝟒 𝒄𝟒 +𝒂𝟒
1) ≤ 0 17. 𝒂𝟐 +𝒃𝟐 + 𝒃𝟐 +𝒄𝟐 + 𝒄𝟐 +𝒂𝟐 ≥

⇒ 𝑎2 − 1 ≥ 0 and -4(a-1)(a+3) ≤ 0 (a) a+ b+ c (b) 𝒂𝟐 + 𝒃𝟐 +


𝒄𝟐 (c) ab+ bc+ ca
⇒ a ≤ −1 or a ≥ 1 𝑎𝑛𝑑 𝑎 ≤ −3 𝑜𝑟 𝑎 ≥ 1
(d) none
i.e., 𝑎 ≤ −3 𝑜𝑟 𝑎 ≥ 1.
Ans. (b) (𝑎2 − 𝑏 2 )2 ≥ 0

31
Solving Mathematical Problems

⇒ 𝑎4 + 𝑏 4 ≥ 2𝑎2 . 𝑏 2 As 𝑥 2 + 𝑦 2 = 𝑐 2 , then 𝑥 2 𝑦 2 is maximum


𝑐2
4 4
⇒2𝑎 + 2𝑏 ≥ 𝑎 + 𝑏 + 2𝑎 . 𝑏 = 4 4 2 2 when 𝑥 2 = 𝑦 2 = 2
(𝑎2 + 𝑏 2 )2 𝑐2 4
∴ 𝑧𝑚𝑖𝑛. = 𝑐4
= 𝑐 2.
𝑎4 +𝑏4 𝑎2 +𝑏 2
⇒ 𝑎2 +𝑏2 ≥ …………….(1) 4
2
𝒏+𝟏 𝟐𝒏
𝑏 4 +𝑐 4 𝑏 2 +𝑐 2
20. 𝒏𝒏 ( ) >
𝟐
Similarly, 𝑏2 +𝑐 2 ≥ ……………(2)
2
(a) n! (b) (𝒏!)𝟐
𝑐 4 +𝑎4 𝑐 2 +𝑎2
And, 𝑐 2 +𝑎2 ≥ ……………………(3) (c) (𝒏!)𝟑 (d) none.
2
1
13 +23 +⋯+𝑛3
(1)+(2)+(3) implies Ans. (c) > (13 . 23 . … . 𝑛3 )𝑛
𝑛
𝑎4 +𝑏4 𝑏 4 +𝑐 4 𝑐 4 +𝑎4 [∵AM> GM]
+ 𝑏2 +𝑐 2 + 𝑐 2 +𝑎2 ≥ 𝑎2 + 𝑏 2 + 𝑐 2 .
𝑎2 +𝑏2
1
𝑛(𝑛+1)2
⇒ > {(𝑛!)3 }𝑛
18. Let m > 1, n ∊ℕ, then 𝟏𝒎 + 𝟐𝒎 + 4

𝟐𝟐𝒎 + 𝟐𝟑𝒎 + ⋯ + 𝟐𝒏𝒎−𝒎 > 𝑛+1 2𝑛


⇒ 𝑛𝑛 ( ) > {(𝑛!)3 }.
2
(a) 𝒏𝟏−𝒎 (b) (𝟏 − 𝒎)𝒏
(c) 𝒏𝟏−𝒎 (𝟐𝒏−𝟏 ) (d) none 21. If 𝒂𝟏 , 𝒂𝟐 , 𝒂𝟑 , … . , 𝒂𝒏 are non- negative
and 𝒂𝟏 , 𝒂𝟐 , 𝒂𝟑 , … . , 𝒂𝒏 = 𝟏,
1𝑚 +2𝑚 +22𝑚 +23𝑚 +⋯+(2𝑛−1 )𝑚
Ans. (c) > then (1+𝒂𝟏 )(1+𝒂𝟐 ) …(1+𝒂𝒏 )≥
𝑛
1+2+4+⋯+2𝑛−1
( )𝑚
𝑛 (a) 𝟐𝒏 (b) 𝟑𝒏
[∵ m> 0 and AM of mth power > mth power (c) 𝟒𝒏 (d) none
of AM] 1+𝑎𝑖
Ans. (a) ( ) ≥ √𝑎𝑖, where i= 1(1)n.
2
⇒ 1𝑚 + 2𝑚 + 22𝑚 + 23𝑚 + ⋯ + (AM ≥ GM)
2𝑛−1
(2𝑛−1 )𝑚 > 𝑛( 𝑛 )𝑚 > 𝑛1−𝑚 (2𝑛−1 )𝑚
Putting the all I value and then multiplies the
in equations,
19. Let 𝒙𝟐 + 𝒚𝟐 = 𝒄𝟐 , then the least value
of 𝒙−𝟐 + 𝒚−𝟐 is (1+𝑎1 )(1+𝑎2 ) … (1+𝑎𝑛 )≥
2𝑛 √𝑎1 , 𝑎2 , 𝑎3 , … . , 𝑎𝑛
(a) c (b) 𝒄𝟐
(c) 𝒄𝟑 (d) none ⇒ (1+𝑎1 )(1+𝑎2 ) … (1+𝑎𝑛 )≥
𝑥 2 +𝑦 2 𝑐2 2𝑛 (∵𝑎1 , 𝑎2 , 𝑎3 , … . , 𝑎𝑛 = 1)
Ans. (d) Let z= 𝑥 −2 + 𝑦 −2 = = 𝑥2𝑦2
𝑥2𝑦2
and

It will be minimum when 𝑥 2 𝑦 2 will be


maximum.

32
Solving Mathematical Problems
𝜋
22. If 𝒂𝟏 , … . , 𝒂𝒏 are positive real nos. [∵ 0 < x <4 ∴ [𝑥] = 0]
whose product is a fixed number c, then
𝒙 𝟏
the minimum value of 𝒂𝟏 + 𝒂𝟐 + ⋯ + 25. Let g(x) = ∫𝟎 𝒇(𝒕)𝒅𝒕 , 𝒘𝒉𝒆𝒓𝒆 𝟐 ≤
𝒂𝒏−𝟏 + 𝟐𝒂𝒏 is 𝒇(𝒕) ≤ 𝟏, 𝒕 ∈ [𝟎, 𝟏]𝒂𝒏𝒅𝟎 ≤ 𝒇(𝒕) ≤
𝟏
𝟏 𝟏
𝒇𝒐𝒓 𝒕 ∈ (𝟏, 𝟐]. Then
(a) 𝒏(𝟐𝒄) 𝒏 (b) (𝒏 + 𝟏)𝒄 𝒏 𝟐
𝟏
(c) 𝟐𝒏𝒄𝒏 (d) done 𝟑 𝟏
(a) − 𝟐 ≤ 𝒈(𝟐) < 𝟐 (b) 𝟎 ≤ 𝒈(𝟐) < 2
𝟑 𝟓
Ans. (a) AM ≥ GM (c) 𝟐 < 𝑔(𝟐) ≤ 𝟐 (d) 2 < g(2 )< 4
1 1
2 1
So, LHS ≥ 𝑛(𝑎1 … 2𝑎𝑛 )𝑛 = 𝑛(2𝑐)𝑛 Ans. (b) 𝑔(2) = ∫0 𝑓(𝑡) 𝑑𝑡 = ∫0 𝑓(𝑡) 𝑑𝑡 +
2 1
𝒙 𝒆𝐜𝐨𝐬 𝒕 ∫1 𝑓(𝑡) 𝑑𝑡 𝑎𝑠 2 ≤ 𝑓(𝑡) ≤ 1 𝑓𝑜𝑟 0 ≤ 𝑡 ≤ 1,
23. If f(x) = ∫𝟎 𝒆𝐜𝐨𝐬 𝒕+𝒆−𝐜𝐨𝐬 𝒕 𝒅𝒕, then 2f(𝜋) =
1 1 1
1
(a) 0 (b) 𝜋 (c) –𝜋 ∴∫ 𝑑𝑡 ≤ ∫ 𝑓(𝑡)𝑑𝑡 ≤ ∫ 1 𝑑𝑡
0 2 0 0
(d) none of these
1
Ans. (b) 𝑜𝑟, 2 ≤
1
𝜋 𝑒 cos 𝑡 ∫0 𝑓(𝑡)𝑑𝑡 ≤ 1 ………. (1)
𝑓(𝜋) = ∫0 𝑒 cos 𝑡+𝑒 −cos 𝑡 𝑑𝑡……….(1)
1
𝜋 𝑒 −cos 𝑡 𝑎𝑠 0 ≤ 𝑓(𝑡) ≤ 𝑓𝑜𝑟 1 < 𝑡 ≤ 2,
𝑓(𝜋) = ∫0 𝑒 −cos 𝑡 +𝑒 cos 𝑡 𝑑𝑡 …………(2) 2
2 2 2
1
[ since cos (𝜋-t)= - cost] ∴ ∫ 0 𝑑𝑡 ≤ ∫ 𝑓(𝑡) 𝑑𝑡 ≤ ∫ 𝑑𝑡
1 1 1 2
𝜋
∴ 2f(𝜋)= ∫0 𝑑𝑡 = 𝜋
𝑜𝑟, 0 ≤
2 1
∫1 𝑓(𝑡) 𝑑𝑡 ≤ 2 ……………. (2)

24. Let [x] denotes the greatest integer 1 3


𝝅
(1) + (2) ⟹ 2 ≤ 𝑔(2) ≤ 2
less than or equal to x, then ∫𝟎 𝒔𝒊𝒏𝒙 𝒅(𝒙 − 𝟒

[𝒙])= ∴ g(2) satisfies the inequality 0≤ 𝑔(2) < 2.

𝟏 26. The tangent at point P of a curve


(a) ½ (b) 1 -
√𝟐 meets the y- axis at B, the line through P
(c) 1 (d) none of these parallel to y-axis meets the x-axis at A. If
𝜋/4 the area of 𝛥AOB is constant, the curve is
Ans. (b) ∫0 sin 𝑥 𝑑(𝑥 − [𝑥]) =
𝜋 (a) parabola (b) hyperbola
𝜋/4 1
∫0 sin 𝑥 𝑑𝑥 =−[cos 𝑥] 04 = − [ − 1] =
√2 (c) ellipse (d) circle
1
1−
√2 Ans. (b)

33
Solving Mathematical Problems

Let P= (x, y) 𝒅𝒇
(c) 𝒅𝒌 − 𝟐𝒇(𝒌). 𝒄𝒐𝒔𝟐 𝒌 = 0
Equation of tangent to the curve at P(x, y) is (d) none of these
1
𝑑𝑦
Y- y = 𝑑𝑥 (𝑋 − 𝑥) Ans. (a) 𝑓(𝑥) = 1−cos 𝑘 cot 𝑘 = 𝑐𝑜𝑠𝑒𝑐 2 𝑘

𝑑𝑦 𝑑𝑓
When X= 0, Y= y – x 𝑑𝑥 = 2 𝑐𝑜𝑠𝑒𝑐 𝑘 (– 𝑐𝑜𝑠𝑒𝑐 𝑘 cot 𝑘)
𝑑𝑘
𝑑𝑦 = −2𝑓(𝑘) cot 𝑘
∴B≡ (0, 𝑦 − 𝑥 𝑑𝑥 )
𝑑𝑓
Area of 𝛥AOB ant=k 𝑜𝑟 + 2𝑓(𝑘)𝑐𝑜𝑡𝑘 = 0
𝑑𝑘
1 𝑑𝑦 𝑥 2 𝑑𝑦
∴2 𝑥 (𝑦 − 𝑥 𝑑𝑥 ) ⟹ 𝑥𝑦 − = ±2𝑘
𝑑𝑥

𝑥 2 𝑑𝑦
⟹ − 𝑥𝑦 = ±2𝑘 = 𝑐
𝑑𝑥 28. The largest value of ‘c’ such that there
𝑑𝑦 1 2 exists a differentiable function f(x) for –c<
⟹ + 𝑦 (− ) = 2
𝑑𝑥 𝑥 𝑥 x < c that satisfies the equation 𝒚𝟏 = 𝟏 +
1 𝒚𝟐 with f(0)= 0 is
𝐼. 𝐹. = 𝑒 − log 𝑥 =
𝑥 (a) 1 (b) 𝜋
𝝅 𝝅
1 𝐶 (c) 𝟑 (d) 𝟐
𝑠𝑜𝑙𝑢𝑡𝑖𝑜𝑛 𝑖𝑠 𝑔𝑖𝑣𝑒𝑛 𝑏𝑦 𝑦, = ∫ 3 𝑑𝑥 + 𝑎
𝑥 𝑥
𝑑𝑦
Ans. (d) = 1 + 𝑦 2 ⟹ tan−1 𝑦 = 𝑥 +
𝐶 𝑑𝑥
𝑜𝑟 𝑦 = 𝑥 (− 2 ) + 𝑎𝑥 𝑘
2𝑥

𝑜𝑟 2𝑥𝑦 = −𝐶 + 2𝑎𝑥 2 ∵ f(x) satisfies the equation

∴tan−1 𝑓(𝑥) = 𝑥 + 𝑘
𝑜𝑟 2𝑎𝑥 2 − 2𝑥𝑦 − 𝐶 = 0 ………….(1)
Now, f(0)= 0= k= 0
Here h= -1, a= a, b= 0 𝜋 𝜋
⟹ x= tan−1 𝑓(𝑥) ∴ − 2 < 𝑥 < 2
∴ ℎ2 > ab. Hence curve (1) is a
𝟐+𝐬𝐢𝐧 𝒙 𝒅𝒚
hyperbola 29. If y= (x) and (𝒅𝒙) = − 𝐜𝐨𝐬 𝒙,
𝒚+𝟏
𝒌 𝝅
27. The function f(k) = 𝒅𝒌 ∫𝟎
𝒅 𝒅𝒙 𝒚(𝟎) = 𝟏, 𝒚 (𝟐 ) equals:
𝟏−𝐜𝐨𝐬 𝒌.𝐜𝐨𝐬 𝒌
satisfies the differentiable equation (a) 1/3 (b) 2/3 (c) -
𝒅𝒇 1/3 (d) 1
(a) 𝒅𝒌 + 𝟐𝒇(𝒌). 𝐜𝐨𝐭 𝒌 = 𝟎
𝒅𝒇
(b) 𝒅𝒌 + 𝟐𝒇(𝒙). 𝐜𝐨𝐬 𝒌 = 𝟎

34
Solving Mathematical Problems
2+sin 𝑥 𝑑𝑦
Ans. (a) Given, = − cos 𝑥 ISI OBJECTIVE SAMPLE PAPER
𝑦+1 𝑑𝑥
………..(1) & 𝑦(0) = WITH SOLUTIONS
1 ………….(2)
SET – 5
𝑑𝑦 cos 𝑥
(1) ⟹ ∫ = −∫ 𝑑𝑥
𝑦+1 2 + sin 𝑥

⟹ log(𝑦 + 1) = − log(2 + sin 𝑥) + log 𝑐 There will be 30 questions in MMA Paper.


𝑐 For each question, exactly one of the four
⟹𝑦+1= choices is correct. You get four marks for
2 + sin 𝑥
each correct answer, one mark for each
𝑐 unanswered question, and zero mark for
⟹𝑦 = − 1 …………..(3)
2+sin 𝑥 each incorrect answer.
𝑐
𝐴𝑙𝑠𝑜 𝑔𝑖𝑣𝑒𝑛 𝑦(0) = 1 ⟹ 1 = −1⟹𝑐
2 𝝅
𝒔𝒊𝒏𝟐 𝒏𝒙
=4 1. If 𝒂𝒏 =∫ 𝒅𝒙, then
𝟐
𝟎 𝒔𝒊𝒏𝟐 𝒙
𝒂𝟏 𝒂𝟓𝟏 𝒂𝟏𝟎𝟏
2 − sin 𝑥
∴ 𝑓𝑟𝑜𝑚 (3), 𝑦(𝑥) = [𝒂𝟐 𝒂𝟓𝟐 𝒂𝟏𝟎𝟐 ]=
2 + sin 𝑥 𝒂𝟑 𝒂𝟓𝟑 𝒂𝟏𝟎𝟑
𝜋 2−1 1
∴𝑦=( )= = (a) 1 (b) 0
2 2+1 3
(c) -1 (d) none of these

Ans. (b) 𝑎𝑛+2 + 𝑎𝑛 − 2𝑎𝑛+1 = 0


30. A permutation of 1, 2, …., n is chosen
⟹ 𝑎1 , 𝑎2 , 𝑎3 , … 𝑎𝑟𝑒 𝑖𝑛 𝐴. 𝑃.
at random. Then the probability that the
numbers 1, 2 appear as neighbor equals ∴ 𝑎1 + 𝑎101 = 2𝑎1 + 2𝑎1 + 100𝑑 =
𝟏 𝟐 2(𝑎1 + 50𝑑) = 2𝑎51
(a) (b) 𝒏
𝒏
𝟏 𝟏 𝑎2 + 𝑎102 = 2𝑎52 , 𝑎3 + 𝑎103 = 2𝑎53
(c) 𝒏−𝟏 (d) 𝒏−𝟐

(𝑛−1)! 1
Ans. (a) P(A)= = 𝑛.
𝑛!
2. If 𝒕𝒓 =𝟐𝒓⁄𝟑 +𝟐−𝒓⁄𝟑 , then
Since there are n! Permutations total. Since ∑𝟏𝟎𝟎 𝟑 𝟏𝟎𝟎
𝒓=𝟏 𝒕𝒓 − 𝟑 ∑𝒓=𝟏 𝒕𝒓 +1 =
1, 2 appear as neighbour, so taking it as a
𝟐𝟏𝟎𝟏+𝟏 𝟐𝟏𝟎𝟏−𝟏
group, so there are total (n-1)! as number of (a) (b)
𝟐𝟏𝟎𝟎 𝟐𝟏𝟎𝟎
favorable cases. 𝟐𝟐𝟎𝟏−𝟏
(c) (d) None of these
𝟐𝟏𝟎𝟎

Ans. (c)

𝑡𝑟 3 = 2𝑟 + 2−𝑟 + 3𝑡𝑟

35
Solving Mathematical Problems

∴∑100 3 100 𝑟 100 100


𝑟=1 𝑡𝑟 = ∑𝑟=1 2 + ∑𝑟=1 2𝑟 + 3 ∑𝑟=1 𝑡𝑟
1 or, mn𝑥 2 − (2𝑚𝑛 − 𝑚 − 𝑛)𝑥 + 𝑚𝑛 − 𝑚 −
𝑛+1=0
1 1
(1− 100 )
= 2(2100 − 1 + 2 2
1 + 3 ∑100
𝑟=1 𝑡𝑟 = 5. If ∑𝒏𝒓=𝟏 𝒓𝟒 = 𝒂𝒏 , then ∑𝒏𝒓=𝟏 𝒓𝟒 (𝟐𝒓 −
1−
2
1 𝟏)𝟒 =
2101 − 2 + 1 − 2100 + 3 ∑100
𝑟=1 𝑡𝑟
(a) 𝒂𝟐𝒏 +𝒂𝒏 (b) 𝒂𝟐𝒏 -𝒂𝒏
2201 −1
= − 1 + 3 ∑100
𝑟=1 𝑡𝑟 (c) 𝒂𝟐𝒏 -16𝒂𝒏 (d)𝒂𝟐𝒏 +16𝒃𝒏
2100

Ans. (c) ∑𝑛𝑟=1(2𝑟 − 1)4 = 14 + 34 + 54 +


⋯ + (2𝑛 − 1)4
3. If ∑𝒏𝒓=𝟏 𝒓.r! = 100! - 1, then n equals
= [14 + 24 + 34 + ⋯ + (2𝑛)4 ] − [24 +
(a) 100 (b) 101 44 + 64 + ⋯ + (2𝑛)4 ]
(c) 99 (d) none of these
= 𝑎2𝑛 − 24 (14 + 24 + 34 + ⋯ + 𝑛4 ) =
Ans. (c) 𝑎2𝑛 − 16𝑎𝑛 .
𝑡𝑟 = r. r != (r+1 -1)r!= (r+1)! –r! 6. If positive numbers a, b, c be in H.P.,
then equation 𝒙𝟐 − 𝒌𝒙 + 𝟐𝒃𝟏𝟎𝟏 − 𝒂𝟏𝟎𝟏 =
∴∑𝑛𝑟=1 𝑡𝑟 = (𝑛 + 1)! − 1! = (𝑛 + 1)! − 1
𝟎 (𝒌 ∊ 𝑹) has
4. If m = ∑∝𝒓=𝟎 𝒂𝒓 , n=∑∝𝒓=𝟎 𝒃𝒓 where
(a) both roots positive
0<a<1,0<b<1,then the quadratic equation
(b) both roots negative
whose
(c) one positive and one negative root
roots are a and b is
(d) both roots imaginary.
(a) mn𝒙𝟐 +(m+n-2mn)x+mn-m-n+1=0
Ans. (c)
(b) mn𝒙𝟐 +(2mn-m-n)x+mn-m-n+1=0
a, b, c are in H.P.
(c) mn𝒙𝟐 +(2mn+m+n)x+mn+m+n+1=0
(d) mn𝒙𝟐 -(2mn+m+n)x+mn+m+n+1=0 ⟹H.M. of a and c= b⟹√𝑎𝑐 > b (∵G.M. >
H.M.)
Ans. (a)
1 𝑚−1 𝑛−1
Since A .M. > G.M.
m = 1−𝑎 ⟹ 𝑎 = , 𝑠𝑖𝑚𝑖𝑙𝑎𝑟𝑙𝑦 𝑏 = .
𝑚 𝑛
𝑎101 +𝑐 101
∴ > (√𝑎𝑐)101 > 𝑏101 [∵ √𝑎𝑐 >
Required quadratic equation is 2
𝑏]
2
𝑥 − (𝑎 + 𝑏)𝑥 + 𝑎𝑏 = 0
⟹2𝑏101 − 𝑎101 − 𝑐 101 < 0
𝑚−1 𝑛−1 (𝑚−1)(𝑛−1)
or, 𝑥 2 − ( + ) 𝑥+ =0
𝑚 𝑛 𝑚𝑛 Let f(x)= 𝑥 2 − 𝑘𝑥+2𝑏101 − 𝑎101-𝑐 101

36
Solving Mathematical Problems

Then f(−∞) = ∞ > 0, 𝑓(0) = 2𝑏101 − ∴ q- p= r- q


𝑎101 − 𝑐101 < 0, 𝑓(∞) = ∞ > 0.
∴ 𝑚7𝑝 , 𝑚7𝑞 , 𝑚7𝑟 are in G.P.
Hence equation f(x)= 0 has one root in (-
10. Let n be a positive integer and (𝟏 +
∞, 0)and other in(0, ∞).
𝒙 + 𝒙𝟐 )𝒏 = 𝒂𝟎 + 𝒂𝟏 𝒙 + ⋯ + 𝒂𝟐𝒏 𝒙𝟐𝒏 , then
7. If the sum of the series ∑∝𝒏=𝟎 𝒓𝒏 , |r|<1, is the value of 𝒂𝟎 𝟐 − 𝒂𝟏 𝟐 + 𝒂𝟐 𝟐 … . . +𝒂𝟐𝒏 𝟐 is
s, then sum of the series ∑∝𝒏=𝟎 𝒓𝟐𝒏 is
(a) 0 (b) 𝒂𝟎
𝟐𝒔
(a) 𝒔𝟐 (b) 𝒔𝟐−𝟏 (c) 𝒂𝒏 (d) 𝒂𝟐𝒏
𝒔𝟐 𝒔𝟐
(c)𝟐𝒔+𝟏 (d)𝟐𝒔−𝟏 Ans. (c) Replacing x by (- 1/x), we get

1 1 𝑛 𝑎1 𝑎2
Ans. (d) (1 − + 2 ) = 𝑎0 − + 2 + ⋯
𝑥 𝑥 𝑥 𝑥
s= ∑∞ 𝑛 2 3 1 𝑎2𝑛
𝑛=0 𝑟 = 1 + 𝑟 + 𝑟 + 𝑟 + ⋯ 𝑡𝑜 ∞ = − 𝑎2𝑛−1 . 2𝑛−1 + 2𝑛
1 𝑥 𝑥
1−𝑟
or, (1 − 𝑥 + 𝑥 2 )𝑛 = 𝑎0 𝑥 2𝑛 − 𝑎1 𝑥 2𝑛−1 +
1 𝑠−1
∴ r=1- = 𝑎2 𝑥 2𝑛−2 + ⋯ + 𝑎2𝑛 ……………….(1)
𝑠 𝑠

∑∞ 2𝑛 1 1 𝑠2 And given (1 + 𝑥 + 𝑥 2 )𝑛 = 𝑎0 + 𝑎1 𝑥 +
𝑛=0 𝑟 = 1−𝑟 2 = (𝑠−1)2
= 2𝑠−1
1− 2
𝑠 ⋯ + 𝑎2𝑛 𝑥 2𝑛 …………………(2)
𝟐 𝟒 𝟐𝒏
8. The limit of the product √𝟓,√𝟓,…. √𝟓 Multiplying corresponding sides of (1) and
as n→∞ is (2), we have

(a) 𝟓
𝟏
(b) 𝐥𝐨𝐠 𝟏𝟎 𝟓 (1 + 𝑥 2 + 𝑥 4 )𝑛 = (𝑎0 + 𝑎1 𝑥 + 𝑎2 𝑥 2 +
⋯ + 𝑎2𝑛 𝑥 2𝑛 ) × (𝑎0 𝑥 2𝑛 − 𝑎1 𝑥 2𝑛−1 +
(c) 1 (d) 5
𝑎2 𝑥 2𝑛−2 + ⋯ + 𝑎2𝑛 ) …….. (3)
Ans. (d) Required limit=
1 1 1 1 1 1 1 1 (1 + 𝑥 2 + 𝑥 4 )𝑛 = (𝑎0 + 𝑎1 𝑥 2 + 𝑎2 𝑥 4 +
+ + +⋯+ 𝑛
Lt 5 . 5 . 5 … . 5
2 4 8 2𝑛 = Lt 5 2 4 8 2 = ⋯ + 𝑎𝑛 𝑥 𝑛 + ⋯ + 𝑎2𝑛 𝑥 4𝑛 ) ……………..(4)
𝑛→∞ 𝑛→∞
1
5 2
1 =5 Equating coefficient of 𝑥 2𝑛 on both sides of
1−
2
(3) and (4)
9. If numbers p, q, r are in A.P. , then
𝑎0 2 − 𝑎1 2 + 𝑎2 2 … . . +𝑎2𝑛 2 = 𝑎𝑛 .
𝒎𝟕𝒑 , 𝒎𝟕𝒒 , 𝒎𝟕𝒓 (m>0) are in
11. The set of all real number x such that
(a) A.P. (b) G.P.
||3-x|-|x+2||=5 is
(c) H.P. (d)none of
these (a) [3, ∞) (b) (-∞, −𝟐]
(c) (-∞, −𝟐] ∪[3, ∞) (d) (-
𝑚7𝑞 𝑚7𝑟
Ans. (b) 𝑚7𝑝 = 𝑚7(𝑞−𝑝) , 𝑚7𝑞 = 𝑚7(𝑟−𝑞) ∞, −𝟑] ∪[2, ∞)

37
Solving Mathematical Problems

Ans. (c) (||3 − x| − |x + 2||)2 = 25 (a) |y(x)|⟶∞ 𝒂𝒔 𝒙 → ∞


(b) |y(x)|⟶𝟎 𝒂𝒔 𝒙 → ∞
⤇(3 − x)2 + (x + 2)2 − 2|3-x||x+2|= 25
(c) 𝐥𝐢𝐦 |𝐲(𝐱)| exists & is finite
⤇ x 2 − 𝑥 − |−x 2 + 𝑥 + 6| = 6 𝒙→±∞
(d) none
2
So, it is clear that −x + 𝑥 + 6 < 0 , i.e.
−x 2 + 𝑥 + 6 ≥ 0 Ans. (a) 𝑚2 + 2𝑐𝑚 + 𝑘 = 0

−2𝑐±√4𝑐 2 −4𝑘 −2𝑐±√4 ( 𝑐 2 −𝑘)


(x-3)(x+2)≥ 0 ∴m= = =
2 2
−2𝑐±2𝑎
So, x ≤ −2 & 𝑥 ≥ 3 [∵ 𝑐 − 𝑘 = 𝑎2 = 𝑐 − 𝑘]
2 2
2

∴x ∊ (-∞, −2] ∪[3, ∞) −𝑐−𝑎 −𝑐+𝑎


= ,
2 2

The general solution of the given L.D.E. is y


𝑐+𝑎
12. The differential equation of the system = 𝑐1 𝑒 𝑚1 𝑥 + 𝑐2 𝑒 𝑚2 𝑥 = 𝑐1 𝑒 −( 2
)𝑥
+
of circle touch the y – axis at the origin is −(
𝑐−𝑎
)𝑥
𝑐2 𝑒 2

𝒅𝒚
(a) 𝒙𝟐 + 𝒚𝟐 − 𝟐𝒙𝒚 𝒅𝒙= 0 So, |y(x)|⟶∞ 𝑎𝑠 𝑥 → ∞
𝒅𝒚
(b) 𝒙𝟐 + 𝒚𝟐 + 𝟐𝒙𝒚 𝒅𝒙=0
14. Let y be a function of x satisfying
𝒅𝒚
𝟐 𝟐 𝒅𝒚 = 𝟐𝒙𝟑 √𝒚 − 𝟒𝒙𝒚. If y(0)= o and then
(c) 𝒙 − 𝒚 − 𝟐𝒙𝒚 𝒅𝒙 = 0 𝒅𝒙

𝒅𝒚 y(1)equals
(d) 𝒙𝟐 − 𝒚𝟐 + 𝟐𝒙𝒚 𝒅𝒙 = 𝟎
𝟏
(a) (b) 1/e
𝟒𝒆𝟐
Ans. (d) 𝑥 2 + 𝑦 2 − 2𝑎𝑥= 0
(c) 𝒆𝟏/𝟐 (d) 𝒆𝟑/𝟐
𝑑𝑦
2x+2y𝑑𝑥 − 2𝑎=0 𝑑𝑦
Ans. (a) 𝑑𝑥 (4𝑥)𝑦=2√𝑦𝑥 3 (Bernoulli’s
𝑑𝑦 𝑥 2 +𝑦 2 Equation)
⤇2(x+ y𝑑𝑥 )=2 ( )
2𝑥

𝑑𝑦 Putting√𝑦 = 𝑧, the equation reduces to


⤇2𝑥 2 + 2𝑥𝑦 𝑑𝑥 = 𝑥 2 + 𝑦 2
𝑑𝑧
𝑑𝑦
+ (2𝑥)𝑧= 𝑥 3 (linear in z)
2 2 𝑑𝑥
⤇𝑥 − 𝑦 + 2𝑥𝑦 𝑑𝑥 = 0.
2
∴ I. F.= e∫ 2𝑥𝑑𝑥 = 𝑒 𝑥
13. Let y(x) be a non-trivial solution of the
second order liner differential equation Multiplying and integrating

𝒅𝟐 𝒚 2 2
𝒅𝒚
+ 𝟐𝒄 𝒅𝒙 + 𝒌𝒚 = 𝟎, 𝒘𝒉𝒆𝒓𝒆 𝒄 < 0, 𝑘 > z𝑒 𝑥 = ∫ 𝑥 3 𝑒 𝑥 𝑑𝑥 (put 𝑥 2 = 𝑢)
𝒅𝒙𝟐
0, 𝒄𝟐 − 𝒌. Then 1 2
= 2 (𝑥 2 − 1)𝑒 𝑥 + 𝑐

38
Solving Mathematical Problems

∴ General solution is given by:- √𝑦 = ⇒𝑥 𝑥 𝑦 𝑦 𝑧 𝑧 × (𝑥 𝑥 𝑦 𝑦 𝑧 𝑧 )2 ≥


1
(𝑥 2 − 1) + 𝑐𝑒 −𝑥
2 𝑥 𝑥+𝑦+𝑧 . 𝑦 𝑥+𝑦+𝑧 . 𝑧 𝑥+𝑦+𝑧
2

1
⇒ (𝑥 𝑥 𝑦 𝑦 𝑧 𝑧 )3 ≥ (𝑥𝑦𝑧)3𝑎
Since y(0)=0, so, c= 2
⇒ 𝑥 𝑥 𝑦 𝑦 𝑧 𝑧 ≥ (𝑥𝑦𝑧)𝑎
1 1
∴y(1)= (2𝑒)2 = 4𝑒 2.
17. The number of integers between 1
15. Let 𝒙𝒊 are non -ve reals and s= 𝒙𝟏 + and 567 are divisible by either 3 or 5, is
𝒙𝟐 + ⋯ + 𝒙𝒏 , 𝒕𝒉𝒆𝒏 𝒙𝟏 𝒙𝟐 + 𝒙𝟐 𝒙𝟑 + … + (a) 200 (b) 250
𝒙𝒏−𝟏 𝒙𝒏 ≤ (c) 300 (d) none
𝒔𝟐 𝒔𝟐
(a) (b) Ans. (d) Let z= {1, 2, 3, …., 566, 567}
𝟐 𝟑
𝒔𝟐 𝑧
(c) 𝟒
(d) none P = {x ∊ 3 𝑑𝑒𝑣𝑖𝑑𝑒𝑠 𝑥} and

𝑧
Ans. (c) (𝑥1 + 𝑥3 + 𝑥5 + ⋯ )(𝑥2 + 𝑥4 + Q = { x ∊ 5 𝑑𝑒𝑣𝑖𝑑𝑒𝑠 𝑥 }
𝑥6 + ⋯)≥ 𝑥1 𝑥2 + 𝑥2 𝑥3 + … + 𝑥𝑛−1 𝑥𝑛
Here, |P|= 189 [∵ 567= 189× 3]
As when expanding LHS, we must get RHS
and many additional non- negative terms And |Q|= 113 [∵ 567= 113× 5 + 2]
since 𝑥𝑖 ≠ 0.
P ∩ 𝑄= set of multiple of both 3 and 5,
Thus maximum achieved by taking 𝑥1 =
𝑥, 𝑥2 = 𝑠 − 𝑎 and all other terms 0, but | P ∩ 𝑄 |=37; |P ∪ 𝑄|= 189+ 113- 37= 265.

𝑠2 𝑠 18. Sets A and B have 3 and 6 elements


x(s-x) ≤ with equality when x = 2 (using respectively. The minimum number of
4
AM ≥ GM ) elements in A ∪B is

16. For any positive reals x, y, z and a is (a) 3 (b) 6


the arithmetic mean of x, y, z then 𝒙𝒙 𝒚𝒚 𝒛𝒛 (c) 9 (d) none
is
Ans. (b) n(A ∪ B)≥ max{𝑛 (𝐴), 𝑛 (𝐵)}
(a) ≥ (𝒙𝒚𝒛)𝒂 (b) < (𝒙𝒚𝒛)𝒂
(c) >(𝒙𝒚𝒛)𝒂 (d) none Thus n (A ∪ B)≥ max {3, 6} = 6.

Ans. (a) Let ≥ 𝑦 ≥ 𝑧 , then 𝑥 𝑥 𝑦 𝑦 ≥ 𝑥 𝑦 𝑦 𝑥 , 19. A has n elements. How many (B, C)
𝑥 𝑥 are such that ≠ 𝑩 ⊆ 𝑪 ⊆ 𝑨 ?
as (𝑦) 𝑥 ≥ (𝑦) 𝑦 is obviously true.
(a)𝟐𝒏 (b) 𝟑𝒏 (c)
𝑦 𝑧 𝑧 𝑦 𝑧 𝑥 𝑥 𝑧
Similarly, 𝑦 𝑧 ≥ 𝑦 𝑧 𝑎𝑛𝑑𝑧 𝑥 ≥ 𝑧 𝑥 𝟒𝒏 (d) none
Multiplying all these, (𝑥 𝑥 𝑦 𝑦 𝑧 𝑧 )2 ≥ 𝑛
Ans. (b) There are (𝑚 ) choices for a subject
𝑥 𝑦+𝑧 . 𝑦 𝑧+𝑥 . 𝑧 𝑥+𝑦 B with m elements.

39
Solving Mathematical Problems

Then each of the remaining n-m elements 0 −1/3 0

can be in C or not, so there are 2𝑛−𝑚 ∫ 𝑓(𝑥) 𝑑𝑥 = ∫ 𝑑𝑥 + ∫ (−1) 𝑑𝑥


−10 −10 −1/3
choices for C 1
−3 0
= [𝑥] − (𝑥) 1
Thus the total no of pairs (B, C) is −10 −3
∑ 2𝑛−𝑚 . 𝑛𝑐𝑚 = ∑ 2𝑚 . 𝑛𝑐𝑚 = (1 + 2)𝑛 = 3𝑛 1 1
(from binomial theorem) [∵𝑛𝑐𝑚 = 𝑛𝑐𝑛−𝑚 ] = (− + 10) − (0 + )
3 3
2
20. The value of the integral = 10 −
𝟐[𝒙]
3
𝟎 |𝟑𝒙−[𝒙]| 𝝅
∫−𝟏𝟎 𝟐[𝒙] dx, where [.] denotes greatest 𝒃 𝐬𝐢𝐧 𝒙
𝟑𝒙−[𝒙] 21. The equation ∫𝟒𝝅 (𝒂|𝐬𝐢𝐧 𝒙| + 𝟏+𝐜𝐨𝐬 𝒙 +

𝟒
integer function is
𝒄) 𝒅𝒙= 0 gives a relation between
(a) 0 (b) 10
(c) -10 (d) none of (a) a, b and c (b) a and b
these (c) b and c (d) a and c
𝜋
Ans. (d) Ans. (d) I = 2a∫04 |sin 𝑥| 𝑑𝑥 + 0 +
𝜋 𝜋
2[𝑥] 𝜋
| | 4
Let 𝑓(𝑥) = 3𝑥−[𝑥] ∫ 𝑐 𝑑𝑥 = 2𝑎 ∫04 𝑠𝑖𝑛𝑥 𝑑𝑥 + 𝑐. 2

𝜋
2[𝑥] 4
3𝑥−[𝑥]
𝜋
𝜋
Clearly f is not defined if x= 0 and when = −2𝑎[𝑐𝑜𝑠𝑥] 4 + 𝑐
3x= [x] 0 2
1 𝜋
1 = −2𝑎 ( − 1) + 𝑐
So in (-10, 0), f is not defined at x= − 3 . √2 2

1 22. Let f(x) = max. {2- x, 2, 1+ x} then


When x∊(−10, − 3) 𝟏
∫−𝟏 𝒇(𝒙) 𝒅𝒙=
[x] < 0 and 3x- [x]< 0
(a) 0 (b) 2 (c)
[𝑥] 9/2 (d) none of these
So, 3𝑥−[𝑥] > 0 ⟹ 𝑓(𝑥) = 1

1
Ans. (c) ∴ f(x) = 2-x, x≤ 0
When x ∊ (− , 0)
3
= 2, 0≤x≤1
[x] < 0 and 3x- [y] > 0⟹ f(x)= -1
= 1+ x, x ≥1
1 0
I= ∫−1 𝑓(𝑥) 𝑑𝑥 = ∫−1 𝑓(𝑥)𝑑𝑥 +
1 0 1
∫0 𝑓(𝑥)𝑑𝑥 = ∫−1(2 − 𝑥)𝑑𝑥 + ∫0 2 𝑑𝑥

40
Solving Mathematical Problems

𝑥2 0 1 Since y is independent of x, therefore y has


= [2𝑥 − ] + 2[𝑥]
2 −1 0 same value for all x.
1 𝑥+10
= 0 − (−2 − ) + 2(1 − 0) ∴∫𝑥 𝑓(𝑥) 𝑑𝑥= 0
2
9
= 𝟏 𝟐
25. If ∫𝟎 𝒙𝒆𝒙 𝒅𝒙 = 𝒌 ∫𝟎 𝒆𝒙 𝒅𝒙, then
𝟏 𝟐
2
23. Let f(x) be a continuous function such (a) k > 1 (b) 0 < k < 1
that f(a-x)+f(x)=0 for all x ∊[0, a]. (c) k=1 (d) none
𝒂 𝒅𝒙
Then ∫𝟎 equals Ans. (b) Here 0 < x < 1
𝟏+𝒆𝒇(𝒙)
2 2 1 2
(a) a (b) a/2 (c) ½ ⟹0< x𝑒 𝑥 < 𝑒 𝑥 ⟹ 0 < ∫0 𝑥𝑒 𝑥 𝑑𝑥 <
f(a) (d) none of these 1 2 1 2 1 2
∫0 𝑒 𝑥 𝑑𝑥 ⟹ 0 < 𝑘 ∫0 𝑒 𝑥 𝑑𝑥 < ∫0 𝑒 𝑥 𝑑𝑥
Ans. (b) Given, f(a- x)= - f(x)
⟹0<𝑘
𝑎 𝑑𝑥 𝑎 𝑑𝑥 1
2
Now 2I= ∫0 + ∫0 = <1 [𝑑𝑖𝑣𝑖𝑑𝑖𝑛𝑔 𝑏𝑦 ∫ 𝑒 𝑥 𝑑𝑥]
1+𝑒 𝑓(𝑥) 1+𝑒 𝑓(𝑎−𝑥)
𝑎 𝑑𝑥 𝑎 𝑑𝑥 𝑎 0
∫0 + ∫0 1+𝑒 −𝑓(𝑥) = ∫0 𝑑𝑥 = 𝑎
1+𝑒 𝑓(𝑥)
26. Consider the parabola 3𝒚𝟐 − 𝟒𝒚 −
𝑎 𝟔𝒙 + 𝟖=0. The points on the axis of this
∴𝐼=
2 parabola from where 3 distinct normals
24. Let f(x) be an integrable odd function can be drawn are given by
in [-5, 5] such that f(10+ x)= f(x), 𝟐 𝟐𝟗
(a) (𝟑 , 𝒉) , 𝒘𝒉𝒆𝒓𝒆 𝒉 > 𝟏𝟖
𝟏𝟎+𝐱
then ∫𝐱 𝐟(𝐭)𝐝𝐭 equals 𝟏
(b) (𝒉, 𝟑) , 𝒘𝒉𝒆𝒓𝒆 𝒉 >
𝟏𝟗
𝟏𝟖
𝟓
(a) 0 (b) 2∫𝐱 𝐟(𝐱)𝐝𝐱 𝟐 𝟐𝟗
(c) (𝒉, 𝟑) , 𝒘𝒉𝒆𝒓𝒆 𝒉 > 𝟏𝟖
(c) > 0 (d) none of these
(d) none of these
𝑥+10
Ans. (a) Let y= ∫𝑥 𝑓(𝑡) 𝑑𝑡…… (1) Ans. (c)
𝑑𝑦
Then, 𝑑𝑥 = 𝑓(𝑥 + 10). 1 − 𝑓(𝑥) = 0 [∵ 2 2 10
Given parabola is (𝑦 − ) = 2 (𝑥 − )
3 9
f(10+x)= f(x)]
10 2
Let X= 𝑥 − ,𝑌 = 𝑦 − 3
∴ y is independent of x. 9

Putting x= -5 in (1), we get ∴𝑌 2 = 2𝑥 becomes the equation of parabola


with reference to the new origin.
5
y= ∫−5 𝑓(𝑡) 𝑑𝑡 = 0 …………. (2)
Hence equation of normal will be

41
Solving Mathematical Problems

𝑚3 ∴ From (A), a(𝑥1 + 𝑥2 ) + 𝑏 = 2𝑎𝑥3 + 𝑏


Y= mX – m- 𝑥 − 2
𝑥1 +𝑥2
[∵ three normals are drawn from point on ⟹ = 𝑥3
2
the axis (H, 0) (say)]
28. Let P (𝛼,β) be any point on parabola
𝑚2 𝒚𝟐 = 𝟒𝒙(𝟎 ≤ 𝜷 ≤ 𝟐). M is the foot of
∴ H= 1 + ⟹ m= ±√2𝐻 − 1
2 perpendicular from the focus S to the
For m to be real, H > ½ tangent at P, then the maximum value of
area of
10 1 29
⟹ℎ− > 2 ⟹ ℎ > 18
9 (a) 1 (b) 2 (c)
𝝅 𝝅
[where h is the abscissa w.r.t. the previous (d)
𝟑 𝟔
co-ordinate system]
Ans. (a) Let 𝛼= 𝑡 2 , 𝛽 = 2𝑡
Hence the points are given by
2 29 ∴ 0≤ 2t ≤ 2 ⟹ 0≤ t ≤ 1
(ℎ, 3) , 𝑤ℎ𝑒𝑟𝑒 ℎ > 18.
Equation of tangent at (𝑡 2 , 2𝑡) 𝑖𝑠 𝑦𝑡 = 𝑥 +
27. A (𝒙𝟏 , 𝒚𝟏 ) and B (𝒙𝟐 , 𝒚𝟐 ) are any two 𝑡2
points on the parabola y= c𝒙𝟐 + 𝒃𝒙 + 𝒂.
If P(𝒙𝟑 , 𝒚𝟑 ) be the point on the arc AB If S be the focus, then S ≡(1, 0)
where the tangent is parallel to the chord |1+𝑡 2 |
AB, then SM= √1+𝑡 2 = √1 + 𝑡 2

(a) 𝒙𝟐 is the A.M. between 𝒙𝟏 𝒂𝒏𝒅 𝒙𝟑 PS= √(𝑡 2 − 1)2 + 4𝑡 2 = (𝑡 2 + 1)


(b) 𝒙𝟐 is the G.M. between 𝒙𝟏 𝒂𝒏𝒅 𝒙𝟑
PM= √𝑃𝑆 2 − 𝑆𝑀2 = 𝑡 √𝑡 2 + 1
(c) 𝒙𝟐 is the H.M. between 𝒙𝟏 𝒂𝒏𝒅 𝒙𝟑
(d) none of these Area of 𝛥 PMS= ½ .PM.SM= ½
.t√𝑡 2 + 1. √𝑡 2 + 1
Ans. (d) Slope of tangent at p=
𝑑𝑦 𝑦 −𝑦 𝑡(𝑡 2 +1)
𝑎𝑡 (𝑥3 , 𝑦3 ) = 2𝑎𝑥3 + 𝑏 = 𝑥2−𝑥1 =
𝑑𝑥 2 1 2
[given]……….(A)
Which is an increasing function hence its
∵ A and B lie on the parabola, maximum value occurs at t= 1

∴𝑦1 = 𝑎𝑥1 2 + 𝑏𝑥1 + 𝑐 ………………(1) ∴ Maximum area= 1 sq. unit.

And 𝑦2 = 𝑎𝑥2 2 + 𝑏𝑥2 + 𝑐 ……………..(2)

∴ 𝑦1 − 𝑦2 = [𝑎(𝑥1 + 𝑥2 )(𝑥1 − 𝑥2 ) + 29. The point A on the parabola 𝒚𝟐 = 𝟒𝒙


𝑏](𝑥1 − 𝑥2 ) for which |AC-AB | is maximum, where
𝑦 −𝑦
∴ 𝑥2−𝑥1 = 𝑎 (𝑥1 + 𝑥2 ) + 𝑏 B≡ (𝟎, 𝒂) 𝒂𝒏𝒅 𝑪 ≡ (−𝒂, 𝟎) is
2 1

42
Solving Mathematical Problems

(a) (a, 2a) (b) (4a, 4a) ISI OBJECTIVE SAMPLE PAPER
(c) (a- 2a) (d) none of WITH SOLUTIONS
these
SET – 6
Ans. (a)

For any three points A, B, and C

|AC - AB|≤ BC There will be 30 questions in MMA Paper.


For each question, exactly one of the four
∴ required point A will be on the choices is correct. You get four marks for
intersection of BC and the parabola. each correct answer, one mark for each
unanswered question, and zero mark for
∴ A≡ (a, 2a) [∵ AB ia tangent to the each incorrect answer.
parabola]

30. The mean and variance of a binomial 1. If x ∊={1, 2, 3,……, 9}and


variable X are 2 and 1 respectively. If X fn(x)=xxx……x (n digits),then
takes values greater than 1, then its 𝒇𝟐𝒏 (3)+𝒇𝒏 (2) =
probability will be (a) 2𝒇𝟐𝒏 (1)
𝟓 𝟏𝟏 (b) 𝒇𝟐𝒏 (1)
(a) 𝟏𝟔 (b) 𝟏𝟔
𝟏 (c) 𝒇𝟐𝒏 (1)
(c) 𝟒 (d) none of these
(d)−𝒇𝟐𝒏 (4)
Ans. (b)
Ans. (c)
Given, np= 2, npq= 1
𝑓𝑛 (𝑥) = 𝑥. 1 + 𝑥. 102 + 𝑥. 103 +
∴ q= ½ , p= ½ , n= 4 (10𝑛 −1) 𝑥
⋯ 𝑥. 10𝑛−1 = 𝑥 = 9 (10𝑛 − 1)
10−1
Now p(X> 1)= 1- P(X≤ 1) 2
3
∴𝑓𝑛 2 (3) + 𝑓𝑛 (2) = [9 (10𝑛 − 1)] +
= 1- [P(X=0)+P(X= 1)] 2 1
(10𝑛 − 1) = 9 (10𝑛 − 1)(10𝑛 − 1 + 2) =
9
1 4 10𝑛 −1
= 1- [4𝐶0 𝑝0 𝑞 4 + 4𝐶1 𝑝1 𝑞 3 ] = 1 − [(2) + = 𝑓2𝑛 (1)
9
1 1 3 5 11
4. 2 (2) ] = 1 − 16 = 16 2. If 𝒂𝒊 ∊R-{0}, i=1, 2, 3, 4 and x∊R and
(∑𝟑𝒊=𝟏 𝒂𝟐𝒊 )𝒙𝟐 - 2x(∑𝟑𝒊=𝟏 𝒂𝒊 𝒂𝒊 +1) + ∑𝟒𝒊=𝟐 𝒂𝟐𝒊 ≥
0,

Then 𝒂𝟏 ,𝒂𝟐 ,𝒂𝟑 ,𝒂𝟒 are in

(a) A.P. (b) G.P.


(c) H.P. (d) none of these

43
Solving Mathematical Problems

Ans. (b) 1 𝑛
21= ∑𝑘𝑛=1 [3 + 90] , 𝑤ℎ𝑒𝑟𝑒 𝑚 = 𝑘!
Given quadratic expression≥ 0 1 1 1 2 1 59
= [3 + 90] + [3 + 90] + ⋯ + [3 + 90] +
∴D≤0 1 60 1 61 1 𝑘
[3 + 90] + [3 + 90] + ⋯ + [3 + 90]
⟹(∑3𝑖=1 𝑎𝑖 𝑎𝑖 + 1)2 −
(∑3𝑖=1 𝑎𝑖 2 )(∑4𝑖=1 𝑎𝑖 2 ) ≤ 0 = (0 + 0 + ⋯ 𝑡𝑜 59 𝑡𝑒𝑟𝑚𝑠). +(1 + 1 +
⋯ 𝑡𝑜 (𝑘 − 59)𝑡𝑒𝑟𝑚𝑠)
⟹(𝑎1 𝑎2 + 𝑎2 𝑎3 + 𝑎3 𝑎4 )2 − (𝑎1 2 + 𝑎2 2 +
𝑎3 2 )(𝑎2 2 + 𝑎3 2 + 𝑎4 2 ) ≤ 0 ∴ 21= k- 59⟹ k= 80.

⟹(𝑎2 2 − 𝑎1 𝑎3 )2 + (𝑎3 2 − 𝑎2 𝑎4 )2 + 5. Let f: R→R such that f(x) is


continuous and attains only rational value
(𝑎2 𝑎3 − 𝑎1 𝑎4 )2 = 0
at all real x and f(3)=4. If
⟹(𝑎2 2 − 𝑎1 𝑎3 )2 = 0, (𝑎3 2 − 𝑎2 𝑎4 )2 = 𝒂𝟏 ,𝒂𝟐 ,𝒂𝟑 ,𝒂𝟒 ,𝒂𝟓 are in H.P., then
0, (𝑎2 𝑎3 − 𝑎1 𝑎4 )2 = 0 ∑𝟒𝒓=𝟏 𝒂𝒓 𝒂𝒓+𝟏 =
𝑎 𝑎 𝑎
⟹𝑎2 = 𝑎3 = 𝑎4 (a) f(5).𝒂𝟏 𝒂𝟓 (b) f(3).𝒂𝟒 𝒂𝟓
1 2 3
(c) f(3).𝒂𝟏 𝒂𝟐 (d) f(2).𝒂𝟏 𝒂𝟑
𝟏 𝒓 𝟏
3. Let a = 𝒏! + ∑𝒏−𝟏
𝒓=𝟏 (𝒓+𝟏)!, b = 𝒎! + Ans. (a)
𝒓
∑𝒎−𝟏
𝒓=𝟏 (𝒓+𝟏)! then a+b equals
Since f(x) is continuous and attains only
rational values
(a) 0 (b) 1 (c)2
(d) none of these ∴ f(x)= constant= 4
Ans. (c) ∴ f(2) = f(3) = f(5) = 4
𝑟 𝑟+1−1 1 1
(𝑟+1)!
= (𝑟+1)!
= 𝑟! − (𝑟+1)! Since 𝑎1 , 𝑎2 , 𝑎3 , 𝑎4 , 𝑎5 are in H.P.

𝑟 1 ∴ 𝑎1 𝑎2 + 𝑎2 𝑎3 + 𝑎3 𝑎4 + 𝑎4 𝑎5 =
∴ ∑𝑛−1
𝑟=1 (𝑟+1)! = 1 − 𝑛! ⟹ 𝑎 = 1
4𝑎1 𝑎5 = 𝑓(5). 𝑎1 𝑎5
1 𝑟
Similarly, 𝑚! + ∑𝑚
𝑟=1 (𝑟+1)! = 1 6. If three successive terms of a G.P. with
common ratio r >1 from the sides of a
∴ a= 1, b=1 ⟹a+ b= 2 triangle and [r] denotes the integral part
𝟏 𝒏 of x, then [r] + [-r]=
4. If ∑𝒌𝒏=𝟏[𝟑 + 𝟗𝟎] =21, where [x] denotes
the integral part of x, then k= (a) 0 (b) 1
(c) -1 (d) none
(a) 84 (b) 80
(c) 85 (d) none of these Ans. (b)

Ans. (b) Since root of equation

44
Solving Mathematical Problems

F(x)= 𝑥 2 + 2(𝑎 − 3)𝑥 + 9 = 0 lie between- (a) 5/4 (b) 7


6 and 1 (c) 4 (d) 2

∴(i) D≥ 0 (ii) f(-6)> 0 (iii) f(1) > 0 (iv) - Ans. (c)


𝛼+𝛽 𝛼+𝛽
6< (v) 1 > 𝑥
2 2 Given, F(x)= ∫0 𝑓(𝑡) 𝑑𝑡 ………(1)
27
Hence 6 ≤ 𝛼 < 𝐹(𝑥 2 ) = 𝑥 2 (1 + 𝑥) ………..(2)
4

∴|a| =6 From (1), F’(x)= f(x)


6−2 4 18
𝑎3 = 2 + 3𝑑 = 2 + 3. =2+7= ∴ f(4)= F’(4)…………..(3)
21 7

1 1 From (1),
1 1 − 1 2 3
6 2
= 2 + 18. ( 21 ) = 2 − 7 = 14
ℎ18
F’(𝑥 2 ).2x= 2x+ 3𝑥 2
18 14 2+3𝑥
∴ 𝑎3 ℎ18 = . = 12 ⟹ F’(𝑥 2 ) = [∵ 0 < 𝑥 < ∞ ∴ 𝑥 ≠ 0]
7 3 2

7. If 𝒙𝟏 , 𝒙𝟐 , 𝒙𝟑 , 𝒙𝟒 , 𝒙𝟓 are in H.P. then ⟹ 𝐹 ′ (4) =


2+6
= 4 [Put x= 2]
𝟏 2
(∑𝟒𝒌=𝟏 𝒙𝒌 𝒙𝒌+𝟏 ) is a root of equation
𝒙𝟏 𝒙𝟓
∴ from (3), f(4)= 4
𝟐
(a) 𝒙 − 𝟑𝒙 + 𝟐 = 𝟎 ∞ 𝒅𝒙
(b) 𝒙𝟐 − 𝟓𝒙 − 𝟒 = 𝟎 9. If n > 1 then ∫𝟎 =
(𝒙+√𝟏+𝒙𝟐 ) 𝒏

(c) 𝒙𝟐 − 𝟗𝒙 + 𝟐𝟎 = 𝟎 𝒏 𝒏𝟐 −𝟏
(a) 𝒏𝟐 −𝟏 (b)
(d) 𝒙𝟐 − 𝟔𝒙 − 𝟖 = 𝟎 𝒏
𝒏 𝟏−𝒏𝟐
(c) − 𝒏𝟐 −𝟏 (d)
Ans.(c) 𝒏

Ans. (a)
𝑥1 , 𝑥2 , 𝑥3 , 𝑥4 , 𝑥5 are in H.P.

∴∑4𝑘=1 𝑥𝑘 𝑥𝑘+1 = 𝑥1 𝑥2 + 𝑥2 𝑥3 + 𝑥3 𝑥4 + Put z= x+√1 + 𝑥 2


𝑥4 𝑥5 = 4𝑥1 𝑥5
∴ z- x= √1 + 𝑥 2 => 𝑧 2 + 𝑥 2 − 2𝑧𝑥 = 1 +
1 𝑧 2 −1
∴𝑥 ∑4𝑘=1 𝑥𝑘 𝑥𝑘+1 = 4 𝑥 2 => 𝑥 =
1 𝑥5 2𝑧

Clearly, 4 is a root of equation 1 [𝑧. 2𝑧 − (𝑧 2 − 1). 1]


∴ 𝑑𝑥 = 𝑑𝑧
2 𝑧2
𝑥 2 − 9𝑥 + 20 =0. 𝑧2 + 1
= 𝑑𝑧
2𝑧 2

𝒙
When x= 0, z= 1 and when x= ∞, 𝑧 = ∞
8. Let f : (0, ∞)⟶R and F(x)= ∫𝟎 𝒇(𝒕) 𝒅𝒕
If F(𝒙𝟐 ) = 𝒙𝟐 (𝟏 + 𝒙), 𝒕𝒉𝒆𝒏 𝒇(𝟒) =

45
Solving Mathematical Problems
∞ 1 𝑧 2 +1 1 ∞ then one of the possible value of k is
∴ I= ∫1 𝑑𝑧 = 2 ∫1 (𝑧 −𝑛 +
𝑧𝑛 2𝑧 2
1 𝑧 −𝑛+1 𝑧 −𝑛−1 ∞ (a) -4 (b) 0
𝑧 −𝑛−2 ) 𝑑𝑧 = 2 [ −𝑛+1 + ]
−𝑛−1 1
(c) 2 (d) 16
1 1 1 1 2𝑛
= [0 − ( − )] = (− ) Ans. (d)
2 1−𝑛 1+𝑛 2 1 + 𝑛2
𝑛
= 2 𝑑 𝑒 sin 𝑥
𝑛 −1 Given, 𝑑𝑥 (𝑓(𝑥)) = ,𝑥 > 0
𝑥

10. If f(x)= ae2x+ bex +cx satisfies the 4 2𝑒 𝑠𝑖𝑛𝑥


2

conditions f(0)= -1, f’(log 2)= 28, 𝑛𝑜𝑤 𝐼 = ∫1 𝑑𝑥 [put z= 𝑥 2 , 𝑑𝑧 =


𝑥
2𝑥 𝑑𝑥]
𝐥𝐨𝐠 𝟒 𝟑𝟗
∫𝟎 [𝒇(𝒙) − 𝒄𝒙]𝒅𝒙 = 𝟐
, then 2
4 2𝑒 𝑠𝑖𝑛𝑥 16 𝑒 sin 𝑧
∴ I= ∫1 𝑑𝑥 = ∫1 𝑑𝑧 =
𝑥2 𝑧
(a) a= 5, b=6, c= 3 (b) a= 5, b= - 6,
[𝑓(𝑧)] 16 = 𝑓(16) − 𝑓(1)
c= 0 (c) a= -5, b=6, c= 3 (d) 1

none

Ans. (b)
∴ f(k)= f(16)
Given f(x)= 𝑎𝑒 2𝑥 + 𝑏𝑒 𝑥 + 𝑐𝑥 ……(1)
∴ one possible value of k= 16
𝑔𝑖𝑣𝑒𝑛, 𝑓(0) = −1 ⟹ 𝑎 + 𝑏 = −1…..(2) 𝟐
12. All the values of a for which ∫𝟏 [𝒂𝟐 +
f’(x)= 2 𝑎𝑒 2𝑥 + 𝑏𝑒 𝑥 + 𝑐𝑥 (𝟒 − 𝟒𝒂)𝒙 + 𝟒𝒙𝟑 ] 𝒅𝒙 ≤ 𝟏𝟐 are given by

∴f’ (log 2)= 𝑎𝑒 log𝑒 4 + 𝑏𝑒 log𝑒 2 + 𝑐 (a) a= 3 (b) a ≤ 𝟒


(c) 0≤ 𝒂 ≤ 𝟑 (d) none
Given 8a+ 2b+c= 28……….(3)
of these
log 4 39
Given, ∫0 (𝑎𝑒 2𝑥 + 𝑏𝑒 𝑥 ) 𝑑𝑥 = Ans. (a)
2

𝑎 log 4 39 2
⟹ [ 𝑒 2𝑥 + 𝑏𝑒 𝑥 ] = ∫ [𝑎2 + (4 − 4𝑎)𝑥 + 4𝑥 3 ] 𝑑𝑥
2 0 2 1
𝑎 𝑎 39 2 2
⟹ 2 𝑒 log 16 + 𝑏𝑒 log 4 − (2 + 𝑏) = = 𝑎2 [𝑥] + (2 − 2𝑎)[𝑥 2 ]
2 1 1
2
⟹ 15a + 6b= 39……….(4) + [𝑥 4 ]
1
Thus a= 5, b= -6, c= 0 = 𝑎2 + (2 − 2𝑎)3 + 15 , 𝐺𝑖𝑣𝑒𝑛, 𝑎2
𝒅 𝒆𝐬𝐢𝐧 𝒙
− 6𝑎 + 21 ≤ 12
11. Let 𝒇(𝒙) = ,𝒙 > ⟹ 𝑎 − 6𝑎 + 9 ≤ 0 ⟹ (𝑎 − 3)2
2
𝒅𝒙 𝒙
𝟐
𝟒 𝟐𝒆𝐬𝐢𝐧 𝒙 ≤ 0 ⟹ (𝑎 − 3)2 = 0
0. 𝐼𝑓 ∫𝟏 𝒅𝒙 = 𝒇(𝒌) − 𝒇(𝟏),
𝒙 ⟹𝑎=3

46
Solving Mathematical Problems

(𝟐𝒓)𝒌 1
13. 𝐋𝐭 ∑𝒏𝒓=𝟏 𝒏𝒌+𝟏 , 𝒌 ≠ −𝟏, is equal to 1
𝒏→∞ =∫ 𝑑𝑥
0 √𝑥 (3√𝑥 + 4)2
𝟐𝒌 𝟐𝒌
(a) 𝒌−𝟏 (b) 3
𝒌 Put z= 3√𝑥 + 4, then dz= 2 𝑑𝑥
√𝑥
𝟏 𝟐𝒌
(c) 𝒌−𝟏 (d) 𝒌+𝟏
When x= 0, z= 4, when x= 1, z= 7
Ans. (d) 2 7 𝑑𝑧 2 1 2 1
∴Reqd. limit= 3 ∫4 = 3 [− 𝑧] 74 = − 3 [7 −
𝑧2
(2𝑟)𝑘
Reqd. limit= Lt ∑𝑛𝑟=1 𝑛𝑘+1 = 1 2 3
] = − 3 (− 28) = 14
1
𝑛→∞ 4
𝑟𝑘 𝑟 𝑘
Lt 2𝑘 ∑𝑛𝑟=1 𝑛𝑘 .𝑛 = Lt 2𝑘 ∑𝑛𝑟=1 (𝑛) =
𝑛→∞ 𝑛→∞ 16. If f(x)= excosx.sinx, |x|≤ 𝟐 =
1 1 𝑥 𝑘+1 2𝑘 𝟑
= 2𝑘 ∫0 𝑥 𝑘 𝑑𝑥 = 2𝑘 . [ 𝑘+1 ] 10 = 𝑘+1 𝟐, 𝒐𝒕𝒉𝒆𝒓𝒘𝒊𝒔𝒆 𝒕𝒉𝒆𝒏 ∫𝟐 𝒇(𝒙)𝒅𝒙 is equal to
𝑛

𝒏! 𝟏 (a) 0 (b) 1
14. 𝐋𝐭 {(𝒌𝒏)𝒏 } 𝒏 , 𝒌 ≠ 𝟎, is equal to
𝒏→∞ (c) 2 (d) 3
𝒌 𝒆
(a) 𝒆 (b) 𝒌 Ans. (c)
𝟏
(c) 𝒌𝒆 (d) none of 3 2 3
∫ 𝑓(𝑥) 𝑑𝑥 = ∫ 𝑓(𝑥) 𝑑𝑥 + ∫ 𝑓(𝑥) 𝑑𝑥
these −2 −2 2
2
Ans. (c) = ∫ 𝑒 cos 𝑥 . sin 𝑥 𝑑𝑥
−2
3
1 𝑛! 1/𝑛 1 𝑛! 1/𝑛 1
Let P= Lt ( ) = 𝑘 Lt (𝑛𝑛 ) = 𝑘𝑒 + ∫ 2 𝑑𝑥 = 0 + 2(3 − 2),
𝑛→∞ 𝑘 𝑛𝑛 𝑛→∞
2

= 2 [∵𝑒 cos 𝑥 . sin 𝑥 𝑖𝑠 𝑎𝑛 𝑜𝑑𝑑 𝑓𝑢𝑛𝑐𝑡𝑖𝑜𝑛 =


√𝒏 2]
15. 𝐋𝐭 ∑𝒏𝒓=𝟏 ( =
𝒏→∞ √𝒓 (𝟑√𝒓+𝟒√𝒏)𝟐
18. The area of the region enclosed by the
𝟏 𝟏
(a) 𝟕 (b) 𝟏𝟎 curves y= xex and y= xe –x and the line x=
𝟏 1, is
(c) 𝟏𝟒 (d) none of these
(a) 1/e (b) 1- 1/e
Ans. (c) (c) 2/e (d) 1- 2/e
Required limit Ans. (c)
√𝑛
= Lt ∑𝑛𝑟=1 2 = y=x𝑒 𝑥 …….(1)
𝑛→∞ 𝑟
√𝑟.𝑛(3√𝑛+4)
y= x𝑒 −𝑥 ……(2)
1 1
Lt ∑𝑛𝑟=1 2 .𝑛
𝑛→∞ 𝑟 𝑟
√ (3+√ +4)
𝑛 𝑛
equating y from (1) and (2) we get

47
Solving Mathematical Problems

x𝑒 𝑥 = 𝑥𝑒 −𝑥 ⟹ 𝑥(𝑒 𝑥 − 𝑒 −𝑥 ) = 0 1
Required area = 2∫0 𝑥𝑒 𝑥 𝑥 𝑑𝑥 = 2[𝑥𝑒 𝑥 −
⟹x= 0 𝑒 𝑥 ] 10 = 2[(𝑒 − 𝑒) − (0 − 𝑒 0 )] = 2

1
∴ Required area= ∫0 (𝑦1 − 𝑦2 ) 𝑑𝑥 =
1
∫0 (𝑥𝑒 𝑥 − 𝑥𝑒 −𝑥 ) 𝑑𝑥 = [𝑥𝑒 𝑥 − 20. A bag contains unlimited number of
𝑒 𝑥 — 𝑥𝑒 −𝑥 − 𝑒 −𝑥 ) 10 white, red, black, and blue balls. The
number of ways of selecting 10 balls so
= (𝑒 − 𝑒) − (0 − 𝑒 0 ) that there is at least one ball of each
+ [(𝑒 −1 + 𝑒 −1 ) − (0 + 1)] colour is
2
= (a) 180 (b0 270
𝑒
(c) 192 (d) none
19. The area bounded by y = xe|x| and the
lines |x|=1, y= 0 is Ans. (d) Number of ways= coefficient of
𝑋10 𝑖𝑛 (𝑋 + 𝑋 2 + 𝑋 3 + ⋯ )4
(a) 1 (b) 2
(c) 4 (d) 6 = coefficient of 𝑋10 𝑖𝑛 𝑋 4 (1 − 𝑋)−4

Ans. (b) = coefficient of 𝑋 6 𝑖𝑛 (1 − 𝑋)−4


(6+1)(6+2)(6+3)
For x ≥ 0, curve is y= x𝑒 𝑥 ……..(1) = [∵coefficient of 𝑥 𝑟 𝑖𝑛(1 −
1.2.3
(𝑟+1)(𝑟+2)(𝑟+3)
𝑋)−4 = ]
1.2.3

𝑑𝑦 7×8×9
For curve (1), 𝑑𝑥 = 𝑒 𝑥 (1 + 𝑥) > 0 = 1×2×3 = 84.

∴ y is increasing. 21. The number of ways of selecting r


balls with replacement out of n balls
𝑑2𝑦
= 𝑒 𝑥 (2 + 𝑥) > 0 numbered
𝑑𝑥 2
1, 2, 3, …., 100 such that the largest
∴ curve is convex downward.
numbered selected is 10 is 271, then r=
For x ≤ 0, y= x𝑒 −𝑥
(a) 3 (b) 4
𝑑𝑦 (c) 5 (d) none
∴𝑑𝑥 = 𝑒 −𝑥 (1 − 𝑥) > 0
Ans. (a) from the given condition, we can
∴ y is increasing write
𝑑2 𝑦 10𝑟 − 9𝑟 = 271,
= 𝑒 −𝑥 − 𝑒 −𝑥 (1 − 𝑥) > 0
𝑑𝑥 2
= −𝑒 −𝑥 (2 − 𝑥) < 0 Applying Trial and error method:-

∴ curve is concave downward. r= 1, 10-9= 1

48
Solving Mathematical Problems

r =2, 102 − 92 = 19 No. of ways to predict wrongly exactly r


matches =10𝑐𝑟 . 2𝑟 110−𝑟
r = 3, 103 − 93 = 271
∴ The required number is 310 −
∴ r= 3.
∑4𝑟=1 10𝑐𝑟 . 2𝑟
22. N men and n women sit along a line
24. Let 1 to 20 are placed in any around a
alternatively in x ways and along a circle
circle. Then the sum of some 3
in y ways such that x= 10y, then the
consecutive numbers must be at least
number of ways in which n men can sit at
a round table so that all shall not have (a) 30 (b) 31
same neighbours is (c) 32 (d) none
(a) 6 (b) 12 Ans. (c) Suppose 𝑥1 , 𝑥2 , … . , 𝑥20 be the
(c) 36 (d) numbers placed around the circle. Now the
none mean of the 20 sums of 3 consecutive
𝑥 2.⎿𝑛⎿𝑛
numbers such as (𝑥1 + 𝑥2 + 𝑥3 ), (𝑥2 + 𝑥3 +
Ans. (b) 𝑦 = ⎿𝑛−1 ⎿𝑛 = 2𝑛 𝑥4 ), …..,

⇒ x = 2ny = 10y ⇒ n = 5 (𝑥19 + 𝑥20 + 𝑥21 ), (𝑥20 + 𝑥1 + 𝑥2 ) is


1 3×20×21
1
{3(𝑥1 + 𝑥2 + ⋯ + 𝑥20 )} = =
20 2×20
Hence the required number = 2 × ⎿4= 12.
31.5

Thus from Pigon hole principle that at least


23. A contest consists of predicting the one of the sums must be ≥ 32.
result (win, draw or defeat) of 10
matches. The number of ways in which
one entry contains at least 6 incorrect 25. The number of different seven–digit
results is numbers can be written using only there
digits 1, 2, 3 under the condition that the
(a) ∑𝟏𝟎
𝒓=𝟔 𝟏𝟎𝒄𝒓 . 𝟑
𝒓
(b) digit 2 occurs twice in each number is
∑𝟓𝒓=𝟏 𝟏𝟎𝒄𝒓 . 𝟐𝒓 (c)
(a) 512 (b) 640
∑𝟏𝟎
𝒓=𝟔 𝟏𝟎𝒄𝒓 (d) none
(c) 672 (d) none
Ans. (d) Since total number of ways
Ans. (c) We have to put 2 twice in each
predicting the results of one match is 3 , so
numbers, so any 2 out of the 7 places can be
results of 10 match is 310 , now number of
chosen in 7𝑐2 ways. The remaining 5 places
ways that the result of one match is correct
can be filled with the other two numbers in
is 1 and also number of ways to predict
25 ways.
wrongly of one match is 2 .

49
Solving Mathematical Problems

The required numbers of numbers are 7𝑐2 × Since 𝑖5 is even for only m values of s, by
25 = 672. P.H.P. , one of the m+ 1 , numbers,
𝑖1 , 𝑖2 , … . , 𝑖𝑛 , say it is odd, where t is also
odd. Hence t-𝑖𝑡 is even and the product (1-
𝒌 𝑴−𝒌 𝑴−𝒌 𝑖1 )(2-𝑖2 )….(n-𝑖𝑛 ) is even.
26. The value of {∑𝟏𝟎𝟎
𝒊=𝟎 ( 𝒊 )(𝟏𝟎𝟎−𝒊)(𝟏𝟎𝟎−𝒊)} /
𝑴
(𝟏𝟎𝟎 ), where M - k > 100, k >100, is

−𝟏 √ 𝒏−√𝒏−𝟏
(a)
𝒌
(b)
𝑴
28. The value of ∑∞
𝒏=𝟏 𝐬𝐢𝐧 =
𝑴 𝒌 √𝒏(𝒏+𝟏)
𝒌
(c) 𝑴𝟐 (d) none 𝝅 𝝅
(a) 𝟒 (b) 𝟐
𝑀−𝑘 𝝅 𝝅
𝑘 𝑀−𝑘 𝑀
Ans. (a) {∑100
𝑖=0 ( 𝑖 )(100−𝑖)(100−𝑖 )}/ (100)
(c) - 𝟐 (d) 𝟑

𝑘
𝑘
( 𝑖 )(100−𝑖) 𝑀−𝑘
𝑖(𝑘𝑖)(100−𝑖
𝑀−𝑘
) Ans. (b)
= (𝑀−100)∑100
𝑖=0[ ( 𝑀 ) − ∑100
𝑖=0 𝑀 ]
100 (𝑀−100)(100 )
√𝑛−√𝑛−1
𝑡𝑛 = sin−1 ( )
𝑘 𝑀
√𝑛(𝑛+1)
𝑀 .100 (100 )
𝑘 (100) 𝑀 𝑘
= 𝑀−100 . 𝑀 − 𝑀 = 𝑀. 1 1
(100) (𝑀−100) (100)
∴𝑡𝑛 = sin−1 − sin−1
√ 𝑛 √𝑛+1

1
∴𝑆𝑛 = sin−1 (1) − sin−1
√𝑛+1
27. Let n be an odd positive integer. If
𝒊𝟏 , 𝒊𝟐 , … . , 𝒊𝒏 is a permutation of 1, 2, 3, ∴ 𝑆∞ = Lt 𝑆𝑛 = sin−1(1) − sin−1 (0)
𝑛→∞
…., n.
𝜋 𝜋
Then (1-𝒊𝟏 )(2-𝒊𝟐 )….(n-𝒊𝒏 )is = −0=
2 2
(a) Odd (b) even
(c) prime (d) none
29. The number of ways to give 16
Ans. (b) since n is odd, let n= 2m+ 1, where different things to 3 persons, according as
m is a non-negative integer. A< B < C so that B gets 1 more than A
and C get 2 more than B, is
Then set s ={1, 2, …, n} contains m+ 1 odd
nos, namely 2, 4, …, 2m. (a) 4!5!7! (b)
𝟒!𝟓!𝟕!
𝟏𝟔!
𝟏𝟔!
This is also true for the (c) 𝟒!𝟓!𝟕! (d) none
permutation 𝑖1 , 𝑖2 , … . , 𝑖𝑛 of s.
Ans. (c) Here x+ y +z=16, x= y+1, y= z+2
Consider m+ 1 numbers1 − 𝑖1 , 3 − 𝑖3 , …. n
- 𝑖𝑛 which are of the from r - 𝑖𝑟 , where r is ∴x=4, y=5, z= 7
odd.

50
Solving Mathematical Problems

∴ Required number of ways = 16𝑐4 × 1 1 1


Ans:- (a) + +
16! 1−𝑥 1−𝑦 1−𝑧
12𝑐5 × 7𝑐7 = 4!5!7!
3−2(𝑥+𝑦+𝑧)+ (𝑥𝑦+𝑦𝑧+𝑧𝑥)
= (1−𝑥)(1−𝑦)(1−𝑧)

3−2×2007+4011
30. For how many positive integers n less = (1−𝑥)(1−𝑦)(1−𝑧) = 0
than 17,⎿n+⎿n+1+⎿n+2 is an integral
multiple of 49? (a) 4 2. In a 𝛥ABC, if r= 𝒓𝟐 + 𝒓𝟑 +
(b) 5 (c) 6 𝒓𝟏 , 𝒂𝒏𝒅 𝒂𝒏𝒈𝒍𝒆(𝐴) >
𝝅 𝒔
(d) none 𝒕𝒉𝒆𝒏 𝒕𝒉𝒆 𝒓𝒂𝒏𝒈𝒆 𝒐𝒇 is equal to
𝟑 𝒂

Ans. (b) ⎿n+⎿n+1+⎿n+2= (a) (½, 2) (b) (½, ∞)


⎿n{1+(n+1)+(n+2)(n+1)} (c) (½, 3) (d) (3, ∞)
= ⎿n(𝑛 + 2)2 Ans. (a)

Since 49 divides (𝑛 + 2)2 ⎿n, so either 7 𝑟 = 𝑟2 + 𝑟3 − 𝑟1


devides (n+2) or 49 divides ⎿n. Thus n=5,
Δ Δ Δ Δ
12, 14, 15, 16, i.e. number of integers are 5. = s−b + s−c − s−a
s

1 1 1 1
⟹ + = +
s s−a s−b s−c
ISI OBJECTIVE SAMPLE PAPER
WITH SOLUTIONS 2s − a s(s − a)
⟹ =
2s − b − c (s − b)(s − c)
SET – 7
2s − a A s
⟹ = cot 2 ⟹
a 2 a
1 A s
= (cot 2 + 1) ⟹
There will be 30 questions in MMA Paper. 2 2 a
For each question, exactly one of the four 1
∊ ( , 2)
choices is correct. You get four marks for 2
each correct answer, one mark for each
unanswered question, and zero mark for
each incorrect answer.
3. If 𝒂𝟏 , 𝒂𝟐 , … . , 𝒂𝒏 are positive real nos,
𝒂 𝒂 𝒂 𝒂
then 𝒂𝟏 + 𝒂𝟐 + ⋯ + 𝒂𝒏−𝟏 + 𝒂𝒏 is always
𝟐 𝟑 𝒏 𝟏
1. Let x, y, z be different from 1 satisfying
x+ y +z = 2007, 𝟏⁄
i) ≥ 𝒏 ii) ≤ 𝒏 iii) 𝒏 𝒏 iv) none of
𝟏 𝟏 𝟏 these.
Then the value of 𝟏−𝒙 + + is
𝟏−𝒚 𝟏−𝒛
Ans:- AM ≥ GM gives
𝟏
(a) 0 (b) 1 (c) 2008 (d) 𝟐𝟎𝟎𝟖

51
Solving Mathematical Problems
𝑑𝑡
𝑎1
+ ⋯+
𝑎𝑛−1 𝑎
+ 𝑎𝑛 ≥ 𝑛√𝑎1 . … .
𝑎 𝑎𝑛−1 𝑎𝑛
.𝑎 =1 y𝑒 −𝑡 . (1 + 𝑡) = ∫ 𝑒 −𝑡 . (1 + 𝑡) (1+𝑡) = 𝑒 −𝑡 +
𝑎2 𝑎𝑛 1 2 𝑎𝑛 1
𝑐
𝑎1 𝑎𝑛−1 𝑎𝑛
∴ 𝑎 +⋯+ +𝑎 ≥𝑛
2 𝑎𝑛 1 When y(0) = -1, ⤇ c= 0.

∴y𝑒 −𝑡 . (1 + 𝑡) = −𝑒 −𝑡

4. The maximum possible value of x𝒚𝟐 𝒛𝟑 ∴y= − 1+𝑡


1
∴y(1)= - ½.
subject to the condition xyz ≥ 𝟎 and
6. If the quadratic equation 𝒙𝟐 + 𝒂𝒙 +
x+y+z = 3 is
𝒃 + 𝟏 = 𝟎 has non- zero
i) 1 ii) 𝟏⁄𝟖 iii) 𝟏⁄𝟒 iv)
Integer solutions, then
𝟐𝟕⁄
𝟏𝟔
a) 𝒂𝟐 + 𝒃𝟐 is a prime number
Ans:- x+ y+ z = 3 b) 𝒂𝒃 is prime number
2𝑦 𝑧
c) Both a) and b)
⇒ x. + 3. 3 = 3 d) Neither a) nor b)
2

Applying AM ≥ GM, Ans:- (d) 𝛼+𝛽= -a, 𝛼𝛽= (b+ 1)

𝑥+2.
2𝑦
+3.
𝑧
𝑦 𝑧 ∴ 𝑎2 + 𝑏 2 = (α + β)2 + (αβ − 1)2
So, 2 3
≥ 6√𝑥( 2)2 (3)3
1+2+3
= (𝛼 2 + 1)( 𝛽 2 + 1)
3 x𝑦 2 𝑧 3
⇒ (6)6 ≥ 𝟏⁄ 𝟏⁄
22 .33 7. Let u = (√𝟓 − 𝟐) 𝟑 − (√𝟓 + 𝟐) 𝟑 and
𝟏 𝟏
⇒ x𝑦 2 𝑧 3 ≤ 16.
27 v= (√𝟏𝟖𝟗 − 𝟖) ⁄𝟑 − (√𝟏𝟖𝟗 + 𝟖) ⁄𝟑,

𝒅𝒚 Then for each positive integer n, 𝒖𝒏 +


5. If y(t) is a solution of (1+t) 𝒅𝒕 − 𝒕𝒚 = 𝟏
𝒗𝒏+𝟏 = ?
and y(t) then y(1) equals
(a) -1 (b) 0
(a) ½ (b) e + ½ (c) e+ ½ (c) 1 (d) 2
(d)- ½
Ans:- (b) 𝑢3 = (√5 − 2) − (√5 + 2) −
𝑑𝑦 𝑡 1
Ans:- (d) 𝑑𝑡 − 1+𝑡 𝑦 = 1+𝑡 1⁄
3
1⁄
3
3(√5 − 2) (√5 + 2) . (𝑢)
𝑡
− ∫ 𝑑𝑡
∴I.F. = 𝑒 1+𝑡 = 𝑒 −(𝑡−log(1+𝑡)) = i.e. 𝑢3 = −4 − 3𝑢
𝑒 −𝑡 . (1 + 𝑡)
⇒ (u-1)( 𝑢2 − 𝑢 + 4)= 0
Multiplying and integrating
𝑢2 − 𝑢 + 4 is always +ve. So, u= 1

Similarly 𝑣 3 + 15𝑣 + 16 = 0

52
Solving Mathematical Problems

⇒ (v +1)( 𝑣 2 − 𝑣 + 16)= 0 (a) 80 (b) 80.2


(c) 80.5 (d) 81
⇒ v= -1
2007 2006 2004 2003
𝑛 𝑛+1 Ans:- (b) x= [3.2005 . 2005 . 2005 . 2005]
So, for each n, 𝑢 + 𝑣 =0
2 1 1 2
= [3(1+2005)(1+2005 )(1-2005)(1-2005)]

8. The number of real values of x 4 1


= [3(1-(2005)2)( 1-(2005)2 )]
satisfying the equation
𝟏⁄ 𝟏 ⇒ x=2.
𝒙. 𝟐 𝒙 + 𝒙 . 𝟐𝒙 = 4 is / are
11. A graph defined in polar co –
(a) 1 (b) 2 𝟏
ordinates by r (𝜃) = cos 𝜃+𝟐. The smallest
(c) 3 (d)4
x –co- ordinates of any point on this
Ans:- (a) if x < 0, LHS = -ve but RHS= +ve graph is

If x = 0, LHS= not defined. (a) 1/16 (b) -1/16


(c) 1/8 (d) -1/8
If x > 0, use AM ≥ GM inequality
Ans:- (b) x =r cos𝜃
1 1 1
𝑥. 2 ⁄𝑥 + . 2𝑥 ≥ 2√2 ⁄𝑥+𝑥 1
𝑥 = cos 2 𝜃 + 2 𝑐𝑜𝑠𝜃
≥ 2. √22 =4 1
= (𝑐𝑜𝑠𝜃 + 4)2 = 1/16
1⁄ 1
⇒ 𝑥. 2 𝑥 = . 2𝑥 ; so, x= 1.
𝑥
12. A monic polynomial is one in which
9. Let f (x) and g (x) be functions, which the coefficient of the highest order term is
take integers as arguments. Let 1. The monic polynomial P(x) (with
f (x+ y) = f (x) + f (y) + 8 for all integers x integer coefficient) of least degree that
and y. Let f (x) = x for all negative satisfies P (√𝟐 + √𝟓)= 0 is
numbers x and let g(8)=17, then f (0)=?
(a) 𝒙𝟒 − 𝒙𝟑 − 𝟏𝟒𝒙𝟐 + 𝟗=0
(a) 8 (b) 9 (c) (b) 𝒙𝟒 − 𝟏𝟒𝒙𝟐 + 𝟗= 0
17 (d) 72
(c) 𝒙𝟒 + 𝒙𝟑 − 𝟏𝟒𝒙𝟐 + 𝟗=0
Ans:- (c) put x = -8, y= 8 in the given (d) 𝒙𝟒 + 𝟏𝟒𝒙𝟐 − 𝟗
functional equation,
ANS:- (b) Let x=√2 + √5 . Squaring, 𝑥 2 =
𝟐𝟎𝟎𝟕.𝟐𝟎𝟎𝟔.𝟐𝟎𝟎𝟒.𝟐𝟎𝟎𝟑
10. Let x = [ 𝟏 ], where [x] 7 + 2√10
× (𝟐𝟎𝟎𝟓)𝟒
𝟑
denotes the greatest integer integer less ⇒ 𝑥 2 − 7 = 2√10. Squaring again, 𝑥 4 −
((𝒙+𝟏).𝒙𝟐 )+𝟏
than or equal to x. then is 14𝑥 2 + 9= 0
(𝒙𝟐 +𝟏)

53
Solving Mathematical Problems

13. The number of distinct real roots of ⇒ a (x+1)+b(y-1)=0⇒ x=-1 , y= 1


the equation 𝒙𝟒 + 𝟖𝒙𝟐 + 𝟏𝟔 = 𝟒𝒙𝟐 −
If a-b = -c ⇒ ax+ by+ (b-a) =0
𝟏𝟐𝒙 + 𝟗 is
⇒ a (x- 1)+ b(y +1)=0
(a) 1 (b) 2
(c) 3 (d) 4 ⇒ x=1, y= -1.
Ans:- (a) (𝑥 2 + 4)2 = (2𝑥 − 3)2 ⇒ 𝑥2 + 16. The value of k for which the
4 = ±(2𝑥 − 3) inequality k𝐜𝐨𝐬 𝟐 𝒙 − 𝒌𝒄𝒐𝒔𝒙 + 𝟏 ≥ 𝟎 ∀ 𝒙 ∊
Giving x2 – 2x + 7 = 0 and x2 +2x +1 = 0. (−∞, ∞)holds is
𝟏
Solving x = - 1 only one real root. (a) k < - (b) k > 4
𝟐
𝟏
14. If in an isosceles triangle with base ‘a’, (c) − 𝟐 ≤ 𝒌 ≤ 𝟒 (d)
𝟏
vertical angle 20° and lateral side of each ≤𝒌≤𝟓
𝟐
wih length ‘b’ is given then the value of
𝒂𝟑 + 𝒃𝟑 equals Ans:- (c) kcos 2 𝑥 − 𝑘𝑐𝑜𝑠𝑥 + 1 ≥ 0 ∀ 𝑥 ∊
(−∞, ∞)
(a) 3ab (b) 3a𝒃𝟐
(c) 3𝒂𝟐 𝒃 (d) 3 ⇒ k (cos 2 𝑥 − 𝑐𝑜𝑠𝑥) +1 ≥ 0…………(i)
𝑎 1 1
Ans:- (b) sin 10° = 2𝑏 ⇒ sin 30° = But cos2 𝑥 − 𝑐𝑜𝑠𝑥 = (𝑐𝑜𝑠𝑥 − 2)2 − 4
3 𝑠𝑖𝑛10° − 4𝑠𝑖𝑛3 10° 1
⇒ − 4 ≤ cos 2 𝑥 − 𝑐𝑜𝑠𝑥 ≤ 2
1 3𝑎 4𝑎3
⇒ 2 = 2𝑏 − 8𝑏3 1
From (i) we get 2k+1 ≥ 0 ⇒ 𝑘 ≥ − 2
3𝑎 4𝑎3
⇒1= − 8𝑏3 𝑘
𝑏
⇒ −4 +1 ≥ 0
⇒ 𝑎3 + 𝑏 3 = 3𝑎𝑏 2 .
⇒k≤4
𝟐 𝟐 𝟐
15. If 𝒂 + 𝒃 + 𝒄 − 𝟐𝒂𝒃 = 𝟎, then the 1
point of concurrency of family of lies ax+ ⇒ −2 ≤ 𝑘 ≤ 4
by+ c= 0 lies on the line
17. Consider two series (i)
𝝅 𝝅
(a) y= x (b) y = x+ 1 ∑∞
𝒏=𝟏 𝒔𝒊𝒏 (𝒊𝒊) ∑∞ 𝒏
𝒏=𝟏(−𝟏) 𝒄𝒐𝒔 , then
𝒏 𝒏
(c) y = -x (d)3x= y
(a) Both (i) and (ii) converge
Ans:- (c) (𝑎 − 𝑏)2 − 𝑐 2 = 0 (b) (i) converges, (ii) diverges
⇒ (a-b -c)(a- b+ c)=0 (c) (i) diverges, (ii) converges
If a- b= c ⇒ ax + by+ (a-b)=0 (d) both (i) and (ii) diverges.

54
Solving Mathematical Problems

Ans:- (d) ∑∞
𝑛=1 𝑠𝑖𝑛 𝑛
𝜋
=
𝜋 1 𝜋
− 3! (𝑛 )3 + ∑∞
𝒏=𝟏 𝒂𝒏 is convergent. If p is a real
𝑛
√𝒂𝒏
⋯ = 𝑈𝑛 number such that the series∑ 𝒏𝒑
1 𝑈𝑛 diverges, the
Let 𝑉𝑛 = 𝑛 , ∴ lim = 𝜋≠0
𝑛→∞ 𝑉𝑛
𝟏
(a) P< ½ (b) P≤ 𝟐
Since ∑ 𝑈𝑛 𝑑𝑖𝑣𝑒𝑟𝑔𝑒𝑠, 𝑠𝑜 𝑑𝑜𝑒𝑠 ∑ 𝑉𝑛 . 𝟏 𝟏
(c) 𝟐 <P≤ 𝟏 (d) 𝟐 ≤ P ≤ 𝟏

𝜋
(𝑖𝑖) ∑(−1)𝑛 𝑐𝑜𝑠 𝑐𝑜𝑛𝑣𝑒𝑟𝑔𝑒𝑠 Ans:- (a) ∑ 𝑎𝑛 is convergent
𝑛
𝑛=1
𝑎𝑛
∞ ⤇∑ is convergent.
𝑛
𝜋
𝑖𝑓 ∑ |(−1)𝑛 𝑐𝑜𝑠 | 1
𝑛 ∑(𝑎𝑛 + ) is convergent. By AM≥ GM
𝑛=1 𝑛

𝜋 √𝑎𝑛
= ∑ 𝑐𝑜𝑠 𝑐𝑜𝑛𝑣𝑒𝑟𝑔𝑒𝑠. ⇒∑ 1 is converges
𝑛 𝑛2
𝑛=1

𝜋 S0, for P < ½ , the series is divergent.


But lim 𝑐𝑜𝑠 𝑛 = 𝑐𝑜𝑠𝜃 = 1 ≠ 0
𝑛→∞ 20. A rigid body is spinning about a fixed
𝜋 point (3, -2, -1) with angular velocity of 4
So, ∑∞
𝑛=1 𝑐𝑜𝑠 𝑛 diverges.
rad/sec., the axis of rotation being in
𝟏 𝟏 the direction of (1, 2, -2), then the velocity
18. If a= 𝐥𝐢𝐦 (𝟏 + 𝟐 + ⋯ + 𝒏) 𝒂𝒏𝒅 𝒃 =
𝒏→∞ of the particle at the point (4, 1, 1) is
𝟏 𝟏 𝟏
𝐥𝐢𝐦 (𝟏 + 𝟐 + ⋯ + 𝒏) then
𝒏→∞ 𝒏 (a) 4/3 (1, -4, 10) (b) 4/3 (4, -10, 1)
(c) 4/3 (10, -4, 1) (d) 4/3(10, 4, 1)
(a) both a= ∞ 𝒂𝒏𝒅 𝒃 = ∞
Ans:- (c)
(b) a= ∞ and b= 0
̂
𝑖̂+2𝑗̂ −2𝑘 4
(c) a= ∞ and b=1 𝜔⃗ = 4( ) = 3 (𝑖̂ + 2𝑗̂ − 2𝑘̂)
√1+4+4

(d) none. r⃗ = O⃗P- O⃗A

= (4î+ĵ+k̂)-(3î-2ĵ-k̂)= î+3ĵ+ 2k̂


1
Ans:- (d) {𝑢𝑛 }= {𝑛}, lim 𝑢𝑛 = 0, ∴ 𝑎 = 0
𝑛→∞
4
So, By Cauchy’s first v̂= 𝜔̂× 𝑟⃗ = 3 ( î + 2ĵ − 2k̂) × ( î + 3ĵ +
1 1 4
theorem lim
1+ +⋯+
2 𝑛
=0 2k̂) = 3 (10𝑖̂ − 4𝑗̂ + 𝑘̂)
𝑛→∞ 𝑛

21. A particle has an angular speed of 3


∴ b= 0.
rad /sec and the axis of rotation passes
19. Let {𝒂𝒏 } be a sequence of non- through the point (1, 2, 2) and (1, 2, -2),
negative real numbers such that the series then the velocity of the particle at the
point P(3, 6, 4) is
55
Solving Mathematical Problems
𝟑 =1- [P(A).P(E/A)+P(B).P(E/B)]
(a) (22, 8, -2)
√𝟏𝟕
1 35 1 20 55 11 29
𝟑 =1- [2 . 100 + 2 . 100 = 200] = (1 − 40) = 40
(b) (𝟐𝟐, 𝟎, 𝟐)
√𝟏𝟕

𝟑 23. for any two events A and B in a


(c) (𝟐𝟐, −𝟖, −𝟐)
√𝟏𝟕 sample space
𝟑 𝑷(𝑨)+𝑷(𝑩)−𝟏
(d) (𝟐𝟐, −𝟖, 𝟐) (a) P (A/B)≥ , 𝑷(𝑩) ≠ 𝟎 is
√𝟏𝟕 𝑷(𝑩)
always true.
Ans:- (c)
(b) P (A∩B̅) = P (A)- P(A∩B) does not
O⃗A= î + ĵ + 2k̂
hold
O⃗B =î + 2ĵ − 2k̂
(c) P (A∪B) =1-P (A̅).P(B̅) if A and B are
∴ A⃗B= ĵ-4k̂ independent

⟹ | A⃗B |= √17 (d) P (A∪B) =1-P (A̅).P(B̅) if A and B are


disjoint
AP⃗= (3î+6ĵ+4k̂)- (î+ĵ+2k̂)
Ans. (c)
= 2î+5ĵ+2k̂
P(A/B͞)+P(A͞/B͞)= 1,
(ĵ-4k̂)
3
∴𝜔⃗ =
√17 ∴ P(A͞+B͞)= 1-P(A/B͞)

(ĵ-4k̂)× (2î + 5ĵ + 2k̂) =


3
v⃗= 𝜔̂× 𝑟⃗= 1 − 𝑃(𝐴 ∪ 𝐵) 𝑃(𝐴 ∪ 𝐵)′
√17 =
3 𝑃(𝐵) 𝑃(𝐵′)
(22î − 8ĵ − 2k̂)
√17 𝑃(𝐴′ ∩ 𝐵 ′ ) 𝐴′
= = 𝑃 ( ′)
22. In a group of equal number of boys 𝑃(𝐵′) 𝐵
and girls, 20% girls and 35% boys are
24. one hundred identical coins, each with
graduate. If a member of the group is
probability P, of showing up heads are
selected at random, then the probability
tossed. If 0 < P < 1 and the probability of
of this member not being a graduate is
heads showing on 50 coins is equal to that
𝟏𝟏 𝟗 of the heads showing on 51 coins, then p=
(a) (b) (c)
𝟒𝟎 𝟐𝟎
𝟏𝟏 𝟐𝟗 𝟏 𝟒𝟗
(d) 𝟒𝟎 (a) 𝟐 (b) 𝟏𝟎𝟏
𝟐𝟎
𝟓𝟎 𝟓𝟏
Ans. (d) Let A and B denotes the events that (c) 𝟏𝟎𝟏 (d) 𝟏𝟎𝟏
the member selected at random is a boy and
Ans. (d)
a girl respectively. Let E denotes the event
that the member selected is a graduate. Here n= 100, p= p, q= 1-p
Reqd. prob.
Given, p(50) = p(51)

56
Solving Mathematical Problems

⟹100𝐶50 𝑝50 (1 − 𝑝)50 = 100𝐶51 𝑝51 (1 − Ans. (d)


49
𝑝)
For f(x) to be defined
100! 100!
⟹ (1 − 𝑝) = 𝑝 (i) [2𝑥 2 − 3]= -1, 0, 1
50! 50! 51! 49!
⟹ 51(1 − 𝑝) = 50𝑝 ⟹ 𝑝 ⟹ -1 ≤ 2𝑥 2 − 3 < 2 ⟹2 ≤2𝑥 2 < 5
51
= 5
101 ⟹ 1≤ 𝑥 2 < 2
25. A box contains 24 identical balls of
1 ≤ 𝑥 2 ⟹ 𝑥 ≤ −1 𝑜𝑟 𝑥 ≥ 1
which 12 are white and 12 are black. The
⟹{ 2 5 5 5
balls are drawn at random from the box 𝑥 < ⟹ −√ < 𝑥 < √
2 2 2
one at a time with replacement. The
probability that a white ball is drawn for 5 5
the 4th time on the 7th draw is ⟹ −√2 < 𝑥 ≤ −1 𝑜𝑟 1 ≤ 𝑥 < √2

𝟓 𝟐𝟕
………..(A)
(a) 𝟔𝟒 (b) 𝟑𝟐
𝟓 𝟏 (ii) 𝑥 2 − 5𝑥 + 5 > 0 ⟹ 𝑥 <
(c) 𝟑𝟐 (d) 𝟐 5−√5 5+√5
𝑜𝑟 𝑥 > …….(B)
2 2
Ans. (c) Probability of drawing a white ball (iii) log 1 (𝑥 2 − 5𝑥 + 5) > 0
12 1 2
in any draw=24 = 2
1 0
A white ball will be drawn for the 4 time th ⟹ 𝑥 2 − 5𝑥 + 5 < ( )
2
on the 7th draw ball is drawn in the 7th draw
⟹ 𝑥 2 − 5𝑥 + 5 < 1 ⟹ 𝑥 2 − 5𝑥 +
and 3 white balls are drawn in the first 6
4<0
draws.
⟹ 1< x< 4 ……………(C)
∴ Required probability
5−√5
3 3 1 3 1 3 1 5 From (A), (B) and (C), 1 ≤ x <
= 6𝐶3 𝑝 𝑞 . 𝑝 = 20. ( ) . ( ) . = 2
2 2 2 32
27. If f(x)= 𝐋𝐭 𝐋𝐭 𝒄𝒐𝒔𝟐𝒎 𝒏! 𝝅𝒙, then
26. If [x]denotes the integral part of x, 𝒎→∞ 𝒏→∞
then the domain of the function range of f(x)is

f(x)= 𝐬𝐢𝐧−𝟏 [𝟐𝒙𝟐 − 𝟑] + 𝐥𝐨𝐠 𝟐 {𝐥𝐨𝐠 𝟏/𝟐 (𝒙𝟐 − (a) [0, 1] (b) [0, 1]
𝟓𝒙 + 𝟓)} is (c) (0, 1) (d) {0}

Ans. (b)
𝟓 𝟓
(a) (−√𝟐 , −𝟏) (b) (𝟏, √𝟐)
When x is rational say p/q , then n! x𝜋 is a
𝟓 𝟓 multiple of 𝜋 and 𝑐𝑜𝑠 2 𝑛! 𝑥𝜋 = 1
(c) (−√𝟐 , −𝟏) ∪ (𝟏, √𝟐)
∴ Lt 𝑐𝑜𝑠 2𝑚 𝑛! 𝑥𝜋 = Lt 1𝑚 = 1
(d) none of these 𝑚→∞ 𝑚→∞

57
Solving Mathematical Problems

∴ f(x)= 1 𝑘
|𝑡|
(1 + 𝑡 2 )(2 + 𝑡 2 ) = 𝑎𝑟𝑒𝑎 𝑜𝑓 𝛥𝑃𝑂𝑄
When x is irrational, 1
= 2 |𝑡 2 . 2𝑡1 − 2𝑡. 𝑡1 2 |
n! x𝜋 ≠a multiple of 𝜋
= |𝑡 2 𝑡1 − 𝑡𝑡1 2 |
2
∴𝑐𝑜𝑠 𝑛! 𝑥𝜋 ≠ 1
2
𝑡 2 +2 (𝑡 2 +2)
2
∴ 0≤ 𝑐𝑜𝑠 𝑛! 𝑥𝜋 < 1 = |−𝑡 2 ( )−𝑡 |
2 𝑡2

∴ Lt 𝑐𝑜𝑠 2𝑚 𝑛! 𝑥𝜋 = (𝑡 2 +2)
𝑚→∞ = (𝑡 2 + 2) |𝑡 + |
𝑡
Lt (𝑐𝑜𝑠 2 𝑛! 𝑥𝜋)𝑚 =0
𝑚→∞
(𝑡 2 +2) (1+𝑡)2
= (𝑡 2 + 2) |𝑡 + | = (𝑡 2 + 2)2
Thus f(x)= 0, when x is rational 𝑡 |𝑡|

= 1, when x is irrational ∴ k= 2

∴ Range f= {0, 1} 29. If {x} denotes the fractional part of x,


𝟑𝟐𝟎𝟎
then { }=
28. The normal at any point P (𝒕𝟐 , 𝟐𝒕) on 𝟖

the parabola 𝒚𝟐 = 𝟒𝒙 meets the curve 𝟏 𝟑


(a) 𝟖 (b) 𝟖
again at Q, the area of 𝛥POQ, O being the 𝟓
𝒌 (c) 𝟖 (d) none of these
origin is |𝒕| (𝟏 + 𝒕𝟐 )(𝟐 + 𝒕𝟐 ) then
Ans. (d)
(a) k > 2 (b) k=2
(c) k < 2 (d) k= 1 3200 9100 (1 + 8)100
= =
8 8 8
Ans. (b)
1 + 100𝐶1 8 + 100𝐶1 82 + ⋯ + 8100
=
Given P ≡ (𝑡 2 , 2𝑡) 8
1
= + 𝑎𝑛 𝑖𝑛𝑡𝑒𝑔𝑒𝑟
Given parabola is 𝑦 2 = 4𝑥 ……….. (1) 8

Here a= 1. 3200 1
∴{ }=8
8
2
Let Q=( 𝑡1 , 2𝑡1)
30. The number of distinct terms in the
Since normal at P meet the curve again at Q. expression of (𝒙𝟏 + 𝒙𝟐 + ⋯ + 𝒙𝒑 )𝒏

2 𝑡 2 +2 (a) 𝒏 + 𝒑 − 𝟏𝑪𝒏 (b) n+ p+ 1


∴ 𝑡1 = −𝑡 − 𝑡 = ………… (2)
𝑡
(c) n+1 (d) 𝒏 + 𝒑 − 𝟏𝑪𝒑+𝟏
Now O≡ (0, 0), P≡ (𝑡 2 , 2𝑡), Q=( 𝑡1 2 , 2𝑡1 )
Ans. (d) Number of terms
Given,
𝑛 + 𝑝 − 1𝐶𝑛 = 𝑛 + 𝑝 − 1𝐶𝑝−1

58
Solving Mathematical Problems

ISI OBJECTIVE SAMPLE PAPER (c) f(x) is neither continuous nor


WITH SOLUTIONS differentiable at x=0

(d) f(x) is a periodic function


SET – 8
Ans. (c)
𝑥
𝑡𝑟+1 =
There will be 30 questions in MMA Paper. (𝑟𝑥 + 1){(𝑟 + 1)𝑥 + 1}
For each question, exactly one of the four (𝑟 + 1)𝑥 + 1 − (𝑟𝑥 + 1)
choices is correct. You get four marks for =
(𝑟𝑥 + 1)[(𝑟 + 1)𝑥 + 1]
each correct answer, one mark for each
unanswered question, and zero mark for 1 1
each incorrect answer. = −
(𝑟𝑥 + 1) (𝑟 + 1)𝑥 + 1
𝑛−1
1. If [x] denotes the integral part of x, 1
∴ 𝑆𝑛 = ∑ 𝑡𝑟+1 = 1, 𝑥 ≠ 0 = 0, 𝑥
then 𝐋𝐭
𝐬𝐢𝐧[𝒄𝒐𝒔𝒙]
= 𝑛𝑥 + 1
𝑟=0
𝒙→𝟎 𝟏+[𝒄𝒐𝒔𝒙]
=0
𝐬𝐢𝐧 𝟏
(a) 0 (b) 1 (c) 1
𝟐
∴ Lt 𝑆𝑛 = Lt (1 − 𝑛𝑥+1)
(d) does not exist 𝑛→∞ 𝑛→∞

Ans. (a) Lt [cos 𝑥] = 0 1, 𝑥 ≠ 0


Thus, f(x)= {
𝑥→0−0 0, 𝑥 = 0
[∵ when x ⟶0- 0, 0 < cosx < 1 ] and ∴ Lt 𝑓(𝑥) = 1 𝑎𝑛𝑑 𝑓(0) = 0
𝑥→0
Lt [cos 𝑥] = 0
𝑥→0+0 Hence f(x) is neither continuous nor
differentiable at x= 0
[∵ when x ⟶0+ 0, 0 < cosx < 1 ]
sin[cos 𝑥] sin 0
Clearly f(x) is not a periodic function.
∴ Lt = =
𝑥→0−0 1+[cos 𝑥] 1+0
𝐥𝐨𝐠(𝟐+𝒙)−𝒙𝟐𝒏 𝒔𝒊𝒏𝒙
0 Lt
sin[cos 𝑥]
=
sin 0
=0 3. Let f(x)= 𝐋𝐭 then f(x) is
𝒏→∞ 𝟏+𝒙𝟐𝒏
𝑥→0+0 1+[cos 𝑥] 1+0
discontinuous at
∴Required limit = 0
(a) x= 1 only (b) x=-1 only
𝒙
2. Let f(x)= 𝐋𝐭 ∑𝒏−𝟏
𝒓=𝟎 (𝒓𝒙+𝟏){(𝒓+𝟏)𝒙+𝟏)} then (c) x= -1, 1 only (d) no point
𝒏→∞
Ans. (c)
(a) f(x) is continuous but not
differentiable at x= 0

(b) f(x) is both continuous and


differentiable at x= 0

59
Solving Mathematical Problems

Lt 𝑋 2𝑛 = Lt (𝑥 2 )𝑛 From graph it is clear that f(x) is continuous


𝑛→∞ 𝑛→∞
at all x and differentiable at all x except x= -
∞, 𝑥2 > 1
1 and x= 1
= {0, 0 ≤ 𝑥2 < 1
1, 𝑥2 = 1 5. If f(x) = p |sin x|+𝒒𝒆|𝒙| +
∞, 𝑥 < −1 𝑜𝑟 𝑥 > 1
𝒓|𝒙|𝟑 𝒂𝒏𝒅 𝒇(𝒙) is differentiable at x=0,
= { 0, −1<𝑥 <1
1, 𝑥 = ±1 then

∴ 𝑓(𝑥) (a) p= q= r=0


−𝑠𝑖𝑛𝑥, 𝑥 < −1 𝑜𝑟 𝑥 > 1 (b) p=0, q=0, r== any real number
log(2 + 𝑥) , −1<𝑥 <1
= { (c) q=0, r=0, p is any real number
log(2 + 𝑥) − 𝑠𝑖𝑛𝑥
, 𝑥 = ±1 (d) r=0, p=0, q is any real number
2
Lt 𝑓(𝑥) = −𝑠𝑖𝑛1, Lt 𝑓(𝑥) Ans. (b)
𝑥→1+0 𝑥→1−0
= 𝑙𝑜𝑔3, Lt 𝑓(𝑥) 𝑓(ℎ)−𝑓(0)
𝑥→1+0
f’(0- 0)= Lt =
ℎ→0−0 ℎ
= Lt log(2 + 𝑥) = 0, 𝑝|sin ℎ|+𝑞𝑒 |ℎ| +𝑟|ℎ|3 −𝑞
𝑥→−1 Lt
ℎ→0−0 ℎ
Lt 𝑓(𝑥) = Lt (− sin 𝑥) = 𝑠𝑖𝑛1
𝑥→1−0 𝑥→−1 −𝑝 sin ℎ + 𝑞𝑒 ℎ − 𝑟ℎ3 − 𝑞
= Lt
Clearly f(x) is discontinuous only at two ℎ→0−0 ℎ
points x= -1, 1 𝑠𝑖𝑛ℎ 𝑞(𝑒 ℎ − 1)
= Lt {−𝑝 − − 𝑟ℎ2 }
4. The function f(x) = max {(1-x), (1+x), 2} ℎ→0−0 ℎ −ℎ
is, where x ∊(-∞, ∞) = −𝑝 − 𝑞

(a) discontinuous at all points Similarly, f’ (0+0)= p+q


(b) differentiable at all points Since f(x) is differentiable at x= 0
(c) differentiable at all points except -1
∴ f’(0- 0) = f’(0+ 0)⟹ - p- q= p+ q
and 1 (d) continuous at all points
except -1 and 1 ⟹ p+ q= 0

Ans. (c) Here r may be any real number.

We draw the graph of y= 1- x, y= 1+ x and y 6. Let f(x)= 𝒙𝟑 − 𝒙𝟐 + 𝒙 + 𝟏, 𝒈(𝒙) =


=2 𝒎𝒂𝒙. {𝒇(𝒕), 𝟎 ≤ 𝒕 ≤ 𝒙}, 𝟎 ≤ 𝒙 ≤ 𝟏
f(x)= max.{1-x, 1+x, 2} = 𝟑 − 𝒙, 𝟏 < 𝑥 ≤ 2
∴ f(x)= 1- x, x ≤ -1= 2, -1≤ x≤ 2 = 1+x, x ≥ then in [0, 2] the points where g(x) is not
2 differentiable is (are)

60
Solving Mathematical Problems

(a) 1 (b) 2 (c) f"(x) exists for all x in R


(c) 1 and 2 (d) none of these (d) f(x) is discontinuous at all integer
points in R
Ans. (a) 𝑓(𝑡) = 𝑡 3 − 𝑡 2 + 𝑡 + 1
Ans. (d)
∴ f’ (t)= 3𝑡 2 − 2𝑡 + 1 > 0
Sin 𝜋 [x+ 1]= 0
∴ f(t) is an increasing function.
Also [x+ 1]= [x]+ 1
Since 0≤ t ≤ x
[𝑥] 𝜋
3 2 ∴ f(x)= 1+[𝑥] sin [𝑥]+1 𝑎𝑡 𝑥 = 𝑛, 𝑛 ∊
∴ max f(t)= f(x)= 𝑥 − 𝑥 + 𝑥 + 1
𝑛 𝜋
3 2
𝐼, 𝑓(𝑥) = sin 𝑓𝑜𝑟 𝑛 − 1 < 𝑥 <
Thus g(x)= 𝑥 − 𝑥 + 𝑥 + 1, 0 ≤ 𝑥 ≤ 1= 3- 1+𝑛 𝑛+1
𝑛, [𝑥] = 𝑛 − 1
x, 1 < x ≤ 2
𝑛−1 𝜋
The only doubtful point for differentiability ∴ f(x)= sin 4
𝑛
of g(x) in [0, 2] is x = 1
𝑛−1 𝜋
3 2 ℎ𝑒𝑛𝑐𝑒 Lt 𝑓(𝑥) sin ,
Clearly, Lt 𝑔(𝑥) = 1 − 1 + 1 + 1 = 𝑥→𝑛−0 𝑛 4
𝑥→1+0
2 𝑛 𝜋
∴ f(n)= 1+𝑛 sin 𝑛+1
Lt 𝑔(𝑥) = Lt (3 − 𝑥) = 2 𝑎𝑛𝑑 𝑔(1)
𝑥→1+0 𝑥→1 ∴ f(x) is discontinuous at all n ∊ I
=2
[𝒕𝒂𝒏𝟐 𝒙]−𝟏 𝝅
8. Let f(x) = , 𝒙 ≠ 𝒏𝝅 ± 𝟒 = 𝟎, 𝒙 =
∴ g(x) is continuous at x= 1 𝒕𝒂𝒏𝟐 𝒙−𝟏
𝝅
𝒏𝝅 ± 𝟒 then f(x) is
Also g’ (x)= 3𝑥 2 − 2𝑥 + 1, 0 ≤ 𝑥 < 1 =
−1, 1 < 𝑥 ≤ 2 (a) continuous at all x (b) continuous at
𝝅 𝝅
x= 𝟒 (c) discontinuous at x= 𝟒 (d)
∴ g’(1-0)= 3. 12 − 2.1 + 1 = 2 and
g’(1+0)= -1 none

Hence g(x) is not differentiable at x= 1. Ans. (c)

Since tan x is not defined at


7. If [x] denotes the integral part of x and
𝝅
𝒔𝒊𝒏[𝒙+𝟏]+𝐬𝐢𝐧 𝝅[𝒙+𝟏] 𝜋
f(x)= [x]{ }, then X= (2n+1) 2 , 𝑛 ∊
𝟏+[𝒙]
𝐼, 𝑡ℎ𝑒𝑟𝑒𝑓𝑜𝑟𝑒 𝑓(𝑥)𝑖𝑠 𝑑𝑖𝑠𝑐𝑜𝑛𝑡𝑖𝑛𝑢𝑜𝑢𝑠 𝑎𝑡 𝑥 =
𝜋
(a) f(x) is continuous in R (2𝑛 + 1) , 𝑛 ∊ 𝐼
2
(b) f(x) is continuous but not
𝜋
differentiable in R Now f ( 4 ) = 0

61
Solving Mathematical Problems

[𝑡𝑎𝑛2 𝑥] − 1 Ans. (b)


Lt
𝜋
𝑓(𝑥) = Lt
𝜋 2
𝑥→ −0 𝑥→ −0 𝑡𝑎𝑛 𝑥 − 1 𝜋 𝜋
4 4 For 0 < 𝑥 < 𝑜𝑟 < 𝑥 < 𝜋 0 < sin 𝑥 <
0−1 2 2
= Lt
𝜋 2
= ∞ 1
𝑥→ −0 𝑡𝑎𝑛 𝑥 − 1
4

𝜋 ∴ for n > 1, sin x > 𝑠𝑖𝑛4 𝑥


Hence f(x) is discontinuous at x= 4
3(sin 𝑥−𝑠𝑖𝑛4 𝑥) 𝜋
∴ f(x)=[ ] = 3, 𝑥 ≠ = 3, 𝑥 =
𝒙 𝟏 sin 𝑥−𝑠𝑖𝑛𝑛 𝑥 2
9. Let f(x)= ∫𝟎 𝒕 𝒔𝒊𝒏 𝒕 𝒅𝒕, then the number 𝜋
2
of points of discontinuity of f(x) in (0, 𝜋) is
Thus in (0, 𝜋) , f(x) = 3
(a) 0 (b) 1
(c) 2 (d) more than 2 Hence f(x)is continuous and differentiable at
𝜋
Ans. (a) x= 2

𝑥
1 11. If[x] denotes the integral part of x and
𝑓(𝑥) = ∫ 𝑡 sin 𝑑𝑡 f(x) = [n+ psin x], 0< x < 𝜋, n∊I and p is a
0 𝑡
prime number, then the number of points
1
∴ f’(x)= x sin 𝑥 where f(x) is not differentiable is

Clearly f’(x) is a finite number at all x in (0, (a) p-1 (b) p


𝜋). (c) 2p-1 (d) 2p +1

∴ f(x) is differentiable and hence continuous Ans. (c) [x] is not differentiable at integral
at all x in (0, 𝜋) points.

10. if [x] denotes the integral part of x and Also [n+ p sin x]= n+ [p sin x]
in (0, 𝜋), we define ∴ [p sin x] is not differentiable, where p sin
𝟐(𝒔𝒊𝒏𝒙−𝒔𝒊𝒏𝒏 𝒙)+|𝒔𝒊𝒏𝒙−𝒔𝒊𝒏𝒏 𝒙| x is an integer. But p is prime and 0 < sin x
f(x)= [𝟐(𝒔𝒊𝒏𝒙−𝒔𝒊𝒏𝒏 𝒙)−|𝒔𝒊𝒏𝒙−𝒔𝒊𝒏𝒏 𝒙|] = 𝟑, 𝒙 =
≤1 [∵ 0 < x < 𝜋]
𝝅 𝝅
𝒙≠ then for n > 1
𝟐 𝟐
∴ p sin x is an integer only when
(a) f(x) is continuous but not 𝑟
𝝅 sinx = , where 0< r ≤ p and r ∊ N
differentiable at x= 𝟐 𝑝

(b) both continuous and differentiable 𝜋


𝝅
For r= p, sin x= 1 ⟹ x= 2 in (0, 𝜋)
at x= 𝟐
𝑟
(c) (c) neither continuous nor For 0 < r < p, sin x= 𝑝
𝝅
differentiable at x= 𝟐
𝑟 𝑟
(d) 𝐋𝐭𝝅 𝒇(𝒙) 𝒆𝒙𝒊𝒔𝒕 𝒃𝒖𝒕 𝐋𝐭𝝅 𝒇(𝒙) ≠ ∴ x= sin−1 𝑝 𝑜𝑟 𝜋 − sin−1 𝑝
𝒙→ 𝒙→
𝟐 𝟐
𝝅
𝒇( 𝟐 )

62
Solving Mathematical Problems

Number of such values of x= p- 1+p- 1= 2p ⟹ f’(x)= 0∀x∊ R


-2
𝒅 𝒙𝟑
14. Let f(t)in t. then 𝒅𝒙 {∫𝒙𝟐 𝒇(𝒕)𝒅𝒕}
∴ Total number of points where f(x) is not
differentiable (a) has a value 0 when x= 0
(b) has a value 0 when x=1 , x= 4/9
= 1+ 2p – 2= 2p – 1

𝒔𝒆𝒄𝟒 𝜽 𝒕𝒂𝒏𝟒 𝜽 𝟏 (c) has a value 𝟗𝒆𝟐 − 𝟒𝒆 when x=e


12. If + = 𝒂+𝒃 , 𝒕𝒉𝒆𝒏 (d) has a differential coefficient 27e -8
𝒂 𝒃
when x= e
(a) |b|=|a| (b) |b|≤ |𝒂|
(c) |b|≥ |𝒂| (d) none of these 𝑑 𝑥3
Ans. (c) 𝑑𝑥 ∫𝑥 2 𝑓 (𝑡) 𝑑𝑡 = 𝑓 (𝑥 3 ). 3𝑥 2 −
Ans. (b) 𝑓(𝑥 2 ). 2𝑥

𝑠𝑒𝑐 4 𝜃 𝑡𝑎𝑛4 𝜃 1 𝑠𝑒𝑐 2 𝜃 − 𝑡𝑎𝑛2 𝜃 = 𝑙𝑜𝑔𝑥 3 . 3𝑥 2 − log 𝑥 2 . 2𝑥


+ = =
𝑎 𝑏 𝑎+𝑏 𝑎+𝑏 = 9𝑥 2 𝑙𝑜𝑔𝑥 − 4𝑥 𝑙𝑜𝑔𝑥
= 𝑥 𝑙𝑜𝑔𝑥(9𝑥 − 4)
𝑠𝑒𝑐 2 𝜃
⟹ 𝑎(𝑎+𝑏) [(𝑎 + 𝑏)𝑠𝑒𝑐 2 𝜃 − 𝑎] +
𝑑𝑧
𝑡𝑎𝑛2 𝜃 2 𝑙𝑒𝑡 𝑧 = 𝑥 log 𝑥 (9𝑥 − 4)𝑡ℎ𝑒𝑛
[(𝑎 + 𝑏)𝑡𝑎𝑛 𝜃 + 𝑏] = 0 𝑑𝑥
(𝑎+𝑏)𝑏
= (1 + log 𝑥)(9𝑥 − 4)
⟹ a𝑡𝑎𝑛2 𝜃 + 𝑏𝑠𝑒𝑐 2 𝜃 = 0 + 9𝑥 log 𝑥
𝑏 𝑑𝑧
⟹𝑠𝑖𝑛2 𝜃 = − 𝑎 𝑖𝑠 𝑛𝑜𝑛-negative and ≤ 1⟹ 𝑎𝑡 𝑥 = 𝑒, = 2(9𝑒 − 4) + 9𝑒 = 27𝑒 − 8
𝑏 𝑑𝑥
|𝑎| ≤ 1
15. If a,
13. If c be a positive constant and |f(y)- 𝜶𝟏 , 𝜶𝟐 , … 𝜶𝟐𝒏−𝟏 , 𝒃 𝒂𝒓𝒆 𝒊𝒏 𝑨. 𝑷. , 𝒂, 𝜷𝟏 , 𝜷𝟐 , … 𝜷𝟐𝒏−𝟏 , 𝒃
f(x)|≤ 𝒄(𝒚 − 𝒙)𝟐 for all real x and y, then are in G.P. and

(a) f(x)= 0 for all x (b) f(x)= x for all x a, 𝜸𝟏 , 𝜸𝟐 , … 𝜸𝟐𝒏−𝟏 , 𝒃 are in H.P., where a,
(c) f’(x)= 0 for all x (d) f’(x)= c for all x b are positive, then the equation

Ans. (c) 𝜶𝒏 𝒙𝟐 − 𝜷𝒏 𝒙 + 𝜸𝒏 = 𝟎 has

Given, |f(y)- f(x)| ≤𝑐(𝑦 − 𝑥)2 , 𝑐 > 0 (a) real and equal roots
(b) real and unequal roots
𝑓(𝑦)−𝑓(𝑥)
⟹|f(y)- f(x)| ≤𝑐|𝑦 − 𝑥|2 ⟹ | |≤
𝑦−𝑥
𝑓(𝑦)−𝑓(𝑥)
(c) imaginary roots
𝑐|𝑦 − 𝑥| ⟹ Lt | | ≤ Lt 𝑐|𝑦 − 𝑥| (d) roots which are in A.P.
𝑦→𝑥 𝑦−𝑥 𝑦→𝑥

Ans. (c)
⟹ |𝑓 ′ (𝑥)| ≤ 0

⟹|𝑓 ′ (𝑥)| ≤ 0 for all real x

63
Solving Mathematical Problems

The middle terms of the A.P., G.P. and H.P. (c) 2 (d) none of these.
are 𝛼𝑛 , 𝛽𝑛 𝑎𝑛𝑑 𝛾𝑛 respectively
Ans. (c)
∴ 𝛼𝑛 = A.M. of a and b,
1 ! +2 ! +3 ! +4! = 33
𝛽𝑛 = G.M. of a and b,
The digits at units place in each of 5 !, 6 !,…
𝛾𝑛 = H.M. of a and b, is 0

∵ AH=𝐺 2 ∴𝑎4 = 𝑎5 = 𝑎6 = ⋯ = 3

∴ 𝛼𝑛 𝛾𝑛 = 𝛽𝑛 2 Clearly 𝑎4 , 𝑎5 , 𝑎6 , … are in A.P. and G.P. but


not in H.P. as they are equal.
Now, discriminant of given equation =𝛽𝑛 2 −
4𝛼𝑛 𝛾𝑛 = −3𝛼𝑛 𝛾𝑛 < 0 (∵𝛼𝑛 , 𝛾𝑛 are 18. If (2+ x)(2+𝒙𝟐 )(𝟐 + 𝒙𝟑 ) … (𝟐 + 𝒙𝟏𝟎𝟎 ) =
positive) ∑𝒏𝒓=𝟎 𝒙𝒓 , then n equals

16. If 𝒂𝒏 = the digit at units place in the (a) 2550 (b) 5050
number 1! +2! +3! +…+n! for n≥ 4, (c) 𝟐𝟖 (d) none of these.
100×101
then 𝒂𝟒 , 𝒂𝟓 , 𝒂𝟔 , … are in Ans. (b) 𝑥 𝑛 = 𝑥1+2+3+⋯+100 = 𝑥 2 =
𝑥 5050
(a) A.P. only (b) G.P. only (c)
A.P. and G.P. only (d) A.P., G.P., ⟹ n= 5050
and H.P.
19. If p, q, r, s ∊ R, then equation (𝒙𝟐 +
Ans.(c) 𝒑𝒙 + 𝟑𝒒)(- 𝒙𝟐 + 𝒓𝒙 + 𝒒)(- 𝒙𝟐 + 𝒔𝒙 − 𝟐𝒒)=
1 ! +2 ! +3 ! +4! = 33 0 has

The digits at units place in each of 5 !, 6 !,… (a) 6 real roots


is 0 (b) at least two real roots

∴𝑎4 = 𝑎5 = 𝑎6 = ⋯ = 3 (c) 2 real and 4 imaginary roots


(d) 4 real and 2 imaginary roots.
Clearly 𝑎4 , 𝑎5 , 𝑎6 , … are in A.P. and G.P. but
Ans. (b)
not in H.P. as they are equal.
𝐷1 + 𝐷2 + 𝐷3 = 𝑝2 − 12𝑞 + 𝑟 2 + 4𝑞 +
𝑠 2 + 8𝑞 = 𝑝2 + 𝑟 2 + 𝑠 2 ≥ 0
17. Let p, q, r ∊ 𝑹+ and 27pqr ≥ (𝒑 + 𝒒 +
⟹ at least one of 𝐷1 , 𝐷2 , 𝐷3 ≥ 0
𝒓)𝟑 and 3p +4q +5r = 12 then 𝒑𝟑 + 𝒒𝟒 +
𝒓𝟓 is equal to 20. If a, b, c, d, are four non-zero real
numbers such that (𝒅 + 𝒂 − 𝒃)𝟐 +
(a) 3 (b) 6
(𝒅 + 𝒃 − 𝒄)𝟐 = 0 and roots of the equation

64
Solving Mathematical Problems

a(b-c) 𝒙𝟐 + b(c-a)x + c(a-b)= 0 and real Hence, equation F’(x)= 0 i.e. equation
and equal, then a, b, c f(x). (𝑥 2 + 𝑝𝑥 + 𝑞) = 0

(a) are equal (b) are not equal (c) are i.e. equation 𝑥 2 + 𝑝𝑥 + 𝑞 = 0 has at least
zero (d) none of the above one root (here exactly one root) in (0, 1) and
exactly one root in (1, 2).
Ans. (a) Equation 𝑎(𝑏 − 𝑐)𝑥 2 + 𝑏(𝑐 −
𝑎)𝑥 + 𝑐(𝑎 − 𝑏)= 0 has equal roots 22. If a, b, c, ∊ R, a ≠ 0 and (𝒃 − 𝟏)𝟐 <
2𝑎𝑐
4𝒂𝒄, then the number of roots of the
⟹ b= 𝑎+𝑐 …….(1) system of equation (in three unknowns
𝒙 𝟏 , 𝒙𝟐 , 𝒙𝟑 )
(𝑑 + 𝑎 − 𝑏)2 + (𝑑 + 𝑏 − 𝑐)2 =0
𝒂𝒙𝟏 𝟐 + 𝒃𝒙𝟏 + 𝒄 = 𝒙𝟐
⟹ a-b = b- c = -d
𝒂𝒙𝟐 𝟐 + 𝒃𝒙𝟐 + 𝒄 = 𝒙𝟑
⟹ 2b= a+ c ……(2)
4𝑎𝑐
𝒂𝒙𝟑 𝟐 + 𝒃𝒙𝟑 + 𝒄 = 𝟏 is
⟹ 𝑎+𝑐= a +c
(a) 0 (b) 1
⟹ (𝑎 − 𝑐)2 = 0⟹ a= c
(c) 2 (d) 3
From (2), b= a
Ans. (a) Let f(x) = 𝑎𝑥 2 + (𝑏 − 1)𝑥+c
thus a= b= c.
Given system of equation is equivalent
21. If p, q be non-zero real numbers and
𝟏 𝑓(𝑥1 )=𝑥2 −𝑥1
f(x)≠ 0 in [0, 2] and ∫𝟎 𝒇(𝒙). (𝒙𝟐 + 𝒑𝒙 + 𝑡𝑜 𝑓(𝑥2 )=𝑥3 −𝑥2 }
𝑓(𝑥3 )=𝑥1 −𝑥3
𝟐
𝒒)𝒅𝒙 = ∫𝟎 𝒇(𝒙). (𝒙𝟐 + 𝒑𝒙 + 𝒒)𝒅𝒙 = 𝟎
then equation 𝒙𝟐 + 𝒑𝒙 + 𝒒= 0 has ⟹ 𝑓(𝑥1 ) + 𝑓(𝑥2 ) + 𝑓(𝑥3 ) = 0

∴ 𝑎𝑓(𝑥1 ) + 𝑎𝑓(𝑥2 ) + 𝑎𝑓(𝑥3 ) = 0 (not


(a) two imaginary roots
(b) no root in (0, 2) possible)

(c) one root in (0,1) and other in (1,2) As (𝑏 − 1)2 − 4𝑎𝑐 < 0.
(d) one root in (-∞, 𝟎) and other in (2,∞) ∴ 𝑎𝑓(𝑥1 ), 𝑎𝑓(𝑥2 ), 𝑎𝑓(𝑥3 ) > 0.
Ans. (c) Hence given system of equation has no real
Let F(x) = ∫ 𝑓(𝑥)(𝑥 2 + 𝑝𝑥 + 𝑞)𝑑𝑥, root.

Then according to question

F(1) –F(0) = 0, F(2) – F(1) = 0

∴ F(0) = F(1) and F(1)= F(2)

65
Solving Mathematical Problems

23. If α, 𝛽 are the roots of the equation 25. 𝟐𝒔𝒊𝒏𝒙 + 𝟐𝒄𝒐𝒔𝒙 ≥ 𝟐𝟏 −


𝟏
√𝟐
𝒙𝟐 -ax +b= 0 and 𝑨𝒏 = 𝜶𝒏 + 𝜷𝒏 then
which of the following is true? (a) only for x ≥0 (b) only for x≤ 0
(c) for all real x (d) only for x ≠0
(a) 𝑨𝒏+𝟏 = 𝒂𝑨𝒏 + 𝒃𝑨𝒏−𝟏
(b) 𝑨𝒏+𝟏 = 𝒃𝑨𝒏 + 𝒂𝑨𝒏−𝟏 Ans. (c)

(c) 𝑨𝒏+𝟏 = 𝒂𝑨𝒏 − 𝒃𝑨𝒏−𝟏 Since A.M. ≥ G.M


(d) 𝑨𝒏+𝟏 = 𝒃𝑨𝒏 − 𝒂𝑨𝒏−𝟏
2sin 𝑥 +2cos 𝑥
∴ ≥ √2sin 𝑥 . 2cos 𝑥 =
2
Ans.(a) 1 1 𝜋
( ) sin(𝑥+ )
2(2)(sin 𝑥+cos 𝑥) = 2 √2 4
𝛼 +𝛽= a, 𝛼𝛽= b
1 𝜋 1
sin 𝑥 cos 𝑥 1+ sin(𝑥+ ) 1−
𝑛 𝑛 ⟹2 +2 ≥2 4 ≥2
Given, 𝐴𝑛 = 𝛼 + 𝛽 √2 √2

Now, 𝐴𝑛+1 = 𝛼 𝑛+1 + 𝛽 𝑛+1 [∵ least value of sin (𝑥 + 4 ) = −1]


𝜋

= (𝛼 𝑛 + 𝛽 𝑛 )( 𝛼 +𝛽)-𝛼𝛽(𝛼 𝑛−1 + 𝛽 𝑛−1)

= a𝐴𝑛 + 𝑏𝐴𝑛−1
26. How many different nine digit
numbers can be formed from the number
223355888 by rearranging its digits so
24. If x satisfies |x-1|+|x-2|+|x-3|≥ 6, then that the odd digits occupy even positions?

(a) 0≤ 𝒙 ≤ 𝟒 (b) 𝒙 ≤ −𝟐 𝒐𝒓 𝒙 ≥ 𝟒 (a) 16 (b) 36


(c) 𝒙 ≤ 𝟎 𝒐𝒓 𝒙 ≥ 𝟒 (d) 𝒙 ≥ 𝟎 (c) 60 (d) 180

Ans. (c) Ans. (c)

For x ≤ 1, −3𝑥 + 6 ≥ 6 ⟹ 𝑥 ≤ 0 Number of digits= 9


……(A)
Number of odd digits = 4, number of even
For 1≤ 𝑥 ≤ 2, −𝑥 + 4 ≥ 6⟹ x≤ −2 digits= 5

(not acceptable as 1≤ 𝑥 ≤ 2) Number of even places= 4

For x≥ 3, 3𝑥 − 6 ≥ 6 ⟹ 𝑥 ≥ 4 Odd digits can be arranged in even paces in


………(B) |4̲
ways. Even digits can be arranged in
|2̲|2̲
|5̲
From (A) and (B) all positive value of x are remaining 5 places in |2̲|3̲ ways
given by x≤ 0 𝑜𝑟 𝑥 ≥ 4
|4̲ |5̲
∴ Required number = |2̲|2̲ . |2̲|3̲ = 60

66
Solving Mathematical Problems

27. For 2≤ 𝒓 ≤ 𝒏, (𝒏𝒓) + 𝟐(𝒓−𝟏


𝒏 𝒏
) + (𝒓−𝟐)= 𝑠𝑖𝑛𝑐𝑒 𝑠𝑖𝑛20 𝑥 𝑎𝑛𝑑 𝑐𝑜𝑠 48 𝑥 𝑐𝑎𝑛′𝑡 𝑏𝑒 𝑧𝑒𝑟𝑜 𝑎𝑡 𝑎 𝑡𝑖𝑚𝑒

(a) (𝒏+𝟏 ) (b) 2(𝒏+𝟏 ) ⟹ 0 < f(x) ≤ 1


𝒓−𝟏 𝒓+𝟏
(c) 2(𝒏+𝟐
𝒓
) (d) (𝒏+𝟐
𝒓
) Hence range of f(x) = (0, 1)

Ans. (d)

(𝑛𝑟) 𝑠𝑡𝑎𝑛𝑑𝑠 𝑓𝑜𝑟 𝑛𝐶𝑟 30. Let x, y, z = 105, where x, y, z ∊N.


Then number of ordered triplets (x, y, z)
Now 𝑛𝐶𝑟 + 2𝑛𝐶𝑟−1 + 𝑛𝐶𝑟−2 satisfying the given equation is:

= (𝑛𝐶𝑟 + 𝑛𝐶𝑟−1 ) + (𝑛𝐶𝑟−1 + 𝑛𝐶𝑟−2 ) = (a) 15 (b) 27


𝑛 + 1𝐶𝑟 + 𝑛 + 1𝐶𝑟−1 = 𝑛 + 2𝐶𝑟 (c) 6 (d) none of these

Ans. (b)

28. If ∑𝟏𝟎 −𝟏
𝑿𝒊 = 𝟓𝝅, 𝒕𝒉𝒆𝒏 ∑𝟏𝟎 𝟐 105= 3× 5 × 7
𝒊=𝟏 𝐬𝐢𝐧 𝒊=𝟏 𝑿𝒊 =

(a) 0 (b) 5 When no 1 is taken as a solution, number of


(c) 10 (d) none of these solutions= |3̲ = 6

Ans. (c) When only 1’ s taken, number of solutions=


3𝐶2 . |3̲ = 18
10
𝜋
∑ sin−1 𝑥𝑖 = 5𝜋 = 10. When two 1’s are taken, number of solutions
2 |3
̲
𝑖=1 = 3𝐶1 . |2̲ = 3
𝜋
⟹ sin−1 𝑥𝑖 = 2 , ∀𝑖 ⟹ 𝑥𝑖 = 1∀𝑖
∴ Reqd. number= 6+18+3 =27
⟹ ∑10 2
𝑖=1 𝑥𝑖 = 1 Second method: xyz = 3× 5 × 7

3 will be a factor of x or y or z in 3 ways


29. Range of f(x) = 𝒔𝒊𝒏𝟐𝟎 𝒙 + 𝒄𝒐𝒔𝟒𝟖 𝒙 is 5 will be a factor of x or y or z in 3 ways
(a) [0, 1] (b) (0, 1) 7 will be a factor of x or y or z in 3 ways
(c) (0, ∞) (d) none of
these ∴Total number of ways=3× 3 × 3 = 27

Ans. (b)

0 ≤ 𝑠𝑖𝑛2 𝑥 ≤ 1 ⟹ 𝑠𝑖𝑛20 𝑥 ≤ 𝑠𝑖𝑛2 𝑥

Thus 0 ≤ 𝑠𝑖𝑛20 𝑥 ≤ 𝑠𝑖𝑛2 𝑥 ………. (1)

Again 0 ≤ 𝑐𝑜𝑠 48 𝑥 ≤ 𝑐𝑜𝑠 2 𝑥 ………..(2)

67
Solving Mathematical Problems

ISI OBJECTIVE SAMPLE PAPER f’(x) ≤ 0


WITH SOLUTIONS 3
∴ −𝛼 2 + 𝑝𝛼 − 4 ≤ 0 ⟹4𝛼 2 − 4𝑝𝛼 + 3 ≥
SET – 9 0, ∀ 𝛼 ∊ 𝑅

∴ D ≤ 0⟹ 16𝑝2 − 48 ≤ 0 ⟹ −√3 ≤ 𝑝 ≤
√3
There will be 30 questions in MMA Paper.
For each question, exactly one of the four 2. Consider the following statements S
choices is correct. You get four marks for and R. S: both sinx and cosx are
each correct answer, one mark for each 𝝅
decreasing function in (𝟐 , 𝝅) & R: If a
unanswered question, and zero mark for
each incorrect answer. differentiable function decreases in (a, b)
then its derivative also decreases in (a, b).
𝒙 Which of the following are true?
1. If f(x) = (𝒑𝜶 − 𝜶𝟐 − 𝟐)𝒙 − ∫𝟎 (𝒄𝒐𝒔𝟒 𝒕 +
𝒔𝒊𝒏𝟐 𝒕 − 𝟐) 𝒅𝒕 is a decreasing function of x (a) both S and R are wrong
for all x ∊R and 𝛼∊ R, where 𝛼 being (b) S is correct and R is wrong
independent of x, then (c) both S and R are correct but R is not
the correct expiation for S
(a) p∊ (-∞, 𝟏) (b) p∊ (-1, √𝟑)
(c) p∊ (1, ∞) (d) none of these (d) S is correct and R is the correct
explanation for S
Ans. (b) Given, 𝑓(𝑥) = (𝑝𝛼 − 𝛼 2 − 2)𝑥 −
𝑥
∫0 (𝑐𝑜𝑠 4 𝑡 + 𝑠𝑖𝑛2 𝑡 − 2) 𝑑𝑡 ……….(1) Ans. (b) From the trend of value of sin x and
cos x we know sin x and cosx decrease in
∴𝑓 ′ (𝑥) = 𝑝𝛼 − 𝛼 2 − 2 − (𝑐𝑜𝑠 4 𝑥 + 𝜋
< 𝑥 < 𝜋. So, the statement S is correct.
𝑠𝑖𝑛2 𝑥 − 2) = −𝛼 2 + 𝑝𝛼 − (𝑐𝑜𝑠 4 𝑥 + 2

𝑠𝑖𝑛2 𝑥) The statement R is incorrect cos x is a


differentiable function which decreases in
= −𝛼 2 + 𝑝𝛼 − (𝑐𝑜𝑠 4 𝑥 + 𝑐𝑜𝑠 2 𝑥 + 1) 𝜋
( 2 , 𝜋) but its d.c. – sin x is increasing in
= −𝛼 2 + 𝑝𝛼 − 𝜋
( 2 , 𝜋)
1 2 3 3
[(𝑐𝑜𝑠 2 𝑥 − 2) + 4] = −𝛼 2 + 𝑝𝛼 − 4 −
𝒙𝟐 +𝟏 −𝒕𝟐
2 1 2 3. If f(x)= ∫𝒙𝟐 𝒆 𝒅𝒕, then the interval
(𝑐𝑜𝑠 𝑥 − 2)
in which f(x) is increasing is
3
Clearly f’(x) ≤ −𝛼 2 + 𝑝𝛼 − 4 [∵ (a) (0, ∞) (b) (-∞, 𝟎)
1 2 (c) [-2, 2] (d) none of these
𝑀𝑖𝑛. 𝑣𝑎𝑙𝑢𝑒 𝑜𝑓 (𝑐𝑜𝑠 2 𝑥 − 2) = 0]
Ans. (b)
For f(x) to be decreasing for all real x,

68
Solving Mathematical Problems

𝑥 2 +1 𝒙𝟐 −𝟐
−𝑡 2 ′ (𝑥) 5. If ∫ =
𝑓(𝑥) = ∫ 𝑒 𝑑𝑡 𝑓 𝒙𝟐 +𝟐
𝒅𝒙
(𝒙𝟒 +𝟓𝒙𝟐 +𝟒) 𝐭𝐚𝐧−𝟏 ( )
𝑥2 𝒙
2 2 4
= 𝑒 −(𝑥 +1) . 2𝑥 − 𝑒 −𝑥 . 2𝑥 𝒍𝒐𝒈|𝒇(𝒛)| + 𝒄, then
2𝑥
= (𝑥 2 +1)2 [1 (a) f(z) = 𝐭𝐚𝐧−𝟏 𝒛 , 𝒘𝒉𝒆𝒓𝒆 𝒛 = √𝒙 + 𝟐
𝑒 𝟐
−𝑥 4 +(𝑥 2 +1)
2 (b) f(z) = 𝐭𝐚𝐧−𝟏 𝒛 , 𝒘𝒉𝒆𝒓𝒆 𝒛 = 𝒙 + 𝒙
−𝑒 ]
𝒙+𝟐
2𝑥 (c) f(z)= 𝐬𝐢𝐧−𝟏 𝒛 , 𝒘𝒉𝒆𝒓𝒆 𝒛 =
2𝑥 2 +1 𝒙
= 2 +1)2 [1 − 𝑒 ]
𝑒 (𝑥 (d) none of these
2 +1
2(𝑒 2𝑥 − 1)
= (𝑥 2 +1)2
(−𝑥) Ans. (b)
𝑒
𝑥 2 −2
But 𝑒 2𝑥
2 +1
>1 I= ∫ 𝑥2 +2
𝑑𝑥
(𝑥 4 +5𝑥 2 +4) tan−1 ( )
𝑥

∴ f’(x)> 0 in (-∞, 0) and hence f(x) is


Dividing numerator & denominator by 𝑥 2
increasing in (-∞, 0)
we have
𝒙 (𝒕−|𝒕|)𝟐 2
4. The value of ∫𝟎 𝒅𝒕 is equal to 1− 2
𝟏+𝒕𝟐 =∫ 𝑥
𝑑𝑥 =
4 2
(𝑥 2 +5+ 2 ) tan−1 (𝑥+ )
𝑥 𝑥
(a) 4(x-𝐭𝐚𝐧−𝟏 𝒙) if x < 0 (b) 0 if > 0 1− 2
2
𝑥
(c) 𝐥𝐨𝐠(𝟏 + 𝒙𝟐 ) if x > 0 (d) none of ∫ 2 2 𝑑𝑥
[(𝑥+ ) 2 +1] tan−1 (𝑥+ )
𝑥 𝑥
these
2
Ans. (a) 𝑙𝑒𝑡 tan−1 (𝑥 + ) = 𝑢
𝑥
𝑥
(𝑡 − |𝑡|)2 1 2
𝐼=∫ 𝑑𝑡 ⟹ . (1 − ) = 𝑑𝑥 = 𝑑𝑢
0 1 + 𝑡2 2 𝑥2
1 + (𝑥 + 𝑥 ) 2
𝑐𝑎𝑠𝑒 𝐼: 𝑥 > 0 , 𝑡ℎ𝑒𝑛 0 < 𝑡 < 𝑥, |𝑡| = 𝑡
1
𝑥 (𝑡−𝑡)2
𝑛𝑜𝑤 𝐼 = ∫ 𝑑𝑢 = log |𝑢| + 𝑐
∴ I= ∫0 1+𝑡 2 𝑑𝑡 =0 𝑢
2
= 𝑙𝑜𝑔 |tan−1 (𝑥 + )|
Case II: x < 0, then x < t< 0⟹ |t|= -t 𝑥
= tan−1 𝑧, 𝑤ℎ𝑒𝑟𝑒 𝑧
𝑥 (𝑡+𝑡)2 𝑥 4𝑡 2 𝑥 2
∴ I= ∫0 𝑑𝑡 = ∫0 𝑑𝑡 = 4 ∫0 (1 − = (𝑥 + )
1+𝑡 2 1+𝑡 2
1
𝑥
) 𝑑𝑡 = 4[𝑡 − tan−1 𝑡] 𝑥0 = 4(𝑥 −
1+𝑡 2 𝟏
tan −1
𝑥) 6. ∫ 𝒙 𝐥𝐨𝐠 (𝟏 + 𝒙) 𝒅𝒙 = 𝒇(𝒙) 𝐥𝐨𝐠(𝒙 + 𝟏) +
𝒈(𝒙)𝒙𝟐 + 𝑳𝒙 + 𝒄, then
𝟏
(a) L= 1 (b) f(x) = 𝟐 𝒙𝟐
(c) g(x) = log x (d) none of these
69
Solving Mathematical Problems

Ans. (d) (𝑥 − 1). 1 − (𝑥 − 2). 1


𝑑𝑥 = 𝑑𝑥
(𝑥 − 1)2
1
I= ∫ 𝑥 log (1 + 𝑥) 𝑑𝑥 = ∫ 𝑥 𝑙𝑜𝑔 (𝑥 + 3
= − 𝑑𝑥
1)𝑑𝑥 − ∫ 𝑥 𝑙𝑜𝑔 𝑥 𝑑𝑥 (𝑥 − 1)2

𝑥2 1 𝑥2 𝑥2 1 4
= log(𝑥 + 1) − ∫ 𝑑𝑥 − 𝑙𝑜𝑔𝑥 𝑛𝑜𝑤 𝐼 = ∫ 𝑧 −5/4 𝑑𝑧 = 𝑧 −1/4 + 𝐶
2 2 1+𝑥 2 3 3
1 4 𝑥 − 1 1/4
+ ∫ 𝑥 𝑑𝑥 = .( ) +𝐶
2 3 𝑥+2
𝑥2 𝑥2
= log(𝑥 + 1) − 𝑙𝑜𝑔𝑥
2 2
1 1 𝟏+𝒏.𝒙𝒏−𝟏 −𝒙𝟐𝒏
− ∫ (𝑥 − 1 + ) 𝑑𝑥 8. ∫ 𝒆𝒙 𝒅𝒙=
2 𝑥+1 (𝟏−𝒙𝒏 )√𝟏−𝒙𝟐𝒏
1
+ ∫ 𝑥 𝑑𝑥 𝒆𝒙 √𝟏−𝒙𝟐𝒏 𝒆𝒙 √𝟏−𝒙𝒏
2 (a) +𝒄 (b) +𝒄
𝟏−𝒙𝟐𝒏 𝟏−𝒙𝒏
𝑥2 𝑥2 1 𝒆𝒙 √𝟏−𝒙𝟐𝒏
= log(𝑥 + 1) − log 𝑥 − log(𝑥 + 1) (c) +𝒄 (d) none of these
𝟏−𝒙𝒏
2 2 2
𝑥
+ +𝑐 Ans. (c)
2
1−𝑥 2𝑛 +𝑛𝑥 𝑛−1 √1−𝑥 2𝑛
𝑥2 1 1 I= ∫ 𝑒 𝑥 [(1−𝑥 𝑛)√1−𝑥 2𝑛] 𝑑𝑥 = ∫ 𝑒 𝑥 [ +
𝑓(𝑥) = − , 𝑔(𝑥) = − log 𝑥 1−𝑥 𝑛
2 2 2
𝑛𝑥 𝑛−1 1−𝑥 𝑛
1 √
(1−𝑥 𝑛 )2 1+𝑥 𝑛
] 𝑑𝑥 = ∫ 𝑒 𝑥 {𝑓(𝑥) +
𝐿= .
2 𝑓 ′ (𝑥)]𝑑𝑥,
𝒅𝒙
7. ∫ 𝟑 𝟓 =
(𝒙−𝟏)𝟒 (𝒙+𝟐)𝟒 1 − 𝑥 2𝑛
𝑤ℎ𝑒𝑟𝑒 𝑓(𝑥) = √ = 𝑒 𝑥 𝑓(𝑥) + 𝐶
𝟏 1 − 𝑥𝑛
𝟒 𝒙−𝟏 𝟒 𝟒 𝒙−𝟏
(a) ( ) +𝒄 (b) 𝟑 √𝒙+𝟐 + 𝒄 1 − 𝑥 2𝑛
𝟑 𝒙+𝟐
= 𝑒𝑥 +𝐶
𝟏 1 − 𝑥𝑛
𝒙+𝟐 𝟒
(c) (𝒙−𝟏) +𝒄 (d) none (𝒙+𝟏)
9. ∫ 𝒙(𝟏+𝒙𝒆𝒙)𝟐 𝒅𝒙 = 𝒍𝒐𝒈|−𝒇(𝒙)| + 𝒇(𝒙) +
Ans. (a) 𝒄 𝒕𝒉𝒆𝒏 𝒇(𝒙) =
𝑑𝑥 𝟏 𝟏
𝐼= ∫ 5
(a) 𝒙+𝒆𝒙 (b) 𝒙+𝒙𝒆𝒙
𝑥+2 4 𝟏
(𝑥 − 1)2 (
𝑥 − 1)
(c) (𝟏+𝒙𝒆𝒙)𝟐 (d) none

𝑥+2 Ans. (b)


𝑝𝑢𝑡 𝑧 = , 𝑡ℎ𝑒𝑛
𝑥−1
Put z= x𝑒 𝑥 , then dz = (𝑒 𝑥 + 𝑥𝑒 𝑥 ) dx

70
Solving Mathematical Problems
𝑑𝑧 1 1 1 𝜋
I= ∫ 𝑧(1+𝑧)2 = ∫ [𝑧 − 1+𝑧 − (1+𝑧)2 ] 𝑑𝑧 = 4
𝑛𝑜𝑤 𝐼𝑛 = ∫ 𝑡𝑎𝑛𝑛−2 𝑥𝑠𝑒𝑐 2 𝑥 𝑑𝑥
𝑧 1 𝑥𝑒 𝑥 1 0
log 1+𝑧 + 1+𝑧 + 𝐶 = 𝑙𝑜𝑔 |1+𝑥𝑒 𝑥 | + 1+𝑥𝑒 𝑥 + 𝑐 1
= ∫ 𝑧 𝑛−2 𝑑𝑧, 𝑝𝑢𝑡𝑡𝑖𝑛𝑔 𝑧
1 1 0
= log |1 − | + +𝐶
1 + 𝑥𝑒 𝑥 1 + 𝑥𝑒 𝑥 = tan 𝑥
𝑧 𝑛−1 1
= [ 𝑛−1 ] 10 = 𝑛−1
𝝅
𝐬𝐢𝐧(𝟐𝒏−𝟏)𝒙
10. If 𝑰𝒏 = ∫𝟎𝟐 𝒅𝒙, 𝒂𝒏𝒅 𝒂𝒏 =
𝒔𝒊𝒏𝒙
𝝅
𝐬𝐢𝐧 𝒏𝜽
∫𝟎𝟐 ( 𝐬𝐢𝐧 𝜽 ) 𝟐 𝒅𝜽, 𝒕𝒉𝒆𝒏 𝒂𝒏+𝟏 − 𝒂𝒏 = 12. If f(𝛼)= f(𝛽) and n ∊N, then the value
𝜷 ′′
(a) 𝑰𝒏 (b) 2𝑰𝒏 of ∫𝜶 (𝒈(𝒇(𝒙))) 𝒈′ (𝒇(𝒙)). 𝒇′ (𝒙) 𝒅𝒙 =
(c) 𝑰𝒏 + 𝟏 (d) 0
(a) 1 (b) 0
Ans. (c) 𝑎𝑛+1 − 𝑎𝑛 = 𝜷𝒏+𝟏 −𝜶𝒏+𝟏
(c) (d) none of these
𝜋 𝒏+𝟏
𝑠𝑖𝑛2 (𝑛+1)𝑥−𝑠𝑖𝑛2 𝑛𝑥
∫ 2 𝑑𝑥
0 𝑠𝑖𝑛2 𝑥
Ans. (b)
𝜋
2 sin(2𝑛 + 1) 𝑥𝑠𝑖𝑛 𝑥 Put z = g(f(x)), then dz = g’(f(x)) f’(x) dx
=∫ 𝑑𝑥
0 𝑠𝑖𝑛2 𝑥
𝜋 𝑧 𝑛+1
2 sin(2𝑛 + 1)𝑥 = ∫ 𝑧 𝑛 𝑑𝑧 = 𝑛+1
= ∫ 𝑑𝑥
0 sin 𝑥 1 𝑛+1 𝛽
= 𝐼𝑛+1 𝐼 = 𝑛+1 [{𝑔(𝑓(𝑥))] ] 𝛼
=
1 𝑛+1 𝑛+1
𝝅 [[{𝑔(𝑓(𝛼))] − [{𝑔(𝑓(𝛽))] ]= 0
𝑛+1
11. If n ≠ 𝟏, ∫𝟎 (𝒕𝒂𝒏𝒏 𝒙 + 𝒕𝒂𝒏𝒏−𝟐 𝒙) 𝒅(𝒙 −
𝟒
[∵ f(𝛼)= f(𝛽)]
[𝒙])=
13. Let [x] denotes the integral part of a
𝟏 𝟏
(a) 𝒏−𝟏 (b) 𝒏+𝟏 real number x and {x} = x- [x], then
𝟏 𝟐 solution of 4{x}= x+ [x] are
(c) 𝒏 (d) 𝒏−𝟏
𝟐 𝟒
(a) ± 𝟑 , 𝟎 (b) ± 𝟑 , 𝟎
Ans. (a)
𝟓
𝜋
(c) 0, 𝟑 (d) ±𝟐, 𝟎
Let 𝐼𝑛 = ∫0 4 (𝑡𝑎𝑛𝑛 𝑥 + 𝑡𝑎𝑛 𝑛−2
𝑥) 𝑑(𝑥 − [𝑥])
Ans. (c)
𝜋
ℎ𝑒𝑟𝑒 0 < 𝑥 < ∴ [𝑥] = 0 ∴ 𝑥 − [𝑥] = 𝑥 4{x}= x+ [x]= [x]+ {x} +[x]
4
2
⟹ {x}= 3 [𝑥] …….(1)

Since 0 ≤ {𝑥} < 1

71
Solving Mathematical Problems
2 3 𝟐 −𝟒𝒙+𝟓)
∴ 0≤ 3 [𝑥] < 1 ⟹ 0 ≤ [𝑥] < 2 15. The roots of equation 𝟕 𝐥𝐨𝐠𝟕(𝒙
are
Hence [x]= 0, 1
(a) 4, 5 (b) 2, -3
2
∴{x}= 0, 3 [from (1)] (c) 2, 3 (d) 3, 5

5 Ans. (c)
∴ x= [x]+ {x}= 0, 3
Given, 𝑥 2 − 4𝑥 + 5 = 𝑥 − 1
14. The maximum number of real roots of
the equation 𝒙𝟐𝒏 − 𝟏= 0 (n ∊N) is ⟹ 𝑥 2 − 5𝑥 + 6 = 0⟹ x= 2, 3

(a) 2 (b) 3 𝒂𝟐 𝒃𝟐 𝒄𝟐
16. Equation 𝒙−𝜶 + 𝒙−𝜷 + 𝒙−𝜸 = 𝒎 − 𝒏𝟐 𝒙
(c) n (d) 2n
(a, b, c, m, n ∊ r) has necessarily
Ans. (a)
(a) all the roots real
2𝑛 2𝑛
𝑥 −1=0⟹ 𝑥 = 1 = cos 0 + 𝑖 sin 0 (b) all the roots imaginary
2𝑟𝜋 2𝑟𝜋 𝑟𝜋 (c) two real and two imaginary roots
∴ 𝑥 = cos + 𝑖 sin = cos +
2𝑛 2𝑛 𝑛
𝑟𝜋 (d) two rational and two irrational roots
𝑖 sin , 𝑟 = 0, 1, … . , (2𝑛 − 1)
𝑛
Ans.(a)
𝑟𝜋
x will be real only when sin =0
𝑛
Let p + iq be a root of given equation, then
𝑟𝜋
or = 𝑚𝜋 𝑎2 𝑏2 𝑐2
𝑛
+ 𝑝−𝛽+𝑖𝑞 + 𝑝−𝛾+𝑖𝑞 = 𝑚 − 𝑛2 (𝑝 +
𝑝−𝛼+𝑖𝑞
or r = mn = a multiple of n 𝑖𝑞)
But, r= 0, 1, 2,, …, 2n- 1 𝑎2 [𝑝−𝛼−𝑖𝑞] 𝑏 2 [(𝑝−𝛽)−𝑖𝑞] 𝑐 2 [(𝑝−𝛾)−𝑖𝑞]
⟹ (𝑝−𝛼)2 +𝑞2 + + =
(𝑝−𝛽)2 +𝑞2 (𝑝−𝛾)2 +𝑞2
∴ r = 0, n 𝑚 − 𝑛2 𝑝 − 𝑖𝑛2 𝑞
∴ 𝑥 2𝑛 − 1 = 0 has only two real root 1, -1. Equating imaginary parts we get
Second method: Let f(x) = 𝑥 2𝑛 − 1 𝑎2 𝑏2 𝑐2
𝑞 [{(𝑝−𝛼)2 +𝑞2 + (𝑝−𝛽)2 +𝑞2 + (𝑝−𝛾)2 +𝑞2 } +
Then, f’(x) = 2𝑛𝑥 2𝑛−1
𝑛2 ] = 0
Sign scheme for f’(x) is
∴ q= 0.
Hence graph, of y= f(x) will either intersect
x –axis at two points or touch x-axis or will Hence p +iq= p= a real number.
not interest x-axis or will not interest x-axis. 17. If a, b, c ∊ { 1, 2, 3, 4, 5}, the number
Therefore eqn. f(x)= 0 has two distinct real of equations of the form 𝒂𝒙𝟐 + 𝒃𝒙 + 𝒄 =
roots or two equal real roots or no real root.
𝟎 which have real roots is

72
Solving Mathematical Problems

(a) 25 (b) 26 (a) A.P. (b) G.P. (c) H.P. (d) none of
(c) 207 (d) 24 these.

Ans. (d) Ans. (a) (𝑥 − 𝑎1 + 𝑎2 )2 + (𝑥 − 𝑎2 +


𝑎3 )2 + ⋯ + (𝑥 − 𝑎𝑛−1 + 𝑎𝑛 )2 = 0
𝑏2
For real roots ac ≤ 4
⟹𝑎1 − 𝑎2 = 𝑎2 − 𝑎3 = ⋯ = 𝑎𝑛−1 −
B 𝑏 2 Possible value No. of 𝑎𝑛 = 𝑥
4 of ac such that possible
𝑏2
⟹ 𝑎1 , 𝑎2 , 𝑎3 , … . 𝑎𝑛 are in A.P. with
ac ≤ pairs (a, c)
4 common difference x.

2 1 1 1 19. Let f(x) = 𝒂𝒙𝟐 + 𝒃𝒙 + 𝒄 and g(x) =


af(x) + bf′(x) + cf″(x) If f(x) > 0 for all x ,
3 2.25 1.2 3
then the sufficient condition for g(x) to be
4 4 1, 2, 3, 4 8 > 0 v x is

5 6.25 1, 2, 3, 4, 5, 6 12 (a) c > 0 (b) b > 0


(c) b< 0 (d) c < 0
Total 24
Ans. (d)

g(x)= 𝑎(𝑎𝑥 2 + 𝑏𝑥 + 𝑐) + 𝑏(2𝑎𝑥 + 𝑏) +


Value of ac Possible pairs (a, c) 𝑐 + 2𝑎 = 𝑎2 𝑥 2 + 3𝑎𝑏𝑥 + 𝑏 2 + 3𝑎𝑐

1 (1, 1) discriminant of its corresponding equation ,

2 (1, 2), (2, 1) D = 9𝑎2 𝑏 2 − 12𝑎3 𝑐

3 (1, 3), (3, 1) = 9𝑎2 𝑏 2 − 36𝑎3 𝑐 + 24𝑎3 𝑐

4 (1, 4), (4, 1), (2, 2) = 9𝑎2 (𝑏 2 − 4𝑎𝑐) + 24𝑎3 𝑐 ……(1)

5 (1, 5), (5, 1) Since f(x)> 0, ∀ 𝑥 ∊ 𝑅

6 (2, 3), (3, 2) ∴ a > 0 and 𝑏 2 − 4𝑎𝑐< 0

Hence number of quadratic equations For g(x)> 0 ∀ 𝑥 ∊ 𝑅, 𝑎2 > 0 𝑎𝑛𝑑 𝐷 < 0


having real roots = 24
But from (1), D < 0 when c < 0
18. If x, 𝒂𝟏 , 𝒂𝟐 , 𝒂𝟑 , … , 𝒂𝒏 ∊ 𝑹 𝒂𝒏𝒅 (𝒙 −
21. The constant term of the quadratic
𝒂𝟏 + 𝒂𝟐 )𝟐 + (𝒙 − 𝒂𝟐 + 𝒂𝟑 )𝟐 + ⋯
expression
𝟏 𝟏
+(𝒙 − 𝒂𝒏−𝟏 + 𝒂𝒏 )𝟐 = 0, ∑𝒏𝒌=𝟏 (𝒙 − ) (𝒙 − 𝒌) 𝒂𝒔 𝒏 ⟶ ∞ is
𝒌+𝟏
then 𝒂𝟏 , 𝒂𝟐 , 𝒂𝟑 , … , 𝒂𝒏 are in

73
Solving Mathematical Problems

(a) -1 (b) 0 3
Thus |𝑧| > 1, 𝑐𝑙𝑒𝑎𝑟𝑙𝑦 |𝑧| > 4
(c) 1 (d) none of these
22. Number of solutions of 𝟑|𝒙| = |𝟐 −
Ans. (c) Constant term
|𝒙|| is
1 1 1 1
c = 1.2 + 2.3 + ⋯ + 𝑛(𝑛+1) = 1 − 𝑛+1 (a) 0 (b) 2
(c) 4 (d) infinite
1
lim 𝑐 = lim (1 − )=1
𝑛→∞ 𝑛→∞ 𝑛+1 Ans. (b)
𝝅
21. If 𝜽𝒊 ∊ [𝟎, 𝟔 ] , 𝒊 = 𝟏, 𝟐, 𝟑, 𝟒, 𝟓 and Given equation is
𝒔𝒊𝒏𝜽𝟏 𝒛𝟒 + 𝒔𝒊𝒏𝜽𝟐 𝒛𝟑 + 𝒔𝒊𝒏𝜽𝟑 𝒛𝟐 + 1 𝑥
(3) = 2 − 𝑥, − ∞ < 𝑥 ≤ −2
𝒔𝒊𝒏𝜽𝟒 𝒛 + 𝒔𝒊𝒏𝜽𝟓 = 𝟐 then z satisfies
𝟑 𝟏 𝟏 𝟑 = 2+ x, -2 ≤ 𝑥 ≤ 0
(a) |z|> 𝟒 (b) |z|< 𝟐 (c) < |z|< 𝟒 (d)
𝟐
none of these 3𝑥 = 2 − 𝑥, 0 ≤ 𝑥 ≤ 2

Ans.(a) = x – 2, 2≤ 𝑥 < ∞
𝜋
Since 0≤ 𝜃𝑖 , ≤ At x = 2, 3𝑥 − 𝑥 + 2 = 9
6

1 For x >2, 3𝑥 − 𝑥 + 2 > 9


∴ 0≤ 𝑠𝑖𝑛𝜃𝑖 , ≤ 2
(As 3𝑥 − 𝑥 + 2 is an increasing function for
From given condition x > 2)
|2|= |𝑠𝑖𝑛𝜃5 + 𝑧𝑠𝑖𝑛𝜃4 + 𝑧 2 𝑠𝑖𝑛𝜃3 + 1 𝑥
𝑧 3 𝑠𝑖𝑛𝜃2 + 𝑧 4 𝑠𝑖𝑛𝜃1 | For x= -2, (3) + 2 + 𝑥 = 9

⟹ 2≤ |𝑠𝑖𝑛𝜃5 | + |𝑧||𝑠𝑖𝑛𝜃4 | + 1 𝑥
For x < -2, (3) + 2 + 𝑥 < 9
1
|𝑧|2 |𝑠𝑖𝑛𝜃3 |+|𝑧|3 |𝑠𝑖𝑛𝜃2 |+|𝑧|4 |𝑠𝑖𝑛𝜃1 | ≤ +
2
1 1 1 1 1 1 𝑥
|𝑧| + |𝑧| + 2 |𝑧|3 +2 |𝑧|4 | < 2 |𝑧| + [as (3) + 2 + 𝑥 𝑖𝑠 𝑑𝑒𝑐𝑟𝑒𝑎𝑠𝑖𝑛𝑔]
2 2 2
|𝑧|2 + ⋯ 𝑡𝑜 ∞ ….(1)
Hence given equation has only two solutions
When|𝑧| < 1, from (1), -2 and 2.

1 1
2 < 2 . 1−|𝑧| 23. The number of real roots of the
𝟏
equation (𝟗 + 𝒔𝒊𝒏𝒙)𝟏−𝒙 + (𝟏𝟎 +
1 3 𝟏 𝟏
∴ 1 − |𝑧| < 4 ⟹ |𝑧| > 4 𝒔𝒊𝒏𝒙)𝟏−𝒙 = (𝟏𝟏 + 𝒔𝒊𝒏𝒙)𝟏−𝒙 for x ∊ (0, 1)
3
is
When |z|> 1, clearly |𝑧| > 4

74
Solving Mathematical Problems

(a) exactly one (b) at least ∴𝑚≤1


one (c) at most one (d)
none of these Let f(x) = (𝑥 − 𝑎1 )101 + (𝑥 − 𝑎2 )101 + ⋯ +
(𝑥 − 𝑎𝑛 )101
Ans. (a)
∴ f’(x) > 0 ⟹ f(x) is an increasing function
Given eqn. is f(x) = 1,
Also f(−∞) = −∞ < 0 𝑎𝑛𝑑 𝑓(∞) = ∞ >
1
1 1−𝑥 0
where f(x) = (1 + 10+sin 𝑥) − (1 −
1
1 ∴ f(x) = 0 has exactly one real root
1−𝑥
)
10+sin 𝑥
∴ n= 1
Clearly
Hence m ≤ 𝑛.
1
f(0)= 5 < 1 𝑎𝑛𝑑 𝑓(1 − 0) = ∞
25. If m be number of integral solutions of
Also f(x) is an increasing function equation 𝟐𝒙𝟐 − 𝟑𝒙𝒚 − 𝟗𝒚𝟐 − 𝟏𝟏 = 𝟎 and
n be the number of real solutions of
∴ f(x)= 1 only for one value of x. equation 𝒙𝟑 − [𝒙] − 𝟑= 0, then m =

24. If 0 < 𝛼r < 1 for r= 1, 2, 3, …, k and m (a) n (b) 2n


be the number of real solutions of (c) n/2 (d) 3n
equation
Ans. (b)
∑𝒌𝒓=𝟏(𝒂𝒓 )𝒙 = 𝟏 & n be the number of real
Given, 2𝑥 2 − 3𝑥𝑦 − 9𝑦 2 − 11 = 0
solution of equation ∑𝒌𝒓=𝟏(𝒙 − 𝒂𝒓 )𝟏𝟎𝟏 =
𝟎, then ⟹ (2x +3y)(x- 3y)= 11

(a) m= n (b) m ≤ n 2𝑥 + 3𝑦 = 1 2𝑥 + 3𝑦 = 11
∴ }, }
(c) m ≥ n (d) m > n 𝑥 − 3𝑦 = 11 𝑥 − 3𝑦 = 1

Ans. (b) 2𝑥 + 3𝑦 = −1 2𝑥 + 3𝑦 = −11


}, }
𝑥 − 3𝑦 = −11 𝑥 − 3𝑦 = −1
Let 𝛼 be a root of eqn.
∴ x= 4, y= 1, x= -4, y= -1
𝑥 𝑥 𝑥
𝑎1 + 𝑎2 + ⋯ + 𝑎𝑘 = 1
…………….(1) ∴ m= 2

Then when x < 𝛼, L.H.S. of (1)> 1 Again, given

And when x > 𝛼, L.H.S. of (1)< 1 𝑥 3 − [𝑥] − 3 = 0 ⟹𝑥 3 − (𝑥 − 𝛼) − 3 = 0,

Hence, eqn. (1) cannot have more than one Where 𝛼= {x}= x-[x]
root.
⟹𝑥 3 − 𝑥 = 3 − 𝛼. But 0 ≤ 𝛼 < 1

75
Solving Mathematical Problems
𝜋
∴ 2 < 𝑥3 − 𝑥 ≤ 3 ∴ k= sin−1 1 = (∵k> 0)
2

For x ≥ 2, 𝜋
∴ [k] = [2 ]= 1
𝑥 3 − 𝑥 = 𝑥(𝑥 2 − 1) ≥ 2(22 − 1) = 6
Given equation is (x-1)(x-𝛼)-1= 0
For x≤ −1, 𝑥 3 − 𝑥 = 𝑥(𝑥 2 − 1) < 0
⟹ (x- 1)(x+ 𝛼)= 1 ……. (1)
3
For -1 < x< 0, 𝑥 − 𝑥 < 1 < 2
We have to find integral value of 𝛼 for
3 3
For 0 < x ≤ 1, 𝑥 − 𝑥 < 𝑥 < 1 < 2 which equation (1) has integral roots.

For x= 0, 𝑥 3 − 𝑥 = 0 < 2 ∴ x and 𝛼 are integers.

∴1 < x< 2 ∴ [x]=1 From (1), (i) x- 1 = 1⟹ x= 2

∴Given equation becomes X+ 𝛼= 1⟹ 𝛼= 1- x= -1


1
(ii) x- 1 = -1⟹ x= 0
𝑥 3 − 4= 0⟹x= 43
X+ 𝛼= -1 ⟹ 𝛼= -1
∴ n= 1
Thus, 𝛼= -1.
Thus m= 2, n=1

26. If [x] denotes the integral part of x


𝟏+𝒕𝟐
and k = 𝒔𝒊𝒏−𝟏 > 0, then integral 27. If [x] denotes the integral part of x
𝟐𝒕
|𝒙|
value of 𝛼 for which the equation (x- and m= [𝟏+𝒙𝟐 ] , 𝒏 =
[k])(x+𝛼) - 1 = 0 has integral roots is 𝟏
𝒊𝒏𝒕𝒆𝒈𝒓𝒂𝒍 𝒗𝒂𝒍𝒖𝒆𝒔 𝒐𝒇 , then
𝟐−𝒔𝒊𝒏𝟑𝒙
(a) 1 (b) 2
(a) m≠ n (b) m > n
(c) 4 (d) none of these
(c) m + n = 0 (d) 𝒏𝒎 = 𝟎
Ans. (d)
Ans. (a)
1+𝑡 2 1+𝑡 2
For sin−1 𝑡𝑜 𝑏𝑒 𝑑𝑒𝑓𝑖𝑛𝑒𝑑, | |≤1 |𝑥|
2𝑡 2𝑡 0 ≤ 1+𝑥 2 < 1 ;
1+𝑡 2
⟹ <1 |𝑥|
2𝑡 ∴ m= [1+𝑥2 ] = 0
⟹ 1+ |𝑡|2 ≤ 2|𝑡|
Again 1≤ 2 − sin 3𝑥 ≤ 3
2
⟹ (1+ |𝑡|) ≤ 0 1 1
∴3 ≤ 2−sin 3𝑥 ≤ 1
⟹ (1+ |𝑡|)2 = 0 ⟹ |𝑡| = 1
1
∴ n= integral value of 2−sin 3𝑥= 1
⟹ t= ±1

76
Solving Mathematical Problems

∴m≠ n is the correct choice. Let an and bn respectively denote the


constant term and the coefficient of x in
fn(x). Then
28. If 1 lies between the roots of equation (a) an = 4, bn = −4n
𝒚𝟐 − 𝒎𝒚 + 𝟏 = 𝟎 and [x] denotes the
𝒎
𝟒|𝒙| (b) an = 4, bn = −4n2
integral part of x, then [(𝒙𝟐 +𝟏𝟔) ] =
(c) an = 4(n−1)! , bn = −4n
(a) 1 (b) 0 (c)
undefined (c) 2 (d) an = 4(n−1)! , bn = −4n2

Ans. (b) Ans. (a) an = (an-1 – 2)2 ; an-1 = 4 ; an = 4

Since 1 lies between the roots of equation fn(x) = (fn-1(x) − 2)2 = (fn-1(x))2 - 4 fn-1(x) + 4

𝑦 2 − 𝑚𝑦 + 1 = 0, So, bn = 2an-1bn-1 – 4bn-1 = 4bn-1 = 4nb0 = - 4n

∴ f(1)< 0

⟹ 2-m < 0 ⟹ m > 2 …… (1) 30. Let x be a positive real number. Then
4|𝑥|
Let y= 𝑥 2 +16 =
4|𝑥|
=
4𝑧
, where z= |x| (a) x2 + π2 + x2π > xπ + (π + x)xπ
|𝑥|2 +16 𝑧 2 +16
(b) xπ + πx > x2π + π2x
∴ y𝑧 2 − 4𝑧 + 16𝑦 = 0 (c) πx + (π + x)xπ > x2 + π2 + x2π
Since z is real, (d) none of the above

1 1 Ans. (a) By A.M > GM inequality, we have


∴ 16- 64𝑦 2 ≥ 0 ⟹ − 2 ≤ 𝑦 ≤ 2
(i) Π2 + x2π > 2πxπ
1
∴ 0≤ 𝑦 ≤ 2 [∵ y > 0] (ii) x2 + x2π > 2πxπ
(iii) x2 + π2 > 2πx
1
∴0≤ 𝑦 𝑚 ≤ 2𝑚 < 1
So we have x2 + π2 + x2π > xπ + (π + x)xπ
∴ [𝑦 𝑚 ] = 0 …….. (2)

29. Let {fn(x)} be a sequence of


polynomials defined inductively as

f1(x) = (x − 2)2

fn+1(x) = (fn(x) − 2)2 , n ≥ 1.

77
Solving Mathematical Problems

ISI OBJECTIVE SAMPLE PAPER Clearly, bc = 𝐷2 ⟹ 𝑟𝑜𝑜𝑡𝑠 𝑜𝑓 𝑒𝑞𝑛.


WITH SOLUTIONS b𝑥 2 + 2𝑑𝑥 + 𝑐 = 0 are real and equal.
SET – 10 2. If n𝒄𝟎 − 𝒏𝒄𝟏 + 𝒏𝒄𝟐 − 𝒏𝒄𝟑 + ⋯ +
(−𝟏)𝒓𝒏𝒄𝒓 = 𝟐𝟖, then n is equal to

(a) 6 (b) 7
There will be 30 questions in MMA Paper. (c) 8 (d) 9
For each question, exactly one of the four
choices is correct. You get four marks for Ans. (d) (𝑛0) − (𝑛1) + (𝑛2) − (𝑛3) + ⋯ +
each correct answer, one mark for each
unanswered question, and zero mark for (−1)𝑟 (𝑛𝑟)
each incorrect answer.
= (𝑛−1
0
) − ((𝑛−1
0
) + (𝑛−1
1
)) + ((𝑛−1
1
)+

1. If the tangents at points P and Q of the (𝑛−1


2
)) − ((𝑛−1
2
) + (𝑛−1
3
)) + ⋯ +
parabola 𝒚𝟐 = 𝟒𝒂𝒙 meet at R and b, c, d (−1)𝑟 (((𝑛−1 ) + (𝑛−1))
𝑟−1 𝑟
be the length of perpendiculars from P,
Q, R to any tangent to the parabola, then = (−1)𝑟 (𝑛−1)
𝑟
the roots of equations b𝒙𝟐 + 2dx+ c = 0 are
necessarily ∴(−1)𝑟 (𝑛−1
𝑟
) = 28 ⇒ r is even

(a) imaginary (b) real and equal 7×8


∴ (𝑛−1
𝑟
) = 28 = 7× 4 = = 8𝑐2
(c) real and unequal (d) rational 2

Ans. (b) ⇒ n= 9.
𝝅
Let P ≡ (a𝑡1 2 , 2𝑎𝑡1 ), Q ≡ ( a𝑡2 2 , 2𝑎𝑡2 ) 3. If x ∊ (𝟎, 𝟐 ) and cosx= ½ then the value
𝒄𝒐𝒔𝒏𝒙
of ∑∞ is equal to
Then R ≡ [a𝑡1 𝑡2 , 𝑎(𝑡1 + 𝑡2 )] 𝒏=𝟎 𝟑𝒏

Let any tangent to the parabola be yt = x + (a) 1 (b) -1


a𝑡 2 (c) 2 (d) -2
𝑐𝑜𝑠𝑥 𝑐𝑜𝑠2𝑥
or, x- yt+ a𝑡 2 = 0 ………….(4) Ans. (a) let c= 1+ 3
+ 32
+⋯

b= length of perp. from P to line (4) = 𝑠𝑖𝑛𝑥 𝑠𝑖𝑛2𝑥


And S = + +⋯
|a𝑡1 2 −2𝑎𝑡1 𝑡+a𝑡 2 | |𝑎| 3 32
√1+𝑡 2
= √1+𝑡 2 (𝑡1 − 𝑡)2
𝑒 𝑖𝑥 𝑒 𝑖𝑥
⇒c + iS= 1+ + +⋯
|𝑎| 2 3 32
c = √1+𝑡 2 (𝑡2 − 𝑡)
1 3
= 𝑒𝑖𝑥
=
|a𝑡1 2 −𝑎(𝑡1 +𝑡2 )+a𝑡 2 | |𝑎| 1− 3−𝑐𝑜𝑠𝑥−𝑖 𝑠𝑖𝑛𝑥
d= √1+𝑡 2
= √1+𝑡 2
|(𝑡1 − 3

𝑡)2 (𝑡2 − 𝑡)| Comparing real parts

78
Solving Mathematical Problems
3(3−𝑐𝑜𝑠𝑥) (a) 200 (b) 300
c = (3−𝑐𝑜𝑠𝑥)2 +𝑠𝑖𝑛2 𝑥
(c) 400 (d) none
1
⇒ c =1 (cosx = 3) Ans. (d) No. of digits used for numbering
pages 1 to 9 = 1 ×9 = 9.
4. The number of value of n for which
𝟑𝟗 + 𝟑𝟏𝟐 + 𝟑𝟏𝟓 + 𝟑𝒏 a perfect cube, is Similarly, 10 to 99 = 90×2 = 180, 100 to
999 = 900×3 = 2700
(a) 2 (b) 6
(c) 8 (d) none Number of digits will remain after using
2889 (=9+ 180+ 2700)digits = 3189- 2889=
Ans. (d) 39 + 312 + 315 + 3𝑛 300,
= 39 (1 + 34 + 36 + 3𝑛−9 ) The digits can be used for numbering 300
= (33 )3 {1 + 3 . 32 + 3. (32 )2 + (32 )3 + ÷4 = 75 pages, i.e. from 1000 to 1074.
3𝑛−9 − 3. (32 )2 } Hence the book has 1074 pages.
= (33 )3 (1 + 32 )3, 7. The unit’s digits of 𝟑𝟏𝟎𝟎𝟏 . 𝟕𝟏𝟎𝟎𝟐 . 𝟏𝟑𝟏𝟎𝟎𝟑
⇒3𝑛−9 − 35 = 0 is

⇒ n-9 =5 (a) 1 (b) 3


(c) 5 (d) none
⇒ n = 14.
Ans. (d) unit digit in 31001 is 3 ;
5. The number of integral solution of xy =
71002 is 9;
𝟐𝟐 . 𝟑𝟒 . 𝟓𝟕 (𝒙 + 𝒚)is
And 31001 is 7;
(a) 675 (b) 680
(c) 685 (d) none ∴Ans. is = 3× 9 × 7 = 9 (𝑢𝑛𝑖𝑡 𝑑𝑖𝑔𝑖𝑡)
Ans. (a) Let N = 22 . 34 . 57 ∴ 9 is in unit place.
Then, xy = N (x+ y) 8. The number of pairs of positive integer
⇒ xy- Nx- Ny= 0 (x, y) which satisfy the equation 𝒙𝟐 +
𝒚𝟐 = 𝒙𝟑 is
⇒(x- N)(y- N)=𝑁 2 = 24 . 38 . 514
(a) 0 (b) 1
∴ The number of integral solutions = (4+ (c) 2 (d) none
1)(8+ 1)(14+ 1) = 675.
Ans. (d) 𝑦 2 = 𝑥 2 (𝑥 − 1)
6. A printer numbers the pages of a book
starting with 1 and uses 3189 digit in all, So, if k is an integer satisfying x - 1 =𝑘 2
then the number of pages are ⇒ x= 𝑘 2 + 1

79
Solving Mathematical Problems

Thus there are infinitely many solutions. (a) {x:x ≥ 𝟎} (b) {x: x > 0}∪
{−𝟏} (c) {-1, 1}
9. If 16−𝒙𝟐 > |𝑥 − 𝑎| is to be satisfied by (d) none
at least one non- negative values of x, then
complete set of values of ‘a’ is 𝑥+1 |𝑥+1|2
Ans. (b) | |+|x+1|=
𝑥 |𝑥|
𝟔𝟓
(a) (-8, 8) (b) (-16, 𝟒 ) 1 |𝑥+1|
𝟔𝟓
⇒ |x+1| (|𝑥| + 1 − )= 0
|𝑥|
(c) (-8, ) (d) none
𝟒
⇒ |x+1|= 0 or, 1+ |𝑥| − |x + 1| = 0
Ans. (b) 16−𝑥 2 > |𝑥 − 𝑎|
⇒|x+1|= 0
⇒𝑥 2 − 16 < 𝑥 − 𝑎 < 16−𝑥 2
⇒ x+ 1 > 0 and x ≠ 0
⇒𝑥 2 − 16 − 𝑥 < −𝑎 < 16 − 𝑥 2 − 𝑥
i.e. x = -1 ,or >0 i.e. {x:x > 0}∪
⇒x + 16-𝑥 2 > 𝑎 > −16 + 𝑥 2 + 𝑥 {−1} .
65 1
⇒ 4 − (𝑥 − 2)2 > 𝑎 > −16 + 𝑥 2 + 𝑥; 𝑥 ≤ 12. The sum of the cubes of the root of
0 equation

∴ a ∊ (-16,
65
) (∵x ∊ℝ) 𝒙𝟒 + 𝒂𝒙𝟑 + 𝒃𝒙𝟐 + 𝒄𝒙 + 𝒅 = 𝟎 is
4
(a) 𝒂𝟑 − 𝟑𝒄 (b) 3ab -
10. Number of positive solutions for 𝒙𝟐 −
𝒂𝟑 (c) 3ab- c
𝟐 − 𝟐[𝒙] = 𝟎, where [ .]= the greatest
(d) none
integer, is
Ans. (d) let 𝛼1 , 𝛼2 , 𝛼3 , 𝛼4 be the root of the
(a) 0 (b) 1
equation
(c) 2 (d) none
Here 𝑎0 = 1, 𝑎1 = 𝑎, 𝑎2 = 𝑏, 𝑎3 = 𝑐, 𝑎4 =
Ans. (b) 𝑥 2 − 2 − 2[𝑥] ≥ 0 ⇒ [𝑥] = −1
𝑑
When [x]=-1, then 𝑥 2 − 2 = −2 ⇒ 𝑥 =
∴𝑎0 𝑠1 + 𝑎1 = 0 ⇒ 𝑠1 + 𝑎 = 0 ⇒ 𝑠1 = −𝑎,
0, which is not possible.
Now, 𝑎0 𝑠2 + 𝑎1 𝑠1 + 2𝑎2 = 0
When [x]=1 ⇒ 𝑥 2 = 4 ⇒ 𝑥 =
±2(impossible) ⇒ 𝑠2 + 𝑎 (−𝑎) + 2𝑏 = 0

When [x]= 2 ⇒ 𝑥 2 = 6 ⇒ 𝑥 = ±√6, i.e. ⇒ 𝑠2 = 𝛼 2 − 2𝑏,


only one possible value , i.e. √6 .
⇒ 𝛼1 2 + 𝛼2 2 + 𝛼3 2 + 𝛼4 2 = 𝑎 − 2𝑏,
𝒙+𝟏 (𝒙+𝟏)𝟐
11. The solution set of | |+|x+1|= is ∴𝑎0 𝑠3 + 𝑎1 𝑠2 + 𝑎1 𝑠1+3𝑎3 = 0
𝒙 |𝒙|

⇒ 𝑠3 = −𝑎3 + 3𝑎𝑏 − 3𝑐

80
Solving Mathematical Problems

⇒𝛼1 3 + 𝛼2 3 + 𝛼3 3 + 𝛼4 3 = 3𝑎𝑏 − 𝑎3 − 15. The number of positive integers which


3𝑐. are less than or equal to 1000 and are
divisible by none of 17, 19 and 23 equals
13. The integral roots of 𝟓𝒙𝟑 − 𝟏𝟏𝒙𝟐 +
𝟏𝟐𝒙 − 𝟐 = 𝟎 are (a) 854 (b) 153
(c) 160 (d) none.
(a) (1, 2, 3) (b) (-1, -2, -3)
(c) (0, 1, 2) (d) none Ans. (a) A: integers divisible by 17

Ans. (d) f(x)= 5𝑥 3 − 11𝑥 2 + 12𝑥 − 2 = 0 B: integers divisible by 19

Where constant term = -2, and divisors of C: integers divisible by 23


constant term are ±1, ±2, i.e. the possible
n(𝐴 ∪ 𝐵 ∪ 𝐶)𝑐 = 1000 − 𝑛(𝐴) − 𝑛(𝐵) −
value of integral roots are ±1, ±2
𝑛(𝐶) + 𝑛(𝐴 ∩ 𝐵) + 𝑛(𝐵 ∩ 𝐶) + 𝑛(𝐶 ∩
Now, f(1)≠ 0, 𝑓(−1) ≠ 0; 𝑓(2) ≠ 𝐴) − 𝑛(𝐴 ∩ 𝐵 ∩ 𝐶)
0; 𝑓(−2) ≠ 0.so it has no integral roots. 1000 1000 1000
= 1000− [ ]−[ ]−[ ]+
17 19 23
𝟐 𝟐
14. If f(x) = 𝒙 + 𝟐𝒃𝒙 + 𝟐𝒄 and g(x)= 1000 1000 1000 1000
[17×19]+ [17×23] + [19×23] − [17×19×23]
−𝒙𝟐 − 𝟐𝒄𝒙 + 𝒃𝟐 are such that min f(x),
then relation between b and c, is = 1000- 58- 52- 43+3+2+2-0
𝒃
(a) |c|> √𝟐 (b) 0 < c < 𝟐 = 854
(c) |c|< √𝟐|b| (d)
16. Let {𝒂𝒏 } be a sequence of real
none
numbers. Then 𝐥𝐢𝐦 𝒂𝒏 exists if and only if
𝒏→∞
Ans. (a) f(x) = (𝑥 + 𝑏)2 + 2𝑐 2 − 𝑏 2
(a) 𝐥𝐢𝐦 𝒂𝟐𝒏 and 𝐥𝐢𝐦 𝒂𝟐𝒏+𝟏 exist
𝒏→∞ 𝒏→∞
⇒ min f(x) = 2𝑐 2 − 𝑏 2 (b) 𝐥𝐢𝐦 𝒂𝟐𝒏 and 𝐥𝐢𝐦 𝒂𝟐𝒏+𝟐 exist
𝒏→∞ 𝒏→∞
g(x) = 𝑏 2 +𝑐 2 - (𝑥 + 𝑐)2 (c) 𝐥𝐢𝐦 𝒂𝟐𝒏 and 𝐥𝐢𝐦 𝒂𝟐𝒏+𝟏 and
𝒏→∞ 𝒏→∞

⇒ max g(x) = 𝑏 2 +𝑐 2 𝐥𝐢𝐦 𝒂𝟑𝒏 exist


𝒏→∞
(d) None of the above
Thus, min f(x) > max g(x)
Ans. (a) If a sequence converges then all of
⇒ 2𝑐 2 − 𝑏 2 > 𝑏 2 +𝑐 2 its subsequences converges.
⇒ |c|> √2|𝑏| ∴ lim 𝑎𝑛 converges ⤇ lim 𝑎2𝑛 and
𝑛→∞ 𝑛→∞
lim 𝑎2𝑛+1 exist
𝑛→∞

𝑎2𝑛 & 𝑎2𝑛+1 cover all the terms in 𝑎𝑛

So, converse is also true.

81
Solving Mathematical Problems

17. In the Taylor expansion of the Since | 𝑠𝑖𝑛2𝑥 |≤ 1 , 𝑠𝑜, 2 + 4𝑠𝑖𝑛2𝑥 ≤ 2+4 =
𝒙
function f(x) = 𝒆 about x=3, the
𝟐 6
coefficient of (𝒙 − 𝟑)𝟓 is 20. The pages of book are numbered
𝟑
𝟏 𝟑
𝟏
consecutively starting from pages. A total
(a) 𝒆𝟐 . 𝟓! (b) 𝒆𝟐 . 𝟐𝟓 𝟓! of 2989 digits was used to number the
𝟑
𝟏 pages. Then the number of pages is
(c) 𝒆−𝟐 . 𝟐𝟓 𝟓! (d) none
divisible by
𝑓 (𝑟) (𝑥0 ) 𝑓 5 (3)(𝑥−3)5
Ans. (b) (𝑥 − 𝑥0 )𝑟 = = (a) 2 (b) 3
𝑟! 5!
3 (c) 5 (d) 7
𝑒 2 (𝑥−3)5
25 5!
Ans. (a) 2989 = 189 + 2800=
18. Let f(x, y) = 189+2700+100 = 1× 9 + 20 × 9 + 300 ×
𝟐 +𝒚𝟐 )
𝒆−𝟏/(𝒙 𝒊𝒇 (𝒙, 𝒚) ≠ (𝟎, 𝟎) 9 + 25 × 4
{
𝟎 𝒊𝒇 (𝒙, 𝒚) = (𝟎, 𝟎)
From 1 to 9 pages, there are 9 digits
Then f(x, y) is
From 10 to 99 pages, there are 20×9 digits
(a) Not continuous at (0, 0)
From 100 to 999 pages, there are 300 × 9
(b) Differentiable at (0, 0)
digits.
(c) Continuous at (0, 0) but does not
From 1000 to 1024 pages, there are 25 × 4
have first order partial derivatives
digits.
(d) Continuous at (0, 0) and has first
So, there are total 1024 pages in the book.
order partial derivatives but not
differentiable at (0, 0) 21. Let A be a set of n elements. The
number of ways, we can choose an
Ans. (a) check yourself.
ordered pair (B, C), where B, C are
19. The maximum value of disjoint subsets of A, equals
𝟏 + 𝒔𝒊𝒏𝟐 𝒙 𝒄𝒐𝒔𝟐 𝒙 𝟒𝒔𝒊𝒏𝟐𝒙
[ 𝒔𝒊𝒏𝟐 𝒙 (a) n2 (b) n3
𝟏 + 𝒄𝒐𝒔𝟐 𝒙 𝟒𝒔𝒊𝒏𝟐𝒙 ] is
𝟐
𝒔𝒊𝒏 𝒙 𝟐
𝒄𝒐𝒔 𝒙 𝟏 + 𝟒𝒔𝒊𝒏𝟐𝒙 (c) 2n (d) 3n

(a) 0 (b) 2 (c ) 4 (d) 6 Ans. (d) 3n (Give reason)

1 −1 0 22. Consider the following system of


Ans. (d) ∆= [ 0 1 −1 ] equivalences of integers.
2 2
𝑠𝑖𝑛 𝑥 𝑐𝑜𝑠 𝑥 1 + 4𝑠𝑖𝑛2𝑥
[𝑅1′ = 𝑅1 − 𝑅2][ 𝑅2′ = 𝑅2 − 𝑅3 ] x ≡ 2 mod 15 &
x ≡ 4 mod 21.
= (1+ 4𝑠𝑖𝑛2𝑥 + 𝑐𝑜𝑠 2 𝑥) + ( 𝑠𝑖𝑛2 𝑥)

= 2+ 4𝑠𝑖𝑛2𝑥

82
Solving Mathematical Problems

The number of solutions in x, where 1 ≤ x Ans. (b) From 1st equation, f(0, 1) = 2
≤ 315, to the above system of equivalences
From 3rd equation, f(1, 1) = f(0, f(1, 0)) =
is
f(1, 0) + 1 (from 1st equation) = 3
(a) 0 (b) 1
(c) 2 (d) 3 26. Let S be the set of real numbers x for
which the power series ∑∞ 𝒏=𝟏[𝟏 −
𝒏 𝒏
Ans. (a) Use Chinese Remainder Theorem, (−𝟐) ]𝒙 converges. Then s equals
there will be no solution.
𝟏 𝟏
(a) {0} (b) (-𝟐 , 𝟐)
23. The set of complex numbers z 𝟏 𝟏
satisfying the equation (3 + 7i)z + (10 − (c) (-𝟐 , 𝟐) (d) (-1, 1)
2i)𝒛̅ + 100 = 0 represents, in the complex
Ans. (b) ∑∞ 𝑛
𝑛=1 𝑎𝑛 𝑥 converges if |x| <
plane, 1
1
(a) a straight line lt |𝑎𝑛 |𝑛
𝑛→∞

(b) a pair of intersecting straight lines 1


i.e. |x| < 1
(c) a point lt [1−(−2)𝑛 ]𝑛
𝑛→∞

(d) a pair of distinct parallel straight lines 1


i.e. |x| <- 2.
Ans. (c) Hint: Put z = x + iy, 𝑧̅ = x – iy
𝟐𝝅𝒊𝒌 𝟐𝝅𝒊(𝒌−𝟏)
27. There are 30 questions in a multiple
24. The 𝐥𝐢𝐦 ∑𝒏𝒌=𝟏 ∣𝒆 𝒏 −𝒆 𝒏 ∣ 𝒊𝒔 choice test. A student gets 1 mark for
𝒏→∞
each unattempted question, 0 mark for e
(a) 2 (b) 2e wrong answer and 4 marks for each
(c) 2π (d) 2i correct answer. A student has answered x
2𝜋𝑖𝑘 2𝜋𝑖
question correctly and has score then the
Ans. (c) lim ∑𝑛𝑘=1 ∣ 𝑒 𝑛 ∣∣ 𝑒 𝑛 −1∣= number of possible values of x is
𝑛→∞
2𝜋𝑖
lim 𝑛 ∣ 𝑒 𝑛 − 1 ∣ = 2π (a) 15 (b) 10
𝑛→∞
(c) 6 (d) 5
25. For non-negative integers m, n define
Ans. (c)
a function as follows

f(m, n) = 60 =10 × 4 + 20 × 1, x =10


𝒏+𝟏 𝒊𝒇 𝒎 = 𝟎 = 11 × 4 + 16 × 1 + 3 × 0 , x= 11
{𝒇(𝒎 − 𝟏, 𝟏) 𝒊𝒇 𝒎 ≠ 𝟎, 𝒏 = 𝟎
𝒇(𝒎 − 𝟏, 𝒇(𝒎, 𝒏 − 𝟏) 𝒊𝒇 𝒎 ≠ 𝟎, 𝒏 ≠ 𝟎 = 12× 4 + 12 × 1 + 6 × 0 , x= 12

Then the value of f(1, 1) is = 13× 4 + 8 × 1 + 9 × 0 , x= 13

(a) 4 (b) 3 = 14 × 4 + 4 × 1 + 12 × 0 , x= 14
(c) 2 (d) 1

83
Solving Mathematical Problems

= 15 × 4 + 0 × 1 + 15 × 0, x= 15 ISI OBJECTIVE SAMPLE PAPER


Total, 6 possible values are there, 10≤ 𝑥 ≤ WITH SOLUTIONS
15.
SET – 11
28. The number of permutations of {1, 2,
3, 4, 5} that keep at least one integer fixed
is There will be 30 questions in MMA Paper.
For each question, exactly one of the four
(a) 81 (b) 76
choices is correct. You get four marks for
(c) 120 (d) 60 each correct answer, one mark for each
unanswered question, and zero mark for
Ans. (c) There are total 5! Permutations of
each incorrect answer.
these 5 digits.

1. Let f(x) = cosx(sinx +


29. If f(x) is a real value function such √𝒔𝒊𝒏𝟐 𝒙 + 𝒔𝒊𝒏𝟐 𝜽), where ‘𝜃’ is a
that 2f(x) + 3f(-x) = 15- 4x, ∀ x ∊ℝ, then given constant,
f(2) is
then maximum value of f(x) is
(a) -15 (b) 22
(a) √𝟏 + 𝐜𝐨𝐬𝟐 𝜽 (b) √𝟏 + 𝐬𝐢𝐧𝟐 𝜽
(c) 11 (d) 0
(c) |cos𝜃| (d) none
Ans. (c) put x= -x , 2f (-x) + 3f(x) = 15+ 4x
Ans. (b) {𝑓(𝑥)𝑠𝑒𝑐𝑥 − 𝑠𝑖𝑛𝑥}2 = 𝑠𝑖𝑛2 𝑥 +
2f(x) + 3f(-x) = 15- 4x 𝑠𝑖𝑛2 𝜃,

_______________________________ ⤇ 𝑓 2 (𝑥)(1 + 𝑡𝑎𝑛2 𝑥) − 2𝑓(𝑥𝑡𝑎𝑛𝑥) =


𝑠𝑖𝑛2 𝜃
Solving equations, we get f(x) = 3+4x, f(2)
= 11. ⤇ 𝑓 2 (𝑥)𝑡𝑎𝑛2 𝑥 − 2𝑓(𝑥𝑡𝑎𝑛𝑥) + 𝑓 2 (𝑥) −
𝑠𝑖𝑛2 𝜃 = 0

⤇4𝑓 2 (𝑥) ≥ 4𝑓 2 (𝑥){ 𝑓 2 (𝑥) − 𝑠𝑖𝑛2 𝜃}


30. If M is a matrix of 3×3 order such
that ⤇𝑓 2 (𝑥) ≤ 1 + 𝑠𝑖𝑛2 𝜃
[0 1 2] M = [1 0 0] and [3 4 5] M = i.e. |f(x)|≤ √1 + 𝑠𝑖𝑛2 𝜃.
[0 1 0]. Then [6 7 8] M is equal to
𝟏+√𝟐+𝟑√𝟑+⋯+𝒏√𝒏
(a) [2 1 -2] (b) [0 1 0] 2. 𝐥𝐢𝐦
𝒏→∞ 𝒏
(c) [-1 2 0] (d) [9 10
(a) equals 0 (b) equals 1
8]
(c) equals ∞ (d) none
Ans. (b) Do yourself.

84
Solving Mathematical Problems

Ans. (b) Cauchy’s First limit theorem:- 1


= 1−𝑥 + log(1 − 𝑥)-1
𝑢1 +𝑢2 +⋯+𝑢𝑛
If lim 𝑢𝑛 = 𝑙, 𝑡ℎ𝑒𝑛 lim =𝑙.
𝑛→∞ 𝑛→∞ 𝑛 𝑥
= 1−𝑥 + log(1 − 𝑥)
1
Here lim 𝑢𝑛 = lim 𝑛𝑛 =
𝑛→∞ 𝑛→∞ 5. The polar equation r= acos𝜃 represents
1, 𝑠𝑜, 𝑏𝑦 Cauchy’s first limit theorem
(a) a spiral (b) a parabola
𝑢1 +𝑢2 +⋯+𝑢𝑛
lim = (c) a circle (d) none
𝑛→∞ 𝑛
1+√2+3√3+⋯+𝑛√𝑛
𝑛→∞
lim 𝑛
= 1. Ans. (c) 𝑟 2 = 𝑎𝑟𝑐𝑜𝑠𝜃

𝟑 𝟑.𝟓 ∴ 𝑥 2 + 𝑦 2 = 𝑎𝑥 (since r = 𝑥 2 + 𝑦 2 , 𝑥 =
3. The sum of the series 1+ + +
𝟒 𝟒.𝟖 𝑎𝑐𝑜𝑠𝜃 )
𝟑.𝟓.𝟕
+ ⋯ is
𝟒.𝟖.𝟏𝟐
∴𝑥 2 + 𝑎𝑥 + 𝑦 2 = 0
(a) 𝒆𝟐 (b) 3
𝑎 𝑎2
(c) √𝟓 (d) √𝟖 ∴(x+2)2 + 𝑦 2 = 4

3 3 3 𝑎 𝑎
1 1 This is a circle of radius 2 and centre (-2 , 0).
Ans. (d) √8 = 22 = (2)−2 = (1 − 2)−2

3
(− )(− −1)
3 6. The value of the infinite product P =
1 3 1
= 1+(2) (2) + 2 2
(− 2)2 + ⋯ 𝟕 𝟐𝟔 𝟔𝟑 𝒏𝟑 −𝟏
2! × 𝟐𝟖 × 𝟔𝟓 × … × 𝒏𝟑 +𝟏 × … is
𝟗
3 3.5 3.5.7
= 1+4 + 4.8 + 4.8.12 + ⋯ (a) 1 (b) 2/3
(c) 7/3 (d) none
4. If 0 < x < 1, then the sum of the infinite
𝟏 𝟐 𝟑
series 𝟐 𝒙𝟐 + 𝟑 𝒙𝟑 + 𝟒 𝒙𝟒 + ⋯ is Ans. (b)

23 −1 33 −1
(a) 𝐥𝐨𝐠 𝟏−𝒙
𝟏+𝒙 𝒙
(b) 𝟏−𝒙 + 𝐥𝐨𝐠(𝟏 + 𝒙) P= 23 +1 . 33 +1…….
𝟏 𝒙
(c) 𝟏−𝒙 + 𝐥𝐨𝐠(𝟏 − 𝒙) (d) 𝟏−𝒙 + (2−1)(22 +1+2) (3−1)(32 +1+3)
= ((2+1)(22 +1−2))( (3+1)(32 +1−3 )…..
𝐥𝐨𝐠(𝟏 − 𝒙)
1.2.3.4….. 7.13.21……
1 2 2 3 3 4 = (3.4.5.6……)( 3.7.13…….. )
Ans. (b) 2 𝑥 + 3 𝑥 + 4 𝑥 + ⋯
1.2 2
1 2 1 3 1 4 = = 3.
= (1-2) 𝑥 + (1-3) 𝑥 + (1-4)𝑥 +…. 3

(𝟏+𝒔𝒊𝒏𝝅𝒙)𝒕 −𝟏
2 3 4
= {𝑥 +𝑥 +𝑥 +…. }-{2 𝑥 + 3 𝑥 + 4 𝑥 +
1 2 1 3 1 4 7. If f(x) = 𝐋𝐭 , then range of
𝒊→∞ (𝟏+𝒔𝒊𝒏𝝅𝒙)𝒕 +𝟏

⋯} f(x) is

1 1 (a) {-1, 1} (b) {0, 1}


= {1+x+𝑥 2 +….}-{x+2 𝑥 2 + 3 𝑥 3 +…}-1
(c) {-1, 1} (d) {-1, 0, 1}

85
Solving Mathematical Problems

Ans. (d) Hence range of f= {n, n+ 1}

(1+sin 𝜋𝑥)𝑡 −1 𝒙 𝒙
f(x)= Lt = 9. If f(x) = 𝐋𝐭 + (𝒙+𝟏)(𝟐𝒙+𝟏) +
𝑡→∞ (1+sin 𝜋𝑥)𝑡 +1 𝒏→∞ 𝒙+𝟏
𝒙
1−
1
(1+sin 𝜋𝑥)𝑡 (𝟐𝒙+𝟏)(𝟑𝒙+𝟏)
+ ⋯ + to n terms, then range
1 𝑡, sin 𝜋𝑥 > 0
1+(
1+sin 𝜋𝑥
) of f(x) is
0−1 =
, sin 𝜋𝑥 < 0 (a) {0, 1} (b) {-1, 0} (c)
0+1
1−1
{ 1+1 , sin 𝜋𝑥 = 0 {-1, 1} (d) none of these
1, sin 𝜋𝑥 > 0 Ans. (a)
{−1, 𝑠𝑖𝑛𝜋𝑥 < 0
0, 𝑠𝑖𝑛𝜋𝑥 = 0 1
𝑆𝑛 = (1 − )
∴ Range f= {-1, 0, 1} 1+𝑥
1 1
+ ( − )
𝝅 1 + 𝑥 1 + 2𝑥
8. If f: (𝟎, 𝒏) → 𝑹, 𝒅𝒆𝒇𝒊𝒏𝒆𝒅 𝒃𝒚 𝒇(𝒙) = 1 1
∑𝒏𝒌=𝟏[𝟏 + 𝐬𝐢𝐧 𝒌𝒙], where [x] denotes the +( − )
1 + 2𝑥 1 + 3𝑥
integral part of x, then range of f(x) is +⋯
1
(a) {n-1, n+1} (b) {n-1, n, n+1} +(
1 + (𝑛 − 1)𝑥
(c) {n, n+1} (d) none of these 1
− )
Ans. (c) 1 + 𝑛𝑥
1
= 1−
f(x )= ∑𝑛𝑘=1(1 + [sin 𝑘𝑥]) = 𝑛 + [sin 𝑥] + 1 + 𝑛𝑥
[sin 2𝑥] + … + [sin 𝑛𝑥] …….(1)
𝑏𝑢𝑡 Lt 𝑛𝑥 = ∞, 𝑥 > 0 = −∞, 𝑥 < 0 =
𝑛→∞
𝜋
case 1: when kx ≠ for k= 1, 2, 3, …, n 0, 𝑥 = 0
2

since 0 < kx < 𝜋 and kx ≠


𝜋 ∴𝑓(𝑥) = Lt 𝑆𝑛 = 1, 𝑤ℎ𝑒𝑛 𝑥 ≠ 0
2 𝑛→∞
= 0, 𝑤ℎ𝑒𝑛 𝑥 = 0
∴ 0 < sin kx < 1, for k= 1, 2, …., n
Hence range f= {0, 1}
∴ [sin kx]= 0, for k= 1, 2, 3, …., n
𝝅𝒙 𝝅𝒙
10. Period of f(x) = sin (𝒏−𝟏)! + 𝐜𝐨𝐬 𝒏! is
∴ from (1), f(x)= n
𝜋 (a) n ! (b) 2 (n!)
When exactly one of x, 2x, 3x, …, nx is 2 .
(c) 2 (n-1)! (d) none of these
Here not more than one of x, 2x, 3x, …, nx
𝜋 𝜋𝑥
can be 2 . In this case one of sin x, sin 2x, …, Ans. (b) sin (𝑛−1)! is a periodic function with
𝜋
sinmnx is 1 and other lie between 0 and 1 period 2𝜋+ (𝑛−1)! = 2(𝑛 − 1)!

∴ from (1), f(x)= n+1 𝜋𝑥 𝜋


𝑃𝑒𝑟𝑖𝑜𝑑 𝑜𝑓 cos = 2𝜋 + = 2𝑛!
𝑛! 𝑛!
86
Solving Mathematical Problems

𝐿. 𝐶. 𝑀. 𝑜𝑓 2(𝑛 − 1)! 𝑎𝑛𝑑 2(𝑛!)𝑖𝑠 2(𝑛!) Ans. (b) Period of cos x= 2𝜋 and period of
2𝜋
cos ax= |𝑎|
∴ 𝑃𝑒𝑟𝑖𝑜𝑑 𝑜𝑓 𝑓(𝑥)𝑖𝑠 2(𝑛!)
2𝜋 2𝜋
11. Period of the function cos {(x+3)- Period of f(x) = L.C.M. of 𝑎𝑛𝑑 =
1 |𝑎|
[x+3]}, where [x] denotes the integral part 𝐿.𝐶.𝑀.𝑜𝑓 2𝜋 𝑎𝑛𝑑 2𝜋
of x is 𝐻.𝐶.𝐹.𝑜𝑓 1 𝑎𝑛𝑑 |𝑎|

(a) 1 (b) 2 (c) Since k= H.C.F. of 1 and |a|


𝜋 (d) 2𝜋 1 |𝑎|
∴ 𝑘= an integer= m (say) and = an integer
𝑘
Ans. (a)
= n (say)
x- [x] is a periodic function with period 1. 𝑛 𝑛
∴ |a|=𝑚 ⟹ 𝑎 = ± 𝑚 = a rational number.
∴ (x+ 3)- [x+ 3] is a periodic function with
period 1. 14. Let f : R⟶ R defined by f(x)= 𝒙𝟑 +
𝒙𝟐 + 𝟏𝟎𝟎𝒙 + 𝟓 𝐬𝐢𝐧 𝒙, then f is
∴ cos {(x+ 3)}- [x+ 3]] is a periodic function
with period 1. (a) many-one onto (b) many-one into
(c) one-one onto (d) one-one into
𝟑
12. If f(x) = 𝟐𝒔𝒊𝒏 𝝅𝒙+𝒙−[𝒙] , where [x]
Ans. (c)
denotes the integral part of x is a periodic
function with period 𝑓(𝑥) = 𝑥 3 + 𝑥 2 + 100𝑥 + 5 sin x
(a) 1 (b) 2 (c) ∴ f’(x)= 3𝑥 2 + 2𝑥 + 100 + 5 cos 𝑥
𝜋 (d) none of these
= 3𝑥 2 + 2𝑥 + 94 + (6 + 5 𝑐𝑜𝑠𝑥) > 0
Ans. (b)
∴ f is an increasing function and
Period of x- [x] is 1 and period of 𝑠𝑖𝑛3 𝜋𝑥 is consequently a one –one function.
2.
Clearly f(−∞)=−∞ , f(∞)=∞ and f(x) is
L.C.M. of 1 and 2 is 2 continuous, therefore range f= R= co
∴ f(x) is a periodic function with period 2. domain f. Hence f is onto.

𝒔𝒊𝒏𝟏𝟎𝟏 𝒙
13. If f(x)= cos x+ cos ax is a periodic 15. Let f(x) = 𝒙 𝟏 , where [x] denotes
[ ]+
𝝅 𝟐
function, then a is necessarily
the integral part of x is
(a) an integer (b) a rational number
(a) an odd function
(c) an irrational number (d) an event
(b) an even function
number
(c) neither odd nor even function
(d) both odd and even function

87
Solving Mathematical Problems

Ans. (a) when x= n𝜋, n 𝜖 I, sin x = 0 and Now sin x + cosec x = k ⟹ sin x + cosec x
𝑥 1 =2
[𝜋 ] + 2 ≠ 0
⟹(sin 𝑥 − 1)2 = 0 ⟹ sin 𝑥 = 1
∴ f(x) = 0
∴ cosec x = 1
∴ when x = n𝜋, f(x)= 0 and f(-x)= 0
Hence 𝑠𝑖𝑛𝑛 𝑥 + 𝑐𝑜𝑠𝑒𝑐 𝑛 𝑥 = 2
∴ f(-x)= f(x)
𝑥 17. If f(x+ y) = f(x) + f(y) – xy – 1for all x,
When x ≠ 𝑛𝜋, 𝑛 𝜖 𝐼, 𝜋 ≠ 𝑎𝑛 𝑖𝑛𝑡𝑒𝑔𝑒𝑟
y ∊R and f(1)=1, then the number of
𝑥 𝑥 𝑥 𝑥 solutions of f(n)= n, n ∊N is
∴ [ ] + [− ] = −1 ∴ [− ] = −1 − [ ]
𝜋 𝜋 𝜋 𝜋
(a) 0 (b) 1 (c)
𝑥 1 𝑥 1 𝑥 1 2 (d) more than 2
⟹ [− ] + = − [ ] − = − ([ ] + )
𝜋 2 𝜋 2 𝜋 2
Ans. (b)
𝑠𝑖𝑛101 (−𝑥) − sin 𝑥
Now 𝑓(−𝑥) = 𝑥 1 = 𝑥 1 =
[− ]+
𝜋 2
−([ ]+ )
𝜋 2
Given
sin 𝑥
𝑥 1 = 𝑓(𝑥) f(x+ y)= f(x)+f(y)- xy- 1∀𝑥, 𝑦, 𝜖 𝑅
[ ]+
𝜋 2
……………(1)
Hence in all cases f(-x)= f(x)
f(1)= 1 ……………..(2)

f(2) = f(1+1)= f(1)+f(-1)-1-1= 0


16. . If k be the value of x at which the
function f(3 )= f(2+1)= f(2)+f(1)-2.1-1= -2

𝒙 f(n+1) = f(n) +f(1) – n – 1 = f(n)- n< f(n)


f(x) =∫−𝟏 𝒕(𝒆𝒕 − 𝟏)(𝒕 − 𝟏)(𝒕 − 𝟐)𝟑 (𝒕 −
𝟑)𝟓 𝒅𝒕 has maximum value and sinx + Thus f(1) > f(2) > f(3)> …and f(1)= 1
cosecx = k, then for n ∊N, 𝒔𝒊𝒏𝒏 𝒙 +
∴f(1)= 1 and f(n)< 1, for n> 1
𝒄𝒐𝒔𝒆𝒄𝒏 𝒙 = …
Hence f(n)= n, n 𝜖 N has only one solution
(a) 2 (b) -2
𝝅 n= 1
(c) 𝟐 (d) 𝜋
18. If f is an increasing function and g is a
Ans. (a) f’(x) = 𝑥(𝑒 𝑥 − 1)(𝑥 − 1)(𝑥 − decreasing function such that g(f(x))
2)3 (𝑥 − 3)5 exists, then
By Sign Rule we get (a) g(f(x)) is an increasing function
(b) g(f(x)) is an decreasing
f(x) has max. at x = 2
(c) nothing can be said
∴k=2
(d) g(f(x)) is a constant function

88
Solving Mathematical Problems

Ans. (b) (𝑛 − 1) (𝑛 − 2)
=𝑛+ + +⋯
2 3
f’ (x)> 0 (∵ f(x) is an increasing function) [𝑛 − (𝑛 − 1)]
+
g’(x)< 0 𝑛
1 1 1
(∵g(x) is a decreasing function) = 𝑛 (1 + + +⋯+ )
2 3 𝑛
(g(f(x)))’= g’ (f(x)).f’(x) < 0 1 2 𝑛−1
− ( + + ⋯+ )
2 3 𝑛
∴g(f(x)) is a decreasing function
1 1
= 𝑛𝑓(𝑛) − [(1 − ) + (1 − ) + ⋯
19. f: R⟶ R, f(x)=x|x| is 2 3
1
+ (1 − )]
(a) one-one and onto 𝑛
(b) one-one but not onto
= 𝑛𝑓(𝑛) − (𝑛 − 1) + 𝑓(𝑛) − 1
(c) not one-one but onto = (𝑛 + 1)𝑓(𝑛) − 𝑛
(d) neither one-one nor onto
21. The period of f(x) = 𝒆𝐬𝐢𝐧{𝒙} +
Ans. (a) 𝝅
𝐬𝐢𝐧 (𝟐 [𝒙]) is ([.]) and {.} are the greatest
−𝑥 2 , 𝑥 ≤ 0 integer function and fraction function
𝑓(𝑥) = {
𝑥2, 𝑥 ≥ 0
(a) 1 (b) 4
2
−2𝑥 , 𝑥 ≤ 0 (c) 2 (d) not periodic
∴ 𝑓 ′ (𝑥) = {
2𝑥, 𝑥 ≥ 0
Ans. (b)
∴ f’(x)=>0 ∴ f(x) is an increasing function
and consequently it is a one –one function. Period of {x} i.e. x- [x] is 1 and period of
𝜋
sin ( 2 [𝑥]) 𝑖𝑠 4
Also f(−∞)= -∞, f(∞)=∞,

Hence range f= R L.C.M. of 1 and 4 is 4

𝟏 𝟏 ∴ Period of f(x) is 4
20. Let f(r) = 1+ 𝟐 +𝟑 + ⋯ +
𝟏
, 𝒕𝒉𝒆𝒏 ∑𝒏𝒊=𝟏 𝒇(𝒊)= 22. If f(x) = (𝒂 − 𝒙𝒏 )𝟏/𝒏 , x > 0 and g(x)> x
𝒓
⋁x ∊R, then for all x > 0
(a) (n+1) f(n)-(n- 1) (b) (n+ 1) f(n)- n
(a) g(g(x))= f(f(x)) (b) g(g(x))>2
(c) n f(n)- (n-1) (d) (n-1) f(n)
f(f(x)) (c) g(g(x))< f(f(x)) (d)
Ans. (b) g(g(x))> f(f(x))

1
f(1)+ f(2)+ …+f(n)= 1+ (1 + 2) + (1 + 2 +
1 Ans. (d)
1 1 1
) + ⋯ (1 + 2 + 3 + ⋯ + 𝑛)
1
f(x)= (𝑎 − 𝑥 𝑛 )1/𝑛 , 𝑥 > 0
3

89
Solving Mathematical Problems
1
∴ f(f(x))= f(y), where y= f(x)= (1 − 𝑦 𝑛 )𝑛 = Clearly 𝑒 2𝑥 ≥ 1 𝑓𝑜𝑟 𝑥 ≥ 0
1
{1 − (𝑎 − 𝑥 𝑛 )} 𝑛 = 𝑥, 𝑥 > 0 ∴y≥ 0 for x ≥ 0

Given, g(x)-> 0 ∀ x 𝜖 R ∴ Range f = [0, ∞) ≠ co domain f.

∴ g(g(x))-g(x)> 0 ∀ x 𝜖 R Hence f is not onto.

[Putting g(x) in place of x] Thus f is a many-one into mapping.

Adding we get, g(g(x))-x> 0 24. Given, y= sgn(x), then

⟹ g(g(x))> x ⟹g(g(x))> f(f(x)), x > 0 (a) |x|= x sgn(x)


(b) sgn(sgn(x)) = sgn(x)
[∵ f(f(x))= x, x > 0]
(c) x= |x|sgn(x)
23. Let f : R ⟶R be a function defined by (d) all of (a), (b), (c)
𝒆|𝒙| −𝒆−𝒙
f(x)= 𝒆𝒙 +𝒆−𝒙 , then
Ans. (d)
(a) ‘f’ is one-one and onto 1, 𝑥 > 0
(b) ‘f’ is one-one but not onto 𝑓(𝑥) = 𝑠𝑔𝑛(𝑥) = { 0, 𝑥 = 0
−1, 𝑥 < 0
(c) ‘f’ is not one-one but onto
(d) ‘f’ is neither one-one nor onto 𝑥, 𝑥 > 0
𝑥 𝑠𝑔𝑛 (𝑥) = { 0, 𝑥 = 0 = |𝑥|
Ans. (d) −𝑥, 𝑥 < 0

𝑒 𝑥 − 𝑒 −𝑥 𝑠𝑔𝑛(1), 𝑥 > 0
𝑓(𝑥) = , 𝑥 ≥ 0 = 0, 𝑥 ≤ 0
𝑒 𝑥 + 𝑒 −𝑥 𝑠𝑔𝑛 (𝑠𝑔𝑛(𝑥)) = { 𝑠𝑔𝑛(0), 𝑥 = 0
𝑠𝑔𝑛(−1), 𝑥 < 0
Since f(x)= 0, for all x ≤ 0 1, 𝑥 > 0
= { 0, 𝑥 = 0 = 𝑠𝑔𝑛(𝑥)
∴ f(x) is a many –one function
−1, 𝑥 < 0
𝑒 𝑥 −𝑒 −𝑥
Let y= 𝑒 𝑥 +𝑒 −𝑥 , 𝑥 ≥ 0 𝑥, 𝑥 > 0
|𝑥|𝑠𝑔𝑛(𝑥) = {0, 𝑥 = 0 = 𝑥, ∀ 𝑥 𝜖 𝑅
𝑒 𝑥 −𝑒 −𝑥 1 𝑥, 𝑥 < 0
⟹ =
𝑒 𝑥 +𝑒 −𝑥 𝑦
25. If 𝒍𝟏 𝒂𝒏𝒅 𝒍𝟐 are the side length of two
𝑒𝑥 1+𝑦
⟹𝑒 −𝑥 = 1−𝑦 variables squares 𝒔𝟏 𝒂𝒏𝒅 𝒔𝟐 , respectively.
If 𝒍𝟏 = 𝒍𝟐 + 𝒍𝟐 𝟑 + 𝟔, then the rate of
1+𝑦 1 1+𝑦
⟹𝑒 2𝑥 = 1−𝑦 ⟹ 𝑥 = 2 log 1−𝑦 change of the area of 𝒔𝟐 with respect to
rate of change of the area of 𝒔𝟏 when
𝑒 𝑥 − 𝑒 −𝑥 𝑒 2𝑥 − 1 𝒍𝟐 = 𝟏 is
𝑦= = ,𝑥 ≥ 0
𝑒 𝑥 + 𝑒 −𝑥 𝑒 2𝑥 + 1

90
Solving Mathematical Problems

(a) 3/2 (b) 2/3 ⤇ 1 − 𝑎2 = 0 ⤇ a= 1, 1+2ab= 0 1+2b= 0


(c) 4/3 (d) none ⤇b=-½.

Ans. (d) Let ∆1 𝑎𝑛𝑑 ∆2 be the area of the


sequences 𝑠1 𝑎𝑛𝑑 𝑠2 ,

∆1=𝑙1 2 𝑎𝑛𝑑 ∆2 = 𝑙2 2
𝒙𝟐
𝒆𝒙 −𝒙−
𝟐
𝑑∆1 𝑑∆2 28. 𝐥𝐢𝐦 𝒔𝒊𝒏 is equal to
∴ = 2𝑙1 𝑎𝑛𝑑 = 2𝑙2 . 𝒙→𝟎 𝒙𝟐
𝑑𝑙1 𝑑𝑙2

𝑑∆ 𝑙 𝑑𝑙 𝑙 1
(a) 0 (b) ½
⤇ 𝑑∆2 = 𝑙2 . 𝑑𝑙2 = 𝑙2 . 1+3𝑙 (c) 1 (d) does
1 1 1 1 2
not exist
𝑑∆2 1
When𝑙2 = 1, 𝑙1 = 8 , 𝑡ℎ𝑒𝑛 = 32.
𝑑∆1 2
Ans. (a) For x⟶ 0, 𝑒 𝑥 ≏ 1 + 𝑥 + 𝑥 ⁄2!
2 2
(1+𝑥+𝑥 ⁄2)−(1+𝑥+𝑥 ⁄2)
𝟒𝒙 So, lim 𝑠𝑖𝑛 =
26. The limit 𝐥𝐢𝐦 (𝟓+𝟒𝒙)𝟐𝒙 is 𝑥→0 𝑥2
𝒙→∞
lim sin(0) = 0.
𝑥→0
(a) 𝒆−𝟓/𝟐 (b) 𝒆−𝟐/𝟓
𝟏+𝒙 𝟏
(c) 𝒆−𝟓 (d) none 29. The limit 𝐥𝐢𝐦 𝐥𝐨𝐠(𝟏−𝒙)𝒙 equals
𝒙→𝟎+
1 1
Ans. (a) lim ( 5 ) 2𝑥 = 5 𝑥 2 = (a) 0 (b) 1 (c)
𝑥→∞ 1+4𝑥 { lim (1+
4𝑥
) }
𝑥→∞
5 2 (d) does not
𝑒 −4×2 = 𝑒 −5/2 . exist

1+𝑥 1
Ans. (c) L= log(1−𝑥)𝑥
27. Let a and b be real numbers such that
1 1+𝑥
𝐥𝐢𝐦 √𝒙𝟐 − 𝒙 − 𝟏 − 𝒂𝒙 − 𝒃 = 𝟎. 𝑙𝑜𝑔𝐿 = 𝑥 log(1−𝑥)
𝒙 →∞

1 𝑥3
Then the value of b is lt log 𝐿= lt {2(𝑥 + + ⋯ )} = 2.
𝑥→0+ 𝑥→0+ 𝑥 3

(a) -1 (b) - ½
30. If 0 < c < d, then the sequence 𝒂𝒏 =
(c) 0 (d) ½ 𝟏
(𝒄𝒏 + 𝒅𝒏 )𝒏 is
Ans. (b) lim (√𝑥 2 − 𝑥 − 1 −𝑎𝑥 − 𝑏) =
𝑥 →∞ (a) Bounded & monotone decreasing
(𝑥 2 −𝑥−1)−(𝑎𝑥+𝑏)
lim (b) bounded & monotone increasing
𝑥 →∞ √𝑥 2 −𝑥−1+𝑎𝑥 2 +𝑏

= (c) monotone increasing & unbounded for


2
(1−𝑎2 )𝑥 −(1+2𝑎𝑏)𝑥−(1+𝑏 2 )
1<c<d
lim √𝑥 2 −𝑥−1+𝑎𝑥 2 +𝑏
=0
𝑥 →∞

91
Solving Mathematical Problems
2
(d) monotone decreasing & unbounded ∴ 𝑃2 - 2q ≥ 0 [∵∑100
𝑖=1 𝑎𝑖 ≥ 0]
for 1 < c < d.
𝑃2
𝑛 𝑛 ∴q≤ .
Ans. (b) 0 < c < d ⤇ 0 < 𝑐 < 𝑑 ∀ 𝑛 ∊ ℕ 2

∴𝑐 𝑛 + 𝑑𝑛 < 2𝑑 𝑛 2. A club with x members is organized


into four committees such that (a) each
1
or, 𝑎𝑛 < 2𝑛 . 𝑑∀ 𝑛 ∊ ℕ member is in exactly two committees, (b)
only two committees have exactly one
1
or, d < 𝑎𝑛 < 𝑑. 2𝑛 member in common, then x has

By squeeze theorem, lim(𝑎𝑛 )= d. So, the (a) Exactly two values both between 4
sequence is bounded and monotone and 8
increasing. (b) Exactly one value between 4 and 8
(c) Exactly two values both between 8
and 16
(d) Exactly one value between 8 and
ISI OBJECTIVE SAMPLE PAPER 16
WITH SOLUTIONS
Ans. (b) Four committees are there, let us
SET – 12 denote member

A , B, C, D, E, F, …..

There will be 30 questions in MMA Paper. 1st combination :- ADE


For each question, exactly one of the four [(a) each member is exactly in two
choices is correct. You get four marks for committees
each correct answer, one mark for each
unanswered question, and zero mark for 2nd combination:- ABF (b)
each incorrect answer. only two committees exactly one member
common]
1. For positive real numbers 3rd combination:- BCE
𝒂𝟏 , 𝒂𝟐 , … … . 𝒂𝟏𝟎𝟎 , let P= ∑𝟏𝟎𝟎
𝒊=𝟏 𝒂𝒊 𝒂𝒏𝒅 𝒒 =
∑𝟏≤𝒊≤𝒋≤𝟏𝟎𝟎 𝒂𝒊 𝒂𝒋 , then 4th combination:- CDF

𝑷𝟐 𝑷𝟐
𝟏 𝟏 𝟏
(a) q = 𝟐 (b) 𝒒𝟐 ≤ 3. Let A = (𝟏 𝟐 𝟐) and B =
𝟐
𝑷𝟐 𝟏 𝟐 𝟑
(c) q < (d) none 𝟏 𝟎 𝟎
𝟐
(𝟏 𝟏 𝟎). Then
Ans. (c) 𝑎1 + 𝑎2 + ⋯ … . +𝑎100 = P, 𝟏 𝟏 𝟏
2 (a) there exists a matrix C such that A=
𝑃2 = (𝑎1 + 𝑎2 + ⋯ … . +𝑎100 )2 = ∑100
𝑖=1 𝑎𝑖 +
100
2 ∑𝑖<𝑗 𝑎𝑖 𝑎𝑗 BC= CD.

92
Solving Mathematical Problems

(b) there is no matrix C such that A= BC. ⤇ (𝜆-1)(𝜆-1)(𝜆-4)= 0

(c) there exists a matrix C such that A= ⤇ 𝜆= 1, 1, 4 are the eigen value of X.
BC, but A ≠ CB
6. If f(x) = 𝒙𝟐 and g(x) = 𝒙𝒔𝒊𝒏𝒙 + 𝒄𝒐𝒔𝒙
(d) there is no matrix C such that A= CB. then
1 0 0 (a) f and g agree at no points
Ans. (c) B=(1 1 0) is an lower (b) f and g agree at exactly one points
1 1 1
triangular mtx. (c) f and g agree at exactly two point
1 1 1 (d) f and g agree at more than two points
Take, C = (0 1 1) is an upper triangular Ans. (c) So, we can two graphs meet at
0 0 1
mtx. exactly two points.

1 1 1 1 1 1 7. A subset S of the set of numbers {2, 3,


∴BC= (1 2 2) , 𝐶𝐵 ≠ ( 1 2 2) 4, 5, 6, 7, 8, 9, 10} is said to be good if it
1 2 3 1 2 3 has exactly 4 elements and their gcd = 1.
Then the number of good subset is
𝒂 𝟏
4. If the matrix A = [ ] has 1 as an
𝟐 𝟑 (a) 0126 (b) 125
eigenval then trace (A) is
(c) 123 (d) 121
(a) 4 (b) 5
Ans. (d) Total number of subsets containing
(c) 6 (d) 7
exactly 4 elements from 9 elements are
Ans. (b) |A- 𝜆I| = 0 ⇒a = 2 9!
= 9𝑐4 = 4!5! = 126.
∴ trace (A) = 2+ 3 = 5= sum of diagonal
elements. Now, gcd = 1, so we need not to take into
count these subset: {2, 4, 6, 8}, {2, 4, 6, 10},
5. The eigen values of the matrix X= {2, 4, 8, 10},{2, 6, 8, 10}, {4, 6, 8, 10}.
𝟐 𝟏 𝟏
(𝟏 𝟐 𝟏) are So, there are total (126 - 5) = 121 good
𝟏 𝟏 𝟐 subsets.
(a) 1, 1, 4 (b) 1, 4, 4
(c) 0, 1, 4 (d) 0, 4, 4
8. In how many ways can there persons,
Ans. (a) |X-𝜆I|= 0 each throwing a single die once,
2−𝜆 1 1 make a score of 11?
⤇[ 1 2−𝜆 1 ]= 0
1 1 2−𝜆 (a) 22 (b) 27 (c) 24 (d) 38
3 2
⤇ 𝜆 − 6𝜆 + 9𝜆 − 4 = 0 Ans. (b) we can use a tree diagram here:-

93
Solving Mathematical Problems

1<→5→6, ,4
5
∶3!𝑤𝑎𝑦𝑠
∶3 𝑤𝑎𝑦𝑠
𝑐𝑜𝑠|𝑥| = 𝑐𝑜𝑠𝑥 is differentiable at x= 0.

→4 , 5 ∶3!𝑤𝑎𝑦𝑠 So, 𝑎1 = 0 is the only criteria for f(x) to be


2 < →3 , 6 ∶3! 𝑤𝑎𝑦𝑠
differentiable.
→4 , 5 ∶3 𝑤𝑎𝑦𝑠
3< →3 , 6 ∶3 𝑤𝑎𝑦𝑠 11. f(x)is a differentiable function on the
real line such that 𝐥𝐢𝐦 𝒇(𝒙) =
So, there are total (3!+3 +3!+3!+3+3)= 27 𝒙⟶∞

ways. 𝟏 𝒂𝒏𝒅 𝐥𝐢𝐦 𝒇′(𝒙) = 𝜶 , then


𝒙⟶∞

Explanation: - For {1, 6, 4} there are total 3! (a) 𝛼 must be 0 (b) 𝛼 need not to
Ways, since{1, 6, 4}, {4, 6, 1}, {1, 4, 6}, {4, be 0, but |𝛼|<1 (c) 𝛼 > 1
1, 6}, {6, 4, 1} are six possibilities, but in (d) 𝛼 < -1
case of {1, 5, 5} there are total 3 ways, since
𝑘
there are {1, 5, 5}, {5, 1, 5}, {5, 5, 1} only 3 Ans. (a) Let f(x)=1+𝑥 𝑃 , 𝑝 > 0
such permutations.
So, lt 𝑓(𝑥) = 1,
𝑥⟶∞

𝑘(−𝑃)
9. 𝒙𝟐 + 𝒙 + 𝟏 is a factor of (𝒙 + 𝟏)𝒏 − ∴ lim 𝑓 ′(𝑥) = lt = 0 = 𝛼.
𝑥⟶∞ 𝑥⟶∞ 𝑥 𝑃+1
𝒙𝒏 − 𝟏, wherever 𝝅
12. ∫𝟎 𝐦𝐢𝐧(𝒔𝒊𝒏𝒙, 𝒄𝒐𝒔𝒙)𝒅𝒙 equals
(a) n is odd (b)
n is odd and a multiple of 3 (a) 1-2√𝟐 (b) 1
(c) 0 (d) 1-√𝟐
(c) n is even multiple of 3 (d)
n is odd and not a multiple 3 𝜋
𝑠𝑖𝑛𝑥, 0 ≤ 𝑥 ≤ 4
Ans. (d) f(x)= { 𝜋
Ans. (c) Let n= 1, (x+1)-x-1= 0 𝑐𝑜𝑠𝑥, 4 ≤ 𝑥 ≤ 𝜋

n=2, (𝑥 + 1)2 − 𝑥 2 − 1 = +2𝑥 𝜋


𝜋
∴ ∫0 min(𝑠𝑖𝑛𝑥, 𝑐𝑜𝑠𝑥)𝑑𝑥 = ∫04 𝑠𝑖𝑛𝑥𝑑𝑥 +
n= 3, (𝑥 + 1)3 − 𝑥 3 − 1 = 3𝑥 2 + 3𝑥 𝜋 1
∫𝜋 𝑐𝑜𝑠𝑑𝑥 = (1 − √2) + (− √2) = 1 − √2
1
4

𝟐
13. The value of the integral ∫−𝟐 𝐦𝐢𝐧{|𝒙 −
10. The map f(x) = 𝒂𝟎 𝒄𝒐𝒔|𝒙| + 𝒂𝟏 𝒔𝒊𝒏|𝒙| +
𝟏|, |𝒙 + 𝟐|}𝒅𝒙 , is
𝒂𝟐 |𝒙|𝟑 is different at x=o if and only if
𝟏𝟏 𝟗
(a) (b) 𝟒
(a) 𝒂𝟏 = 𝟎 𝒂𝒏𝒅 𝒂𝟐 = 𝟎 𝟒
𝟏𝟏 𝟗
(b) 𝒂𝟎 = 𝟎 𝒂𝒏𝒅 𝒂𝟏 = 𝟎 (c) 𝟐
(d) 𝟐

(c) 𝒂𝟏 = 𝟎 2
Ans. ∫−2 min{|𝑥 − 1|, |𝑥 + 2|}𝑑𝑥
(d) 𝒂𝟎 , 𝒂𝟏 , 𝒂𝟐 can take any real value
1
− 2
Ans. (c) |𝑥|3 is differentiable at x=0. = ∫−22|𝑥 + 2|𝑑𝑥 + ∫−1|𝑥 − 1|𝑑𝑥
2

94
Solving Mathematical Problems

14. Let f(x) be a given differentiable ∴ r = 0, 2, 4,…, 32 means r can take 17


function. Consider the following different values.
𝒅𝒚
differential equation in y f(x)𝒅𝒙 =
𝒚𝒇′ (𝒙) − 𝒚𝟐 .
16. Let P is an odd prime and n= 1+p!,
The general solution of the equation is then total number of prime in the list n+1,
given by n+2, n+3, ……, n+p-1 is equal to
𝒙+𝒄 𝒇(𝒙)
(a) y= − 𝒇(𝒙) (b) 𝒚𝟐 = (a) P- 3 (b) P- 5
𝒙+𝒄
𝒇(𝒙) [𝒇(𝒙)]𝟐
(c) 0 (d) none
(c) y = (d) y =
𝒙+𝒄 𝒙+𝒄
Ans. (c) ∵ n= 1+p!
𝑑𝑦 𝑦𝑓 ′ (𝑥)−𝑦 2
Ans. (c) 𝑑𝑥 = ∴ n+ r= (r+1)+p!
𝑓(𝑥)

1 𝑑𝑦 1 𝑓′(𝑥) 1 1 If 1 ≤ 𝑟 ≤ 𝑝 − 1, 𝑡ℎ𝑒𝑛 2 ≤ 𝑟 + 1 ≤ 𝑝 and


⇒ -𝑦 2 𝑑𝑥 + 𝑦 𝑓(𝑥) = 𝑓(𝑥) [put 𝑦 = v, -
1 𝑑𝑦 𝑑𝑣
clearly,
= 𝑑𝑥 ]
𝑦 2 𝑑𝑥
(n+ r) is divisible by r+1. ∴ n+ r can’t be a
𝑑𝑣 𝑓′(𝑥) 1 prime
⇒𝑑𝑥 + 𝑣. 𝑓(𝑥) = 𝑓(𝑥) which is a linear
equation in v Hence, there is no prime in the given list.
𝑓′(𝑥)
∴ I.F. = 𝑒 ∫ 𝑑𝑥 = 𝑒 log 𝑓(𝑥) = 𝑓(𝑥).
𝑓(𝑥)

1
17. The remainder obtained when 1! +2!
Hence solution is: v. f(x)= ∫ 𝑓(𝑥) . 𝑓(𝑥)𝑑𝑥 + +3!+….+ 95! is divided by 15, is
𝑓(𝑥)
𝑐 = 𝑥 + 𝑐 𝑖. 𝑒. 𝑦 = . (a) 3 (b) 5
𝑥+𝑐
(c) 7 (d) none
15. Number of integral terms in the
expansion of (√𝟔 + √𝟕)𝟑𝟐 = Ans. (a) here 1! +2! +3! +4! = 33 and n! is
divisible by 15 where n ≥ 5.
(a) 15 (b) 17
(c) 19 (d) none The remainder is same as the remainder
obtained by dividing 33 with 15, i.e., 3.
Ans. (b) (√6 + √7)32 =
𝑟 32−𝑟
∑32
𝑟=0 32𝑐𝑟 . 62 . 7 2

𝒆𝒕𝒂𝒏𝒙 −𝒆𝒙
𝑟 32−𝑟 18. The value of 𝐋𝐭 is
For integral terms 2 and both are 𝒙→𝟎 𝒕𝒂𝒏𝒙−𝒙
2
𝑟
integers and w is in turn possible if 2 is an (a) 0 (b) 1
integer. (c) e (d) none

95
Solving Mathematical Problems

𝑒 tan 𝑥 −𝑒 𝑥 𝑒 tan 𝑥 −1 𝑥2
Ans. (b) Lt = Lt 𝑒 𝑥 ( tan 𝑥− 𝑥 ) = We have, f(𝑥 2 )= ∫0 𝑓(𝑡) 𝑑𝑡 = 𝑥 2 + 𝑥 3
𝑥→0 tan 𝑥− 𝑥 𝑥→0
𝑒 0 . log 𝑒 𝑒 = 1
Differentiating both sides, we get;

f(𝑥 2 ).2𝑥 = 2𝑥 + 3𝑥 2
|𝒙|
19. Total number of solutions of sinx = 𝟏𝟎 ⟹ f(𝑥 2 )= 1+(3/2)x
is
⟹ f(4)= 1+3/2(2)= 4
(a) 0 ; (b) 3 ; (c) 4 ; (d) none
22. The equation of a curve is y= f(x). The
Ans. (d) Two graphs meet exactly 6 times, tangents at (𝛼, f(𝛼)), (𝛽,f(𝛽)) and (𝛾, f(𝛾))
hence, it has 6 solutions. Draw graph 𝝅 𝝅 𝝅
make angles 𝟔 , 𝟑 , 𝟒 respectively with the
yourself.
positive direction of the x- axis. Then the
20. Let f(x)= value of
𝒙𝟑 𝐬𝐢𝐧 𝒙 𝐜𝐨𝐬 𝒙 𝜸 𝜸
∫𝜷 𝒇′ (𝒙). 𝒇′′ (𝒙)𝒅𝒙 + ∫𝜶 𝒇′′ (𝒙)𝒅𝒙 is equal
|𝟔 −𝟏 𝟎 | 𝒘𝒉𝒆𝒓𝒆 𝒑 𝒊𝒔 𝒂 𝒄𝒐𝒏𝒔𝒕𝒂𝒏𝒕 , 𝒕𝒉𝒆𝒏 𝒂𝒕 𝒙 =
𝒑 𝒑𝟐 𝒑𝟑 to
𝒅𝟑 (𝒇(𝒙)) 𝟏 𝟏
𝟎, = (a) − (b)
𝒅𝒙𝟑
√𝟑 √𝟑

(a) p (b) 0 (c) 0 (d) none of


(c) 𝒑𝟐 (d) –p these

𝑑3 𝑑3 Ans. (a)
Ans. (b) 𝑑𝑥 3 (𝑥 3 ) = 3! = 6, 𝑑𝑥 3 (sin 𝑥) =
1
𝑑3 Given, 𝑓 ′ (𝛼) = , 𝑓 ′ (𝛽) = √3, 𝑓 ′ (𝛾) = 1
− cos 𝑥, 𝑑𝑥 3 (𝑐𝑜𝑠𝑥) = 𝑠𝑖𝑛𝑥 √3

𝛾 𝛾
6 − cos 𝑥 sin 𝑥 Now ∫0 𝑓 ′ (𝑥)𝑓 ′′ (𝑥) 𝑑𝑥 + ∫𝛼 𝑓 ′′ (𝑥) 𝑑𝑥 =
𝑑3 𝑓(𝑥)
∴ = |6 −1 0 | 𝑎𝑡 𝑥 = 0, 1 2 1
[2 (𝑓 ′ (𝑥)) ] 𝛽𝛾 + [𝑓 ′ (𝑥)] 𝛼𝛾 = 2 (𝑓 ′ (𝑦)) −
2
𝑑𝑥 3
𝑝 𝑝2 𝑝3
1 1
{𝑓 ′ (𝛽)}2 + 𝑓 ′ (𝛾) − 𝑓 ′ (𝛼) = (1 − 3) +
6 −1 0 2 2
𝑑 3 𝑓(𝑥) 1 1
= |6 −1 0 | = 0 1− = −
𝑑𝑥 3 √3 √3
𝑝 𝑝2 𝑝3
23. A rod of length 10ft sides with ends on
21. Let f : (0, +∞) → 𝑹 𝒂𝒏𝒅 𝑭(𝒙) = the co-ordinates axes. If the end on x-axis
𝒙
∫𝟎 𝒇(𝒕)𝒅𝒕 𝒊𝒇 𝑭(𝒙𝟐 ) = 𝒙𝟐 (𝟏 + 𝒙), then f(4) moves with constant velocity of 2ft/
equals minute, then the magnitude of the velocity
of the middle point at the instant the rod
(a) 5/4 (b) 7
makes an angle of 30° with x-axis is
(c) 4 (d) 2
(a) 2ft / sec (b) 3 ft / sec
Ans. (c)
(c) √𝟑 ft / sec (d) none of these

96
Solving Mathematical Problems

Ans. (a) ∴𝛼 3 + 𝛽 3 = 𝑎2 𝑎𝑛𝑑 𝛾 3 + 𝛿 3 = 𝑎2

Let AB be the position of rod at any time t 𝛼 3 − 𝛾 3 = 𝛿 3 − 𝛽 3 ……………(2)


and p be its middle point.
𝑑𝑦 𝑥2
From(1), 𝑑𝑥 = − 𝑦 2
𝑥 𝑦
Let OA= x, OB= y, then P ≡ (2 , 2)
Equation of tangent at P(𝛼, 𝛽) 𝑖𝑠 𝑦 − 𝛽 =
𝑥 2 + 𝑦 2 = 102 𝛼2
(𝑥 − 𝛼)…………(3)
𝛽2

𝑑𝑥 𝑑𝑦 According to question, (3) passes through


∴2𝑥 𝑑𝑡 + 2𝑦 𝑑𝑡 = 0
Q ∴ 𝛼 2 (𝛼 − 𝛾 ) = 𝛽 2 (𝛿 − 𝛽) ……(4)
𝑑𝑦 𝑥 𝑑𝑥 𝑥
= − = − .2
𝑑𝑡 𝑦 𝑑𝑡 𝑦 𝛼 2 + 𝛾 2 + 𝛼𝛾 𝛽 2 + 𝛿 2 + 𝛽𝛽
=
𝑥 𝛼2 𝛽2
𝑤ℎ𝑒𝑛 𝜃 = 30°, = cot 30° = √3 𝛾2 𝛾
𝑦 ⟹1+ 2+
𝛼 𝛼
𝑑𝑦 𝛿2 𝛿
∴ = −2√3𝑓𝑡/𝑠𝑒𝑐 = 1+ 2+
𝑑𝑡 𝛽 𝛽

1 𝑑𝑥 2 1 𝑑𝑦 2 𝛾 2 𝛿 2 𝛾 𝛿 𝛾 𝛿

𝑛𝑜𝑤 𝑉 = ( ) +( ) ⟹ ( ) −( ) =( − )⟹ − +1
2 𝑑𝑡 2 𝑑𝑡 𝛼 𝛽 𝛼 𝛽 𝛼 𝛽
𝛾 𝛿
1 = 0[∵ ≠ ]
= √22 + 12 = 2 𝑓𝑡/𝑠𝑒𝑐 𝛼 𝛽
2
24. Two persons are moving on the curve
𝒙𝟑 + 𝒚𝟑 = 𝒂𝟑 . When the position of first 25. The triangle formed by the tangents to
and second persons are (𝛼, 𝛽) and (𝛾, 𝛿) the curve f(x) = 𝒙𝟐 + 𝒃𝒙 − 𝒃 at the point
the second persons is in the direction of (1, 1) and the co-ordinate on the first
the instantaneous motion, then quadrant. If its area is 2 then the value of
𝜸 𝜹 𝜶 𝜷 b is
(a) 𝜶 + 𝜷 + 𝟏 = 𝟎 (b) 𝜸 + 𝜹 − 𝟏 =
𝟎 (c) 𝛼𝛾 +𝛽𝛿 = 1 (d) none (a) -1 (b) 3
of these (c) -3 (d) 1
𝑑𝑦
Ans. (a) Ans. (c) 𝑑𝑥 = 2𝑥 + 𝑏

Given curve is 𝑥 3 + 𝑦 3 = 𝑎3 ……….(1) ∴ The equation of the tangent at (1, 1) is


Let P≡ (𝛼, 𝛽), 𝑄 ≡ (𝛾, 𝛿) y-1= (2+b)(x-1)
Since P and Q lie on(1) or (2+b)x- y= 1+ b

97
Solving Mathematical Problems
1+𝑏 𝟖
∴ OA= 2+𝑏 𝑎𝑛𝑑 𝑂𝐵 = −(1 + 𝑏) 27. Equation sin x + 2sin 2x +3 sin 3x= 𝝅
has at least one root in
Since 𝛥 AOB lies in the first quadrant,
𝟑𝝅 𝝅
1+𝑏 (a) (𝝅, ) (b) (𝟎, 𝟐 )
∴ 2+𝑏 > 0𝑎𝑛𝑑 1 + 𝑏 < 0 𝟐
𝝅
(c) ( 𝟐 , 𝝅) (d) none of these
∴1+b < 0, 2+b < 0⟹ b < -2………..(1)
Ans. (b)
Now, area (𝛥AOB)= 2
8
1 1+𝑏
Let f(x) = 𝜋 𝑥 + cos 𝑥 + cos 2𝑥 + cos 3𝑥
∴ 2= 2 .2+𝑏 {−(1 + 𝑏)}
8
or, 4(2+b)+ (1 + 𝑏)2 = 0 𝑡ℎ𝑒𝑛 𝑓 ′ (𝑥) = 𝑥 − 𝑠𝑖𝑛𝑥 − 2sin 2𝑥
𝜋
− 3 sin 3𝑥
or, 𝑏 2 + 6𝑏 + 9 = 0
f(x) is continuous and differentiable at every
Or (𝑏 + 3)2 = 0 point
∴ b= -3 > 1 𝜋
Also f(0)= f(2 ) ∴ By Rolle’s theorem
26. If 2a+ 3b +6c = 0, then equation 𝒂𝒙𝟐 + 𝜋
f’(c)= 0 for at least one c in (0, 2 )
𝒃𝒙 + 𝒄 = 𝟎 has at least one root in

(a) (-1, 1) (b) (1, 2)


(c) (-1, 0) (d) (2, 3) 28. Let f(x) = 1+|x- 2|+|sin x|, then
Ans. (a) Lagrange’s mean value theorem is
applicable for f(x) in
Let f’(x) = a𝑥 2 + 𝑏𝑥 + 𝑐, then
(a) [0, 𝜋] (b) [𝜋, 2𝜋]
𝑎𝑥 3 𝑏𝑥 2 2𝑎𝑥 3 +3𝑏𝑥 2 +6𝑐𝑥 𝝅 𝟑𝝅 𝝅 𝝅
f(x) = + + 𝑐𝑥 = (c)[ 𝟐 , ] (d) [− 𝟐 , 𝟐 ]
3 2 6 𝟐

2𝑎 + 3𝑏 + 6𝑐 Ans. (b)
𝑓(1) = = 0, 𝑓(0) = 0
6
f(x) is not differentiable at x= 2 and x= n𝜋,
∴ f(0)= f(1)
n ∊I
∴ there exists 𝛼, 0< 𝛼 < 1 such that f’ (𝛼)= 0
29. let f(x) and g(x) be differentiable
2
i.e., equation a𝑥 + 𝑏𝑥 + 𝑐 = 0 has at least functions for 0≤ 𝒙 ≤ 𝟏 such that f(0) = 2,
one root in (0, 1). g(0)= 0, f(1)= 6.Let there exist a real
number c in (0, 1) such that f’(c)= 2g’(c),
then g(1)=

98
Solving Mathematical Problems

(a) 1 (b) 2 ISI OBJECTIVE SAMPLE PAPER


(c) -2 (d) -1 WITH SOLUTIONS
Ans. (b)
SET – 13
Let 𝜙(x)= f(x)+Ag(x)

Then 𝜙’(x)= f’(x)+ Ag’(x)


There will be 30 questions in MMA Paper.
Choosing A such that φ(0)= 𝜙(1), For each question, exactly one of the four
choices is correct. You get four marks for
𝑓(1)−𝑓(0) each correct answer, one mark for each
We have A= - 𝒈(𝟏)−𝒈(𝟎)
unanswered question, and zero mark for
each incorrect answer.
for this value of A using Rolle’s theorem for
𝜙(x) in (0, 1), we have
1. When m, n are positive integers, then
𝜙’(c)= 0 for some c ∊ (0, 1) (m+n)! is divisible by
𝑓 ′ (𝑐) 𝑓(1) − 𝑓(0)
⟹ = −𝐴 = (a) m!n! (b) m! +n!
𝑔′ (𝑐) 𝑔(1) − 𝑔(0) (c) m!-n! (d) none
6−2 4
= ⟹2=
𝑔(1) − 0 𝑔(1) Ans. (a) (m+n)!=
⟹ 𝑔(1) = 2 1.2…….m(m+1)(m+2)….(m+n)
𝑚! .(m+1)(m+2)….(m+n)n!
30. Which of the following function does = 𝑛!
not obey mean value theorem in [0, 1]
= m! n! × an integer
𝟏 𝟏 𝟐 𝟏
(a) f(x)= 𝟐 - x, x < ½; f(x)=(𝟐 − 𝒙) , 𝒙 ≥ 𝟐 (𝑚+𝑛)!
𝐬𝐢𝐧 𝒙 ⇒ = 𝑎𝑛 𝑖𝑛𝑡𝑒𝑔𝑒𝑟.
(b) f(x) = , 𝒙 ≠ 𝟎; 𝒇(𝒙) = 𝟏, 𝒙 = 𝟎 𝑚!𝑛!
𝒙
2. If n and r are positive integers such
(c) f(x)= x |x| that 0 < r< n, then the roots of the
(d) f(x)= |x|
quadratic equation 𝒏𝒄𝒓−𝟏 𝒙𝟐 + 𝟐. 𝒏𝒄𝒓 𝒙 +
Ans. (a) 𝒏𝒓+𝟏 = 0 are

Let f(x) = 𝑥 3 − 3𝑥 + 𝑘 (a) rational (b) imaginary


(c) real & distinct (d)
Then f’(x) = 3(𝑥 2 − 1) < 0 𝑖𝑛 (0, 1) none
⟹ f’(x) has no root in (0, 1) Ans. (c) Discriminant(D) = 4 {(𝑛𝑐𝑟 )2 −
But f(x) = 0 has two distinct roots 𝛼and 𝛽 in 𝑛𝑐𝑟−1 . 𝑛𝑐𝑟+1 } = 4 (a - b),
(0, 1)
Where, a = (𝑛𝑐𝑟 )2 and b = 𝑛𝑐𝑟−1 . 𝑛𝑐𝑟+1
⟹ f’(x)= 0 has at least one root in (𝛼, 𝛽).

99
Solving Mathematical Problems
𝑎 𝑟+1 𝑛−𝑟+1 1 1 𝒎 𝒇(𝒎)
∴𝑏 = . = (1 + 𝑟 ) (1 + 𝑛−𝑟) > 1 (a) 𝒏
(b) 𝒇(𝒏)
𝑟 𝑛−𝑟
as, n > r 𝒎
(c) 𝒇(𝟏) (d) none
𝒏

∴a>b
Ans. (c) since f(x+ y)= f(x)+f(y) ∀ x, y
⇒ D > 0.
So, f(x) =ax is the functional form, where a=
3. Given, f(x+y) = f(x)+ f(y) ∀ 𝒙, 𝒚 ∊ ℝ constant.
and f(1)=3, then the value of ∑𝒏𝒓=𝟏 𝒇(𝒓)is ∴f(1)=a
𝟑
(a) 𝟑𝒏 − 𝟏 (b) 𝟐 (𝟑𝒏 − 𝑚 𝑚
i.e. f( 𝑛 )= a( 𝑛 )= f(1).
𝑚
𝑛
𝟑
𝟏) (c) 𝟐 . 𝟑𝒏 − 𝟏
6. Let f(x)= (𝒙 − 𝒂)𝟑 + (𝒙 − 𝒃)𝟑 + (𝒙 −
(d) none
𝒄)𝟑 , a < b < c. Then no. of real roots of
Ans. (b) let f(x)= 𝑎 𝜆𝑥 , 𝜆 is a constant f(x) = 0 is

f(1)= 𝑎 𝜆 = 3 (a) 3 (b) 2


(c) 1 (d) none
∴ ∑𝑛𝑟=1 𝑓(𝑟) = ∑𝑛𝑟=1 𝑎 𝜆𝑟 = 𝑎 𝜆 + 𝑎2𝜆 +
𝑎3𝜆 + … . +𝑎𝑛𝜆 Ans. (c) f′(x)= 3{(𝑥 − 𝑎)2 + (𝑥 − 𝑏)2 +
(𝑥 − 𝑐)2 }> 0
𝑎𝜆 (𝑎𝑛𝜆 −1) 3(3𝑛 −1) 3
= = = 2 (3𝑛 − 1) ∴ f(x) is an increasing function
(𝑎𝜆 −1) 3−1

𝟏 𝟏 Note that f(x)< 0 if x< a


4. If f(x). 𝒇 (𝒙) = f(x) + 𝒇 (𝒙) and f(4)= 65,
then f(6)is f(x) > 0 if x > c
(a) 201 (b) 205 ∴ there is one root.
(c) 215 (d) none

Ans. (d) f(x)= 𝑥 𝑛 ± 1, where n ∊ I


7. A, B are two square matrix such that
1st case: f(4)= 4𝑛 + 1= 65 ⇒ 4𝑛 = 64 ⇒ n=3 AB = A and BA = B, then
2nd case: f(4)= 4𝑛 − 1=65 ⇒ 4𝑛 =66, (a) A, B both are idempotent
impossible as n ∊ I (b) only A is idempotent.

∴ f(x)= 𝑥 3 + 1 (c) only B is idempotent


(d) none
∴ f(6) =63 + 1 =217.
Ans. (a) (AB)A = A× 𝐴 = 𝐴2
5. Let f be a function of a real variable
such that it satisfies f(x +y) = f(x)+f(y) ⇒ A (BA)= 𝐴2
𝒎
)∀ 𝒙, 𝒚 ∊ 𝑰, then 𝒇 ( 𝒏 )

100
Solving Mathematical Problems

⇒ AB= 𝐴2 number of 2× 2 square matrices with


elements 1, -1 is 24 = 16.
⇒A = 𝐴2
Out of these 16 matrices, following matrices
Hence A is idempotent are singular
Similarly, B is idempotent 1 1 −1 −1 1 1 −1 −1
[ ],[ ],[ ],[ ],
−1 −1 1 1 1 1 −1 −1
8. Let B is non- singular matrix and A is a
square matrix, then det(𝑩−𝟏 𝑨𝑩) = −1 1 1 −1 1 −1 −1 1
[ ],[ ],[ ],[ ].
−1 1 1 −1 −1 1 1 −1
(a) det B (b) det A
Thus the number of non-singular matrices =
(c) det(AB) (d) none
16-8 =8
Ans. (b) det(𝐵 −1 𝐴𝐵) = 8 1
det(𝐵 −1 ) det(𝐴) det(𝐵) = ∴ required probability is = 16 = 2.
1
. det(𝐴) . det(𝐵) = det 𝐴
det(𝐵) 11. Let 𝑨𝟏 , 𝑨𝟐 , … . , 𝑨𝒏 are n independent
𝟏
9. A subset A of the set X= {1, 2, 3, events with P(𝑨𝒋 ) = 𝟏+𝒋 (𝟏 ≤ 𝒋 ≤ 𝒏). The
…..,100}is chosen at random. The set X is probability that not one of 𝑨𝟏 , 𝑨𝟐 , … . , 𝑨𝒏
reconstructed by replacing the elements occur is
of A and another subset B of x is chosen
𝟏 𝟏
at random. The probability that A∩B (a) (𝒏+𝟏)!
(b) 𝒏+𝟏
contains exactly 10 elements is 𝒏!
(c) (𝒏−𝟏)! (d) none
𝟑 𝟏
(a) (𝟏𝟎𝟎
𝟏𝟎
)(𝟒)𝟗𝟎 (b) (𝟏𝟎𝟎
𝟏𝟎
)(𝟐)𝟏𝟎𝟎
Ans. (c) Required probability = P (𝐴1 ∩
𝟐
(c) (𝟏𝟎𝟎
𝟏𝟎
)(𝟑)𝟏𝟎𝟎 (d) none 𝐴2 ∩ … . .∩ 𝐴𝑛 )

Ans. (d) A and B can be chosen in general in = P (𝐴1 )P(𝐴2 )…. P(𝐴𝑛 )
={∑90
𝑟=0 90𝑐𝑟 2
90−𝑟
}.100𝑐10 Ways 1 2 𝑛 𝑛!
= 2 × 3 × … .× 𝑛+1 = (𝑛+1)!
(Ans.)
90 90
= (1 + 2) . 100𝑐10 = 100𝑐10 . 3 ways.
1 𝑗
[∵P (𝐴𝑗 )= 1-𝐴𝑗 ) = 1 − 1+𝑗 = 1+𝑗]
10. Let A be a 2× 2 matrix to be written
down using the numbers 1, -1 as elements. 12. Given a circle of radius r. Tangents
The probability that the matrix is non- are drawn from points A and B lying on
singular is one of its diameters which meet at a point
(a)1/2 (b) 3/8 P lying on another diameter
(c) 5/8 (d) none perpendicular to the other diameter. The
minimum area of the triangle PAB is
Ans. (a) A 2× 2 matrix has 4 elements each
of which can be chosen in 2 ways, so, total

101
Solving Mathematical Problems

(a) 𝒓𝟐 (b) 2𝒓𝟐 ⤇ (𝑋 − 𝑋2 )(𝑋 − 𝑋3 ) … . (𝑋 − 𝑋𝑛 ) =


𝒓𝟐 𝑋 𝑛 +𝑎𝑋+𝑏
(c) 𝜋𝒓𝟐 (d) (𝑋−𝑋1 )
𝟐

Ans. (b) ⤇ (𝑋1 − 𝑋2)( 𝑋1 − 𝑋3)( 𝑋1 −


𝑋4)….. (𝑋1 − 𝑋𝑛 ) =
OP = r sec 𝛼 𝑋 𝑛 +𝑎𝑋+𝑏
lt , [𝐵𝑦 𝐿′ ℎ𝑜𝑠𝑝𝑖𝑡𝑎𝑙 𝑟𝑢𝑙𝑒]
𝑋→𝑋1 (𝑋−𝑋1 )
OA = r cosec 𝛼
1 = n𝑋1 𝑛−1 + 𝑎.
Area of 𝛥 APB = 2 . 𝑂𝑃. 2𝑂𝐴 = 𝑂𝑃. 𝑂𝐴 =
2𝑟 2
sin 2𝜃
15. Let these three values of X such that
2
∴Minimum area = 2𝑟 X, [X], {X} are in H.P. then the number of
(𝒙−𝟏)(𝒙−𝟐)𝟐 (𝒙+𝟒)
values of x are
13. Solution set of (𝒙+𝟐)(𝒙−𝟑)
≥ 𝟎 𝒊𝒔
(a) 1 (b) 2
(a) (−∞, −𝟒] ∪ (−𝟐, 𝟏] ∪ {𝟐} ∪ (𝟑, ∞) (c) 3 (d) none
(b) [-4, -2)∪ [𝟏, 𝟐] ∪ (𝟑, ∞) 2 {𝑋}𝑋
Ans. (a) [X]= {𝑋}+𝑋 ; Now, we know X =
(c) (−∞, −𝟒] ∪ {𝟐} ∪ (𝟑, ∞) [X]+ {X}, putting value of X , we get
(d) none
⤇ [𝑋]2 = 2 {𝑋}2
(𝑥−1)(𝑥−2)2 (𝑥+4)
Ans. (a) G(x) = (𝑥+2)(𝑥−3)
≥0
1
⤇ {𝑋}2 = 2 [𝑋]2
+−+−−+
So, sign change is like this :- −4 −2 ,
1 2 3 [𝑋]2
⤇0< < 1,
2
⤇ x ∊ (−∞, −4] ∪ (−2, 1] ∪ {2} ∪ (3, ∞).
⤇ 0 < [𝑋]2 < 2, ⤇ 0 < [X] < √2 ⤇ [X]= 1
1
⤇ {X}= ,
√2
𝒏
14. If 𝑿𝟏 , 𝑿𝟐 , … . , 𝑿𝒏 are the roots of 𝒙 +
1
𝒂𝒙 + 𝒃 = 𝟎, then the value of So, X = 1+ is the only value.
√2

(𝑿𝟏 − 𝑿𝟐 )( 𝑿𝟏 − 𝑿𝟑 )( 𝑿𝟏 − 𝑿𝟒 )….(𝑿 −
𝑿𝒏 ) =
𝟐𝒈(𝒙)
𝒏−𝟏
16. Let f(x) = 𝐬𝐢𝐧−𝟏 𝟏+(𝒈(𝒙))𝟐 , then
(a) n𝑿𝟏 + 𝒃 (b) n𝑿𝟏
(c) n𝑿𝟏 𝒏−𝟏 + 𝒂 (d) none (a) f(x) is decreasing if g(x) is increasing
and |g(x)|> 1
Ans. (c) 𝑥 𝑛 + 𝑎𝑥 + 𝑏 = (𝑋 − 𝑋1 )(𝑋 −
𝑋2 )(𝑋 − 𝑋3 ) … . (𝑋 − 𝑋𝑛 ) (b) f(x) is an increasing function if g(x) is
increasing and |g(x)|≤ 𝟏

102
Solving Mathematical Problems
3
(c) f(x) is decreasing if g(x) is decreasing This will be true if 𝑝𝑞 − 𝑞 2 − 2 ≤
and |g(x)|> 1
0 𝑜𝑟 2𝑞 2 − 2𝑝𝑞 + 3 ≥ 0 𝑓𝑜𝑟 𝑎𝑙𝑙 𝑞 ∊ 𝑅 ⟹
(d) none of these 𝐷≤0

Ans. (c) 𝑜𝑟 4𝑝2 − 24 ≤ 0 𝑜𝑟 − √6 ≤ 𝑝 ≤ √6.

𝑓(𝑥)
2
𝑔(𝑥) 18. The lengths of the sides of a triangle
= sin−1
1 are 𝛼- 𝛽, 𝛼 +𝛽 and √𝟑𝜶𝟐 + 𝜷𝟐 , (𝜶 > 𝛽 >
1+( )
𝑔(𝑥)2 0). Its largest angle is
1
= sin−1 (sin 2𝜃) , 𝑤ℎ𝑒𝑟𝑒 tan 𝜃 = 𝟑𝝅 𝝅
𝑔(𝑥) (a) (b) 𝟐
𝟒
𝟐𝝅 𝟓𝝅
= 2 𝜃 = 2 cot −1(𝑔(𝑥)), |𝑔(𝑥)| > 1 (c) (d)
𝟑 𝟔

2 Ans. (c)
𝑡ℎ𝑢𝑠 𝑓 ′ (𝑥) = 1+(𝑔(𝑥))2 𝑔′ (𝑥), |𝑔(𝑥)| ≤ 1 =
2
− 1+(𝑔(𝑥))2 𝑔′ (𝑥), |𝑔(𝑥)| > 1 . 𝛼, 𝛽 > 0

𝒙 ∴ 𝛼+ 𝛽 > 𝛼- 𝛽
17. Let f(x)− ∫𝟎 (𝒄𝒐𝒄𝟒 𝜽 + 𝒔𝒊𝒏𝟒 𝜽)𝒅𝜽 +
(𝒑𝒒 − 𝒒𝟐 − 𝟏)𝒙 then for f(x) to be a Also 𝛼- 𝛽 > 0
decreasing function in R for all real
∴ 𝛼 > 𝛽 then 3𝛼 2 + 𝛽 2 − (𝛼 + 𝛽)2 =
values of q independent of x, the set of
2𝛼(𝛼 − 𝛽) > 0
values of p is

(a) [10, √𝟏𝟎] (b) [−√𝟏𝟎, √𝟏𝟎] ⟹√3𝛼 2 + 𝛽 2 > 𝛼 + 𝛽


(c) [-√𝟏𝟎, 𝟎] (d) none of these Let a = 𝛼 − 𝛽, 𝑏 = 𝛼 + 𝛽, 𝑐 = √3𝛼 2 + 𝛽 2
Ans. (d) 𝑎2 +𝑏2 −𝑐 2
Now cos 𝐶 = =
2𝑎𝑏
𝑓 ′ (𝑥) = −(𝑐𝑜𝑠 4 𝑥 + 𝑠𝑖𝑛4 𝑥) 2𝛼2 +2𝛽 2 −3𝛼2 −𝛽2 1
= −2
+ (𝑝𝑞 − 𝑞 2 − 1) 2(𝛼2 −𝛽 2 )
= −(𝑐𝑜𝑠 2 𝑥 + 𝑠𝑖𝑛2 𝑥)2 2𝜋
+ 2𝑠𝑖𝑛2 𝑥𝑐𝑜𝑠 2 𝑥 + 𝑝𝑞 − 𝑞 2 ∴𝐶 = 3
−1
19. The set of critical points of the
1 function
= 𝑝𝑞 − 𝑞 2 + 𝑠𝑖𝑛2 2𝑥 − 2
2 𝒙 𝟏
f(x) = 𝒙 − 𝑰𝒏 𝒙 + ∫𝟐 ( 𝒕 − 𝟐 − 𝟐 𝐜𝐨𝐬 𝟒𝒕) 𝒅𝒕
1 3
But f’(x) ≤ pq - 𝑞 2 − 2 + 2 = 𝑝𝑞 − 𝑞 2 − 2 is
For f(x) to be decreasing in R, f’(x) ≤ 0

103
Solving Mathematical Problems
𝝅 𝝅 𝝅 𝒏𝝅 f(sin 𝛾) = (sin 𝛾- sin 𝛼)(sin 𝛾- sin 𝛽)> 0
(a) {𝟐 , 𝒏𝝅 + 𝟔 } , 𝒏 ∊ 𝑰 (b) {𝟔 , ±
𝟐
𝝅
, 𝒏 ∊ 𝑵} (c) {n𝜋}, n ∊I (d) none Hence equation f(x) = 0 has one root
𝟔
of these between sin 𝛼 and sin 𝛽 and other between
sin 𝛽 and sin 𝛾.
Ans. (b)
21. If f(x)= ∑𝟏𝟎𝟎 𝒓
𝒓=𝟎 𝒂𝒓 𝒙 and f(0) and f(1) are
1 1
f’(x)= 1 − 𝑥 + 𝑥 − 2 − 2 cos 4𝑥 𝑜𝑟 𝑓 ′ (𝑥) = odd numbers, then for any integer x
−1 − 2 cos 4𝑥
(a) f(x) is odd or even according as x is
for critical points f’(x)= 0 odd or even

1 2𝜋 (b) f(x) is even or odd according as x is


⟹ cos 4𝑥 = − 2 = cos ⟹ 4𝑥 = 2𝑛𝜋 ±
3 odd or even
2𝜋 𝑛𝜋 𝜋
⟹𝑥= ± 6,𝑛 ∊ 𝐼
3 2
(c) f(x) is even for all integral value of x
But for log x to be defined, x > 0
(d) f(x) is odd for all integral values of x
𝜋 𝑛𝜋 𝜋
∴x= 6, ± 6,𝑛 ∊ 𝑁
2 Ans. (d) Given f(x)= 𝑎0 + 𝑎1 𝑥 + 𝑎2 𝑥 2 +
𝝅 ⋯ + 𝑎100 𝑥100
20. If 0 < 𝛼< 𝛽 < 𝛾< 𝟐 , then the equation
𝟏 𝟏 𝟏 ∴ f(2m) = 𝑎0 + 𝑎𝑛 (even number)
+ 𝒙−𝒔𝒊𝒏𝜷 + 𝒙−𝒔𝒊𝒏𝜸 = 𝟎 has
𝒙−𝒔𝒊𝒏𝜶
= f(0) + an even number
(a) imaginary roots
(b) real and equal roots = an odd number

(c) real and unequal roots f(2m +1) = 𝑎0 + 𝑎1 (1 + 2𝑚) + 𝑎2 (1 +


(d) rational roots 2𝑚)2 + ⋯ + 𝑎100 (1 + 2𝑚)100

Ans. (c) = 𝑎0 + 𝑎1 + ⋯ + 𝑎100 + 𝑎𝑛 even


𝜋 number
0<𝛼<𝛽<𝛾<2
= f(1) + an even number
⟹ sin 𝛼< sin 𝛽 < sin 𝛾
= an odd number
Given equation is
22. Let P= √𝟑 𝒆𝒊𝝅/𝟑 , 𝑸 =
(x- sin𝛽)(x- sin𝛾 )+(x- sin𝛼) (x- sin𝛾) +( x-
√𝟑 𝒆−𝒊𝝅/𝟑 𝒂𝒏𝒅 𝑹 = √𝟑 𝒆−𝒊𝝅 form a
sin𝛼)(x- sin𝛽 )= 0 triangle PQR in the Argand plane. Then
Or, f(x)= 0 𝛥PQR is

f(sin 𝛼) = (sin 𝛼- sin 𝛽)(sin 𝛼- sin 𝛾)> 0 (a) isosceles (b) equilateral
(c) scalene (d) none of these
f(sin 𝛽) = (sin 𝛽- sin 𝛼)(sin 𝛽- sin 𝛾)< 0

104
Solving Mathematical Problems

Ans. (b) ⟹ |2z − 1|2 = |z − 2|2

The points P, Q, R lie on a circle |z|= √3 ⟹ (2z -1) (2z̅ -1)= (z- 2) (z̅ -2)
2𝜋
with a difference in argument = for any ⟹ 4zz̅- 2z̅- 2z +1 = zz̅ -2z̅ -2z +4
3
two complex numbers.
⟹3|z|2 = 3 ⟹|z|= 1
∴ 𝛥 PQR is equilateral.
Again
23. A, b and C are points represented by
complex numbers 𝒛𝟏 , 𝒛𝟐 , 𝒂𝒏𝒅 𝒛𝟑 . If the |𝑧1 − 𝑧2 | =|𝑧1 − 𝛼 + 𝑧2 − 𝛽 + 𝛼 + 𝛽|≤ |𝑧1 −
circumcentre of the 𝛥ABC is at the origin 𝛼|+|𝑧2 − 𝛽|+|𝛼+𝛽|< 𝛼+𝛽+ |𝛼+ 𝛽|
and the altitude AD of the triangle meets
= 2|𝛼+ 𝛽|[∵ 𝛼, 𝛽> 0]
the circumcircle again at P, then P
represents the complex number 1𝑧 +𝑧
2 1 𝑧 +𝑧
2
∴ | 𝛼+ | < 2 ⟹ | 𝛼+ | < 2|𝑧|
𝛽 𝛽
−𝒛𝟏 𝒛𝟐 𝒛𝟏 𝒛𝟑
(a) (b)
𝒛𝟑 𝒛𝟐 25. If 1, 𝜶𝟏 , 𝜶𝟐 , … , 𝜶𝟑𝒏 be the roots of
−𝒛𝟐 𝒛𝟑 𝒛𝟏 𝒛𝟐
(c) 𝒛𝟏
(d) 𝒛𝟑 equation 𝒙𝟑𝒏+𝟏 − 𝟏 = 𝟎 , and 𝜔 be an
imaginary cube root of unity, then
Ans. (c) (𝝎𝟐 −𝜶𝟏 )(𝝎𝟐 −𝜶𝟐 )…(𝝎𝟐 −𝜶𝟑𝒏 )
(𝝎−𝜶𝟏 )(𝝎−𝜶𝟐 )…(𝝎−𝜶𝟑𝒏 )
=
|𝑧1 |= |𝑧2 |𝑧3 | =|z|= r (say)
(a) 𝜔 (b) –𝜔
∴ |𝑧1 | 2 = |𝑧2 | 2 = |𝑧3 | 2 = |𝑧|2 = 𝑟 2 (c) 1 (d) 𝝎𝟐
⟹𝑧1 𝑧̅1 = 𝑧2 𝑧̅2 = 𝑧3 𝑧̅3 = 𝑧𝑧̅ = 𝑟 2 Ans. (c)
𝑧−𝑧 1 3 𝑧 −𝑧
3 𝑧−𝑧1 𝑧 −𝑧3
∴𝑧̅ −̅̅̅ = ̅̅̅−𝑧 ⟹ 𝑟2 𝑟2
= − 𝑟32 𝑟2
= Since 1, 𝛼1 , 𝛼2 , … , 𝛼3𝑛 are the roots of
𝑧 1 𝑧 ̅̅̅
2 3 − −
𝑧 𝑧1 𝑧2 𝑧3
equation
−𝑧𝑧1 = 𝑧2 𝑧3
𝑧
𝑥 3𝑛+1 − 1 = 0
⟹ z = - 𝑧3
1
∴ 𝑥 3𝑛+1 − 1 = (x- 1) (x-𝛼1 ) (x-𝛼2 ) … (x-
24. If |2z- 1|= |z- 2| and 𝒛𝟏 , 𝒛𝟐 , 𝒛𝟑 are 𝛼3𝑛 )
complex numbers such that |𝒛𝟏 − 𝜶|< 𝛼,
𝑥 3𝑛+1 −1
|𝒛𝟐 − 𝜷|< 𝜷, then |
𝒛𝟏 + 𝒛𝟐
| ⟹ (x-𝛼1 ) (x-𝛼2 ) … (x- 𝛼3𝑛 ) = 𝑥−1
𝜶+𝜷

⟹ (x-𝛼1 ) (x-𝛼2 ) … (x - 𝛼3𝑛 ) = 1+ x+𝑥 2 +


(a) < |z| (b) <2|z|
⋯ + 𝑥 3𝑛 ……………(1)
(c) > |z| (d) > 2|z|
(𝜔 2 −𝛼1 )(𝜔2 −𝛼2 )…(𝜔 2 −𝛼3𝑛 )
Ans. (b) ∴ (𝜔− 𝛼1 )(𝜔− 𝛼2 )…(𝜔−𝛼3𝑛 )
=
𝜔 6𝑛+2 −1 𝜔−1 (𝜔 2 −1) (𝜔−1)
|2z- 1|= |z- 2| . 𝜔3𝑛+1 −1 = =1
𝜔 2 −1 (𝜔 2 −1)(𝜔−1)

105
Solving Mathematical Problems

26. If 𝜔 be an imaginary cube root of According to question,


𝟏
unity and 𝒙𝟏 , 𝒙𝟐 , 𝒙𝟑 ∊ 𝑹, ∑𝟑𝒓=𝟏 𝒙 =
𝒓 +𝝎 𝑥 𝑛 − 1= (x- 1) (x- 𝛼) (x- 𝛼 2 )… (x - 𝛼 𝑛−1 )
𝟏 𝟏
𝟐𝝎𝟐 , ∑𝟑𝒓=𝟏 𝒙 𝟐
= 𝟐𝝎 then ∑𝟑𝒓=𝟏 𝒙 =
𝒓 +𝝎 𝒓 +𝟏 Taking logarithm, we get
(a) -2 (b) 2 log(𝑥 𝑛 − 1) = log(𝑥 − 1) + log(𝑥 − 𝛼) +
(c) 0 (d) none of … + log(𝛼 𝑛−1 )
these
Differentiating w.r.t.x, we get
Ans. (b)
𝑛𝑥 𝑛−1 1 1 1 1
= 𝑥−1 + 𝑥−𝛼 + 𝑥−𝛼2 + ⋯ + 𝑥−𝛼𝑛−1
Consider the equation 𝑥 𝑛 −1

1 1 1 2 𝑥 𝑛 −1 1 1 1
+𝑥 +𝑥 =𝑥 ⟹ (𝑥−1 + 𝑥−𝛼 + ⋯ + 𝑥−𝛼𝑛−1 )= n
𝑥1 +𝑥 2 +𝑥 3 +𝑥 𝑥 𝑛−1

………………..(1) 3𝑛 −1 1 1
⟹ (2 + ∑𝑛−1
𝑟=1 3−𝛼𝑟 ) = 𝑛,
𝑥 𝑛−1
Equation (1) is
[Putting x= 3]
x (x+𝑥2 ) (x+𝑥3 ) + 𝑥(𝑥 + 𝑥1 )(𝑥 + 𝑥3 ) +
𝑥(𝑥 + 𝑥1 )(𝑥 + 𝑥2 ) − 2(𝑥 + 𝑥1 )(𝑥 + 𝒙𝟐 + 𝒙 𝒙 − 𝟏 𝒙 + 𝟏
𝑥2 )(𝑥 + 𝑥3 )= 0 28. If | 𝒙 𝟐𝒙 𝟑𝒙 − 𝟏| = 𝒑𝒙𝟒 +
𝟒𝒙 + 𝟏 𝒙 − 𝟐 𝒙 + 𝟐
or, 𝑥 3 − (𝑥1 𝑥2 + 𝑥2 𝑥3 + 𝑥3 𝑥1 ) − 𝒒𝒙𝟑 + 𝒓𝒙𝟐 + 𝒔𝒙 + 𝟏 be an identity in x
2𝑥1 𝑥2 𝑥3 = 0 ……………(2) and 𝜔 be an imaginary cube root of unity,
𝒂+𝒃𝝎+𝒄𝝎𝟐 𝒂+𝒃𝝎+𝒄𝝎𝟐
then 𝒄+𝒂𝝎+𝒃𝝎𝟐 + 𝒃+𝒄𝝎+𝒂𝝎𝟐 =
from given conditions it is clear that 𝜔 and
𝜔2 are the roots of equation (1) i.e. of eqn.
(a) p (b) 2p
(2).
(c) -2p (d) –p
Therefore, if 𝛾 be its third roots, then
Ans. (a)
𝜔+ 𝜔2 + 𝛾= 0 ⟹ 𝛾= 1
Equating the coefficient of 𝑥 4 , we get
∴ 1 is a root of equation (1)
P=1. (2-3)= -1
1
⟹ ∑3𝑟=1 =2 Given expression
𝑥𝑟 +1

𝜔 2 (𝑐+𝑎𝜔+𝑏𝜔2 ) 𝜔 (𝑏+𝑐𝜔+𝑎𝜔 2 )
27. If 1, 𝛼, 𝜶𝟐 , … , 𝜶𝒏−𝟏 be the nth roots of = + = 𝜔2 +
𝑐+𝑎𝜔+𝑏𝜔2 𝑏+𝑐𝜔+𝑎𝜔2
𝟑𝒏 −𝟏 𝟏 𝟏
unity, then ( 𝟑𝒏−𝟏 ) (∑𝒏−𝟏
𝒓=𝟏 𝟑−𝜶𝒓 + 𝟐) = 𝜔 = −1 = 𝑝

(a) –n (b) 0 29. If the equation- (𝒑 + 𝒊𝒒)𝒙𝟐 + (𝒎 +


(c) n (d) 1 𝒊𝒏)𝒙 + 𝒓 = 𝟎 has real roots where p, q, m,
n and r are real (r≠ 𝟎) then 𝒑𝒏𝟐 + 𝒓𝒒𝟐 −
Ans. (c) 𝒎𝒏𝒒 =
106
Solving Mathematical Problems

(a) 1 (b) 0 1. Let X be a non-empty set and let P(X)


(c) -1 (d) none of these denotes the collection of all subsets of X.
Define
Ans. (b)
F: X× 𝑷(𝑿) → 𝑹 by
Let 𝛼 be the real root satisfying the given
equation, then 𝟏 𝒊𝒇 𝒙 ∈ 𝑨
f(x, A) = {
𝟎 𝒊𝒇 𝒙 ∉ 𝑨
(p𝛼 2 + 𝑚𝛼 + 𝑟) + 𝑖 (𝑞𝛼 2 + 𝑛𝛼)= 0
Then f(x, A∪B) equals
Equating real and imaginary parts, we have
𝑛 p𝑛2 𝑚𝑛 𝑛 (a) f(x, A) + f(x, B)
𝛼 = − 𝑞 𝑎𝑛𝑑 − + 𝑟 = 0 [𝛼 = − 𝑞 ]
𝑞2 𝑞 (b) f(x, A) + f(x, B) – 1
⟹ p𝑛2 + r𝑞 2 − 𝑚𝑛𝑞 = 0 (c) f(x, A) + f(x, B) – f(x, A)f(x, B)
(d) none
30. If n(A)= p, n(B)= q and total number
of mappings from A to 343, then p - q = Ans. (c) n(A∪B) = n(A) + n(B) – n(A∩B)
(a) 3 (b) -3 So, f(x, A∪B) = f(x, A) + f(x, B) – f(x,
(c) 4 (d) none of these A)f(x, B)
Ans. (d) 𝟏
2. The set { 𝒙 ∶ ∣∣ 𝒙 + 𝒙 ∣∣> 6 } equals the
Total number of mapping from A to B = 𝑞 𝑝 set

Given 𝑞 𝑝 = 343 = 73 ⟹ 𝑞 = 7, 𝑝 = 3 (a) (0, 𝟑 − 𝟐√𝟐) ∪ (𝟑 + 𝟐√𝟐, ∞)


∴ p – q= - 4 (b) (-∞, −𝟑 − 𝟐√𝟐) ∪ (−𝟑 + 𝟐√𝟐, ∞)

(c) (-∞, 𝟑 − 𝟐√𝟐) ∪ (𝟑 + 𝟐√𝟐, ∞)


(d) none
ISI OBJECTIVE SAMPLE PAPER
Ans. (c) x2 – 6x + 1 = 0 gives us x =
WITH SOLUTIONS −6±√36−4
= 3 ± 2√2
2
SET – 14
So, set is (-∞, 3 − 2√2) ∪ (3 + 2√2, ∞)

3. If f : (3, 6) ⟶(2, 5) is a function defined


There will be 30 questions in MMA Paper. 𝒙
by f(x)= x-[𝟑] (where [.] denotes the
For each question, exactly one of the four
choices is correct. You get four marks for greatest integer function), then 𝒇−𝟏 𝒙
each correct answer, one mark for each
unanswered question, and zero mark for (a) x -1 (b) x +1 (c) x (d) none of these
each incorrect answer.
Ans. (b)

107
Solving Mathematical Problems

3<x<6 𝑓(𝑥) = [𝑥]2 − [𝑥 2 ]


𝑥 0, 0 ≤ 𝑥 < √2
⟹ [3] = 1 ∴ 𝑓(𝑥) = 𝑥 − 1 ∴ 𝑓 −1 (𝑥) =
= −1, √2 ≤ 𝑥 < √3
𝑥+1 −2, √3 ≤ 𝑥 < 2
4. The possible values of ‘a’ for which the { 0, 𝑥=2

function f(x) = 𝒆𝒙−[𝒙] + 𝐜𝐨𝐬 𝒂𝒙 (where [.] ∴ Range of f(x) is {-2, -1, 0}
denotes the greatest integer function) is
periodic with finite fundamental period is 6. Let f: R⟶R be a function defined by
f(x)= max [x, 𝒙𝟑 ]. The set of all points
(a) 𝜋 (b) 0 where f(x) is not differentiable is
(c) 1 (d) 2
(a) {-1, 1} (b) {-1, 0} (c) {0,
Ans. (a) 1} (d) {-1, 0, 1}
Period of x- [x] is 1. Ans. (d) 𝑓(𝑥) =
max{𝑥, 𝑥 3 }
∴ Period of 𝑒 𝑥−[𝑥] is 1 and period of cos ax is
2𝜋
𝑥, 𝑥 ≤ −1
|𝑎|
𝑥3 , −1<𝑥 ≤0
⟹ 𝑓(𝑥) = {
When a= 𝜋, period of cos ax = 2 𝑥, 0<𝑥 ≤1
𝑥3, 𝑥 ≥1
When a= 2𝜋, period of cos ax= 1
Clearly f(x) is non-differentiable at x= -1, 0,
In first case period of f(x) will be 2 and in 1
second case it will be 1.
7. Let f: R⟶R be a function defined by
But when a = 1 or 2. f(x)= 2x + sinx, x ∊R. Then f is

Period of cos ax will be an irrational number (a) one-to-one and onto


T and L.C.M. of 1 and T will not exist. (b) one-to-one but not onto

5. If f(x)= [𝒙]𝟐 − [𝒙𝟐 ], (where [.] denotes (c) onto but not one-to-one
the greatest integer function), and x ∊[0, (d) neither one-to-one nor onto
2], then the range of f(x) is
Ans. (a)
(a) {-1, 0, 2} (b) {0, 1, 2} (c) {-2, -1, 0} (d)
f’(x)= 2+ cosx > 0∀ x 𝜖 R
{-2, 0, 1}
⟹ f(x) is increasing
Ans. (c)
⟹ f is one-one

Also, as x⟶∞ , f(x)⟶ ∞ 𝑎𝑛𝑑 𝑎𝑠 𝑥 ⟶


−∞, 𝑓(𝑥) ⟶ −∞

108
Solving Mathematical Problems

More over f is continuous log(1 + ℎ) − ℎ + ℎ log(1 + ℎ)


= Lt
ℎ→0 ℎ log(1 + ℎ)
⟹ Range of R ⟹ f is onto. 2
ℎ ℎ3
(ℎ − 2 + 3 − ⋯ ) − ℎ + ℎ log(1 + ℎ)
8. f : [0, ∞) → [𝟎, ∞), 𝒅𝒆𝒇𝒊𝒏𝒆𝒅 𝒃𝒚 𝒇(𝒙) = = Lt
𝒙 ℎ→0 ℎ log(1 + ℎ)
is
𝟏+𝒙
ℎ2 ℎ ℎ2 ℎ2 log(1 + ℎ)
(a) one-one and onto − 2 (1 − 3 + 4 − ⋯ ) +

= Lt 2
(b) one-one but not onto ℎ→0 ℎ log(1 + ℎ)

(c) onto but not one-one 2
1 ℎ ℎ log(1 + ℎ)
− 2 (1 − 3 + 4 − ⋯ ) +
(d) neither one-one nor onto ℎ
= Lt
ℎ→0 log(1 + ℎ)
Ans. (b) ℎ

f : [0, ∞)⟶ [0, ∞) ; 𝑓(𝑥) = 1+𝑥 = 1 − 1+𝑥


𝑥 1 1
−2 + 1 1
= =
1 1 2
∴f’(x)= (1+𝑥)2 > 0
𝟏
10. The value of 𝐥𝐢𝐦 𝒏𝟒 [𝟏(∑𝒏𝒌=𝟏 𝒌) +
𝒏→∞
Hence f(x) is increasing and therefore one-
𝟐(∑𝒏−𝟏 𝒏−𝟐
𝒌=𝟏 𝒌) + 𝟑(∑𝒌=𝟏 𝒌)] + ⋯ + 𝒏. 𝟏] will
one.
be
Also 0≤ x < ∞ ∴ f(0)≤ f(x) < f(∞) 𝟏 𝟏
(a) 𝟐𝟒 (b) 𝟏𝟐
𝑥
⟹0 ≤ f(x) < Lt 𝟏 𝟏
𝑥→∞ 1+𝑥 (c) 𝟔 (d) 𝟑

⟹ 0≤ f(x) < 1 Ans. (a)


Hence range f(x) = [0, 1), therefore f is not (r+1) th term of the series
onto.
𝑛−𝑟
𝒙 𝟏
|
𝒙−𝟏 𝒍𝒐𝒈𝒙
| 𝑡𝑟+1 = (𝑟 + 1). ∑ 𝑘
9. The value of 𝐥𝐢𝐦 (𝒙−𝟏)𝒍𝒐𝒈𝒙 is 𝑘−1
𝒙→𝟎
(𝑟 + 1)(𝑛 − 𝑟)(𝑛 − 𝑟 + 1)
(a) 1 (b) -1 =
2
(c) ½ (d) -½
𝑟[(𝑛 − 𝑟)2 + 𝑛 − 𝑟]
Ans. (c) = + 𝑃(𝑛),
2

Required limit= Lt
𝑥 log 𝑥−(𝑥−1)
= 𝑛2 − (2𝑛 + 1)𝑟 + 𝑛 + 𝑟 2
𝑥→1 (𝑥−1) log 𝑥 𝑤ℎ𝑒𝑟𝑒 𝑃(𝑛) =
(1+ℎ) log(1+ℎ)−ℎ
2
Lt [𝑝𝑢𝑡𝑡𝑖𝑛𝑔 𝑥 = 1 + ℎ] 𝑟 2
ℎ→0 ℎ log(1+ℎ) = (𝑛 − (2𝑛 + 1)𝑟 + 𝑟 2
2
+ 𝑛) + 𝑃(𝑛)

109
Solving Mathematical Problems

1 𝒎𝒙 𝒎𝒙
= [𝑟 3 − (2𝑛 + 1)𝑟 2 + (𝑛2 + 𝑛)𝑟] + 𝑃(𝑛) (a) 𝒙! . 𝒆−𝒎 (b) . 𝒆𝒎
𝒙!
2
(c) 1 (d) 0
∴sum of the series= ∑𝑛−1
𝑟=0 𝑡𝑟+1 =
1 Ans. (a)
[∑𝑛−1 3
𝑟=0 𝑟 − (2𝑛 + 1) ∑𝑛−1 2
𝑟=0 𝑟 + 𝑛(𝑛 +
2
1) ∑𝑛−1 𝑚 𝑥 𝑚 𝑛−𝑥
𝑟=0 𝑟 ] + 𝑞(𝑛) Lt 𝑛𝐶𝑥 ( ) (1 − )
𝑛→∞ 𝑛 𝑛
Where q(n) is a polynomial in n of degree 3 𝑚 𝑥
(1 − 𝑛)
𝑛! 𝑚 𝑥
= Lt ( )
1 (𝑛−1)𝑛 2 (𝑛−1)𝑛(2𝑛−1) 𝑛→∞ 𝑥! (𝑛 − 𝑥)! 𝑛 𝑚 𝑥
= 2 [{ } − (2𝑛 + 1) + (1 − 𝑛 )
2 6
(𝑛−1)𝑛
𝑛 (𝑛 + 1) ] + 𝑞(𝑛) 𝑚 𝑥
2 𝑚 𝑥 (1 − 𝑛 ) 𝑛(𝑛 − 1) … (𝑛 − 𝑥 + 1)
= Lt .
𝑛→∞ 𝑥! 𝑚 𝑥 𝑛𝑥
1 (𝑛2 − 2𝑛 + 1)𝑛2 (1 − 𝑛 )
= [
2 4
𝑚 𝑥 −𝑚 1 𝑥−1
(𝑛2 − 𝑛)(4𝑛2 − 1) = 𝑒 . Lt [1 (1 − ) … (1 − )]
− 𝑥! 𝑛→∞ 𝑛 𝑛
6 𝑚𝑥
𝑛2 (𝑛2 − 1) = 𝑒 −𝑚
+ ] + 𝑞(𝑛) 𝑥!
2
𝟏 𝟏

1 1 4 1 13. If f(x) = 𝐥𝐢𝐦 𝒏𝟐 (𝒙𝒏 − 𝒙𝒏+𝟏 ) , 𝐱 >


𝒏→∞
∴ 𝑅𝑒𝑞𝑑. 𝑙𝑖𝑚𝑖𝑡 = ( − + )
2 4 6 2 𝟎 𝐭𝐡𝐞𝐧 ∫ 𝐱 𝐟(𝐱)𝐝𝐱 equals to
1 3 2 1
= ( − )= 𝒙𝟐
2 4 3 24 (a) (b) 0
𝟐
𝒇(𝒙) 𝟏/𝒙 𝒙𝟑
11. If 𝐋𝐭 [𝟏 + 𝒙 ] = 𝒆𝟑 , then the (c) (d) none of these
𝒙→∞ 𝒙 𝟐
value of the function f(x) may be
Ans. (d)
𝒙𝟐
(a) (b) 𝒙𝟐 1 1
𝟐 𝑓(𝑥) = Lt 𝑛2 (𝑥 𝑛 − 𝑥 𝑛+1 ) , 𝑥
(c) 2𝒙𝟐 (d) 3𝒙𝟐 𝑛→∞
> 0[∞/ 0 𝑓𝑟𝑜𝑚]
Ans. (c)
=
𝑓(𝑥) 1/𝑥 1
log 𝑥
1
log 𝑥
When 𝑓(𝑥) = 𝑘𝑥 2 , Lt [1 + 𝑥 + ] = Lt 𝑛2 [𝑐 𝑛 −𝑒 𝑛+1 ]
𝑥→0 𝑥 𝑛→∞
1/𝑥
Lt [1 + 𝑥(1 + 𝑘)] = 𝑒 𝑘+1
𝑥→0 𝑙𝑜𝑔𝑥
2
𝑙𝑜𝑔𝑥 2
= Lt 𝑛 [{1 + +( ) +⋯}
For 𝑒 𝑘+1 = 𝑒 3 , 𝑘 = 2 𝑛→∞ 𝑛 𝑛

2
𝑙𝑜𝑔𝑥 𝑙𝑜𝑔𝑥 2
∴ f(x) = 2𝑥 − {1 + +( )
𝑛+1 𝑛=1
𝒎 𝒎 𝒏−𝒙
12. 𝐋𝐭 𝒏𝒄𝒙 ( 𝒏 ) 𝒙 (𝟏 − 𝒏 ) equals to + ⋯ }]
𝒏→∞

110
Solving Mathematical Problems

1 1 15. 𝐋𝐭
𝒙 𝐬𝐢𝐧{𝒙}
, where {x} denotes the
= Lt 𝑛2 [( − ) log 𝑥 𝒙→𝟏 𝒙−𝟏
𝑛→∞ 𝑛 𝑛+1
1 1 fractional part of x, is equal to
+ (log 𝑥)2 { 2 − }
𝑛 (𝑛 + 1)2 (a) -1 (b) 0
1 1
+ (log 𝑥)3 { 3 − } (c) 1 (d) does not
𝑛 (𝑛 + 1)3 exist
+⋯]
Ans. (d)
𝑛2
= Lt log 𝑥 Lt {𝑥} = Lt (𝑥 − [𝑥]) = 1 − 0 = 1
𝑛→∞ 𝑛(𝑛 + 1)
𝑥→1−0 𝑥→1−0
1
+ 𝑛2 { 2 Lt {𝑥} = Lt (𝑥 − [𝑥]) = 1 − 1 = 0
𝑛 𝑥→1−0 𝑥→1−0
1
− } (log 𝑥)2 + ⋯ 𝑥 sin{𝑥} 𝑥
(𝑛 + 1)2
∴ Lt = Lt sin{𝑥}
= log 𝑥 𝑥→1−0 𝑥 − 1 𝑥→1−0 𝑥 − 1
= −∞. sin(1) = −∞

𝑥 sin{𝑥} 𝑥 sin{𝑥} {𝑥}


𝐱 𝐧 −𝐱 −𝐧 Lt = Lt .
14. If f(x) = 𝐥𝐢𝐦 𝐱𝐧 +𝐱−𝐧 , 𝟎 < 𝐱 < 𝟏, 𝐧 ∊ 𝑥→1−0 𝑥−1 𝑥→1−0 {𝑥} 𝑥−1
𝐧→∞
−𝟏
𝐍, 𝐭𝐡𝐞𝐧 ∫(𝐬𝐢𝐧 𝐱) 𝐟(𝐱)𝐝𝐱 is equal to 𝑥 sin{𝑥} 𝑥 − [𝑥]
= Lt .
𝑥→1−0 {𝑥} 𝑥−1
(a) – [𝐱𝐬𝐢𝐧−𝟏 𝒙 + √𝟏 − 𝒙𝟐 ] + 𝒄 𝑥 sin{𝑥} 𝑥 − 1
(b) 𝐱𝐬𝐢𝐧−𝟏 𝒙 + √𝟏 − 𝒙𝟐 + 𝒄 = Lt .
𝑥→1−0 {𝑥} 𝑥−1
= 1×1×1= 1
(c) constant
(d) none of these Since L.H. limit ≠R.H. limit
Ans. (a) ∴ limit does not exist.
𝑥 𝑛 − 𝑥 −𝑛 𝟏
𝑓(𝑥) = Lt , 0 < 𝑥 < 1, 𝑛𝜖 𝑁 16. If a = 𝐋𝐭 ∑𝒏𝒓=𝟏 (𝒓+𝟐)𝒓! 𝒂𝒏𝒅 𝒃 =
𝑛→∞ 𝑥 𝑛 + 𝑥 −𝑛 𝒏→∞
𝒆𝐬𝐢𝐧 𝒙 −𝒆𝒙
𝑥 2𝑛 −1 (𝑥 2)𝑛 −1
𝐋𝐭 , then
𝒙→𝟎 𝐬𝐢𝐧 𝒙− 𝒙
= Lt = Lt = −1
𝑛→∞ 𝑥 2𝑛 +1 𝑛→∞ (𝑥 2)𝑛 +1
(a) a= b (b) a= 2b
∴ ∫ sin−1 𝑥 . 𝑓(𝑥)𝑑𝑥 = − ∫ 1. sin−1 𝑥 𝑑𝑥 (c) 2a = b (d) a+ b = 0

Ans. (c)
= − [𝑥 sin−1 𝑥
2𝑥 1 𝑟+1 (𝑟 + 2) − 1
−∫ 𝑑𝑥] 𝑡𝑟 = = =
2√1 − 𝑥 2 (𝑟 + 2)𝑟! (𝑟 + 2)! (𝑟 + 2)!
1 1
= − [𝑥 sin−1 𝑥 + √1 − 𝑥 2 ] = −
(𝑟 + 1)! (𝑟 + 2)
+𝑐

111
Solving Mathematical Problems
𝑛
1 1 ∴ Lt (𝑠𝑒𝑐 2𝑛 𝑘! 𝜋𝑏) = 1 = 𝑎
𝑛→∞
∑ 𝑡𝑟 = −
2! (𝑛 + 2)!
𝑟=1 𝟏
18. 𝐋𝐭 𝐋𝐭 𝟐 is
𝒏→∞𝒙→𝟎 𝟐 𝟐 𝟐 𝒕𝒂𝒏 𝒙
1 1 1 (𝟏𝒄𝒐𝒕 𝒙 +𝟐𝒄𝒐𝒕 𝒙 +⋯+𝒏𝒄𝒐𝒕 𝒙 )
∴ 𝑎 = Lt [ − ]= equal to
𝑛→∞ 2 (𝑛 + 2)! 2

𝑒 𝑠𝑖𝑛𝑥 − 𝑒 𝑥 (a) 0 (b) ∞


𝑏 = Lt (c) n (d) none of these
𝑥→0 𝑠𝑖𝑛𝑥 − 𝑥
𝑒 𝑠𝑖𝑛𝑥−𝑥 − 1
= Lt 𝑒 𝑥 . ( ) Ans. (a)
𝑥→0 𝑠𝑖𝑛𝑥 − 1
2𝑥 2𝑥
= 𝑒 0. 1 = 1 Lt (1𝑐𝑜𝑡 + 2𝑐𝑜𝑡 +⋯
𝑥→0
2 𝑡𝑎𝑛2 𝑥
∴ 2a = b + 𝑛𝑐𝑜𝑡 𝑥 ) [∞0 𝑓𝑜𝑟𝑚]

17. If 𝐋𝐭 (√𝒙𝟐 − 𝒙 + 𝟏 − 𝒂𝒙 − 𝒃) = 1
𝒙→∞ = Lt (1𝑦 + 2𝑦 + ⋯ + 𝑛𝑦 )𝑦 [𝑤ℎ𝑒𝑟𝑒 𝑦
𝟎, 𝒕𝒉𝒆𝒏 𝒇𝒐𝒓 𝒌 ≥ 𝟐, 𝐋𝐭 𝒔𝒆𝒄𝟐𝒏 (𝒌! 𝝅𝒃)= 𝑦→∞
𝒏→∞ = 𝑐𝑜𝑡 2 𝑥 ]
(a) a (b) –a
(c) 2a (d) b 1 𝑦 2 𝑦 𝑛−1 𝑦
= Lt 𝑛 [( ) + ( ) + ⋯ + ( )
𝑦→∞ 𝑛 𝑛 𝑛
Ans. (a)
1
𝑦
Lt (√𝑥 2 − 𝑥 + 1 − 𝑎𝑥 − 𝑏) + 1) ]
𝑥→∞
= 0[ℎ𝑒𝑟𝑒 𝑎 > 0 𝑓𝑜𝑟 𝑖𝑓 𝑎
≤ 0, 𝑡ℎ𝑒𝑛 𝑙𝑖𝑚𝑖𝑡 = ∞] 1 1 𝑦 2 𝑦 𝑛−1 𝑦
= 𝑛. Lt 𝑦 [ ) + ( ) + ⋯ + ( ) ]
𝑦→∞ 𝑒 𝑛 𝑛 𝑛
𝑥 2 − 𝑥 + 1 − (𝑎𝑥 + 𝑏)2
⟹ Lt = 𝑛. 𝑒 0 = 𝑛
𝑛→∞ √𝑥 2 − 𝑥 + 1 + 𝑎𝑥 + 𝑏
(1 − 𝑎2 )𝑥 2 − (1 + 2𝑎𝑏)𝑥 + 1 − 𝑏 1
⟹ Lt ∴ 𝑅𝑒𝑞𝑑. 𝑙𝑖𝑚𝑖𝑡 = Lt =0
𝑛→∞ √𝑥 2 − 𝑥 + 1 + 𝑎𝑥 + 𝑏 𝑛→∞ 𝑛
=0
𝒍𝒐𝒈𝒙𝒏 −[𝒙]
19. 𝐋𝐭 , 𝒏 ∊ 𝑵, where [x] denotes
[𝒙]
This is possible only when 1- 𝑎2 = 0 and 1+ 𝒙→∞

2ab= 0 the integral part of x, is equal to

(a) 0 (b) 1
∴ a= 1(∵ a > 0) and b= - ½
(c) -1 (d) ∞
Now k! 𝜋 b= k! 𝜋 (- ½ )= an integer multiple
Ans. (c)
of 𝜋 as k ≥ 2
𝑙𝑜𝑔𝑥 𝑛 − [𝑥] 𝑛 log 𝑥 − [𝑥]
∴𝑠𝑒𝑐 2 𝑘! 𝜋𝑏 = 1 Lt = Lt
𝑥→∞ [𝑥] 𝑥→∞ [𝒙]

112
Solving Mathematical Problems

𝑙𝑜𝑔𝑥 [𝑥] 𝛼 𝛼 2 𝛼 𝑛
𝑛 𝑥 − 𝑥 = 𝛽 𝑛 [1 + + ( ) + ⋯+ ( ) ]
= Lt … … … . (1) 𝛽 𝛽 𝛽
𝑥→∞ [𝑥]
𝑥 𝛼 𝑛+1
( ) −1
𝛽
1 = 𝛽𝑛 𝛼
𝑙𝑜𝑔𝑥 ∞ 𝑥 −1
𝑏𝑢𝑡 Lt [∞ from] = Lt = 0 and 𝛽
𝑥→∞ 𝒙 𝑥→∞ 1
[𝑥]
Lt 1 4𝑛+1 − 1
𝑥→∞ 𝑥
= ( )
2𝑛 3. 2𝑛
𝑥 − [𝑥] [𝑥]
= Lt = Lt (1 − ) = 1 − 0
𝑥→∞ 𝑥 𝑥→∞ 𝑥 21. If [x]denotes the integral part of x,
=1 then

0−1 𝐥𝐨𝐠 𝒆 [𝒙]


∴ 𝑓𝑟𝑜𝑚, 𝑟𝑒𝑞𝑑. 𝑙𝑖𝑚𝑖𝑡 = = −1 𝐋𝐭 =
𝒙→∞ 𝒙
1
(a)-1 (b) 1
20. If 𝛼 = min. of (𝒙𝟐 + 𝟐𝒙 + 𝟑) and 𝛽=
𝟏 (c) 0 (d) none of these
𝐋𝐭 ∑𝒏𝒓=𝟏 (𝒓+𝟐)𝒓! , 𝒕𝒉𝒆𝒏 ∑𝒏𝒓=𝟎 𝜶𝒓 𝜷𝒏−𝒓 =
𝒏→∞
Ans. (c)
𝟐𝒏+𝟏 +𝟏 𝟐𝒏+𝟏 −𝟏
(a) (b)
𝟑.𝟐𝒏 𝟑.𝟐𝒏 𝑥 − 1 < [𝑥] ≤ 𝑥
𝟒𝒏+𝟏 −𝟏
(c) (d) none of these
𝟑.𝟐𝒏 ⟹ 𝑙𝑜𝑔𝑒 (𝑥 − 1) < 𝑙𝑜𝑔𝑒 [𝑥] ≤ 𝑙𝑜𝑔𝑒 𝑥
Ans. (c) log 𝑒 (𝑥 − 1) log 𝑒 [𝑥]
⟹ Lt ≤ Lt
4. 1. 3 − 22
𝑥→∞ 𝑥 𝑥→∞ 𝑥
𝛼 = 𝑚𝑖𝑛. 𝑜𝑓 𝑥 + 2𝑥 + 3 = 2 log 𝑒 𝑥
4 ≤ Lt
𝑥→∞ 𝑥
=2
𝑛 𝑛
1 1
1 𝑟+1 𝑥 − 1 log 𝑒 [𝑥] 𝑥
𝛽 = Lt ∑ = Lt ∑ ⟹ Lt ≤ Lt ≤
𝑛→∞ (𝑟 + 2)𝑟! 𝑛→∞ (𝑟 + 2)! 𝑥→∞ 1 𝑥→∞ 𝑥 1
𝑟=1 𝑟=1
𝑛
(𝑟 + 2) − 1 log 𝑒 [𝑥] log 𝑒 [𝑥]
⟹ 0 ≤ Lt ≤ 0 ⟹ Lt
= Lt ∑ 𝑥→∞ 𝑥 𝑥→∞ 𝑥
𝑛→∞ (𝑟 + 2)!
𝑟=1 =0
𝑛 𝒙
1 1 ∫𝟏 |𝒕−𝟏|𝒅𝒕
= Lt ∑ ( − ) 22. 𝐋𝐭 =
𝑛→∞ (𝑟 + 1)! (𝑟 + 2)! 𝒙→𝟏+𝟎 𝐬𝐢𝐧(𝒙−𝟏)
𝑟=1
1 1 1 (a) -1 (b) 0
= Lt [ − ]=
𝑛→∞ 2 (𝑛 + 2)! 2 (c) 1 (d) none of these
𝑛
Ans. (b)
𝑛𝑜𝑤 ∑ 𝛼 𝑟 𝛽 𝑛−𝑟 = 𝛽 𝑛 + 𝛼𝛽 𝑛−1 + ⋯ + 𝛼 𝑛
𝑟=0

113
Solving Mathematical Problems
𝑥
∫1 (𝑡−1)𝑑𝑡 When x ⟶ 𝑎𝑚 + 0, 𝛼𝑟 = 1, 𝑟 =
Reqd. limit = Lt =
𝑥→1+0 sin(𝑥−1) 1, 2, 3, … , 𝑚 = −1, 𝑟 = 𝑚 + 1, 𝑚 + 2, … , 𝑛
(𝑡−1)2 𝑥 (𝑥−1)2
[ ]1
2
Lt = Lt 2
= ∴ Lt (𝛼1 , 𝛼2 , … , 𝛼𝑛 ) = 1𝑚 (−1)𝑛−𝑚
𝑥→1+0 sin(𝑥−1) 𝑥→1+0 sin(𝑥−1) 𝑥→𝑎𝑚 +0
𝑥−1 𝑥−1
Lt .( )=1×0=0 = (−1)𝑛−𝑚
𝑥→1+0 sin(𝑥−1) 2

𝟒 𝑤ℎ𝑒𝑛 𝑥 ⟶ 𝑎𝑚 − 0,
23. 𝐋𝐭 ∏𝒏𝒓=𝒏 𝐥𝐨𝐠 𝒓−𝟏 (𝒓) =
𝒏→∞
𝛼𝑟 = 1, 𝑓𝑜𝑟 𝑟 = 1, 2, … , 𝑚 − 1
(a) 0 (b) ∞ = −1, 𝑓𝑜𝑟 𝑟
(c) k (d) k! = 𝑚, 𝑚 + 1, … , 𝑛
Ans. (c) ∴ Lt (𝛼1 , 𝛼2 , … , 𝛼𝑛 )
𝑥→𝑎𝑚 −0
𝑛𝑘 𝑛𝑘 = 1𝑚−1 (−1)𝑛−𝑚+1
𝑙𝑜𝑔𝑟
Lt ∏ log 𝑟−1 𝑟 = Lt ∏ = (−1)𝑛−𝑚+1
𝑥→∞ 𝑛→∞ log(𝑟 − 1)
𝑟=𝑛 𝑟=𝑛
log 𝑛 log(𝑛 + 1) log(𝑛 + 2) Since L.H. limit ≠ R.H. limit
= Lt . .
𝑛→∞ log(𝑛 − 1) log 𝑛 log(𝑛 + 1)
Lt (𝛼1 , 𝛼2 , … , 𝛼𝑛 ) 𝑑𝑜𝑒𝑠 𝑛𝑜𝑡 𝑒𝑥𝑖𝑠𝑡
𝑥→𝑎𝑚
log 𝑛𝑘
… 25. If 𝑺𝒏 = ∑𝒏𝒓=𝟏 𝒂𝒓 𝒂𝒏𝒅 𝐋𝐭 𝒂𝒏 =
log(𝑛𝑘 − 1) 𝒏→∞
𝑺𝒏+𝟏 −𝑺𝒏
𝒂, 𝒕𝒉𝒆𝒏 𝐋𝐭
log 𝑛𝑘 𝒏→∞ √∑𝒏 𝒓
= Lt 𝒓=𝟏
𝑛→∞ log(𝑛 − 1)
𝑘 log 𝑛 ∞ (a) a (b) 2a
= Lt [ 𝑓𝑟𝑜𝑚]
𝑛→∞ log(𝑛 − 1) ∞ (c) –a (d) 0

1 Ans. (d)
𝑘. 𝑛 1
= Lt = Lt 𝑘 (1 − ) = 𝑘
𝑛→∞ 1 𝑛→∞ 𝑛 𝑆𝑛+1 − 𝑆𝑛 𝑎𝑛+1
𝑛−1 Lt = Lt
𝑛→∞ √∑𝑛 𝑟 𝑛→∞
𝑟=1 √𝑛(𝑛 + 1)
2
𝑎𝑛+1
𝒙−𝒂
Lt 𝑛
24. If 𝜶𝒓 = |𝒙−𝒂𝒓| , 𝒓 = 𝟏, 𝟐, 𝟑, . . , 𝒏 & 𝜶𝟏 < = 𝑛→∞ =0 [
𝒓
1
𝜶𝟐 < ⋯ < 𝜶𝒏 . Then 𝐋𝐭 (𝜶𝟏 𝜶𝟐 … 𝜶𝒏 ),
𝒙→𝜶𝒎 √1 (1 + 𝑛)
2
𝟏 ≤ 𝒎 ≤ 𝒏 is equal to ∵ Lt 𝑎𝑛+1 = 𝑎]
𝑛→∞
𝒎−𝟏 𝒎
(a) (−𝟏) (b) (−𝟏)
(c) (−𝟏)𝒎+𝟏 (d) none

Ans. (d)

114
Solving Mathematical Problems

𝒏𝒌 𝒔𝒊𝒏𝟐 𝒏! 28. If [x] denotes the integral part of x,


26. If 𝐋𝐭 = 𝟎 for
𝒏→∞ 𝒏+𝟏 [𝒙]+[𝟐𝒙]+[𝟑𝒙]+⋯+[𝒏𝒙]
then 𝐋𝐭 is
𝒏→∞ 𝒏𝟐
(a) all k (b) 0≤k < 1
(c) k= 1 (d) for k > 1 (a) x (b) x/2
(c) x/3 (d) independent of x
Ans. (b)
Ans. (b)
𝑛𝑘 𝑠𝑖𝑛2 𝑛! 𝑛𝑘
Lt = Lt . (𝑠𝑖𝑛2 𝑛!) nx – 1 < [nx] ≤ nx, for n= 1, 2, …, n
𝑛→∞ 𝑛 + 1 𝑛→∞ 𝑛 + 1

Lt 𝑠𝑖𝑛2 𝑛! does not exist but if ⟹ ∑𝑛𝑛=1(𝑛𝑥 − 1) < ∑𝑛𝑛=1[𝑛𝑥] ≤ ∑𝑛𝑛=1 𝑛𝑥


𝑛→∞
𝑛
𝑛𝑘 𝑛𝑘 𝑠𝑖𝑛2 𝑛! 𝑛(𝑛 + 1) 𝑛(𝑛 + 1)
Lt = 0, 𝑡ℎ𝑒𝑛 Lt =0 ⟹ 𝑥 − 𝑛 < ∑[𝑛𝑥] ≤
𝑛→∞ 𝑛 + 1 𝑛→∞ 𝑛 + 1 2 2
𝑛=1

𝑛𝑘 𝑛(𝑛 + 1) 1 ∑𝑛𝑛=1[𝑛𝑥]
𝑏𝑢𝑡 Lt =0⟺0 ≤𝑘≤1 ⟹ 𝑥 − <
𝑛→∞ 𝑛 + 1 2𝑛2 𝑛 𝑛2
𝑛(𝑛 + 1)
≤ 𝑥
2𝑛2
{𝒙}+{𝟐𝒙}+{𝟑𝒙}+⋯+{𝒏𝒙}
27. 𝐋𝐭 where {x} = x- 𝑛(𝑛 + 1) 1
𝒏→∞ 𝒏𝟐
⟹ Lt ( 2
𝑥− )
[x] denotes the fractional part of x, 𝑛→∞ 2𝑛 𝑛
𝑛(𝑛 + 1) 𝑥
(a) 1 (b) 0 ≤ 𝑙 Lt 𝑥 ⟹ ≤𝑙
𝑛→∞ 2𝑛2 2
(c) ½ (d) none of these 𝑥 𝑥
≤ ⟹𝑙=
2 2
Ans. (b)

0≤ {nx} < 1, for n= 1, 2, 3,…, n


29. If [x] denotes the integral part of x,
0 ∑𝑛
𝑛=1{𝑛𝑥} 1
⟹ 0≤ ∑𝑛𝑛=1{𝑛𝑥} < 𝑛 ⟹ 𝑛2 ≤ <𝑛 and 𝒂𝒏 = ∑𝒏−𝟏
𝑛2 𝒓=𝟎 [𝒙 +
𝒓 𝒂𝟏 +𝒂𝟐 +⋯+𝒂𝒏
0 ∑𝑛𝑛=1{𝑛𝑥} 1 𝒏
] , 𝒕𝒉𝒆𝒏 𝐋𝐭 𝒏𝟐
=
Lt 2 ≤ Lt ≤ 2
< Lt 𝒏→∞
𝑛→∞ 𝑛 𝑛→∝ 𝑛 𝑛→∞ 𝑛
𝒙 𝒙
(a) 𝟐 (b) 𝟑
∑𝑛𝑛=1{𝑛𝑥}
⟹0≤ Lt ≤0 (c) x (d) none of these
𝑛→∞ 𝑛2
{𝑥} + {2𝑥} + ⋯ + {𝑛𝑥} Ans. (a)
⟹ Lt =0
𝑛→∞ 𝑛2 1 2 𝑛−1
[𝑥] + [𝑥 + ] + [𝑥 + ] + ⋯ + [𝑥 + ]
𝑛 𝑛 𝑛
= [𝑛𝑥]

∴ 𝑎𝑛 = [𝑛𝑥]

115
Solving Mathematical Problems

𝑎1 + 𝑎2 + ⋯ + 𝑎𝑛 ISI OBJECTIVE SAMPLE PAPER


∴ Lt
𝑛→∞ 𝑛2 WITH SOLUTIONS
[𝑥] + [2𝑥] + [3𝑥] + ⋯ + [𝑛𝑥] 𝑥
= Lt =
𝑛→∞ 𝑛2 2 SET – 15

𝟏 𝜽 𝟏 𝜽
30. 𝐋𝐭 (𝐭𝐚𝐧 𝜽 + 𝟐 𝒕𝒂𝒏 𝟐 + 𝟐𝟐 𝒕𝒂𝒏. 𝟐𝟐 +
𝒏→∞ 𝒙𝟑 𝒔𝒊𝒏𝒙 𝒄𝒐𝒔𝒙
𝟏 𝜽
⋯ + 𝟐𝒏 𝒕𝒂𝒏 𝟐𝒏 )= 1. Let f(x) = 𝒅𝒆𝒕 ( 𝟔 −𝟏 𝟎 )
𝟐
𝒑 𝒑 𝒑𝟑
𝟏 𝟏 𝒅𝟑
(a) 𝜽 (b) 𝜽 − 𝟐𝒄𝒐𝒕𝟐𝜽 where p is a constant. Then 𝒅𝒙𝟑 {𝒇(𝒙)} at x
(c) 2 cot2𝜃 (d) none = 0 is
Ans. (b) tan 𝜃 = cot 𝜃 − 2 cot 2𝜃 (a) p (b) p+ 𝒑𝟐
1 𝜃 1 𝜃 (c) p + 𝒑𝟑 (d) independent of p
∴ 2 tan 2 = 2 cot 2 − 𝑐𝑜𝑡𝜃
Ans. (d)
… … …
6 − cos 𝑥 sin 𝑥
1 𝜃 1 𝜃 1 𝜃 f‴(x) = |6 −1 0 |
tan = cot − 𝑐𝑜𝑡
2𝑛 2𝑛 2𝑛 2𝑛 2𝑛−1 2𝑛−1 𝑝 𝑝2 𝑝3

∴𝑆𝑛 =
1
cot
𝜃
− 2 cot 2𝜃 6 −1 0
2𝑛 2𝑛 ∴ f‴(0) = |6 −1 0|=0
𝑝 𝑝2 𝑝3

2. The number of solutions of the


equations 𝒙𝟐 − 𝒙𝟑 = 𝟏
∴ Reqd. limit = Lt 𝑆𝑛 = −𝒙𝟏 + 𝟐𝒙𝟑 = 𝟐
𝑛→∞
𝒙𝟏 − 𝟐𝒙𝟐 = 𝟑 is

(a) zero (b) one


1 (c) two (d) infinite
Lt − 2𝑐𝑜𝑡2𝜃
𝑛→∞ 𝜃
2𝑛 tan 2𝑛 𝜃 Ans. (b)
𝜃 . 2𝑛
( 2𝑛 ) 0 1 −1
1 Hence 𝛥 = |−1 0 2|
= − 𝑐𝑜𝑡2𝜃 1 −2 0
𝜃
= -1 (-2)-1 (2)=0

Hence given system of equations is


consistent and since have no two equations

116
Solving Mathematical Problems

are identical, therefore number of solutions X= 0, we get 𝑎1 = 4𝑎


is one.
Differentiating again and putting x= 0
3. If p(x), q(x), r(x) be three quadratic
expressions in x and f(x) = We get 𝑎2 = 6𝑎2 + 4𝑏
𝒑(𝒙) 𝒒(𝒙) 𝒓(𝒙)
Clearly, 𝛥= −(𝑎0 3 + 𝑎1 3 + 𝑎2 3 − 3𝑎0 𝑎1 𝑎2 )
| 𝒑′ (𝒙) 𝒒′ (𝒙) 𝒓′ (𝒙) | , where dash
𝒑″ (𝒙) 𝒒″ (𝒙) 𝒓″ (𝒙) ∵ 𝑎0 + 𝑎1 + 𝑎2 ≠ 0 ∴ 𝑎0 = 𝑎1 = 𝑎2
denotes the derivative with respect to x, 1 5
then ∴ 1= 4a= 6𝑎2 + 4𝑏 ⟹ 𝑎 = 4 , 𝑏 = 32

(a) f(x) = f(0) (b) f(x) = 2f(0) 5. If 𝛼, 𝛽, 𝛾 ∊ R, then


(c) f(x) = -f(0) (d) none of these (𝒆𝒊𝜶 + 𝒆−𝒊𝜶 )𝟐 (𝒆𝒊𝜶 − 𝒆−𝒊𝜶 )𝟐 𝟏
|(𝒆𝒊𝜷 + 𝒆−𝒊𝜷 )𝟐 (𝒆𝒊𝜷 − 𝒆−𝒊𝜷 )𝟐 𝟏| equals
Ans. (b) Let p(x) = 𝑎𝑥 2 + 𝑏𝑥 + 𝑐, 𝑞(𝑥) =
(𝒆𝒊𝜸 + 𝒆−𝒊𝜸 )𝟐 (𝒆𝒊𝜸 − 𝒆−𝒊𝜸 )𝟐 𝟏
𝑚𝑥 2 + 𝑛𝑥 + 𝑘, 𝑟(𝑥) = 𝑢𝑥 2 + 𝑣𝑥 + 𝑤
(a) 𝒆𝒊(𝜶+𝜷+𝜸) (b) 𝒆−𝒊(𝜶+𝜷+𝜸)
f(x) =
(c) 𝒆𝒊𝜶𝜷𝜸 (d) none of these
2
𝑎𝑥 2 + 𝑏𝑥 + 𝑐 𝑚𝑥 2 𝑛𝑥 + 𝑘 𝑢𝑥 2 + 𝑣𝑥 + 𝑤 Ans. (d)
| 2𝑎𝑥 + 𝑏 2𝑚𝑥 + 𝑛 2𝑢𝑥 + 𝑣 | =
𝑎 𝑚 𝑢 Applying
𝑎 𝑘 𝑤 2
2 |𝑏 𝑛 𝑣 | 𝑅1→𝑅 𝑅→𝑅
1 −𝑥𝑅2 +𝑥 𝑅3
−2𝑥𝑅 𝐶1 → 𝐶1 − 𝐶2 , we have 𝛥= 0
2 2 3
𝑎 𝑚 𝑢
6. If 𝒇𝒏 (𝒙) = (𝒏𝒙 + 𝒏−𝒙 )𝟐 , 𝒈𝒏 (𝒙) =
= 2f(0)
𝒇𝟐 (𝒙) 𝒈𝟐 (𝒙) 𝟏
4. If (𝟏 + 𝒂𝒙 + 𝒃𝒙𝟐 )𝟒 = 𝒂𝟎 + 𝒂𝟏 𝒙 + (𝒏𝒙 − 𝒏−𝒙 )𝟐 then |𝒇𝟑 (𝒚) 𝒈𝟑 (𝒚) 𝟏| is
𝒇𝟓 (𝒛) 𝒈𝟓 (𝒛) 𝟏
𝒂𝟐 𝒙𝟐 + ⋯ + 𝒂𝟖 𝒙𝟖 , where a,
b, 𝒂𝟎 , 𝒂𝟏 , … , 𝒂𝟖 ∊ 𝑹 such that 𝒂𝟎 + 𝒂𝟏 + (a) equal to zero (b) independent of
𝒂𝟎 𝒂𝟏 𝒂𝟐 𝟏
𝒂𝟐 ≠ 𝟎 𝒂𝒏𝒅 |𝒂𝟏 𝒂𝟐 𝒂𝟎 | = 0 then x, y, z (c) 𝟐𝒙 𝟑𝒚 𝟓𝒛 (d) 𝟐𝒙 𝟑𝒚 𝟓𝒛
𝒂𝟐 𝒂𝟎 𝒂𝟏
Ans.(a)
(a) a= 3/4, b= 5/8 (b) a= 1/4, b=
5/32 (c) a= 1, b= 2/3 (d) (2𝑥 + 2−𝑥 )2 (2𝑥 − 2−𝑥 )2 1
none 𝛥 = |(3𝑦 + 3−𝑦 )2 (3𝑦 − 3−𝑦 )2 1| =
(5𝑧 + 5−𝑧 )2 (5𝑧 − 5−𝑧 )2 1
Ans. (b) 4 (2𝑥 − 2−𝑥 )2 1
|4 (3𝑦 − 3−𝑦 )2 1| = 0
Putting x= 0, 𝑎𝑛 = 1
4 (5𝑧 − 5−𝑧 )2 1
Differentiating both sides and putting

117
Solving Mathematical Problems

7. If ∑𝒏𝒏=𝟏 𝜶𝒏 = 𝒑𝒏𝟐 + 𝒒𝒏 + 𝒓 and 𝑿 𝒀 𝒁


𝑺 𝒕𝟏
𝜶𝟏 , 𝜶𝟐 , 𝜶𝟑 ∊ {𝟏, 𝟐, 𝟑, … , 𝟗} and and |𝑿𝟏 𝒀𝟏 𝒁𝟏 | + | 𝟏 | = 𝒙𝒏 , then
𝑺𝟐 𝒕𝟐
𝑿𝟐 𝒀𝟐 𝒁𝟐
25𝜶𝟏 , 𝟑𝟕𝜶𝟐 , 𝟒𝟗𝜶𝟑 be three digit numbers,
𝜶𝟏 𝜶𝟐 𝜶𝟑 n=
then | 𝟓 𝟕 𝟗 |=
𝟐𝟓𝜶𝟏 𝟑𝟕𝜶𝟐 𝟒𝟗𝜶𝟑 (a) 1 (b) 2
(c) 3 (d) none of
(a) 𝜶𝟏 + 𝜶𝟐 + 𝜶𝟑 (b) 𝜶𝟏 − these
𝜶𝟐 + 𝜶𝟑 (c) 7
Ans. (c)
(d) 0
𝛥=
Ans. (d)
𝑋 𝑆𝑋 𝑡𝑋
𝛼𝑛 = 2𝑝𝑛 + 𝑞 |𝑋1 𝑆𝑋1 + 𝑆1 𝑋 𝑡𝑋1 + 𝑡1 𝑋 |
𝑋2 𝑆𝑋2 + 2𝑆1 𝑋1 + 𝑆2 𝑋 𝑡𝑋2 + 2𝑡1 𝑋1 + 𝑡2 𝑋
𝛼1 𝛼2 𝛼3
𝛥= | 5 7 9 |= =
25𝛼1 37𝛼2 49𝛼3 𝑋 0 0
𝛼1 𝛼2 𝛼3 𝑋
| 1 𝑆1 𝑋 𝑡1 𝑋 | [𝐶𝐶3→→𝐶𝐶3 −𝑡𝐶1
]
2 2 −𝑆𝐶1
| 5 7 9 | [𝑅3 → 𝑅3 − 𝑅1 − 10𝑅2 ] 𝑋2 2𝑆1 𝑋1 + 𝑆2 𝑋 2𝑡1 𝑋1 + 𝑡2 𝑋
200 300 400
𝑆1 𝑡1
𝛼1 2𝑝 2𝑝 = 𝑋2 | |=
𝐶 →𝐶 −𝐶 2𝑆1 𝑋1 + 𝑆2 𝑋 2𝑡1 𝑋1 + 𝑡2 𝑋
=| 5 2 2 | = 0 [𝐶23→ 𝐶32−𝐶21] 𝑆 𝑡
200 100 100 𝑋 3 | 1 1 | [𝑅3 → 𝑅2 − 2𝑋1 𝑅1 ]
𝑆2 𝑡2
𝒑(𝒙) 𝒒(𝒙) 𝒓(𝒙)
∴ n= 3
8. If f(x) = | 𝒑(𝜶) 𝒒(𝜶) 𝒓(𝜶) |, where
𝒑 (𝜶) 𝒒′ (𝜶) 𝒓′ (𝜶)

10. If f(x) = cos[𝜋]x+ cos[𝜋x], where [y]is
p(x), q(x), r(x) are polynomials of degree the greatest integer function of y, then
3, 4, 5 respectively, then (𝒙 − 𝜶)𝒎 is a 𝝅
f( 𝟐 )=
factor of F(x), where
(a) 0 (b) cos3
(a) m ≥ 1 (b) m ≥ 2
(c) cos4 (d) none of these
(c) m= 2 (d) m ≥ 3
Ans. (c)
Ans. (b) F(𝛼) = 0, F’(𝛼) = 0
f(x) = cos [𝜋]x + cos [𝜋x]
∴ (𝑥 − 𝛼)2 will necessarily be a factor or
F(x) ∴ m≥ 2 𝜋 𝜋 π2 3𝜋
∴ 𝑓 ( 2 ) = 𝑐𝑜𝑠[π] 2 + cos [ 2 ] = cos +
2
9. If Y= SX, Z= tX, all the variables being cos 4 = 0 + cos 4 = cos 4
differentiable functions of x, and lower
suffices denotes the derivative w.r.t. to x

118
Solving Mathematical Problems

11. Let g(x) = (a) |z|< 5 (b) |z| > 6


𝒇(𝒙 + 𝜶) 𝒇(𝒙 + 𝟐𝜶) 𝒇(𝒙 + 𝟑𝜶) (c) 1 <|z|< 3 (d) |z|=5
| 𝒇(𝒙) 𝒇(𝟐𝜶) 𝒇(𝟑𝜶) |,

𝒇 (𝜶) 𝒇′(𝟐𝜶) 𝒇′(𝟑𝜶) Ans. (a)
𝒈(𝒙)
where 𝛼 is a constant, then 𝐥𝐭 𝒙⟶𝟎 = |𝑧|2 +2|𝑧|+6
𝒙 Since log 1 (2|𝑧|2 −2|𝑧|+1) < 0
2

(a) 0 (b) 1 |𝑧|2 +2|𝑧|+6


(c) -1 (d) none of these ∴ 2|𝑧|2 −2|𝑧|+1 > 1

Ans. (a) ⟹ |𝑧|2 − 2|𝑧| − 5 < 0


𝑔(𝑥) 0
Lt [0 𝑓𝑟𝑜𝑚 𝑎𝑠 𝑔(0) = 0] ⟹ (|z|- 5)(|z| +1) < 0 ⟹ |z| < 5
𝑥→0 𝑥

𝑔′(𝑥) 14. If |z|= maximum {|z+2|, |z-2|}, then


= Lt = 𝑔′ (0) = 0
𝑥→0 1
(a) |z-z̅|= ½ (b) |z +z̅|=2
12. Choose any 9 distinct integers. These 9 (c) |z +z̅|= ½ (d) |z-z̅|= 2
integers can be arranged to from 9!
determinants each of order 3. The sum of Ans. (b)
these 9! determinants is |z| =|z +2|
(a) 0 (b) > 0 ⟹ zz̅ = (z +2)(z̅+ 2)
(c) < 0 (d) 9!
⟹ z+ z̅= -2 ⟹ |z +z̅|= 2
Ans. (a) Let the nine distinct digits be
𝑎1 , 𝑎2 , … , 𝑎9 |z|= | z -2| ⟹ zz̅ = (z -2)(z̅- 2)

Let 𝛥1 = one of the 9! Determinates then ⟹ z+ z̅= 2 ⟹ |z +z̅|= 2


there exists
15. Let a be a complex number such that
𝛼 Determinant 𝛥2 = adding 9! Determinants |a|< 1 and 𝒛𝟏 , 𝒛𝟐 , 𝒛𝟑 , … be the vertices of a
obtained by interchanging 𝑅1 𝑎𝑛𝑑 𝑅2 𝑖𝑛 𝛥1 polygon ∋ 𝒛𝒌 = 𝟏 + 𝒂 + 𝒂𝟐 + ⋯ + 𝒂𝒌−𝟏
such that for all k= 1, 2, 3, … then 𝒛𝟏 , 𝒛𝟐 , … lie
within the circle
𝛥1 + 𝛥2 = 0
𝟏 𝟏
Thus 9! /2 pairs of determinants will be (a) |𝒛 − 𝟏−𝒂| = |𝒂−𝟏|
there such that sum of each pair is zero.
𝟏 𝟏
(b) |𝒛 + 𝒂+𝟏| = |𝒂+𝟏|
∴ Required sum = 0
𝟏
13. If a complex number z satisfies (c) |𝒛 − 𝟏−𝒂| = |𝒂 − 𝟏|
|𝒛|𝟐 +𝟐|𝒛|+𝟔
𝐥𝐨𝐠 𝟏 (𝟐|𝒛|𝟐 −𝟐|𝒛|+𝟏) < 0, then locus of point 𝟏
𝟐 (d) |𝒛 + 𝒂+𝟏| = |𝒂 + 𝟏|
represented by z is

119
Solving Mathematical Problems

Ans. (a) Ans. (b)

Given 𝑧𝑘 = 1 + 𝑎 + 𝑎2 + ⋯ + 𝑎𝑘−1 =
1−𝑎𝑘 1 1 1 1
Required number= 5!(|2̲̲ − |3̲ + |4̲̲ − |5)
1−𝑎 ̲ ̲
̲ ̲ ̲̲
̲
̲ ̲ ̲
̲ ̲
̲
1 𝑎𝑘
⟹ 𝑧𝑘 − 1−𝑎 = − 1−𝑎 1 1 1 1
= 120 (2 − 6 + 24 − 120)
1 |𝑎| 𝑘 1
⟹ |𝑧𝑘 − 1−𝑎| = |1−𝑎| < |1−𝑎| [∵ |𝑎| < 1]
= 60 − 20 + 5 − 1 = 44
⟹ 𝑧𝑘 lies within the circle 18. Number of divisors of 𝟐𝟐 . 𝟑𝟑 . 𝟓𝟑 . 𝟕𝟓 of
1 1 the form 4n+1, n ∊N is
|𝑧 − 1−𝑎| = |1−𝑎|
(a) 46 (b) 47
16. The number of ordered triplets of (c) 96 (d) none of these
positive integers which satisfy the
Ans. (b) 4n + 1 is an odd number
inequality 20 ≤ 𝒙 + 𝒚 + 𝒛 ≤ 𝟓𝟎 is
Divisor will be of the from 4n+1 if and only
(a) (𝟓𝟎
𝟑
) − (𝟏𝟗
𝟐
) (b) (𝟓𝟎
𝟐
)−
if it is the product of
(𝟏𝟗
𝟑
) (c) (𝟓𝟎
𝟑
) − (𝟐𝟎
𝟑
)
(d) none (i) All numbers of the from 4k +1,
or
Ans. (a) 20 ≤ 𝑥 + 𝑦 + 𝑧 ≤ (ii) 2 or 4 numbers of the from 4n
50, 𝑤ℎ𝑒𝑟𝑒 𝑥, 𝑦, 𝑧, ∊ 𝑁 +3

⟹ 17 ≤ 𝑎 + 𝑏 + 𝑐 ≤ 47, ∴ Number of divisors of

Where a = x - 1, b = y - 1, c = z - 1 N= 22 . 33 . 53 . 73 , which are of the from 4n


+1 excluding 1
∴ Reqd. number = (𝟏𝟗
𝟏𝟕
) + (𝟐𝟎
𝟏𝟖
) + ⋯ + (𝟒𝟗
𝟒𝟕
)
= number of terms in the product
= (𝟏𝟗
𝟐
) + (𝟐𝟎
𝟐
) + ⋯+ (𝟒𝟗
𝟐
)
(1+32 )(1 + 5 + 52 + 53 )(1 + 72 + 74 ) +
= (𝟓𝟎
𝟑
) − (𝟏𝟗
𝟐
) number of terms in the product

17. Let A= {𝒙𝟏 , 𝒙𝟐 , 𝒙𝟑 , 𝒙𝟒 , 𝒙𝟓 }. Then the (3 + 33 )(7 + 73 + 75 )(1 + 5 + 52 + 53 ) −


number B= {𝒚𝟏 , 𝒚𝟐 , 𝒚𝟑 , 𝒚𝟒 , 𝒚𝟓 }. Then the 1
number of one-one mappings, from A to
= 2 × 4 × 3 + 2 × 3 × 4 − 1 = 47
B such that f(𝒙)𝒊 ≠ 𝒚𝒊 , 𝒊 = 𝟏, 𝟐, … , 𝟓 is
19. If m = number of distinct rational
(a) 40 (b) 44 𝒑
numbers 𝒒 ∊ (𝟎, 𝟏) such that p, q ∊ {1, 2,
(c) 6 (d) 24

120
Solving Mathematical Problems

3, 4, 5} and n = number of mappings from Ans. (c)


{1, 2, 3} onto {1, 2}, then m –n is
Clearly f(x) = g(x)
(a) 1 (b) -1
But here sec x and tan x should be defined
(c) 0 (d) none of these
𝜋
Ans. (c) ∴𝑥 ≠ (2𝑛 + 1) 2 , 𝑛 ∈ 𝐼

𝜋
n= 23 − 2 = 6 ∴ x ∊ R – {x : x=(2𝑛 + 1) 2 , 𝑛 ∈ 𝐼 }
2 1
𝑚 = 4 + 3 + 2 + 1 − 1 = 9 (𝑎𝑠 = 2) 22. Consider a set P consisting of n
4
∴m–n=3 elements. A subset ‘A’ of P is chosen
thereafter set P is reconstructed by
20. There are n different books each replacing the elements of A and finally
having m copies. If the total number of another subset ‘B’ of P is chosen. The
ways of making a selection from them is number of ways of choosing ‘A’ and ‘B’
255 and m-n+1 = 0. Then distance of such that (A∪B) is a proper subset of P is
point (m, n) from the origin is
(a) 𝟒𝒏 (b) 𝟒𝒏 − 𝟑𝒏
(a) 3 (b) 4 (c) 𝟒𝒏 − 𝟐𝒏 (d) none of these
(c) 5 (d) none of these
Ans. (b)
Ans. ( c) Total number of selections =
(𝑚 + 1)𝑛 − 1 (number of ways in which no For any element 𝑎𝑖 of P, there are 4
book is selected = 1) possibilities:

Given, (𝑚 + 1)𝑛 − 1 = 255 ⟹ (𝑚 + (i) 𝑎𝑖 ∊ 𝐴 𝑎𝑛𝑑 𝑎𝑖 ∊ 𝐵


1)𝑛 = 256 (ii) 𝑎𝑖 ∊ 𝐴 𝑏𝑢𝑡 𝑎𝑖 ∉ 𝐵
(iii) 𝑎𝑖 ∉ 𝐴 𝑏𝑢𝑡 𝑎𝑖 ∊ 𝐵
∴ 𝑛𝑛 = 256 = 44 [∵ m+ 1= n] (iv) 𝑎𝑖 ∉ 𝐴 𝑎𝑛𝑑 𝑎𝑖 ∉ 𝐵
⟹ n= 4, m= 3 Total number of ways for one element of P
and two subsets A and B = 4
∴ Required distance= √𝑚2 + 𝑛2 = 5
𝒙 𝒙 ∴ Total number of ways for n elements
21. Let f(x) = 𝒔𝒊𝒏𝟐 + 𝒄𝒐𝒔𝟐 𝒂𝒏𝒅 𝒈(𝒙) = 𝑎1 , 𝑎2 , … , 𝑎𝑛 of P and two subsets A and B=
𝟐 𝟐
𝟐 𝟐
𝒔𝒆𝒄 𝒙 − 𝒕𝒂𝒏 𝒙 𝒂𝒍𝒔𝒐 𝒇(𝒙) = 𝒈(𝒙),then 4𝑛

(a) x ∊R Number of ways in which one particular


(b) x ∊𝛷 (empty set) element 𝑎𝑖 ∊ 𝐴 ∪ 𝐵 = 3
𝝅
(c) x ∊R - {x: x = (2n+1) 𝟐 where n ∊ I} ∴ number of ways in which all elements
(d) none of these 𝑎1 , 𝑎2 , … , 𝑎𝑛 ∊ 𝐴 ∪ 𝐵 =3𝑛

121
Solving Mathematical Problems

∴ number of ways in which at least one ⟹ n= 3m


element of p does not belong to
⟹ n= 45
𝑛 𝑛
A∪ B = 4 − 3
24. Triplet (x, y, z) is chosen from the set
𝑛 𝑛
∴ Required number = 4 − 3 . {1, 2, 3, …, n}, such that x ≤y < z. The
number of such triplets is
23. Suppose 𝑨𝟏 , 𝑨𝟐 , 𝑨𝟑 , 𝑨𝟒 , … , 𝑨𝟑𝟎 are
thirty sets each with five elements and (a) 𝒏𝟑 (b) 𝒏𝑪𝟑
𝑩𝟏 , 𝑩𝟐 , … 𝑩𝒏 are n sets each with three (c) 𝒏𝑪𝟐 (d) 𝒏𝑪𝟐 + 𝒏𝑪𝟑
elements such that ⋃𝟑𝟎 𝒏
𝒊=𝟏 𝑨𝒊 = ⋃𝒊=𝟏 𝑩𝒊 = 𝒔.
Ans. (d)
If each element of S belongs to exactly ten
of the 𝑨𝒊 ′s and exactly 9 of 𝑩𝒊 ′𝒔 then the Number of selections when x < y < z = (𝑛3)
value of n is
Number of selections when x= y < z = (𝑛2)
(a) 15 (b) 135
(c) 45 (d) 90 (Here in fact we have to select only two
numbers out of n numbers).
Ans. (c)
∴ required number= (𝑛3)+(𝑛2)
Since 𝐴𝑖 has 5 elements, we have
25. There are n locks and n keys. If all
∑30
𝑖=1 𝑛(𝐴𝑖 ) = 5 × 30 = 150 locks and keys are to be matched 100%
……………..(1) then maximum numbers of trials
Suppose S has ‘m’ distinct elements. required are
𝒏(𝒏−𝟏)
Since each element of S belongs to exactly (a) (b) ∑𝒏𝒓=𝟏 𝒓
𝟐
𝑛+1
10 of 𝐴𝑖 ′𝑠 we also have (c) ( 2 ) (d) (𝑛−1
2
)
30
Ans. (a)
∑ 𝑛(𝐴𝑖 ) = 10𝑚,
𝑖=1 Maximum number of trials to match first
lock and key is (n- 1).
From (1) and (2) , we get 10m = 150
Max. number of trails to match second lock
⟹ m= 15
and key is (n- 2) and so on.
Since 𝐵𝑖 has 3 elements and each element of (𝑛−1)𝑛
S belongs to exactly 9 of 𝐵𝑖 ′𝑠 , we have ∴ Required number = ∑𝑛−1
𝑟=1 𝑟 = 2

𝑛 𝑛
26. If 𝐬𝐢𝐧 𝒙 + 𝐜𝐨𝐬 𝒙 + 𝐭𝐚𝐧 𝒙 + 𝐜𝐨𝐭 𝒙 +
∑ 𝑛(𝐵𝑖 ) = 3𝑛 𝑎𝑛𝑑 ∑ 𝑛(𝐵𝑖 ) = 9𝑚 𝐬𝐞𝐜 𝒙 + 𝒄𝒐𝒔𝒆𝒄 𝒙 = 𝟕 𝒂𝒏𝒅 𝒔𝒊𝒏𝟐𝒙 = 𝒂 −
𝑖=1 𝑖=1
𝒃√𝟕 ordered pair (a, b) can be
⟹ 3n= 9m
122
Solving Mathematical Problems

(a) (6, 2) (b) (8, 3)


(c) (22, 8) (d) (11, 4)
1 1 1
∴𝑢𝑟 = |4̲ − |3̲ + |4̲̲ = 12 − 4 + 1
Ans. (c) |2 ̲
̲ ̲ ̲
̲ ̲
̲ ̲ ̲
̲
̲ ̲
̲
̲
̲ ̲
̲ ̲
̲
̲ ̲
sin 𝑥 + cos 𝑥 + tan 𝑥 + cot 𝑥 + sec 𝑥 ( ̲̲̲ ̲ )
+ 𝑐𝑜𝑠𝑒𝑐 𝑥 = 7
28. Number of positive unequal integral
1 solutions of equation x+ y+ z = 6 is
⟹ (sin 𝑥 + cos 𝑥) + ( )
sin 𝑥 . cos 𝑥
(sin 𝑥 + cos 𝑥) (a) 4! (b) 3!
+ =7
sin 𝑥 . cos 𝑥 (c) 6! (d) 2×4!

2 Ans. (b)
⟹ (sin 𝑥 + cos 𝑥) (1 + )
sin 2𝑥
2 Given x+ y+ z = 6 …….(1)
= (7 − )
sin 2𝑥
x, y, z ∊ N and are unequal.
4 4
⟹ (1 + sin 2𝑥) (1 + 𝑠𝑖𝑛2 2𝑥 + sin 2𝑥) = ⟹ x, y, z ∊ {1, 2, 3} and are unequal
4 28
49 + 𝑠𝑖𝑛2 2𝑥 − sin 2𝑥 (Squaring both sides)
∴ Required number of solutions= 3! = 6
⟹ 𝑠𝑖𝑛2 2𝑥 − 44𝑠𝑖𝑛2 2𝑥 + 36𝑠𝑖𝑛2𝑥 = 0

⟹ sin2x = 22 - 8√7 29. The plain containing the two straight


lines r⃗= a⃗+ 𝜆b⃗ and r⃗= b⃗+ 𝜇a⃗ is
27. If ur denotes the number of one–one
functions from (a) [r⃗ a⃗ b⃗] = 0
{𝒙𝟏 , 𝒙𝟐 , … , 𝒙𝒓 } 𝒕𝒐 {𝒚𝟏 , 𝒚𝟐 , … , 𝒚𝒓 } such that (b) [r⃗ a⃗ a⃗ × 𝒃⃗] = 0
f(𝒙𝒊 ) ≠ 𝒚𝒊 , for i= 1, 2, 3, …, r then 𝒖𝟒 =
(c) [r⃗ b⃗ a⃗ × 𝒃⃗] = 0
(a) 9 (b) 44 (d) none
(c) 265 (d) none of these
Ans. (a)
Ans. (a)
Given lines are r⃗ = a⃗ + 𝜆b⃗ …………..(1)
𝑢𝑟 = number of ways of putting
𝑥1 , 𝑥2 , … , 𝑥𝑟 in r corresponding place so that r⃗ = b⃗ + 𝜇a⃗ ……………..(2)
no 𝑥1 is put in the corresponding place
lines (1) and (2) intersect at (a⃗ + b⃗ )

1 1 (−1)𝑟
Then the plane passes through (a⃗ + b⃗ )
= |r̲(|2̲̲ − |3̲ + ⋯ + )
̲ ̲
̲ |𝑟̲ ̲
̲
̲ ̲ Also, line (1) is parallel to b⃗ and line (2) is
̲ ̲
̲
̲
̲
parallel to a⃗ ⟹ (a⃗ ×b⃗ ) is normal to plane
containing these lines.

123
Solving Mathematical Problems

∴ Eqn. of reqd. plane is TOPIC-WISE OBJECTIVE


[r⃗ - (a⃗ +b⃗ )].(a⃗ ×b⃗ ) = 0 SOLVED PROBLEMS

r⃗(a⃗ ×b⃗ ) - (a⃗ +b⃗ ). (a⃗ ×b⃗) = 0

⟹ [r⃗ a⃗ b⃗] = 0 THEORY OF EQUATIONS


30. Let a⃗= 2î + ĵ - 2k̂ and b⃗= î + ĵ . If c⃗ is 1. The sum of the roots of equation 𝒙𝟕 +
a vector such that a⃗.c⃗ = |c⃗|, |c⃗-a⃗| = 2√𝟐 𝟗𝒙𝟔 − 𝟐 = 𝟎 is
and angle between | (a⃗× 𝒃⃗) × 𝒄⃗| = (a) 0 (b) 3
(c) -9 (d) 7
(a) 2/3 (b) 1/3
(c) 3/2 (d) 1 Solution:- (c)
Ans. (c) Given a⃗ = 2 î+ ĵ- 2 k̂ 𝑐𝑜𝑒𝑓𝑓𝑖𝑐𝑖𝑒𝑛𝑡 𝑜𝑓 𝑥 6
Sum of the roots = (− 𝑐𝑜𝑒𝑓𝑓𝑖𝑐𝑖𝑒𝑛𝑡 𝑜𝑓 𝑥 7 )
b⃗= î+ ĵ
= −9.
a⃗ .c⃗=|c⃗|……………(1)
2. The sum of all the roots of the equation
|c⃗-a⃗|= 2√2 …………….(2)
∣ 𝒙 − 𝟐 ∣𝟐 +∣ 𝒙 − 𝟐 ∣ −𝟐 = 𝟎 is
𝜋
Angle between (a⃗ ×b⃗ ) and c⃗= 6
(𝒂) 4 (b) 2
𝜋
Now, |(a⃗ ×b⃗ )× 𝑐⃗|= |a⃗ ×b⃗ ||c⃗|sin 6 = (c) 6 (d) none.
3
|𝑐⃗|……(3) Solution:- (a) ∣ x -2∣ = t
2

From (3), |𝑐⃗ − 𝑎⃗|2 = 8 𝑡2 + 𝑡 − 2 = 0

⟹(𝑐⃗ − 𝑎⃗).( 𝑐⃗ − 𝑎⃗)= 8 ∴ t = 1, -1, but -1 is not acceptable.

⟹|𝑐⃗|2 + |𝑎⃗|2 − 2𝑎⃗. 𝑐⃗ = 8 So, x = 3, 1.

⟹ |𝑐⃗|2 + 9 − 2|𝑐⃗| = 8 (from (1)) Sum of the roots = 4.

∴ |𝑐⃗| = 1
3
From (3), |(a⃗ ×b⃗ )× 𝑐⃗ | = . 3. The equation 𝟑𝒙𝟏𝟎 + 𝟕𝒙𝟔 + 𝟓𝒙𝟒 +
2
𝟐𝒙𝟐 + 𝟏 = 𝟎 has
(a) 10 (b) 6 (c) 2
(d) zero real roots.

Solution:-

124
Solving Mathematical Problems

Let 𝑓(𝑥) = 3𝑥10 + 7𝑥 6 + 5𝑥 4 + 2𝑥 2 + 1 6. The equation (𝒙 − 𝒂)𝟑 + (𝒙 − 𝒃)𝟑 +


𝑓(−𝑥) = 3𝑥10 + 7𝑥 6 + 5𝑥 4 + 2𝑥 2 + 1 (𝒙 − 𝒄)𝟑 has
(a) All real roots (b) one real two
Here f(x) and f(-x) has no sign change.
imaginary roots (c) three real
By sign rule, f(x) = 0 has no real roots. roots (d) none

4. The equations 𝒙𝟐 − 𝒌𝒙 − 𝟐𝟏 = Solutions:- (b) Differentiating the


𝟎 𝒂𝒏𝒅 𝒙𝟐 − 𝟑𝒌𝒙 + 𝟑𝟓 = 𝟎 have a function w.r.t. x, we get
common root then the value of k is equal
𝑓 ′ (𝑥) = 3{(𝑥 − 𝑎)2 + (𝑥 − 𝑏)2
to
+ (𝑥 − 𝑐)2 } > 0 ∀ 𝑥
(a) -6 (b) 4
𝑆𝑜, 𝑓 ′ (𝑥) = 0 has no repeated roots.
(c) 5 (d) 6

Solution:- (b) From the above two 7. If √𝒂 + √𝒃 be one of the roots of the
equations, we have given equation (a and b are not perfect
squares) with rational coefficients, then
𝑥2 𝑥 1 lowest degree of such an equation must
= − =
−35𝑘 − 63𝑘 35 + 21 −3𝑘 + 𝑘 be
7𝑘 28 (a) 2 (b) 3
∴𝑥= ,𝑥 =
4 𝑘 (c) 4 (d) none.

𝑆𝑜, 𝑘 2 = 16 Solutions:- (c) Irrational roots occur in pairs.

⇨ 𝑘 = ± 4. If one root is √𝑎 + √𝑏, then the other roots


5. The numbers of solutions of the equation are: √𝑎 − √𝑏, −√𝑎 + √𝑏, √𝑎 − √𝑏.
∣ 𝒙 ∣= 𝐜𝐨𝐬 𝒙 is
∴ Number of roots are = 4.
(a) 1 (b) 2
(c) 3 (d) So, the lowest degree is 4.
none
8. If 𝜶𝟏 , 𝜶𝟐 , … . , 𝜶𝒏 are the roots of the
Solution: (b) Two graphs below intersect at equations 𝒙𝒏 − 𝒏𝒂𝒙 − 𝒃 = 𝒄 𝒂𝒏𝒅 𝒕𝒉𝒆𝒏
two points only. (𝜶𝟏 − 𝜶𝟐 )(𝜶𝟏 − 𝜶𝟑 ) … . (𝜶𝟏 − 𝜶𝒏 )
equals
(a) nα (b) n (𝜶𝟏 𝒏−𝟏 + 𝒂)
(c) na. 𝜶𝟏 𝒏−𝟏 (d)
none

Solution:- (b) (𝑥 − 𝛼1 )(𝑥 − 𝛼2 ) … (𝑥 −


𝛼𝑛 ) = 𝑥 𝑛 − 𝑛𝑎𝑥 − 𝑏

125
Solving Mathematical Problems

Differentiating w. r. t. x, Solution:- Since roots are imaginary, so Δ=


𝑏 2 − 4𝑎𝑐 < 0
(𝑥 − 𝛼2 )(𝑥 − 𝛼3 ) … (𝑥 − 𝛼𝑛 ) + (𝑥 −
𝛼1 )(𝑥 − 𝛼3 ) … . (𝑥 − 𝛼𝑛 ) + ⋯ + (𝑥 − ∴The roots α and β are given by, α=
𝛼1 )(𝑥 − 𝛼2 ) … (𝑥 − 𝛼𝑛−1 ) = 𝑛𝑥 𝑛−1 + 𝑛𝑎 −𝑏+𝑖√4𝑎𝑐−𝑏 2 −𝑏−𝑖√4𝑎𝑐−𝑏 2
𝑎𝑛𝑑 𝛽 = .
2𝑎 2𝑎
Putting x = 𝛼1 , we get _____
And α = β͞, so ∣ α∣ = ∣ β∣ .
(𝛼1 − 𝛼2 )(𝛼1 − 𝛼3 ) … (𝛼1 − 𝛼𝑛 ) =
𝛼1 𝑛−1 . 𝑛 + 𝑛𝑎. 𝑏2 4𝑎𝑐−𝑏 2 𝑐
Moreover, ∣ α∣ = √4𝑎2 + = √𝑎 ⇒∣
4𝑎2

𝛼 ∣> 1 (∵ 𝑐 > 𝑎).

9. Let p, q ∈{1, 2, 3, 4}, the number of ∴ ∣ α∣ = ∣ β∣ & ∣ α∣ >1.


equations of 𝒑𝒙𝟐 + 𝒒𝒙 + 𝟏 = 𝟎 having
real roots are
(a) 7 (b) 8 NUMBER THEORY
(c) 9 (d) none
11. The congruence 35x ≡14 (mod 21) has
Solutions:- (a) Δ= 𝑞 2 − 4𝑝 ≥ 0 for real (a) 7 solutions (b) 6 solutions
roots. (c) Unique solution (d) No
solution
i.e. 𝑞 2 ≥ 4𝑝
Solution:- (a) 35x -14 is divisible by 21
Now, if p =1, then 𝑞 2 ≥ 4, i.e. q= 2, 3, 4.
gcd (35, 21) = 7
If p = 2, then 𝑞 2 ≥ 8, i.e. q = 3, 4.
and 7 divides 14; hence the given
If p = 3, then 𝑞 2 ≥ 12, q = 4
congruence has 7 solution.
If p =4, then 𝑞 2 ≥ 16, 𝑖. 𝑒. 𝑞 = 4

There are 7 such favourable cases.

12. The maximum value of f(x) = (𝒙 −


10. If 0 <a < b <c and the roots α, β of the 𝟐)𝒏 (𝟑 − 𝒙)𝒏 for a natural number n≥ 1
equation 𝒂𝒙𝟐 + 𝒃𝒙 + 𝒄 = are imaginary, and 2 ≤ x ≤ 3 is
then 𝟏 𝟏
(a) (b) 𝟒𝒏
𝟐𝒏
(a) ∣α∣ <1 (b) ∣β∣ <1 𝟏 𝟏
(c) 𝟖𝒏 (d) 𝟏𝟔𝒏
(c) ∣α∣ = ∣β∣ (d)
none Solution:- (b) [If a + b = λ is given then ab is
𝜆
maximum when a = b = 2.]

126
Solving Mathematical Problems

Here (x -2) + (3- x) = 1 then(x- 2) = ½ = (3- x) 𝟏 𝟏


15. The sum if the series + 𝟔.𝟕 + ⋯ +
𝟓.𝟔
𝟏
1 𝑛 1 𝑛 1 𝑛 is
∴ [𝑓(𝑥)]𝑚𝑎𝑥 = (2) (2) = (4) . 𝟏𝟎𝟒.𝟏𝟎𝟓
𝟐𝟐 𝟐𝟎
(a) (b) 𝟐𝟏
𝟐𝟏
13. If a ≡ b (mod n). Prove that gcd(a, n) is
(c) 1 (d) None
(a) gcd (b, n) (b) b
1 1 1
(c) n (d) none Solution:- (b) 5.6 + 6.7 + ⋯ + 104.105 =
1 1 1 1 1 1 1
Solution:- (a) gcd (a, n) = d (5 − 6) + (6 − 7) + ⋯ + (104 − 105) = 5 −
1 20
⤇d ∣ a, d∣ n but n∣ (a-b) = 21.
105

𝟏 𝟏
⤇d ∣ a –b , d ∣ a 16. The sum of the series + +
𝟏+√𝟐 √𝟐+√𝟑
𝟏 𝟏
⤇d ∣ a –(a –b)= b +⋯+ is
√𝟑+√𝟒 √𝟗𝟗+√𝟏𝟎𝟎

⤇d ∣ a, d ∣ b.
(a) 1 (b) 9
Same as gcd (b, n) = d. So, gcd (a, n) = gcd (c) 10 (d) None
(b, n). 1−√2 √2−√3 √3−√4
Solution:- (b) S = + + +
1−2 2−3 3−4
14. The highest power of 3 contained in √99−√100
⋯+ = −1 + 10 = 9.
1000! Is 99−100

(a) 493 (b) 494


17. The last digit of 𝟒𝟑𝟏𝟕 is
(c) 495 (d)
(a) 3 (b) 7
496
(c) 1 (d)
Solution:- (d) P= 3, n = 1000. None

The highest power of 3 contained in n! Is Solution:- (a) 43 ≡ 3 (mod 10)


given by
(43)17 ≡ 317 (mod 10) ;
𝑛 𝑛 𝑛
𝑘(𝑛!) = [ ] + [ 2 ] + [ 3 ] + ⋯ i.e. last digit of 4317 is the last digit of 317 .
𝑝 𝑃 𝑃

1000 1000 1000 Now, 34 ≡ 1 (𝑚𝑜𝑑 10)


𝑆𝑜, 𝐴𝑛𝑠 = [ ]+[ ]+[ 3 ]
3 9 3 (34 )4 ≡ 1 (𝑚𝑜𝑑 10)
1000 1000
+[ 4 ]+[ 5 ]
3 3 So, the last is 3.
1000
+[ 6 ]
3 18. The remainder when (𝟐𝟐𝟐𝟐)𝟓𝟓𝟓𝟓 is
divisible by 7 is
= 496.
(a) 3 (b) 5
(c) 7 (d) 9
127
Solving Mathematical Problems

Solution:- (b) 2222 ≡ 3 mod 7 244 ≡ 1 (𝑚𝑜𝑑 23)

(2222)3 ≡ 27 mod 7 244 ≡ 24 (𝑚𝑜𝑑 23)

(2222)3 ≡ (-1) mod 7 241 . 8 ≡ 3 × 8 (𝑚𝑜𝑑 23)

(2222)5553 ≡ (−1)1851 mod 7 241 ≡ 3 (𝑚𝑜𝑑 23)

(2222)2 ≡ 9 mod 7 So, the least positive residue is 3.

(2222)5553 ≡ −9 mod 7− 9 ≡ 5 mod 7. 21. The remainder when 4(29)! +5! Is


divisible by 31 is
19. The unit digit of (𝟐𝟑𝟑𝟕)𝟐𝟑𝟑𝟕 is (a) 3 (b) 5
(a) 3 (b) 5 (c) 7 (d)
(c) 7 (d) 9 None
Solution:- (c) Last digits Solution :-(d) Wilson’s theorem states that “
If p is a prime then (p -1)!+ 1 ≡ 0(mod p).
(2337)1 ≡ 7 (𝑚𝑜𝑑 10)
The converse of this theorem is also true.
(2337)2 ≡ 9 (𝑚𝑜𝑑 10)
So, by Wilson’s theorem,
3
(2337) ≡ 3 (𝑚𝑜𝑑 10)
(30)! +1 ≡ 0 (mod 31), since 31 is prime.
4
(2337) ≡ 1 (𝑚𝑜𝑑 10)
(31 -1)(29)! +1 ≡ 0 (mod 31)
5
(2337) ≡ 5 (𝑚𝑜𝑑 10)
⇨-29! +1 ≡ 0(mod 31)
2337 ≡ 1 (mod 4)
⤇29! -1 ≡ 0 (mod 31)
584 4
((2337) ) . 2337 ≡ 7(𝑚𝑜𝑑 10)
⤇ 4(29)! -4≡ 0 (mod 31)
Unit digit is 7.
⤇ 4(29)! -4 +124 ≡ 0 (mod 31)
𝟒𝟏
20. The least positive residue in 𝟐 (mod 23)
is ⤇ 4(29)! +120 ≡ 0 (mod 31)
(a) 3 (b) 5 ⤇ 4(29)! +5! ≡ 0(mod 31)
(c) 7 (d) 9

Solution:- If a ≡ b (mod m) then b is said to


be the residue of a modulo m. 22. The smallest positive integer that has
remainder 4, 3 and 1 when divided by 5,
If a ≡ b (mod m) then 𝑎𝑛 ≡ 𝑏 𝑛 ∀ 𝑛 ∈ 𝐼 + . 7, and 9, respectively, is
But the 23 is a prime & 2 is a prime to 23. (a) 211 (b) 201
By Fermat’s theorem, 222 ≡ 1 (𝑚𝑜𝑑 23) (c) 199 (d) 189

128
Solving Mathematical Problems

Solution:- X ≡ 4 (mod 5) Ends in 9 : 310 , 330 , 350 , 370 , 390 , …. ;

X ≡ 3 (mod 7) Ends in 1 : 320 , 340 , 360 , 380 , 3100 , …..


𝟗
X ≡ 1 (mod 9) 24. The last digit of 𝟗𝟗 is
Let X = 4 +5t, t ∈Z (a) 1 (b) 7 (c) 9 (d) none
So, 4 + 5t ≡ 3 (mod 7) ⇨ 5t +1 ≡ 0 (mod 7) Solution:- The last digit of 93 is 9.
Let, t = 4 + 7u, u ∈ℤ The last digit of 99 is 9.
So, X = 4 +5 (4+ 7u) = 24+ 35u. 9
Thus, the last digit of 99 is 9.
24 +35u ≡ 1(mod 9) ⤇ 35u + 23 ≡ 0 (mod 9)
⤇ 36u –u ≡ -23 (mod 9) ⤇- u ≡ -23 (mod 9)
LINEAR ALGEBRA
⤇ 23 –u ≡ 0 (mod 9)
25. Which of the following sets is not LIN?
∴ u = 5 +9v (let), v ∈ ℤ
(a) {𝟏, 𝑿, 𝟏 + 𝑿 + 𝑿𝟐 } in a vector space
∴ X = 24 + 35 (5+ 9v) = 199+ 315v. of all polynomials over the field of
real numbers.
So, the smallest positive solution is 199. (b) {𝟏, 𝑿, 𝑿𝟐 , … } in a vector space of all
polynomials over the field of real
numbers.
23. The last digit of 𝟑𝟖𝟎 is (c) {(𝟏, 𝟏, 𝟎, 𝟎), (𝟎, 𝟏, −𝟏, 𝟎), (𝟎, 𝟎, 𝟎, 𝟑)}
(a) 3 (b) 9 in 𝑽𝟒 (𝑹).
(c) 7 (d) 1 (d) {(𝟏, 𝟐, 𝟏), (𝟑, 𝟏, 𝟓), (𝟑, −𝟒, 𝟕)} in
𝑽𝟑 (𝑹).
Solution:- (d) 31 ≡ 3 (𝑚𝑜𝑑 10)
Solution:- (a) Let a, b, c be scalars such that
32 ≡ 9 (𝑚𝑜𝑑 10)
a(1) + bX +c (1 + 𝑋 + 𝑋 2 ) = 0 ⇒ (a+ c) + (b
33 ≡ 7 (mod 10)
+c)X + c𝑋 2 = 0 ⇒ a+ c = 0, b+ c = 0, c = 0
34 ≡ 1 (mod 10)
⇒ a = 0, b = 0, c = 0.
5
3 ≡ 3 (mod 10)
∴ The vectors 1, X, 1 + 𝑋 + 𝑋 2 are LIN over
380 =(316 )5 = (33×5 . 3)5 = (325 )3 . 35 ≡ the field of real no’s.
33 . 3 (𝑚𝑜𝑑 10) ≡ 1 (mod 10)
(𝑏)𝑎0 1 + 𝑎1 𝑋 + 𝑎2 𝑋 2 + ⋯ = 0
More explicitly, we can write :-
By definition of equality of two polynomials
𝑎0 = 𝑎1 = 𝑎2 = ⋯ = 0

129
Solving Mathematical Problems

∴ The vectors are LIN. 𝑥 𝑦


Solution:- (d) 𝑤1 ∩ 𝑤2 = {[ ] ∶ 𝑥, 𝑦 ∈
0 0
(c) a(1,1, 0, 0) + b(0, I, -1, 0) + c(0, 0, 0, 3) = ℝ}; dim (𝑤1 ∩ 𝑤2 ) = 1.
0
28. Let T:𝑹𝒏 ⟶> 𝑹𝒎 , m >n, be a linear
𝑎=0 transformation.
𝑎+𝑏 =0
⇒ { ⇒ 𝑎 = 𝑏 = 𝑐 = 0 is the
−𝑏 = 0 Consider the following statements about T:
3𝑐 = 0
only solution. (i)T can be one to one (ii) T can onto (iii) dim
(T(𝑹𝒏 ))≥ n
∴The vectors are LIN.
(A) Only (i) is true (B) only (ii) is false
1 3 3
(𝑑)│2 1 −4│ = 0 ⇒ rank (A) <3 ⇒ the (C) only (ii) is true (D) only (iii) is
1 5 7 true
set of vectors are linearly dependent.
Solution:- (B) T:𝑅 𝑛 ⟶
> 𝑅 𝑚 , m >n.
26. The eigenvalue of 𝑨𝟒 , where A =
𝟏 𝟎 −𝟏 i.e. no. Of elements in domain < no. of
(𝟗 𝟒 𝟏 ), is elements in range.
𝟑 𝟏 𝟏
(a) 3, 4, 5, (b) 1, 2, 3, (c) 5, 6, 7, (d) i.e. T can be one to one is true statement.
none
But T can be onto is false as m > n.
Solution:- (d) ∣ A –λI∣ = 0
dim(T(𝑅 𝑛 ))≥ n is also a true statement as n <
1−𝜆 0 −1 m.
⇒ 𝑑𝑒𝑡 [ 9 4−𝜆 1 ]=0 ⇒
3 1 1−𝜆
(1 − 𝜆)(2 − 𝜆)(3 − 𝜆) = 0 ⇒ 𝜆 = 1, 2, 3
𝒂 −𝒃
4
29. If A = [ ] ∈ 𝒌𝟐×𝟐 , 𝒃 ≠ 𝟎 then A
∴For the matrix 𝐴 , the eigen values 𝒃 𝒂
are: (1)4 = 1, (2)4 = 16, (3)4 = 81. has eigen values if k is
(A) R (B) C
27. Let, 𝑴𝟐×𝟐 (𝑹) be the vector space of all (C) Q (D) All
2× 𝟐 matrices over R and above
𝒙 𝒚
Let 𝒘𝟏 = {[ ] : 𝒙, 𝒚, ∈ ℝ} & 𝒘𝟐 = Solution:- (D) Ch. Equation ∣ A- λI∣ = 0
𝟎 𝒙
𝒙 𝒚
{[ ] ∶ 𝒙, 𝒚, 𝒛 ∈ ℝ} then dim (𝒘𝟏 ∩ 𝑎−𝜆 −𝑏
𝒛 𝟎 ⇒ [ ]=0
𝑏 𝑎−𝜆
𝒘𝟐 ) is
⇒ 𝜆2 − 2𝑎𝜆 + (𝑎2 + 𝑏 2 ) = 0
(a) 2 (b) 3
(c) 4 (d) 1

130
Solving Mathematical Problems

2𝑎 ± √4𝑎2 − 4(𝑎2 + 𝑏 2 ) 31. The equations have:


⇒𝜆= =𝑎±𝑏 x –y +2z =4
2
3x +y +4z = 6
Hence the eigenvalues of A be a + b and a –
x +y +z = 1
b.

Then A has eigenvalues when k = R, C, Q (a) Unique solution (b) infinite


solution (c) no solution
(d) none of these
30. The no. of solution of the system of 1 −1 2 4
equation Solution:- (b) A = [3 1 4 |6] ∼
1 1 1 1
2x +y –z = 7 1 −1 2 4
[0 4 −2 |−6]
X -3y +2z = 1 0 0 0 0

X +4y -3z = 5 ∴ rank (A ⋮ B) = 2 = rank (A) < 3

(a) Unique solution (b) no solution ∴ the given system of equations are
(c) many solution (d) exactly two consistent & have infinite number of
solution solutions.

2 1 −1 𝟏 −𝟏
𝑥 7 32. If the matrix A = [ ], then find
Solution:- (b) (1 −3 2 ) (𝑦) = (1) −𝟏 𝟏
1 4 −3 𝑧 5 the matrix 𝑨𝒏+𝟏
(a) 2𝑨𝒏 (b) 𝟐𝒏 𝑨
⇒ 𝐴𝑋˷ = 𝑏˷
(c) 𝟐𝒏−𝟏 𝑨 (d) A
2 1 −1 2 1 −1 2 1 −1
[1 −3 2 ] ~ [−1 −4 3 ] ~ [1 4 −3] Solution:- (b) ∣A- λI∣ = 0
1 4 −3 1 4 −3 0 0 0
1−𝜆 −1
⇒ [ ] = 0 ⇒ 𝜆2 − 2𝜆 = 0
Rank (A) = 2 −1 1 − 𝜆

2 1 −1 : 7 2 1 −1 : 7 So 𝐴2 − 2𝐴 = 0
[1 −3 2 : 1] = [−1 −4 3 : −6]
1 4 −3 : 5 1 4 −3 : 5 ⇒ 𝐴3 = 2𝐴2 = 4𝐴
2 1 −1 : +7
= [−1 −4 3 : −6] ∴ 𝐴𝑛+1 = 2𝑛 𝐴
0 0 0 : −1
33. The following system of linear equation is
Rank (A ⋮ b) = 3 constant if α, β don’t equal to
x +3y +z = 3
∴ rank (A) ≠ rank (A ⋮ b) 2x +3y +5z = 4
The system has no solution. 4x +9y +αz = β

131
Solving Mathematical Problems

(a) (1, 3) (b) (5, 10) (b) The eigenvalues of real symmetric
(c) (7, 10) (d) None matrix are real.
(c) The eigenvalues of skew
Solution:- (c) Given system of equation can
Hermition matrix are real.
be expressed as AX = B
(d) The eigenvalues of unitary matrix
Argumented matrix [A ∣ B] = may be real.
1 3 1 3
Solution:- (c) As the eigenvalues of skew-
[2 3 5 | 4 ] ∼
4 9 𝛼 𝛽 Hermition matrix are either pure imaginary
1 3 1 3 or zero.
𝑅 ⟷𝑅 −2𝑅
[0 −3 3 | −2 ] 𝑅23 ⟷𝑅23 −4𝑅
0 −3 𝛼 − 4 𝛽 − 12 36. Let 𝑴𝒏 (𝑹) be the set of n×m matrices
with real entries, if all A ∈ 𝑴𝒏 (𝑹) have
1 3 1 3
both negative and positive eigen values
∼ [0 −3 3 | −2 ] 𝑅3 ⟷ 𝑅3 − 𝑅2
0 0 𝛼 − 7 𝛽 − 10 then the set is having
(a) Positive semi definite matrices
Given system of linear equation be constant only
(b) Positive & negative semi definite
i.e. rank (A) = rank (A∣ B) = 3
matrices
i.e. α -7 ≠ 0 , β -10 ≠ 0 (c) Negative definite matrices only
(d) Indefinite matrices
⇒ α ≠ 7 & β ≠ 10
Solution:- (d) A positive and positive semi
34. Let x & y in 𝑹𝒏 be non zero row vectors
definite matrices have positive eigenvalues
from the matrix A = x𝒚𝑻 , wherer 𝒚𝑻
only.
denote the transpose of y. Then the rank
of A is Negative definite matrices have negative
(a) 0 or 1 (b) 2 eigenvalues only.
(3) at least n/2 (d)
But indefinite matrices have both positive
none
and negative eigenvalues.
Solution:- (a) A = x𝑦 𝑇
37. Which of the following is true?
Then A is a matrix of order 1× 1. 𝟐 𝟐
(a) The matrix [ ] is diagonalisable
𝟏 𝟐
If A is non -zero then rank (A) = 1 𝟏 𝟎
(b) The matrix [ ] is diagonalisable
𝟏 𝟓
If A i zero mtx. Then rank (A) = 0. 𝟐 𝟏
(c) The matrix [ ] is diagonalisable
𝟎 𝟓
35. Which of the following is false? 𝟓 𝟎
(d) The matrix[ ] is not
(a) The eigenvalues of Hermition 𝟏 𝟏
diagonalisable
matrix are real.

132
Solving Mathematical Problems

Solution:- (b) We know that any matrix (2 { 𝒇 ∈ 𝑽 ∣ 𝒇(−𝑿) = 𝒇(𝑿) }𝒂𝒏𝒅 𝑽𝟐 =


×2) is said to be diagonalisable if it has two {𝒇 ∈ 𝑽 ∣ 𝒇(−𝑿) = −𝒇(𝑿)},
different eigen values. (A) Neither 𝑽𝟏 nor 𝑽𝟐 is a subspace
of V.
1 0
[ ] is lower triangular matrix, its (B) 𝑽𝟏 is a subspace of V, but not 𝑽𝟐 .
1 5
diagonal entries are its eigen values since (C) 𝑽𝟐 is a subspace of V, but not 𝑽𝟐 .
they are distinct . (D) 𝑩𝒐𝒕𝒉 𝑽𝟏 , 𝑽𝟐 are subspaces of V.

⇒ the matrix is said to be diagonalisable. Solution:- (D) The necessary and sufficient
condition for a non-empty subset W of a
38. If V is vector space on the field ℤ / 3ℤ and vector space V(F) to be subspace of V is
𝒅𝒊𝒎 ℤ (𝑽) = 𝟑 then V has
𝟑ℤ
a, b 𝜖 F and for all α, β ∈ W ⇒ aα + bβ ∈W
(a) 27 elements (b) 9 elements
(c) 30 elements (d) 15 λa ∈ W
elements
Hence both 𝑉1 , 𝑉2 satisfied these conditions.
Solution:- (a) since, dim V = 3 and ℤ / 3ℤ =
41. Let P be an n × m idempotent matrix, i.e.
{0, 1, 2}
𝑷𝟐 = 𝑷. Which of the following is FALSE?
So, they are 33 elements in V which can be (a) 𝑷𝑻 is idempotent.
expressed as the linear combination of (b) The possible eigenvalues of P can be
elements of basis. zero.
(c) The non-diagonal entries of P can be
39. If V is a vector space over the field ℤ /7ℤ
zero.
and 𝒅𝒊𝒎 ℤ (𝒀) = 𝟐 then V has
𝟕ℤ (d) There may be infinite no. of n× m
(a) 49 elements (b) 14 elements non- singular matrices that one
(c) 128 elements (d) None idempotent.

Solution:- (a) Since, dim V = 2 and ℤ /7ℤ = Solution:- (d) Given P be an n × m


{0, 1, 2, 3, 4, 5, 6} idempotent matrix s. t 𝑃2 = 𝑃

So, there may be 72 elements which can be If P is idempotent then 𝑃𝑇 is also


expressed which can be expressed as the idempotent P (P –I) = 0
linear combination of elements of basis &
their coefficient could only be {0, 1, 2, 3, 4, ⇒ the possible eigenvalues of P are 0 and 1.
5, 6}. ⇒ non-diagonal entries of P can be zero.
40. If V is the real vector space of all mapping So, (d) is false.
from ℝ to ℝ, 𝑽𝟏 =

133
Solving Mathematical Problems

42. If A is a 3 ×3 non-zero matrix such that 1 w 𝑤2


𝑨𝟐 = 0, then the number of non-zero
1 1 w 𝑤2
eigen values of A is
(a) 0 (b) 1 W w 𝑤2 1
(c) 2 (d) 3
𝑤2 𝑤2 1 w
2
Solution:- (a) 𝐴 = 0

A is nilpotent matrix, so all of its eigenvalues


will be zero. 45. The irreducible polynomials in C[X] are
the polynomials of degree
43. Let T : ℝ𝟑 ⟶ ℝ𝟐 be a linear (a) 0 (b) 1
transformation defined by T (x, y, z) = (x (c) 2
+) (d)None
(a) 0 (b) 1
Solution:- (b) The polynomials of degree
(c) 2 (d) 3
0 are the invertible element of C[X].
Solution:- (b) T(x, y, z) = (x +y, x –z) Now, (x By the fundamental theorem of algebra,
+y, x –z) = (0, 0) any polynomial of positive degree has a
⇒ (x +y) = 0; (x –z) = 0 ⇒x = -y = z root in C and hence a linear factor.
Therefore, any polynomial of degree
⇒ N(T) = (a, -a, a) │a∈ R𝜁 greater than 1 are reducible and those of
degree 1 are irreducible.
i.e. nullity = 1.
46. Which of the following statement is
false :
ABSTRACT ALGEBRA (a) The polynomial 𝑿𝟑 − 𝑿 + 𝟏 is
irreducible in ℤ / 2ℤ [X]
44. Let G be the set of cube roots of unity. (b) The polynomial 𝑿𝟐 − 𝟑 is
Then under multiplication of complex irreducible in Q[X].
numbers (c) The polynomial 𝑿𝟐 + 𝟏 is
(a) G is a group of finite order irreducible in ℤ / 5ℤ [X].
(b) G is an abelian group (d) The polynomial 𝑿𝟐 + 𝟏 is
(c) G is a cyclic group irreducible in ℤ/7ℤ [X].
(d) None of the above.
Solution:- (c) The polynomial 𝑋 2 + 1 is
Solution:- reducible in ℤ/5ℤ[X].

(c) Here G = {1, W, 𝑊 2 } As ℤ5 = {0, 1, 2, 3, 4}𝑎𝑛𝑑 𝑓(𝑥) = 𝑥 2 + 1.

So, G is an abelian cyclic group of order 3, f(3) = 10 ≡ 0 (mod 5)


since
134
Solving Mathematical Problems

⟾ 𝑋 2 + 1 is reducible in ℤ /5ℤ[X].

47. Let f : G ⟶ H be a group


homomorphism from a group G into a
group H with kernel K. If O(G) = 75,
O(H) = 45, O(K) = 15. Then the order
of the image f(G) is :
(a) 3 (b)5
(c) 15 (d) 45
𝐺 X -1 0 ½ 1 2
Solution:- (b) f(G) ≅ 𝐾.
P(X) 0 -1 0 4 27
𝐺 𝑂(𝐺) 75
O{f(G)} = O(𝐾) = 𝑂(𝐾) = 15 = 5.

48. Which of the following is a cyclic ∴ P(X) = (𝑋 + 1)2 (2𝑋 − 1).


group?
(i) ℤ𝟏𝟐 × ℤ𝟗 (ii) ℤ𝟏𝟎 × 50. Any group of 2p, where p is a prime
ℤ𝟖𝟓 (iii) ℤ𝟐𝟐 × number has a normal subgroup of order
ℤ𝟐𝟏 × ℤ𝟔𝟓 (iv) None p, then the index of subgroup H in G is
(a) P (b) 2
Solution:- (iii) Any group ℤ𝑝 × ℤ𝑟 is said
𝑷𝟐
to be cyclic if the greatest common divisor (c) (d)
𝟐
(GCD) of p and r is equal to 1. Similarly, none
for ℤ𝑝 × ℤ𝑞 × ℤ𝑟 is called cyclic if GCD
Solution:- (b) Given that O(G) = 2p.
between any two is equal to one.
Since p is prime, and
(i) GCD of (12, 9) = 3
(ii) GCD of (10, 85) = 5 By Cauchy’s theorem, G has an element of
(iii) GCD of (22, 21) & (21, 65) & order p then the cyclic group, H = {a,
(22, 65) = 1. 𝑎2 , … . , 𝑎𝑃 }is a subgroup of order p.
⟾ℤ22 × ℤ21 × ℤ65 is a cyclic group. 𝑂(𝐺) 2𝑝
∴ The index of H in G is = 𝑂(𝐻) = = 2.
𝑝
49. Factor P(X) = 2𝑿𝟑 + 𝟑𝑿𝟐 − 𝟏 in Q[X]
(i) (𝑿 − 𝟏)𝟐 (𝟐𝑿 + 𝟏) (ii) (𝑿 + 51. If 1, ℤ𝟏 , ℤ𝟐 , … , ℤ𝟏𝟏 are the 12 roots of
𝟏)𝟐 (𝟐𝑿 − 𝟏) (iii) (𝑿 − unity forming the cyclic group under
𝟏)𝟐 (𝟐𝑿 − 𝟏) (iv) (𝑿 + multiplication. Then ℤ𝟗 generates a
𝟏)𝟐 (𝟐𝑿 + 𝟏) cyclic sub group of the above containing:
(a) 12 elements (b) 9
Solution:- (ii) elements (c) 8
elements (d) none

135
Solving Mathematical Problems

Solution:- (d) The integral divisors of 9 are REAL ANALYSIS


1, 3, 9.
54. If f and F be both continuous in [a, b] and
∴ All the elements of order 1, 3, 9 will give are derivable in (a, b) and F’(x) = f’(x) ∀ x
subgroups. in (a, b) then f(x) and F(x) differ b
(A) 1 in [a, b] (B) x in [a, b]
So, {ℤ9 } has the subgroups {e}, (ℤ1 ), (ℤ3 ),
(C)constant in [a, b] (D)
(ℤ9 ).
none
So there is 4 elements.
Solution:- (c) Since they are continuous
52. The n equal rotations of a regular
f’(x) = F’(x)
polygon of n sides
(a) From an abelian but not cyclic Let 𝜙(x)= f(x) – F(x)
group
(b) From a cyclic group ⇒ 𝜙’(x) = f’(x)-F’(x)= 0 i.e. 𝜙 (x) =
(c) Don’t from a cyclic group constant.
(d) From non-abelian non-cyclic 55. Let 𝒇𝑿 (𝒙) = 𝒏 𝒔𝒊𝒏𝟐𝒏+𝟏 𝒙 𝐜𝐨𝐬 𝒙, then
group. 𝒏
the value of 𝐋𝐭 ∫𝟎𝟐 𝒇𝒏 (𝒙) 𝒅𝒙 −
𝒏→∞
Solution:- (b) The rotations are the 𝝅/𝟐
∫𝟎 𝐋𝐭 (𝒇𝒏 (𝒙))𝒅𝒙 is
generators of the group G. 𝒏→∞
(a) ½ (b) 0
Hence, G must be a cyclic group. (c) -½ (d) -∞

53. Let G = {z ∈ c : ∣ z∣ = 1}, then under Solution:- (d) Value =


2𝑛+1+1 1+1
multiplication of complex numbers 𝑛⎾(
2
)⎾ (
2
)
Lt [ 2𝑛+1+1+2 ]−
(a) G is a group of order (finite) 𝑛→∞ 2⎾(
2
)
(b) G is a group of infinite 𝜋/2
Lt 𝑛𝑠𝑖𝑛2𝑛+1 𝑥 cos 𝑥 𝑑𝑥
∫0
(c) G is a cyclic group 𝑛→∞

(d) None of the above. 𝑛 ⎾𝑛+1


= Lt −
𝑛→∞ 2 ⎾𝑛+2
Solution:- (c) Let G ={z ∈ c : ∣ z∣ = 1} and 𝜋/2 𝑛 1
∫0 ∞ = Lt − ∞ = −∞ .
𝑛→∞ 2 𝑛+1
𝑧1 𝑧2 ∈ 𝐺.
56. If {𝒔𝒏 } be a convergent sequence of
Then 𝑧1 𝑧2 ∈ G ⟾
∣ 𝑧1 ∣ = 1, ∣ 𝑧2 ∣ = 1. 𝟏
positive numbers ∋ 𝒔𝒏 = 𝟐 (𝒔𝒏−𝟏 +

∣ 𝑧1 𝑧2 ∣ = ∣ 𝑧1 ∣ ∣ 𝑧2 ∣ = 1. 𝒔𝒏−𝟐 )∀𝒏 ≥ 𝟐 then 𝐋𝐭 𝒔𝒏 = ?
𝒏→∞
𝟏 𝟏
∴ G is closed for multiplication. (a) 𝟑 (𝒔𝟐 + 𝒔𝟑 ) (b) 𝟑 (𝒔𝟑 +
𝟏 𝟏
And ∃ inverse of every element in G. Hence, 𝒔 ) (c) 𝟑 (𝒔𝟏 +
𝟐 𝟏
G is multiplicative group. 𝟐𝒔𝟐 ) (d) None

136
Solving Mathematical Problems

Solution:- (c) Let Lt 𝑠𝑛 = 𝑙 𝑥1 +𝑥2


𝑛→∞ if 𝑥2 > 𝑥1 , then 𝑓 ′ ( )=
2

1 𝑓 ′ (𝑥1 ) ≥ 1 ∀𝑥1 ∈ [0, 1]


𝑠𝑛 = (𝑠𝑛−1 + 𝑠𝑛−2 )
2 𝑓(𝑥2 ) − 𝑓(𝑥1 ) > (𝑥2 − 𝑥1 )𝑓 ′ (𝑥1 )
1 ≥ (𝑥2 − 𝑥1 )∀𝑥2
𝑠3 = (𝑠 + 𝑠1 ) ≥ 𝑥1 ∀ 𝑥1 , 𝑥2 ∈ [0, 1]
2 2
1 ⇒ 𝑓(𝑥2 ) − 𝑥2 ≥ 𝑓(𝑥1 ) − 𝑥1 .
𝑠4 = (𝑠 + 𝑠2 )
2 3
1
𝑠𝑘−1 = (𝑠𝑘−2 + 𝑠𝑘−3 ) 58. A function f is defined as {0, 1}, by f(x) =
2
𝟏 𝟏 𝟏
1 ∀ > 𝒙 > 𝒏+𝟏 for all n = 1, 2, 3, .... if
𝒙 𝒙
𝑠𝑘 = (𝑠𝑘−1 + 𝑠𝑘−2 )
2 given that f ∈ R {0, 1} then evaluate
𝟏
1 1 ∫𝟎 𝒇(𝒙) 𝒅𝒙
Adding all these, 𝑠𝑘 + 2 𝑠𝑘−1 = 2 (𝑠1 + 2𝑠2 )
𝝅𝟐 𝝅𝟐
1 1 (𝑨) (B) +𝟏
⇒ 𝑙 + 2 𝑙 = 2 (𝑠1 + 2𝑠2 ) 𝟔 𝟔
𝝅𝟐
(C) −𝟏 (D) none
1 𝟔
⇒ 𝑙 = 3 (𝑠1 + 2𝑠2 )
1
Solution:- (C) ∫0 𝑓(𝑥) 𝑑𝑥 =
57. For every function f : [0, 1] ⟶ℝ, which is 1 1 1
twice differentiable and satisfies f’ (x) ≥1 Lt ∑𝑛𝑟=1 𝑟 {𝑟 − 𝑟+1}
𝑛→∞
∀ x ∈ [0, 1], we must have
1 1 1 1
(a) f’’(x) ≥ 0∀ x∈[0, 1 = Lt {(1 − ) + ( − ) + ⋯
𝑛→∞ 2 2 2 3
(b) f(x) ≥ x ∀ x ∈ [0, 1] 1 1 1
(c) f(𝒙𝟐 ) − 𝒙𝟐 ≤ 𝒇(𝒙𝟏 ) − + ( − )}
𝑛 𝑛 𝑛+1
𝒙𝟏 ∀ 𝒙𝟏 , 𝒙𝟐 ∈ [𝟎, 𝟏]𝒘𝒊𝒕𝒉 𝒙𝟐 ≥
1 1
𝒙𝟏 = Lt {(1 + + ⋯ + )
𝑛→∞ 22 𝑛2
(d) f(𝒙𝟐 ) -𝒙𝟐 ≥ 𝒇(𝒙𝟏 ) − 1
𝒙𝟏 ∀ 𝒙𝟏 , 𝒙𝟐 ∈ [𝟎, 𝟏] with 𝒙𝟐 ≥ − (1 − )}
𝑛+1
𝒙𝟏
𝜋2
Solution:- (d) Taylor’s formula gives = − 1.
6
f(𝑥2 )- f(𝑥1 ) > (𝑥2 − 59. Let f be a differentiable function defined
𝑥 +𝑥
𝑥1 )𝑓 ′ ( 1 2 2) ∀ 𝑥1 , 𝑥2 ∈ on [0, 1],
[0, 1] 𝑤𝑖𝑡ℎ 𝑥2 > 𝑥1 k ∈ (0, 1) ∋ f(x) < f(k) = f(0) ∀ x ∈ [0,
1], x ≠ k, then
(a) f’(k) = 0 and f’ (0)= 0
(b) f’(k) = 0 and f(0) = 0

137
Solving Mathematical Problems

(c) f’(k) = 0 and f’(0) ≤ 0 And so on.


(d) f’(k) > 0 and f’(0) ≤ 0.
Since 𝑎1 , 𝑎3 , 𝑎5 , … .. is a decreasing
Solution:- (c) f(k) = f(0) is maximum of f in [0, sequence and 𝑎2 , 𝑎4 , 𝑎6 , …. is a increasing
1] sequence.
1 1
∴ f’(k) = 0 {𝑎𝑛 } converges to (𝑎1 + 2𝑎2 ) = (1 +
3 3
2 2
f’(0)= Lt
𝑓(ℎ)−𝑓(0) ) = 3.
2
ℎ→0 ℎ
2𝑎𝑛+1 −𝑎𝑛
Since f(h) < f(0) ∀ h ∈ [0, 1] Again, 𝑏𝑛 = 𝑎𝑛

⇒ f(h) –f(0) < 0 𝑏1 = 0


𝑓(ℎ)−𝑓(0) 𝑏2 = 2
⇒ < 0 [∵ ℎ > 0]

2
i.e. f’(0) ≤ 0. 𝑏3 =
3
60. Let {𝒂𝒏 } 𝒂𝒏𝒅 {𝒃𝒏 } be sequence of real 6
nos. Defined as 𝒂𝟏 = 𝟏 𝒂𝒏𝒅 𝒇𝒐𝒓 𝒏 ≥ 𝑏4 =
5
𝟏, 𝒂𝒏+𝟏 = 𝒂𝒏 + (−𝟏)𝒏 𝟐𝒏 , 𝒃𝒏 =
𝟐𝒂𝒏+𝟏 −𝒂𝒏 10
then 𝑏5 =
𝒂𝒏 11

(a) {𝒂𝒏 } converges to zero and {𝒃𝒏 } is a ⁞


Cauchy sequence ⁞

(b) {𝒂𝒏 } converges to non-zero and {𝒃𝒏 } is a And so on.


Cauchy sequence.
𝑏1 , 𝑏3 , 𝑏5 …. are increasing sequence &
Solution :- (b) 𝑎1 = 1 𝑏2 , 𝑏4 , 𝑏6 , …. are decreasing sequence the
{𝑏𝑛 } converges to limit
(−1) 1 1 1
𝑎2 = 𝑎1 + = (𝑏1 + 2𝑏2 ) = (0 + 2.2) = 4
2 2 3 3
1 3 1 ∴ {𝑏𝑛 } is a Cauchy sequence.
𝑎3 = 𝑎2 + 2
= [𝑎2 = ]
2 4 2
61. On x = c [0, 1] define T: x ⟶ > x by T(f(x))
1 5 3 𝒙
𝑎4 = 𝑎3 − 3 = [𝑎3 = ] = ∫𝟎 𝒇(𝒕) 𝒅𝒕 ∀ f in x then
2 8 4
a) T is one-one and onto b) T is
1 11 5 one- one but not onto c) T is not
𝑎5 = 𝑎4 + 4 = [𝑎4 = ]
2 16 8 one-one but onto d) T is neither
⁞ ⁞ one-one nor onto.

138
Solving Mathematical Problems
𝑥 5
Solution:- (a) T(f(x)) = ∫0 𝑓(𝑡) 𝑑𝑡 ∀ f in x ∴ R ∫0 𝑓(𝑥) 𝑑𝑥= 6.

⇒ Let 𝑥1 , 𝑥2 ∈ 𝑋 Evaluating the Lebergue integral [0, 2+ δ[ , δ


>0],
S.t., T(f(𝑥1 )) = T (f(𝑥2 ))
𝑥 𝑥 We get
⇒ ∫0 1 𝑓(𝑡) 𝑑𝑡 = ∫0 2 𝑓(𝑡) 𝑑𝑡.
5
∴ L∫0 𝑓(𝑥) 𝑑𝑥 = 6.
⇒ 𝑥1 = 𝑥2
0[1- 0]+ 1 [(2 -1)+ (4 -3)]+ 2[(3- 2) + (5 -4)] =
i.e. T is one-one
6.
for each T(f(x)) , ∃ only one x ∈ X,
63. Let f be an one to one function from the
𝑥
S.t, T(f(x)) = ∫0 𝑓(𝑡) 𝑑𝑡 closed interval [-1, 1] to the set of real
numbers ℝ, then
⇒T is onto. a) f must not be onto
𝟓 b) Range of f must contain a rational
62. Evaluate ∫𝟎 𝒇(𝒙) 𝒅𝒙, 𝒊𝒇 𝒇(𝒙) =
number.
𝟎, 𝒊𝒇 𝟎 ≤ 𝒙 ≤ 𝟏 c) Range of f must contain an
{ 𝟏, {𝟏 ≤ 𝒙 < 𝟐} ∪ {𝟑 ≤ 𝒙 < 𝟒} irrational no.
𝟐, {𝟐 ≤ 𝒙 < 𝟑} ∪ {𝟒 ≤ 𝒙 < 𝟓}
d) Range of f must contain both
By using Riemann & Lebesgue definition of rational and irrational nos.
the integral
Solution:- d) y = sin−1 𝑥 (let)
𝟓 𝟓
a) R ∫𝟎 𝒇(𝒙) 𝒅𝒙 > 𝑳 ∫𝟎 𝒇(𝒙) 𝒅𝒙 x ∈ [-1 , 1]
𝟓 𝟓
b) R∫𝟎 𝒇(𝒙) 𝒅𝒙 < 𝑳 ∫𝟎 𝒇(𝒙) 𝒅𝒙 𝜋 𝜋
𝟓 𝟓
y ∈ [-2 , 2 ]
c) R∫𝟎 𝒇(𝒙) 𝒅𝒙 = 𝑳 ∫𝟎 𝒇(𝒙) 𝒅𝒙
d) None. ⇒ range of f must contain both rational and
irrational nos. f is onto here.
Solution:- c) Using Riemann definition of the
integral (where the subdivision is taken of 64. The sequence
the segment [0, 5] by the division points √𝟏𝟏, √𝟏𝟏 + √𝟏𝟏, √𝟏𝟏 + √𝟏𝟏 + √𝟏𝟏, ….
𝑥0 , 𝑥1 , … , 𝑥𝑛 on X-axis) the upper & lower converges to
Riemann sums tend to the common value. 𝟏+√𝟒𝟑 𝟏+√𝟒𝟓
a) b)
𝟐 𝟐
0(1- 0) +1 (2- 1) + 2( 3- 2) + 1 (4 -3) +2 (5 -4) 𝟏+√𝟐𝟑 𝟏+√𝟐𝟗
c) d)
𝟐 𝟐
=6
Solution:- b) 𝑆 2 = 2 + 𝑆 ⇒ 𝑆 2 − 𝑆 − 11 =
(Since the function is constant an each of
1+√45
the subintervals) 0⇒𝑆= 2

139
Solving Mathematical Problems

65. The sequence log(1+𝑎𝑥)−log(1−𝑏𝑥)


f(0)= Lt 𝑓(𝑥) = Lt
𝑥→0 𝑥→0 𝑥
√𝟐, √𝟐 + √𝟐, √𝟐 + √𝟐 + √𝟐, .....
converges to 𝑎 log(1 + 𝑎𝑥)
= Lt
a) 2 b) 3 𝑥→0 𝑎𝑥
−𝑏 log(1 − 𝑏𝑥)
𝟐+√𝟐 √𝟑+𝟏 + Lt
c) d) 𝑥→0 −𝑏𝑥
𝟑 𝟐

Solution:- a) 𝑆 2 = 2 + 𝑆 ⇒ 𝑆 2 − 𝑆 − 2 = = 𝑎. 1 + 𝑏. 1
0 ⇒ (𝑆 + 1)(𝑆 − 2) = 0 ⇒ 𝑆 = 2.
= 𝑎 + 𝑏.
𝟏
𝟏 𝐬𝐢𝐧 𝒙
66. For x > 0, 𝐋𝐭 [(𝐬𝐢𝐧 𝒙) + (𝒙) 𝒙 ] is 68. If a is a real number then
𝒙→𝟎
a) 0 b) -1 𝟏 𝒂/𝒏 𝒏
𝐋𝐭 [ ] is equal to
𝒏→∞ −𝒂/𝒏 𝟏
c) 1 d) 2
a) 𝑰𝟐×𝟐 b) 𝑶𝟐×𝟐
1
1 sin 𝑥 c) 1’ d) None.
Solution:- c) L = Lt (sin 𝑥)𝑥 + Lt (𝑥)
𝑥→0 𝑥→0
1 𝑎/𝑛
1 sin 𝑥 Solution:- b) A = [ ]=
log( ) −𝑎/𝑛 1
=0+ Lt 𝑒 𝑥 [ Lt (𝑑𝑒𝑐𝑖𝑚𝑎𝑙)) ∞ 1 𝑛 𝑎
𝑥→0 𝑥→0
[ ]
𝑛 −𝑎 𝑛
= 0]
Let n = 𝑟 cos 𝜃 , 𝑎 = 𝑟 sin 𝜃.
1
log( )
Lt 𝑥
𝑎
= 𝑒 𝑥→0𝑐𝑜𝑠𝑒𝑐 𝑥 ⇒ r = √𝑛2 + 𝑎2 ; 𝜃 = tan−1 (𝑛)
Applying L’ Hospital’s rule, we get 𝑟 cos 𝜃 sin 𝜃
𝐴= [ ] ⇒ 𝐴𝑛
−1 𝑛 − sin 𝜃 cos 𝜃
𝑥( 2 )
Lt 𝑥 Lt
sin 𝑥
.tan 𝑥 𝑟 𝑛 cos 𝑛𝜃 sin 𝑛𝜃
L=𝑒 𝑥→0 −𝑐𝑜𝑠𝑒𝑐 𝑥 cot 𝑥 = 𝑒 𝑥→0 𝑥 = 𝑛( )
𝑛 − sin 𝑛𝜃 cos 𝑛𝜃
= 𝑒 0 = 1. ⇒ 𝐴𝑛
𝑛
𝑎2 cos 𝑛𝜃 sin 𝑛𝜃
= (√1 + ) ( )
𝐥𝐨𝐠(𝟏+𝒂𝒙)−𝐥𝐨𝐠(𝟏−𝒃𝒙) 𝑛2 − sin 𝑛𝜃 cos 𝑛𝜃
67. The function f(x) = is
𝒙
not defined at x = 0. The value which 𝐴𝑛
should be assigned to & at x = 0, so that ⇒
𝑛
f(x) is continuous at x = 0, is 𝑛 cos 𝑛𝜃 sin 𝑛𝜃
a) a –b b) a +b 𝑎2 2
= (1 + 2 ) ( 𝑛 𝑛 )
c) 𝐥𝐨𝐠 𝒂 + 𝐥𝐨𝐠 𝒃 d) 𝑛 sin 𝑛𝜃 cos 𝑛𝜃

none 𝑛 𝑛
𝐴𝑛 0 0
Solution:- b) ⇒ Lt = (1 + 0) ( )
𝑛→∞ 𝑛 0 0
140
Solving Mathematical Problems

0 0 So, the series is absolutely convergent.


⇒ Lt 𝐴𝑛 = ( ) = 𝑂2×2 .
𝑛→∞ 0 0
𝐬𝐢𝐧 𝒙
(−𝟏)𝒏 70. 𝐋𝐭 [ ] is equal to
69. The series ∑ (𝒏+𝟏)𝒑 is 𝒙→𝟎 𝒙
a) 1 b) 0
a) Conditionally convergent if 0<p ≤ 1
c) does not exist d)
b) Absolutely convergent if p >1
none
c) Oscillatory if p ≤0
d) All above sin 𝑥
Solution:- b) since, │ │ <1
𝑥

Solution: d) Case I: p ≤ 0 sin 𝑥


⟹ tends to 1 forms the values that
𝑥
p = -q are less than one as x ⟶ 0. Thus, Lt [
sin 𝑥
]
𝑥→0 𝑥
Then the given series becomes =0

∑(−1)𝑛 (𝑛 + 1)𝑞 71. The net profit of an industry in a year is


given by y = 2ax -𝒙𝟐 , where x denotes the
= −2𝑞 + 3𝑞 − 4𝑞 + 5𝑞 − ⋯
input.
This is an Oscillatory series. Then the profit increases in relation to
x if
Case II:- 0 < p < 1 a) 0 < a < x b) x = a
This series is : c) a < x < 2a d) x <a
𝑑𝑦
1 1 1 1 Solution:- d) y = 2ax -𝑥 2 ⟹ 𝑑𝑥 = 2𝑎— 2𝑥
∑ 𝑈𝑛 = − 𝑝 + 𝑝 − 𝑝 + 𝑝 − ⋯
2 3 4 5
Profit will increase if 2a -2x > 0 i.e. x < a
By Leibnitz’s test this series is convergent.
𝟒+𝟑𝒂
1 1 1 72. If 𝒂𝟏 = 𝟏 𝒂𝒏𝒅 𝒂𝒏+𝟏 = 𝟑+𝟐𝒂𝒏 , 𝒏 ≥ 𝟏;
Also, ∣ ∑ 𝑈𝑛 ∣ = 2𝑝 + 3𝑝 + 4𝑝 + ⋯ 𝒏
then 𝐋𝐭 𝒂𝒏 = 𝒍. The l is equal to
𝒏→∞
This series is P-series and p ≤ 1. a) −√𝟐 b) √𝟐 c) 2 d) none
So, it is divergent. 4+3𝑙
Solution: b) 𝑙 = 3+2𝑙 ⟹ 𝑙 2 = 2 ⟹ 𝑙 = √2.
Case III: p >1 , the series is
Since lim 𝑎𝑛 = lim 𝑎𝑛+1 = 𝑙.
𝑛→∞ 𝑛→∞
1 1 1
∑ 𝑈𝑛 = − 𝑝 − 𝑝 − 𝑝 − ⋯ 𝒏𝑷 𝒔𝒊𝒏𝟐 (𝒏!)
2 3 4 73. 𝐋𝐭 , 𝟎 < 𝑷 < 𝟏, is equal to
𝒏→∞ 𝒏+𝟏
By Leibnitz’s test this series is convergent. a) 0 b) ∞
1 1 1
c) 1 d) none
∣ ∑ 𝑈𝑛 ∣ = 2𝑝 + 3𝑝 + 4𝑝 + ⋯

The series is also convergent.


141
Solving Mathematical Problems

𝑠𝑖𝑛2 (𝑛!) 𝑈𝑛 sin 1/𝑛


Solution : a) Lt 1 (∵ 0 < 𝑃 < ∴ lim = lim =1
𝑛→∞ 𝑛1−𝑃 (1++𝑛) 𝑛→∞ 𝑉𝑛 𝑛→∞ 1/𝑛

1) So, ∑ 𝑉𝑛 𝑑𝑖𝑣𝑒𝑟𝑔𝑒𝑠.
𝑠𝑎𝑚𝑒 𝑟𝑒𝑎𝑙 𝑛𝑜. 𝑖𝑛 [0, 1]
= [∵ 1 − 𝑃 > 𝑛0] ∴ By limit comparison test, ∑ 𝑈𝑛 is also
∞ diverges.
=0
76. If 𝒇𝒏 (𝒙) be a function defined on [0, 1]
𝝅
74. The series ∑∞
𝒏=𝟏 𝐬𝐢𝐧 𝒏𝑷 is
and then the sequence {𝒇𝒏 (𝒙) }, where
a) Convergent for all values of P 𝒇𝒏 (𝒙) = 𝒙𝒏 , is __________
b) Convergent for p ≤ 1 and divergent a) Uniformly convergent in [0, 1]
for P >1 b) Uniformly convergent in (0, 1)
c) Convergent for P > 1 and c) Uniformly convergent in ℝ
divergent for p ≤1 d) None
d) Divergent for all values of P. Solution:- b) lim 𝑓𝑛 (𝑥) lim 𝑥 𝑛 =
𝑛→∞ 𝑛→∞
𝜋 𝜋 𝜋3 0, 𝑤ℎ𝑒𝑟𝑒 0 ≤ 𝑥 < 1
Solution: c) 𝑈𝑛 = sin 𝑛𝑃 = − 3!𝑛3 𝑃 + {
𝑛𝑃 1, 𝑤ℎ𝑒𝑟𝑒 𝑥 = 1.
1
⋯ 𝑎𝑛𝑑 𝑉𝑛 = 𝑛𝑃
Then the sequence is point wise convergent
𝑈𝑛 in [0, 1] and uniformly convergent in (0, 1).
∴ Lt = 𝜋≠0
𝑛→∞ 𝑉𝑛
77. Which of the following functions is
∑ 𝑉𝑛 is convergent for P > 1 and divergent uniformly continuous on the domain as
for P≤ 1 stated?
(a) f(x) = 𝒙𝟐 , x ∈ ℝ (b) f(x) =
∴ ∑ 𝑈𝑛 is convergent for P > 1 and divergent 𝟏
, 𝒙 ∈ [𝟏, ∞) (c) f(x) =
for P ≤ 1. 𝒙
𝝅 𝝅
𝐭𝐚𝐧 𝒙 , 𝒙 ∈ (− 𝟐 , 𝟐 ) (d) f(x) =
75. Let 𝑼𝒏 = 𝐬𝐢𝐧(𝟏/𝒏) and consider the
[x], x ∈ [0, 1]
series ∑ 𝑼𝒏 .
Which of the following statement is 1
Solution:- (b) f(x) = 𝑥 is uniformly
true?
continuous in [1, ∞).
a) ∑ 𝑼𝒏 is convergent

b) 𝑼𝒏 ⟶ 𝟎 𝒂𝒔 𝒏 ⟶ ∞
c) ∑ 𝑼𝒏 is divergent SET, COMBINATORICS,
PROBABILITY
d) ∑ 𝑼𝒏 is absolutely convergent.
1 78. The number of non–empty of a set
Solution:- c) 𝑈𝑛 = sin(1/𝑛) , 𝑉𝑛 = 𝑛
consisting 6 elements is

142
Solving Mathematical Problems

(a) 63 (b) 64 (a) 50 ≤ n ≤ 60 (b) 49 ≤n ≤59


(c) 65 (d) none (c) 40 ≤ n ≤60 (d) 49 ≤ n ≤ 60

Solution:- (a) The no. of non-empty Sol. (d) No of friends = n


subset of a set consisting n element is
Months (holes) (m) = 12.
𝑛
=2 .
By extended pigeon –Hole principle,
79. Le A and B be two sets having 7
𝑛−1
common elements, then the number of [ ]+1=5
𝑚
elements common to 𝑨 × 𝑩 𝒂𝒏𝒅 𝑩 × 𝑨
is 𝑛−1
⟹[ ]+1 = 5
(a) 0 (b) 𝟐𝟕 12
(c) 49 (d) none.
⟹ 49 ≤ 𝑛 ≤ 60.
Sol. (c) The no. of common elements to ×
82. Let U be the set of positive integers not
𝐵 𝑎𝑛𝑑 𝐵 × 𝐴 is = 𝑛2 .
exceeding 1000 then the number of
80. The number of squares that can be sets of such integers which are not
formed on a chess board is divisible by 3, 5, 7 is
(a) 204 (b) 224 (a) 255 (b) 456
(c) 230 (d) None (c) 457 (d) 256

Solution:- (a)A chess board has 9 equi- Sol. (c)


spaced horizontal and vertical lines we
A : integers divisible by 3
need to choose two consecutive
horizontal and vertical lines to make a 1 × B : integers divisible by 5
1 square from among these which is done
in 8 × 8 = ways. C : integers divisible by 7
1000 1000
Similarly, 2 × 2 square needs 3 n(A)= [ ] = 333, 𝑛(𝐵) = [ ]=
3 5
consecutive horizontal and vertical lines, 1000
200, 𝑛(𝐶) = [ ] = 142
i.e. in 7 × 7 = 72 ways. 7

1000
∴ Total number of squares = 82 + 72 + 𝑛(𝐴 ∩ 𝐵) = [ ] = 66, 𝑛(𝐵 ∩ 𝐶)
15
62 + 52 + ⋯ + 12 1000
=[ ] = 28, 𝑛(𝐴 ∩ 𝐶)
8 35
8(8 + 1)(16 + 1) 1000
= ∑ 𝑖2 = = 204 =[ ] = 47
6 21
𝑖=1

81. How many friends must you have to 1000


𝑛(𝐴 ∩ 𝐵 ∩ 𝐶) = [ ] = 9.
guarantee that it least five of them will 105
have birthdays in the same month?

143
Solving Mathematical Problems

By inclusion –exclusion principle, 𝟏 𝟏


(a) 𝟒 (b) 𝟕
𝟏
n(A ∪B ∪C) = n(A)+ n(B)+ n(C) –n(A∩ (c) 𝟖 (d) None.
B) –n (B∩ C) –n(A∩ C) +n(A∩ B∩C) =
543. Sol. (a) The unit place of 7𝑘, where k is an
integer will be 9, 3, 1, 7.
So, required answer is = 𝑛(𝐴 ∪ 𝐵 ∪ 𝐶)𝑐
71 = 7, 72 = 49, 73 = 343, 74
= 1000 − 𝑛(𝐴 ∪ 𝐵 ∪ 𝐶) = 457. = 2407, … , 77
≡ 1 (𝑙𝑎𝑠𝑡 𝑑𝑖𝑔𝑖𝑡)
83. A and B toss a fair win each
simultaneously 50 times. The Now 7𝑚 + 7𝑛 is divisible by 5 if m = 3 or
probability that both of them will not 7 and n = 7 or 3.
get tail in the same toss is
𝟑 𝟓𝟎 𝟐 Also, 7𝑚 + 7𝑛 is divisible by 5 if m = 9 or
(a) (𝟒) (b) (𝟕) 𝟓𝟎
1 and n = 1 or 9.
𝟏 𝟓𝟎
(c) (𝟖) (d) none. Now, 7𝑚 + 7𝑛 is divisible by 5 only when
the last digit in the unit place is zero.
Sol. (a) There are four possibilities in
each toss, i.e.. ∴ Required probability is =
4 1
= 22 = 4.
1
24
A = tail B = Head
85. The total number of subsets of a set of
A = Head B = tail 12 elements are
(a) 144 (b) 𝟏𝟐𝟏𝟐
A = Head B = Head (c) 47900 (d)
A = tail B= tail 4096

Total number of cases = 450 Sol. (d) Answer = 212 = 4096.

In each case there are 3 possibilities of 86. The total number of non-empty even
not getting tail on the same toss, subsets of a set having n elements is
(a) 𝟐𝐧−𝟏 (b) 𝟐𝐧−𝟏 −
∴ Favourable cases = 350 . 𝟏 (c) 𝟐𝐧
(d) 𝟐𝐧+𝟏 + 𝟏
3 50
Hence the required probability is (4) .
Sol. (b) If a set having n elements then
84. If the integers m and n are chosen at total no. of subsets is = 2𝑛
random between 1 to 100.
Total no. of even subsets is = 2𝑛−1 .
Then the probability that a number
of the form 𝟕𝒎 + 𝟕𝒏 is Excluding the empty set 𝜙, we have
2𝑛−1 − 1 as total number of non-empty
subsets.

144
Solving Mathematical Problems

87. A bar of unit length is broken into 3 𝟏 𝟏


(a) 𝟐 (b) 𝟒
parts x, y, z. The probability that a 𝟏
(c) 𝟔 (d) none
triangle can be formed from the
resulting parts is 𝑟
𝟏 𝟏 Sol. (b) AB = r ; CD = 2
(a) 𝟐 (b) 𝟑
𝟏 n(S) = the area of the circle of radius r =
(c) 𝟒 (d) None
𝛱(𝑟 2 )
Sol. (c) Let z = 1- (x +y) 𝑟
n(E) = the area of the circle of radius 2 =
x > 0, y > 0, (x +y) <1. 𝑟 2
𝛱 (2)
1 1
The sample space is 2 𝑋 ∣ 𝑋 ∣= 2 = interior
𝑟 2
𝑛(𝐸) 𝛱( ) 1
of a unit triangle with unit legs. ∴ P(E) = 𝑛(𝑠) = 2
= 4.
𝛱𝑟 2

Then two conditions are needed to satisfy


90. Two finite sets have m and elements.
to from a triangle:
The total number of subsets of the first
(a) The sum of the two sides is greater set is 56, more than the total number of
than the third side subsets of the second set, The value of
(b) The difference between any two m, n are
sides is smaller than the third one. (a) 7, 6 (b) 6, 3
(c) 5, 1 (d) none
The area of the new triangle domain is =
1 1
×4=8
1 Sol. (b) We know that 2𝑚 − 2𝑛 = 56.
2

1 By trial, m = 6, n = 3
8 1
∴ Prob. is = 1 = 4.
2 So, (b) is correct.

88. Total number of non-negative integer 91. Total number of polynomials of the
solutions of 𝒙𝟏 + 𝒙𝟐 + 𝒙𝟑 = 𝟏𝟎 is form 𝒙𝟑 + 𝒂𝒙𝟐 + 𝒃𝒙 + 𝒄 = 𝟎 which is
(a) 𝟏𝟎𝑪𝟐 (b) 𝟏𝟎𝑪𝟑 divisible by 𝒙𝟐 + 𝟏, where 𝒂, 𝒃, 𝒄 ∈
(c) 𝟏𝑪𝟐 (d) none. {𝟏, 𝟐, 𝟑, … , 𝟏𝟎} is

Sol. (c) 𝑛 + 𝑟 − 1𝐶𝑟−1 = 10 + 3 − (a) 15 (b) 10 (c) 5 (d) none


1𝐶3−1 = 12𝐶2
Solution: (b)
89. A point is selected at random from the
Take 𝑖 3 + 𝑎𝑖 2 + 𝑏𝑖 + 𝑐 = 0
interior of a circle.
The probability that the point is and −𝑖 3 + 𝑎𝑖 2 − 𝑏𝑖 + 𝑐 = 0
closed to the centre than the
implying (𝑏 − 1)𝑖 + (𝑐 − 𝑎) = 0
boundary of the circle is

145
Solving Mathematical Problems

Thus, b = 1 and a = c. 𝒅𝒚
93. The differential equation │ 𝒅𝒙 │ =
So, total number of polynomials equals to │𝒚│, 𝒚(𝟎) = 𝟏, 𝒚 ≠ 𝟎 has
(10
1
) = 10. (a) Unique solution (b) non-
trivial solution (c) finite
number of solution (d) infinite
number of solution
92. Let 𝒙 + 𝒚 = 𝟐𝒂, where a is a constant,
and all values of x lying between 0 and Solution:- (b) The equation is │y│ = 𝑒 𝑥+𝑐
2a are equally likely. Then the chance
𝟑𝒂𝟐 y(0) = 1, gives 𝑒 𝑐 = 1 = 𝑒 0 ⟹ 𝑐 = 0.
that 𝒙𝒚 > is
𝟒
So, y = ex and y = - ex are two solutions.
(a) ½ (b) 1/3 (c) ¼ (d) none
So, the ODE has non-trivial solution.
Solution:- (a)
[Note: Trivial solution:- A solution in
Let OP = a, AP = x, AQ = y, and x + y = which ever variable has zero value is
2a. called trivial solution.
Now, AB2 = AP, PQ = xy. Infinite solution:- If the constant(s) of the
solution of the ODE remain undetermined
Also, MP = MO, and NO = NQ.
then the equation has infinite number of
3 solutions.]
If A lies in MN then AB > a√4

𝟑𝒂𝟐 𝑴𝑵 𝟏
Thus, 𝑃 (𝒙𝒚 > )= =𝟐 𝒅𝟐 𝒚
𝟒 𝑨𝑩
94. Number of solution of the ODE +
𝒅𝒙𝟐
𝜫
𝟒𝒚 = 𝟎, 𝒚(𝟎) = 𝟎, 𝒚 ( 𝟒 ) = 𝟏 is
(a) 0 (b) 1 (c) 2 (d) None

Solution:- (b)
𝑑2 𝑦
+ 𝜆𝑦 = 0 (𝜆 > 0) has the general
𝑑𝑥 2
solution

y = 𝑐1 cos √𝜆𝑥 + 𝑐2 sin √𝜆𝑥 ; where λ > 0.


𝑑2 𝑦
So, 𝑑𝑥 2 + 4𝑦 = 0 has the solution

y = 𝑐1 cos 2𝑥 + 𝑐2 sin 2𝑥

DIFFERENTIAL EQUATIONS 𝑦(0) = 0 ⟹ 𝑐1 = 0

146
Solving Mathematical Problems

𝛱 97. One of the integrating factors of the


𝑦 ( ) = 1 ⟹ 𝑐2 = 1.
4 ODE
(𝒚𝟐 − 𝟑𝒙𝒚)𝒅𝒙 + (𝒙𝟐 − 𝒙𝒚)𝒅𝒚 = 𝟎
∴ y = sin 2x is the unique solution of the
is
given differential equation. 𝟏 𝟏
(a) (𝒙𝟐 𝒚𝟐 ) (b) (𝒙𝟐 𝒚)
𝟏 𝟏
(c) (𝒙𝒚𝟐 ) (d) (𝒙𝒚)
𝒅𝒚
95. The solution of the ODE 𝒅𝒙 = 𝒙,
Solution:- (b)
𝒚(𝟎) = 𝟎 is
(a) Unbound (b) positive M = 𝑦 2 − 3𝑥𝑦
(c) negative (d) zero
N = 𝑥 2 − 𝑥𝑦
Solution:- (b)
As M dx + N dy = 0 is homogeneous.
∫ 𝑑𝑦 = ∫ 𝑥 𝑑𝑥
1 1
An I.F. is = = (−2𝑥 2 𝑦).
𝑀𝑥+𝑁𝑦
⟹ 𝑦 = 𝑥2 + 𝑐
1
𝑦(0) = 0 𝑔𝑖𝑣𝑖𝑛𝑔 𝑐 = 0 So, 𝑥 2 𝑦 is an I. F. by ignoring the constant.

∴ y = 𝑥 2 is the solution which is always


positive.
98. General solution of (𝒙𝒚𝒔𝒊𝒏(𝒙𝒚) +
𝒄𝒐𝒔(𝒙𝒚))𝒚𝒅𝒙 + (𝒙𝒚𝒔𝒊𝒏(𝒙𝒚) −
𝒅𝟐 𝒚 𝒄𝒐𝒔(𝒙𝒚))𝒅𝒚 = 𝟎 is
96. Number of solutions of the ODE =
𝒅𝒙𝟐
𝟎, 𝒚(𝟎) = 𝟏 is (a) ysin(xy) = cx (b) xsec(xy) = cy
(a) 0 (b) 1 (c)
infinite no. of solutions (c) ytan(xy) = cx (d) none.
(d) none Solution: (b)
Solution:- (c) xysin(xy)(ydx+xdy)+cos(xy)(ydx-
𝑑2 𝑦 xdy)=0
= 0 has the solution y = 𝑐1 𝑥 + 𝑐2
𝑑𝑥 2
implies tan(xy).d(xy)+dx/x – dy/y = 0
y(0)= 1 ⟹ 𝑐2 = 1.
implies log⃒secxy⃒ + log ⃒x/y⃒ = c’
∴ y = 𝑐1 𝑥 + 1, 𝑐1 is arbitrary constant.
implies xsec(xy) = cy
⟹ The ODE has infinite number of
solutions.
99. The solution of the curve y = f(x)
satisfying the differential equation
147
Solving Mathematical Problems

𝒅𝒚
√(𝒙 − 𝒚) = |𝒙𝟐 − 𝒚𝟐 | and passing 𝑥𝑑𝑦 − 𝑦𝑑𝑥 1 𝑦 2
𝒅𝒙 = √1 − ( ) 𝑑𝑥
𝑥2 𝑥 𝑥
through the point (1,0) is

(a) (𝒙 − 𝟏) = 𝒚𝟐 (𝒙𝟐 − 𝒚𝟐 ) 𝑦 1 𝑦 2
→ 𝑑 ( ) = √1 − ( ) 𝑑𝑥
(b) 𝒚𝟐 = 𝒙 − 𝟏 𝑥 𝑥 𝑥
𝟏
(c) (𝟐𝒙 − 𝟑) + (𝒙𝟐 −𝒚𝟐 ) = 𝟎 𝑦
𝑑 (𝑥 ) 1
(d) None → = 𝑑𝑥
2 𝑥
√1 − (𝑦)
Solution: (c) 𝑥
𝑦
𝑑𝑦 → 𝑠𝑖𝑛−1 ( ) = log(𝑥) + 𝑐
√(𝑥 − 𝑦) = |𝑥 2 − 𝑦 2 | 𝑥
𝑑𝑥

𝑑(𝒙𝟐 − 𝒚𝟐 )
→ = 𝑑𝑥
2(𝒙𝟐 − 𝒚𝟐 )2

−1
→ = 2𝑥 + 𝑐
(𝒙𝟐 − 𝒚𝟐 )

Which passes through (1,0). ISI SUBJECTIVE SAMPLE


Thus, - 1 = 2x + c which gives c = - 3. PAPER WITH SOLUTIONS

Hence the curve is SET – 1


1
(2𝑥 − 3) + =0 1. Find all real numbers satisfying 𝟔𝒙 +
(𝑥 2 − 𝑦2)
𝟐𝟐𝒙 + 𝟐𝟒𝒙 − 𝟑𝟔𝒙 − 𝟏𝟔𝒙 = 𝟏.

Ans:- Rewrite the given relation as:


100. The solution of the equation 𝒙𝒅𝒚 −
𝒚𝒅𝒙 = √(𝒙𝟐 − 𝒚𝟐 )𝒅𝒙 subject to the 6𝑥 + 4𝑥 − 36𝑥 + 24𝑥 − 16𝑥 = 1
condition y(1)=0, is Let 6𝑥 = 𝑎, 4𝑥 = 𝑏, we have

(a) y = xsin(logx) a+ b -𝑎2 + 𝑎𝑏 − 𝑏 2 = 1


(b) y = x2sin(logx)
⇒𝑎2 − 𝑎𝑏 + 𝑏 2 − 𝑎 − 𝑏 + 1=0
(c) y = x2(x – 1)
(d) None ⇒ 2𝑎2 − 2𝑎𝑏 + 2𝑏 2 − 2𝑎 − 2𝑏 + 2=0

Solution: (a) ⇒ (𝑎2 − 2𝑎𝑏 + 𝑏 2 )+ (𝑎2 − 2𝑎 + 1)+ (𝑏 2 −


2𝑏 + 1)=0

148
Solving Mathematical Problems
𝑥
⇒ (𝑎 − 𝑏)2 + (𝑎 − 1)2 + (𝑏 − 1)2 =0 Ans:- Define , F (x) = ∫0 𝑓(𝑡)𝑑𝑡 and G
𝑥
∴ a= 1 and b= 1 when a= b. (x)= ∫0 (𝑓(𝑡))2 𝑑𝑡

⇒ 4𝑥 = 1 𝑎𝑛𝑑 6𝑥 = 1, giving x= 0 only. Since f: (0, ∞) ⟶ (0, ∞)

2. Two boxes contain between them 65 we have F (x)> 0 ∀ 𝑥 > 0


balls of several different sizes. Each ball is 1 1
white, black, red, or yellow. If you take Also, 𝐺(𝑥) = {𝐹(𝑥)}2 , from the given
2 𝑥
any five balls of the same colour, at least condition on differentiation, we have
two of them will always be of the same 1 1 1
size (radius). Prove that there are at least 𝐺′(𝑥)= 𝑥 . 2𝐹(𝑥). 𝐹 ′ (𝑥) − 𝑥 2 (𝐹(𝑥))2
2
three balls which lie in the same box, have 1 2
the same colour and are of the same size. This means that (𝐹(𝑥))2=𝑥 𝐹(𝑥) 𝐹 ′ (𝑥) −
2
1
Ans:- we will make repeated use of pigon– 𝑥 2 (𝐹(𝑥))2
hole- principle (PHP). As there are 65 balls
1 𝑥𝐹′ (𝑥) 2 𝑥𝐹′ (𝑥)
and 2 boxes , one of these boxes must or, 2 ( ) =2 −1
𝐹(𝑥) 𝐹(𝑥)
65
contain at least [ 2 ]+1 = 33 balls.
Solving this equation as a quadratic in
𝑥𝐹′ (𝑥)
Consider that box, now we have four colours we have
𝐹(𝑥)
(white, black, red, yellow) and hence there
33 𝑥𝐹′ (𝑥)
must be at least ( )+1 = 9 balls of the same = 2 ± 2 = 𝑘(say)
4 𝐹(𝑥)
colour.
𝑑𝐹(𝑥) 𝑑𝑥
On integration, we obtain ∫ =𝑘 ∫
𝐹(𝑥) 𝑥
There can be at most 4 different sizes
available for these 9 balls of the same ⇒ ln 𝐹(𝑥)= klnx + ln𝜆 ⇒ 𝐹(𝑥)= 𝜆𝑥 𝑘
colour, For if there were 5 (or
more)different sizes, then collection of 5 ⇒ f (x)=𝜆k𝑥 𝑘−1 ⇒ f (1) = 1
balls, all of different sizes, would not satisfy
the given property. ⇒ 𝜆k=1
1−√2
Thus of these 9 balls there must be at least 3 ∴ f(x)= 𝑥 𝑘−1 = 𝑥1+√2 /𝑥
balls of the same size.

3. Find all continuous function f : (0,


√[𝒙]+√{𝒙}
∞)⟶(0, ∞) ∋ f (1)= 1 and 4. Let x ≥ 𝟏, 𝒇(𝒙) = , where [.]
√𝒙
𝟐 denotes G.I.F. and { } denotes fractional
𝟏 𝒙 𝟐
𝟏 𝒙
∫ (𝒇(𝒕)) 𝒅𝒕 = (∫ 𝒇 (𝒕)𝒅𝒕) part. Determine the smallest number k ∋
𝟐 𝟎 𝒙 𝟎
f(x)≤ 𝒌 for each x ≥ 𝟏

Ans:- Let x = a+ b where a= [x], b= {x}

149
Solving Mathematical Problems

f(x)=
√𝑎+√𝑏 When a ∊ [1, 3] , the area is a quadrilateral
√𝑎+𝑏 𝑎 𝑎 𝑎 𝑎+2 4−𝑎
with vertices at ( 3 , 3), (2 , 0), ( , ) and
3 3
𝑎+𝑏+2√𝑎𝑏 2√𝑎𝑏
(f(𝑥))2 = =1+ (1, 1)
𝑎+𝑏 𝑎+𝑏
1 (𝑎−2)2
Using AM≥ 𝐺𝑀, ≤ 1 + 1 ⇒ 𝑓(𝑥) ≤ √2. So, the net area is 3 − which also does
6
not exceed 1/3.
5. Solve the equation (√𝟐 + √𝟐)𝒙 +
(√𝟐 − √𝟐)𝒙 = 𝟐𝒙 When a ∊ [3, 4], the area is same as when a
∊ [0, 1]
√2 𝜋 𝜋
Ans:- 1+ = 1 + 𝑐𝑜𝑠 4 = 2 cos2 8
2 8. Determine a value of the parameter 𝜃 ∋
2+ √2 𝑥⁄ 2− √2 𝑥⁄ f(x)= 𝐜𝐨𝐬𝟐 𝒙 + 𝐜𝐨𝐬𝟐 (𝒙 + 𝜽) −
( ) 2 +( ) 2
4 4 𝒄𝒐𝒔𝒙𝒄𝒐𝒔(𝒙 + 𝜽).
𝜋 𝜋
= (𝑐𝑜𝑠 8 )𝑥 + (𝑠𝑖𝑛 8 )𝑥 Is a constant function of x?

⇒ x= 2 Ans:- f(x)=sin2 𝜃 + (2 𝑐𝑜𝑠𝜃 −


1)(cos2 𝑥𝑐𝑜𝑠𝜃 − 𝑠𝑖𝑛2𝑥𝑠𝑖𝑛𝜃)
6. Let f(x) be a polynomial with real
coefficient for which the equation f(x)= x The function f(x)is constant when
has no real solution. Prove that the (2 𝑐𝑜𝑠𝜃 − 1)= 0
equation f(f(x))= x has no real solution, 𝜋 3
either. i.e. 𝜃= and the constant value is .
3 4

𝒏𝑪𝟎 𝒏𝑪𝟏 𝒏𝑪𝟐 𝒏𝑪


Ans:- Suppose, if possible that f(f(a))=a, let 9. If − + − ⋯ + (−𝟏)𝒏 𝒏+𝟐
𝒏
=
𝟐 𝟑 𝟒
b= f(a), then f(b)=a by hypothesis b ≠ a. 𝟏
, then what is the value of n?
Assume that a < b then f(a)- a > 0 and f(b)-b 𝟏𝟗𝟗𝟗×𝟐𝟎𝟎𝟎
< 0. So, by intermediate value theorem f(x) -
Ans:- (a)
x = 0 should be a root between (a, b).
(1 − 𝑥)𝑛 = 𝑐0 − 𝑐1 𝑥 + 𝑐2 𝑥 2 − ⋯ +
But this contradicts our assumption. Hence,
(−1)𝑛 . 𝑐𝑛 𝑥 𝑛
f(f(x)) can have no real solution.
⟹ x(1 − 𝑥)𝑛 = 𝑐0 𝑥 − 𝑐1 𝑥 2 + 𝑐2 𝑥 3 … +
7. Let a ∊ [0, 4]. Prove that the area
(−1)𝑛 . 𝑐𝑛 𝑥 𝑛+1
bounded by the curves y=1 -|x-1| and y=
|2x -a| can’t exceed 1/3. Integrating between the limits 0 and 1,
Ans:- when a ∊ [0, 1].the area is a triangle 𝑥2 𝑥3 𝑥4
We get [𝑐0 − 𝑐1 + 𝑐2 − ⋯+
formed by (0, 0), (1⁄2 , 0) and (1, 1) with 2 3 2
𝑥 𝑛+2 1 (1−𝑥)𝑛+1 1
area equals 1/4. (−1)𝑛 . 𝑐𝑛 ] = [−𝑥. 𝑛+1 ] 0 +
𝑛+2 0
1 (1−𝑥)𝑛+1
∫0 1. 𝑛+1
𝑑𝑥

150
Solving Mathematical Problems
(1−𝑥)𝑛+2 1 Log u=cos y logx
= 0 − [(𝑛+1)(𝑛+2)] 10 = (𝑛+1)(𝑛+2)
1 𝑑𝑢 1 𝑑𝑦
𝑐0 𝑐1 𝑐𝑛 1 . = 𝑐𝑜𝑠𝑦. 𝑥 − 𝑙𝑜𝑔𝑥. 𝑠𝑖𝑛𝑦. 𝑑𝑥
∴ − + ⋯+ (−1)𝑛 𝑛+2 = (𝑛+1)(𝑛+2)
𝑢 𝑑𝑥
2 3
𝑑𝑢 𝑐𝑜𝑠𝑦 𝑑𝑦
1 1 ∴𝑑𝑥 = 𝑥 𝑐𝑜𝑠𝑦 ( − 𝑙𝑜𝑔𝑥. 𝑠𝑖𝑛𝑦. 𝑑𝑥 ) …..(1)
𝑥
Given, (𝑛+1)(𝑛+2)
= 1999×2000
v=𝑦 𝑐𝑜𝑠𝑥 => 𝑙𝑜𝑔𝑣 = 𝑐𝑜𝑠𝑥𝑙𝑜𝑔𝑦
So n = 1998.
𝑑𝑣 1 𝑑𝑦
= 𝑦 𝑐𝑜𝑠𝑥 {−𝑙𝑜𝑔𝑦. 𝑠𝑖𝑛𝑥 + 𝑐𝑜𝑠𝑥. 𝑦 . 𝑑𝑥 }
10. For what value of m the sum 𝑑𝑥
𝟏𝟎 𝟐𝟎 𝒑 ………………(2)
∑𝒎
𝒊=𝟎( 𝒊 )(𝒎−𝒊) , 𝒘𝒉𝒆𝒓𝒆 (𝒒) = 𝟎 if p< q, is
maximum? Now, u+ v=1
𝑑𝑢 𝑑𝑣
Ans:- (c) + = 0.
𝑑𝑥 𝑑𝑥

∑10
𝑖=0 10𝐶𝑖 = 20𝐶𝑚−𝑖 = 10𝐶0 20𝐶𝑚 + 1 𝑑𝑦 𝑑𝑦
⤇𝑦 𝑐𝑜𝑠𝑥 . 𝑐𝑜𝑠𝑥. 𝑦 . 𝑑𝑥 − 𝑥 𝑐𝑜𝑠𝑦 . 𝑙𝑜𝑔𝑥𝑠𝑖𝑛𝑦 𝑑𝑥 =
10𝐶1 20𝐶𝑚−1 + ⋯ + 10𝐶𝑚 20𝐶0 ……..(1) 𝑐𝑜𝑠𝑦
𝑦 𝑐𝑜𝑠𝑥 . 𝑙𝑜𝑔𝑦. 𝑠𝑖𝑛𝑥 − 𝑥 𝑐𝑜𝑠𝑦 . 𝑥
(1 + 𝑥)20 = 20𝐶0 + 20𝐶1 𝑥 + ⋯ +
𝑐𝑜𝑠𝑦
20𝐶𝑚−1 𝑥 𝑚−1 + 20𝐶𝑚 𝑥 𝑚 + ⋯ + 20𝐶20 𝑥 20 𝑑𝑦 𝑦 𝑐𝑜𝑠𝑥 .𝑙𝑜𝑔𝑦.𝑠𝑖𝑛𝑥−𝑥 𝑐𝑜𝑠𝑦 .
𝑥
⤇𝑑𝑥 = 1 .
𝑦 𝑐𝑜𝑠𝑥 .𝑐𝑜𝑠𝑥. −𝑥 𝑐𝑜𝑠𝑦 .𝑙𝑜𝑔𝑥.𝑠𝑖𝑛𝑦
……………….(2) 𝑦

(1 + 𝑥)10 = 10𝐶0 + 10𝐶1 𝑥 + ⋯ + 10𝐶10 𝑥10 2. Find the inverse of the following matrix
……………….(3) with R1 = (c0 , c1 , c2 , c3 ); R2 = (c2 , c3 , c0 ,
c1 );
Multiplying (2) and (3) and equating the co-
efficient of 𝑥 𝑚 , we get R3 = (c3 , -c2 , c1 , -c0 ); R4 = (c1 , -c0 , c3 , -c2
),
10𝐶0 20𝐶𝑚 + 10𝐶1 20𝐶𝑚−1 + ⋯ +
𝟏+√𝟑 𝟑+√𝟑
10𝐶𝑚 20𝐶0 = 30𝐶𝑚 , 30𝐶𝑚 will be greatest where 𝒄𝟎 = , 𝒄𝟏 = , 𝒄𝟐 =
𝟒√𝟐 𝟒√𝟐
when m= 15. 𝟑−√𝟑 𝟏−√𝟑
, 𝒄𝟑 =
𝟒√𝟐 𝟒√𝟐

Ans:- Put 𝑐0 2 + 𝑐1 2 + 𝑐2 2 + 𝑐3 2 = 1
ISI SUBJECTIVE SAMPLE 1 3 −3
𝑐0 𝑐3 = − 16 , 𝑐2 𝑐1 = 16 , 𝑐0 𝑐2 = ,
PAPER WITH SOLUTIONS 16
−3
𝑐1 𝑐3 = .
16
SET – 2
Here 𝐴2 = 𝐼 ⤇ 𝐴 = 𝐴−1
𝒅𝒚
1. Find 𝒅𝒙 𝒊𝒇 𝒙𝒄𝒐𝒔𝒚 + 𝒚𝒄𝒐𝒔𝒙 = 𝟏

Ans:- u= 𝑥 𝑐𝑜𝑠𝑦

151
Solving Mathematical Problems

3. True/False: If f is a continuous function 2


⇒ 15𝑦 2 = 2 ⇒ y= ±√15
on ℝ ∋ f(x+ y)= f(x)+ f(y) ∀ x, y∊ ℝ.

Then f(0)= 0 ∀ x∊ℝ. 3 2 3


∴ x= 2 (±√15) = ±√10 ∴ Max (z)=
Ans:- False
3 2 5
√ + √15 = .
f(x+ y)= f(x) .f(y) 10 √30

Let f(x) = 𝑎 𝑥 , f(y)=𝑎 𝑦 ∀ x, y∊ ℝ

f(x+ y)= 𝑎 𝑥 + y 6. For any positive a, b prove that (𝒂 +


𝟏 𝟐 𝟏
) + (𝒃 + )𝟐 ≥ 𝟖.
f(0)=1 ≠0. 𝒂 𝒃

𝟏⁄ Ans: AM ≥ GM
4. 𝐥𝐭 (𝒄𝒐𝒔𝒙) 𝒙𝟐 =?
𝒙⟶𝟎 1 1
(𝑎 + 𝑎)2 + (𝑎 + 𝑎)2 ≥
1⁄
Ans:- (𝑐𝑜𝑠𝑥) 𝑥2 = 𝑘, 𝑠𝑎𝑦 1 1
2√(𝑎 + 𝑎)2 + (𝑏 + 𝑏)2
1
∴ lnk= 𝑥 2 𝑙𝑛(𝑐𝑜𝑠𝑥)
1 𝑎 𝑏
≥ 2(ab+𝑎𝑏 + 𝑏 + 𝑎)
𝑙𝑛 cos 𝑥 0
∴ lt (𝑙𝑛𝑘) = lt ( )=
𝑥⟶0 𝑥⟶0 𝑥2 0 1
− tan 𝑥 0 −𝑐𝑜𝑠𝑒𝑐 2 𝑥 1 ≥ 2(2+2) [∵ ab + 𝑎𝑏 ≥ 2]
lt (0) = lt = −2
𝑥⟶0 2𝑥 𝑥⟶0 2

1
7. Let A & B be two invertible n ×n real

∴ lt 𝑘 = 𝑒 2 matrices. Assume that A + B is invertible.
𝑥⟶0

Show that 𝑨−𝟏 + 𝑩−𝟏 is also invertible.

5. Maximize x+ y subject to the condition Ans:- A, B are invertible


that 2𝒙𝟐 + 𝟑𝒚𝟐 ≤ 𝟏. A + B is invertible.
𝑥2 𝑦2
Ans:- 1⁄ + 1⁄ ≤ 1 |A||𝐴−1 + 𝐵 −1||B| = |B+ A|≠0
2 3
⇒ |𝐴−1 + 𝐵 −1| ≠0 as |A|, |B|≠ 0
Let z= x+ y
𝑑𝑦 𝑑𝑦 2𝑥 ⇒ 𝐴−1 + 𝐵 −1is invertible.
Now, 4x + 6y𝑑𝑥 = 0 ⇒𝑑𝑥 = − 3𝑦

2𝑥
At the touching point. -3𝑦 = -1
8. Let A be n ×n orthogonal mtx where A
3𝑦 is even and suppose |A| =-1. S.T. |I−A|= 0,
⇒ 2x= 3y and 2𝑥 2 + 3𝑦 2=1 ⇒ 2 ( 2 )2 +
where I denotes n ×n identity mtx.
(3𝑦 2 ) = 1
Ans:- 𝐴−1 = 𝐴𝑇 |A| = −1
152
Solving Mathematical Problems
1
⇒𝜆= 𝜆 ⇒ ∏𝑛𝑖=1 𝜆𝑖 = −1 then at least 10. If 0 < x < 𝜋, and f(x)
one 𝜆𝑖 = −1
=√𝟐 + √𝟐 + √𝟐 + ⋯ + √𝟐(𝟏 + 𝐜𝐨𝐬𝐱) there
⇒ 𝜆= ± 1
being n number of 2’s, then 𝐋𝐭 𝒇(𝒙) = ⋯
∴ Characteristics Equation is |𝜆𝐼𝑛 − 𝐴|= 0 𝒏→∞

⇒|𝐼𝑛 − 𝐴 |= 0 for 𝜆𝑖 = +1 Ans:- (b)

Let y= Lt 𝑓(𝑥)
𝑛→∞

9. If f(x+ y)=f(x).f(y)for all x and y f(1)= 2 𝑤ℎ𝑒𝑛 𝑛 → ∞, 𝑦 2 = 2 + 𝑦


and 𝜶𝒏 = 𝒇(𝒏), 𝒏 ∊ 𝑵, then find equation
of the circle having (𝜶𝟏 , 𝜶𝟐 ) and (𝜶𝟑 , 𝜶𝟒 ) ⟹ 𝑦2 − 𝑦 − 2 = 0
as the ends of its one diameter? ⟹ y = 2, -1
Ans:- (a) ⟹ y = 2 (∵ y > 0)
Given f(x+ y) = f(x)f(y), for all x, y
………………….(1)

f(1)= 2 ……………..(2)

Putting x = 1, y = 1 in (1), we get

f(2) = (𝑓(1))2 = 22
ISI SUBJECTIVE SAMPLE
Putting x = 2, y = 1 in (1), we get
PAPER WITH SOLUTIONS
f (3) = f(2)f(1) = 22 . 2= 23
SET – 3
Similarly, f(n) = 2𝑛 , n ∊ N

Given 𝛼𝑛 = 𝑓(𝑛)
1. Let s= {(𝒂𝟏 , 𝒂𝟐 , 𝒂𝟑 , 𝒂𝟒 ): 𝒂𝒊 ∊ ℝ, 𝒊 =
∴ 𝛼𝑛 =2𝑛 , n ∊ N ∴ 𝛼1 = 2, 𝛼2 = 𝟏, 𝟐, 𝟑, 𝟒 ; 𝒂𝟏 + 𝒂𝟐 + 𝒂𝟑 + 𝒂𝟒 = 𝟎 }
4, 𝛼3 = 8, 𝛼4 = 16
And T= {(𝒂𝟏 , 𝒂𝟐 , 𝒂𝟑 , 𝒂𝟒 ): 𝒂𝒊 ∊ ℝ, 𝒊 =
Let P ≡ (2, 4), Q ≡ (8, 16) 𝟏, 𝟐, 𝟑, 𝟒 ; 𝒂𝟏 − 𝒂𝟐 + 𝒂𝟑 − 𝒂𝟒 = 𝟎 }
∴ Equation of circle having PQ as a Find a basis for S∩T. Also find its
diameter is dimension.
(x - 2)(x - 8)+ (y - 4)(y - 16)= 0 Ans:- S∩T={(𝑎1 , 𝑎2 , 𝑎3 , 𝑎4 ): 𝑎𝑖 ∊ ℝ, 𝑖 =
1, 2, 3, 4 ; 𝑎1 − 𝑎2 + 𝑎3 − 𝑎4 = 0 }

153
Solving Mathematical Problems

Let x̰ ∊ S∩T, then 𝑘


Now, x+𝑛 = [𝑥] + 𝑦 + 𝑛
𝑘

𝑎1 + 𝑎2 = −𝑎3 − 𝑎4……………..(1) 𝑃+𝑘−1 𝑘 𝑃+𝑘


Also, <𝑦+𝑛<
𝑛 𝑛
𝑎1 − 𝑎2 =-𝑎3 + 𝑎4 ………………(2)
𝑃−1+𝑘
So, long as < 1 , i.e. , k < n-(P-1)
_________________________________ 𝑛

𝑘
𝑎1 = −𝑎3 So, 𝑦 + 𝑛 < 1 and consequently

𝑎2 =-2𝑎3 − 𝑎4 𝑘
[x+𝑛] = [x]for k= 0, 1, 2, …, n-P.
Here, x̰= (−𝑎3 , −2𝑎3 − 𝑎4 , 𝑎3 , 𝑎4 ) 𝑘
But [x+𝑛] = [x]+1 for k= n-P+1, …..n-1.
= 𝑎3 (−1, −2, 1, 0) + 𝑎4 (0, −1, 0, 1)
1 𝑛−1
∴[x]+ [x+𝑛]+…+[x+ ]
Here, {(−1, −2, 1, 0), (0, −1, 0, 1)} forms a 𝑛

basis. ([𝑥] + [𝑥] + ⋯ + [𝑥]) +


=⏟
And also dim(S∩T)= 2. (([𝑥] + 1) + ([𝑥] + 1) + ⋯ + ([𝑥 + 1))]

𝟏
2. Find the following limit: 𝐥𝐢𝐦 ( + = n[x]+(P-1)……………………(1)
𝒙→∞ √𝒏𝟐 +𝟏
𝟏 𝟏
+⋯+ ) Also, [nx]=[n[x]+ny]= n[x]+(P-1)
√𝒏𝟐 +𝟐 √𝒏𝟐 +𝒏

𝑛 Since P-1 ≤ ny < P……(2)


Ans:- Let 𝑢𝑛 = √𝑛2
+𝑛
From equation (1) & (2),
𝑛 1
∴ lim 𝑢𝑛 = lim = lim = 1.
𝑥→∞ 𝑥→∞ √𝑛2 +𝑛 𝑥→∞ √1+ 1 1 𝑛−1
𝑛 [x]+[x+𝑛]+…+[x+ ] = [nx].
𝑛

By Cauchy’s first theorem:- 4. Prove that for n > 1, 1+𝟐𝟐 + 𝟑𝟐 + ⋯ +


𝟏 𝟏
𝑢1 +⋯+𝑢𝑛
lim ( ) = 1. 𝟏 𝟏
𝑥→∞ 𝑛 <2−𝒏
𝒏𝟐
1 1 1
So, lim (√𝑛2 + √𝑛2 + ⋯ + √𝑛2 )= 1. 1 5 1 3 6
𝑥→∞ +1 +2 +𝑛 Ans:- P(1)= 1+22 = 4 < 2 − 2 = 2 = 4.
3. for any real number x and for any
The statement is true for n= 2.
positive integer n show that
Let , the statement is true for n= m.
𝟏 𝟐 𝒏−𝟏
[x]+[x+𝒏] + [𝒙 + 𝒏] + ⋯ + [𝒙 + ]=
𝒏 1 1 1 1
[𝒏𝒙] ∴P(m)= 1+22 + 32 + ⋯ + 𝑚2 < 2 − 𝑚.

Ans:- Let x= [x]+y, where 0 ≤ y < 1, Now, we need to show that the statement is
also true for n=m+1.
Let p be an integer such that P-1 ≤ 𝑛𝑦 < 𝑃

154
Solving Mathematical Problems
1 1 1 1 Let us assume p > 1 and q > 1. We arrange
P(m+1)= 1+22 + 32 + ⋯ + 𝑚2 < 2 − (𝑚+1)2
pq integers in q rows of p columns as
1 1 1 follows:
< 2 − 𝑚 + 𝑚(𝑚+1). [∵(𝑚+1)2 <
1
∀ 𝑚 > 1] 1 2 …… r p
𝑚(𝑚+1)

1
p+1 p+2 ……. p+r 2p
< 2-𝑚+1.
2p+1 2p+2 ….. 2p+r 3p
∴ The statement is true for n= m+1
… …. …. …….
So, for all n ∊ ℕ the statement is true.
(q-1)p+1 (q-1)p+2 ….. (q-1)p+r qp
Hence proved.
The number of integers among these, that
5. Show that (a) 𝛷(p)= p-1; are prime to pq is (pq) [By lemma].

(b) 𝛷(pq)=𝛷(p)𝛷(q); where p and q are The number of integers in the first row that
prime numbers. the prime to p is (p) [Lemma]

Ans:- (a) Let us take k as a positive integer Each column in the arrangements
and p be prime. The positive integer ≤ 𝑝𝑘 contain(q)integers prime to q [By lemma 3]
which are not prime to 𝑝𝑘 are p, 2p, 3p,…,
Hence (pq) = 𝛷(p)𝛷(q).
(𝑝𝑘−1 )p. therefore , the number of positive
integers less than 𝑝𝑘 and prime to 𝑝𝑘 is
𝑝𝑘 − 𝑝𝑘−1
6. Determine x, y, and z so that the 3× 3
1 matrix with the following row vectors is
Hence (𝑝𝑘 )= 𝑝𝑘 − 𝑝𝑘−1 = 𝑝𝑘 (1 − 𝑝).
orthogonal:
For k= 1, (p)= p-1. (𝟏⁄ , 𝟏⁄ , 𝟏⁄ ), (𝟏⁄ , − 𝟏⁄ , 𝟎),
√𝟑 √𝟑 √𝟑 √𝟐 √𝟐
To prove this , we use: (x, y,z).

(i) a is prime to pq if and only if a is 1⁄ 1⁄ 1⁄


√3 √3 √3
prime to p and a is prime to q.
(ii) If r be the residue of a modulo q Ans:- 1⁄ − 1⁄ 0
√2 √2
and r is prime to q then a is prime 1⁄ 1⁄ − 2⁄
( √6 √6 √6)
to q.
(iii) If c be an integer and a is prime 𝑖𝑠 𝑎𝑛 𝑜𝑟𝑡ℎ𝑜𝑔𝑜𝑛𝑎𝑙 𝑚𝑎𝑡𝑟𝑖𝑥.
to q then the number of integers
in the set {c, c+a, c+2a, …, c+(n-
1)a}that are prime to q is 𝛷(q)
7. Solve: dy/dx= (y+2)/(x-2)
(b) Since (1)=1, the theorem is trivial
true when p and q equals 1.

155
Solving Mathematical Problems
𝑑𝑦 𝑑𝑥 𝑝 𝑄
Ans:- ∫ 𝑦+2 = ∫ 𝑥−2 tan + tan
2 2 𝜋
∴ 𝑝 𝑄 = tan = 1
1 − tan tan 4
2 2
⤇ log|𝑦 + 2| = log|𝑥 − 2| + log |𝑐|
𝑏
2 2 −
⤇ (𝑦 + 2) = 𝑘(𝑥 − 2) is the required 𝑎
⟹ 𝑐 =1 ⟹𝑏 =𝑐−𝑎 ⟹𝑎+
1−
solution. 𝑎
𝑏=𝑐
10. If (𝟏 + 𝒙)𝒏 = ∑𝒏𝒓=𝟎 𝒂𝒓 𝑿𝒓 𝒂𝒏𝒅 𝒃𝒓 =
8. If w is a complex cube root of unity 𝒂𝒓 (𝟏𝟎𝟏)𝟏𝟎𝟎
then show that 𝟏+𝒂 𝒂𝒏𝒅 ∏𝒏𝒓=𝟏 𝒃𝒓 = , then n
𝒓−𝟏 𝟏𝟎𝟎!
=?
𝒂𝟑 + 𝒃𝟑 + 𝒄𝟑 − 𝟑𝒂𝒃𝒄
= (𝒂 + 𝒃 + 𝒄)(𝒂 + 𝒃𝒘 Ans:- (b) (1 + 𝑥)𝑛 = ∑𝑛𝑟=0 𝑎𝑟 𝑎𝑟 = 𝑛𝐶𝑟
+ 𝒄𝒘𝟐 )(𝒂 + 𝒃𝒘𝟐 + 𝒄𝒘). 𝑎𝑟+𝑎𝑟−1 𝑛𝐶𝑟 +𝑛𝐶𝑟−1
𝑎𝑟
𝑏𝑟 = 1 + 𝑎 = = =
−1 𝑎𝑟−1 𝑛𝐶𝑟−1
Ans:- 𝑎3 + 𝑏 3 + 𝑐 3 − 3𝑎𝑏𝑐 𝑛+1
𝑟

𝑛+1𝐶𝑟 .𝑛𝐶𝑟−1 𝑛+1


𝑟
2 2 2 = =
= (a+ b +c)( 𝑎 + 𝑏 + 𝑐 − 𝑎𝑏 − 𝑏𝑐 − 𝑐𝑎) 𝑛𝐶𝑟−1 𝑛𝐶𝑟−1 𝑟

𝑛
= (a+ b +c){ 𝑎2 + 𝑏 2 𝑤 3 + 𝑐 2 𝑤 3 + (𝑤 + 𝑛 + 1 𝑛 + 1 𝑛 + 1 (𝑛 + 1)2
𝑤 2 )𝑎𝑏 + (𝑤 + 𝑤 2 )𝑏𝑐 + (𝑤 + 𝑤 2 )𝑐𝑎} ∏ 𝑏𝑟 = . … =
1 2 𝑛 𝑛!
𝑟=1
2 2 3 2 3
= (a+ b +c){ 𝑎 + 𝑏 𝑤 + 𝑐 𝑤 + 𝑤𝑎𝑏 + (𝑛+1)𝑛 (101)100
𝑤 2 𝑎𝑏 + 𝑤𝑏𝑐 + 𝑤 2 𝑏𝑐 + 𝑤 2 𝑏𝑐 + 𝑤𝑐𝑎 + Given, =
𝑛! 100!
𝑤 2 𝑐𝑎}
∴ n = 100
2 2
= (a+ b +c){a(a +b𝑤 + 𝑐𝑤)+bw(a+b𝑤 +
𝑐𝑤)+c𝑤 2 (𝑎 + 𝑏𝑤 2 + 𝑐𝑤))

= (a+ b+ c)(a+ b𝑤 2 + 𝑐𝑤)(a+ bw+ c𝑤 2 ). ISI SUBJECTIVE SAMPLE


PAPER WITH SOLUTIONS

𝝅
SET – 4
9. In a 𝛥PQR, ⦟R = 𝟐 . If
𝑷 𝑸
𝒕𝒂𝒏 𝒂𝒏𝒅 𝒕𝒂𝒏 are the roots of
𝟐 𝟐
equation a𝒙𝟐 + 𝒃𝒙 + 𝒄= 0 (a≠0), then 1. If A and B are real orthogonal matrices
show that a + b = c. of the same order and |B|+|A|= 0.
𝑝 𝑄 Prove that |A+ B|= 0
Ans:- (a) tan 2 + tan 2 =
𝑏 𝑃 𝑄 𝑐 𝑃 𝑄 𝜋
− 𝑎 , tan 2 tan 2 = 2 ∴2+2= Ans:- |A|+|B|=0
4

⇒ |A|= −|B|

156
Solving Mathematical Problems

|A|.|B|=−1 [∵ |B|=|𝐵 −1 |as they are (ii) 𝐥𝐢𝐦


𝒖𝒏+𝟏
=𝟐
𝟏
𝒏⟶∞ 𝒖𝒏
orthogonal]
Ans:- (i) 0 < 𝑢1 < 1
Let, C = A (𝐴𝑇 + 𝐵 𝑇 )B

⇒ |C|= |A𝐴𝑇 𝐵 + 𝐴𝐵 𝑇 𝐵| = |𝐵 + 𝐴| ⇒ 0 < √1 − 𝑢1 < 1


………..(i)
⇒ 0 < 1 -√1 − 𝑢1 < 1
𝑇 𝑇 𝑇 𝑇
And |C|= |A||𝐴 + 𝐵 ||𝐵| = −|𝐴 + 𝐵 |
i.e. 0 < 𝑢2 < 1
𝑇
⇒ - |(𝐴 + 𝐵) | = −|𝐴 + 𝐵|…………..(ii)
Similarly, 0 < 𝑢3 < 1 ….. and so on.
|𝐴 + 𝐵| = −|𝐴 + 𝐵|
Let 0 < 𝑢𝑛 < 1, then 0 < 1- √1 − 𝑢𝑛 < 1,
⇒ 2 |A +B|=0 i.e. 0 < 𝑢𝑛+1 < 1

⇒ |A+ B|=0 Thus {𝑢𝑛 } is bounded.


2. Determine whether there is a one –to – Again, 𝑢𝑛+1 − 𝑢𝑛 = 1-√1 − 𝑢𝑛 -𝑢𝑛
one function f: ℝ⟶ ℝ such that
𝟏 =(1- 𝑢𝑛 )- √1 − 𝑢𝑛
f(𝒙𝟐 )-[f(𝒙)]𝟐 ≥ 𝟒 ∀ 𝒙
= (√1 − 𝑢𝑛 )2 − √1 − 𝑢𝑛
1
Ans:- Take x= 0, then f(0) – (f(0))2 ≥ 4
= √1 − 𝑢𝑛 (√1 − 𝑢𝑛 − 1)
2 1 1
⇒ (f(0)) + (2)2 − 2. 2 . f(0) ≤ 0
< 0 as 0 < √1 − 𝑢𝑛 < 1
1 2
⇒ (f(0) − ) ≤0
2 ∴ 𝑢𝑛+1 < 𝑢𝑛 as 0 < √1 − 𝑢𝑛 <1
1 1
⇒ f(0) − 2 = 0 ⇒ f(0) = 2 ∴ {𝑢𝑛 } is monotonically decreasing. ∴ {𝑢𝑛 }
1 converges two zero.
Also, taking x=1 we have f(1) - 2 = 0 ∴
𝑢𝑛+1
f(0) = f(1)= 2
1 (ii) Let lim 𝑢𝑛 = 𝑙, then lim
𝑛⟶∞ 𝑛⟶∞ 𝑢𝑛

1− √1−𝑙 𝑙 1 1
∴ This is not one -to –one function. ∴ lim = lim = =
𝑛⟶∞ 𝑙 𝑛⟶∞ 𝑙(1+ √1−𝑙 ) 1+ √1−0 2

; Since 𝑢𝑛 converges to zero.

3. if 0 < u < 1 and 𝒖𝒏+𝟏 = 𝟏 − 4. Let g: ℝ⟶ ℝ be a continuous function


𝒙−𝟏
√𝟏 − 𝒖𝒏 ∀ 𝒏 > 1, ∋ g(x) = g ( 𝟐
) ∀ x.

Prove that (i) {𝒖𝒏 } converges to zero Show that g must be a constant function.

157
Solving Mathematical Problems
𝑥−1 𝑥
Ans:- g(x) = g ( ) Ans:- F (x) = ∫0 𝑓(𝑡)𝑑𝑡 =
2
𝑥 𝑥 𝑘+1
𝑥−1
−1
∫0 (∑𝑛𝑘=0 𝑎𝑘 𝑡 𝑘 )𝑑𝑡 = ∑𝑛𝑘=0 𝑎𝑘 . 𝑘+1
𝑥−1 2 𝑥−3
⇒g( )=g( )=g( )
2 2 4
Clearly F (x) satisfies the conditions of
𝑥−1
Again putting x= Rolle’s Theorem as F (0) = 0 and
2
𝑎
𝑥−1 𝑥−7 F (1) =∑𝑛𝑘=𝑜 𝑘+1
𝑘
=0
g( )= g ( ) and so on
2 2

𝑥−1 𝑥−(2𝑛 −1) Hence ∃ a ‘c’ ∊ (0, 1) ∋ F′(c) =0 ⇒ F (c) =0


Generally we have, g( )= g ( )
2 2𝑛

𝑥 1
∴ g (x)= g (2𝑛 − 1 + 2𝑛 ) 𝟏 𝟏 𝟏 𝟏
7. Show that 1+ 𝟐 + 𝟑 + 𝟒 + ⋯ . + can
𝒏
∴ lim 𝑔 (𝑥) = 𝑔 (−1) never be an integer value.
𝑛⟶∞
1 1 1
⇒ g (x)= g (-1)= constant ∀ x. Ans:- We are to show:- 1+ 2 + 3 + 4 +
1 𝑎𝑛 𝑜𝑑𝑑 𝑛𝑢𝑚𝑏𝑒𝑟
5. Find the greatest and least value of the ⋯.+ = 𝑎𝑛 𝑒𝑣𝑒𝑛 𝑛𝑢𝑚𝑏𝑒𝑟 ∀ 𝑛 > 1
𝑛
function f (x)= 𝒙𝟑 − 𝟑𝒙𝟐 + 𝟐𝒙 + 𝟏 in [2, 1 1 1 1
3]. Let, P (n): 1+ 2 + 3 + 4 + ⋯ . + 𝑛=
𝑎𝑛 𝑜𝑑𝑑 𝑛𝑢𝑚𝑏𝑒𝑟
∀𝑛 >1
Ans:- f (x)= 𝑥 3 − 3𝑥 2 + 2𝑥 + 1 𝑎𝑛 𝑒𝑣𝑒𝑛 𝑛𝑢𝑚𝑏𝑒𝑟

1 3 𝑎𝑛 𝑜𝑑𝑑 𝑛𝑢𝑚𝑏𝑒𝑟
f′(x)= 3𝑥 2 − 6𝑥 + 2; f′ (2) = 2 > 0; f′ When n=2, LHS= 1+ 2 = 2 = 𝑎𝑛 𝑒𝑣𝑒𝑛 𝑛𝑢𝑚𝑏𝑒𝑟
(3) =11 > f′ (2)
∴ P (2) is true. Let P (m)be true
∴ f(x) is an increasing function
1 1 1 1
⇒ 1+ 2 + 3 + 4 + ⋯ . + =
Note that f″(x) = 6x -6 > 0 ∀ x ∊ [2, 3] 𝑎𝑛 𝑜𝑑𝑑 𝑛𝑢𝑚𝑏𝑒𝑟 𝑘
𝑚
= (𝑠𝑎𝑦)
𝑎𝑛 𝑒𝑣𝑒𝑛 𝑛𝑢𝑚𝑏𝑒𝑟 𝑟
∴ f(x) is concave.
𝑘 1
Now, P (m+1) = + 𝑚+1
Thus the function has min. value at x= 2 and 𝑟
max. Value at x= 3
m is odd or even,
∴ minimum value= f(2)=1 ∴ maximum
But in case, it can be shown that
value = f(3) = 7
𝑎𝑛 𝑜𝑑𝑑 𝑛𝑢𝑚𝑏𝑒𝑟
P (m)= 𝑎𝑛 𝑒𝑣𝑒𝑛 𝑛𝑢𝑚𝑏𝑒𝑟

6. Let F (x) = ∑𝒏𝒌=𝟎 𝒂𝒌 𝒙𝒌 , where 𝒂𝒌 satisfy ∴P (n) is true for all n ∊ℕ


𝒂
∑𝒏𝒌=𝟎 𝒌 = 𝟎 so that there exists a real
𝒌+𝟏
root of f (x) = 0 in the interval (0, 1)

158
Solving Mathematical Problems

8. Give an example of a function f:[a, closed and bounded subset of ℝ. Hence, S is


b]⟶ ℝ ∋ |f (x)- f (y)| < (x-y)∀ x, y ∊ [a, b]. complete.

Prove that any function satisfying the Assume S is infinite.


above condition also satisfies
Then there is a limit point 𝑥 ∊ 𝑆;
𝒃 𝟏
|∫𝒂 𝒇 (𝒙)𝒅𝒙 − (𝒃 − 𝒂)𝒇(𝒂)| ≤ 𝟐 (𝒃 − 𝒂)𝟐 ,
i.e. there is a sequence {𝑥𝑛 } of distinct
provided f (x) is integrable on [a, b]. points in S which converges to x.
Ans:- f(x)= sin 𝑥 Also, as all points are in S, 𝑓(𝑥𝑛 ) = 𝑓(𝑥) =
0 ∀ 𝑛 ∊ 𝑁.
| f (x)- f (y)| = |sin 𝑥 − sin 𝑦|
𝑥+𝑦 𝑥−𝑦 𝑥−𝑦 We now show that f’(x) =0.
= |2cos ( ) sin ( )| ≤ 2|sin ( )|≤
2 2 2
2|
𝑥−𝑦
|=|x- y| Since.|𝑥𝑛 − 𝑥|→0 ,
2
so
[∵ for x > 0, sin 𝑥 ≤ 𝑥]
𝑓(𝑥+(𝑥𝑛 −𝑥))−𝑓(𝑥)
f (x)− f (y) f (x)− f (y) f’(x) = lim
[| |≤ 1 ∴ | lt | ≤ 1⇒ 𝑛→∞ 𝑥𝑛 −𝑥
𝑥−𝑦 𝑥⟶𝑦 𝑥−𝑦
|f′(y)|≤ 1] = lim
𝑓(𝑥𝑛 )−𝑓(𝑥)
𝑛→∞ 𝑥𝑛 −𝑥
𝑢
Let ∫−∞ 𝑓(𝑥)𝑑𝑥 = 𝐹(𝑢)
=0
By Taylor’s theorem:- F(b)= F (a)+ (b-a)f′ The last equality holds since f(x) = f(𝑥𝑛 ) =0
(𝑏−𝑎)2 ″
(a)+ 𝑓 (𝑐 ∗) holds ∀ 𝑛 ∊ 𝑁.
2

(𝑏−𝑎)2
⇒ F (b)-F (a) = (b-a) f (a) + f′(c*)
⎿2
10. If f’(a) = f’’(a) = f’’’(a)=0 but
𝑏
⇒ |∫𝑎 𝑓 (𝑥)𝑑𝑥 − (𝑏 − 𝑎)𝑓(𝑎) |= 𝒇(𝒊𝒗) (a)>0 and 𝒇(𝒊𝒗) (x) is continuous at
(𝑏−𝑎)2 (𝑏−𝑎)2 x=a. Then show that f(a) is local
| 𝑓 ′ (𝑐 ∗)| ≤ .
2 2 minimum.

Ans:- Toylor’s theorem states that


9. Let f: ℝ → ℝ be differentiable and (𝑥−𝑎)2
f(x) = f(a) + (x-a)f’(a) + f”(a)
assume there is no x 𝒊𝒏 ℝ ℈ f(x) = f’(x) = 2!
(𝑥−𝑎)𝑛
0. Show that S = {x| 0≤ 𝒙 ≤ 𝟏 ; 𝒇(𝒙) = 𝟎} +……+ 𝑓 𝑛 (𝑐), a < c < x
𝑛!
is finite.
Now, applying Taylor’s theorem,
Ans:- Consider 𝑓 −1 ({0}). Since {0} is
closed and f is continuous 𝑓 −1 ({0}) is (𝑥−𝑎)4
f(x) = f(a) + 𝑓 (𝑖𝑣) (c)
4!
closed. Therefore, S=[0,1]∩ 𝑓 −1 ({0}) is

159
Solving Mathematical Problems

(𝑥−𝑎)4 ⟹cos 𝛼1 . cos 𝛼2 … . cos 𝛼𝑛 =


=> f(x) –f(a)= 𝑓 (𝑖𝑣) ≥ 0 ∀ 𝑎 ∊ (𝑎−∊
4!
sin 𝛼1 . sin 𝛼2 … . sin 𝛼1 …………….(1)
, 𝑎+∊)
Now, (cos 𝛼1 . cos 𝛼2 … . cos 𝛼𝑛 )2 =
i.e. f(x) ≥ f(a).
(cos 𝛼1 . cos 𝛼2 … . cos 𝛼𝑛 )(cos 𝛼1 . cos 𝛼2 … . cos 𝛼𝑛 )
So, f(a) is the local minimum.
=
(cos 𝛼1 . cos 𝛼2 … . cos 𝛼𝑛 )(
sin 𝛼1 . sin 𝛼2 … . sin 𝛼𝑛 ) [from (1)]
ISI SUBJECTIVE SAMPLE
1 1
PAPER WITH SOLUTIONS = 2𝑛 sin 2𝛼1 . sin 2𝛼2 … . sin 2𝛼𝑛 ≤ 2𝑛

SET – 5 ∴ (cos 𝛼1 . cos 𝛼2 … . cos 𝛼𝑛 )2 ≤ 2𝑛


1

1 1
∴cos 𝛼1 . cos 𝛼2 … . cos 𝛼𝑛 ≤ √2𝑛 ≤ 𝑛
1. The four digit number aabb is a 22

square. Find the number. [∵cos 𝛼𝑖 ≥ 0]

Ans:- aabb = 𝑛2

Then 𝑛2 = 1100a + 11b 3. f(x, y) = 0 is a circle such that f(0, 𝜆) = 0


and f(𝜆,0) = 0 have equal roots and f(1,1)=
= 11(100a + b) - 2 then the radius of the circle is

= 11(99a + a + b) (a) 4 (b) 8


(c) 2 (d) 1
Since, 𝑛2 is divisible by112 , we see that 11 |
(a+b) Ans. (c)

i.e. a+b=11. Since 𝑛2 𝑖𝑠 a square , bcan’t be Let f(x, y) = 𝑥 2 + 𝑦 2 + 2𝑔𝑥 + 2𝑓𝑦 + 𝑐 =


0,1,2,3,5,7 or 8 . Checking the remaining we 0 𝑏𝑒 𝑡ℎ𝑒 𝑟𝑒𝑞𝑢𝑖𝑟𝑒𝑑 𝑐𝑖𝑟𝑐𝑙𝑒
see that 7744= 882
f(0, 𝜆) = 𝜆2 + 2𝑓𝜆 + 𝑐 = 0 ………….(1)
2. Find the maximum value of
𝒄𝒐𝒔𝜶𝟏 . 𝒄𝒐𝒔𝜶𝟐 . 𝒄𝒐𝒔𝜶𝒏 , under the f(𝜆, 0) = 𝜆2 + 2𝑔𝜆 + 𝑐= 0…………….(2)
restrictions ∵ (1) and (2) have equal roots.
0 <𝜶𝟏 , 𝜶𝟐 , … 𝜶𝒏 ≤ 𝟐 𝒄𝒐𝒕𝜶𝟏 . 𝒄𝒐𝒕𝜶𝟐 . 𝒄𝒐𝒕𝜶𝒏 ∴ D= 0
= 1.
⟹ 𝑓 2 = 𝑔2 = 𝑐
Ans.
∴ f(x, y) = 𝑥 2 + 𝑦 2 + 2𝑔𝑥 ± 2𝑓𝑦 + 𝑔2 = 0
Given, (cot 𝛼1 ). (cot 𝛼2 ) …. (cot 𝛼𝑛 ) = 1
f(1, 1) = - 2 ⟹ 𝑔2 + 2𝑔 ± 2𝑔 + 4 = 0

160
Solving Mathematical Problems

when f = - g, 𝑔2 = −4 (not possible) 𝒙𝟐 + 𝟏 𝒙 + 𝟏 𝒙+𝟐


|𝟐𝒙 + 𝟑 𝟑𝒙 + 𝟐 𝒙 + 𝟒 | = 𝒎𝒙𝟒 +
∴ f = g and g = f = -2 and c = 4 𝒙 + 𝟒 𝟒𝒙 + 𝟑 𝟐𝒙 + 𝟓
𝒏𝒙𝟑 + 𝒑𝒙𝟐 + 𝒒𝒙 + 𝒓 be an identity in x,
∴ Radius of circle is 2.
then the variable line always passes
𝟏 𝟏 𝟏 𝟐𝒎 through the point
4. If 𝟏!𝟗! + 𝟑!𝟕! + 𝟓!𝟓! = , then
𝒏!
orthocenter of the triangle having sides x (a) (-r, m) (b) (-m, r)
- y + 1 = 0, x + y + 3 = 0 and 2x + (c) (r, m) (d) (2r, m)
5y - 2= 0 is
Ans. (c)
(a) (2m- 2n, m- n) (b) (2m- 2n, n-
Let the variable line be ax + by + c= 0
m) (c) (2m- n, n+ m) (d)
none Given,
Ans. (a) 3𝑎+𝑏+𝑐 −𝑎+2𝑏+𝑐 𝑎+3𝑏+𝑐
√𝑎2 +𝑏2
+ √𝑎2 +𝑏 2
+ √𝑎2 =0
+𝑏2
2𝑚 2 2 1
= |1̲|9 + |3|7̲ + |5|5 ⟹ 3a+ 6b+ 3c⟹ 0 ⟹ a+ 2b +c = 0
̲̲
|𝑛 ̲ ̲ ̲̲

1
= |1̲0 [2. 10𝐶1 + 2. 10𝐶3 + 10𝐶5 ] ⟹ line ax + by +c = 0 passes through the
̲
point (1, 2)
1
= |10 [10𝐶1 + 10𝐶3 + 10𝐶5 + 10𝐶7 + Also in the given identity putting x= 0, we
̲
10𝐶9 ] [∵ 10𝐶1 = 10𝐶9 , 10𝐶3 = 10𝐶7 ] get r= 1 and equating the coefficient of 𝑥 4 ,
we get m = 6 – 4 = 2.
29
= |10
̲ 6. If a, b, c are positive integers such that
abc + ab + bc + ca + a + b + c = 1000.
∴ m= 9, n= 10
Find the value of a + b +c ?
Two sides of the given triangle are x - y + 1
= 0 and x + y + 3 = 0. Ans:- abc +ab + ac + bc +a + b+ c = 1000
Clearly they are perpendicular. Therefore ⇒ a (bc +b+ c)+ a+ b+ c+ bc +1 = 1000+1
orthocenter of the triangle will be point of
intersection of these two lines which is (-2, - ⇒ a (bc +b+ c+ 1)+ (b+ c+ bc+1) = 1001
1).
⇒ (a+ 1) (b+ 1) (c+ 1) = 1001 = 13× 7 ×
5. If the algebraic sum of the 11
perpendicular distance from the points (3,
⇒ a= 12, b= 6, c = 10,
1), (-1, 2), and (1, 3) to a variable line be
zero, and ∴ a + b + c = 28.

161
Solving Mathematical Problems

7. Find minimum value of 𝟐𝐜𝐨𝐬 𝒙 + 1 3 615


∴ f (1234) + f (1234) + …+ f (1234) +f (1234) +
617

𝟐𝐬𝐢𝐧 𝒙 , 𝟎 ≤ 𝒙 ≤ 𝟐𝝅 ? 619 1233


f (1234) + …+ f (1234)
Ans:- By AM ≥ 𝐺𝑀 inequality, we have
= 308 + 0.5 = 308.5
cos 𝑥 sin 𝑥 √2cos 𝑥+sin 𝑥
2 +2 ≥2
sin 𝑥+cos 𝑥
= 2. 2 𝒃−𝒂
2 9. Show that 𝟏+𝒃𝟐 < 𝐭𝐚𝐧−𝟏 𝒃 − 𝐭𝐚𝐧−𝟏 𝒂 <
Note that the RHS will be minimum if 𝒃−𝒂
.
𝟏+𝒂𝟐
sin 𝑥 + cos 𝑥 is minimum, i.e. when both sin
x and cos x are minimum, i.e. at x= 2250 , Ans:- Let f(x)=tan−1 𝑥.
1
sin x = - = cos 𝑥 𝑓(𝑏)−𝑓(𝑎)
√2
From Mean-value theorem, 𝑏−𝑎
= 𝑓 ′ (𝑐)
2 1
− −
∴ 2cos 𝑥 + 2sin 𝑥 ≥ 2.2 √2
= 2.2 √2

tan−1 𝑏−tan−1 𝑎 1
= 𝑓 ′ (𝑐) = 1+𝑐 2 ; a < c < b.
2
𝑏−𝑎
1
1− 𝑏−𝑎
∴ The minimum value is 2 √2
→ tan−1 𝑏 − tan−1 𝑎 = 1+𝑐 2.

𝑏−𝑎 tan−1 𝑏−tan−1 𝑎 𝑏−𝑎


∴ < < 1+𝑎2 .
1+𝑏 2 1
𝒆𝟐𝒙−𝟏 𝟏 𝟑
8. f (x) = 𝟏+𝒆𝟐𝒙−𝟏 . Then f(𝟏𝟐𝟑𝟒)+ f(𝟏𝟐𝟑𝟒)+ …
𝟏𝟐𝟑𝟏 𝟏𝟐𝟑𝟑
+ f (𝟏𝟐𝟑𝟒)+ f (𝟏𝟐𝟑𝟒)= ?
10. Let s=√𝟏 + √𝟐 + √𝟑 + ⋯ + √𝟏𝟎𝟎𝟎𝟎
1 𝑒 1−1 1 𝟏𝟎𝟎𝟎
Ans:- f (2)= 1+𝑒 1−1 = 2 and I=∫𝟎 √𝒙 𝒅𝒙. Show that I ≤ 𝒔 ≤ 𝑰 +
𝟏𝟎𝟎.
Now, f (x) + f(1-x)=1
1000 1
1 1
Ans:- I=∫0 √𝑥 𝑑𝑥 = ∫0 √𝑥 𝑑𝑥 +
So, f (1234)+ f(1- 1234)=1 2 10000
∫1 √𝑥 𝑑𝑥 + ⋯ + ∫9999 √𝑥 𝑑𝑥
⁞ 1 2
→∫0 0. 𝑑𝑥 + ∫1 √1 dx
615 10000 1
Now there are 308 terms up to f (1234)+ f (1- +….+∫9999 √9999 𝑑𝑥 ≤ 𝐼 ≤ ∫0 1. 𝑑𝑥 +
615 2 10000
)= 1 ∫1 √2 𝑑𝑥 + ⋯ + ∫9999 √10000 dx
1234

617
Now, f(1234)+ f(1-1234) =1
617 →√1 +√2 +…..+√9999 ≤ 𝐼 ≤ √1 +√2
+…..+√10000
1 1
⇒ f (2)+ f (2) = 1
→I ≤ 𝑆
617
i.e. f (1234) = ½ Also, S ≤ 𝐼 + √10000

162
Solving Mathematical Problems

∴ I ≤ 𝑆 ≤ 𝐼 + 100 Ans:- x = yz, y = zx , z = xy

∴ xyz = (𝑥𝑦𝑧)2

i.e. xyz= 0 or 1

ISI SUBJECTIVE SAMPLE now, xyz = 𝑥 2 = 𝑦 2 = 𝑧 2


PAPER WITH SOLUTIONS ⇒ |x|= |y|= |z|= 1
SET – 6 However the remaining 4 cases are: (1, 1, 1),
(-1, -1, -1), (-1, 1, -1) or (1, -1, -1); i.e. it has
1. Prove that 4 solutions.

𝟏 𝟏 𝟏 𝟒 3. If X= {n: n is a positive integer, n≤ 𝟓𝟎},


1 < 𝟏𝟎𝟎𝟏 + 𝟏𝟎𝟎𝟐 + ⋯ + 𝟑𝟎𝟎𝟏 < 𝟑 A = {n ∊ X:n is even} and B= {n ∊ X: n is
1 a multiple of 7}, then what is the number
Ans:- consider 2001 numbers 𝑘 , 1001 ≤
of elements in the smallest subset of X
𝑘 ≤ 3001 containing both A and B ?
Using AM- HM inequality, we get Ans:- The number of integers ≤n and
𝑛
1 divisible by k is given by [𝑘 ], where [ .]
(∑3001 3001
𝑘=1001 𝑘 )(∑𝑘=1001 𝑘 ≥ (2001)
2

denotes the greatest integer function.


But ∑3001
𝑘=1001 𝑘 = (2001) 2
50 50
Accordingly, n(A)= [ 2 ] = 25 , n(B)= [ 7 ] =
1
Hence we get the inequality ∑3001
𝑘=1001 𝑘 > 1 7
50
On the other hand grouping 500 terms at a n (A∩ 𝐵)= [14]=3.
time, we also have
n (A ∪ B)= 𝑛 (𝐴) + 𝑛 (𝐵) − n (A ∩ 𝐵)=
1 500 500 500 500
S= ∑3𝑛+1
𝑘=𝑛+1 𝑘 < 1000 + 1500 + 2000 + 2500 + 25+7-3
1 1 1 1 1 1 3851 4 =29.
< + 3 + 4 + 5 + 3001 = 3000 < 3
3001 2

1
[Remarks:- if S =∑3001
𝑘=1001 𝑘, there are (2n+ 𝟏 𝑨𝒓
4. If = ∑𝒏𝒓=𝟎 , then Ar
𝒙(𝒙+𝟏)(𝒙+𝟐)….(𝒙+𝒏) 𝒙+𝒓
1) terms in the sum and the middle term is
=?
1 29 7
; then < 𝑆 < 6 ].
2𝑛+1 27 Ans:- By method of representation for
partial fraction Ar , is obtained by putting x+
2. How many ordered triplet (x, y, z) of
r= 0,
non zero real numbers have the property
that each number is the product of the i.e. x= - r in all factors, except (x+ r).
other two?

163
Solving Mathematical Problems

∴ Ar =
1
𝒂𝟏 𝒂 𝒂𝒏−𝟏 𝒂
−𝑟(−𝑟+1)…(−𝑟+𝑟+1)(−𝑟+𝑟+1)(−𝑟+𝑟+2)…….(−𝑟+𝑛) 7. Show that + 𝒂𝟐 + ⋯ + + 𝒂𝒏 > 𝑛
𝒂𝟐 𝟑 𝒂𝒏 𝟏
1
= 𝑟 Ans:- AM > GM
(−1) {𝑟!}{(𝑛−𝑟)!}

1 𝑎1 𝑎2 𝑎𝑛 𝑎1 𝑎2 𝑎𝑛
( + + ⋯+ ) > ( . .…. )
𝑛 𝑎2 𝑎3 𝑎1 𝑎2 𝑎3 𝑎1
5. If 𝒂𝟏 , 𝒂𝟐 , … . , 𝒂𝒏 are non-ve
and 𝒂𝟏 , 𝒂𝟐 , … . , 𝒂𝒏 = 𝟏. Show (1+𝒂𝟏 ) 𝑎 𝑎 𝑎
or, (𝑎1 + 𝑎2 + ⋯ + 𝑎𝑛) > 𝑛
(1+𝒂𝟐 )….. (1+𝒂𝒏 )≥ 𝟐𝒏 2 3 1

Ans:- By AM ≥ GM
1+𝑎𝑖 8. If 𝒂𝟏 , 𝒂𝟐 , … . , 𝒂𝒏 be non- negative real
( )≥ √𝑎𝑖 ∀ 𝑖 = 1(1)𝑛 numbers such that
2

Multiplying all these 𝒂𝟏 + 𝒂𝟐 + 𝒂𝟑 + … . + 𝒂𝒏 = 𝒎, then


𝒎𝟐
(1+𝑎1 ) (1+𝑎2 )….. (1+𝑎𝑛 )≥ prove that ∑𝒊<𝑗 𝒂𝒊 𝒂𝒋 ≤ 𝟐
𝑛
2 √𝑎1 , 𝑎2 , … . , 𝑎𝑛 = 2 𝑛 Ans:- 𝑚2 = (𝑎1 + … . + 𝑎𝑛 )2

𝑚2 = 𝑎1 2 + ⋯ + 𝑎𝑛 2 + 2 ∑𝑖<𝑗 𝑎𝑖 𝑎𝑗

6. If 𝒂𝒊 > 0 ∀ 𝑖 = 1(1)𝑛. Prove that 𝑚2


⇒ ∑𝑖<𝑗 𝑎𝑖 𝑎𝑗 ≤ . [∵𝑎1 2 + ⋯ +
𝟏 𝟏 2
(𝒂𝟏 + 𝒂𝟐 + … . + 𝒂𝒏 ) (𝒂 + 𝒂 + ⋯ + 𝑎𝑛 2 ≥ 0]
𝟏 𝟐
𝟏
) > 𝒏𝟐
𝒂𝒏

Ans:- AM> GM 9. Let A be a set containing n elements. If


the number of elements in the set,
𝑎1 , 𝑎2 , … . , 𝑎𝑛 1
> (𝑎1 , 𝑎2 , … . , 𝑎𝑛 )𝑛
𝑛 B = {(x, y, z) : x ∊A, y ∊A, z ∊A and x, y, z,
1 1 1 are not all distinct} is equal to 280,
+ +⋯+ 1 1 1 1
𝑎1 𝑎2 𝑎𝑛
And > (𝑎 . 𝑎 . … . 𝑎 )𝑛
𝑛 1 2 𝑛 then find the value of n?
⇒ (𝑎1 + 𝑎2 + … . + 𝑎𝑛 ) > Ans:- (b)
1
𝑛(𝑎1 , 𝑎2 , … . , 𝑎𝑛 ) 𝑛
According to question 𝑛3 − (𝑛3). 3! = 280
1
1 1 1 1
And 𝑎 + 𝑎 + ⋯ + 𝑎 > n(𝑎 )𝑛
1 2 𝑛 1 …….𝑎𝑛 ∴ 𝑛3 − 𝑛(𝑛 − 1)(𝑛 − 2) = 280

⇒ (𝑎1 + 𝑎2 + … . + 𝑎𝑛 ) (𝑎 + 𝑎 + ⋯ +
1 1
⟹ n(𝑛2 − 𝑛2 + 3𝑛 − 2)= 280
1 2
1
) > 𝑛2 ⟹ n(3n- 2)= 280= 10(3.10- 2)
𝑎𝑛

164
Solving Mathematical Problems

∴ n= 10. 𝑥 2 +𝑥 𝑛𝜋
∴ =
6 2

⇒ 𝑥 2 + 𝑥 − 3𝑛𝜋 = 0
10. If the maximum number of trails
Here 𝑏 2 − 4𝑎𝑐 = 1 − 4.1(−3𝑛𝜋)
required to open all locks when there are
n locks and n keys is 105, then what is the = 1+12 𝑛𝜋 ≥ 0 ∀ 𝑛 = 0, 1, 2, … …
value of n?
∴there are infinitely many roots.
Ans:- Max. number of trails to open the first
lock = n – 1 2. Find the no. of real roots of the
polynomial f(x) = 𝒙𝟓 + 𝒙𝟑 − 𝟐𝒙 + 𝟏.
Max. number of trails to open the second
lock = n – 2 and so on. Ans:- Descarte’s sign rule:-

∴ Maximum number of trails to open all the f(x)= 0 has two sign changes.
(𝑛−1)𝑛
locks = ( n- 1) + (n -2) +…+1 = ∴ No. of +ve roots ≤ 2
2

(𝑛−1)𝑛
Given, = 105 f(-x)= 0 = -𝑥 5 − 𝑥 3 + 2𝑥 + 1
2

⟹ n(n -1)= 210= 15× 14 - - + +

∴ n= 15 ∴f(-x) has the one sign change.

∴no. of –ve roots ≤1

⇒ there is no negative roots,


ISI SUBJECTIVE SAMPLE
PAPER WITH SOLUTIONS ∴ As complex roots occure in pair, so, there
is one +ve roots.
SET – 7
3. Let f(x)= 𝒙𝟑 + 𝟑𝒙 − 𝟐 , x ∊ ℝ, Show
that f(x)=0 has only one real root.
1. Find the no. of real roots of the
equation Ans:- f′(x)= 3(𝑥 + 1)2 > 0
𝒙𝟐 +𝒙 f(-1)= -6 < 0, f(2)> 0
2cos ( ) = 𝟐𝒙 + 𝟐−𝒙
𝟔
∴ It has one root.
𝑥 2 +𝑥 2𝑥 +2−𝑥
Ans:- cos ( )= ≥ √2𝑥 + 2−𝑥 =
6 2
4. If 𝐥𝐭 𝐟(𝐱) = 𝟏 𝐚𝐧𝐝 𝐥𝐭 𝐟′(𝐱) = 𝛂 ,
1, by AM ≥ GM 𝐱→∞ 𝐱→∞
find 𝛼 .
𝑥 2 +𝑥
But cos ( )≤1 𝑘
6
Ans:- Let f(x)=1+𝑥 𝑝 , p> 0
𝑥 2 +𝑥 𝑛𝜋
∴ cos ( ) = 1 = cos( 2 )
6

165
Solving Mathematical Problems
1 1 1 1
Then lt 𝑓(𝑥) = 1 = 4𝑣 + 4 . 𝑣 = 2𝑣.
𝑥→∞

𝑘(−𝑝) 3
∴ 𝛼= lt 𝑓 ′(𝑥) = lt =0 5 3 1
I = ∫3 ∫1 𝑢. 2𝑣 𝑑𝑢𝑑𝑣 = [[ 4 ] [𝑙𝑛𝑣]53 =
𝑢2
𝑥→∞ 𝑥→∞ 𝑥 𝑝+1
1
25
𝒙 𝒊𝒇 𝒙 ∊ [𝟎, 𝟐] log ( 9 ) .
5. Let f (x)= { and g(x)=
𝟎 𝒊𝒇 𝒙 ∉ [𝟎, 𝟐]
𝟏 𝒊𝒇 𝒙 ∊ [𝟎, 𝟐] 7. Find the value of
{
𝟎 𝒊𝒇 𝒙 ∉ [𝟎, 𝟐] 𝟐
∫𝒙𝟐 +𝒚𝟐 ≤𝟏 ∫ 𝟏+𝒙𝟐 +𝒚𝟐 𝒅𝒙𝒅𝒚 .
Let A = {(x, y): x+ y ≤ 𝟑}, then find the
Ans:- Let x = rcos𝜃, y= r sin𝜃
value of the integral ∫𝑨 ∫ 𝒇(𝒙)𝒈(𝒚)𝒅𝒙𝒅𝒚
As 𝑥 2 + 𝑦 2 ≤ 1
ANS:- ∫𝐴 ∫ 𝑓(𝑥)𝑔(𝑦)𝑑𝑥𝑑𝑦
∴ 0< r < 1
= ∫ ∫ 𝑥 .1. 𝑑𝑥𝑑𝑦
& 0 < 𝜃 < 2𝜋
{(x, y): 0 < x < y < 2 & x+ y≤ 3} where 𝑐𝑜𝑠𝜃 𝑠𝑖𝑛𝜃
J= | |=𝑟
0 < x < 1 0 < y < 2 & 1≤ x < 2 , 0 ≤ 𝑦 < 𝑟𝑠𝑖𝑛𝜃 𝑟𝑐𝑜𝑠𝜃
3−𝑥 2𝜋 1 2
∴ I = ∫0 ∫0 𝑟. 𝑑𝑟𝑑𝜃
1+𝑟 2
1 2 2 3−𝑥
= ∫0 ∫0 𝑥𝑑𝑦𝑑𝑥 + ∫1 (∫0 𝑥𝑑𝑦)𝑑𝑥 = 1+ = [|n (1 + 𝑟 2)]10 [𝜃]2𝜋
2 0
3 𝛼3 19
[ 𝑥2 − ] = .
2 3 1 6 = 2𝜋log 𝑒 2 .
6. Using the change of variable evaluate 8. Give an example of each of the types of
∫𝑹 ∫ 𝒙𝒚𝒅𝒙𝒅𝒚, when the region R is functions
bounded by the curves xy = 1, xy = 3, y = (a) The function which is continuous
3x, y = 5x in the 1st coordinate. but not differentiable at only
Ans:- (i) One point (ii)
two points
Then transformation domain is D = {(u, v): (iii) ten points
1 ≤ 𝑢 ≤ 3, 3 ≤ 𝑣 ≤ 5} (b) The function which is
discontinuous at
𝑢
∴ y = √𝑢𝑣 , 𝑥 = √𝑣 (i) Four points (ii) 15
points.
Jacobian of the transformation is (c) The function which is
differentiable once but not twice at
1 1 1 1
. √𝑢(− 2 . 𝑣√𝑣) (i) One points (ii)
𝑣 2 𝑢
J= | √ √1 1 | three points.
√𝑣. 2√𝑢 √𝑢. 2√𝑣
Ans:- (a) (i) f(x)= |x|

166
Solving Mathematical Problems

(ii) f(x)= |x|+|x-1| [𝑎⃗ + 2 𝑏⃗⃗𝑏⃗⃗ + 2 𝑐⃗𝑐⃗ + 2 𝑎⃗]

(iii) f(x)=∑10 [𝑎⃗𝑏⃗⃗𝑐⃗]


𝑘=1 |𝑥 − 𝑘|
[(𝑎⃗ + 2 𝑏⃗⃗) × (𝑏⃗⃗ + 2 𝑐⃗). (𝑐⃗ + 2 𝑎⃗)]
(b) (i) f(x)= [x], 0< x < 5 =
[𝑎⃗𝑏⃗⃗𝑐⃗]
(ii) f(x)= [x], 0 < x < 16
9[𝑎⃗𝑏⃗⃗ 𝑐⃗]
= =9
−𝑥 2 , 𝑥 ≤ 0 [𝑎⃗𝑏⃗⃗𝑐⃗]
(c) (i) f(x)= { 2
𝑥 ,𝑥 > 0
2
= 𝑥 𝑠𝑖𝑔𝑛(𝑥), 𝑤ℎ𝑒𝑟𝑒 𝑠𝑖𝑔𝑛(𝑥) =
−1 , 𝑖𝑓 𝑥 ≤ 0
{ ISI SUBJECTIVE SAMPLE
1 , 𝑖𝑓 𝑥 > 0
(ii) f(x)= 𝑥 2 𝑠𝑖𝑔𝑛(𝑥) + (𝑥 − PAPER WITH SOLUTIONS
1)2 𝑠𝑖𝑔𝑛(𝑥 − 1) + (𝑥 −
SET – 8
2)2 𝑠𝑖𝑔𝑛(𝑥 − 2).

9. If the angles of a triangle are in the 1. Find the maximum value of (𝟏 +


ratio 1: 2: 3, then the sides opposite to the 𝟐𝒙)𝟐 (𝟑 − 𝟐𝒙).
respective angles are in the ratio
Ans:- AM ≥ 𝐺𝑀
(a) 1 : √𝟐 : √𝟑 (b) 1 : √𝟑 :
1+2𝑥
2 (c) 1 : √𝟐 :3 2. +(3−2𝑥) 1+2𝑥 2
⇒ 2
≥ 3 √( ) (3 − 2𝑥)
(d) 1: 2: 3 2+1 2

4
Ans:- (b) ⇒ (1 + 2𝑥)2 (3 − 2𝑥) ≤ 22 (3)3

Let the angles be x, 2x, 3x, then 2. Minimize 3x+ 4y subject to 𝒙𝟐 𝒚𝟑 = 𝟔.


6x = 180° ⟹ 𝑥 = 30° Ans:- AM ≥ 𝐺𝑀
∴ Angles are 30°, 60°, 90° 3𝑥 3𝑥 4𝑦 4𝑦 4𝑦
+ + + + 3𝑥 4𝑦
⇒ 2 2 3 3 3
≥ 5√( 2 )2 ( 3 )3
5
∴ Ratio of sides is
3𝑥+4𝑦
⇒ ≥ 2, 𝑎𝑠 𝑥 2 𝑦 3 = 6
Sin 30° ∶ sin 60° : sin 90° 𝑖. 𝑒. 1 ∶ √3 ∶ 2 5

10. If a⃗ +b⃗ +c⃗ are non-coplanar, then ⇒ 3x + 4y ≥ 10


[𝒂 ⃗⃗𝒃
⃗⃗+𝟐𝒃 ⃗⃗+𝟐𝒄 ⃗⃗𝒄
⃗⃗+𝟐𝒂
⃗⃗]
⃗⃗𝒄
= (3x+ 4y) attains its minimum sub. To
[𝒂
⃗⃗𝒃 ⃗⃗]
𝑥 2 𝑦 3 = 6 when ‘=’holds in AM ≥ 𝐺𝑀
(a) 3 (b) 9 3𝑥 4𝑦 8
(c) 8 (d) 6 i.e. iff = ⇒ 𝑥 = 9𝑦
2 3

Ans:- (b) 3 4
∴ 𝑥2𝑦3 = 6 ⇒ 𝑦 = ,𝑥 =
2 3

167
Solving Mathematical Problems

3. Evaluate the determinant of the matrix = 𝑠𝑖𝑛2 x+𝑐𝑜𝑠 2 x =1


𝟏 + 𝒙 𝟏 𝒚𝟏 ⋯ 𝟏 + 𝒙 𝒏 𝒚𝒏
𝛥= ( ⋮ ⋱ ⋮ ) ∴ h (5) = h (10) = 10.
𝟏 + 𝒙 𝒏 𝒚𝟏 ⋯ 𝟏 + 𝒙 𝒏 𝒚𝒏
6. Test the convergence of the series x+
1 𝑥1 0 ⋯ 0 𝟐𝟐 𝒙𝟐 𝟑𝟑 𝒙𝟑 𝟒𝟒 𝒙𝟒
+ + + [Assume x> 0 and
Ans:- (a) |𝛥| = 𝑑𝑒𝑡 ( ⋮ ⋱ ⋮) × 𝟐! 𝟑! 𝟒!
1 𝑥𝑛 0 ⋯ 0 examaine all possible cases]
1 𝑦1 0 ⋯ 0
𝑛𝑛 .𝑥 𝑛
𝑑𝑒𝑡 ( ⋮ ⋱ ⋮) = 0 Ans: - we have 𝑢𝑛 = 𝑛!
1 𝑦𝑛 0 ⋯ 0
(𝑛+1)𝑛+1 .𝑥 𝑛+1
𝑢𝑛+1= (𝑛+1_!

4. Do there exist function f : R⟶R and g: 𝑢𝑛 1 𝑛𝑛 (𝑛+1)!


⤇ = . .
𝑢𝑛+1 𝑥 𝑛! (𝑛+1)𝑛+1
R⟶R ∋
1 𝑛𝑛 1 𝑛𝑛
f (g (x))= 𝒙𝟐 𝒂𝒏𝒅 𝒈 (𝒇(𝒙)) = 𝒙𝟑 ∀ 𝒙 ∋ ℝ = 𝑥 . (𝑛+1)𝑛 = 𝑥 . 𝑛𝑛{1+(1/𝑛)}𝑛

𝑢𝑛 1 1 1
Ans:- f(𝑥 3 )= f (g(f(x))= {𝑓(𝑥)}2 ⤇ lim 𝑢 = 𝑥 𝑙𝑖𝑚 1 = .
𝑛+1 [1+ ]𝑛 𝑒𝑥
𝑛
Now x ∊ {-1, 0} ⇒𝑥 3 = x ⇒ f(x) = {𝑓(𝑥)}2
1
∴by ratio test, ∑ 𝑢𝑛 is convergent if 𝑒𝑥 >
⇒ x ∊ {0, 1} 1 1
1 ⤇ 𝑥 < 𝑒 and ∑ 𝑢𝑛 is divergent if x >𝑒
Hence ∃ different a, b ∊ {-1, 0, 1} such that
1
f(a)= f(b) But then 𝑎3 = 𝑔 (𝑓(𝑎)) = But ratio test fails when x = 𝑒. Here we can
𝑔(𝑓(𝑏)) = 𝑏 3 , a contradiction . Thus the use logarithm test.
function f and g satisfying the given 1 𝑢𝑛 1
conditions don’t exist. When x = 𝑒, we have𝑢 = 𝑒. 1
𝑛+1 (1+ )𝑛
𝑛

5. Let f be a twice differentiable function ⤇𝑛𝑙𝑜𝑔 𝑢


𝑢𝑛
= 𝑛 [14 − 𝑛𝑏𝑙𝑜𝑔{1 + 1/𝑛}]
such that 𝑛+1

1 1 1
f″(x) = -f (x) ; f′ (x) =g (x) and h (x) =𝒇𝟐 =n[1-n(𝑛 − 2𝑛2 + 3𝑛3 … …)]
(x) +𝒈𝟐 (x). Given that h (5) =1 and find
1 1
h(10). = n[2𝑛-3𝑛2 + ⋯.]

Ans:- Take f (x) =sinx 1 1


= 2 − 3𝑛 + ⋯
f′ (x) = -cosx=g (x) 𝑢𝑛 1
∴ lim 𝑛𝑙𝑜𝑔 𝑢 = 2 < 1,
𝑛→∞ 𝑛+1
f″ (x) =- sinx= f (x)
1
2 2 ∴ ∑ 𝑢𝑛 is divergent at x= 𝑒.
h(x) = 𝑓 (x) +𝑔 (x)

168
Solving Mathematical Problems

7. Find the maximum value of ∬ 𝒅𝒙𝒅𝒚 as 3𝐴 3𝐵 3𝑐


𝑖. 𝑒. sin = 0 𝑜𝑟 sin = 0 𝑜𝑟 sin
a function of m , 0 < m< 1, where, limit of 2 2 2
𝒙𝟐 𝒚𝟐 =0
integration is 𝛥= {(x, y): 𝒎 + 𝟏−𝒎 ≤ 𝟏}.
2𝜋 2𝜋 2𝜋
Ans:- ∬ dxdy = 𝑜𝑟 𝐴 = 𝑜𝑟 𝐵 = 𝑜𝑟 𝐶 =
3 3 3
2𝜋 1 𝑥
√𝑚 √1 − 𝑚 ∫0 (∫0 𝑟𝑑𝑟) 𝑑𝜃 ; where √𝑚 = 𝐴
𝑦 𝐴𝑙𝑠𝑜, 𝑟 = (𝑠 − 𝑎) tan
𝑟𝑐𝑜𝑠𝜃, = rsin𝜃 2
√1−𝑚
𝐵
= (𝑠 − 𝑏) tan
= 𝜋 √𝑚 √1 − 𝑚 2
𝐶
𝑚+1−𝑚 = (𝑠 − 𝑐) tan
≤ 𝜋. (∵ AM≥ 𝐺𝑀) 2
2

𝜋 ⟹ 𝑟 = √3(𝑠 − 𝑎)𝑜𝑟 𝑟 = √3(𝑠 − 𝑏)𝑜𝑟 𝑟


= 2.
= √3(𝑠 − 𝑐)
8. Find maximum value of xyz subject to
𝒙𝟐 + 𝟐𝒚𝟐 + 𝟗𝒛𝟐 = 𝟔.
10. If (𝟏 + 𝒙 + 𝒙𝟐 + 𝒙𝟑 )𝟏𝟎𝟎 =
𝑥 2 +2𝑦 2 +9𝑧 2
Ans:- AM≥ 𝐺𝑀 => ≥ ∑𝟑𝟎𝟎 𝒓
𝒓=𝟎 𝒃𝒓 𝒙 𝒂𝒏𝒅 𝒌 =
3
3
√𝑥 2 . 2𝑦 2 . 9𝑧 2 ∑𝟑𝟎𝟎 𝟑𝟎𝟎
𝒓=𝟎 𝒃𝒓 , 𝒕𝒉𝒆𝒏 ∑𝒓=𝟎 𝒓. 𝒃𝒓 is

𝑥 2 +2𝑦 2 +9𝑧 2 3 (a) 50.𝟒𝟏𝟎𝟎 (b) 150. 𝟒𝟏𝟎𝟎


⇒( ) ≥ 18𝑥 2 𝑦 2 𝑧 2
3 (c) 300. 𝟒𝟏𝟎𝟎 (d) none of these
⇒ 23 ≥ 18𝑥 2 𝑦 2 𝑧 2 Ans:- (b)
8 2
⇒ (𝑥𝑦𝑧)2 ≤ ⇒ xyz ≤ 3. Given (1 + 𝑥 + 𝑥 2 + 𝑥 3 )𝑛 = ∑3𝑛
𝑟=0 𝑏𝑟 𝑥
𝑟
18

𝑜𝑟(1 + 𝑥 + 𝑥 2 + 𝑥 3 )𝑛 = 𝑏0 + 𝑏1 𝑥 +
𝑏2 𝑥 2 + ⋯ + 𝑏3𝑛 𝑥 3𝑛 …….(1)
9. If in a 𝜟𝑨𝑩𝑪, ∑ 𝐜𝐨𝐬 𝟑𝑨 = 1, then show
that 𝛥ABC is an obtuse angled triangle. Putting (1/x)

Ans:- In place of x, we get

We have A+ B + C= 𝜋 and ∑ cos 3𝐴 = (1 + 𝑥 + 𝑥 2 + 𝑥 3 )𝑛 = 𝑏0 𝑥 3𝑛 + 𝑏1 𝑥 3𝑛−1 +


1 𝑏𝑢𝑡 ∑ cos 3𝐴 = 1 + 𝑏3𝑛 ………..(2)
3𝐴 3𝐵 3𝑐
4 sin . sin . sin
2 2 2 Equating the co- efficient of similar powers
3𝐴 3𝐵 3𝑐 of x on the R.H.S. of (1) and (2) we have
⟹ sin . sin . sin =0
2 2 2 𝑏0 = 𝑏3𝑛 𝑏1 = 𝑏3𝑛−1 , 𝑏𝑟 = 𝑏3𝑛−𝑟

Given, k= ∑3𝑛
𝑟=0 𝑏𝑟 …..(3)

169
Solving Mathematical Problems

Let y= ∑3𝑛
𝑟=0 𝑟 𝑏𝑟 =
1 2𝜋
+ cos 𝑥 𝑖𝑓𝑥 < 3
{ 21 2𝜋
−(2 + cos 𝑥) 𝑖𝑓 𝑥 > 3
Then y = ∑3𝑛
𝑟=0[3𝑛 − (3𝑛 − 𝑟)]𝑏𝑟 =
3𝑛 ∑𝑟=0 𝑏𝑟 − ∑3𝑛
3𝑛
𝑟=0(3𝑛 − 𝑟)𝑏𝑟 2𝜋
1 𝜋 1
I = ∫03 (2 + cos 𝑥)𝑑𝑥 − ∫2𝜋(2 + cos 𝑥)𝑑𝑥
3𝑛 3

= 3𝑛𝑘 − ∑(3𝑛 − 𝑟)𝑏3𝑛−𝑟 [∵ 𝑏𝑟 = 𝑏3𝑛−𝑟 ] 1 2𝜋 2𝜋 1 𝜋 2𝜋


= 2. + sin − [2 (3 ) + sin 𝜋 − sin ]
𝑟=0 3 3 3

3𝑛 𝜋
= 6 + √3.
= 3𝑛𝑘 − ∑ 𝑟 𝑏𝑟 = 3𝑛𝑘 − 𝑦
𝑟=0

3𝑛𝑘 𝟏
∴ 2𝑦 = 3𝑛𝑘 ⟹ 𝑦 = 2. If a circle intersects a hyperbola y = 𝒙
2

Putting x= 1 in the given expansion, at 4 distinct points {𝒙𝒊 , 𝒚𝒊 : 𝒊 = 𝟏(𝟏)𝟒},

We get ∑𝑛𝑟=1 𝑏𝑟 = 4𝑛 ⟹ 𝑘 = 4𝑛 then prove that 𝒙𝟏 𝒙𝟐 = 𝒚𝟑 𝒚𝟒

Putting n= 100, we get Ans:- let the circle be 𝑥 2 + 𝑦 2 = 𝑎2


1
hyperbola be y = 𝑥 is given.
300.4100
y= = 150. 4100
2 1
For points of contact, 𝑥 2 + = 𝑎2
𝑥2

⇒ 𝑥 4 − 𝑥 2 𝑎2 + 1 = 0
1
It has 4 roots, so, Now, we know x = 𝑦
ISI SUBJECTIVE SAMPLE
PAPER WITH SOLUTIONS 1
∴ i.e. 𝑥3 = 𝑦 & 𝑥4 = 𝑦
1
3 4

SET – 9
∴ 𝑥1 𝑥2 = 𝑦3 𝑦4

𝝅 𝟏
1. Evaluate ∫𝟎 | 𝟐 + 𝐜𝐨𝐬 𝒙 |𝒅𝒙
3. If 𝜶𝟏 , 𝜶𝟐 ,…………..𝜶𝒏 be the roots of
1
Ans:- note that, |2 + cos 𝑥| = 𝒙𝒏 +1 =0, then (1- 𝜶𝟏 ) (1-𝜶𝟐 )…(1-𝜶𝒏 )=?
1 1
+ cos 𝑥 𝑖𝑓 cos 𝑥 > − 2 Ans:- 𝑥 𝑛 +1=(x- 𝛼1 ) (x-𝛼2 )…(x-𝛼𝑛 )
2
{ 1 1
− 2 − cos 𝑥 𝑖𝑓 cos 𝑥 < − 2 Putting x=1,

∴ 2=(1- 𝛼1 ) (1-𝛼2 )…(1-𝛼𝑛 )

170
Solving Mathematical Problems
𝟏 𝟏 𝟏 ∴ Possible value of 𝛽 are: 0, 3, 6, 9
4. The equation 𝟑 +𝟐 𝒔𝟐 + 𝟔 𝒔𝟑 = 𝒔 has
exactly _____ solution(s) in [0,1]. Possible values of 𝛶 are: 0, 7
1 1 1
Ans:- f(s)= 3 +2 𝑠 2 + 6 𝑠 3 − 𝑠 Possible values of 𝛼 are: 0, 1, 2, …, 10

f(1)=0 Since 𝛼+𝛽+𝛶= 10

f(0)= 3
1 ∴ Possible triplets (𝛼, 𝛽, 𝛶) will be

(0, 0, 10), (0, 7, 3), (3, 0, 7), (3, 7, 0), (6, 0,


1 𝑠2 +2𝑠−2 1
f′(s)=s+2 𝑠 2 − 1 = = 2 (𝑠 − 𝛼)(𝑠 − 4), (9, 0, 1)
2
𝛽);

here 𝛼= -1 -√3, 𝛽= -1+√3.


6. Find all pairs of prime numbers p, q
>0 𝑖𝑓 𝑥 < 𝛼 𝑜𝑟 𝑥 > 𝛽 such that p+q = 18(p−q). Justify your
∴f′(s)={
<0 , 𝛼 < 𝑥 < 𝛽. answer.

There are two roots. Ans:- Solving the equation we have 19q =
17p.

Pair of prime number is (17, 19)


5. The number of terms free from radical
𝟏 𝟏
sign in the expression of (𝟏 + 𝟑𝟑 + 𝟕𝟕 )𝟏𝟎

is -----------
7. In a group of n persons, each person is
Ans:- The term from radical sign must be of
1 1 asked to write down the sum of the ages
the from (constant)(33 )3𝑚 . (77 )7𝑛 of all the other (n − 1) persons. Suppose
the sums so obtained are S1, . . . , Sn. It is
where 3m+ 7n≤ 10 and m, n are non-
now desired to find the actual ages of the
negative integers.
persons from these values. Formulate the
This inequality has 6 solutions(m, n) ∊ {(0, problem in the form of a system of linear
0), (1, 0), (0, 1), (1, 1), (2, 0), (3, 0)} equations.

∴ Number of terms free from radical sign is Ans:- Let Pi be the actual age of person i.
6.
Then the equation is given by
Second method: General term in the given 𝑆𝑖
|10
̲ 𝛽 𝛶 P i = S i – ∑𝑖 .
𝛼 𝑛−1
expansion= 1 .3 .7 ,3 7
|𝛼
̲ |𝛽̲|𝛶̲̲

Where 𝛼+𝛽+𝛶= 10

171
Solving Mathematical Problems

8. For n ≥ 4, prove that 1! + 2! + · · · + n! ⟹ Lt Lt 𝑐𝑜𝑠 2𝑚 |𝑛


̲𝑥𝜋 = 0
𝑚→∞ 𝑛→∞
cannot be the square of a positive integer.
Thus 𝛼= 1, 𝛽= 0
Ans:- 1! + 2! + 3! = 9 ≡ 3 (𝑚𝑜𝑑 4)
Area of triangle =| ½ [1(1-1) -2 (1- 0)+ 2(0-
1! + 2! + 3! + …. + n! = Odd Number 1)]| = 2
By induction, we can say the given sum
can’t be the square of a positive number.
ISI SUBJECTIVE SAMPLE
PAPER WITH SOLUTIONS
9. If f is continuous in [0, 1], then
𝟐
𝒏
[ ] 𝟏 𝒋 SET – 10
𝐥𝐢𝐦 ∑𝒋=𝟎 𝒇 (𝒏) = ?
𝒏→∞ 𝒏

Ans:- Express limiting sum in the form of 1. Evaluate 𝐥𝐢𝐦{𝒙𝟐 (𝟏 + 𝟐 + 𝟑 +


𝒙→𝟎
1/2
definite integral. i.e. ∫0 𝑓(𝑥)𝑑𝑥 𝟏
⋯ + [∣𝒙∣])}

Ans.
10. Let 𝛼= 𝐋𝐭 𝐋𝐭 𝒄𝒐𝒔𝟐𝒎 𝒏! 𝝅𝒙, where x 1
𝒎→∞ 𝒏→∞ Lim{𝑥 2 (1 + 2 + 3 + ⋯ + [∣𝑥∣])}
is rational, 𝛽= 𝐋𝐭 𝐋𝐭 𝒄𝒐𝒔𝟐𝒎 𝒏! 𝝅𝒙, 𝑥→0
𝒎→∞ 𝒏→∞
where x is irrational, then the area of the 1 1
[∣ 𝑥 ∣] [∣ 𝑥 ∣ + 1] ∣ 𝑥 ∣ +1
triangle having vertices (𝛼, 𝛽), (-2, 1) and = lim 𝑥 2 = lim
𝑥→0 2 𝑥→0 2
(2, 1) is 1
=
(a) 2 (b) 4 2
(c) 1 (d) none of these ∣𝒙∣ 𝟏
2. Find 𝐥𝐢𝐦 𝐬𝐢𝐧 (𝟑 𝒙) .
𝒙→𝟎 √𝒙𝟒 +𝟒𝒙𝟐 +𝟕 √
Ans. (a)
1
𝑝 Ans. −1 ≤ sin (3 𝑥) ≤ 1
When x is rational: let x= 𝑞 . √

−∣ 𝑥 ∣
𝑝
Then |n̲ x= (1 . 2…q …n) 𝑞 = 𝑎𝑛 𝑖𝑛𝑡𝑒𝑔𝑒𝑟 √𝑥 4 + 4𝑥 2 + 7
∣𝑥∣ 1
∴ 𝑐𝑜𝑠 |𝑛2
̲𝑥𝜋 = 1 ≤ sin ( )
√𝑥 4 + 4𝑥 2 + 7 3√𝑥
∣𝑥∣
∴ Lt Lt 𝑐𝑜𝑠 2𝑚 |𝑛
̲𝑥𝜋 = 1 ≤
𝑚→∞ 𝑛→∞ √𝑥 4 + 4𝑥 2 + 7
When x is irrational |𝑛
̲𝑥 is not an integer Taking limits, we will have
∴ 0≤ 𝑐𝑜𝑠 2 |𝑛
̲𝑥𝜋 < 1 ∣𝑥∣ 1
0 ≤ √𝑥 4 sin (3 𝑥) ≤ 0
+4𝑥2 +7 √

172
Solving Mathematical Problems

So answer is 0. 5. Show that

𝟏 𝟏 𝟏
𝐥𝐢𝐦 𝒏𝟐 { √(𝟏 − 𝒄𝒐𝒔 ) √(𝟏 − 𝒄𝒐𝒔 ) √(𝟏 − 𝒄𝒐𝒔 ) … . . ∞ }
𝒏→∞ 𝒏 𝒏 𝒏
3. Evaluate the following two integrals
𝟏
directly and compare them =𝟐.
∬√𝒂∣𝒙∣≤𝟏,√𝒃∣𝒚∣≤𝟏 𝒅𝒙𝒅𝒚 and
Ans. Take y =
∬𝒂𝒙𝟐 +𝒃𝒚𝟐𝟐 ≤𝟏 𝒅𝒙𝒅𝒚 .
√(1 − 𝑐𝑜𝑠 1 ) √(1 − 𝑐𝑜𝑠 1) √(1 − 𝑐𝑜𝑠 1) … . . ∞
Ans. ∬√𝑎∣𝑥∣≤1,√𝑏∣𝑦∣≤1 𝑑𝑥𝑑𝑦 𝑛 𝑛 𝑛

1 1
1
√𝑎 √𝑏
= ∫ ∫ 𝑑𝑥𝑑𝑦
−1 −1
 y2 = y (1 − 𝑐𝑜𝑠 𝑛)
√𝑎 √𝑏 1
 y = (1 − 𝑐𝑜𝑠 𝑛)
4
=
√ab 1
So lim 𝑛2 𝑦 = lim 𝑛2 (1 − 𝑐𝑜𝑠 𝑛) =
𝑛→∞ 𝑛→∞
> ∬ax2 +by22 ≤1 dxdy 1 1
lim 𝑛2 𝑦 = lim 𝑛2 (1 − 1 + 2𝑛2 ) = 2
𝑛→∞ 𝑛→∞
𝜋
= ∞ 𝟐
√𝑎𝑏 6. Show that ∫−∞ 𝒆−𝒙 𝒅𝒙 = √𝝅 .

Ans.
𝒅𝒚 ∞ 2 2
4. Find 𝒅𝒙 where y = (𝒙𝒍𝒐𝒈𝒙 )(𝒍𝒐𝒈𝒙)𝒙 , (∫−∞ 𝑒 −𝑥 𝑑𝑥) =
∞ 2 ∞ 2
when x > 1. (∫−∞ 𝑒 −𝑥 𝑑𝑥) (∫−∞ 𝑒 −𝑥 𝑑𝑥) =
∞ 2 ∞ 2
Ans. Take log both sides, (∫−∞ 𝑒 −𝑥 𝑑𝑥) (∫−∞ 𝑒 −𝑦 𝑑𝑦)
∞ ∞ 2 +𝑦 2 )
logy = (logx)2 + xlog(logx) = ∫−∞ ∫−∞ 𝑒 −(𝑥 𝑑𝑥𝑑𝑦

Now differentiate w.r.t. x both sides, 2𝜋 ∞ 2 2𝜋 𝑑𝜃


= ∫0 ∫0 𝑒 −𝑟 𝑟𝑑𝑟𝑑𝜃 = ∫0 =𝜋
2
𝑑𝑦 2 1
= 𝑦 ( 𝑙𝑜𝑔𝑥 + + log(𝑙𝑜𝑔𝑥)) 7. A total of n balls, numbered 1 through
𝑑𝑥 𝑥 𝑙𝑜𝑔𝑥
n, are put into n urns, also numbered
Put the value of y and get the answer. 1 through n in such a way that ball i is
equally likely to go into any of the
urns 1, 2, 3, …, i. Find the expected
number of urns that are empty.

1 𝑖𝑓 𝑢𝑟𝑛 𝑖𝑠 𝑒𝑚𝑝𝑡𝑦
Ans. Xi = {
0 𝑜𝑡ℎ𝑒𝑟𝑤𝑖𝑠𝑒
E(Xi) = P(Xi = 1)

173
Solving Mathematical Problems
1 1 1 𝑖−1 satisfy x5y3 = x8y5 = e, then what is the
= (1 − 𝑖 ) (1 − 𝑖+1) … (1 − 𝑛) = 𝑛
relation between x & y?
𝑛−1
E(X) = ∑𝑛𝑖=1 𝐸(𝑋𝑖 ) = . Ans.
𝑛

x5y3 = x8y5 = x3y2 = x2 e = e


∑𝒌−𝟏
𝒓=𝟎 𝒙
𝟐𝒓
(from the given relation)
8. If ∑𝒌−𝟏 𝒓
is a polynomial in x for two
𝒓=𝟎 𝒙
values p and q of k, then roots of i.e. x = e, so y = x-1 = e.
equation 𝒙𝟐 + 𝒑𝒙 + 𝒒 = 0 cannot be
rational. Justify.
10. Let 𝝏 be the permutation:
Ans.
1 2 3 4 5 6 7 8 9
∑𝑘−1
𝑟=0 𝑥
2𝑟 3 5 6 2 4 9 8 7 1
∑𝑘−1 𝑟
is a polynomial,
𝑟=0 𝑥 I be the identity permutation and m
be the order of 𝝏, i.e.,
⟹ 1+𝑥 2 + 𝑥 4 + ⋯ +
m = min {positive integers n: 𝝏𝒏 = 𝟏}.
𝑥 2(𝑘−1) 𝑖𝑠 𝑑𝑖𝑣𝑖𝑠𝑖𝑏𝑙𝑒 𝑏𝑦 1 + 𝑥 + 𝑥 2 + ⋯ +
Then what is the value of m?
𝑥 𝑘+1
Ans.
1−𝑥2𝑘
1−𝑥2 1+𝑥 𝑘
⟹ 1−𝑥𝑘
𝑖. 𝑒. 1→3→6→9→1 : order 4
1+𝑥
1−𝑥

2→5→4→2 : order 3
is a polynomial in x.
7→8→7 : order 2
⟹ 1+𝑥 𝑘 is divisible by 1+ x
LCM (4, 3, 2) = 12 = m.
⟹ 1+(−1)𝑘 = 0

⟹ 𝑘 = 𝑎𝑛 𝑜𝑑𝑑 𝑝𝑜𝑠𝑖𝑡𝑖𝑣𝑒 𝑖𝑛𝑡𝑒𝑔𝑒𝑟

⟹ p, q are odd integers SHORT ANSWER TYPE


(since p, q are value of k) QUESTIONS (MAINLY
Now since coefficients of the quadratic
FOR MSTAT EXAM)
equation 𝑥 2 + 𝑝𝑥 + 𝑞 = 0 are odd integers Q1. If 𝒂𝟏 <𝒂𝟐 <………< 𝒂𝒎 , 𝒃𝟏 < 𝒃𝟐 <
and hence its roots cannot be rational. ……<, 𝒃𝒏 and also ∑𝒎 𝒊=𝟏 |𝒂𝒊 −
𝒏
𝒙|=∑𝒋=𝟏 |𝒃𝒋 − 𝒙|, where x is any real
number then prove that ai = bj for all i
9. Let G be a group with identity and n = m.
element e. If x and y are elements in G
Solution: let f(x)=|𝑎1 -x|+|𝑎2 -x|+…..+|𝑎𝑚 -x|

174
Solving Mathematical Problems

And g(x) = |𝑏1 -x|+|𝑏2 -x|+…..+|𝑏𝑛 -x|. and {(1, 0, 1, 0), (3, 0, 1, 0)} respectively.
Find a basic of 𝒘𝟏 ∩ 𝒘𝟐 . Also find a basis
Then we know only points of non-
of 𝒘𝟏 + 𝒘𝟐 containing {(1, 0, 1, 0), (3, 0, 1,
differentiability of f(x) is
0)}. (ᴪ : The set of all real numbers)
𝑎1 ,𝑎2 ,………, 𝑎𝑚 ,and only points of non-
differentiability of g(x) is 𝑏1 , 𝑏2 ,………, 𝑏𝑛 , Solution: 𝑤1 ={(1, 2, 3, 4) , (2, 1, 1, 2)}
𝑤2 = {(1, 0, 1, 0), (3, 0, 1, 0)}
Since, m, n are finite numbers and also
given that f(x)=g(x) Now we will calculate dim(𝑤1𝜐 𝑤2) which is
equal to number of independent rows in
𝑓(𝑎𝑖+ℎ)−𝑓(𝑎𝑖)
So, we may write, =

𝑔(𝑎𝑖+ℎ)−𝑔(𝑎𝑖)
∀ h.

1 2 3 4
So, RHL{f’(ai)}=RHL {g’(ai)}
2 1 1 2
And also, LHL{f’(ai)}=LHL {g’(ai)}
1 0 1 0
But as f(x) is non-differentiable at x=ai,
i.e. Rank(A)=4.
So, LHL{f’(ai)}≠RHL {f’(ai)},
LHL{g’(ai)}≠RHL {g’(ai)→g(x) is also not Now, dim(𝑤1∪ 𝑤2)=dim𝑤1+dim𝑤2 -
differentiable at x=ai. dim(𝑤1 ∩ 𝑤2 )

Now, since both the functions are equal so ⇰ 4 =2 + 2 - dim (𝑤1 ∩ 𝑤2 )


the points of discontinuity are same so m=n.
⇰ dim (𝑤1 ∩ 𝑤2 )=0.
To show the another part, we need to show
i.e. basis of (𝑤1 ∩ 𝑤2 )={(0, 0, 0, 0)}
ai=bi.
⇰ᴪ4 = 𝑤1⊕ 𝑤2
In a similar way, we can say, for any given
𝑏𝑟 there exists 𝑎𝑝 ⇰ basis of 𝑤2 can be extended to form basis
of 𝑤1 + 𝑤2 which is given by
Such that 𝑏𝑟 =𝑎𝑝 .
= {(1, 0, 1, 0), (3, 0, 1, 0), (0, 1, 0, 0), (0, 0,
So, {𝑎1 ,𝑎2 ,………, 𝑎𝑚 } and 0, 1)}
{𝑏1 , 𝑏2 ,………, 𝑏𝑛 } has one –to-one and
onto correspondence. Q3. Two players 𝒑𝟏 and 𝒑𝟐 are playing
the final of a chess championship, which
Therefore, m=n and every ai = bj if i = j. consist of a series of matches. Probability
of 𝒑𝟏 winning a match is 𝟐⁄𝟑and for 𝒑𝟐
is 𝟏⁄𝟑. The winner will be one who is
Q2. Suppose 𝒘𝟏 and 𝒘𝟐 are subspaces of
ahead by 2 games as compared to the
ᴪ𝟒 spanned by {(1, 2, 3, 4) , (2, 1, 1, 2)}
other player and wins at least 6 games.

175
Solving Mathematical Problems

Now, if the player 𝒑𝟐 wins first four Q4. Let 𝑿𝟏 , 𝑿𝟐 , … . , 𝑿𝒏 be a random


matches, find the probability of 𝒑𝟏 sample drawn from a continuous
winning the championship. distribution. The random variables are
ranked in the increasing order of
Solution:- 𝑝1 can win in the following magnitude. 𝑹𝒊 be the rank of the ith
mutually exclusive ways: sample. Find the correlation coefficient
(a) 𝑝1 wins the next six matches. between 𝑹𝟏 and 𝑹𝟐 .
(b) 𝑝1 wins five out of next six matches, Solution:- 𝑅𝑖 be the rank of 𝑋𝑖 .
so that after next six matches score
of 𝑝1 and 𝑝2 are tied up. This is 𝑅𝑖 be the random variable such that P(𝑅𝑖 =
1
continued up to ‘2𝑛 ’ matches (n≥ 0) 𝑟𝑖 ) =𝑛 ; ri=1(1)n .
and finally 𝑝1 wins 2 consecutive
matches. 𝑛(𝑛+1)
∴ ∑𝑛𝑖=1 𝑅𝑖 = , a constant quantity.
2
2
Now, probability of case (a) =(3)6 and And since 𝑅1 , 𝑅2 , … , 𝑅𝑛 is identical random
probability of tie after 6 matches (in case variable, now 𝑅𝑖 is th random variable and
2 1 26 ∑𝑛𝑖=1 𝑅𝑖 is a constant.
(b)) =(3)5(3) =35 .
∴ cov(𝑅1 , ∑𝑛𝑖=1 𝑅𝑖 )=0
Now probability that scores are still tied up
2 1 1 2 4
after another next two matches=3 . 3 +3. 3=9. ⇰ cov(𝑅1 , 𝑅1 + 𝑅2 + ⋯ + 𝑅𝑛 , )=0

[1st match is own by 𝑝1 and 2nd by 𝑝2, or , by ⇰var(𝑅1 ) +cov(𝑅1 , 𝑅2 ) + ….+cov(𝑅1 , 𝑅𝑛 )=0
reversively ]
⇰var(𝑅1 )+(n-1). Cov(𝑅1 , 𝑅2 )=0
Similarly probability that scores are still tied [∵𝑅𝑖 ’s are identically distributed; cov(𝑅𝑖 ,
up after another ‘2n’ matches=(9)𝑛 .
4 𝑅𝑗 )= cov(𝑅𝑖 )]

𝑣𝑎𝑟(𝑅 )
⇰Total probability of 𝑝1 winning the ⇰cov(𝑅1 , 𝑅2 )=- (𝑛−1)𝑖
championship
𝑛2 −1 (𝑛+1)
=− 12 =-
2 26 4 12
(2 2
= (3)6 +35 (∑∞
𝑛=0(9)
2
) ) (𝑛−1)
3
𝑛+1
𝑐𝑜𝑣(𝑅1 ,𝑅2 ) − 1
2 26 2 1 ∴𝛒= = 12
= − (𝑛+1) .
= (3)6 +35 (3)2 ( 4) √𝑣𝑎𝑟(𝑅1 )𝑣𝑎𝑟(𝑅2 ) (𝑛+1)(𝑛−1)
1− 12
9

17 2 6
= ( )
5 3

1088
Q5. Let X and Y be two random variables
= 3645 . with joint P. D. F.

f(x, y) = 1 if –y< x<y , 0<y <1

176
Solving Mathematical Problems

=0 elsewhere The conditional distribution of Y given X= x


is given by,
Find the regression equation of Y on X
and that of probability density function.

Solution:- 𝑓(𝑥,𝑦) 1
𝑓𝑦⁄ (𝑦)= = 1+𝑥 if –x < y < 1
𝑥 𝑓𝑥 (𝑥)
Here –y< x<y and 0<y <1 1 𝑦 𝑑𝑦 1 1
∴ E (Y|X) = ∫−𝑥 (1+𝑥) = (1+𝑥) .2 (1 − 𝑥 2 ) =
⇰ -1< x <1 , which is the marginal range of 1−𝑥
x. .
2

Again, y> -x and y > x Case 2. 0 < x < 1,

∴ y > max (𝑥1 -x) The conditional distribution of Y given X=X


is given by,
∴ max (𝑥1 -x) < y < 1
𝑓(𝑥,𝑦) 1
𝑓𝑌|𝑋 (𝑦)= = 1−𝑥 if x < y < 1
∴ Marginal PDF of X is given by, 𝑓(𝑥)

1 1+𝑥
𝑓𝑋 (𝑥) = ∫max(𝑥 (𝑥, 𝑦) 𝑑𝑦 , -1 < x < 1 Similarly, E (Y|X) =
−x) 2
1

Case –I:- -1 < x <0 ∴ If -1 < x < 1, then regression equation of


Y on X is given by,
∴ max (𝑥1 -x) = -x
1− |𝑥|
Y= .
1 2
∴ 𝑓𝑋 (𝑥) = ∫−x 𝑑𝑦 , if -1 < x < 0
The conditional distribution of X given Y =
= 1+ x if -1 < x <0 y is given by,
Case –II:- 0 < x< 1 𝑓(𝑥,𝑦) 1
𝑓𝑋|𝑌 (𝑥)= = 2𝑦 if –y < x < y
𝑓𝑌 (𝑦)
Max(x, -x) = x,
∴ E (X|Y)=0
1
∴ 𝑓𝑋 (𝑥) = ∫x 𝑑𝑦 , if 0 < x < 1
∴ Regression equation of X and Y is given
= 1- x if 0< x <1 by x =0.

Marginal PDF of Y is given by.


1 Q6. (a) Let 𝒇𝒏 be a sequence of continuous
𝑓𝑦 (𝑦) = ∫−y 𝑑𝑥 , if 0 <y < 1
real valued functions on, [0,1] which
= 2y if 0 <y < 1 converges uniformly to f . Prove that
𝟏
𝐥𝐢𝐦 𝒇𝒏 (𝒙𝒏 ) = 𝒇 (𝟐)for any sequence
Case 1. -1 <x < 0, 𝒏→∞
{𝒙𝒏 }converges to ½.

177
Solving Mathematical Problems

(b) Must the conclusion still hold if the Q7. Let {𝒙𝒏 : 𝒏 ≥ 𝟎} be a sequence of real
convergence is only point wise? Explain. numbers such that 𝒙𝒏+𝟏 = 𝝀𝒙𝒏 + (𝟏 −
𝝀)𝒙𝒏−𝟏 , 𝒏 ≥ 𝟏, for some 0< 𝝀 < 1.
Solution:- (a) Let {𝑥𝑛 } be a sequence in [0,1]
with 𝑥𝑛→1/2 as n→ Fix ∊>0 and let 𝑁0 ∊ (a) show that 𝒙𝒏 = 𝒙𝟎 + (𝒙𝟏 −
𝑁 be such that n ≥ 𝑁0 implies |𝑓𝑛 (𝑥) − 𝒙𝟎 ) ∑𝒏=𝟏 𝒌
𝒌=𝟎( 𝝀 − 𝟏) .
𝑓(𝑥)|< for all x∊[0,1]. Let δ>0 be such that
|𝑓(𝑥) − 𝑓(𝑦)|∊/2 ∀ x, y ∊ [0,1] with |𝑥 − (b) Hence, or, otherwise, show that 𝒙𝒏
𝑦| < 𝛿. Finally, let 𝑁1 ∊ 𝑁 be such that n ≥ converges and find the limit.
𝑁1 𝑖𝑚𝑝𝑙𝑖𝑒𝑠 |𝑥𝑛 − 1/2|<δ. Then n ≥ Max { Solution :- 𝑥𝑛+1 − 𝑥𝑛 = 𝜆𝑥𝑛 +(1 −
𝑁0 , 𝑁1 } inplise 𝜆)𝑥𝑛−1 − 𝜆𝑥𝑛−1 + 𝜆𝑥𝑛−1 − 𝑥𝑛
1
|𝑓𝑛 (𝑥𝑛 ) − 𝑓(2)| ≤ |𝑓𝑛 (𝑥𝑛 ) − 𝑓(𝑥𝑛 )| + =(𝜆 − 1)𝑥𝑛 +
1
|f(𝑥𝑛 ) − 𝑓(2)| 𝑥𝑛−1 (1 − 𝜆 − 𝜆 + 𝜆)

∊ ∊ = (𝜆 − 1)[𝑥𝑛 − 𝑥𝑛−1 ]
≤ + 2 = ∊.
2
= (𝜆 − 1)2 [𝑥𝑛−1 −
(b) Suppose the convergence is only 𝑥𝑛−2 ]
pointwise.

Then the conclusion is false, demonstrating
by an counter example: = (𝜆 − 1)𝑛 (𝑥1− 𝑥0 )

Defining 𝑓𝑛 (𝑥) to be the function, ∴ 𝑥𝑛 − 𝑥𝑛−1 = (𝜆 − 1)𝑛−1 (𝑥1− 𝑥0)

f(x)= 0
1
, if 0≤ 𝑥 < 2 − 2𝑛
1 𝑥𝑛−1 − 𝑥𝑛−2 = (𝜆 − 1)𝑛−2 (𝑥1− 𝑥0 )

1 1 1

= 2nx-(n-1), if 2 − 2𝑛 ≤ 𝑥 < 2
𝑥1− 𝑥0 = (𝜆 − 1)0 (𝑥1− 𝑥0 )
1
i.e. 𝑓𝑛 (𝑥)is constantly zero for, x< ½ - 2𝑛,
Adding we get, 𝑥𝑛− 𝑥0 = (𝑥1− 𝑥0 ) ∑𝑛=1
𝑘=0( 𝜆 −
then it increases linearly until it reaches ‘1’ 𝑘
1)
at x= ½ , and then it remains constantly ‘1’
for x> ½ . ∴𝑥𝑛 = 𝑥0 + (𝑥1− 𝑥0 ) ∑𝑛=1
𝑘=0( 𝜆 − 1)
𝑘

1 1 1
Now, define the sequence, 𝑥𝑛 = 2 − 𝑛, ∴ Lt 𝑥𝑛 = 𝑥0 + (𝑥1− 𝑥0 ). 1−𝜆+1, as n→∞.
n→∞

Then f( 𝑥𝑛 )=0 ∀ 𝑛 ∊ 𝑁 and 𝑥𝑛→1 for n→∞. 1


=𝑥0 + (𝑥1− 𝑥0 ). 2−𝜆.
2

Therefore, f( ½ )=1≠0 = lim 𝑓𝑛 (𝑥𝑛 ) . Q8. Let f:ℝ→ℝ be a continuous function


𝑛→∞
with |f(x)-f(y)|≥ |𝒙 − 𝒚 for every x,y,∊ℝ.
Is f one –to-one? Show that there can’t

178
Solving Mathematical Problems

exist three points a, b, c ∊ ℝ with a< b< c 1 1


⇰AB=[ ]
such that f(a)<f(c)< f(b). 1 1

Solution:- f: ℝ→ℝ such that |f(x)-f(y)|≥ |AB -λI|=(1 − 𝜆)2 − 1=0


|𝑥 − 𝑦| ⇰1 − 𝜆=± 1
𝑓(𝑥)−𝑓9𝑦)
or, | |≥ 0 ∀ 𝑥, 𝑦 ∊ ℝ. ⇰ 𝜆 = 0, 2.
𝑥−𝑦

𝑓(𝑦+ℎ)−𝑓(𝑦) So, 2 is an eigen value of AB.


⇰| ≥ 0, taning x=y+h.|

Q10. Let A and B be n×n real matrices
⇰|f’(y)|≥ 0 such that 𝑨𝟐 = 𝑨, 𝑩𝟐 = 𝑩. Let I-(A+B) is
⇰f is either an increasing or decreasing invertible. Show that R(A)=R(B).
function. Solution:- A[I –(A+B)]=A -𝐴2 -AB
⇰∀ 𝑎 < 𝑏 < 𝑐 → 𝑓(𝑎) < 𝑓(𝑏) < 𝑓(𝑐) = 𝐴2 − 𝐴2 -AB
Or, a> b> c→ 𝑓(𝑎) < 𝑓(𝑏) < 𝑓(𝑐) = -AB.
i.e. a< b<c s.t. 𝑓(𝑎) < 𝑓(𝑏) < 𝑓(𝑐). And,[ I –(A+B)]B=B -AB - 𝐵 2
Q9. (a) Let ṵ and ṽ eigenvectors of A =𝐵 2- AB - 𝐵 2
corresponding to the eigenvalues 1 and 3,
respectively. Prove that ṵ +ṽ is not an =-AB
eigenvector of A.
∴ rank(A)= rank[A(I – A-B)]
(b) Let A and B be real matrices such that
=rank(-AB)=rank(b).
the sum of each row of A is 1 and the sum
of each row of B is 2. Then show that 2 is Q11. Let P be a matrix of order n>1 and
an eigenvalue of AB. entries are positive integers. Suppose 𝑷−𝟏
Solution:-As ṵ and ṽ are given eigen vectors exists and has integer entries, then what
corresponding to eigen values 1 and 3, so, are the set of possible values of |p| ?

𝐴𝑢 = 1. 𝑢 ; 𝐴𝑣 = 3. 𝑣 = 3𝑣. Solution:- P has integer entries,

⇰𝜆1 + 𝜆2 + ⋯ . +𝜆𝑛 =trace(P)=integer.


⇰ A(u+v)=u+3v.

As, RHS is not multiple of u+v, so, u+v ⇰ ∑𝑖<𝑗 𝜆𝑖 𝜆𝑗 = sum of minors
can’t be eigen vector of A. ⁞
1/2 1/2 1 1
(b) A=[ ], B=[ ] ⇰∏𝑛𝑖=1 𝜆𝑖 |p| = integer,
1/2 1/2 1 1

179
Solving Mathematical Problems
1 ⇰a𝛒 +b=0, since, cov(x, y)=𝛒,
Then the eigen-values of 𝑃 −1 are 𝜆𝑖 and
cov(y,y)=var(y)=1 as,E(x)=E(y)=0
they are also integers
&v(x)=v(y)=1’
1
⇰ 𝜆𝑖 = 𝜆𝑖
⇰ b= -a𝛒.
⇰ 𝜆𝑖 = ±1 ∴ (a, b)=a(1-𝛒), a∊ℝ.
So, |P|=∏𝑛𝑖=1 𝜆𝑖 = ±1. ∴ S={(a, b) : (a,b)=a(1-𝛒); a∊ℝ }
Q12. Let X, Y be a bivariate normal ∴ dim(s)=1.
vector such that E(X)=E(Y)=0 and
V(X)=V(Y)=1. Let s be a subset of ℝ𝟐 and Q13. In a knockout tournament, 𝟐𝒏
defined by S={(a, b) : (ax+ by) is equally skilled players namely,
independent of Y}. 𝒔𝟏 , 𝒔𝟐 , 𝒔𝟑 , … . . , 𝒔𝟐𝒏 are participating. In
each round, player are divided in pairs at
(i) show that S be a sub space, random and winner from each pair moves
in the next round. If 𝒔𝟐 reaches semi-final
(ii) Find its dimension.
, then find the probability that 𝒔𝟏 will win
Solution:- S={(a, b) : (ax+by) is independent the tournament.
of Y }
Solution:- In a knockout tournament, 2𝑛
(i) (𝑎1 , 𝑏1), (𝑎2 , 𝑏2 )∊S. equally skilled players namely,
𝑠1 , 𝑠2 , 𝑠3 , … . . , 𝑠2𝑛 are participating.
Then 𝑎1 𝑥 + 𝑏1 𝑦 is independent of y,
similarly, Let 𝐸1 be the event that 𝑠1 wins the
tournament and
𝑎2 𝑥 + 𝑏2 𝑦 is independent of y.
𝐸2 be the event that 𝑠2 reaches the semifinal.
⇰ (𝛂𝑎1 +𝛃𝑎2 )x +(𝛂𝑏1 + 𝜷𝒃𝟐 )y is
independent of y. We are to obtain P(𝐸1 /𝐸2 ).

⇰ (𝛂𝑎1 +𝛃𝑎2 , 𝛂𝑏1 + 𝜷𝒃𝟐 ∊s). Since all the players are of equal skill and
there will be four person in the semifinal.
⇰ (𝑎1 , 𝑏1) +𝛃(𝑎2 , 𝑏2 )∊ s ∀ (𝜶, 𝜷) ∊ ℝ
2𝑛 −1 𝑐3 4
So, P(𝐸2 ) = = 𝑛.
Hence, S is a subspace. 2𝑛 𝑐4 2

(ii) (a, b)∊S. P(𝐸1 ∩ 𝐸2 ) =probability that 𝑠1 and 𝑠2 both


are in the semifinal & then 𝑠1 wins the
⇰ax +by is independent of y. semifinal and also in final
⇰ cov(ax +by, y) =0 2𝑛 −2𝑐2 1 1 3
= . 2 . 2 = 2𝑛(2𝑛−1)
2𝑛 𝑐4
⇰ acov(x,y) + bcov(y, y) =0

180
Solving Mathematical Problems
P(𝐸1 ∩𝐸2 ) Soln:- Let us define the random variable as
Hence, P[𝐸1 /𝐸2 ]= P(𝐸2 )
follows,
3 .2𝑛 3
=2𝑛 (2𝑛−1).4=4(2𝑛−1). X= no. of heads obtained by A.

Q14. Let Y, 𝒀𝟐 , 𝒀𝟑 be i.i.d. continuous Y= No. of heads obtained by B.


r.v.s for i=1, 2. Define 𝑼𝒊 as 𝑼𝒊 =1 if 1
X ∼bin (n+1, 2)
𝒀𝒊+𝟏 > 𝒀𝒊
1
Y ∼ bin (n, 2)
=0 ow
1
Then, (n+1-X) ∼ bin (n+1, )
Find the mean and variance of 𝑼𝟏 + 𝑼𝟐 . 2

1
1 (n-Y) ∼ bin (n, )
Solution:- E(𝑈𝑖 )=1.P[𝑌𝑖+1 > 𝑌𝑖 ]=2 2

1 i> P(A will have more heads than b)


E (𝑈𝑖 )2=12 .P[𝑌𝑖+1 > 𝑌𝑖 ]=2
=(x>Y)
1 1 1 =P(n+1-X > n-Y)
V (𝑈𝑖 ) =2 − 4 =4 =P (Y >X -1)
1 1 =P(Y≥ 𝑋)
E (𝑈1 + 𝑈2 )= 2 + 2 =1. =1 –P(X>Y)
E (𝑈1 𝑈2 ) =1.1.P[𝑌2 > 𝑌1 , 𝑌3 > 𝑌2 ] = ∴ 2P(X>Y)=1
1
P[𝑌3 >𝑌2 > 𝑌1]=6 . 1
⇰ P(X>Y)=2 .
1
Cov(𝑈1 , 𝑈2 )= E(𝑈1 𝑈2 )-E(𝑈1 )E(𝑈2 ) = -12
ii> P(A and b have equal number of
∴ V(𝑈1 + 𝑈2 )=V(𝑈1 )+V(𝑈2 )+2cov(𝑈1 , 𝑈2 ) heads)
1 =P(X=Y)
=3 .
= ∑𝑛𝑖=1 𝑃(𝑋 = 𝑖0 𝑃(𝑌 = 𝑖)
Q15. A and B have respectively (n+1) and =
𝑛+1 1 𝑖 1 𝑛+1−𝑖 𝑛
n coins. If they toss their coin ∑𝑛𝑖=1 ( ) (2) (2) .( 𝑖 )
𝑖
simultaneously. What is the probability
1 𝑖 1 𝑛−𝑖
that, ____ ( ) ( )
2 2
𝑛+1 𝑛 1 2𝑛+1
i>A will have more heads than B. =∑𝑛𝑖=1 ( 𝑖 ) . ( 𝑖 ) (2) =
1 2𝑛+1 𝑛+1 𝑛
ii> A and B will have an equal number of (2) ∑𝑛𝑖=1 ( ) (𝑖)
𝑖
heads. 1 2𝑛+1 (𝑛+1)! 𝑛!
= (2) ∑𝑛𝑖=1 . 𝑖!(𝑛−𝑖)!
𝑖!(𝑛−𝑖+1)!
iii> B will have more heads than A. 1 2𝑛+1 2𝑛+1
= (2) ( )
𝑛
iii> P(B have more heads than A)

181
Solving Mathematical Problems

=P(y>x) (b) P(more than k misprints)


=1-P(X≥ 𝑌)
=P(x>k)
= 1-P(X=Y)-P(X>Y)
1 2𝑛+1 2𝑛+1 1
=1- (2) ( )−2 =P(X≥ k-1)
𝑛
1 1 2𝑛 2𝑛+1
=2[1- (2) ( )] = 1 – P(X≤ k-1)
𝑛
𝑒 −𝜆 .𝜆𝑥
Q16. A book of N pages contains on the = 1- ∑𝑘=1
𝑥=0 𝑥!
average λ misprints per page. Estimate
the probability that a page drawn at Q17. A certain mathematician carries two
random contains, match boxes in his pocket, each time he
wants to use a match, he selects one of
(a) at least one misprints. boxes at random. Each pocket contain n
matchsticks.
(b) More than k misprints.
(a) Find the distribution of the number of
Solution:- Let us define the random variable
sticks in one box, while the other is found
X as follows,
empty.
X= no. of misprints per page,
(b) Also find the distribution of the
The book contains λ misprints per page on number of sticks remaining in one box
an average. become empty.

Since the number of trials i.e. the no. of Solution:- (a) Let us define a r.v. X denoting
words is very large and probability of a the number of the matchsticks remaining in
misprint is very small, hence according to the match box when the other box is found
the definition of poisson distribution, empty.

X ∼ p(λ) Let𝑋𝑖𝑗 , I, j, i≠j denotes the number of


matchsticks remaining in the ith box cohen
𝑒 −𝜆 .𝜆𝑥
∴ P(X = x) = , x=0, 1, 2, …; λ>0 the jth box is found to be empty.
𝑥1

=0 , otherwise The mass points of X are 0, 1, …., N

(a)P(at least one misprint) For any such mass point x,

= P(X≥ 1) P[X=x] =P(𝑋12=x) +P(𝑋21=x)

= 1-P(X<1) We consider the distribution of 𝑋12.

= 1- P(X=0) The second box will be found empty if the


box is chosen for the (N+1) th time. At that
= (1- 𝑒 −𝜆 ) time the first box contain x matches if (N-x)
matches have already taken from it. If the

182
Solving Mathematical Problems

selection of the second box is regarded as (𝒏+𝑿)


Find the distribution of and find out
𝟐
success, then the event.
E(X).
P[𝑋12=x]=P[(N-x) failures occur preceeding
Solution:- R denotes no. of steps at right
the (N+1)th success].
after taking n steps.
1
=p[Z=N-x], where Z ∼ NB(N+1,2) ∴ R∼ Bin(n, p)

Similarly for P[𝑋21=x] L denotes no. of steps at left after taking n


steps.
(b) Let us define a random variable Y
denoting the number of matchsticks ∴ L∼ Bin(n,1-p)
remaining in a matchbox when the
other match box becomes empty. Let us define,

Let 𝑌𝑖𝑗 , I, j, i≠ 𝑗denotes the number of X: the position of the drunk and after n
steps.
matchsticks remaining in the ith box
when jth box becomes empty. R + l= n,
The mass points of Y are 0, 1, 2, …, n. r- l=X.
P[Y=y]=p[𝑌21 =y]+P[𝑌12 =y] 𝑛+𝑋
∴ 2R= n+X ⇰ R= ∼ Bin(n,p)
2
1
Now, P[𝑌12 =y]p[Z=n-y] , Z ∼ N.B(N, ) 𝑛+𝑋
2 ∴ E( )=nP
2
1 1
=(N+N-y-1C N-y) (2)𝑁 (2)𝑁−𝑦 𝑛
⇰ E(X) =2[np - 2]
1
=(2N-y-1C N-y) (2)2𝑁−𝑦 1
=2n (p - 2)
1
Similarly, P[𝑌21 =y]=(2N-y-1C N-y) (2)2𝑁−𝑦 =n (2p-1).
1
∴ P[Y=y] ]=(2N-y-1C N-y) (2)2𝑁−𝑦+1 Q19. Let X be an R.V. with mean 𝜇 and
variance 𝝈𝟐 >0.

If 𝝃𝒒 denotes the 𝒒𝒕𝒉 quantile of X, show


Q18. A drunk man performed a random that
walk over the position 0, ±𝟏, ±𝟐,…. The
drunk man stars from the point o. He 𝟏−𝒒 𝒒
𝜇-𝜎 √ ≤ 𝝃𝒒 ≤ 𝜇+𝜎 √𝟏−𝒒.
takes successive unit steps with 𝒒

probability p at right and probability (1-


ANS:- We know that 𝜉𝑞 statistics the
p) at left. His steps are independent. X be
a location of the drunk man after taking incauality p(X≤ 𝜉𝑞 )≥q
n- steps,

183
Solving Mathematical Problems
𝑋−𝜇 𝜉𝑞 −𝜇 =P (𝜇-3𝜎≤ 𝑋 ≤ μ + 3σ )
∴ P( ≤ )≥𝑞
𝜎 𝜎
1 μ+3σ
𝜉𝑞 −𝜇 = 2 ∫μ−3σ 𝑒 −|𝑥| 𝑑𝑥
If 𝜉𝑞 <𝜇, i.e. <0, we have from one
𝜎
sided chebyshev’s inequality, = ∫0
μ+3σ −𝑥
𝑒 𝑑𝑥 [Since the integrand is an
𝑋−𝜇 𝜉𝑞 −𝜇 1 even function]
q≤ 𝑃[ ≤ ]≤ 𝜉𝑞 −𝜇
𝜎 𝜎 1+( )2
𝜎
= 1-𝑒 −(μ+3σ)
1
∴ q≤ 𝜉𝑞 −𝜇 =.95 [X∼ Laplace (0, 1)]
1+( )2
𝜎

𝜉𝑞 −𝜇 2 1−𝑞 By Chebyshev’s inequality,


⇰( ) ≤
𝜎 𝑞
1
P [|X − μ| ≥ 3𝜎]≤ 32
1−𝑞 𝜉𝑞 −𝜇 1−𝑞
⇰ -√ ≤ ≤√ 1 8
𝑞 𝜎 𝑞
⇰ P [|X − μ| ≤ 3𝜎] ≥ 1 − 9=9=.88
1−𝑞 1−𝑞
⇰ 𝜇-𝜎√ ≤ 𝜉𝑞 ≤ 𝜇+𝜎√ .(Proved) Hence the given probability and the
𝑞 𝑞
Chebyshev’s upperbound is nearer to each
Q20. Let g be a non – negative non other.
decreasing function, prove that if
Q22. For the r. v. X having the following
E(𝐠(|𝐗 − 𝛍|)) exists, where 𝜇= E(X), then
(𝐠(|𝐗−𝛍|)) PDF
prove that if P[|𝐗 − 𝛍| > 𝒕]<
𝒈(𝒕)
𝒆−𝒙 .𝒙𝝀
f(x)= , x> 0 show that P (0< X
𝐸{𝑔|X−μ|} √𝝀+𝟏
ANS:- P[g|X − μ| > 𝑔(𝑡)]≤ 𝝀
𝑔(𝑡) <2(𝝀 + 𝟏))> 𝝀+𝟏
But, g|X − μ| > 𝑔(𝑡)
ANS:- E(X)= (𝜆 + 1)=Y(X)
⇔|X − μ|>t [∵ g is non decreasing &
From Chebyshev’s inequality,
non-negative function]
X−μ 1
𝐸{𝑔|X−μ|} P [| |< t]> 1-𝑡 2
∴ P [|X − μ|>t] < . (Proved) 𝜎
𝑔(𝑡) 1 𝜆
1-𝑡 2 =𝜆+1
Q21. For a Laplaces distribution with
𝟏 1
PDF f(x )= 𝟐 𝒆−|𝒙| , -∞ < 𝑥 < ∞. ⇰ P[-𝜎t < (X − μ)<𝜎t]> 1-𝑡 2
⇰ t= √𝜆 + 1
Find the minimum probability of an
observation lying with in the mean ± 3 s. ⇰ P[-(√𝜆 + 1) (√𝜆 + 1)< (X-
1
d. interval. √𝜆 + 1)<λ+1]> 1- 𝜆+1
ANS:- P (|X − μ| ≤ 3𝜎 𝜆
⇰ P [0, X, 2(𝜆 + 1)]> 𝜆+1

184
Solving Mathematical Problems

Q23. Let the random variables X and Y P[T= t]= P[X=t, Y>t]+ P[Y=t, X>t] + P[x=t,
have the joint probability density Y=t]
function(x, y) given by
=P[x=t]P[Y> t]+ P[X> t]P[Y=t]+p
𝟐 (𝒙+𝟏) [X=t]P[Y= t]
f(x, y)= 𝒚 𝒆 − 𝒚 ; 𝒙 ≥ 𝟎, 𝒚 ≥ 𝟎

=0 ;otherwise Now, P[Y≤t]= P[Y=1]+ P[Y=2]+…+P[Y=t]


1 1 1 1 1
Are the random variables x and Y =(1-2) + (2 − 3) + …+ ( 𝑡 − 𝑡+1)
independent? Justify your answer .
1
= 1- 𝑡+1
Solution:- The marginal p d f of x is given
by : 1
⇰ P[Y>t]= 𝑡+1

F(x) =∫0 𝑦 2 𝑒 −𝑦(𝑥+1) 𝑑𝑦
1
let, 𝑦(𝑥 + 1)=t, (x+1) dy =dt. Similarly, P[X> t]= 𝑡+1

∞ 𝑡2 1 1 1 1 1 1
= ∫0 𝑒 −𝑡 . (𝑥+1) 𝑑𝑡 Hence, P[T=t]=( 𝑡 − 𝑡+1). 𝑡+1 + 𝑡 − 𝑡+1).
(𝑥+1)2
1 1 1
+ ( 𝑡 − 𝑡+1)2
1 ∞ 𝑡+1
= (𝑥+1)3 ∫0 𝑡 2 𝑒 −𝑡 𝑑𝑡
1 1
=𝑡(𝑡+1)2 + 𝑡 2 (𝑡+1)
√(3) 2
=(𝑥+1)3 =(𝑥+1)3 ; x ≥ 0
1 1
∴ E(T) =∑∞
𝑡=1 + ∑∞
𝑡=1
(𝑡+1)2 𝑡 (𝑡+1)
The marginal p d f of Y is given by :
𝜋2 1 1
∞ =( 6 − 1) + ∑∞
𝑡=1 ( 𝑡 − 𝑡+1)
F(y) = ∫0 𝑦 2 𝑒 −𝑦(𝑥+1) 𝑑𝑥
𝜋2
∞ 𝑦2 = 6 -1 + 1
= ∫0 −𝑦 . 𝑒 −𝑦(𝑥+1)
𝜋2
= 𝑦𝑒 −𝑦
;y≥0 =6

As, f(x, y)≠f(x), f(y) Q25. Suppose a random vector (X, Y) has
joint probability density function
So, X and Y are not independent.
f(x, y) =3y on the triangle bounded by the
Q24. Let X and Y are i.i.d. with P[X= x] = lines y=0, y=1 –x and y= 1 +x
𝟏 𝟏
− 𝒙+𝟏, x=1, 2, …
𝒙
find the marginal PDF of X and Y.
𝟏
Find E[Min(X, Y)]. Compute (Y| X≤ 𝟐).

Solution:- Let T= min (x, Y). Solution:- The joint PDF of the random
vector

185
Solving Mathematical Problems

(X, Y) is given by ii> If y be an another r. v. defined as

F(x, y) = 3y, on the shaded triangle of figure Y=0 if X < a ; 1 if X > a


1.
Then S.T. Y and Z=|X-a| will be
From the figure, the range of the marginal independently distributed.

Distributions of X and Y are given by, ANS:- i> It is given that, F(a+x) +F(a-x) =1

0 < x < 1, -1 < y <1, respectively, From the above equation it is clear that the
disth of X is symmetric about ‘a’,
Now, f(x, y) = 3y if 1 –x <y <1+x, 0 < x <
1 Hence, E(X-a) =0

Now, 1+ x> y ⇰ E(X)=a

⇰x > y -1 Ii > It is given that,

And 1 –x< y Y ={ 0 if X < a ; 1 if X≥a

⇰ x-1 > -y And Z=|X-a|

⇰x>1–y Now form the equation, F(x+a) +F(a-x)=1,


1
∴ x> max {(1-y), (y-1)} It is clear that F(a)=2 [since the
∴ max {(1-y), (y-1)}< x < 1 distribution is symmetric about ‘a’]
1 1
Case I:- -1 < y < 0 ∴ Y= { 0 with prob. 2 ; 1, with prob. 2

If -1< y <0 Now, for same Z > 0

⇰ -2 <y-1 < -1 P[Z≤z , Y=0]

And 1 > -y > 0 =P [|X-a|≤ z , X < a]

⇰ 2 > 1-y > 1 = P[-x +a≤ z≤ X-a , X < a]


∴ max {(y-1), (1-y)}= 1-y = P[a-z≤ X≤ a+z, X < a]

∴ 1-y < x < 1 =P [a-z≤ X≤ min (a+z, a)]

Q26. Let x be a continuous random = P [a-z≤ X≤a]


variable with distribution function f(x),
which is such that F(a+x) +F(a-x) =1 for =F(a) –F(a-z)
some fixed a. 1
= 2 – F (a-z)
i> Show that E(X) =a

186
Solving Mathematical Problems
1 Now, E (𝑌𝑥 |X = 1)=M
= 2 [F(a+z) –F(a- z)]
𝑀+𝑚1 𝑀+𝑚2 𝑀+𝑚𝑛
1 E (𝑌𝑥 |X = 2) = + + ….. +
= 2 P [a-z ≤ X ≤ a+ z] 𝑛 𝑛 𝑛

𝑚
1 =M +
= 2 P [|X|≤ a+z] 𝑛

2𝑚
1 E (𝑌𝑥 |X = 3) = M +
= 2 P [|X - a|≤z] , since a > 0 𝑛

(𝑥−1)𝑚
1 In general, E (𝑌𝑥 |X = x) = M +
= 2 P [Z≤z] 𝑛

𝑚
=P [Z≤z]. F (a) Now, E (Y)= ∑𝑛+1
𝑥=1 [𝑀 + (𝑥 − 1) 𝑛 ].P (X=x)

=P [Z≤z]. P [Y=0] Now, P (X= x)= P(X drawings are required


to get the coin A)
Similarly, it can be shown that,
𝑛 𝑛−1 𝑛−2 𝑛−𝑥+1 1
=𝑛+1 . . 𝑛−1 … . 𝑛−𝑥+2 . 𝑛−𝑥+1
P [Z≤z , Y=1]=P [Z≤z]. P [Y=1] 𝑛

1
Hence, =𝑛+1

P [Z≤z]. P [Y= y]= P [Z ≤z, Y=y] 𝑚 1


∴ E(Y)= ∑𝑛+1
𝑥=1 [𝑀 + (𝑥 − 1) 𝑛 ] . 𝑛+1

Hence, Y and Z are independently 1 𝑚 𝑛(𝑛+1)


distributed. = [(𝑛 + 1)𝑀 + . ]
(𝑛+1) 𝑛 2

𝑚
Q27. A bag contains a coin of value M = M + 2.
and a number of other coins whose
aggregate value is m. A person draws So the required value of the expectation is
𝑚
coins one at a time till the draws the coin (M + 2 ).
of value M. Find the value of his
expectation. Q28. Let X & Y be two joining
distributed continuous random variable
ANS:- Let the coins be A, 𝐵1 , 𝐵2 , … . , 𝐵𝑛 . with joint PDF,
Value of A= M & value of 𝐵𝑖 = 𝑚𝑖 (say) 𝟏 𝟏
𝒇𝑿𝒀 (𝒙, 𝒚) = 𝟐𝝅 exp [- {𝒙𝟐 −
√𝟏−𝑷𝟐 𝟐 (𝟏−𝑷𝟐 )
Such that ∑𝑛𝑖=1 𝑚𝑖 = 𝑚. 𝟐𝑷𝒙𝒚 + 𝒚𝟐 }], x ∊ ℝ ,y ∊ ℝ
Let 𝑌𝑥 be the value of the coins if x
I > find the marginal PDF of X
drawings are needed, x= 0, 1, 2, …., n+1.
ii> Find the conditional PDF of Y for
And Y is the total value of the coins
given X = x.
eventually.

Solution :- i> 𝑓𝑋 (𝑥) = ∫−∞ 𝑓𝑋𝑌 (𝑥, 𝑦)𝑑𝑦
Now, E(Y)=E {E(𝑌𝑥 |X = x)}

187
Solving Mathematical Problems
1 ∞ 1 Solution:- The joint PDF of X & y is given
= 2𝜋 ∫−∞ exp [− {𝑥 2 − 2𝑃𝑥𝑦 +
√1−𝑃 2 2 (1−𝑃 2 )
by
𝑦 2 }]dy
1
1 − (𝑥 2 +𝑦 2 )
1 ∞ 1 2 F(x,y)=2𝜋𝜎2 𝑒 2𝜎2 , x∊ℝ, y
= 2𝜋 2
∫−∞ exp [− 2 (1−𝑃 2 )
{(𝑦 − 𝑃𝑥) +
√1−𝑃
2 2 ∊ ℝ, 𝜎 > 0
(1 − 𝑃 )𝑥 }]dy

1 𝑥 2⁄ ∞ (𝑦−𝑃𝑥)2
Let us consider a square S, with vertices (a, -
= 2𝜋 𝑒− 2 ∫−∞ exp[− 2 (1−𝑃2 )] 𝑑𝑦 a), (a, a), (-a, -a)
√1−𝑃 2

Since,
1 ∞
∫−∞ exp[− 2𝜎
1
(𝑦 − 𝜇𝑌)2 ]= 1 The area of the square =4𝑎2 = s
√2𝜋𝜎𝑌 2
𝑌
Consider a circle C with radius = r, and the
Here,Y =Px and 𝜎 2 𝑌 = (1 − 𝑃2 ) centre at (0, 0)
1 ∞ 1
∴ √2𝜋 ∫−∞ exp[− 2(1−𝑃2 ) (𝑦 − Area of C = 𝜋𝑟 2
√(1−𝑃2 )
2
𝑃𝑥) ] 𝑑𝑦 Hence, 𝜋𝑟 2 =4𝑎2 [given]
∞ 1
∴ ∫−∞ exp[− 2(1−𝑃2) (𝑦 − 2𝑎
⇒r =
√𝜋
𝑃𝑥)2 ] 𝑑𝑦=√2𝜋(1 − 𝑃2 )
Therefore, a < r < √2𝑎
𝑥 2
1
∴𝑓𝑋 (𝑥) = 𝑒 − ⁄2 , x∊ℝ,
√2𝜋
Now, P [XY∊ S] =∬𝑥,𝑦∊𝑆 𝑓 (𝑥, 𝑦)𝑑𝑥𝑑𝑦
∴ X∼ N(0, 1).
𝑎 𝑎
= 4 ∫0 ∫0 𝑓 (𝑥, 𝑦)𝑑𝑥𝑑𝑦 [ By symetry]
ii> Conditional PDF of Y for Given X= x is

1 𝑥2⁄ (𝑦−𝑃𝑥)2 P [X, Y∊ C]= ∬𝑥,𝑦∊𝐶 𝑓 (𝑥, 𝑦)𝑑𝑥𝑑𝑦


𝑒− 2 𝑒−
𝑓𝑋𝑌 (𝑥,𝑦) 2𝜋
√1−𝑃2 2(1−𝑃2 )
𝑓𝑦 𝑦
( ) = = 1 −𝑥2⁄
=𝑥 𝑥 𝑓𝑋 (𝑥) 𝑒 2 Now in the first quadrant,
𝑥 √2𝜋

1 (𝑦−𝑃𝑥)2 P [X, Y∊ C] - P [X, Y∊ S]


= 𝑒 − 2(1−𝑃2 )
√2𝜋 (1−𝑃 2 )

𝑌
= ∬𝑥,𝑦∊𝐴 𝑓 (𝑥, 𝑦)𝑑𝑥𝑑𝑦 -
i.e. 𝑋=x∼ N (𝑃𝑥 , ( √1 − 𝑃2 )2 ) , ∞ < 𝑦 < ∞
∬𝑥,𝑦∊𝐵 𝑓 (𝑥, 𝑦)𝑑𝑥𝑑𝑦 [From the figure
𝑦
𝑌 ∞ ( )𝑑𝑦 canceling the common region]
E [𝑋 = 𝑥] =∫−∞ 𝑓𝑋𝑌 𝑥 =Px.
A = shaded region,
Q29. Let x and Y have the circular
normal distribution with zero mean, i.e. X B = dotted region.
& Y ∼ 𝑵𝟐 (0, 0, 𝝈𝟐 , 𝝈𝟐 , 𝟎). Cosider a circle
C and a square S of equal area both with Now, if (x, Y)∊ A, then,
ac (0, 0). Prove that, P [(X , Y)∊s].
𝑥 2 + 𝑦 2 <𝑟 2

188
Solving Mathematical Problems

(𝑥 2 +𝑦 2 ) 𝑟2 Q31. Let X and y be two r. v.’s with


⇒- > - 2 𝜎2
2 𝜎2
means zero, variance unity and
1 𝑟2 correlation coefficient P, then S. T.
⇒ f(x, y)> 2𝜋𝜎2 𝑒 − 2 𝜎2 ………………<i>
E[Max(𝑿𝟐 , 𝒀𝟐 )]≤ 𝟏 + √𝟏 − 𝑷𝟐
If (X, Y)∊ B
Solution:- Max(𝑋 2 , 𝑌 2 ) + Min(𝑋 2 , 𝑌 2 ) =
𝑥2 + 𝑦2 > 𝑟2 𝑋2 + 𝑌2

1 𝑟2 Max(𝑋 2 , 𝑌 2 ) − Min(𝑋 2 , 𝑌 2 ) =
⇒ f(x, y)< 2𝜋𝜎2 𝑒 − 2 𝜎2 ………………<ii>
𝑋2 − 𝑌2
From <i> & <ii> we get, 1
Max(𝑋 2 , 𝑌 2 ) = [ (𝑋 2 + 𝑌 2 ) +
2
∬𝑥,𝑦∊𝐴 𝑓 (𝑥, 𝑦)𝑑𝑥𝑑𝑦 > ∬𝑥,𝑦∊𝐵 𝑓 (𝑥, 𝑦)𝑑𝑥𝑑𝑦 |𝑋 2 − 𝑌 2 |]
1
∴P [X, Y∊ C] > P [X, Y∊ S] E [Max(𝑋 2 , 𝑌 2 )] = 2 [𝐸 (𝑋 2 ) + 𝐸(𝑌 2 ) +
𝐸|(𝑋 + 𝑌)(𝑥 − 𝑌)|]
This inequality similarly holds for the other
quadrants. 1
=2 [1 + 1 𝐸|(𝑋 + 𝑌)(𝑥 − 𝑌)|]
𝐚 𝐱𝟐
𝟏 − 𝐝𝐱
Q30. Show that, ∫𝟎 𝐞 𝟐 < By C – S inequality,
√𝟐𝛑
𝟏
𝟐
√𝟏 − 𝐞−𝟐𝐚𝛑
𝟐 𝐸 2 [|(𝑋 + 𝑌)(𝑥 − 𝑌)|]
≤ 𝐸(𝑋 + 𝑌)2 𝐸(𝑋 − 𝑌)2
Solution:- P (x, Y∊ S)=4
1
i.e. 𝐸 2 |𝑋 2 − 𝑌 2 | ≤ (2 + 2𝐸(𝑋𝑌))(2 −
𝑎 𝑎 1 − (𝑥 2 + 𝑦 2 )
∫0 ∫0 2𝜋𝜎2 𝑒 2 𝑑𝑥𝑑𝑦 2𝐸(𝑋𝑌))
2 2
4 𝑎 − 𝑥 𝑑𝑥 2 ⇒ E |𝑋 2 − 𝑌 2 | ≤ √1 − 𝑃2
= [∫0
𝑒 2 ]
2𝜋
1 2
𝜋⁄ 𝑟 1 −
1 ∴ E [ max(𝑋 2 , 𝑌 2 )]≤ 1 +2 . √1 − 𝑃2
2
P (x, Y∊ C) = 4 ∫0 2 ∫0 2𝜋 𝑒 2𝜎2 𝑅 . RdRd0
≤ 1+ √1 − 𝑃2
𝑟2
4 −
=4 (1-𝑒 2 )
Q32. (a) S.T. for a r.s.
∴ P (x, Y∊ C) > P (x, Y∊ S) 𝑿𝟏 , 𝑿𝟐 , … . , 𝑿𝒏 𝒇𝒓𝒐𝒎 𝒏(𝝁, 𝝈𝟐 ), show that,

2 𝒏
𝑟2 2 √𝒏−𝟏 (𝑿𝟏 −𝐱)
1 − 1 𝑎 − 𝑥 𝑑𝑥
⇒ 4 (1 − 𝑒 2 )> [ ∫0
𝑒 2 ] ∼ 𝒕𝒏−𝟐
2𝜋 𝒏
(𝒏−𝟏)𝑺𝟐 − (𝑿 −𝐱𝟐
√ 𝒏−𝟏 𝟏
𝒏−𝟐
1
x2 2 2a 2
< √1 − e− π
1 a
− dx
⇒, ∫0 e 2
√2π

189
Solving Mathematical Problems

𝑖 Hence 𝑦1 ∼ N (𝜇√𝑛 , 𝜎 2 )and 𝑦𝑖 ∼ N


Solution:- (a) Let 𝑌𝑖 = √𝑖−1 (x𝑖 − 𝑋𝑖 ) =
(0 , 𝜎 2 ) I = 1(1)n independently distributed.
𝑖 𝑋1 +𝑋2 ,….,𝑋𝑖
√ ( − 𝑋𝑖 ) 𝑖 (𝑋𝑖 − x𝑖 )2⁄
𝑖−1 𝑖 Now, ∑𝑛𝑖=2 𝑖−1 𝜎2
𝑖 𝑋1 + ….+ 𝑥𝑖−1 − (𝑖−1)𝑋𝑖 𝑦
=√𝑖−1 ( 𝑖
) =∑𝑛𝑖=2( 𝜎𝑖 )2, which is the sum of squares of
(n-1)
𝑋1 + ….+ 𝑥𝑖−1 − (𝑖−1)𝑋𝑖
= ∀ i=2(1)n.
√𝑖(𝑖−1)
i.i. d. N (0, 1) R.V.’s , follows 𝜆𝑛−1 2
1 1
Let 𝑌1 = 𝑋1 + ⋯ + 𝑋𝑛 . [𝑦𝑖 ∼ N (0 , 𝜎 2 ) , 𝑖 = 2(1)𝑛
√𝑛 √𝑛
𝑦
Hence the transformation reduces Y=AX, ⇒ 𝜎𝑖 ∼ N (0 ,1)]
where
𝑖
1 1 1 1
⇒ ∑𝑛𝑖=2 𝑖−1 (𝑋𝑖 − x𝑖 )2 ∼ 𝜎 2 𝜆𝑛−1 2
A= ………………..
√𝑛 √𝑛 √𝑛 √𝑛
Note :- 𝑦1 = √𝑛 x ∼ N (𝜇√𝑛 , 𝜎 2 )
1 1
0 ……………0
√1.2 √1.2
(c) Consider the transformation
is orthogonal
Y= A(X –𝜇1), where
⦙ ⦙ ⦙ ………… ⦙
1 1 1 1
1 1 1 (𝑛−1) A= ………………..
….. √𝑛 √𝑛 √𝑛 √𝑛
√𝑛(𝑛−1) √𝑛(𝑛−1) √𝑛(𝑛−1) √𝑛(𝑛−1)
−(𝑛−1) 1 1 1
[ Helmert’s transformation] … … … … ..
√𝑛(𝑛−1) √𝑛(𝑛−1) √𝑛(𝑛−1) √𝑛(𝑛−1)

The PDF of (𝑋1 , … . . , 𝑋𝑛 ) is


𝑎31 𝑎32 𝑎33 ……………………..
1 1
− ∑𝑛
(𝑋𝑖 −𝜇 )2
𝑎3𝑛
F(𝑥1 , … . . , 𝑥𝑛 ) = (𝜎√2𝜋)𝑛 𝑒 2 𝑖=1 𝜎2 , 𝑥𝑖 ∊
R 𝑎𝑛1 𝑎𝑛2 𝑎𝑛3 ……………………..
𝑎𝑛𝑛
Here 𝑦1 𝑦=𝑥1 𝑥 and𝑦1 =√𝑛x
Note that y’ y= (𝑋1- 𝜇)’ (X –𝜇)
∴ |J|=1 is the Jacobian of the transformation.
⇒ ∑𝑛𝑖=1 𝑦 2 𝑖 = ∑𝑛𝑖=1(𝑋𝑖 − 𝜇)2
The PDF of (𝑦1 , … . . , 𝑦) is
And |J=1|
g (𝑦1 , 𝑦2, … . . , 𝑦𝑛 )=
+ ∑𝑛 2 2 The PDF of X is
1 𝑖=1 𝑦𝑖 −2𝜇√𝑛 𝑦1 𝑛𝜇
𝑛 −
(𝜎√2𝜋) 𝑒 2𝜎2
1
1 − ∑𝑛
𝑖=1(𝑋𝑖 −𝜇)
2
𝑓𝑋 (𝑥) =(𝜎√2𝜋)𝑛 . 𝑒 2𝜎2 , 𝑥𝑖 ∊ R
(𝑦 −𝜇√𝑛)2 𝑦 2
1 − 1 2 1 − 𝑖2
= {𝜎√2𝜋 𝑒 2𝜎 }∏𝑛𝑖=2{𝜎√2𝜋 𝑒 2𝜎 }
The PDF of Y is

190
Solving Mathematical Problems
1
1 − ∑𝑛 2
𝑖=1 𝑦 𝑖 Q33. Suppose (X, Y)∼ BN(0, 0, 1, 1, P).
𝑓𝑌 (𝑦)= (𝜎√2𝜋)𝑛 . 𝑒 2𝜎2 , 𝑦𝑖 ∊ R
S.T.
⇒ 𝑦𝑖 ∼ N (0 , 𝜎 2 ), i= 1(1) n 𝑿𝟐 −𝟐𝑷𝑿𝒀+𝒀𝟐
i) ∼ 𝝀𝟐 𝟐
𝟏− 𝑷𝟐
1
Here, 𝑦1 = ∑𝑛𝑖=1(𝑋𝑖 − 𝜇)= √𝑛 (x̄- 𝜇)
√𝑛
ii) 𝑴𝒛 (𝒕)= [{𝟏 − (𝟏 + 𝐏)𝐭} {𝟏 + (𝟏 −
−(𝑛−1)(𝑋1 − μ)+ (𝑋2 − μ)+ ….+(𝑋𝑛 − μ) −𝟏⁄
and 𝑦2 = 𝐏)𝐭}] 𝟐
√𝑛(𝑛−1)
Solution :-i) (X, Y)∼ BN(0, 0, 1, 1, P)
𝑛 x−nX1 𝑛
= = -√𝑛−1 (X1 − x)
√𝑛(𝑛−1) ∴ the joint PDF of (X, Y) is given by,

𝑛 𝑓𝑋𝑌 (𝑥, 𝑦)=


Hence, √𝑛−1 (X1 − x)= -𝑦2 ∼ N (0 , 𝜎 2 ) 1
1 − (𝑋 2 −2𝑃𝑋𝑌+𝑌 2 )
2 (1−𝑃2 )
𝑒 ; (x,
𝜎𝑥 𝜎𝑦 √1−𝑃 2 .2𝜋
And ∑𝑛𝑖=3 𝑦 2 𝑖 = ∑𝑛𝑖=1(𝑋𝑖 2
− 𝜇) − 𝑦 2
1
−𝑦 2
2 2
y)∊ ℝ
𝑛
={ ∑𝑛𝑖=1(𝑋𝑖 − 𝜇)2 -n(x − μ)2} -𝑛−1 (X1 − Let, U= X+ Y
2
x)
V= X- Y
𝑛
= ∑𝑛𝑖=1(Xi 2
− x) -𝑛−1 (X1 − x) 2
1 𝑈+𝑉 𝑈−𝑉
∴ |J|= 2, X= ,Y=
2 2
𝑛 X1 −x 𝑌2
Therefore, √ ( )= - ∼ N(0, 1) Now, note that,
𝑛−1 𝜎 𝜎

∑𝑛 2 𝑛
(X −x)2 1 (𝑈 + 𝑉)2 (𝑈 − 𝑉)2
𝑖=1(Xi −x) − 𝑛−1 1 𝑌 { +
And, = ∑𝑛𝑖=3( 𝑖 )2, 1 − 𝑃2 4 4
𝜎2 𝜎
(𝑈 + 𝑉)(𝑈 − 𝑉)
The sum of squares of (n-2) iid N(0, 1) − 𝑃 }
2
R.V.’S, follows 𝜆𝑛−2 2 , independent 1 by
1 𝑈 2 +𝑉 2 (𝑈 2 −𝑉 2 )
defn of t- distn. =1−𝑃2 { − 𝑃 }
4 2

𝑛 X1 −x 1

𝑛−1
(
𝜎
) =2(1−𝑃2){𝑈 2 − 𝑃𝑈 2 + 𝑉 2 − 𝑃𝑉 2 }
∼ 𝑡𝑛−2
2 𝑛
∑𝑛 (X )2
𝑖=1(Xi −x) −𝑛−1 1 −x
√ { }⁄ 1
𝜎2
(𝑛−2) = {𝑈 2 (1 − 𝑃) + 𝑉 2 (1 + 𝑃)}
2(1+𝑃)(1−𝑃)

𝑛 X1 −x 𝑈2 𝑉2

𝑛−1
(
𝜎
) =2(1+𝑃) + 2(1−𝑃), ____________<i>
⇒ { (𝑛−1)𝑆2 −
𝑛
(X −x)2 }
∼ 𝑡𝑛−2
𝑛−1 1
(𝑛−2) ∴ Joint PDF of U and V is given by,

191
Solving Mathematical Problems

𝑓𝑈𝑉 (𝑢, 𝑣)= Q34. X ∼ R(0, 1) find


1 1
1 − 𝑢2 1 − 𝑣2
𝑒 4(1+𝑃) . 𝑒 4(1+𝑃)
i) the distn of 𝑿(𝒓) .
√2 √2𝜋 √1+𝑃 √2 √2𝜋 √1+𝑃
2
, (u,v)∊ℝ
ii) The m.g.f. , mean (E(𝑿(𝒓) )) and
∴ U and V are independent. variance (var(𝑿(𝒓) ))

U ∼ N(0, 2(1+P)) ANS:- i) If X ∼ R (0, 1), then the p.d.f of X


𝑈 𝑈2 is given by,
∴ ∼ 𝑁(0, 1) ⇒ 2(1+𝑃) ∼ 𝜆1 2
√2(1+𝑃)
F(x)=1, 0 < x < 1
𝑉2
Similarly, ∼ 𝜆1 2. The distn function of X is given by,
2(1−𝑃)

𝑥
𝑈2 𝑉2 F(x)= ∫0 𝑑𝑥=x
∴ 2(1+𝑃) + 2(1−𝑃) ∼ 𝜆1 2 [By the
reproductive property of 𝜆2-distribution] ∴ The PDF of 𝑟 𝑡ℎ order statistic is given by,
ii) MGF of X, Y is given By, 𝑛!
g(x) = (𝑟−1)!(𝑛−𝑟)! 𝑥 𝑟−1 (1 − 𝑥)𝑛−𝑟 , 0< x <
𝑀𝑋𝑌 (𝑡)=E(𝑒 𝑡𝑋𝑌 ) 1

= E[E(𝑒 𝑡𝑋𝑌 | 𝑋)] ∴ 𝑋(𝑟) ∼ B (r, n-r+1)

Y| X ∼ N(𝑃𝑋 , (1-𝑃2 )) ii) Let 𝑋(𝑟) be denoted as U, then


1 2 2
∴ E [𝑒 𝑡𝑋.𝑃𝑋 + 𝑡 𝑋 (1 − 𝑃2 )] U ∼ B(r-1, n-r)
2

2 1
= E [𝑒 𝑡𝑋.𝑃𝑋 + 2 𝑃2 (1 − 𝑃2 )𝑋 2 ] 𝑀𝑈 (𝑡)= E(𝑒 𝑈𝑡 )
𝑈2𝑡2
1
{𝑡𝑃+ 𝑡 2 (1−𝑃 2 )}𝑋 2 =E [1+Ut+ + ⋯]
= E [𝑒 2 ] 2!

𝑡𝑟
=
1 2
[∵𝑋 ∼𝜆1 ] 2 = ∑∞ 𝑟
𝑟=0 𝑟! 𝐸(𝑈 )
1 1
[1−2(𝑡𝑃+ 𝑡 2 (1−𝑃2 ))] ⁄2
2
𝑡𝑟
1 =∑∞ 1 1
𝑟=0 𝑟! .𝜇𝑟 [∵𝜇𝑟 = rth order raw
= 1
[1−2𝑡𝑃−𝑡 2 (1−𝑃 2 )] ⁄2 moment about zero]
1 1 1
= E(U)=𝐵(𝑟,𝑛−𝑟+1) ∫0 𝑢. 𝑢𝑟−1 (1 − 𝑢)𝑛−𝑟 𝑑𝑢 ,
√(1−𝑡𝑃)2 −𝑡 2

1 0<x <1
=
√{(1−𝑃)𝑡+1}{(1−𝑡(1+𝑃)}
𝐵(𝑟+1,𝑛−𝑟+1)
=
−1⁄ 𝐵(𝑟,𝑛−𝑟+1)
= [{1 − (1 + P)t} {1 + (1 − P)t}] 2
𝑟
=𝑛+1.

192
Solving Mathematical Problems
𝑟(𝑟+1) ∴ 𝑈1 , 𝑈2 are independently distributed,
E (𝑈 2 )=(𝑛+1)(𝑛+2)
∴ sample range(R)= 𝑋(2) − 𝑋(1)
𝑟(𝑟+1) 𝑟2 𝑟 𝑟+1 𝑟
V(U) =(𝑛+1)(𝑛+2) - (𝑛+1)2 = 𝑛+1 [𝑛+2 − 𝑛+1]
=𝑢2
𝑟(𝑛−𝑟+1)
=(𝑛+1)2 (𝑛+2) ∞
∴ E(R)=E(𝑢2 )= λ∫0 𝑢 𝑒 −𝜆𝑢2 𝑑𝑢
Q35. If 𝑿𝟏 , 𝑿𝟐 be a random sample of size 𝜆 1
= 𝜆2 = 𝜆
2 drawn from a population having p.d.f.
f(x)=𝛌𝐞 −𝝀𝒙 , x> 0, λ>0. Then find the distn ∴ PDF of sample range (R) is 𝑓𝑅 (R)=λ
of the sample range. Is the distn 𝑒 −𝜆𝑅 , R> 0
independent from the sampling
distribution of simple AM? 𝑋(1) + 𝑋(2) 2𝑢1 +𝑢2
Now, simple AM = 2
= 2
=
1
ANS:- 𝑋1 , 𝑋2 be a random sample drawn 𝑢1 +2 𝑢2 = Z, say,
from a population with pdf f(x)= λe −𝜆𝑥 ,
x>0, λ> 0. ∴ Joint PDF of (Z, 𝑢2 ) is given by,

Let us consider the following transformation 𝑓𝑍,𝑢2 (Z, 𝑢2 )= 2𝜆2 𝑒 −2𝜆𝑍 , Z>0

(𝑋1 , 𝑋2) → (𝑋(1) , 𝑋(2)) , where 𝑋(𝑖) =ith So, the distn of sample range & simple AM
order statistic. are different.

∴ Joint distn of 𝑋(1) , 𝑋(2) is given by, Q36. F(x, y) be a joint distribution
function of X and Y. G(𝜉, )be a function
𝑓𝑋(1) ,𝑋(2) (𝑥1 ,𝑥2) = 2𝜆2 𝑒 −𝜆(𝑋(1)+ 𝑋(2)) ∋𝜉= max(X, Y). Show that G(x,
y){𝑭(𝒙, 𝒙)𝒊𝒇 𝒙 < 𝑦
Let us define a variable,
F(x, y)+F(x,x)-F(y, y) if x≥ y
𝑈𝑖 = 𝑋𝑖 − 𝑋(𝑖−1) ∀ i=1, 2.
Solution:- G(x, y)= P [Max(X, Y)≤x,
𝑈1 =𝑋(1) [Assuming𝑋(0) = 0] Min(X, Y)≤ y] if x < y

𝑈2 = 𝑋(2) − 𝑋(1) = P[P [Max(X, Y)≤x, Min(X, Y)≤ x]

∴ 𝑋2=𝑢1 +𝑢2 =P [X≤ x , Y ≤ x]

𝑋(1) ,𝑋(2) 1 0 = F(x, x)


∴ |J|=|J( )|=| |=1
𝑢1 ,𝑢2 1 1
Now, G(x, y)= P [Max(X, Y)≤x, Min(X,
∴ Joint pdf of 𝑢1 , 𝑢2 is given by, Y)≤ y] if x ≤ y

𝑓𝑈1 ,𝑈2 (𝑢1 ,𝑢2) =2𝜆2 𝑒 −𝜆(2𝑢1 +𝑢2 ) = P[𝑋(2) ≤x, 𝑋(1) ≤ 𝑦 ]; A= 𝑋(2) ≤x,
B=𝑋(1) ≤ 𝑦
=2 λ .𝑒 −2𝜆𝑢1 , λ .𝑒 −2𝜆𝑢2 , (𝑢1 ,𝑢2 )>0

193
Solving Mathematical Problems

=P (A) –P(A ∩BC) Q38. 𝑿𝟏 , ………, 𝑿𝒏 , …… be i.i.d r. v. ‘s


𝟏
satisfying P [𝑿𝟏 =𝟐𝒋 ]= 𝟐𝒋 ,
= P[𝑋(2) ≤x] –[P[𝑋(2) ≤x, 𝑋(1) ≥ 𝑦 ]

= P [X≤ x, Y≤ x]- P [y ≤ 𝑋(1) ≤ 𝑋(2) ≤ j = 1, 2, 3, ….. show that WLLN dose not
hold for {𝑿𝒏 }.
𝑥]
𝐿𝑡
Solution:- WLLN holds iff 𝑛⟶∞ [|𝑥| > 𝑛] =
= F(x,x)-(F(y,y)-F(x, y))
0
= F(x,x) +(F(x ,y)-F(y , y)).
Let, 2𝑘 ≤ 𝑛 ≤ 2𝑘+1
𝟏
Q37. f(x,y)= {𝝅 𝒊𝒇 𝒙𝟐 + 𝒚𝟐 ≤ 𝟏 (𝒂)Are
i.e. n=2𝑘 +r
X and Y uncorrelated?
P[X > n]=P [𝑋 ≥ 2𝑘+1]= P [𝑋 = 2𝑘+1]+
𝟐 𝟐
=0 if 𝒙 + 𝒚 > 1 (b)Are X 𝑋 ≥ 2𝑘+2+ ……
and y independent?
1 1
1 =2𝑘+1 +2𝑘+2 + …….
Solution:- f(x,y)= {𝜋 𝐼 𝑥 2 + 𝑦 2 ≤ 1
1 1 1 1
= 2𝑘+1 .(1+ 2 + 4+ …..)= 2𝑘
1 √1−𝑥 2 2
𝑓𝑋 (𝑥) = 𝜋 ∫−√1−𝑥 2 𝑑𝑋 = 𝜋 √1 − 𝑥 2 ; -1 < x
2𝑘+𝑟
<1 ∴ nP [X > n]= 2𝑘
2
𝑓𝑌 (𝑦)=𝜋 √1 − 𝑦 2 ; -1 < y < 1 = 1+
𝑟
2𝑘
2 1 𝑟
E(X)= 𝜋 ∫−1 𝑥 √1 − 𝑥 2 dx = 0 [∵ the 𝐿𝑡
∴ 𝑛⟶∞ 𝐿𝑡
𝑛𝑃[𝑥 > 𝑛]= 𝑘⟶∞ (1 + 2𝑘)=1.
function is odd]
∴ WLLN does not hold
Similarly, E(Y)=0
Q39. There are 10 balls in an urn
1 √1−𝑦2 1 numbered 1 through 10.You randomly
E(XY)=∫−1 ∫−√1−𝑦 2 𝑥𝑦 𝑑𝑥𝑑𝑦 = 0 [∵
𝜋
select 3 of those balls. Let the random
√1−𝑦2 𝑥
∫−√1−𝑦 2 𝜋 𝑑𝑥 = 0] variable Y denotes the maximum of the
three numbers on the extracted balls.
∴ X and Y are uncorrelated. Find the probability mass function of y.
4
You should simplify your answer to a
Note that, f(x,y)=𝜋 √(1 − 𝑥 2 )(1 − 𝑦 2 ) ≠ fraction that does not involve binomial
f(x). f(y). coefficients. Then calculate:

∴ X and Y are not independent. P [Y ≥7].

Solution:- The random variable Y can take


the values in the set {3, 4, …. , 10}. For any
I, the triplet resulting in Y attaining the

194
Solving Mathematical Problems

value i must consist of the ball numbered i larger and the shorter subintervals
and a pair of balls with lower numbers. So , respectively. Show that Y/Z does not have
(𝑖−1)(𝑖−2)
finite expectations.
(𝑖−1
2 ) (𝑖−1)(𝑖−2)
2
Pi =P[Y =i]= (10
= 10.9.8 =
3) 3.2.1
240 Solution:- Let X ∼ U(0, 1) and U be the
length of the intervals (0, X) and (X, 1); i.e.
Since the balls are numbered 1 through 10, Z = min(X, 1-X)
we have
And let, Y = 1- U.
P [Y ≥7] =P [Y= 7] + P [Y= 8] + P[ Y=9] +
P [Y= 10] We can write Y and Z as Y = Y(X), Z =
Z(X); as functions of X ∼ U (0, 1).
6.5 7.6 8.7 9.8
So, P [Y ≥7]= 240 + 240 + 240 + 240
To show the expectation is infinite; we need
5 to show that
=6.
0 𝑌(𝑥)
∫1 𝑑𝑥 =∞
Q40. The number of misprints per page of 𝑍(𝑥)
text is commonly modeled by a poisson
Now, Z = min (X, 1-X) , Y = max(X, 1-X)
distribution . It is given that the
parameter of this distribution is λ = 0.6 1
𝑌(𝑥)
1
max(X,1−X)
for a particular book. Find the But, ∫12 𝑍(𝑥) 𝑑𝑥 = ∫12 min (X,1−X) 𝑑𝑥 =
1
probability that there are exactly two 1−𝑥
∫12 𝑑𝑥 = ∞
misprints on a given page of the book. 𝑥

How about the prob. that there are two or 1


One get the same thing on the interval [2, 1]
more misprints?
as well after a substitution, so the integral on
Solution:- Let X denote the random variable [0, 1] is ∞.
which stands for the number of misprints on
max(X,1−X)
a given page. Then Aliter:- let T (x)= min (X,1−X)

0.62
P [X =2]= 𝑒 −0.6≈ 0.0988 To calculate P [T (X)≤], for some t, If t < 1,
2!
then trivially we get 0.
P [X≥ 2] = 1- P [X < 2]
1−𝑋
Otherwise, P [T (X)≤]= P [ ≤ 𝑡, X∊ (0,
𝑋
= 1- P [X=1] – P[X=0] 1 𝑋 1
) ] + P [1−𝑋 ≤ 𝑡, X∊ ( 2 , 1) ]
2
−0.6 −0.6
=1-𝑒 - 0.6𝑒
1 1
= 2P [X≥ 1+𝑡, X∊ (0, 2)]
≈ 0.122.
1⁄ 2
Q41. The unit interval (0,1)is divided into =2 ∫1⁄ 2 𝑑𝑥 = 1- 𝑡+1.
1+𝑡
two subintervals picking a point at
random from inside the interval. 2
Differentiating, we get f(t) = (𝑡+1)2 for t≥ 1.
Denoting by Y and Z, the lengths of the
195
Solving Mathematical Problems
∞ 2 =
Now, ∫1 𝑑𝑡 = ∞.
(𝑡+1)2 1 2+ 𝑢 2 1 2+ 𝑢 2
𝑒 −2(𝑢1 2 )
. (−𝑢1 ) 𝑒 −2(𝑢1 2 )
. (−𝑢2 )
Q42. Let 𝑿𝟏 , 𝑿𝟐 ∼ R(0, 1). Show | 1 𝑢2 1 1 |
𝑢 . (− 𝑢 ) 𝑢 .𝑢
that____________ 2𝜋{1+( 2 )2 } 1 2𝜋{1+( 2 )2 } 1
𝑢1 𝑢1

1
− (𝑢 2 + 𝑢2 2 ) −𝑢1 −𝑢2
𝑼𝟏 = √−𝟐𝟏𝒏𝑿𝟏 𝐜𝐨𝐬(𝟐𝝅𝑿𝟐 ) 𝑒 2 1
= 𝑢 |− 𝑢2 1 |
2𝜋{1+( 2 )2 }
𝑢1 𝑢1 2 𝑢1
𝑼𝟐 = √−𝟐𝟏𝒏𝑿𝟏 𝐬𝐢𝐧(𝟐𝝅𝑿𝟐 )
1 2+ 𝑢 2
1
= - 2𝜋 . 𝑒 −2(𝑢1 2 )
Are standard normal variables.

Solution:- The PDF of (𝑋1 , 𝑋2) is The PDF of (𝑢1 , 𝑢2 ) is


𝑓𝑋1 ,𝑋2 (𝑥1 , 𝑥2 )= 𝑓(𝑥) = 1 1 2+ 𝑢 2

1 , 0 < 𝑥1 , 𝑥2 < 1 𝑓𝑈1 ,𝑈2 (𝑢1 , 𝑢2 )= 1 . | − . 𝑒 −2(𝑢1 2 )


|.
2𝜋
{
0 , 𝑜𝑤 (𝑢1 , 𝑢2 )∊𝑅 2
1 2 1 2
Here, 𝑢1 = √−21𝑛𝑥1 cos(2𝜋𝑥2 ) 1 1
= . 𝑒 −2 𝑢1 . . 𝑒 −2 𝑢2 , (𝑢1 , 𝑢2 )∊ℝ2
√2𝜋 √2𝜋

𝑢2 = √−21𝑛𝑥1 sin(2𝜋𝑥2 ) = 𝑓𝑢1 (𝑢1 ). 𝑓𝑢2 (𝑢2 ) , 𝑢1 , 𝑢2 ∊ℝ


∴ 𝑢1 2 + 𝑢2 2 = -21n𝑥1 Hence, 𝑈1 , 𝑈2 ∼ N(0, 1).
1 2+ 𝑢 2
⇒𝑥1 = 𝑒 −2(𝑢1 2 )
Q43. Let X, Y ∼ N(0, 1). Show that U = 𝒀
𝑿

𝑢 has a standard Cauchy distribution.


And tan(2𝜋𝑥2 )= 𝑢2
1 𝑿
What would be the distn of |𝒀| ?
1 𝑢
⇒ 𝑥2 = 2𝜋 tan−1 𝑢2
1 (𝑥2 +𝑦2 )
1
Soln:- Here, 𝑓𝑋,𝑌 (𝑥, 𝑦)= 2𝜋 𝑒 − 2 , (x, y)
Note that, 0 < 𝑥1 < 1
∊ℝ2
⇒ -21n𝑥1 > 0 , 0<2𝜋𝑥2 <2𝜋 𝑋
Let, U=𝑌 and V = y.
⇒ √−21𝑛𝑥1 > 0 , -1 ≤ 𝑥
cos(2𝜋𝑥2 ) , sin(2𝜋𝑥2 ) ≤ 1 ∴ u = 𝑦 , v= y [-∞ < 𝑢 < ∞ , −∞ < 𝑣 <
∞]
⇒ 𝑢1 , 𝑢2 ∊ ℝ
⇒ x= uv , y=v
∂𝑥1 ∂𝑥1
∂𝑢 ∂𝑢2
The Jacobian is J = | ∂𝑥1 ∂𝑥2
| 𝜕𝑥 𝜕𝑥
2 𝑣 𝑢
∂𝑢1 ∂𝑢1 ∴J=| |= v = |𝜕𝑢 𝜕𝑣
|
0 1 𝜕𝑦 𝜕𝑦
𝜕𝑢 𝜕𝑣

Clearly, (u,v) ∊ℝ2

196
Solving Mathematical Problems

The PDF (U, V) is ___ ⇒ u and –U have identical distribution]


2
1 2 )𝑣 ∴ 𝑓𝑊 (𝑤)= 𝑓𝑈 (𝑤) ∀ w
𝑓𝑈,𝑉 (𝑢, 𝑣𝑦)= 2𝜋 𝑒 −(1+𝑢 2 |𝑣| , (u,v) ∊ℝ2
𝑋
⇒ W = |𝑌|∼ C (0, 1).
The PDF of U is
∞ Q44. If X, Y ∼ N(0, 1). Find the distns of
𝑓𝑈 (𝑢) =∫−∞ 𝑓𝑈𝑉 (𝑢, 𝑣) 𝑑𝑣 =
𝑿
∞ 1 −(1+𝑢2 )
𝑣2 U = √𝑿𝟐 + 𝒀𝟐 and V = 𝒀
∫−∞ 2𝜋 𝑒 2 |𝑣|𝑑𝑣
1 2 +𝑦 2 )
1
∞ 𝑣 1
− 𝑣 2 (1+𝑢2 )
Solution:- 𝑓𝑥,𝑌 (𝑥, 𝑦) = 2𝜋 .𝑒 −2(𝑥 , (x, y)∊
Or, =[2 ∫0 2𝜋 𝑒 2 𝑑𝑣] =
ℝ2
𝑣2
2 ∞ −(1+𝑢2 )
𝜋
∫0 𝑒 2 𝑣𝑑𝑣 𝑥
Note that, u= √𝑥 2 + 𝑦 2 , v = 𝑦
∞1 1 1 2 )=𝑍]
[ ∫0 . 𝑒 −𝑍 𝑑𝑍 2 𝑣 2(1+𝑢
𝜋 (1+𝑢2 ) ⇒ u = |y|. √1 + 𝑣 2 , x= vy
1 ∞ −(1+𝑢2 )𝑍
= ∫ 𝑒
𝜋 0
𝑑𝑍
𝑢𝑣 𝑢
⇒ x = ± √1+𝑣2 , y = ± √1+𝑣 2
𝑣2
[where, Z= 2 , ⇒dZ = vdv] 𝑢𝑣 𝑢
Let, 𝑥1 = √1+𝑣2 , 𝑦1 = √1+𝑣2
1
[𝜋(1+𝑢2 ) [−𝑒 −𝑍 ]]
Then for a pair (U, V), there are two points
1 ┌(1)
= 𝜋 .(1+𝑢2 ), u ∊ ℝ of (x, y):

1 (𝑥1 , 𝑦1), (−𝑥1 , −𝑦1 )


[𝜋(1+𝑢2 )]
1 The transformation is not one – to one,
= 𝜋(1+𝑢2 ), u ∊ ℝ
Clearly, 0 < u < ∞ , v ∊ ℝ
𝑋
Hence, U=𝑌 ∼ C(0, 1) distn
∂𝑥1 ∂𝑥1 𝑣 𝑢
3
∂u ∂v √𝑣 2 +1 (√𝑣 2 +1) ⁄2
𝑥
Let, w = |𝑌|, The PDF of w is 𝑓𝑊 (𝑤)= P Now, 𝐽1 = |∂𝑦 ∂𝑦1
| =| 1 𝑢𝑣 |
1
3
∂u ∂v √𝑣 2 +1 (√𝑣 2 +1) ⁄2
[W≤ w/Y < 0]P [Y < 0]+ P [W≤ w/Y > 0]P
[Y > 0] 𝑢
= -1+ 𝑣2 = 𝐽2
1 𝑋 𝑋
= 2 {P [−𝑌 ≤ 𝑤] + P [𝑌 ≤ 𝑤]} Hence, The PDF of (U, V)is
1
= 2 {P[-U≤ 𝑤] + P [U ≤ 𝑤]} 𝑓𝑈,𝑉 (𝑢, 𝑣)=
𝑓 (𝑥 , 𝑦 )|𝐽 | + 𝑓𝑥,𝑌 (−𝑥1 , −𝑦1 )|𝐽2 |, 𝑖𝑓 0 < 𝑢 < ∞, −∞ <
1 { 𝑥,𝑌 1 1 1
= .2 .P [U≤ 𝑤 ] [∵ U∼ C (0, 1)is 0 , 𝑜𝑤
2
symmetrical about ‘0’ ]

[⇒𝑓𝑈 (−𝑢) =𝑓𝑈 (𝑢)

197
Solving Mathematical Problems
𝑢2
2 − .|−
𝑢
|,𝑖𝑓0<𝑢< ∞,𝑎𝑛𝑑−∞<𝑣< ∞ ⇒ r = √4𝑢2 + 𝑣 2
= {2𝜋 𝑒 2 1+ 𝑣2

0 , 𝑜𝑤 The PDF of (U, V)is


= −
4𝑢2 +𝑣2
2 .𝑒 2
𝑢2 𝑓𝑈𝑉 (𝑢, 𝑣)=
− 1
{(𝑢𝑒 ) . 𝜋(1+ 𝑣2 ) , 𝑖𝑓 0 < 𝑢 < ∞ 𝑎𝑛𝑑 𝑣 ∊ ℝ
2 2π
1
.( √4𝑢2 + 𝑣 2 )| |, (u, v) ∊ ℝ2
0 , 𝑜𝑤 √4𝑢2 +𝑣 2

𝑢2
Hence, U = √𝑋 2 + 𝑌 2 has the PDF 1
− 1
1 𝑣2
=1 .𝑒 2.
4 . . 𝑒 − 2 ; (u, v) ∊ ℝ2
√2π √2π
𝑢2 2

𝑓𝑈 (𝑢) = {𝑢 𝑒 ,0 < 𝑢 < ∞ 2

0 , 𝑜𝑤 = 𝑓𝑈 (𝑢). 𝑓𝑉 (𝑢) , u, v ∊ ℝ
1
and V ∼ Cauchy (0, 1), independently. Hence, U ∼ N (0, 4) and V ∼ N (0, 1),
independently.
Q45. If X, Y ∼ N(0, 1), Find the distn of
𝑿𝒀 𝑿𝟐 −𝒀𝟐
U= , and V = . Q46. Let 𝑿𝟏 , 𝑿𝟐 ∼ r (0, 1). Find out CDF
√𝑿𝟐 +𝒀 𝟐 √𝑿𝟐 +𝒀𝟐
and hence the PDF of 𝑿𝟏 + 𝑿𝟐 . How
𝑋2 +𝑌2 should the above result be modified in
1
Solution:- 𝑓𝑥,𝑌 (𝑥, 𝑦)= 2𝜋 𝑒 − 2 , (x, y)∊ ℝ2
case 𝑿𝟏 , 𝒂𝒏𝒅 𝑿𝟐 ∼ R (a, b)?
Let, x= rcos𝜃, y = rsin𝜃, Solution:- 𝑓𝑈 (𝑢) = P [U≤ u]
Here, 0< r<∞, 0< 𝜃< 2𝜋, = P [𝑋1 + 𝑋2 ≤ u]
∴ J = r,
=∬𝑋 (𝑥1 , 𝑥2 )𝑑𝑥1 , 𝑑𝑥2
1 + 𝑋2
The PDF of (r, 𝜃)is _____
Here, U = 𝑋1 + 𝑋2 takes values between 0
𝑟2 1
− . ,0<𝑟< ∞ 𝑎𝑛𝑑 0< 𝜃<2𝜋 and 2.
g (r, 𝜃)= {𝑟𝑒 2 2𝜋

0 , 𝑜𝑤 Note that for 0< u <1,


𝑟
Here, u= rsin𝜃cos𝜃 = 2 𝑠𝑖𝑛2 𝜃 P [U≤ u]= P [𝑋1 + 𝑋2 ≤ 𝑢]

And v= rcos2𝜃 𝐴𝑟𝑒𝑎 𝑜𝑓 𝑡ℎ𝑒 𝑟𝑒𝑔𝑖𝑜𝑛 𝐴


=
𝐴𝑟𝑒𝑎 𝑜𝑓 𝑡ℎ𝑒 𝑠𝑎𝑚𝑝𝑙𝑒 𝑠𝑝𝑎𝑐𝑒 (𝛺)
2
Clearly, (U, V) ∊ ℝ
[Using the concept of geometric probability,
∂(r,θ) 1 1 1
𝐽1 = ∂(u,v) = ∂(u,v) = 1 = 𝑟 = 𝐽2 As (𝑋1 , 𝑋2) is uniformly distributed
𝑠𝑖𝑛2 𝜃 rcos2θ
∂(r,θ) |2 |
cos2θ −2𝑠𝑖𝑛𝜃
Over 𝛺, here,
Here, (2𝑢)2 + 𝑣 2 = 𝑟 2 [ a pair(u,v)is a
obtained, for two pairs: (r, 𝜃), (r, 𝜃+2𝜋). The 𝛺 = {(𝑋1 , 𝑋2 ) : 0< 𝑋1 , 𝑋2 < 1}
transformation is not one-to-one]

198
Solving Mathematical Problems

And A ={(𝑋1 , 𝑋2 ) : 𝑋1 + 𝑋2 ≤ 𝑢}≤ 𝛺] = P [-u≤ 𝑋1 − 𝑋2 ≤ u ]


1 2 𝐴𝑟𝑒𝑎 𝑜𝑓 𝑡ℎ𝑒 𝑠ℎ𝑎𝑑𝑒𝑑 𝑟𝑒𝑔𝑖𝑜𝑛
𝑢 1 =𝐴𝑟𝑒𝑎 𝑜𝑓 𝑡ℎ𝑒 𝑠𝑎𝑚𝑝𝑙𝑒 𝑠𝑝𝑎𝑐𝑒 (𝛺)
∴ P [U≤ u] = 2
= 2 𝑢2 , for 0< u< 1.
12

For, 1 ≤ u< 2 , ___ [ Using the concept of Geometric

P [U ≤ u]= P [𝑋1 + 𝑋2 ≤ 𝑢] Probability as (𝑋1 , 𝑋2) is

𝐴𝑟𝑒𝑎 𝑜𝑓 𝑡ℎ𝑒 𝑟𝑒𝑔𝑖𝑜𝑛 𝐴 Uniformly distributed over 𝛺]


= 𝐴𝑟𝑒𝑎 𝑜𝑓 𝑡ℎ𝑒 𝑠𝑎𝑚𝑝𝑙𝑒 𝑠𝑝𝑎𝑐𝑒 (𝛺)
1
12 − (1−𝑢)2
2
1
12 − (2−𝑢)2 P [U ≤ u]=
2 12
= 12
= 1- (1 − 𝑢)2
1
= 1 - (2 − 𝑢)2
2
Hence, the CDF of u is
Hence the CDF of U is___ 𝐹𝑈 (𝑢)=
0 ,𝑢 ≤ 0
0, 𝑢 ≤ 0
1 2 𝑓𝑈 (𝑢)={1 − (1 − 𝑢)2 , 0 < 𝑢 < 1
𝑢 ,0 < 𝑢 < 1 1 , 𝑢≥1
2
1
1 − 2 (2 − 𝑢)2 , 1 ≤ 𝑢 < 2
And the PDF of U is
{ 1, 𝑢 ≥ 2
2 (1 − 𝑢), 0 < 𝑢 < 1
And the PDF of U is _____ 𝑓𝑈 (𝑢)={
0, 𝑜𝑤
𝑢, 0 < 𝑢 < 1
2 − 𝑢, 1 ≤ 𝑢 < 2 ii> Let U= 𝑋1 𝑋2
𝐹𝑈 (𝑢)= {
0 , 𝑜𝑤
Then U takes value between 0 and 1.

Modification:- 𝑋𝑖 ∼ R (a, b) , i=1, 2 For, 0< u< 1:

⇒ 𝑈𝑖 =
𝑋𝑖 −𝑎
∼ R (0, 1), i=1, 2. P [U ≤ u]= P [𝑋1 𝑋2 ≤ u ]
𝑏−𝑎
𝐴𝑟𝑒𝑎 𝑜𝑓 𝑡ℎ𝑒 𝑠ℎ𝑎𝑑𝑒𝑑 𝑟𝑒𝑔𝑖𝑜𝑛
Q47. Let 𝑿𝟏 , 𝑿𝟐 ∼ R (0, 1). Find out CDF = 𝐴𝑟𝑒𝑎 𝑜𝑓 𝑡ℎ𝑒 𝑠𝑎𝑚𝑝𝑙𝑒 𝑠𝑝𝑎𝑐𝑒 (𝛺)
and PDF of 1
𝑢𝑋1 +∫𝑢 𝑥2 𝑑𝑥1
=
i> |𝑿𝟏 − 𝑿𝟐 |, ii>𝑿𝟏 𝑿𝟐 12

1 𝑢
Solution:- =u +∫𝑢 𝑑𝑥1
𝑥1

i> Let U=|𝑋1 − 𝑋2| = u+ u [1n𝑥1 ]1,u =u(1-|nu)


Note that U takes values between 0 and 1. Q48. X and Y ∼ R (0, 1) X & Y are
For, 0< u< 1, independent,

P [U ≤ u]= P [|𝑋1 − 𝑋2 | ≤ u] i> X+Y ∼ ?

199
Solving Mathematical Problems

ii> X- Y ∼? 𝑧 + 1 , 𝑖𝑓 − 1 < 𝑧 < 0


iii> XY ∼? 𝐹𝑍 (𝑧)= { 1 − 𝑧 , 𝑖𝑓 0 < 𝑧 < 1
𝑿 0 , 𝑜𝑤
iv> ∼?
𝒀
v> |𝑿 − 𝒀|∼? iii) Z=XY

ANs:-i) Z= X+Y 0< X, Y< 1 ⇒ 0< XY <1.

0< X, Y<1 ⇒ 0<x < 1

⇒ 0< Z < 2. 𝐹𝑍 (𝑧)= P [XY ≤ 𝑧]

Distribution function of Z is =
0 , 𝑖𝑓 𝑧 ≤ 0
𝐹𝑍 (𝑧)= P [z≤ z] 1 𝑧 1 𝑧/𝑥
{∫0 ∫0 𝑑𝑥𝑑𝑦 + ∫𝑧 ∫0 𝑑𝑥𝑑𝑦 , 𝑖𝑓 0 < 𝑧 < 1
= P[Y≤z-X] 1 , 𝑖𝑓 𝑧 ≥ 1
1𝑧
0 ,𝑧 ≤ 0 = z+∫𝑧 𝑑𝑥 , if 0< z< 1
1 2 𝑥
𝑧 , 𝑖𝑓 0 < 𝑧 < 1
2
= 1 = z + z [|n|-|nz]
1 − 2 (2 − 𝑧)2 , 𝑖𝑓 1 < 𝑧 < 2
{ 1 , 𝑖𝑓 𝑧 ≥ 2 = z- z|nz = z (1-|nz). , if 0< z<1

PDF of Z is, _____ ∴ CDF of z is, _____

𝑧 , 𝑖𝑓 0 < 𝑧 < 1 0 , 𝑖𝑓 𝑧 ≤ 0
𝐹𝑍 (𝑧)= {2 − 𝑧 , 𝑖𝑓 1 < 𝑧 < 2 𝐹𝑍 (𝑧)= {𝑧 (1 − |𝑛𝑧), 𝑖𝑓 0 < 𝑧 < 1
0 , 𝑜𝑤 1 , 𝑖𝑓 𝑧 ≥ 1
ii) Z = X- Y PDF of z is, ___
0< X, Y< 1 −1𝑛𝑧 , 𝑖𝑓 0 < 𝑧 < 1
𝐹𝑍 (𝑧)= {
0 , 𝑜𝑤
⇒ -1< z< 1
𝑋
iv) Z= 𝑌
𝐹𝑍 (𝑧) = P [X-Y ≤z]

=P [Y ≥ 𝑋 − 𝑧 ] 0< X, Y< 1
𝑋
0 , 𝑖𝑓 𝑧 ≤ −1 ⇒ 0<𝑌 < ∞
1
(𝑧 + 1)2 , 𝑖𝑓 − 1 < 𝑧 < 0
2
= 1 ∴ 0< Z <∞
1 − 2 (1 − 𝑧)2 , 𝑖𝑓 0 < 𝑧 < 1
{ 1 , 𝑧≥1 P [Z ≤ z]
𝑋
PDF of Z is, ________ =P [𝑌 ≤ z]

200
Solving Mathematical Problems

= P [Y ≥ ½ X] 0< x < a, 0<y < b

0 , 𝑖𝑓 𝑧 ≤ 0 ⇒ 0< x+y< a+b


1
𝑧 , 𝑖𝑓 0 < 𝑧 < 1
= 2 ∴ P [Z ≤z]= P [Y≤z-X]
1
1 − 2𝑧 , 𝑖𝑓 𝑧 ≥ 1
0 ,𝑧 ≤ 𝑜
{ 1 1 2
× 𝑧 , 𝑖𝑓 0 < 𝑧 ≤ 𝑏
𝑎𝑏 2
1 𝑏
= × 2 × (2𝑧 − 𝑏), 𝑏 < 𝑧 ≤ 0
𝑎𝑏
1 𝑎+𝑏−𝑧)2
, 𝑖𝑓 𝑧 ≤ 0 𝑎𝑏−(
2 2
1
𝑎 <𝑧 <𝑎+𝑏
𝑎𝑏
∴ 𝐹𝑍 (𝑧)= , 𝑖𝑓 𝑧 ≥ 1
2𝑧 2 { 1, 𝑖𝑓 𝑧 ≥ 𝑎 + 𝑏
0 , 𝑜𝑤
{ 𝑧
,𝑎 < 𝑧 ≤ 𝑏
𝑎𝑏
1
v) Z = |𝑋 − 𝑌| ,𝑏 < 𝑧 ≤ 𝑎
= 𝑎
𝑎+𝑏−𝑧
0< X, Y< 1 ,𝑎 < 𝑧 < 𝑎 + 𝑏
𝑎𝑏
{ 𝑜 , 𝑜𝑤
⇒ 0<|𝑋 − 𝑌| < 1

P [Z ≤z]
ii) X- Y = z
= P [X –Z ≤Y ≤ X+Z]
P [Z ≤ z]
0 , 𝑖𝑓 𝑧 ≤ 0
={1 − (1 − 𝑧)2 ,𝑖𝑓 0<𝑧<1 = P [X-Y ≤ z]
1, 𝑖𝑓 𝑧 ≥ 1
= P [Y ≥ X- z]
2(1 − 𝑧) , 𝑖𝑓 0 < 𝑧 < 1 0 , 𝑧 ≤ −𝑏
∴ 𝑓𝑍 (𝑧) = {
0 , 𝑜𝑤 1
(𝑏 + 𝑧)2 , −𝑏 < 𝑧 ≤ 0
2𝑎𝑏
Q49. X ∼ R(0, a) =
1
(𝑏 + 2𝑧)𝑏, 0 < 𝑧 ≤ 𝑎 − 𝑏
2𝑎𝑏
1
Y ∼ R( 0, b) 1− (𝑎 − 𝑧)2 , 𝑎 − 𝑏 < 𝑧 < 𝑎
2𝑎𝑏
{ 1 , 𝑧 ≥𝑎
X and Y are independent & a> b
1
i> X+Y ∼ ? (𝑏 + 𝑧), −𝑏 < 𝑧 ≤ 0
𝑎𝑏
1
,0 < 𝑧 ≤ 𝑎 − 𝑏
ii> X- Y ∼? ∴ 𝑓𝑧 (𝑧)= 𝑎
𝑎−𝑧
iii> XY ∼? ,𝑎 − 𝑏 < 𝑧 < 𝑎
𝑎𝑏
iv>
𝑿
∼? { 0 , 𝑜𝑤
𝒀
v> |𝑿 − 𝒀|∼? iii) Z= XY , 0< z < ab

ANS :- i) X + Y= Z, P [Z ≤ z]

201
Solving Mathematical Problems
𝑧 𝑿𝟏 + 𝑿𝟐
= P [Y ≤ ] 𝒀𝟐 = 𝑿
𝑋 𝟏 + 𝑿𝟐 + 𝑿𝟑

0 ,𝑧 ≤ 0 𝑿𝟏
1 𝑎𝑍
𝒀𝟑 = , are independently
𝑿𝟏 + 𝑿𝟐
= {𝑎𝑏 [𝑧 + ∫𝑧 𝑋
𝑑𝑥 ] , 0 < 𝑧 < 𝑎𝑏
𝑏 distributed.
1 , 𝑧 ≥ 𝑎𝑏
Identify their distribution.

𝑓𝑧 (𝑧)= Solution :- 𝑓𝑋1 ,𝑋2 ,𝑋3 (𝑥1 , 𝑥2 , 𝑥3 )=
[ 1 − 𝑏 + 1𝑛𝑏𝑎 − 1𝑛𝑧]
1
, 0 < 𝑧 < 𝑎𝑏 𝑒 −(𝑥1 +𝑥2 + 𝑥3 ) , 𝑖𝑓 𝑥𝑖 > 0 ∀ 𝑖 = 1, 2, 3
{ 𝑎𝑏 {
0 , 𝑜𝑤
0 , 𝑜𝑤
𝑋 Here, 𝑥1 + 𝑥2 + 𝑥3 = 𝑦1
iv) Z = 𝑌 , 0< z
𝑥1 + 𝑥2 = 𝑦1 𝑦2
𝑋 𝑥1 +𝑥2
P [𝑌 ≤ 𝑧] [∵𝑥1 + 𝑥2 < 𝑥1 + 𝑥2 + 𝑥3 ⇒ <
𝑦1
𝑥1
𝑋 1, <1 ]
P[Y≥ ] 𝑥1 +𝑥2
𝑧

0 , 𝑧 ≤0 𝑥3 = 𝑦1 (1 − 𝑦2 )
1
= {2𝑎𝑏 × 𝑏𝑧 × 𝑏 , 0 < 𝑧 < 1 Clearly, 0< 𝑦1 < ∞ and 0< 𝑦2 , 𝑦3 < 1
𝑎 1
1 − 𝑎. 𝑧 . 2𝑎𝑏 , 1 ≤ 𝑧 𝜕𝑥1 𝜕𝑥1 𝜕𝑥1
𝜕𝑦1 𝜕𝑦2 𝜕𝑦3
𝑏
,0 < 𝑧 < 1 The Jacobian is J= 𝜕𝑦
𝜕𝑥2 𝜕𝑥2 𝜕𝑥2
2𝑎
1 𝜕𝑦2 𝜕𝑦3
∴ 𝑓𝑧 (𝑧)= {𝑎 1
( ),𝑧 ≥ 1 𝜕𝑥3 𝜕𝑥3 𝜕𝑥3
2𝑏 𝑧 2
𝜕𝑦1 𝜕𝑦2 𝜕𝑦3
0 , 𝑜𝑤
𝑦2 𝑦3 𝑦1 𝑦3 𝑦1 𝑦2
v) Z = |X- Y| , 0< z < a
= 𝑦2 (1 − 𝑦3 ) 𝑦1(1−𝑦3 ) − 𝑦1 𝑦2 =
P [X-z ≤Y ≤ X + z] 1 − 𝑦2 −𝑦1 0
1 0 0
0 ,𝑧 ≤ 0 𝑦2 (1 − 𝑦3 ) 𝑦1 (1 − 𝑦3 ) −𝑦1 𝑦2
1 1 1 − 𝑦2 −𝑦1 0
= {1 − 2 (𝑏 − 𝑧) − 2 (𝑎 − 𝑧)2 , 0 < 𝑧 < 𝑎
2

1 , 𝑧 ≥𝑎 = - 𝑦1 2 𝑦2
𝑎 + 𝑏 − 2𝑧 , 0 < 𝑧 < 𝑎
∴ 𝑓𝑧 (𝑧)= { The PDF of (𝑌1, 𝑌2 , 𝑌3 )is
0 , 𝑜𝑤

Q50. let 𝑿𝟏 , 𝑿𝟐 , 𝑿𝟑 be iid RV’s with PDF 𝑓𝑌1 ,𝑌2 ,𝑌3 (𝑦1 , 𝑦2 , 𝑦3 )=
−𝑦1 ⎸𝑦1 2 𝑦2 ⎸,𝑖𝑓
𝒆 −𝒙
,𝒙 > 0 {𝑒 0 < 𝑦1 < ∞ 𝑎𝑛𝑑 0 < 𝑦2 , 𝑦3 < 1
F(x) = { 0 , 𝑜𝑤
𝟎 , 𝒐𝒘

Show that ⟶ 𝒀𝟏 = 𝑿𝟏 + 𝑿𝟐 + 𝑿𝟑

202
Solving Mathematical Problems

= The PDF of U is , ___________


𝑒 −𝑦1 .𝑦1 3−1
. 2𝑦2 .1 , 𝑖𝑓 0 < 𝑦1 < ∞ , 0 < 𝑦2,
{ ┌(3) 𝑦3 < 𝑓𝑈 (𝑢)=
0 , 𝑜𝑤 1 1 6𝑢2 𝑣
∫0 (∫0 𝑑𝑣) 𝑑𝑤 , 𝑖𝑓 0 < 𝑢 < ∞
1 { (1+𝑢)4
0 , 𝑜𝑤
Where, 𝑓𝑌1 (𝑦1 ) =
1 𝑢3−1
𝑒 −𝑦1 .𝑦1 3−1 . (1+𝑢)3+1 , 0 < 𝑢 < ∞
, 0 < 𝑦1 < ∞ ∴ 𝑌1 ∼ 𝐺𝑎𝑚𝑚𝑎(3) = {𝛽(3,1)
{ ┌(3)
0 , 𝑜𝑤
0 , 𝑜𝑤
Hence U ∼ second kind Beta (3, 1)

Q. If 𝑋1 , 𝑋2 , 𝑋3 ∼ N(0, 1). Find the distn s of


𝑓𝑌2 (𝑦2 )
𝑦2, 2−1 (1 − 𝑦2, )1−1 𝑋 + 𝑋 +
𝑌1 = 1 2
𝑋3
, 0 < 𝑦2<1 ∴ 𝑌2 ∼ 𝛽(2, 1) √3
= { 𝛽(2, 1)
0 , 𝑜𝑤 𝑋 − 𝑋
𝑌2 = 1 2
√2
And 𝑓𝑌3 (𝑦3 ) = 𝑋1 + 𝑋2 − 2𝑋3
1 , 0 < 𝑦3 < 1 , ∴ 𝑌3 ∼ 𝑈(0, 1) 𝑌3 =
√6
{
0 , 𝑜𝑤
Solution:-𝑓𝑋1 ,𝑋2 ,𝑋3 (𝑥1 , 𝑥2 , 𝑥3 )=(
[ Due to independence] 1 3⁄ 1 2+ 𝑥 2 + 𝑥 2 )
) 2 . 𝑒 −2(𝑥1 2 3
; xi ∊ ℝ
2𝜋
Q51. let 𝒇𝑿,𝒀,𝒁 (𝒙, 𝒚, 𝒛)=
𝟔 𝑦
, 𝒊𝒇 𝒙, 𝒚, 𝒛 > 0 Note that, y = (𝑦12 ) =
{(𝟏+𝒙+𝒚+𝒙)𝟒 𝑦3
𝟎 , 𝒐𝒘 1 1 1
√3 √3 √3
Be the PDF of (X, Y, Z). find the distn of 1 1 𝑥
− 0 (𝑥12 )
U = X+ Y+ Z. √2 √2 𝑥3
1 1 2

Solution :- U = X+ Y+ Z (√6 √6 √6)

𝑋+𝑌 = a x , wher A is orthogonal, i. e. 𝐴𝐴𝑇 = 𝐼3 .


V= 𝑋+𝑌+𝑍 ℎ𝑒𝑟𝑒, 0 < 𝑈 < ∞ 𝑎𝑛𝑑 0 <
𝑉 ,𝑤 < 1 ∴ x= 𝐴−1 𝑦 = 𝐴𝑇 𝑦 and
𝑋 1 1 1
W= 𝑋+𝑌 √3 √2 √6
𝑥 1 1 1 𝑦
2 (𝑥12 ) = − (𝑦12 )
J= - 𝑢 𝑣 𝑥3 √3 √2 √6 𝑦3
1 2
0 −
𝑓𝑈,𝑉,𝑊 (𝑢, 𝑣, 𝑤)= (√3 √6)
6
. |− 𝑢2 𝑣|, 0 < 𝑢 < ∞ 𝑎𝑛𝑑 0 < 𝑣, 𝑤 < 1 ⇒ 𝑥1 =
𝑦1
+
𝑦2
+
𝑦3
{(1+𝑢)4 √3 √2 √6
0 , 𝑜𝑤

203
Solving Mathematical Problems
𝑦1 𝑦2 𝑦3
𝑥2 = - + By invariance property, x2 is consistent for
√3 √2 √6
𝜇2.
𝑦1 2𝑦3
𝑥3 = +0.y -
√3 √6 But, E (x2 )= v(x̄)+ 𝐸 2 (x)
𝜕(𝑜𝑙𝑑 𝑣𝑎𝑟𝑖𝑎𝑏𝑙𝑒) 𝜕𝑥 𝜕(𝑥 ,𝑥 ,𝑥 )
Jacobian = |𝜕(𝑜𝑙𝑑 𝑣𝑎𝑟𝑖𝑎𝑏𝑙𝑒)|= |𝜕𝑦|or |𝜕(𝑦1 ,𝑦2 ,𝑦3)| 𝜇
=𝑛 + 𝜇 2 ≠ 𝜇 2 [∵ 𝑋𝑖 ∼ N (μ, μ) ]
1 2 3

𝜕𝑥1 𝜕𝑥1 𝜕𝑥1 1 1 1 i.e. x2 is biased for 𝜇 2 .


𝜕𝑦1 𝜕𝑦2 𝜕𝑦3 √3 √2 √6
𝜕𝑥2 𝜕𝑥2 𝜕𝑥2 1 1 1
= 𝜕𝑦 = − = (b) In a normal sample, x̄ and 𝑆 2 are
1 𝜕𝑦2 𝜕𝑦3 √3 √2 √6
𝜕𝑥3 𝜕𝑥3 𝜕𝑥3 1 2 independently distributed.
0 −
𝜕𝑦1 𝜕𝑦2 𝜕𝑦3 (√3 √6)
𝑇
|𝐴 |=± 1. Also, E (x̄)= 𝜇 and E (𝑆 2 )= 𝜇

Note that, yˊy = xˊ AˊAx = xˊx⇒ ∑3𝑖=1 𝑦𝑖 2 = Hence,E(x̄. 𝑆 2 ) = E(x̄) .E (𝑆 2 ), due to


∑3𝑖=1 𝑥𝑖 2 independence.

Clearly, 𝑦𝑖 ∊ ℝ , i=1, 2, 3 = 𝜇2

The PDF of 𝑌1 , 𝑌2 , 𝑌3 is And var(x̄.𝑆 2 )= E (𝐸 2 )2-𝐸 2 (x. 𝑆 2 )

1 3 1
− ∑3𝑖=1 𝑦𝑖 2
= E (x2 . 𝑆 4 ) - 𝜇 4
𝑓𝑌1 ,𝑌2 ,𝑌3 (𝑦1 , 𝑦2 , 𝑦3 )= (2𝜋) ⁄2 . 𝑒 2 .⎸ ±
1⎸, 𝑦𝑖 ∊ ℝ = E (x2 ). E(. 𝑆 4 ) - 𝜇 4

1 1 2 = {v(x̄) +𝐸 2 (x) } {v(𝑆 2 )+ 𝐸 2 (𝑆 2 ) }-𝜇 4


=∏3𝑖=1{ . 𝑒 −2𝑦𝑖 }= ∏3𝑖=1 𝑓𝑌𝑖 (𝑦𝑖 )
√2𝜋
𝜇 2𝜇 2
= {𝑛 + 𝜇 2 } {𝑛−1 + 𝜇 2 }- 𝜇 4
Hence, 𝑌𝑖 ∼ N (0, 1) , i= 1, 2, 3.

Q52. Let 𝑿𝟏 , 𝑿𝟐 , … , 𝑿𝒏 be a r. s. from N (𝜇, ⟶0 as n ⟶∞


𝜇), 𝜇 > 0. Hence, x. 𝑆 2 is consistent as well as
(a) Find a consistent estimator of 𝝁𝟐 . Is it unbiased for 𝜇 2 .
unbiased? Q52. give an example of an estimator
(b) Find out an UE which is consistent? which is

𝜇 (i) Consistent but not unbiased,


Solution :- (a) x̄ ∼ N (𝜇, 𝑛)
(ii) Unbiased but not consistent,
⇒ E (x̄)= 𝜇
𝜇
(iii) Consistent as well as unbiased.
V(x̄)= 𝑛 ⟶0 as n ⟶∞
1
ANs :- (i) Let 𝑇1 = x + n
Hence x̄ is consistent for 𝜇.

204
Solving Mathematical Problems
1
Clearly, 𝑇1 = x + is consistent but E Now, P [|x -𝜇|< ∊]= P [𝜇- ∊ <x < 𝜇+ ∊ ]
n
1 μ+ ∊ 𝑑x
(𝑇1 )= 𝜇 +n ≠ 𝜇 =∫μ− ∊ 𝜋{1+(x –μ)2 }

So, it is not unbiased. 1


= [𝜋 tan−1(x – μ)] μ+ ∊ μ− ∊
[if {𝑇𝑛 } is consistent for 𝜃, the {𝑇𝑛 + 𝑎𝑛 } is
2
consistent for 𝜃 if lim 𝑎𝑛 = 0.] =𝜋 tan−1 ∊ ↛1 as n ⟶∞
𝑛⟶∞

(ii) note that, T =


𝑋1 +𝑋𝑛
is an unbiased Hence x is not consistent for 𝜇.
2
estimator of 𝜇.
2
T ∼ n(𝜇, 𝜎 ⁄2) It can be shown that for large samples,
𝑃 (1−𝑃)
T− μ ∊√2 𝜉𝑝 ∼ 𝑁 (𝜉𝑝 , 𝑛.𝑓2 (𝜉 )),
Now, P [|T- 𝜇|< ∊] = P [| 𝜎 |< ] 𝑝
𝜎
√2
Where, f(.)is the PDF of the distribution.
∊√2
= 2 I[ ]-1
𝜎
For, C (𝜇, 1) distribution, 𝜉𝑝 ∼
1
↛1 as n ⟶∞ 𝑁 (𝜉𝑝 , 4𝑛𝑓2 (𝜇))

Hence, T is unbiased but not consistent for 𝜋2 1


𝜇. ⇒ x ∼ N (𝜇,4𝑛) [∵ f(𝜇)=𝜋]

(iii) Let 𝑋1 , 𝑋2 , … , 𝑋𝑛 be a r. s. from N Hence, for large n, E (x)=𝜇


(𝜇, 𝜎 2 )
𝜋2
V (x)= 4𝑛 ⟶ 0 as n ⟶∞
2
Then x ∼ n (𝜇, 𝜎 ⁄𝑛)
⇒ x(𝜉1 ) is consistent for 𝜇.
2 2
E (x)= 𝜇 , v(x̄) = 𝜎 ⁄𝑛⟶ 0 as n⟶ ∞
𝑃
⇒ x is consistent as well as unbiased. Remarks: - By khinchinte’s WLLN: x → 𝜇,
provided E(𝑋1)=𝜇, the population mean
Q53. Show that for a r. s. from Cauchy exists. In Cauchy population, the population
distribution with location parameter 𝜇, mean does not exist and 𝜇 is not the
i.e. C(𝜇, 1), the sample mean is not populations mean but it is the population
consistent for 𝜇 but the sample median is median. Hence for 𝜇, X̄ is not consistent, but
consistent for 𝜇. x is consistent.

ANS: - let 𝑋1 , 𝑋2 , … , 𝑋𝑛 be a r. s. from C (𝜇,


1)
Q54. let 𝑿𝟏 , 𝑿𝟐 , … , 𝑿𝒏 be a r. s. from the
Then x ∼ c (𝜇, 1) population with PDF

205
Solving Mathematical Problems
−(𝐱− 𝛉),𝐢𝐟 𝐱> 𝜃 ∴ V(X) = E (𝑋 2 ) -𝐸 2 (𝑋)
f (x; 𝜃) = {𝒆
𝟎 , 𝒐𝒘
1 𝜃2
⇒ V(X) = 3 -
Show that 𝑿(𝟏) is consistent for 𝜃. 9

1 1 𝜃2 1 1 𝜃2
ANS :- 𝑓𝑋(1)(𝑥) = n[1 − V (X̄) = 𝑛2 . 𝑛(3 - )= 𝑛 (3 - )
9 9
𝑥
∫0 𝑒 −(𝑥−𝜃) 𝑑𝑥]𝑛−1 . 𝑒 −(𝑥−𝜃) ; x> 𝜃 1 1
∴ lt 𝑉(3X̄) = 9 lt 𝑉(X̄)= 9 lim ( −
𝑛⟶∞ 𝑛⟶∞ 𝑛⟶∞ 𝑛 3
= n [1 + 𝑒 −(𝑥−𝜃) − 1]𝑛−1 . 𝑒 −(𝑥−𝜃) 𝜃2
)=0
9
= n𝑒 −𝑛(𝑥−𝜃) ; x> 𝜃
∴ 3X̄ is a consistent estimator of 𝜃.
P [|𝑋(1) − 𝜃|< ∊]= P [𝜃 <𝑋(1) < 𝜃 +∊ ] =
Q56. Examine whether the WLLN holds
θ +∊ −𝑛(𝑥−𝜃)
n∫𝜃 𝑒 𝑑𝑥 for the following sequences {𝑿𝒏 } of
θ +∊
independent R.Vs:
𝑒−𝑛𝑥
𝑛𝜃 [ ]
= n𝑒 −𝑛 𝜃
I> P [𝑿𝒏 = −𝟐𝒏 ] = 𝟐−𝟐𝒏−𝟏 = P[𝑿𝒏 = 𝟐𝒏 ]
= 1- 𝑒 −𝑛∊
P [𝑿𝒏 = 𝟎]= 1- 𝟐−𝟐𝒏
⟶1 as n ⟶∞ 𝟏 𝟏
II> P [𝑿𝒏 = − 𝒏]= ½ =P [𝑿𝒏 = 𝒏]
∴ 𝑋(1) is consistent for 𝜃.
Solution :- i> 𝜇k= E(𝑋𝑘 )= (-2𝑘 ). 2−2𝑘−1 +
(2𝑘 ). 2−2𝑘−1 + 0. (1 − 2−2𝑘 )
Q55. If 𝑿𝟏 , 𝑿𝟐 , … , 𝑿𝒏 be a r. s. from f(x) = =0
𝟏
(1+𝜃x);
𝟐 And var(𝑋𝑘 )=𝛔𝑘 2 = 𝐸(X𝑘 2 )
-1< x<1, -1<𝜃 <1. Find a consistent
= (−2𝑘 )2 . 2−2𝑘−1 + (2𝑘 )2 . 2−2𝑘−1 + 0
estimator of 𝜃.
1
= 1 , k ∊ℕ
Solution :- f(x)= 2 (1+𝜃x)I -1< x< 1
1 1 1
Now, 𝑛2 ∑𝑛𝑘=1 σ𝑘 2 = 𝑛2 ∑𝑛𝑘=1 1 = 𝑛 ⟶0 as n
1 1 𝜃
∴ E(X) = ∫ (1
2 −1
+ θx)xdx = 3 ⟶∞
1 𝜃
Now, E (X̄) = 𝑛 ∑𝑛𝑖=1 𝐸 (𝑋𝑖 ) = 3 Hence, {𝑋𝑛 } obeys WLLN , by chebyshev’s
WLLN.
⇒ E (3X̄) = 𝜃
II) Here 𝜇k = 0 and 𝛔𝑘 2 = 𝑉(𝑋𝑘 ) =
1 1 1 1
Now, E (𝑋 2 ) = 2 ∫−1 𝑥 2 (1 + θx)dx = 2 𝐸 (𝑋 2 𝑘) = 𝑘 2 , n ∊ℕ
1 1
∫−1(𝑥 2 + 𝜃𝑥 3 )𝑑𝑥 = 1 1 1 𝐶
3 Now, 𝑛2 ∑𝑛𝑘=1 σ𝑘 2 = 𝑛2 ∑𝑛𝑘=1 𝑘 2 < 𝑛2

206
Solving Mathematical Problems
1 Solution :- Here, 𝜇k=0,
[∑𝑛𝑘=1 𝑘 2 is a convergent p-series,

1 1 And σ𝑘 2 = V (𝑋𝑘 ) = 𝐸(𝑋𝑘 2 ) = 2−2𝑘 , 𝑘 ∊


⇒ ∑𝑛𝑘=1 2 < ∑∞
𝑘=1 𝑘 2 = 𝑐 , a finite quantity]
𝑘 ℕ
Hence, {𝑋𝑛 }obeys WLLN, by chebyshev’s 1
Now, 𝑛2 ∑𝑛𝑘=1 σ𝑘 2 =
WLLN.
1 1
1 1 {1−( )𝑛 }
𝟏 ∑𝑛𝑘=1 2−2𝑘 = 2 . 4 4
1
Q57. Let P [𝑿𝒏 = −𝒏𝑷 ]= 𝟐 = 𝑷[𝑿𝒏 = 𝑛2 𝑛 1−
4

𝒏𝑷 ] 1 1 1 1
= . 𝑛2 {1 − (4)𝑛 } < ⟶0 as n ⟶∞
3 3𝑛2
Show that WLLN holds for the sequence
1
{𝑿𝒏 } of independent R.V.’s if P < 𝟐
𝟏
Hence, lim ∑𝑛𝑘=1 σ𝑘 2 = 0
𝑛⟶𝑘 𝑛2

Solution :- here, 𝜇𝑘 = 𝐸 (𝑋𝑘 ) = 0 ⇒ 𝑋𝑛 } obeys WLLN by chebyshev’s


WLLN.
1
σ𝑘 2 = V (𝑋𝑘 ) = 𝐸(𝑋𝑘 )2 = (−𝑘 𝑃 )2 . 2 +
1 II) 𝜇k=0
(𝑘 𝑃 )2 .
2
1
And σ𝑘 2 = 𝐸(𝑋𝑘 2 ) = (−𝑘)2 . 2 +
2𝑃 √𝑘
= 𝑘 , k ∊ℕ 1
(𝑘)2 . 2 + 0
1 1 √𝑘
Now, 𝑛2 ∑𝑛𝑘=1 σ𝑘 2 = 𝑛2 ∑𝑛𝑘=1 𝑘 2𝑃 <
3⁄
1 𝑛
∫ 𝑥 2𝑃 𝑑𝑥 =𝑘 2
𝑛2 1
1 1 3⁄
𝑛2𝑝+1 −1 Now, 𝑛2 ∑𝑛𝑘=1 σ𝑘 2 = 𝑛2 ∑𝑛𝑘=1 𝑘 2
= 𝑛2 (2𝑃+1)
1 𝑘 3⁄ 1 3⁄ 2
For, large n, 𝑛 ∑𝑛𝑘=1(𝑛) 2 ≃ ∫0 𝑥 2 𝑑𝑥 =5
1 𝑛2𝑝+1 −1
Now, 0 ≤ ∑𝑛𝑘=1 σ𝑘 2 < <
𝑛2 𝑛2 (2𝑃+1)
5
𝑛2𝑝−1 3 2𝑛 ⁄2 2
⟶0 as n ⟶∞
1
[if 2P-1< 0, if p < 2] ⇒ ∑ 𝑘 ⁄2 ≃ = 5 √𝑛 ↛ 0 ⟶
(2𝑃+1) 5
∞ as n ⟶ ∞
1 1
⇒ if p < 2 , ∑𝑛𝑘=1 σ𝑘 2 ⟶0 as n ⟶∞
𝑛2 1
As 𝑛2 ∑𝑛𝑘=1 σ𝑘 2 ↛0 as n ⟶∞
1
Hence, {𝑋𝑛 } obeys WLLN if p < .
2 We cannot draw any conclusion by
Q58. Decide whether WLLN holds for the chebyshev’s WLLN, whether WLLN holds
sequence,{ 𝑿𝒏 } o0f independent R.V.’s : or not.

𝟏 Q59. Let (𝑿𝟏 , 𝑿𝟐 , 𝑿𝟑 ) be a r.s. from Bin(1,


P [𝑿𝒏 = ±𝟐−𝒏 ]= 𝟐
p). Is T= 𝑿𝟏 + 𝟐𝑿𝟐 + 𝑿𝟑 sufficient for p
𝟏 ? is 𝑿𝟏 + 𝑿𝟐 + 𝑿𝟑 is sufficient for p?
⟺ P [𝑿𝒏 = −𝟐−𝒏 ]= 𝟐= P[𝑿𝒏 = 𝟐−𝒏 ]

207
Solving Mathematical Problems

ANS :- (i) Here T takes the value 0, 1, 2, 3, 𝜽𝒙𝜽−𝟏 ; 𝟎 < 𝑥 < 1


(i) f(x ; 𝜃)= {
4. 𝟎 , 𝒐𝒘
(𝒙−𝝁)
P [𝑋1 = 1, 𝑋2 = 0, 𝑋3 = 1|𝑇 = 2 ] 𝟏 −
𝟐𝝁𝟐
(ii) f(x ; 𝜇)= |𝝁|√𝟐𝝅 . 𝒆 ; x ∊ℝ
𝑃[𝑋1 =1,𝑋2 =0,𝑋3 =1 ;𝑇=2]
= 𝑃[𝑇=2] 𝒙𝜶−𝟏 (𝟏−𝒙)𝜷−𝟏
,𝟎 < 𝑥 < 1
(III) f (x ; 𝛼,𝛽)= { 𝜷(𝜶,𝜷)
𝑃[𝑋1 =1,𝑋2 =0,𝑋3 =1 ]
= 𝑃[𝑋 𝟎 , 𝒐𝒘
1 =1,𝑋2 =0,𝑋3 =1]+𝑃[𝑃[𝑋1 =0,𝑋2 =1,𝑋3 =0]
(𝒙−𝝁)
− 𝟏,𝒊𝒇 𝒙>𝜇
𝑝2 (1−𝑝) 𝑝
f(x ; 𝜇, 𝜆)= {𝝀 𝒆
𝝀
= = 𝑝+1−𝑝 = p, which (iv)
𝑝 (1−𝑝)+ 𝑝(1−𝑝)2
2
𝟎 , 𝒐𝒘
depends on p. (v) f(x; 𝜇, 𝛔)=
𝟏
𝟏 − (|𝒏𝒙−𝝁)𝟐 , 𝒊𝒇 𝒙>0
Hence T is not sufficient for p 𝒆 𝟐𝝈𝟐
{𝒙𝝈√𝟐𝝅
(ii) Here, 𝑋1 + 𝑋2 + 𝑋3 = 𝑇 𝟎 , 𝒐𝒘
𝜽 𝜶𝜽

Let us consider a specific case, 𝑋1 = 1, 𝑋2 = (vi) f(x;𝛼,𝜃)={𝒙𝜽+𝟏 𝒊𝒇𝒙 > 𝛼


𝟎 , 𝒐𝒘
1, 𝑋3 = 0 and T =1 𝟐(𝜽−𝒙)
;𝟎 < 𝑥 < 𝜃
(vii) f(x; 𝜃) ={ 𝜽𝟐
Here, 𝑋1 + 𝑋2 + 𝑋3 = 1 for, 𝟎 , 𝒐𝒘

{(𝑋1 = 1, 𝑋2 = 1, 𝑋3 = 0),( 𝑋1 = 1, 𝑋2 = Ans:- (I) The joint PDF of 𝑋1 , 𝑋2 , … , 𝑋𝑛 is


0, 𝑋3 = 1),( 𝑋1 = 0, 𝑋2 = 1, 𝑋3 = 1), (𝑋1 =
0, 𝑋2 = 0, 𝑋3 = 1) } f(x) = 𝜃 𝑛 (∏𝑛𝑖=1 𝑥𝑖 )𝜃−1

∴ P [𝑋1 = 1, 𝑋2 = 1, 𝑋3 = 0|𝑇 = 1] = 𝑔𝜃 {∏𝑛𝑖=1 𝑥𝑖 } . h(x). where h(x)=1

P [𝑋1 =1,𝑋2 =1,𝑋3 =0]


, if T = 1 And T (x)= ∏𝑛𝑖=1 𝑥𝑖
={ 𝑃[𝑇=1]
0 , 𝑜𝑤 ∴ By Neyman – Fisher factorization
𝑝2 (1−𝑝)
criterion,
, 𝑖𝑓 𝑇 = 1
= {3𝑝2 (1−𝑝)+ (1−𝑝)2 𝑝
T = ∏𝑛𝑖=1 𝑥𝑖 is sufficient for 𝜃.
0 , 𝑜𝑤
(𝑥−𝜇)
𝑃 1 −
, 𝑖𝑓 𝑇 = 1 (ii) f(x; 𝜇, 𝛔)= |𝜇|√2𝜎 . 𝑒 2𝜎2
={2𝑃+1
0 , 𝑜𝑤
So, X ∼ N (𝜇, 𝜇 2 ), where 𝜇 ≠ 0.
E. T is not sufficient for p.
By Ex.(3). T (x)= (∑𝑛𝑖=1 𝑋𝑖 , ∑𝑛𝑖=1 𝑋𝑖 2 ) is
Q60. Let 𝑿𝟏 , 𝑿𝟐 , … , 𝑿𝒏 be a r. s. from the sufficient for 𝜇.
following PDF & find the non trival
sufficient statistic in each case : Note : - If in the range of 𝑋𝑖 , there is the
parameter of the distribution present then we

208
Solving Mathematical Problems

have to use the concept of indicator = T (∑𝑛𝑖=1 |𝑛𝑥𝑖 , ∑𝑛𝑖=1(|𝑛𝑥𝑖 )2 ; 𝜇, 𝜎). h(x ̰) ;
function(𝑋(1) 𝑜𝑟 𝑋(𝑛) ) or min{𝜒𝑖 }. where,
𝑖
1
(iii> 𝑓𝜃 (𝑥) =
1
𝑥 . (1 − 𝑥) , if 0< x< 1 , h(x ̰) = ∏𝑛 ; T (x ̰)=
𝐵(𝛼,𝛽) 𝑖=1 𝑥𝑖

𝛼, 𝛽> 0 ∑𝑛𝑖=1 |𝑛𝑥𝑖 , ∑𝑛𝑖=1(|𝑛𝑥𝑖 )2

∴ Joint PDF of 𝑋1 , … , 𝑋𝑛 is is sufficient for 𝜇 and 𝛔.

1 𝑛 (𝛼𝜃 )𝑛
f(x ̰)= [𝐵(𝛼,𝛽)] (∏𝑛𝑖=1 𝑥𝑖 )𝛼−1 (∏𝑛𝑖=1 1 − (vi) f (x ̰)= 𝜃 𝑛 𝜃+1 if 𝑥𝑖 > 𝛼
∏𝑛
𝑖=1(𝑥𝑖 )

𝑥𝑖 )𝛽−1 1
= (𝜃𝛼 𝜃 )𝑛 . ∏𝑛 𝜃+1
𝐼(𝑥(1),𝛼 ) if 𝑥(1)> 𝛼
𝑖=1{𝑥𝑖 }
= g(T((x ̰) ; 𝛼, 𝛽)h(x ̰), where, h(x ̰)= 1 and T
where I (a, b)=1 if a > b
(x ̰) =(∏𝑛𝑖=1 𝑥𝑖 , ∏𝑛𝑖=1 1 − 𝑥𝑖 ) is jointly
sufficient for (𝛼, 𝛽) = 0 ow

1 ∑𝑛
(𝑥𝑖 −𝜇)
= g (∏𝑛𝑖=1 𝑥𝑖 , 𝑥(1); ). h (x ̰); where , h
(iv> f(x ̰)= 𝜃𝑛 . 𝑒 − 𝑖=1 𝜎 if 𝑥𝑖 > 𝜇 𝛼, 𝛼
(x ̰)= 1 and hence
1 − ∑𝑛
𝑖=1 𝑥𝑖 −𝑛𝜇
= 𝜎𝑛 . exp{ }. I (𝑥(1) ,𝜇 ) where I (a, T =∏𝑛𝑖=1 𝑥𝑖 , 𝑥(1) is sufficient for 𝜃 and 𝛼.
𝜎
b)= 1 if a ≥ b
2𝑛
(vii> f (x ̰) = 𝜃2𝑛 ∏𝑛𝑖=1(𝜃 −
𝑥𝑖 ) ; 0 <𝑥𝑖 < 𝜃
=0 ow 𝑛
2𝑛
= (𝜃2𝑛) . (𝜃 − 𝑥1 )(𝜃 −
= g(∑𝑛𝑖=1 𝑋𝑖 , 𝑥(1) ; 𝜎,𝜇 ) . h(x ̰), where h(x ̰)=
𝑥2 ) … . (𝜃 − 𝑥𝑛 ); 0 <𝑥𝑖 < 𝜃
1.
These cannot be expressed in the form of
Thus, 𝑋(1) and ∑𝑛𝑖=1 𝑋𝑖 , are jointly sufficient
factorization criterion.
statistic for 𝜇 and 𝛔.
So (𝑋1 , 𝑋2 , … , 𝑋𝑛 ) or(𝑋(1) , 𝑋(2) , … , 𝑋(𝑛) ) are
1
1 − (|𝑛𝑥−𝜇)2 , 𝑖𝑓 𝑥>0
<v> f(x; 𝜇, 𝛔)= 𝑥𝜎√2𝜋 𝑒 2𝜎2 trivally sufficient for here , ̰ .there is no
non- trival sufficient statistic.
The joint PDF of x ̰ is
Q61. Let 𝑿𝟏 , … , 𝑿𝒏 be a r.s. from gamma
F(x ̰) = distn with pdf
1 1
(∏𝑛 𝑛 𝑛 . exp{− 2𝜎2 ∑𝑛𝑖=1(|𝑛𝑥𝑖 −
𝑖=1 𝑥𝑖 )𝜎 (√2𝜋) 𝜶𝑷
𝒇𝜽 (𝒙) = ⎾(𝑷) 𝐞𝐱𝐩[−𝜶𝒙]𝒙𝑷−𝟏 if 0< x< ∞
𝜇)2 ,} if 𝑥𝑖 > 0
, where 𝛼> 0, P > 0
2
∑(1𝑛𝑥𝑖 ) 𝜇 ∑ 1𝑛𝑥𝑖 𝑛𝜇2
1 −( − + 2) 1
= 𝜎𝑛 (√2𝜋)𝑛 . 𝑒 2𝜎 2 𝜎2 𝜎 . (∏𝑛 Show that ∑𝒊 𝑿𝒊 𝒂𝒏𝒅 ∏𝒊 𝑿𝒊 are jointly
𝑙=1 𝑥𝑖 )
sufficient for (𝛼, P)

209
Solving Mathematical Problems

Solution:- f(x )̰ = 𝒇𝜽 (𝒙)=


𝟏
, where -∞ < 𝜃 <∞, is
𝝅[𝟏+ (𝒙− 𝜽)𝟐 ]
𝛼𝑃 𝑛 𝑃−1
{⎾(𝑃)} . exp[−𝛼 ∑𝑖 𝑋𝑖 ] . (∏𝑖 𝑋𝑖 ) considered.

= g (T(x )̰ ; , P). h (x )̰ ; where h(x )̰ =1 Then can you have a single sufficient
statistic for 𝜃?
∴ T (x )̰ = (∑𝑛𝑖=1 𝑋𝑖 , ∏𝑛𝑖=1 𝑋𝑖 ) is jointly
sufficient for (𝛼, P). Solution:- The PDF of (𝑋1 , … , 𝑋𝑛 ) is

𝟏 𝒙⁄ 𝑛
Q62. If f(x)= 𝜽 𝒆− 𝜽; 𝟎 <𝑥< 1
∏ 𝑓 (𝑋𝑖 , 𝜃) =
𝜃. 𝐹𝑖𝑛𝑑 𝑎 𝑠𝑢𝑓𝑓𝑖𝑐𝑖𝑒𝑛𝑡 𝑒𝑠𝑡𝑖𝑚𝑎𝑡𝑜𝑟 𝑓𝑜𝑟 𝜃. 𝜋 𝑛 {∏𝑛𝑖=1[1 + ((𝑋𝑖 − 𝜃)2 ]}
𝑖=1

1 1
Solution:- f(x )̰ = 𝜃𝑛 . 𝑒𝑥𝑝 {− 𝜃 ∑𝑛𝑖=1 𝑥𝑖 } Note that, ∏𝑛𝑖=1{1 + ((𝑋𝑖 − 𝜃)2 }

= g {∑𝑛𝑖=1 𝑥𝑖 , 𝜃}. h (x )̰ ; where h (x ̰)= 1. = {1 + ((𝑥1 − 𝜃)2 }{1 + ((𝑥2 − 𝜃)2 }….
{1 + ((𝑥𝑛 − 𝜃)2 }
∴T = ∑𝑛𝑖=1 𝑥𝑖 is sufficient statistic for 𝜃.
= 1+ term involving one 𝑋𝑖 + term involving
Q63. If 𝒇𝜽 (𝒙)= ½ ; 𝜃 -1< x < 𝜃 -1 , then two 𝑋𝑖 ′𝑠 + ……. + term involving all 𝑋𝑖 ′𝑠.
show that 𝑿(𝟏) 𝒂𝒏𝒅 𝑿(𝒏) are jointly
= 1+ ∑𝑖(𝑥𝑖 − 𝜃)2 + ∑𝑖 ∑≠𝑗(𝑥𝑖 − 𝜃)2 ((𝑥𝑗 −
sufficient for 𝜃. (𝑿𝒊 ∼ 𝑼(𝛉 − 𝟏, 𝛉 + 𝟏))
2
𝜃) + … . + ∏𝑛𝑖=1((𝑥𝑖 − 𝜃)2
1
Solution:- f(x )̰ =(2)𝑛
Clearly, ∏𝑛𝑖=1 𝑓(𝑥𝑖 , 𝜃)cannot be written as g
1
= 2𝑛 . 𝐼 (θ − 1, 𝑥(1) )𝐼 (𝑥(𝑛) , θ + 1); θ − 1< (T (x ̰), 𝜃). h(x ̰)
𝑥(1) < 𝑥(𝑛) < θ + 1 For a statistic other than the trivial choices
1 𝑖𝑓 𝑎 < 𝑏 (𝑋1 , … , 𝑋𝑛 ) or (𝑋(1) , … , 𝑋(𝑛) ).
Where I (a, b)= {
0 𝑖𝑓 𝑎 ≥ 𝑏
1
Hence there is no non-trivial sufficient
= g (T (x )̰ ; 𝜃)h(x )̰ ; where h(x )̰ = 2𝑛 . statistic

∴ T (x )̰ = (𝑋(1) , 𝑋(𝑛) )is jointly sufficient for Therefore, in this case, no reduction in the
𝜃. space is possible.

Q64. let 𝑿𝟏 , 𝑿𝟐 , … , 𝑿𝒏 be a R.S. from c (𝜃, ⇒ The whole set (𝑋1 , … , 𝑋𝑛 ) is jointly
1), where 𝜃 is the location parameter, S. sufficient for 𝜃.
T. there is no sufficient statistic other
Q65. Let 𝑿𝟏 𝒂𝒏𝒅 𝑿𝟐 be iid RVS having
than the trivial statistic (𝑿𝟏 , 𝑿𝟐 , … , 𝑿𝒏 ) or
the discrete uniform distribution on {1, 2,
(𝑿(𝟏) , 𝑿(𝟐) , … , 𝑿(𝒏) )
……, N}, where n is unknown. Obtain the
If a random sample of size n ≥ 2 from a conditional distribution of 𝑿𝟏 , 𝑿𝟐 given
Cauchy distn with p.d.f. (T = max (𝑿𝟏 , 𝑿𝟐 ))

210
Solving Mathematical Problems

Hence, show that T is sufficient for n but 2𝑁−𝑡+1


= 𝑁2
𝑿𝟏 + 𝑿𝟐 is not.
∴ P [𝑋1 = 𝑥1 ; 𝑋2 = 𝑥2 |𝑇 = 𝑡]=
ANS :- (i) p (T= t)= P [max (𝑋1 , 𝑋2 ) = 𝑡] 𝑃 [𝑋1 =𝑥1 ; 𝑋2 =𝑥2 ]
𝑃 [𝑋1 + 𝑋2 =𝑡]
= P [𝑋1 < 𝑡, 𝑋2 = 𝑡]+ P [𝑋1 = 𝑡, 𝑋2 <
1
𝑡]+ P [𝑋1 = 𝑡, 𝑋2 = 𝑡] 𝑁2 1
(𝑡−1) = 𝑖𝑓 𝑋1 + 𝑋2 = 𝑡
𝑡−1
𝑁2
= P [𝑋1 < 𝑡] P [𝑋2 = 𝑡]+ P [𝑋1 = 𝑡] P = 1
𝑁2 1
[𝑋2 < 𝑡] + P [𝑋1 = 𝑡] P [𝑋2 = 𝑡] 2𝑁−𝑡+1 = 2𝑁−𝑡+1 𝑖𝑓 𝑋1 + 𝑋2 = 𝑡
{ 𝑁2
Now, P [𝑋1 < 𝑡]= P [𝑋2 = 1]+ P [𝑋2 =
Which depends on N, so for the 2nd case
2]+ …. + P [𝑋2 = 𝑡 − 1]
(𝑋1 + 𝑋2)is not sufficient.
1 1 1
=⏟ + 𝑁 + ….+
𝑁 𝑁 Q66. Let 𝑿𝟏 , 𝑿𝟐 , … , 𝑿𝒏 be a R.S. from one
of the following two PDFs
(t-1) times
𝟏 ,𝟎 < 𝑥 < 1
𝑡−1 1 If 𝜃 = 0, f(x/𝜃)= {
= & P [𝑋1 = 𝑡]= P [𝑋2 = 𝑡]= 𝑁 𝟎 , 𝒐𝒘
𝑁
𝟏
∴ P [T = t]= 𝑁.
1 𝑡−1
+
𝑡−1 1 1 1
.𝑁 + 𝑁.𝑁 ,𝟎 < 𝑥 < 1
𝟐
𝑁 𝑁 If 𝜃=1, , f(x/𝜃)= { √𝒙
𝟎 , 𝒐𝒘
2 (𝑡−1)+1
= 𝑁2 Find the MLE of 𝜃.
∴ P [𝑋1 = 𝑥1 , 𝑋2 = 𝑥2 |𝑇 = 𝑡]= Solution: - The Likelihood function is
𝑃[𝑋1 =𝑥1 ,𝑋2 = 𝑥2 ]
, 𝑖𝑓 𝑀𝑎𝑥(𝑥1 , 𝑥2 ) = 𝑡
{ 𝑃 [𝑇=𝑡] 𝑥
L (𝜃/ x ̰)= ∏𝑛𝑖=1 𝑓 ( 𝜃𝑖 ) , 𝜃 ∊ 𝛺 = (0, 1)
0 , 𝑜𝑤
1 1
. 1 When 𝜃 = 0, L (𝜃/ x ̰) =
𝑁𝑁
= 2 (𝑡−1)+1 = , which is independent 1 𝑖𝑓 0 < 𝑥𝑖 < 1 ∀ 𝑖 = 1(1)𝑛
2 (𝑡−1)+1
𝑁2 {
0 , 0𝑤
of N.
When 𝜃= 1, L (𝜃/ x ̰)
(ii) T = 𝑋1 + 𝑋2 then, 1
2𝑛
, 0 < 𝑥𝑖 < 1 , 𝑖 = 1(1)𝑛
For 2 ≤ 𝑡 ≤ 𝑁 + 1 ;P [T=t]= P [𝑋1 = 2, ={ √∏𝑛
𝑖=1 𝑥𝑖

𝑋2 = 𝑡 − 1]+ P [𝑋1 = 2, 𝑋2 = 𝑡 − 2] + 0 , 𝑜𝑤
…+P [𝑋1 = 2, 𝑋1 = 𝑡 − 1, 𝑋2 = 1] 𝐿 (θ=1/ x ̰)
Now, 0 ⋛1
(𝑡−1) 𝐿 (θ= )

= 𝑁2
1 1/𝑛
iff ⋛ 1, where G = (∏𝑛𝑖=1 𝑥𝑖 )
For N +2≤ 𝑡 ≤ 2𝑁 ; P [T=t]= P[𝑋1 = 𝑡 − √4𝑛 𝐺 𝑛

𝑁, 𝑋2 = 𝑁] + P [𝑋1 = 𝑡 − 𝑁 + 1, 𝑋2 = 1
iff 4 G ⋚ 1 iff G⋚ 4
𝑁 − 1]+ …..+P [𝑋1 = 𝑁, 𝑋2 = 𝑡 − 𝑁]
211
Solving Mathematical Problems
1
1 𝑖𝑓 𝐺 < 4 Hence, L (𝜇, 𝜎 2 /x ̰) is maximum at (𝜇, 𝜎 2 )=
1 (𝜇, 𝜎 2 )
0 𝑖𝑓 𝐺 > 4
Hence MLE of 𝜃 is 𝜃’ =
0,1 𝑖𝑓 𝐺 = 4
1 Therefore, the MLE of (𝜇, 𝜎 2 ) is
{ (𝜇, 𝜎 2 ) = (x, S 2 ) where nS 2 = ∑𝑛𝑖=1(𝑋𝑖 − x)2 .
Q67. Let 𝑿𝟏 , … , 𝑿𝒏 be a R.S. from N (𝜇, Q68. Let 𝑿𝟏 , … , 𝑿𝒏 be a R.S. from f (x;
𝝈𝟐 ), 𝜇 𝜖R, 𝜎 > 0 𝟏 |𝒙−𝝁|
𝜇,)= 𝟐𝝈 𝒆− 𝝈 ; 𝒙 𝝐 𝑹,where 𝜇 𝜖R, 𝜎 > 0.
𝟐
Find the MLE of (𝜇, 𝝈 ). Find the MLE of 𝜇 and 𝜎.
Solution: - Likelihood function: Solution:- The log- likelihood function is
L (𝜇, 𝜎 2 /x ̰) = 1
1
L (𝜇, 𝜎 2 /x ̰) = -n|n2- n|n𝜎- 𝜎 ∑ | 𝑥𝑖 −
1 − 2 ∑𝑛
𝑖=1(𝑥𝑖 −𝜇)
2
.𝑒 2𝜎 ; 𝑥𝑖 𝜖ℝ where μ ϵ R, σ >
𝜇| ; 𝜇 𝜖𝑅, 𝜎 > 0
(2𝜋𝜎2 )𝑛/2
0
[As∑ | 𝑥𝑖 − 𝜇|is not differentiable w.r.t. 𝜇,
𝑛
⇒ |nL (𝜇, 𝜎 2 /x ̰)= constant (- 2 |𝑛𝜎 2 − hence the derivative technique is not
1 applicable for maximizing |nL w.r.t.𝜇]
∑(𝑥𝑖 − 𝜇)2 )
2𝜎2
We adopt two stage maximization:-
𝜕|𝑛𝐿 1 ∑ 𝑥𝑖
0= = − 2𝜎2 ∑ 2(𝑥𝑖 − 𝜇)(−1) = −
𝜕𝜇 𝜎2 First fix 𝜎, and then maximize |nL for
𝑛𝜇
𝜎2 variation in 𝜇.

𝜕|𝑛𝐿 𝑛 ∑(𝑥𝑖 −𝜇)2 For fixed 𝜎, lnL is maximum,


0= = − 2𝜎2 +
𝜕𝜎2 2𝜎4
iff, ∑ | 𝑥𝑖 − 𝜇|is minimum
𝜇= ́ x
⇒{ 2 1 , the likelihood
𝜎 = 𝑛 ∑(𝑥𝑖 − x)2 iff, 𝜇=x= the sample median
function has a unique solution. = 𝜇, say.
Note that, the matrix of second order partial Now, we maximize |nL (𝜇, 𝜎 2 /x ̰)=-n|n2-
derivatives at (𝜇, 𝜎 2 ) is 1
n|n𝜎- 𝜎 ∑ | 𝑥𝑖 − 𝜇|, 𝑤. 𝑟. 𝑡. 𝜎
𝛿 2 |𝑛𝐿 𝛿 2 |𝑛𝐿
𝜕𝜇 2 𝛿𝜇𝛿𝜎2 𝛿
( 𝛿2 |𝑛𝐿 ) (𝜇, 𝜎 2 )= (𝜇, 𝜎 2 ) Note that 𝛿𝜎|nL (𝜇, 𝜎 2 /x ̰)
𝛿 2 |𝑛𝐿
𝛿𝜎2 𝛿𝜇 𝛿(𝜎2 )2 𝑛 1
= -𝜎 + 𝜎2 ∑ | 𝑥𝑖 − 𝜇|
𝑛
− 𝜎2 0
=( 𝑛 ) is negative definite (n. d.) 𝑛 1
= -𝜎2 {𝜎 − 𝑛 ∑ | 𝑥𝑖 − 𝜇|}
0 − 2𝜎4

212
Solving Mathematical Problems

1 Now we shall maximize L (𝜇, 𝜎/x) w.r.t.𝜎


> 0, 𝜎 < ∑ | 𝑥𝑖 − 𝜇|
{ 𝑛
∑(𝑥𝑖 −𝜇)
1 Now, in L(𝜇, 𝜎/x)= -nln𝜎-
< 0, 𝜎 > ∑ | 𝑥𝑖 − 𝜇| 𝜎
𝑛
𝛿 σ 𝑛 1
By, 1st derivative test, |nL (𝜇, 𝜎 2 /x ̰) is Note that, 𝛿𝜎 𝑙𝑛L(μ, x )= -𝜎 + 𝜎2 ∑(𝑥𝑖 − 𝜇)
1
maximum at 𝜎=𝑛 ∑𝑛𝑖=1 |𝑥𝑖 − 𝜇| 𝑛
= 𝜎2 {𝜎-(x − x(1) )}
Hence, the MLE of 𝜇 and 𝜎 are 𝜇= x ,
1
> 0 𝑖𝑓 𝜎 < x − x(1)
𝜎= 𝑛 ∑ | 𝑥𝑖 − x |. {
< 0 𝑖𝑓 𝜎 > x − x(1)

Hence, L(𝜇, 𝜎/x) is maximum at 𝜎=x −


x(1) = 𝜎
Q69. let 𝑿𝟏 , 𝑿𝟐 , … , 𝑿𝒏 be a R.S. from
(𝒙−𝝁) Therefore, The MLE of 𝜇 and 𝜎 are 𝜇= x(1)
𝟏 − ⁄𝝈 ,𝒊𝒇 𝒙> 𝜇
F(x; 𝜇,)= {𝝈 𝒆 and 𝜎= x − x(1)
𝟎 , 𝒐𝒘
(ii) When 𝜎= 𝜇> 0
Where 𝜇 ∊ R, 𝜎 > 0. Find the MLE of (i) 𝜇
∑(𝑥𝑖 −𝜇)
and 𝜎 1 − ; x(1) ≥ μ
L (𝜇/ x ̰) = {𝜇𝑛 𝑒 𝜇

(ii)𝜇 0 , 𝑜𝑤
when 𝜎= 𝜇 (>0)
L (𝜇/ x ̰) is maximum iff
Solution:- (i) The likelihood function is
For 𝜇 ≤ x(1)
∑(𝑥 −𝜇)
1 − 𝑖 ;𝑖𝑓 𝑥(1) ≥ 𝜇
L (𝜇, 𝜎/x)= {𝜎𝑛 . 𝑒
𝜎
𝛿 𝛿 1
0, 𝑜𝑤 𝑙𝑛L = {−𝑛𝑙𝑛𝜇 − ∑(𝑥𝑖 − 𝜇) }
𝛿𝑢 𝛿𝑢 𝜇
𝜇 ∊ R, 𝜎> 0
𝑛
We adopt two stage maximization. = -𝜇2 (𝜇 − x)

First fix 𝜎, then maximize L (𝜇, 𝜎/x) w.r.t.𝜇 > 0 𝑖𝑓 𝜇 < x


{
< 0 𝑖𝑓 𝜇 > x
For fixed 𝜎, L (𝜇, 𝜎/x) is maximum
⇒ L (𝜇/ x ̰) is maximum at 𝜇 = x
iff ∑(𝑥𝑖 − 𝜇) is minimum subject to 𝜇 ≤
𝑥(1) From the graph for 𝜇 ≤ 𝑥(1), L (μ/ x ̰) is
maximum at 𝜇 = 𝑥(1),
iff 𝜇 is as large as possible subject to the
restriction ≤ 𝑥(1) . Therefore, 𝜇 = 𝑥(1), is the MLE of 𝜇.

iff 𝜇= 𝑥(1) = 𝜇 (say)

213
Solving Mathematical Problems

Q70. Let X be a single observation from 1 ,1 < 𝑥 < 3


For k=2, the test 𝛷 (x)={
the PDF 0 , 𝑜𝑤

𝟏 Is an MP test of 𝐻0 against 𝐻1 of its size


F (x; 𝜃) = {𝝅{𝟏+(𝒙−𝜽)𝟐 } , 𝒙 ∊ ℝ
= E [𝛷 (x)/ 𝐻0 ]= P [1< x<3/𝜃=0]
Show that the test
3 1 1
= ∫1 𝑑𝑥= 𝜋 [tan−1 𝑥]3,1
𝒇(𝒙,𝟏) 𝜋(1+𝑥 2 )
𝟏, 𝒇(𝒙,𝟎) > 𝑘
𝛷 (x)= { 𝟎 , 𝒐𝒘 1
=𝜋 [tan−1 3 − tan−1 1]

1 3−1
Is an MP test of 𝑯𝟎 : 𝜽 = 𝟎 aginst 𝑯𝟏 : 𝜽 = = 𝜋 tan−1 1+3.1
𝟏 of its size. 1 1
= tan−1 .
𝜋 2
Solution:- For a particular value of k, the
test Q71. Find an MP test of testing 𝑯𝟎 such
that 𝑯𝟎 ∶ 𝑿 ∼ 𝒇𝟎 (𝒙) against 𝑯𝟏 ∶ 𝑿 ∼
𝑓(𝑥,1)
1, 𝑓(𝑥,0) > 𝑘 𝒇𝟏 (𝒙) of its size, where
𝛷 (x)= { 0 , 𝑜𝑤
𝟏 𝒙𝟐⁄ ,
𝒇𝟎 (𝒙)= 𝒆− 𝟐𝒙 ∊𝑹
√𝟐𝝅
Is an MP test of 𝐻0 : 𝜃 = 0 aginst 𝐻1 : 𝜃 = 1 𝟏
of its size, by NP lemma, 𝒇𝟏 (𝒙)= 𝟐 𝒆−|𝒙| , 𝒙 ∊ 𝑹

𝑓(𝑥,1) 1+𝑥 2 S.T. the power of the test is greater than


Now, 𝑓(𝑥,0) > 𝑘⇒ 1+(𝑥−1)2 > 𝑘
its size.
⇒ 𝑥 2 (𝑘 − 1) − 2𝑘𝑥 + (2𝑘 − 1) < 0 Solution:- By N-P lemma, for a particular
2𝑘 2𝑘−1 value of k, the test
[If (k-1)> 0, 𝑥 2 − (𝑘−1) 𝑥 + <0
𝑘−1
𝑓 (𝑥)
1 , 𝑓1 (𝑥) > 𝑘
⇒ (x- α)(x-𝛽)< 0 𝛷(x)={ 0
0 , 𝑜𝑤
2𝑘 2𝑘−1
Where, 𝛼+𝛽= (𝑘−1), and 𝛼𝛽= Is an MP test of 𝐻0 against 𝐻1 of its size.
𝑘−1

𝑓 (𝑥)
⇒ 𝛼< x< 𝛽 Now, 𝑓1 (𝑥) > 𝑘
0

In the given MP test 𝛼=1, 𝛽=3 1 2 −2|𝑥|}


⇒ 𝑒 2{𝑥 > 𝑘1
2𝑘
Hence, 1+3 =(𝑘−1) ⇒ k=2] 1
⇒ 𝑒 2 {(|𝑥| − 1)2 − 1} > 𝑘1
𝑓(𝑥,1)
Set, k=2, 𝑓(𝑥,0) > 2
⇒ (|𝑥| − 1)2 > 𝑘2 2 , 𝑘2 > 0

⇒ 1< x < 3
214
Solving Mathematical Problems

⇒ |x|-1 < -𝑘2 or |x|-1>𝑘2 𝑓 (𝑥)


Note that , 𝑓1 (𝑥) > 𝑘
0

⇒ |x|< 𝐶1 or |x|> 𝐶2 2
𝑒𝑥
⇒ > 𝑘1 , say
[Alternative: - note that 𝑓1 (𝑥) has more 1+𝑥 2

probability in its tails and near 0 than 𝑓0 (𝑥) 𝑒𝑥


2

has. If either a very large or very small value Let u (x) = 1+𝑥 2
of x is observed, we suspect that 𝐻1 is true 2 2
(1+𝑥 2 )𝑒 𝑥 .2𝑥−𝑒 𝑥 .2𝑥
rather than𝐻0 . For some 𝐶1 and𝐶2 , we shall Now, u′ (x)= (1+𝑥 2 )2
𝑓1 (𝑥)
reject 𝐻0 iff >𝑘
𝑓0 (𝑥) 2
2𝑥 3 .𝑒 𝑥
= (1+𝑥 2 )2
To |x|< 𝐶1 or |x|<𝐶2 .]
2 2
Hence, for some 𝐶1 and𝐶2 , the test [u′ (0)=0 ⇒2𝑥 3 . 𝑒 𝑥 = 0 ⇒ 𝑥 = 0 𝑜𝑟𝑒 𝑥 =
0 ⇒ 𝑥2 = ∞ ]
1 , |x| < 𝐶1 or |x| < 𝐶2
𝛷(x) = { < 0 , 𝑖𝑓 𝑥 < 0
0 , 𝑜𝑤 ={
> 0 , 𝑖𝑓 𝑥 > 0
Is an MP test of 𝐻0 against 𝐻1 of its size
From the graph ,u(x) >𝑘1
Note, that, 𝛽𝛷( 𝑓1 ) = 𝑃[1 × 1 < 𝐶1 𝑜𝑟 1 ×
1 < 𝐶2 ] ⇔ |x|>𝑐1

𝑓 Hence, for a particular value of 𝑐1, the test


= ∫𝑤1 𝑓1 (𝑥)𝑑𝑥 , 𝑤 = {𝑥: |𝑥| < 𝐶1 𝑜𝑟 |𝑥| >
𝐶2 } 1 , |x| > 𝐶1
𝛷(x) = {
0 , 𝑜𝑤
>∫𝑤 𝑓0 (𝑥)𝑑𝑥 , as 𝑓1 (𝑥) > 𝑓0 (𝑥)∀𝑥𝜖𝑤
Is an MP test 𝐻0 against 𝐻1 of its size.
= 𝑃𝑓0 [1 × 1 < 𝐶1 𝑜𝑟 1 × 1 < 𝐶2 ] Q73. Find on MP test at level 𝛼= 0.05 for
testing 𝑯𝟎 : X ∼ N (0, 1)against 𝑯𝟏 : X
= 𝛽𝛷 (𝑓0 ) . (Proved).
∼C(0, 1).
Q72. Find an MP test of 𝑯𝟎 : X∼N(0, ½ )
Solution:- for a given k, the test 𝛷(x) =
against 𝑯𝟏 : X∼c (0, 1) of its size . 𝑓 (𝑥)
1 , 𝑓1 (𝑥) > 𝑘
Solution :- For a given K , the test 𝛷 (x) = { 0
0 , 𝑜𝑤
𝑓 (𝑥)
1, 𝑓1 (𝑥) > 𝑘
{ 0
Is an MP test of 𝐻0 against 𝐻1 of its size, by
0, 𝑜𝑤 𝑓 (𝑥)
NP lemma. Note that. R (x)= 𝑓1 (𝑥) > 𝑘
0
Is an MP test of 𝐻0 against 𝐻1 of its size,
𝑥2⁄
By N-P lemma, 𝑒 2
⇒ 1+𝑥 2
> 𝑘1 , say.

215
Solving Mathematical Problems
𝑥2⁄ Is an MP test for testing𝐻0 : X ∼ N (0, 1)
𝑒 2
Let u (x) = 1+𝑥 2 against 𝐻1 : X ∼c (0, 1) at level 𝛼= 0.05.

< 0 , 𝑥 < −1 Q74. Let 𝑿𝟏 , … , 𝑿𝒏 be a R.S. from f(x; 𝜃)


> 0 , −1 < 𝑥 < 0 𝜽 𝒆−𝜽𝒙 , 𝒊𝒇 𝒙 > 𝜃
Note that, u′ (x)= { ={
< 0 ,0 < 𝑥 < 1 𝟎 , 𝒐𝒘
> 0 ,𝑥 > 1
Find the size 𝛼 LRT of (i) 𝑯𝟎 :𝜃=
[for k> 0.7979, then the critical region:
𝜽𝟎 𝒂𝒈𝒂𝒊𝒏𝒔𝒕 𝑯𝟏 : 𝜃≠ 𝜽𝟎
|x| > 𝑐2 with size< 0.1118.
(ii) 𝑯𝟎 :𝜃= 𝜽𝟎 𝒂𝒈𝒂𝒊𝒏𝒔𝒕 𝑯𝟏 : 𝜃> 𝜽𝟎
For 0.6524≤ k≤ 0.7979,
(𝒊𝒊𝒊 > 𝑯𝟎 :𝜃≥ 𝜽𝟎 𝒂𝒈𝒂𝒊𝒏𝒔𝒕 𝑯𝟏 : 𝜃< 𝜽𝟎
Then critical region:
Solution:- The likelihood function is
|x|>𝑐1 𝑜𝑟|x| > 𝑐2with size 𝜖 (0.1118, n −𝜃 ∑𝑖=1 𝑥𝑖 𝑛

0.3913) L (x̰ ; θ) = {θ 𝑒 , 𝑖𝑓 𝑥𝑖 > 𝜃


0 , ow
For, k < 0.6524, the critical region: x 𝜖R
Where , 𝜃> 0
with size=1]
(i) To test 𝐻0 :𝜃= 𝜃0 𝑎𝑔𝑎𝑖𝑛𝑠𝑡 𝐻1:
For 𝛼= 0.05, a small quantity, then u(x)> 𝑘1 ,
𝜃≠ 𝜃0 :∼
where 𝑘1 is such that P [u(x)> 𝑘1 /𝐻0 ]=0.05
and from the graph u(x)> 𝑘1 ⇔ |x|> 𝑐2 . Here 𝛺0 = {𝜃0 } and 𝛺= {0, 𝜃> 0}
1
−𝜃0 𝑛x+ nx
1 , |x| > 𝐶2 [∵𝑒 x
Hence, 𝛷(x) = {
0 , 𝑜𝑤

Is an MP test of 𝐻0 against 𝐻1 at level 𝛼= =𝑒


−𝜃0 𝑛x+n
]
0.05, where
The likelihood ratio is
0.05= P [|x| > 𝑐2 /𝐻0 ]
L (x̰ ; θ)
𝜃𝑠𝑢𝑝 𝜖𝛺0 L (x̰ ; 𝜃0 )
=P [[|x| > 𝑐2 /X ∼ N (0, 1)] 𝜆= = 𝑛
𝜃𝑠𝑢𝑝 𝜖𝛺L (x̰ ; θ) ∑
(𝜃ˆ)𝑛 .𝑒 −𝜃 𝑖=1 𝑥𝑖

= 2 [1- ⏀ (𝑐2 )] 1
Where, 𝜃ˆ = is the MLE of 𝜃 under 𝛺.
𝑥
⇒ 1- ⍕(𝑐2 ) = 0.025 = 1-⍕ (0.025)
−𝑛(𝜃0 x−1)
Here, 𝜆= (𝜃0 x)𝑛 . 𝑒
⇒ 𝑐2 = 0.025
−𝑛(𝑦−1)
= 𝑦𝑛. 𝑒 , where y = 𝜃0 x
=1.96
𝑑𝜆 −𝑛(𝑦−1)
Now, 𝑑𝑦= 𝑦 𝑛 . 𝑒 (-
1 , |x| > 1.96
Hence, 𝛷(x) ={ −𝑛(𝑦−1)
0 , 𝑜𝑤 n)+𝑛𝑦 𝑛−1 . 𝑒

216
Solving Mathematical Problems
−𝑛(𝑦−1)
= 𝑛𝑦 𝑛−1 . 𝑒 {1 − 𝑦} (𝜃 x)𝑛 . 𝑒 −𝑛(𝜃0 x−1),𝑖𝑓 𝜃0 x<1
Now, 𝜆= { 0
1 , 𝑖𝑓 𝜃0 x ≥ 1
> 0 𝑖𝑓 𝑦 < 1
={
< 0 𝑖𝑓 𝑦 > 1 From graph, 𝜆< c(<1)

From graph, 𝜆< c ⇒ y< k

⇒ y < 𝑘1 𝑜𝑟 y > 𝑘2 ⇒ 2𝜃0 ∑𝑛𝑖=1 𝑥𝑖 < a , say

⇒ 2 𝜃0 ∑𝑛𝑖=1 𝑥𝑖 < 𝑎 𝑜𝑟 2 𝜃 ∑𝑛𝑖=1 𝑥𝑖 > 𝑏 The size 𝛼 LRT is given

Where, 2n𝑘1 = 𝑎, 2𝑛𝑘2 = 𝑏 by: reject 𝐻0 iff 𝜆 < c, iff 2 𝜃0 ∑𝑛𝑖=1 𝑥𝑖 < 𝑎

Here, the size 𝛼LRT is given by: Where ‘a’ is such that 𝛼= P𝐻0 [2
𝜃0 ∑𝑛𝑖=1 𝑥𝑖 < 𝑎]
Reject 𝐻0 iff 𝜆 > c iff 2 𝜃0 ∑𝑛𝑖=1 𝑥𝑖 ∉ [𝑎, 𝑏]
∴ 𝛼 = P [𝜒 2 2𝑛 < 𝑎]
Where ‘a’, ‘b’ are such that
⇒ a=𝜒 21−𝛼;2𝑛
𝛼 = P𝐻0 [2 𝜃0 ∑𝑛𝑖=1 𝑥𝑖 ∉ [𝑎, 𝑏]]

= 1- P𝐻0 [a ≤ 2 𝜃0 ∑ 𝑥𝑖 ≤ 𝑏] Therefore the size 𝛼 LRT is given by:

𝜒2 1−𝛼;2𝑛
= 1- P [a ≤ 𝜒 2 2𝑛 ≤ 𝑏] Reject 𝐻0 iff ∑𝑛𝑖=1 𝑥𝑖 < .
2𝜃0

= 1- F𝜒 2 2𝑛 (𝑏)+ 𝜒 2 2𝑛 (𝑎) and (𝑘1 )= 𝜆(𝑘2 )⇒ 𝜃𝑠𝑢𝑝 ≥𝜃0 L (x̰ ; θ)


(III) Hint:- 𝜆 =
−𝑛(𝑘1 −1) 𝜃𝑠𝑢𝑝 𝜖𝑅 L (x̰ ; θ)
𝑘1 𝑛 . 𝑒
−𝑛(𝑘2 −1) The size 𝛼 LRT is given by: Reject 𝐻0 iff
=𝑘2 𝑛 . 𝑒 𝜒2 𝛼;2𝑛
∑𝑛𝑖=1 𝑥𝑖 > .
2𝜃0
(II) To test 𝐻0 :≥ 𝜃0 𝑎𝑔𝑎𝑖𝑛𝑠𝑡 𝐻1: 𝜃> 𝜃0 :-

Here, 𝛺0 = {𝜃0 } and 𝛺= { ≥ 𝜃0 }


Q75. let 𝑿𝟏 , 𝑿𝟐 , … , 𝑿𝒏 be a R.S. from N
The likelihood ratio is
(𝜃, 𝝈𝟐 ), 𝜎 known. Derive size 𝛼 LRT for
L (x̰ ; θ) 𝑛
𝜃𝑠𝑢𝑝 𝜖𝛺0 𝜃0 𝑛 .𝑒 −𝜃0 ∑𝑖=1 𝑥𝑖 testing
𝜆= =
𝜃𝑠𝑢𝑝 𝜖𝛺L (x̰ ; θ) 𝜃𝑠𝑢𝑝 ≥𝜃0 {L (x̰ ; θ)}
(i) 𝑯𝟎 :𝜃= 𝜽𝟎 𝒂𝒈𝒂𝒊𝒏𝒔𝒕 𝑯𝟏 : 𝜃≠ 𝜽𝟎
1
For 𝜃 > 0, L (x̰ ; θ) is maximum at 𝜃= x =
(ii) 𝑯𝟎 :𝜃= 𝜽𝟎 𝒂𝒈𝒂𝒊𝒏𝒔𝒕 𝑯𝟏 : 𝜃> 𝜽𝟎
𝜃ˆ
Show that the LRT’s obtained are
∴ Sup L (x̰ ; θ) 𝜃≥ 𝜃0 unbiased.
(𝜃ˆ)𝑛 𝑒 −𝜃ˆ2𝑥𝑖 ,𝑖𝑓𝜃0 < 𝜃ˆ Solution:- the likelihood function is
={
𝜃0 𝑛 . 𝑒 −𝜃0 ∑ 𝑥𝑖 ,𝑖𝑓𝜃0 >𝜃ˆ

217
Solving Mathematical Problems

L (x̰ ; θ) = 𝜃 𝑠𝑢𝑝 𝜖𝛺0 L (x̰ ; θ)


The likelihood ratio is 𝜆=
1 𝜃 𝑠𝑢𝑝𝜖 ΩL (x̰ ; θ)
1 − ∑𝑛 (𝑥 −𝜃)2
(𝜎√2𝜋)𝑛 . 𝑒 2𝜎2 𝑖=1 𝑖 ; 𝑤ℎ𝑒𝑟𝑒 𝜃 𝜖ℝ
L (x̰ ; θ)
= 𝜃sup ≥ 𝜃
0 L (x̰ ; θ)
(i) To test 𝐻0 :𝜃= 𝜃0 𝑎𝑔𝑎𝑖𝑛𝑠𝑡 𝐻1:
𝜃≠ 𝜃0 :- Here L (x̰ ; θ) is maximum at 𝜃= x̄= 𝜃ˆ

Here 𝛺0 = {𝜃0 } and 𝛺={𝜃0 : 𝜃𝜖ℝ} Now, 𝜃 sup ≥ 𝜃0 L (x̰ ; θ) =


𝜃𝑠𝑢𝑝 𝜖𝛺0 L (x̰ ; θ)
L (x̰ ; θˆ), if x > 𝜃0
The likelihood ratio is 𝜆 = {
𝜃𝑠𝑢𝑝𝜖 ΩL (x̰ ; θ) L (x̰ ; 𝜃0 ) , if 𝜃0 ≥ x
𝑛
L (x̰ ,𝜃 ) − (x−𝜃0 )2 ,𝑖𝑓𝜃0 <x
= 𝜃𝑠𝑢𝑝𝜖 RL 0(x̰ ; θ) Here, 𝜆= {𝑒 2𝜎2

1 , 𝑖𝑓 𝜃0 ≥ x
1
1 − ∑𝑛 (𝑥 −𝜃0 )2
( )𝑛 .𝑒 2𝜎2 𝑖=1 𝑖 Note that, 𝜆 <c (< 1)
𝜎√2𝜋
= 1
∑𝑛 (𝑥 −x)2
1 −
( )𝑛 .𝑒 2𝜎2 𝑖=1 𝑖 𝑛
𝜎√2𝜋 − (x−𝜃0 )2 <𝑐
⇒ 𝑒 2𝜎2 , where 𝜃0 < x
1
− 2 {∑(𝑥𝑖 −𝜃0 )2 −∑(𝑥𝑖 −x)2 }
=𝑒 2𝜎 𝑛 (x−𝜃0 )
⇒ > 𝑐1 , where x>𝜃0
𝜎
1
− 2 .𝑛 (x−𝜃0 )2
=𝑒 2𝜎
√𝑛 (x−𝜃0 )
⇒ > k , as (x − 𝜃0 )> 0
𝜎
Note that 𝜆< c
The size 𝛼 LRT is given by: Reject 𝐻0 iff 𝜆
𝑛
− (x−𝜃0 )2 <𝑐 <c
⇒𝑒 2𝜎2

𝑛(x−𝜃0 )2 √𝑛 (x−𝜃0 )
⇒ > 𝑐1 iff > k , where k is such that
𝜎2 𝜎

√𝑛 (x−𝜃0 ) √𝑛 (x−𝜃0 )
⇒| |> k, say. 𝛼 = 𝑃𝐻0 [ > 𝑘]
𝜎 𝜎

The size 𝛼 LRT is given by: = 𝑃 [𝑧 > 𝑘], z ∼N (0, 1)

√𝑛 (x−𝜃0 ) The size LRT is given by: Reject 𝐻0 iff


Reject 𝐻0 iff 𝜆> c iff | |> k, where k 𝜎
𝜎
x > 𝜃0 + 𝑛 𝛼,
is such that √

√𝑛 (x−𝜃0 ) Which is the UMP test for testing 𝐻0 : θ =


𝛼 = 𝑃𝐻0 [| | > 𝑘] = 𝑃 [|𝑧| > 𝑘], z
𝜎 𝜃0 𝑎𝑔𝑎𝑖𝑛𝑠𝑡 𝐻1 : θ > 𝜃0 and is unbiased.
∼N (0, 1)
√𝑛 (x−𝜃0 ) α
𝐻1 : θ ≠ 𝜃0 is 𝛽(𝜃)= 𝑃𝐻0 [| | > 2]
⇒ k = 𝛼/2 𝜎

√𝑛 (x−𝜃0 ) α √𝑛 (𝜃0 −𝜃) α


(II) to test 𝐻0 :𝜃= 𝜃0 𝑎𝑔𝑎𝑖𝑛𝑠𝑡 𝐻1: 𝜃> 𝜃0 :- = 1- ⏀ ( + )+ ⏀( − )
𝜎 2 𝜎 2

Here 𝛺0 = {𝜃0 } and 𝛺= {𝜃0 : 𝜃≥ 𝜃0 }

218
Solving Mathematical Problems

Note that, 𝛽′(𝜃)= ⏀(


√𝑛 (𝜃0 −𝜃) α √𝑛
+ 2)( 𝜎 )- From graph, 𝜆 < c ⇒ t > 𝑘2 𝑜𝑟 𝑡 < 𝑘1
𝜎
√𝑛 (𝜃0 −𝜃) α √𝑛 The LRT of its size is given by;
⏀( − 2)× ( 𝜎 ) ; if 𝜃 > 𝜃0
𝜎
Reject 𝐻0 iff 𝜆 < c iff ∑𝑛𝑖=1 𝑥𝑖 < 𝑘1 or
> 0 𝑖𝑓 θ > 𝜃0
Now, 𝛽′(𝜃)={ ∑𝑛𝑖=1 𝑥𝑖 > 𝑘2 ∋ 𝜆 (𝑘1 )= 𝜆 (𝑘2 )
< 0 𝑖𝑓θ < 𝜃0

Clearly, 𝛽 (𝜃)> (𝜃0 ) ∀ 𝜃 ≠ 𝜃0 Q77. let X be a discrete random variable


with P [X= -1]= P and P [x= k]= (𝟏 −
⇒ power > size 𝒑)𝟐 𝑷𝒌 , where P 𝜖 (0, 1)is unknown. Show
that U(X)is an unbiased estimator of 𝜃 iff
i.e the LRT is unbiased
U (k)= ak , k = -1, 0, 1, 2, …. for some a.
Q76. Let 𝑿𝟏 , … , 𝑿𝒏 be a R.S. from B (1,
Solution:- E [U(x)] = ∑∞
𝑘= −1 𝑎𝑘 . 𝑃 [𝑋 = 𝑘]
p) population. Derive a LRT of its size of
𝑯𝟎 : 𝒑 = {𝒑𝒐}𝒂𝒈𝒂𝒊𝒏𝒔𝒕𝑯𝟏 : 𝒑 ≠ 𝒑𝒐 = -ap + ∑∞ 2 𝑘
𝑘=0 𝑘(1 − 𝑝) 𝑃

Solution: - Here 𝛺0 = {𝑝0 } 𝑎𝑛𝑑 𝛺 = = -ap + a(1 − 𝑝)2 [p+ 2𝑝2 + 3𝑝3 + … ∞]
{𝑝 ; 0 < 𝑝 < 1}
=-ap +a(1 − 𝑝)2 p(1 − 𝑝)2
The likelihood function is
= -ap+ ap
𝑝∑ 𝑥𝑖 (1 − 𝑝)𝑛− ∑ 𝑥𝑖 , 𝑖𝑓 𝑥𝑖 = 0, 1
L (x̰ ; p)= { 0 , 𝑜𝑤 = 0.

Now, ∑∞
𝑥= −1 𝑢 (𝑥)𝑓(𝑥) = 0
The LR is
⇨ u (-1) P [X= -1]+ ∑∞
𝑥=0 𝑢 (𝑥)𝑓(𝑥) = 0
𝑝𝑠𝑢𝑝 𝜖𝛺0 L (x̰ ;p) L (x̰ ;𝑝0 )
𝜆= =L , where
𝑝𝑠𝑢𝑝𝜖 ΩL (x̰ ; p) (x̰ ; pˆ) ⇨ u (-1) p + ∑∞ 2 𝑥
𝑥=0 𝑢 (𝑥)(1 − 𝑝) 𝑝 = 0

⇨ p (u (-1)) + (1 − 𝑝)2 ∑∞ 𝑥
𝑥=0 𝑢 (𝑥) 𝑝 =0

𝑝0 𝑡 (1−𝑝0 )𝑛−𝑡 pˆ −𝑝𝑢(−1)


𝜆= , 𝑡 = ∑ 𝑥𝑖 ⇨ ∑∞ 𝑥
𝑥=0 𝑢 (𝑥) 𝑝 =
pˆt (1−pˆ)n−t 𝑞2

𝑛𝑝0 𝑡 𝑛 (1−𝑝0 ) 𝑛−𝑡


=( ) ( 𝑛−𝑡 ) ⇨ ∑∞ 𝑥
𝑥=0 𝑢𝑥 𝑝 = -u (-1) p (1 − 𝑝)
2
𝑡

𝑛𝑝0 𝑛 (1−𝑝0 ) = -u (-1) p (1+ 2p +3𝑝2 + ⋯)


Now, ln 𝜆= tln( )+ (n-t)ln { }
𝑡 𝑛−𝑡
= -u (-1) [p+2𝑝2 + 3𝑝3 + ⋯]
𝑑 𝑛𝑝0 𝑛 (1−𝑝0 )
And 𝑑𝑡 𝑙n𝜆 = ln( )- ln { }
𝑡 𝑛−𝑡
= -u (-1) ∑∞
𝑥=0 𝑥 𝑝
𝑥
[comparing power
𝑛𝑝0 𝑛−𝑡 series from both side]
= | n( . 𝑛 (1−𝑝 )𝑡)
𝑡 0
⇨ u (x) = -u (-1)x
> 0 , 𝑖𝑓 𝑡 < 𝑛𝑝0
={
< 0 , 𝑖𝑓 𝑡 > 𝑛𝑝0 ⇨ u(x) = ax.
219
Solving Mathematical Problems
−(𝒙− 𝜽) 𝒊𝒇 𝒙 ≥𝜽 ⇨𝑃
Q78. f(x) = {𝒆
𝛼
𝜒2 2 ;1−
𝜃[ 2 ≤𝑋 −𝜃 ≤𝑋 ]=1−𝛼
𝟎 𝒐𝒘 2𝑛 (1) (1)

Find (a) MLE of 𝜃 (b) 95% C. I. for 𝜃 𝜒2 2 ;1−


𝛼
2
⇨𝑃𝜃 [𝑋(1) − ≥ 𝜃 ≥ 𝑋(1) −
2𝑛
Solution:- (a) let 𝑋1 , … , 𝑋𝑛 be a R.S. from 𝛼
f(x). then, 𝜒 2 2 ; 2𝑛 ] = 1 − 𝛼

𝑓𝜃 (𝑥) = exp[− ∑𝑛𝑖=1(𝑥𝑖 − 𝜃)]. 𝐼 𝑋(1) ≥ 𝜃; ∴ Confidence interval for 𝜃 is


𝛼
= L(θ|x̰ ) 2 𝛼 𝜒2 2 ;1−
2
(𝑋(1) − 𝜒 2 ; 2𝑛 , 𝑋(1) − )
2𝑛
The likelihood function will be maximum
when ∑𝑖(𝑥𝑖 − 𝜃)is minimum Q79. suppose 𝑿𝟏 , 𝑿𝟐 , … , 𝑿𝒏 are i.i.d. N (𝜃,
1), 𝜽𝟎 ≤ 𝜽 ≤ 𝜽𝟏 , where 𝜽𝟎 < 𝜽𝟏 are two
i.e. , when 𝜃 is maximum. specified numbers. Find the MLE of 𝜃
and show that it is better than the sample
i.e. 𝜃ˆ𝑀𝐿𝐸 = 𝑋(1)
mean x̄ in the sense of having smaller
(b) CI will based on sufficient statistic mean squared error.

− ∑𝑖 𝑥𝑖 +𝑛𝜃 ; 𝑋(1) ≥𝜃 Solution:- L = ∏𝑛𝑖=1 𝑓 (𝑥𝑖 , 𝜃) =


𝑓𝜃 (x̰ )= {𝑒 1 1
∑(𝑥𝑖 −𝜃)2
0 ; 𝑜𝑤 . 𝑒 −2 ; 𝑥𝑖 𝜖 ℝ
(√2𝜋)𝑛

= 𝑒 − ∑𝑖 𝑥𝑖 +𝑛𝜃 ⏀ (𝜃, 𝑋(1) ) ; where


Here we wish to maximize L w.r.t. 𝜃 subject
1 𝑖𝑓 𝜃 ≤ 𝑖
⏀ (𝜃, 𝑋(1) )= { to the condition𝜃0 ≤ 𝜃 ≤ 𝜃1 ; L will be
0 𝑖𝑓 𝑜𝑤 1 2
∑(𝑥
maximum iff 𝑒 2 𝑖 −𝜃) is minimum. Iff
∴𝑓𝜃 (x̰ ) = 𝑔𝜃 (𝑡). ℎ (x̰ ) ∑(𝑥𝑖 − 𝜃)2 is minimum at 𝜃ˆ𝑀𝐿𝐸 = 𝜃0
Where, 𝑔𝜃 (𝑡) = 𝑒 𝑛𝜃 . ⏀ (𝜃, 𝑡) and ℎ (x̰ ) = For x̄,
𝑒 − ∑ 𝑥𝑖
1
lnL = c-2 ∑(𝑥𝑖 − 𝜃)2
∴ by NF Factorization theorem 𝑋(1) is
sufficient. 𝜕|𝑛𝐿
= ∑(𝑥𝑖 − 𝜃) = 0
𝜕𝜃
Now, the PDF of 𝑋(1) is given by 𝑓𝑋(1) (𝑦) =
⇨ ∑ 𝑥𝑖 = 𝑛𝜃
𝑛𝑒 −𝑛 (𝑦−𝜃) 𝐼 𝑦 ≥ 𝜃
⇨𝜃ˆ𝑀𝐿𝐸 = x ; where 𝜃0 ≤ x ≤ 𝜃1 as 𝜃0 ≤
Note that, it is a shifted exponential
distribution, 𝜃 ≤ 𝜃1

∴ 2n (𝑋(1) − 𝜃) ∼ 𝜒 2 2 E (𝑋 − 𝜃0 )2 = 𝐸 [𝑋 − x) + (𝑋 − 𝜃0 )]2

= E (𝑋 − x)2 + E (𝑋 − 𝜃0 )2
∴ 𝑝𝜃 [𝜒2
𝛼 2 𝛼
2 ;1− 2 ≤2𝑛(𝑋(1) −𝜃)≤𝜒 2 ;2𝑛]=1−𝛼
⇨ E (𝑋 − 𝜃0 )2 ≤ E (𝑋 − x)2.
220
Solving Mathematical Problems

Q80. if T: ℝ 𝟑 ⟶ ℝ 𝟑 be a linear 𝑉 (T(𝑋1 ))


𝑉 (𝑠𝑛 ) ≤ 𝑉 (𝑉𝑛 ) −
transformation defined by 𝑛2
𝑉 (T(𝑋1 ))
T (x, y, z) = (x+y+z, y+z, z) then find 𝑻𝒏 (x, = 𝑛
y, z).
⇨ lt 𝑉 (𝑠𝑛 ) = 0 .
Solution: - 𝑇 2 (x, y, z) = T (x + y+ z, y+z, z) 𝑛⟶∞

= (x+ 2y + 3z, y+ 2z, z)


Q82. Let f (0)=0 then show that
𝑇 3 (x, y, z) = T (x + 2y + 3z, y + 2z, z) 𝒇 (𝒉)+ 𝒇 (−𝒉)
𝐥𝐢𝐦 = 𝒇″ (𝟎)
𝒉⟶𝟎 𝒉𝟐
= (x+ 3y + 6z, y+ 3z, z)
ℎ2
⦙ Solution:- f (h)= f(0)+hf′(0)+ 2! 𝑓 ″ (0)+
ℎ3
𝑛 𝑛 (𝑛+1) 𝑓 ‴ (𝑐) [∵ f(0)= 0]
𝑇 (x, y, z) = (x+ ny + . 𝑧, 𝑦 + 𝑛𝑧, 𝑧). 3!
2
ℎ2
Q81. Let 𝑿𝟏 , 𝑿𝟐 , … , 𝑿𝒏 be i.i.d random Similarly, f(-h)= - hf′(0)+ 𝑓 ″ (0) −
2!
variables with family f(𝜃 ; x ; x 𝝐 ℝ , 𝜃 𝜖 ℎ3
𝑓 ‴ (𝑐)
3!
(0, 1) be the unknown parameter.
Suppose that there exit an unbiased f (h)+ f (-h)= ℎ2 𝑓 ″ (0)
estimator T of 𝜃 based on sample size one,
𝑓 (ℎ)+ 𝑓 (−ℎ)
i.e. ∴ 𝑓 ″ (0) = lt .
ℎ⟶0 ℎ2

E [T (𝑿𝟏 )] = 𝜃. Assume that V (T (𝑿𝟏 )) <


∞.

(I) find the estimator 𝑽𝒏 for 𝜃, E (𝑿𝒏 ) ∍


𝑽𝒏 is constant. Q83. Suppose x has a normal distribution
with mean 0 and variance 25.
(II) Let 𝒔𝒏 be the MVUE of 𝜃 based on
𝑿𝟏 , 𝑿𝟐 , … , 𝑿𝒏 s.t. 𝐥𝐭 𝑽 (𝒔𝒏 ) ⟶ 𝟎 Let Y be an independent RV taking
𝒏⟶∞
values -1 and +1 with equal probability ½
Solution:- (I) 𝑋𝑖 ∼ 𝑓𝜃 (x) .define

Y = T (𝑋1) ∼𝑓𝜃 (t) 𝑿 𝒙


S= X y + 𝒚 , T = xy -𝒚.
Now, Y̅ ⟶ E (𝑌1 )
(I) Find the probability
1 distribution of S.
i.e. 𝑉𝑛 = 𝑛 ∑𝑛𝑖=1 T (𝑋𝑖 ) ⟶ E [T(𝑋1 ) = θ ]
(II) Find the probability
𝑺+𝑻 𝟐
(II) E (Y̅) = 0 distribution of ( )
𝟏𝟎

Solution:- (i) 𝐹𝑆 (𝑠) = 𝑃 [𝑆 ≤ 𝑠]

221
Solving Mathematical Problems

= P [S≤ 𝑠 | 𝑌 = −1]𝑃 [𝑌 = −1] + 𝑃 [𝑆 ≤ ⇨ lim 𝑃 [𝑆𝑛 ≤ 𝑛 + √2𝑛 ] = ⏀(1)


𝑛⟶∞
𝑠|𝑌 = 1] 𝑃 [𝑌 = 1]
1 1 ⇨ lim 𝑃 [𝑆𝑛 > 𝑛 + √2𝑛 ] = 1 − ⏀(1)
= 2 𝑃 [−2𝑋 ≤ 𝑠] + 2 𝑃 [−2𝑥 ≤ 𝑠] 𝑛⟶∞

1 ∞ 𝑡 𝑛⁄
𝑠
=P [x ≤ 2], since ‘X’ is symmetrically ⇨ lim { 1 ∫𝑛+√2 𝑛 𝑒 − ⁄2 𝑡 2−1 𝑑𝑡} =
𝑛⟶∞ 2𝑛⁄2⎾(2)

distributed about ‘0’, ⏀(−1)


𝑥−0 𝑠−0 Q85. let𝒀𝟏 , 𝒀𝟐 , 𝒀𝟑 , 𝒀𝟒 has four
=P[ ≤ ]
𝑆 10
uncorrelated r.v.s with E (𝒀𝒊 ) = i𝜃,
𝑠
= ⏀(10)
V (𝒀𝒊 ) = 𝒊𝟐 𝝈𝟐 , i= 𝟏(𝟏)𝟒 , where 𝜃, 𝜎> 0 are
∴ S ∼ N (0, 102 ). unknown parameter

(Ii) s + T =2 xy Find the values of 𝒄𝟏 , 𝒄𝟐 , 𝒄𝟑 , 𝒄𝟒 for which


∑𝟒𝒊=𝟏 𝒄𝒊 𝒀𝒊 is unbiased for and has least
(𝑆 + 𝑇)2 = 4 𝑥 2 𝑦 2 = 4 𝑥 2 , since P [𝑌 2 = variance.
1]=1
Solution:- E (∑4𝑖=1 𝑐𝑖 𝑌𝑖 )= (∑4𝑖=1 𝑖𝜃 𝑐𝑖 )𝜃,
𝑋
⇨ (𝑆 + 𝑇)2 = ( 5 )2 ∼ 𝜒 2 . ∑ 𝑐𝑖 = 1

Q84. using an appropriate probability Again, V (∑𝑖 𝑐𝑖 𝑌𝑖 )= ∑ 𝑐𝑖 2 𝑖 2 𝜎 2 =𝜎 2 =


distribution or otherwise find the value of ∑ 𝑖 2 𝑐𝑖 2
𝟏 ∞ 𝒕 𝒏⁄
𝐥𝐢𝐦 𝒏 𝒏 . ∫𝒏+√𝟐 𝒏 𝒆− ⁄𝟐 𝒕 𝟐−𝟏 𝒅𝒕 Now, ∑ 𝑐𝑖 𝑌𝑖 is unbiased.
𝒏⟶∞ 𝟐 ⁄𝟐 ⎾( )
𝟐

Solution:- Let {𝑥𝑛 } be a sequence of i.i.d. So, 1 = (∑𝑖 𝑖𝑐𝑖 )2 ≤ (∑𝑖 𝑖 2 𝑐𝑖 2 )(∑𝑖 1) , by
1 1 C-S inequality.
Random variables following Gamma (2 , 2)
1 1
𝑥 1
‘=’ holds when 𝑖𝑐𝑖 = k= 4 ⇨𝑐𝑖 =4𝑖.
𝑒 − ⁄2 .𝑥 ⁄2−1
1⁄ . 1 𝑖𝑓 𝑥 > 0
Here , f (x)= { 2 2 ⎾( )
2 Alternative way:-
0 𝑜𝑤
𝑋𝑖 ∼ (𝜃, 𝜎 2 )
1 𝑛
Then ∑𝑛𝑘=1 𝑋𝑘 = 𝑆𝑛 ∼ 𝐺𝑎𝑚𝑚𝑎(2 , 2 )

E (𝑆𝑛 )= n, V(𝑆𝑛 )= 2n. by lindeberg- lavy 𝑌𝑖 ∼ (𝑖𝜃 , 𝑖 2 𝜎 2 )


central Limit theorem
𝑌𝑖
𝑆𝑛 − 𝐸 (𝑆𝑛 ) ∼ (𝜃, 𝜎 2 )
lim 𝑃 [ ≤ 𝑥] = ⏀ (𝑥) 𝑖
𝑛⟶∞ √V(𝑆𝑛 )
𝑆𝑛 −𝑛 𝐵𝐿𝑈𝐸
⇨ lim 𝑃 [
𝑛⟶∞ √2𝑛
≤ 1] = ⏀(1) Y̅ → 𝜃

222
Solving Mathematical Problems
1 𝑌𝑖 𝜽𝒂𝜽
∑ is BWE for 𝜃 Q87. If f(x) = 𝒙𝜽+𝟏 𝑰 𝒂 < 𝑥 < ∞, 𝜃 > 0, 𝑎 >
4 𝑖

1 0;
∑ 𝑐𝑖 𝑌𝑖 is BWE for 𝜃, where 𝑐𝑖 =
4𝑖
(a) Find UMVUE of 𝜃, when a is known,
Q86. Let 𝑿𝟏 , 𝑿𝟐 , … , 𝑿𝒏 be independently
distributed random variables with (b) Find UMVUE of a, when 𝜃 is known
densities
Solution: - (a) from OPEF, the complete
𝒆𝒊𝜽 −𝒙𝒊 ,𝒊𝒇 𝒙𝒊 ≥𝒊𝜽 sufficient statistic is ∑𝑛𝑖=1 |𝑛𝑋𝑖 and
f (𝒙𝒊 ; 𝜽)= { [Here 𝑋𝑖
𝟎 , 𝒐𝒘 hence∑𝑛𝑖=1 |𝑛 will also be complete
𝑎
𝒙′𝒊 𝒔𝒂𝒓𝒆 𝒏𝒐𝒕 𝒓𝒂𝒏𝒅𝒐𝒎 𝒔𝒂𝒎𝒑𝒍𝒆𝒔] sufficient statistic.
Find a one –dimensional sufficient 𝑋𝑖
Now, |𝑛 ∼ Exp (𝜃)
statistic for𝜃. 𝑎

𝑋𝑖
Solution:- the joint PDF of 𝑋1 , 𝑋2 , … , 𝑋𝑛 is ∴ 2𝜃 ∑𝑛𝑖=1 |𝑛 ∼ 𝜒2𝑛 2
𝑎
𝑛 1 1
⇨E[ 𝑋𝑖 ] = (2𝑛−2) [If X ∼𝜒𝑛 2 E
∏ 𝑓 (𝑥𝑖 ; 𝜃) 2θ ∑𝑛
𝑖=1 |𝑛 𝑎
𝑖=1 1 1
(𝑋) =𝑛−2 ]
𝜃 ∑𝑛 𝑛
𝑖=1 𝑥𝑖 −∑𝑖=1 𝑥𝑖
= {𝑒 ; 𝑖𝑓𝑥𝑖 ≥ 𝑖𝜃 , ∀ 𝑖 = 1(1)𝑛
0 , 𝑜𝑤 𝑛−1
⇨E[ 𝑋 ]= 𝜃
∑𝑖 |𝑛 𝑖
𝑛(𝑛+1)𝜃 𝑎
−∑𝑛
𝑖=1 𝑥𝑖
𝑥𝑖
= {𝑒 ; 𝑖𝑓 ≥ 𝜃∀ 𝑖 = 1(1)𝑛
2
𝑖

0 ; 𝑜𝑤
𝑛(𝑛+1)𝜃
−∑𝑛 𝑥
Function of complete sufficient statistic
𝑖=1 𝑥𝑖 ; 𝑖𝑓 min{ 𝑖𝑖 } ≥ 𝜃
= {𝑒
2
𝑖 𝑛−1
0 ; 𝑜𝑤 ∴ 𝑋 is the required UMVUE.
∑𝑖 |𝑛 𝑖
𝑎
𝑛(𝑛+1)𝜃
−∑𝑛
𝑖=1 𝑥𝑖
𝑥
=𝑒 2 . 𝐼 (𝜃, min{ 𝑖𝑖 }); where 𝜃𝑛 𝑎𝑛𝜃
(b) 𝑓𝑎 (𝑥̰) = 𝜃+1 . ⏀(𝑎, 𝑥); where
1 𝑖𝑓 𝑎 ≤ 𝑏 (∏𝑛
𝑖=1 𝑋𝑖 )
I(a,b)= { 1 , 𝑖𝑓 𝑎 < 𝑥
0 𝑖𝑓 𝑎 > 𝑏 ⏀(𝑎, 𝑥) = {
0, 𝑜𝑤
𝑛(𝑛+1)𝜃
𝑥 𝑛
=𝑒 2 . 𝐼 (𝜃, min{ 𝑖 }). 𝑒 − ∑𝑖=1 𝑥𝑖
𝑖 ∴ 𝑔𝑎 (𝑡) = 𝑎𝑛𝜃 . ⏀(𝑎, 𝑡) With T = 𝑋(1)
= g (T (x̰) ;𝜃). h(x̰), where h(x̰)= 𝑒 − ∑ 𝑥𝑖 ∴ 𝑋(1) is the sufficient statistic for 𝜃.
𝑥
so, T (x̰) = min{ 𝑖𝑖 } is sufficient for 𝜃, by Now, 𝑓𝑋(1) (𝑡) = 𝑃 [𝑋(1) ≤ 𝑡]= 1-P [𝑋(1) >
𝑖
NFFT. 𝑡]= 1- (𝑃[𝑋 > 𝑡])𝑛

∞ 𝑎𝜃
Where, P [X > t]= ∫𝑡 𝜃𝑎𝜃 𝑥 −𝜃−1 𝑑𝑥 = 𝑡𝜃

223
Solving Mathematical Problems

𝑎 𝑛𝜃−1 𝑎 ∴ E [T (𝑋(𝑛) )]=0


∴ 𝑓𝑋(1) (𝑡) = −𝑛𝜃 ( 𝑡 ) . (− 𝑡 2 )
∞ 𝑛𝑥(𝑛)𝑛−1
𝑎𝑛𝜃 ⇨ ∫0 𝑡(𝑥(𝑛) ) . 𝑑𝑥(𝑛) = 0
= n . 𝑡 𝑛𝜃+1 𝐼𝑎 > 0 𝜃𝑛

𝜃
⇨ ∫0 𝑡(𝑥(𝑛) ). 𝑥(𝑛)𝑛−1 𝑑𝑥(𝑛) = 0⇨ G (𝜃)-
Now, if (t) is any arbitrary function of t,
then E [𝜓(t)]=0 G(0)=0

∞ 𝑎𝑛𝜃 ⇨ g (𝜃)= 0 ⇨ u (𝑥(𝑛) ). 𝑥(𝑛)𝑛−1=0 ⇨ u


⇨ ∫𝑎 𝑛𝜃 𝑡 𝑛𝜃+1 𝜓(𝑡)𝑑𝑡 =0
(𝑥(𝑛) )=0
∞ 𝜓(𝑡)
⇨ ∫𝑎 𝑑𝑡 = 0 ∴ 𝑋(𝑛) is complete sufficient for 0.
𝑡 𝑛𝜃+1

g (t), say To find UE of 𝜃 based on𝑋(𝑛) , let us


consider the function,
⇨ G (∞)-G (a)=0
𝑎 𝑖𝑓 𝑋(𝑛) < 1
⇨ g (∞)-0 - g (a)-1=0 h (ˆ𝑋(𝑛) )= {
𝑏𝑋(𝑛) 𝑖𝑓 𝑋(𝑛) ≥ 1
⇨ g (a)=0 ⇨ 𝜓(a)=0
∴ E [h (𝑋(𝑛) )] = 𝜃 [∵ h (𝑋(𝑛) ) is UE of 𝜃]
∴ 𝑋(1) is complete sufficient for 𝜃
𝑛𝑥(𝑛)𝑛−1
⇨ a. P [𝑋(𝑛) ≤ 1]+ b 𝑥(𝑛). 𝑑𝑥(𝑛) =𝜃
Let 𝑋(1) = 𝑇 𝜃𝑛

1 𝑛𝑥(𝑛)𝑛−1
∞ 𝑎𝑛𝜃 ⇨ a ∫0 𝑑𝑥(𝑛) +
Now, E (T)= ∫𝑎 𝑡. 𝑛𝜃 𝑡 𝑛𝜃+1 𝑑𝑡 𝜃𝑛
𝜃 𝑥(𝑛)
𝑏 ∫1 𝑛. 𝑑𝑥(𝑛) = 𝜃
𝜃𝑛
𝑛𝜃−𝑎
=𝑛𝜃+1
𝑛 1 𝑏.𝑛
⇨ a. 𝜃𝑛 . 𝑛 + (𝑛+1)𝜃𝑛 = 𝜃
𝑛𝜃−1
∴ E [t. ]= a
𝑛𝜃 𝑏𝑛
⇨ a+ 𝑛+1 (𝜃 𝑛+1 − 1) = 𝜃 𝑛+1
𝑛𝜃−1 1
∴ UMVUE of a is = (1 − 𝑛𝜃).
𝑛𝜃 𝑏𝑛 𝑏𝑛
⇨ a+ 𝑛+1 𝜃 𝑛+1 -𝑛+1= 𝜃 𝑛+1
Q88. let 𝑿𝟏 , 𝑿𝟐 , … , 𝑿𝒏 ∼ Rec (0, 𝜃) with an
𝑏𝑛 𝑏𝑛
unknown 𝜃 (1, ∞) ∴ 𝑛+1 = 1, 𝑎 = 1, 𝑛+1 = 1. [Equating

Suppose we only observe 𝒛𝒊 = coefficients of 𝜃]


𝑿 𝒊𝒇𝑿𝒊 ≥ 𝟏 1 𝑖𝑓𝑋(𝑛) < 1
{ 𝒊
𝟏 𝒊𝒇 𝑿𝒊 < 1 ∴ h (X) = { 𝑛+1 is UMVUE of
𝑋(𝑛) ≥ 1
𝑛
Derive UMVUE of 𝜃. 𝜃.
Solution:- let T (𝑋(𝑛) ) be an unbiased Q89. Let 𝑿𝟏 , 𝑿𝟐 , … , 𝑿𝒏 be a random
estimator of 0. sample from a distribution having pdf

224
Solving Mathematical Problems
𝜶𝒙𝟎 2𝑛
𝒇𝒐𝒓 𝒙 > 𝒙𝟎 , ( ) 𝑃𝑘 (1 − 𝑃)2𝑛−𝑘
f(x; 𝒙𝟎 , 𝜶)= {𝒙𝜶+𝟏
𝟎 𝒐𝒘 𝑘

Where, 𝒙𝟎 , > 0, 𝛼> 0. Find the maximum In our case, if we want the number of heads
likelihood estimator of 𝛼 if 𝒙𝟎 is known. to be the same as number of tails then k=n
and if we are tossing a fair coin then P =1⁄2.
Solution: - Likelihood function, L (x, 𝛼) is Hence, we get
given by,
P (No. of Heads=No. of Tails)
𝛼𝑛 𝑥0 𝛼𝑛
L (x, 𝛼) = (𝛼+1) 𝑛 𝑛
𝑥1 (𝛼+1) 𝑥2 (𝛼+1)…𝑥𝑛 1
= (2𝑛
𝑛
) (12) (12) = 22𝑛 (2𝑛
𝑛
)
⇨ log 𝐿 = 𝑛 log 𝛼 + 𝛼𝑛 log 𝑥0 − (𝛼 +
Hence, we get that P (No. of Heads > No. of
1) ∑ 𝑥𝑖
Tails)= P (No. of Heads < No. of Tails)
1 𝜕𝐿 𝑛 𝜕𝐿
⇨ 𝐿 . 𝜕𝛼= 𝛼 + 𝑛 log 𝑥0 − ∑ 𝑥𝑖 𝜕𝛼 = 0 (2𝑛
𝑛)
1− 2𝑛
2 1 𝑛 (2𝑛)
𝑛 = = 2 − 22𝑛+1 .
2
⇨ 𝛼+n|n𝑥0 = ∑ 𝑥𝑖

∑ 𝑥𝑖
⇨ 𝛼 −1 = − |n𝑥0
𝑛
Q91. Suppose 𝒂𝒏 ≥ 𝟎 𝒂𝒏𝒅 ∑ 𝒂𝒏 is
1 convergent. Show that
⇨ 𝛼= ∑ 𝑥𝑖
−|n𝑥0
𝑛
𝟏
∑ 𝒊𝒔 𝒅𝒊𝒗𝒆𝒓𝒈𝒆𝒏𝒕.
Thus MLE 𝛼 is given by 𝛼= ∑ 𝑥𝑖
1 𝒏𝟐 𝒂𝒏
−|n𝑥0
𝑛
Solution: Using CS Identity
Q90. A fair coin is flipped 2n times. Find
2
the probability that it comes up heads 2 2 2 1
[(√𝑎1 ) + (√𝑎2 ) + ⋯ + (√𝑎𝑛 ) ] [( )
more often than it comes up tail. 1. √𝑎1
2
1 2 1
Solution: - P (No. of Heads > No. of Tails)+ +( ) + ⋯+ ( ) ]
2√𝑎2 𝑛√𝑎𝑛
P (No. of Heads= No. of Tails)+ P (No. of
1 1 1 1 2
Heads < No. of Heads < No. of Tails)=1. ≥ [ + + +⋯+ ]
1 2 3 𝑛
Assuming you are tossing a fair coin, by
RHS is divergent and ∑ 𝑎𝑛 is convergent. Hence
symmetry, we also have that
1
P (No. of Heads > No. of Tails) ∑ 𝑖𝑠 𝑑𝑖𝑣𝑒𝑟𝑔𝑒𝑛𝑡
𝑛2 𝑎𝑛
= P (No. of Heads < No. of Tails)

If we want to get k heads in 2n tosses, where Q92. Let 𝑭 ∶ ℝ𝒏 → ℝ be defined by


the probability of getting a head is P then the
probability is. 𝑭(𝒙𝟏 , 𝒙𝟐 , … , 𝒙𝒏 ) = 𝐦𝐚𝐱{|𝒙𝟏 |, |𝒙𝟐 |, … , |𝒙𝒏 |}

225
Solving Mathematical Problems

Show that F is a uniformly continuous function. 1 1

𝐻𝑒𝑛𝑐𝑒 ∫ 𝑥 𝑓(𝑥)𝑑𝑥 = 𝜇 ∫ 𝑥 𝑛 𝑑𝑥
𝑛
Solution: Take 0 0

𝑋1 = (𝑋11 , 𝑋12 , 𝑋13 , … , 𝑋𝑛 ) , 𝑋2 1


𝜇
= (𝑋21 , 𝑋22 , … , 𝑋2𝑛 ) ⇒ ∫ 𝑥 𝑛 𝑓(𝑥) 𝑑𝑥 =
𝑛+1
0
|𝐹(𝑋11 , 𝑋12 , … , 𝑋𝑛 ) − 𝐹(𝑋21 , 𝑋22 , … , 𝑋2𝑛 )| =
1
|𝑋 |, |𝑋22 |, … , 𝜇
max{|𝑋11 |, |𝑋12 |, … , |𝑋1𝑛 |} – max { 21 } ⇒ lim ∫ 𝑥 𝑛 𝑓(𝑥)𝑑𝑥 = lim =0
|𝑋2𝑛 |
𝑛→∞ 𝑛→∞ 𝑛 + 1
0
≤ |max{|𝑋11 − 𝑋21 |, |𝑋12 − 𝑋22 |, … , |𝑋1𝑛
− 𝑋2𝑛 |}| Q94. (a) Suppose the series

(𝑋 − 𝑋21 )2 + (𝑋12 − 𝑋22 )2 + ⋯ ∑ 𝒂𝒏
≤ √ 11
+(𝑋1𝑛 − 𝑋2𝑛 )2 𝒏=𝟏

is convergent (𝒂𝒏 ≥ 𝟎)
≤ |𝑋1 − 𝑋2 |

So, for any given ∈> 0, 𝑐ℎ𝑜𝑜𝑠𝑒 𝛿 =∈ Is it true that


, 𝑡ℎ𝑒𝑛 |𝐹(𝑋1 ) − 𝐹(𝑋2 )| <∈ ∀ |𝑋1 − 𝑋2 | < 𝛿 ∞

∑ √𝒂𝒏 𝒂𝒏 − 𝟏 𝒂𝒍𝒔𝒐 𝒄𝒐𝒏𝒗𝒆𝒓𝒈𝒆𝒏𝒕?


𝒏=𝟏

Q93. Let f be a continuous function on [0, 1]. (b) Is the converse of the statement (a) true?
Evaluate
Solution: (a) AM ≥ GM gives
𝟏

𝐥𝐢𝐦 ∫ 𝒙𝒏 𝒇(𝒙) 𝒅𝒙 𝑎𝑛 + 𝑎𝑛−1


𝒏→∞ √𝑎𝑛 𝑎𝑛−1 ≤
𝟎 2

Solution: Applying Mean Value theorem for If


integrals ∞

∑ 𝑎𝑛
𝑓 ∶ [0, 1] → ℝ 𝑎𝑛𝑑 𝑔 ∶ [0, 1] → ℝ
𝑛=1

be two integrable functions and g(X) has the is convergent then


same sign on [0, 1]

𝑏 𝑏
𝑎𝑛 + 𝑎𝑛−1

2
𝑇ℎ𝑒𝑛 ∫ 𝑓(𝑥) 𝑔(𝑥)𝑑𝑥 = 𝜇 ∫ 𝑔(𝑥)𝑑𝑥 𝑛=1
𝑎 𝑎
is also convergent.
Take 𝑔(𝑥) =
𝑥 𝑛 ; 𝑥 𝑛 𝑑𝑜𝑒𝑠 𝑛𝑜𝑡 𝑐ℎ𝑎𝑛𝑔𝑒 𝑠𝑖𝑔𝑛 𝑜𝑛 [0, 1] Therefore √𝑎𝑛 𝑎𝑛−1 is convergent.

(b) Take 𝑎𝑛 = 𝑛2 (for odd n); 𝑎𝑛 = 𝑛−102 (for


even n). Then

226
Solving Mathematical Problems

𝑆 → choose rationals as intermediate points.
∑ 𝑎𝑛
𝑛=1 𝑆 ′ → choose irrationals as intermediate points.

is divergent but We know that



lim 𝑆 → 𝐵 𝑎𝑛𝑑 lim 𝑆′ → 𝐵
∑ √𝑎𝑛 𝑎𝑛−1 𝑖𝑠 𝑐𝑜𝑛𝑣𝑒𝑟𝑔𝑒𝑛𝑡 ‖𝑃‖ ‖𝑃‖

𝑛=1
As ‘f’ is Riemann integrable
(By p−series)
⇒ lim 𝑆 − 𝑆 ′ → 0
‖𝑃‖→0

⇒ lim 𝑢(𝑥)(𝑏 − 𝑎) − 𝑣(𝑥)(𝑏 − 𝑎) → 0


Q95. Evaluate the value of ‖𝑃‖→0

𝒂 ⇒ lim 𝑢(𝑥) − 𝑣(𝑥) → 0


‖𝑃‖→0
𝒏𝒙
𝐥𝐢𝐦 ∫ 𝒆 𝒅𝒙
𝒏→∞
𝟎 ⇒ 𝑢(𝑥) = 𝑣(𝑥)

Solution:  If 𝑢(𝑥) = 𝑣(𝑥) 𝑓𝑜𝑟 𝑥 ∈ [𝑎, 𝑏] and given


𝑎
u is continuous function, f is integrable
𝑙𝑡 𝑙𝑡 𝑒 𝑛𝑥 𝑎 (To show).
𝐼= ∫ 𝑒 𝑛𝑥 𝑑𝑥 = [ ]
𝑛→∞ 𝑛→∞ 𝑛 0
0 If ‘f’ is Riemann integrable then it satisfies the
𝑙𝑡 1 𝑛𝑎 following property that for every partition ‘P’
= [𝑒 − 1] and choice of intermediate points 𝜉,
𝑛 → ∞𝑛
𝑙𝑡 lim 𝑆(𝑃, 𝑓) = 𝐵 ↝
= 𝑎𝑒 𝑛𝑎 ‖𝑃‖→0
𝑛→∞
𝑎 𝑟𝑒𝑎𝑙 𝑛𝑢𝑚𝑏𝑒𝑟, 𝑤ℎ𝑒𝑛 𝑆(𝑃, 𝑓, 𝜉) is the Riemann
∞ ,𝑎 > 0 sum.
𝐼={
0 ,𝑎 ≤ 0
Q97. Let f be a bounded twice differentiable
real valued function on ℝ ∋ 𝒇′′ (𝒙) ≥ 𝟎 for all
Q96. Let 𝒖, 𝒗 ∶ [𝒂, 𝒃] → ℝ be continuous. x. Show that f is a constant.
Define 𝒇: [𝒂, 𝒃] → ℝ by Solution: 𝑓 ′′ ≥ 0 ⇒ 𝑓 is convex
𝒖(𝒙) 𝒊𝒇 𝒙 𝒊𝒔 𝒓𝒂𝒕𝒊𝒐𝒏𝒂𝒍 𝑓(𝑥) ≥ 𝑓(𝑥0 ) + 𝑓 ′ (𝑥0 )(𝑥 − 𝑥0 )
𝒇(𝒙) = {
𝒗(𝒙) 𝒊𝒇 𝒙 𝒊𝒔 𝒊𝒓𝒓𝒂𝒕𝒊𝒐𝒏𝒂𝒍
Case I: 𝑓 ′ (𝑥0 ) > 0
Show that f is Riemann integrable on [a, b] if
and only if 𝒖(𝒙) = 𝒗(𝒙) 𝒇𝒐𝒓 𝒂𝒍𝒍 𝒙 ∈ [𝒂, 𝒃] ⟹ 𝑓(𝑥) → ∞ 𝑎𝑠 𝑥 → ∞

Solution: Now pick any partition ‘P’. ⟹ 𝑓(𝑥)𝑖𝑠 𝑐𝑜𝑛𝑠𝑡𝑎𝑛𝑡 [∵ 𝑓 𝑖𝑠 𝑏𝑜𝑢𝑛𝑑𝑒𝑑]

Construct two Riemann sums Case II : 𝑓 ′ (𝑥0 ) < 0

𝑆(𝑃, 𝑓, 𝜉) 𝑎𝑛𝑑 𝑆 ′ (𝑃, 𝑓, 𝜉 ′ ) ⇒ 𝑓(𝑥) → ∞ 𝑎𝑠 𝑥 → −∞

227
Solving Mathematical Problems

⇒ 𝑓(𝑥) 𝑖𝑠 𝑐𝑜𝑛𝑠𝑡𝑎𝑛𝑡 𝑎𝑠 𝑓 𝑖𝑠 𝑏𝑜𝑢𝑛𝑑𝑒𝑑 For rank(A) = 𝑟𝑎𝑛𝑘 (𝐴̅) ⇒ 𝑐 − 9𝑎 − 7𝑏 = 0 ⇒


9𝑎 + 7𝑏 = 𝑐 is the required condition.
Case III: 𝑓 ′ (𝑥0 ) = 0
For general solution:
𝑓 ′ (𝑥) = 0 ∀ 𝑥 ∈ ℝ
𝑥 + 3𝑦 − 2𝑧 = 𝑎
⇒ 𝑓 𝑖𝑠 𝑎 𝑐𝑜𝑛𝑠𝑡𝑎𝑛𝑡 }
−2𝑦 + 𝑧 = 𝑎 + 𝑏

Take 𝑧 = 𝑡,
Q98. Find conditions on a, b, c ∈ ℝ to ensure 𝑎+𝑏−𝑡 5𝑎 + 3𝑏 + 𝑡
that the following system is consistent, and in 𝑦 = −( ); 𝑥 =
2 2
that case, find the general solutions:

𝒙 + 𝟑𝒚 − 𝟐𝒛 = 𝒂
Q99. Prove or disprove:
−𝒙 − 𝟓𝒚 + 𝟑𝒛 = 𝒃
(a) ∃ a linear map 𝑻 ∶ ℝ𝟐 → ℝ𝟒 such that
𝟐𝒙 − 𝟖𝒚 + 𝟑𝒛 = 𝒄
Range (T) = {(𝒙𝟏 , 𝒙𝟐 , 𝒙𝟑 , 𝒙𝟒 ) ∈ ℝ𝟒 ∶ 𝒙𝟏 + 𝒙𝟐 +
Solution: 𝒙𝟑 + 𝒙𝟒 = 𝟎}
𝑥 𝑎
(b) ∃ a linear map 𝑻 ∶ ℝ𝟐 → ℝ𝟑 ∋
𝐴3×3 ( ) = (𝑏)
𝑦
𝑧 𝑐
Range (T) = { (𝒙𝟏 , 𝒙𝟐 , 𝒙𝟑 ) ∈ ℝ𝟑 ∶ 𝒙𝟏 + 𝒙𝟐 +
1 3 −2 𝒙𝟑 = 𝟎}
𝐴 = (−1 −5 3 ) 𝑅′
⏟ 2 ⇋ 𝑅1
2 −8 3 Solution: (a) NO
1 3 −2
+ 𝑅2 (0 −2 1 ) 𝑇 ∶ ℝ2 → ℝ 4
2 −8 3
1 3 −2 dim(𝑅 2 ) = dim(𝑅𝑎𝑛𝑔𝑒 (𝑇) + ker(𝑇))
𝑅′3 = 𝑅3 − 2𝑅1 (0
⏟ −2 1)
0 −14 7 ⇒2=3+𝑥

1 3 −2 ⇒ dim(𝑅𝑎𝑛𝑔𝑒 (𝑇)) = 3
𝑅′
⏟ 3 = 𝑅3 − 7𝑅2 (0 −2 1 )
0 0 0 (b)dim(𝑅𝑎𝑛𝑔𝑒 (𝑇)) = 3

So, Rank (A) = 2 dim(𝑅 2 ) = 2 + dim(ker 𝑇)


1 3 −2 𝑎 ⇒ dim(ker 𝑇) = 0,
̅
𝐴 = (−1 −5 3 |𝑏) ⏟
𝑠𝑜𝑚𝑒 𝑜𝑝𝑒𝑟𝑎𝑡𝑖𝑜𝑛𝑠
2 −8 3 𝑐 Yes, linear transformation is possible.
1 3 −2 𝑎
(0 −2 1 𝑎+𝑏 ) 𝑇 ∶ ℝ2 → ℝ 3
0 0 0 𝑐 − 9𝑎 − 7𝑏
𝑇(𝑥1 , 𝑥2 ) → (𝑥1 , 𝑥2 , −𝑥1 − 𝑥2 )

228
Solving Mathematical Problems

Each of this is easy to establish. Thus, when the


parameter is 1000 as in the problem at hand,
Q100. Consider a circle which is tangent to the
the maximum product must be 3332 × 22 .
y−axis at 0. Show that the slope at (x, y)
satisfies

𝒅𝒚 𝒚𝟐 − 𝒙𝟐 Q2. Each of the numbers 𝒂𝟏 , 𝒂𝟐 … , 𝒂𝒏 is 1 on -


=
𝒅𝒙 𝟐𝒙𝒚 1. We have S = 𝒂𝟏 𝒂𝟐 𝒂𝟑 𝒂𝟒 + 𝒂𝟐 𝒂𝟑 𝒂𝟒 𝒂𝟓 + ⋯ +
𝒂𝒏 𝒂𝟏 𝒂𝟐 𝒂𝟑 .Then show that 4/n.
Solution: General equation of such circle would
be, (𝑥 − 𝑟)2 + 𝑦 2 = 𝑟 2 … … … . (∗) Sol. This is a number theoretic problem, but it
can also be solved by invariance. If we no place
⇒ 𝑥 2 + 𝑦 2 − 2𝑥𝑟 = 0 … … … (1) any 𝑎𝑖 by - 𝑎𝑖 , Then s. does not change mod 4
Differentiating (*), 2(𝑥 − 𝑟) + 2𝑦𝑦 ′ = 0 since four cyclically adjacout terms change their
sing. Indeed, if two of those terms are positive
𝑟−𝑋 and two negative, nothing changes. It one or
𝑦′ = … … . (2)
2𝑦 three have the same sing, S changes by ±4.

Eliminating ‘r’ from (1) & (2) gives Finally, if all four are of the same sing, then S
changes by ±8.

𝑦2 − 𝑥2
𝑦 =
2𝑥𝑦 Initially we have S = 0 which implies S = 0 mods.
Now, step by step, we change each negative
sing into a positive sing. This does not change S
mod = 4. At the end, we still have S= 0 mod 4,
𝑛𝑖 .𝑠4
but also S = .
MODEL TEST PAPERS WITH 𝑛

SOLUTIONS

MODEL TEST PAPER 1 Q3. Draw the graph │y│≤ │x│≤ 1.

Sol. The graph of│y│≤ │x│≤1. will be

Q1. Find positive numbers n and 𝒂𝟏 , 𝒂𝟐 , … , 𝒂𝒏


such that 𝒂𝟏 + 𝒂𝟐 + ⋯ + 𝒂𝒏 =
𝟏𝟎𝟑 𝒂𝒏𝒅 𝒂𝟏 𝒂𝟐 … 𝒂𝒏 is as large as possible.

Sol.:- We are led to discover that in a max


product (i) no 𝑎𝑖 will >4 (ii) no 𝑎𝑖 will = 1, (iii) all
𝑎𝑖 ′𝑠 can be taken to be 2 or 3 (As 4 = 2 × 2 and
4 = 2+ 2), (iv) at most two 𝑎𝑖 ′𝑠 will equal 2 (As
2 × 2 × 2 < 3 × 3 and 2+ 2+ 2 = 3+ 3).

229
Solving Mathematical Problems
𝐴𝐼 𝐵𝐼 𝐶𝐼 (𝑎+𝑏)(𝑏+𝑐)(𝑐+𝑎)
f(a, b, c) = 𝐴𝐷 . 𝐵𝐸 . 𝐶𝐹 = (𝑎+𝑏+𝑐)3

8 𝑎+𝑏+𝑐 3 8
(𝑎+𝑏+𝑐)3
( 3 ) 𝑤ℎ𝑖𝑐ℎ 𝑖𝑠 27
.

This is the right side of the inequality chain. To


prove the left side, we use the triangle
inequality

(a+ b –c) (a +c –b) (b+ c –a ) >0 (2)

For a more economical evaluation, we


introduce the elementary symmetric functions.

= a+ b +c, v = ab + bc + ca, w = abc (3)

Putting (3) into (2), we get −𝜇3 + 4𝜇𝑣 − 8𝑤 >


0 (4)
1
Q4. In a triangle ABC the bisectors AD, BE, CF On the other band 4 < 𝑓(𝑎, 𝑏, 𝑐) (5)
𝟏 𝑰𝑨 𝑰𝑩 𝑰𝑪
meet at the point I. Show that 𝟒 < 𝑨𝑫 . 𝑩𝑬 . 𝑪𝑭 =
𝟖
Give −𝜇3 + 4𝜇𝑣 − 4𝑤 > 0 (6)
𝟐
.
Now (4) is obviously correct. Hence (6) is also
Sol. A bisector of a triangle divides the opposite correct. Here we profitably used the elementary
side in the ratio of the other two sides. symmetric functions. They are useful in cares
(𝑎𝑏) (𝑎𝑐)
when we are dealing with functions which are
Hence, p = CD =(𝑏+𝑐) , q = DB = (𝑏+𝑐). symmetric in their variables.

Thus we have Here is the simplest proof of (5) : set a = y+ z, b


𝑥 𝑦
= z+ x, c = x+ y. with r = 𝑥+𝑦+𝑧 , 𝑠 = (𝑥+𝑦+𝑧) , 𝑡 =
𝐴𝐼 𝑏 + 𝑐 𝐴𝐼 𝐴𝐼 𝑏+𝑐
= 𝑏; 𝑝 , = = 𝑧
,
𝐼𝐷 𝑎 𝐴𝐷 𝐴𝐼 + 𝐼𝐷 𝑎 + 𝑏 + 𝑐 (𝑥+𝑦+𝑧)

Similarly, 𝐴𝐼 1
We get = (1 + 𝑟),
𝐼𝐷 2
𝐵𝐼 𝑎+𝑐 𝐶𝐼 𝑎+𝑐
= , = 𝐵𝐼 1 𝐶𝐼 1
𝐵𝐸 𝑎 + 𝑏 + 𝑐 𝐶𝐹 𝑎 + 𝑏 + 𝑐 = (1 + 𝑠), = (1 + 𝑡), 𝑟 + 𝑠 + 𝑡 = 1,
𝐵𝐸 2 𝐶𝐹 2
1 1
𝑓(𝑎, 𝑏, 𝑐) = (1 + 𝑟)(1 + 𝑠)(1 + 𝑡) =
8 8
1
(1 + 1 + 𝑟𝑠 + 𝑠𝑡 + 𝑡𝑟 + 𝑟𝑠𝑡) > .
4

Applying the GM- AM inequality to the Q5. Suppose regular polygon of number of
numerator we get sides 𝒑𝟏 , 𝒑𝟐 , … , 𝒑𝒌 meet at a common verstex

230
Solving Mathematical Problems

so that no gaps is left, what is the relation Sol. (i) 𝑥1 ≤ 5 ⟹ 𝑥1 = = 5 𝑜𝑟 𝑥1 = 4 𝑜𝑟 𝑥1 =


between 𝒑𝟏 , 𝒑𝟐 , … , 𝒑𝒌 ? 3 𝑜𝑟 𝑥1 = 2 𝑜𝑟 𝑥1 = 1 number of solution in
positive integers is
Sol.: The sum of the exterior angles of a convex
polygon of any number of sides is 2𝜋. For a 4𝑐0 + 4𝑐0 + 2𝑐0 + 1𝑐0 + 0𝑐0 = 5 = 5𝑐1
regular polygon of p sides, each extarias angle
2𝜋 (ii)𝑥1 + 𝑥2 ≤ 5 ⟹ 𝑥1 + 𝑥2 =
would be and each interior angle would be
𝑝 5, 4, 3, 2. Number of solutions in
2𝜋
𝜋− . positive integers is
𝑝

If k polygons of 𝑝1 , 𝑝2 , … , 𝑝𝑘 sides meet at a 4𝑐1 + 3𝑐1 + 2𝑐1 + 1𝑐1 += 10 = 5𝑐2


point and there is no gap, then since the sum
of the angles at a point is 2II, it follows that
2𝜋 2𝜋 2𝜋
𝜋− + (𝜋 − ) + ⋯ + (𝜋 − ) = 2𝜋 (iii) 𝑥1 + 𝑥2 + 𝑥3 ≤ 5 ⟹ 𝑥1 + 𝑥2 + 𝑥3 =
𝑝1 𝑝2 𝑝𝑘
5, 4, 3. Number of solutions in positive integers
1 1 1 𝑘 is
𝑜𝑟, + + ⋯+ = −1
𝑝1 𝑝2 𝑝𝑘 2
4𝑐2 + 3𝑐2 + 2𝑐2 = 10 = 5𝑐3
(i) 𝑥1 + 𝑥2 + 𝑥3 + 𝑥4 ≤ 5 ⟹ 𝑥1 +
𝑥2 + 𝑥3 + 𝑥4 = 5, 4. Number of
solutions in positive integers is
4𝑐3 + 3𝑐3 = 5 = 5𝑐4 .
(ii) 𝑥1 + 𝑥2 + 𝑥3 + 𝑥4 + 𝑥5 ≤ 5 ⟹
𝑥1 + 𝑥2 + 𝑥3 + 𝑥4 + 𝑥5 = 5.
Number of solutions in positive
integers is 4𝑐4 = 5𝑐5 .

Total number of solutions in positive integer


is

5𝑐1 + 5𝑐2 + 5𝑐3 + 5𝑐4 = 25 − 1 = 31

The conjecture is that number of solutions


Q6. Find the number of solutions in positive in positive integers of
integers 𝒙𝟏 ≤ 𝟓, 𝒙𝟏 + 𝒙𝟐 ≤ 𝟓, 𝒙𝟏 + 𝒙𝟐 + 𝒙𝟑 ≤ 𝑥1 + 𝑥2 + ⋯ + 𝑥𝑘 ≤ 𝑥 𝑖𝑠 𝑛𝑐𝑘 .
𝟓,

𝒙𝟏 + 𝒙𝟐 + 𝒙𝟑 + 𝒙𝟒 ≤ 𝟓, 𝒙𝟏 + 𝒙𝟐 + 𝒙𝟑 + 𝒙𝟒 +
𝒙𝟓 ≤ 𝟓. Q7. Let f and g be real valued functions defined
for all real values of x and y and satisfying the
Make a conjecture about the number of equation
solution on positive integers of 𝒙𝟏 + 𝒙𝟐 + ⋯ +
𝒙𝒌 ≤ 𝒙, 𝒙 ≥ 𝒌.

231
Solving Mathematical Problems

f(x+ y) + f(x –y )= 2 f(x) .g(y) ∀ x, y. Prove that if 1 1


𝑁𝑜𝑤, 𝑦 = , ⟹ 𝑥2 + = 𝑟2,
f(x) is not identically zero and if 1 f(x) │≤│∀x, 𝑥 𝑥
then │g(y)│ ≤ 1∀y. ⟹ 𝑥 4 − 𝑟 2 𝑥 2 + 1 = 0,

Sol. Since │f (x) │ is bounded, it has a least ∴ Product of roots, i.e. 𝑥1 𝑥2 𝑥3 𝑥4 = 1,


upper bound M and since f is not identically 0. 1 1
M > 0. = 𝑥1 𝑥2 = . . = 𝑥1 𝑥2 = 𝑦3 𝑦4
𝑥3 𝑥4
MW, suppose that the inequality │g(y)│ ≤1
does not hold fordly. Then these is a point 𝑦0
such that │g(𝑦0 )│ >1. Q10. Prove the identity 𝒔𝒊𝒏𝟐𝒑 𝒙 + 𝒔𝒊𝒏𝟐𝒑 𝟐𝒙 +
𝟏
Using the given equation and the triangle 𝒔𝒊𝒏𝟐𝒑 𝟑𝒙 + ⋯ + 𝒔𝒊𝒏𝟐𝒑 𝒏𝒏 = +
𝟐
𝟏.𝟑.𝟓…(𝟐𝒑−𝟏) 𝝅
inequality. We get 2│f(x)││g(𝑦0 )│= │f(x+ 𝒏( ); if x = 𝒂𝒏𝒅 𝒑 < 2𝒏 (p is a
𝟐.𝟒.𝟔…𝟐𝒑 𝟐𝒏
𝑦0 )+ f(x -y)│≤ │1 f(x+ 𝑦0 )│+│f(x -𝑦0 )│≤
positive integer).
2M.
𝑀 Sol. S = 𝑠𝑖𝑛2𝑝 𝑥 + 𝑠𝑖𝑛2𝑝 2𝑥 + ⋯ + 𝑠𝑖𝑛2𝑝 𝑛𝑥
Hence │f(x)│≤ < 𝑀, contradicting the
│𝑔(𝑦0 )│
𝑛
fact that M is least upper bound of │f(x)│. We 1
= ∑ 𝑠𝑖𝑛2𝑝 × 𝑠𝑖𝑛2𝑝 𝑙𝛼 = (−1)𝑝 .
conclude that │g(y)│≤ for all y. 2𝑝 − 1
𝑙=1

𝑝−1
1
= ∑(−1)𝑘 2𝑝𝐶𝑘 𝑐𝑜𝑠2(𝑝 − 𝑘)𝑙𝛼 + . 2𝑝𝐶𝑝 .
Q.8. Show that for any odd prime p, there 𝑖 2𝑝
𝑘=0
exists a positive integers n such that
𝒏 (−1)𝑝
𝒏, 𝒏𝒏 , 𝒏𝒏 ………..all leave the same remainder 𝑝−1
∴ S = 2𝑝−1 . ∑𝑘=0(−1): 2𝑝𝐶𝑘 ∑𝑛 𝑐𝑜𝑠2(𝑝 −
upon division by p where n does not leave a 𝑛
𝑘)𝑙𝛼 + 𝑝 . 2𝑝𝐶𝑝 .
remainder of 0 or 1 upon division by p.

Sol. We claim that n = 2p -1 will satisfy the given Put 2 (p –k )= 𝜆, then ∑𝑛𝑙=𝜆 2𝑐𝑜𝑠2(𝑝 −
𝑛+1
𝑠𝑖𝑛𝑛𝜆 cos
conditions. First 1n = 2p -1 is odd and n = 2p -1 𝑘)𝑙𝛼 = 𝑐𝑜𝑠𝜆 + ⋯ + 𝑐𝑜𝑠𝑛𝜆 = 2
𝜆
sin
= -1 (mod p). So for any power of n, we have 2
𝑛 𝑛
𝑛𝑛 = (−1)𝑛 = −1(𝑚𝑜𝑑 𝑝). …………….. (i)

𝑛
∴ All the numbers 𝑛, 𝑛𝑛 , 𝑛𝑛 , … … leave a Case A: Equation. (i) = 0 (if k is of the same point
remainder of p -1 when divided by p. Finally, y as p) (k ≡p) and
since p is an odd prime, 2p -1 cannot be 0 or 1. Case B: Equation. (i) = -1(if k and p are of
different point) k = (p +1).

(−1)𝑝 𝑝−1 1
Q9. If a circle intersects the hyperbola 𝒚 =
𝟏
at S= 2𝑝−1 . ∑𝑘=0(−1)𝑘 2𝑝𝐶𝑘 + 𝑛. 22𝑝 . 2𝑝𝐶𝑝 =
𝒙
1 𝑝−1 𝑛 1
four distinct points (𝒙𝒊 , 𝒚𝒊 ), i = 1, 2, 3, 4, then 22𝑝 −1
× ∑𝑘=0 2𝑝𝐶𝑘 + 22𝑝 . 2𝑝𝐶𝑝 = 2 +
show that 𝒙𝟏 𝒙𝟐 = 𝒚𝟑 𝒚𝟒 . 1.3.5….(2𝑝−1)
𝑛 ( 2.4.6….2𝑝 ) .
Sol. Equation of circle 𝑥 2 + 𝑦 2 = 𝑟 2

232
Solving Mathematical Problems

term as m+ n- 2 and common difference 2 and


the last term as given above.
MODEL TEST PAPER - 2

Q1. In a city, there are m – roads from North to


South and x−roads going from East to West, 𝟏 𝟏 𝟏
Q2. Show that when 𝒑 > 1, 𝟏𝒑 + 𝟐𝒑 + ⋯ + 𝒏𝒑 >
what is the length of the shortest path from 𝒙+𝟏 𝟏 𝟏 𝟏 𝟏
∫ 𝒙𝒑
𝒅𝒙 > 𝟐𝒑
+ 𝟑𝒑 … + 𝒏𝒑
North – East corner to the South-West corner
and how many such shortest paths are there? 𝟏 𝟏
What will be the lengths of other possible 𝒐𝒓 𝑺𝟏 > {𝟏 − }
𝒑+𝟏 (𝒏 + 𝟏)𝒑 + 𝟏
paths, if no part of a road is allowed to be > 𝑺𝒑 + (𝒏 + 𝟏)𝒑 − 𝟏
travelled twice?
𝟏 𝟏
Sol. 𝒐𝒓 {𝟏 − } < 𝑺𝒑
𝒑+𝟏 (𝒏 + 𝟏)𝒑 + 𝟏
𝟏 𝟏
< (𝟏 − )
𝒑+𝟏 (𝒏 + 𝟏)𝒑+𝟏
𝟏
+𝟏−
(𝒏 + 𝟏)𝒑

𝒔𝒉𝒐𝒘 𝒂𝒍𝒔𝒐 𝒕𝒉𝒂𝒕 𝒂𝒔 𝒏 → ∞ and p > 1.


𝟏 𝟏 𝟏
𝒑+𝟏
≤ ∑∞
𝟏 𝒏𝒑 ≤ 𝟏 + 𝒑+𝟏

Sol.

Let F(m, n) be the number of paths for reaching


from (1, 1) to (m, n) then are has to reach either
(m -1, n) or (m, n -1) before reaching the vertex
(m, n). This gives the functional equations.

F(m, n) = F(m- n) + F(n, m- 1). Knowing that F(1,


2) = 1, F(2, 1) = 1, we can deduce successively.
The values of F(2, 1) , F(3, 1); …….. F(m, n).
Alternatively we know that we have to cover m- This follows from the fact that the area under
1 step towards the west and n -1 steps towards 1
the curve 4 = 𝑥 𝑝 between x= 1 and x = n+ 1 lies
the South out of a total of m+ n- 2 steps so that
between the area of the horizontally shaded
the number of different paths is
rectangles and the sum of the horizontally and
(𝑚+𝑛−2)! vertically shaded rectangles.
(𝑚+𝑛−2
𝑚−1
) = (𝑚−1)!(𝑛−1)!

The shortest path is of length m+ n- 2 and the


longest path has the length mn if mn are both Q3. Given a parallelogram ABCD, can you
even and is mn−1 if one or both are odd and construct a quadrilateral PQRS of which the
the other lengths will be in AP with the first middle points of the sides are the four vertices

233
Solving Mathematical Problems

A, B, C, D of the given parallelogram? How QR and RS. As such PQRS is the required
many such quadrilaterals quadrilateral. However in this quadrilateral
angle Q and length l are arbitrary and therefore
Can be constructed? What are the maximum in general we can obtain a double infinity set of
and minimum areas of the these quadrilaterals with their middle points of the
quadrilaterals? What can you say about their
sides at the vertices of the given parallelogram.
perimeter?
In the given figure the area of the triangle PAB =
Sol. Given any quadrilateral PQRS we can easily 1
.
show that if we join the middle points of its 4

sides. The quadrilateral obtained will always be The area of the triangle PQS and the area of the
a parallelogram. Here we are concerned with 1
triangle RQC is the area of the triangle RQS,
the inverse problem. We are given the 4

parallelogram and we want to reconstruct the therefore the seem of the areas of the triangle
1
quadrilateral from which the given APB and RCD is 4 the area of the quadrilateral
parallelogram can be obtained by joining the PQRS. Similarly seem of the areas of triangles
middle points of the sides. QAD and SBC = ¼ of the quadrilateral PQRS,
therefore area of the quadrilateral outside the
parallelogram= the area of parallelogram and
therefore the area of the quadrilateral is double
the area of parallelogram.

But the area of parallelogram is fixed, therefore


the area of quadrilateral is also fixed. Thus
although we have an infinity of quadrilaterals for
the same parallelogram and these will have
different sides and angles, the areas of these
quadrilaterals will be the same. The quadrilateral
with the minimum perimeter will arise when the
Let ABCD be the parallelogram. Through A, we
quadrilateral is a square.
draw any line PAQ inclined at an arbitrary angle
O to the side AB such that AP = AQ = l where l is Q4. n! is defined only for n = 0 and for positive
any arbitrary length. integral values of n. Define a function for all
positive real values of x so that it reduces to n!
Now join P to B and produce it to S, that PB =
when x is non-negative integer n. How many
BS. Now join SC and QD and produce then to
such functions can be defined? Can you define
meet at R.
such a function which is both continuous and
In the triangle PQS, A and B are the middle differentiable for all non-negative real values
points of the two sides (by our construction) of x?
and therefore AB is l/2 and is parallel to QS in
Sol. We know that f(0) = 1, f(1) = 1, f(2) = 2, f(3)
the triangle RSQ, CD is parallel to QS and is
= 6, f(4) = 24, f(50 = 120, ………..
equal to ½ of QS. Since CD is parallel and = AB,
therefore C and D must be the middle points of

234
Solving Mathematical Problems

Now join (0, 1) to (1, 1) : (1, 1) to (2, 2) and (2, through the hole. What will be the curve
2) to (3, 6) and show by straight line segments traced by this pencil as the disc rolls.
and find equations of all the straight segments.
Sol. Let the rolling disc. Be initially at the lowest
Thus we find f(x) = 1, 0 ≤x ≤ 1 position and let P the point market on the disc.
Let Q be other point on the disc, then as the
= x, 1 ≤x ≤ 2. disc rolls every point on the circumference of
= 2(2x -3), 2 ≤ x ≤ 3. the disc between A and Q cares in contact in
succession with points on the ring between A
= 6(3x -8), 3 ≤ x ≤ 4. and Q arc AQ = arc AQ so that ∠𝐴𝑂𝑄 = 𝜃 = arc
AQ’ /a = arc.
= …………

= n! [1+ n(x –n )], n ≤ x ≤ n+1.

Thus the above function is defined for all non-


negative values of x and it reduce to n! when x
is a non-negative integer n. The function is
continuous at all positive integral values of n.

In the above discussion, we have fitted straight


line segments. If we fit parabolic segments of
the second degree, we can get a function which
is both continuous and differentiable
everywhere, but its second order derivative will
not exist for positive integral values of x. If we
AQ/a and as the disc continuous, to roll further
want there to exist, we would have to fit
the point P will come into contact with point P’
parabolic segments of the third degree.
where AOP = d AO’P’ = 2d and OP and P’ lie on a
straight line. Let P” be the new position of P
then O’P” P’ is a straight line. This shows that
locus of P is the line OP’. As the disc rolls the
marked point moves from P to P’ along the
diameter through O and P and this point moves
from P’ to P, then from P to O, then from O to B
and finally from B to the initial position of the
marked point. Thus the locus of any point on
the rim of the disc will be the diameter of the
vig passing through the initial position of the
marked point.

Q5. A circular disc rolls inside a circle of Q6. a, b, c are any three digits from 0 to 9.
double its radius. A hole is made in the disc Show that
near the circumstance. A pencil is passed

235
Solving Mathematical Problems

1. abcabc is divisible by 7, 11, 13. The last equation is an identity. For the proof,
2. If abc ≤ 500 then abc (twice abc) abc is we must reverse these steps. The only
divisible by (𝟕 × 𝟏𝟏 × 𝟏𝟑)𝟐 questionable step is from (e) to 1: the proof is
3. If abc ≤ 333 then abc (three times abc) valid only if we do not divide by zero in going
(three times abc) abc is divisible by from (2) to (1). Then do yourself.
(𝟕 × 𝟏𝟏 × 𝟏𝟑)𝟑
Q8. If 𝑭𝟏 denote the term in the Fibonacci
What is the final quotient in each care? sequence, then show that 𝑭𝒏+𝟏 𝟐 + 𝑭𝒏 𝟐 =
𝑭𝟐𝒏+𝟏 .
Sol. 7 × 11 × 13 = 1001 ⟹ (𝑎𝑏𝑐) ×
(1001) = 𝑎𝑏𝑐𝑎𝑏𝑐 Sol. The result holds for n= 1. So suppose the
result holds for integer k, then
⟹ (𝑎𝑏𝑐) × (1001)2 = 𝑎𝑏𝑐 (𝑡𝑤𝑖𝑐𝑒 𝑎𝑏𝑐)𝑎𝑏𝑐
𝐹𝑘+2 2 + 𝐹𝑘+1 2 = (𝐹𝑘+1 + 𝐹𝑘 )2 + 𝐹𝑘+1 2 =
⟹ (𝑎𝑏𝑐) × (1001)3 = 𝑎𝑏𝑐 (𝑡ℎ𝑟𝑒𝑒 𝑎𝑏𝑐)
𝐹𝑘+1 2 + 2𝐹𝑘+1 𝐹𝑘 + 𝐹𝑘 2 + 𝐹𝑘+1 2 = (𝐹𝑘+1 2 +
(𝑡ℎ𝑟𝑒𝑒 𝑎𝑏𝑐)𝑎𝑏𝑐. 𝐹𝑘 2 ) + (2𝐹𝑘+1 𝐹𝑘 + 𝐹𝑘+1 2 ) = 𝐹2𝑘+1 +
(2𝐹𝑘+1 𝐹𝑘 + 𝐹𝑘+1 2 ), the last step by the
inductive assumption.
Q7. Let 𝛼be a fixed real number such that 0 < 𝛼
𝐬𝐢𝐧 𝜽+𝐬𝐢𝐧(𝜽+𝜶)
We should be done it we could show
< 𝜋 and F(𝜃) = 𝒘𝒉𝒆𝒓𝒆 𝟎 ≤ 𝜽 ≤
𝐜𝐨𝐬 𝜽−𝐜𝐨𝐬(𝜽+𝜶) 2𝐹𝑘+1 𝐹𝑘 + 𝐹𝑘+1 2 = 𝐹2𝑘+1 , for we could then
𝝅 − 𝜶. Show that F is a constant. continue the previous argument and then you
do yourself.
Sol. Suppose that F is a const. then F(𝜃) + F(O) ∀
𝜃, 0 ≤ 𝜃 ≤ 𝜋 − 𝛼. That is 𝟏
Q9. Show that ∫𝟎 𝐥𝐨𝐠 𝒙 𝐥𝐨𝐠(𝟏 − 𝒙)𝒅𝒙 = 𝟐 −
sin 𝜃+sin(𝜃+𝛼) 𝑠𝑖𝑛𝛼 𝝅𝟐
= 1−𝑐𝑜𝑠𝛼 ……… (i), 𝟔
.
cos 𝜃−cos(𝜃+𝛼)

1 𝑥2 𝑥3
[sin 𝜃 + sin(𝜃 + 𝛼)][1 − 𝑐𝑜𝑠𝛼] = 𝑠𝑖𝑛𝛼 Sol. 1 = ∫0 log 𝑥 (𝑥 + + + ⋯ ) 𝑑𝑥
2 3

cos 𝜃 − cos(𝜃 + 𝛼) …………… (ii) 1 ∞

= ∫ log 𝑥 ∑ −𝑥 𝑘 + 𝑑𝑥
sin 𝜃 + sin(𝜃 + 𝛼) − 𝑠𝑖𝑛𝜃𝑐𝑜𝑠𝛼 − sin(𝜃 + 0 𝑘=1
𝛼) cos 𝛼 ∞
1 1
= − ∑ ∫ 𝑥 𝑘 𝑙𝑜𝑔𝑥𝑑𝑥
= 𝑠𝑖𝑛𝛼𝑐𝑜𝑠𝜃 − 𝑠𝑖𝑛𝛼cos(𝜃 + 𝛼) ………… (iii), 𝑘 0
𝑘=1

1
sin 𝜃 + sin(𝜃 + 𝛼) − [𝑠𝑖𝑛𝜃𝑐𝑜𝑠𝛼 + 1
𝑁𝑜𝑤 ∫ 𝑥 𝑘 𝑑𝑥 = ( )
𝑠𝑖𝑛𝛼𝑐𝑜𝑠𝜃] − [sin(𝜃 + 𝛼) cos 𝛼 − 𝑠𝑖𝑛𝛼𝑐𝑜𝑠(𝜃 + 0 𝑘+1
𝛼)] = 𝜃 ………….. (iv)
Differentiating centre the sign of integration
sin 𝜃 + sin(𝜃 + 𝛼) − sin(𝜃 + 𝛼) − sin(𝜃 + 1
with respect to k, we get ∫0 𝑥 2 𝑙𝑜𝑔𝑥𝑑𝑥 =
𝛼 − 𝛼) = 0…………. (v) 1
− (𝑘+1)2

236
Solving Mathematical Problems
∞ ∞
1 𝑘+1−𝑘 plays are game with each of other players and
𝐼= ∑ = ∑ rules are such that no ties can occur. If
𝑘(𝑘 + 1)2 𝑘(𝑘 + 1)2
𝑘=1 𝑘=1
𝑾𝒏 𝒂𝒏𝒅 𝑳𝒏 be the number of games won and
∞ last respectively by player 𝒑𝒏 ,Show that
1 1
=∑ ( − ) ∑𝒏𝒓=𝟏 𝑾𝒓 𝟐 = ∑𝒏𝒓=𝟏 𝑳𝒓 𝟐 .
𝑘(𝑘 + 1) (𝑘 + 1)2
𝑘=1

∞ ∞ Sol:
1 1 1
= ∑ ( − )−∑
𝑘 𝑘+1 (𝑘 + 1)2 Let ∑𝑛𝑟=1 𝑊𝑟 2 = ∑𝑛𝑟=1 𝐿𝑟 2 𝑡ℎ𝑒𝑛 ∑𝑛𝑟=1(𝑊𝑟 2 −
𝑘=1 𝑘=1
𝐿𝑟 2 ) = 0.
𝜋2 𝜋2
=1−( − 1) = 2 − . ∑𝑛𝑟=1(𝑊𝑟 − 𝐿𝑟 )(𝑊𝑟 + 𝐿𝑟 ) = 0 𝑏𝑢𝑡 𝑊𝑟 + 𝐿𝑟 =
6 6
𝑛 − 1for each. r, so (𝑛 − 1) ∑𝑛𝑟=1(𝑊𝑟 − 𝐿𝑟 ) =
0,

Q10. Let r and s be nonzero integers. Prove 𝑛 𝑛 𝑛

that (𝒓𝟐 − 𝒔𝟐 )𝒙𝟐 − 𝟒𝒓𝒔𝒙𝒚 − (𝒓𝟐 − 𝒔𝟐 )𝒚𝟐 has ⟹ ∑(𝑊𝑟 − 𝐿𝑟 ) = 0, ⟹ ∑ 𝑊𝑟 = ∑ 𝐿𝑟 .


𝑟=1 𝑟=1 𝑟=1
no solution in integers x, y.
This last equation is true, since the total
Sol. The L.H.s. can be factorized so that{(𝑟 −
number of games won by the players has to
𝑠)𝑥 − (𝑟 + 𝑠)𝑦} {(𝑟 + 𝑠)𝑥 − (𝑟 − 𝑠)𝑦} = 1.
equal the total number of games lost.
Both factors are integers. As such the only
possibility is that (r –s )x – (r +s )y- k = 0. (r+ s)x + Q2. If x, y, z, are positive integers, show that
(r –s )y –k = 0, (x, y) (x, z) (y, z) [𝒙, 𝒚, 𝒛]𝟐 =
𝑥 𝑦 [𝒙, 𝒚][𝒙, 𝒛][𝒚, 𝒛](𝒙, 𝒚, 𝒛)𝟐 , where (a, …, g) and
Where k is either 1 or -1, so that = =
𝑘𝑟 −𝑘𝑠 [a, …, g] denote gcd (a, …, g) and lcm (a, …, g)
1
𝑟 2 +𝑠2
. respectively.

𝑘𝑟 𝑘𝑠 Sol. Because of unique factorization, if suffices


𝑥= ,𝑦 = 2 , 𝑠𝑜 𝑡ℎ𝑎𝑡 𝑥 2 + 𝑦 2
𝑟2 +𝑠 2 𝑟 + 𝑠2 to show that for each prime p, the power of p
𝑘2 on the left side (in its prime factorization) is
= 2 ,
𝑟 + 𝑠2 equal to the power of n on the right side. So let
𝑜𝑟 (𝑥 2 + 𝑦 2 )(𝑟 2 + 𝑠 2 ) = 𝑘 2 = 1, = 1 𝑟 2 + 𝑠 2 x = 𝑝𝑎 𝑟, 𝑦 = 𝑝𝑏 𝑠 𝑎𝑛𝑑 𝑧 = 𝑝𝑐 𝑡 , for integers r,
= 1, ⟹ 𝑟 = 1, 𝑠 = 0, 𝑜𝑟 𝑟 s, t each relatively prime to p. We may assume
= 0, 𝑠 = 1. (as of symmetry and by relabeling if necessary
that) a ≤ b≤ c. Then the power of p in the
But r and s are non-zero integers. unique factorization of [𝑥, 𝑦, 𝑧]2 is 2c; the
powers of p in (x, y), (x, z) and (y, z) are a, a and
b respectively. Hence the power of p on the left
MODEL TEST PAPER – 3 side is 2a + b + 2c. In the same manner, the
power of p on the R. S. is b + c + c + 2a = 2a+ b +
Q1. In a round robin tournament with n- 2c.
players 𝒑𝟏 , 𝒑𝟐 , … . , 𝒑𝒏 where n > 1, each player

237
Solving Mathematical Problems

Q3. Suppose f(x) = a, 𝐬𝐢𝐧 𝒙 + 𝒂𝟐 𝐬𝐢𝐧 𝟐𝒙 + ⋯ + 𝐴𝑙𝑠𝑜 𝑓(𝑥) + 𝑓(0) = 𝑓(𝑥) ⇒ 𝑓(0) = 0 ⇒ 𝐷
𝒂𝒏 𝐬𝐢𝐧 𝒏𝒙 𝒘𝒉𝒆𝒓𝒆 𝒂𝟏 , 𝒂𝟐 , … , 𝒂𝒏 are real = 0.
numbers and n is a positive integer. If (𝒙)𝟏 ≤
𝑓(𝑥) = 𝑐 tan−1 𝑥.
𝐬𝐢𝐧 𝒙 ∀ 𝒙, show that 𝟏𝒂, +𝟐𝒂𝟐 + ⋯ + 𝒏𝒂𝒏 𝟏 ≤
𝟏. Q5. Find two non-congruent similar triangles
with sides of integral length having the length
Sol. Let we try inducting on the number of
of two sides of one triangle equal to two sides
terms in f(x). When n= 1, f(x) =
of the other triangle.
𝑎1 sin 𝑥 𝑎𝑛𝑑 𝑠𝑖𝑛𝑐𝑒 1 𝑓(𝑥)1 ≤∣ sin 𝑥 ∣
𝜋
, 𝑖𝑡 𝑓𝑜𝑙𝑙𝑜𝑤𝑠 𝑡ℎ𝑎𝑡 ∣ 𝑎1 ∣=∣ 𝑎1 sin ( 2 ) ∣= ∣ Sol. Let a, b, c ; b, c, d, be the lengths of the
𝑓
𝜋
(2) ∣≤ ∣
𝜋
sin ( 2 ) ∣= 1. sides of the two triangles and let a < b. Since the
triangles are similar.
Suppose the result holds for k and consider the 𝑎 𝑏 𝑐 𝑏2 𝑐2
functions f(x) = 𝑎1 sin 𝑥 + 𝑎2 sin 2𝑥 + ⋯ + = = , 𝑠𝑜 𝑡ℎ𝑎𝑡 𝑐 = ,𝑑 = but c < a+ b
𝑏 𝑐 𝑑 𝑐 𝑏
𝑎𝑘 sin 𝑘2 + 𝑎𝑘 + 1𝑠𝑖𝑛 (𝑘 + 1)𝑥 for some choice 𝑏2 𝑏2 𝑏
⇒ 𝑎
< 𝑏 + 𝑎 ⇒ 𝑎2 − 𝑎 − 1 < 0.
of real numbers 𝑎1 , 𝑎2 , … , 𝑎𝑘+1 and suppose
that if∣ 𝑓(𝑥) ∣≤∣ sin 𝑥 ∣ ∀ 𝑥. Since sin(𝑘 + 𝑏 𝑏 1 𝑈𝑠+1
⇒ ( − 𝑟) ( + ) < 0 𝑤ℎ𝑒𝑟𝑒 𝑟 = .
𝑎 𝑎 𝑟 2
1) 𝑥 = sin 𝑘𝑥 𝑐𝑜𝑠𝑥 + sin 𝑥 cos 𝑥 𝑘𝑥, we can
coride 𝑓(𝑥) = (𝑎1 + 𝑎𝑘 + 1𝛼 sin 𝑘𝑥). We have 𝑏
= 1.618 … . . ⇒ < 𝑟.
𝑎
now recoritten 𝑓(𝑎) as a sum of k terms, more
or less of type from which we can apply the 𝑏
As such we can take 𝑎 any rational number
induction assumption. Then you do yourself. 𝑏 4
between 1 and 1.6. The suppose we take = .
𝑎 3
Q4. Find all functions which are everywhere
16𝑘 64𝑘
differentiable and satisfy f(x) + f(y) = ∴ 𝑎 = 3𝑘, 𝑏 = 4𝑘, 𝑐 = 𝟑
,𝑑 = 9
𝒙+𝒚
𝒇( ) ∀ 𝒙, 𝒚 for which xy ≠ 1.
𝟏−𝒙𝒚 If we choose k = 9
Sol. Differentiating partially with respect to x, y a = 27, b = 36, c = 48, d = 64.
𝑥+𝑦 1+𝑦 2
f’(x) = 𝑓 ′ (1−𝑥𝑦) (1−𝑥𝑦)2 Length of all the sides are integers and
𝑎 𝑏 𝑐
′ (𝑦) ′(
𝑥+𝑦
) 1−𝑦 2 𝑏
= 𝑐
= 𝑑 is satisfied.
𝑓 =𝑓 1−𝑥𝑦
(1−𝑥𝑦)2
Q6. Let a, b, c be three real numbers such that
Dividing(1 + 𝑥 2 )𝑓 ′ (𝑥) = (1 + 𝑦 2 )𝑓 ′ (𝑦). a < b < c f(x) is continuous in [a, c] and
Since L.H.S. is a function of x only and R.H.S. is differentiable in (a, c). Also f’(x) is strictly
a function of y only, each must be a constant, so increasing in (a, c). Prove that (c - b) f(a) + (b -
that a) f(c) > (c - a) f (b).

𝑓(𝑏)− 𝑓(𝑎)
𝐶 Sol. : By mean value theorem =
𝑓(𝑥) = , 𝑓(𝑥) = 𝐶 tan−1 𝑥 + 𝐷. 𝒃−𝒂
1 + 𝑥2 𝑓 ′ (𝑢),
𝑎<𝑢<𝑏

238
Solving Mathematical Problems
𝑓(𝑐)−𝑓(𝑏) Q8. Find the number of points in the cartesian
𝑐−𝑏
= 𝑓 ′ (𝑣), 𝑏 < 𝑣 < 𝑐
plane with integral co- ordinate satisfying the
Since f’(x) is strictly increasing f’(u) < f’(v) inequalities ∣x∣ ≤ k, ∣y∣ ≤ k, ∣x - y∣ ≤ k.

𝑓(𝑏)− 𝑓(𝑎)
<
𝑓(𝑐)−𝑓(𝑏) Sol. : We must –k ≤ x ≤ k, -k ≤ y ≤ k, -k ≤ y –
𝑏−𝑎 𝑐−𝑏
x ≤ k.
𝑜𝑟 𝑓(𝑏)(𝑐 − 𝑏 + 𝑏 − 𝑎) − 𝑓(𝑎)(𝑐 − 𝑏) −
The points are bounded by the straight lines
𝑓(𝑐)(𝑏 − 𝑎) < 0.
x = -k , x = k
𝑜𝑟, (𝑐 − 𝑏)𝑓(𝑎) + (𝑎 − 𝑐)𝑓(𝑏) + (𝑏 −
𝑎)𝑓(𝑐) > 0. y = -k , y = k

𝑜𝑟, (𝑏 − 𝑐)𝑓(𝑎) + (𝑐 − 𝑎)𝑓(𝑏) + (𝑎 − y = x – k, y = x + k


𝑏)𝑓(𝑐) < 0.

𝟏 𝟏
Q7. Evaluate bin 𝒏 ∑𝒏𝒌=𝟏 ∑𝒏𝒋=𝟏 𝒋𝟐 +𝒌𝟐.

Sol.

When
x = −k, we get y = −k, −(k − 1), … . 0 k + 1
x = −(x − 1)y = −k, … … 1, k + 2 }
………………………………………………..
𝑥 = −1, 𝑦 = −𝑘, … 𝑘 − 1, 𝑘 + 𝑘
𝑗
1 𝑥 𝑑𝑥 𝑑𝑦 𝑥 = 0, 𝑦 = −𝑘, … 𝑘, 𝑘 + 𝑘 + 1
I= lim ∑𝑛𝑘=1 ∑𝑛𝑗=1 𝑗2 𝑘2 . 𝑛2 𝑛
= ∫ ∫ 𝑥 2 +𝑦2 =
𝑛→∞ + 𝑥 = 1, 𝑦 = −(𝑘 + 1), … . , 𝑘, 𝑘 + 𝑘
𝑛2 𝑛2
𝐷1 +𝐷2 𝑥 𝑑𝑥 𝑑𝑦 𝑥 𝑑𝑥 𝑑𝑦 𝑥 = 2, 𝑦 = −(𝑘 − 2), … . , 𝑘, 𝑘 + 𝑘 − 1
∫ ∫𝐷 𝑥 2 +𝑦2
+ ∫ ∫𝐷 𝑥 2 +𝑦 2 ………………………………………..
1 2
𝑥=𝑘 𝑦 = 0 … … … … 𝑘, 𝑘 + 1 }
𝜋
𝑠𝑒𝑐𝜙
4
= ∫ cos 𝜙 𝑑𝜙 ∫ 𝑑𝑟 Total number of points
0
𝜋
4
𝑐𝑜𝑠𝑒𝑐𝜙 = 2 [(k +1) + …+(k +k)] + 2k +1
+ ∫ cos 𝜙 𝑑𝜙 ∫ 𝑑𝑟
= k (3k +1)+ 2k + 1
𝜋 𝜋
𝜋 2 𝜋 𝜋/4 2 = 3𝑘 2 + 3𝑘 + 1
= + ∫ cot 𝜙 𝑑𝜙 = + [𝑙𝑜𝑔 𝑠𝑖𝑛𝜙] 𝜋
4 0 4
4 = (𝑘 + 1)3 − 𝑘 3 .
𝜋 1 𝜋 1
= − log = + log 2.
4 √2 4 2

239
Solving Mathematical Problems
1
Q9. Let 𝒂𝒙𝟐 + 𝒃𝒙𝒚 + 𝒄𝒚𝟐 𝒂𝒏𝒅 𝑨𝒙𝟐 + 𝑩𝒙𝒚 + By another method, area = 2 𝑎2 sin 60° =
𝑪𝒚𝟐 be two positive definite forms which are √3 2 √3 √3
𝑎 = [(𝑥1 − 𝑥2 )2 + (𝑦1 − 𝑦2 )] = ×
not proportional. Prove the from 4 4 4
𝑎𝑛 𝑖𝑛𝑡𝑒𝑔𝑒𝑟
(𝒂𝑩 − 𝒃𝑨)𝒙𝟐 + 𝟐(𝒂𝑪 − 𝒄𝑨)𝒙𝒚 + (𝒃𝑪 −
𝒄𝑩)𝒚𝟐 is indefinite. And this is not an integer, since √3 is irrational.

Sol. : Since the two forms are positive definite. The contradiction in the result obtained by the
two methods proves the required result.
a>0:c>0 𝑏 2 − 4𝑎𝑐 < 0

A>0:C>0 𝐵2 − 4𝑎𝑐 < 0


MODEL TEST PAPER - 4
To show that the new form is indefinite, we
𝟏
have to show that its discriminant D > 0. Now Q1. Solve 𝒙𝒏+𝟏 = (𝒙𝒏 +
𝟐
𝟏
D = 4(𝑎𝐶 − 𝑐𝐴)2 − 4(𝑎𝐵 − 𝑏𝐴)(𝑏𝐶 − 𝑐𝐵) ) , 𝒙𝟎 𝒂 𝒈𝒊𝒗𝒆𝒏 𝒄𝒐𝒎𝒑𝒍𝒆𝒙 𝒏𝒖𝒎𝒃𝒆𝒓
𝒙𝒏

𝑎2 𝐷 = {2𝑎 (𝑎𝐶 − 𝑐𝐴) − 𝑏(𝑎𝐵 − 𝑏𝐴)}2 − 𝒂𝒏𝒅 𝒇𝒊𝒏𝒅 𝐥𝐢𝐦 𝒙𝒏 .


𝒏→∞
(𝑏 2 − 4𝑎𝑐)(𝑎𝐵 − 𝑏𝐴)2
1+𝑧 𝑥 −1
Sol. : Substituting 𝑥𝑛 = 1−𝑧𝑛 , 𝑧𝑛 = 𝑥𝑛 +1
And D would be > 0 unless 𝑛 𝑛

1+ 𝑧 +1 1 1−𝑧 1−𝑧 1+𝑧 2


𝑎𝐵 − 𝑏𝐴 = 0, 𝑎𝐶 − 𝑐𝐴 = 0 We get, 1−𝑧 𝑛+1 = 2 (1−𝑧𝑛 + 1+𝑧𝑛 ) = 1−𝑧𝑛 2
𝑛 𝑛 𝑛 𝑛

𝑎 𝑏 𝑐
i.e. unless 𝐴 = 𝐵 = 𝐶 𝑧𝑛+1 = 𝑧𝑛 2 = 𝑧𝑛−1 4 = ⋯ = 𝑧0 2𝑛+1 ∣ 𝑧𝑛 + 1 ∣
= ∣ 𝑧0 ∣ 2𝑛+1
Which is given to be not true. As such D > 0 and
4
the given form is indefinite. 𝑥𝑛−1 𝑥0+1 2
𝑜𝑟, ( )= ( )
𝑥𝑛+1 𝑥0+1
4
𝐴𝑙𝑠𝑜 ∣ 𝑧𝑛 ∣= ∣ 𝑧0 ∣2
Q10. Prove that there is no equilateral triangle
all of where vertices have integral co- 𝐴𝑠 𝑛 → ∞, 𝑖𝑓 ∣ 𝑧0 ∣ > 1, ∣ 𝑧𝑛 ∣→ ∞
ordinates.
𝑖𝑓 ∣ 𝑧0 ∣< 1, ∣ 𝑧𝑛 ∣→ 0.
Let (𝑥1 , 𝑦1 ), (𝑥2 , 𝑦2 ) be the vertices of the
𝐼𝑓 ∣ 𝑧0 ∣= 1, 𝑧0 = 𝑇ℎ𝑒𝑛 𝑧𝑛→1
triangle and let a be the length of the side of
if k is of the form
the equilateral triangle.
𝑎
𝑥1 𝑦1 1 2𝑏
, 𝑎, 𝑏 are integers and 𝑧𝑛 diverges if k is not of
1
The area of the triangle = ∣ 𝑥2 𝑦2 1∣= this form Thus if Re 𝑥0 > 0, 𝑡ℎ𝑒𝑛 𝑥𝑛 → 1
2
𝑥3 𝑦3 1
𝑎𝑛 𝑖𝑛𝑡𝑒𝑔𝑒𝑟 Re 𝑥0 < 0, 𝑡ℎ𝑒𝑛 𝑥𝑛 → −1

Re 𝑥0 = 0, 𝑡ℎ𝑒𝑛 𝑥𝑛 𝑑𝑖𝑣𝑒𝑟𝑔𝑒𝑠.

240
Solving Mathematical Problems

Q2. If 𝑪𝟎 , 𝑪𝟏 , 𝑪𝟐 , … . . , 𝑪𝒏 are real numbers P is a perfect cube if (n- 1) (n+ 1) (n+ 2)= 𝑛3 +
satisfying 𝑪𝟎 + 𝑪𝟏 + ⋯ + 𝑪𝒏 = 𝟎, Show that 2𝑥 − 𝑥 − 2 is a perfect cube, but if n > 2.
𝑪𝟎 + 𝟐𝒄, 𝒙 + ⋯ + (𝒏 + 𝟏)𝑪𝒏 𝒙𝒏 = 𝟎 has at
least one real roof. 𝑛3 < 𝑛3 + 2𝑛(𝑛 − 2) + 3𝑛 − 2

(𝑛 + 1)3 > 𝑛3 + 2𝑛2 − 𝑛 − 2, so that we have


Sol. : If f(x)= 𝐶0 + 2𝑐, 𝑥 + ⋯ + (𝑛 + 1)𝐶𝑛 𝑥 𝑛
a cube lying between two consecutive cubes
1 which is not possible. As such when n is odd. P
∫ 𝑓(𝑥)𝑑𝑥 = 𝐶0 + 𝐶1 + … . . + 𝐶𝑛 = 0
0 cannot be a perfect cube.

By the mean value theorem of integral calculus, Care (ii) let n be even so that (n+ 1) and (n- 1)
there exist on (0 <f < 1) such that f(f) = 0. are odd and n+ 2 is a consecutive even term so
that if p is a perfect cube then so will be n (n -1)
So that f(x) = 0 has at least one real roof. (n+ 2)= 𝑛3 + 𝑛2 and if n > 2 , 𝑛3 < 𝑛3 + 𝑛2 +
2𝑛 < (𝑛 + 1)3

Q3. Prove that the product of four consecutive So that if p is a perfect cube., we shall again
positive integers cannot be a perfect square on have a cube numbers lying between two
a perfect cube. consecutive cubes which is again impossible.

Sol. :Let the form integers be n – 1, n, n + 1, n + Thus p cannot be a perfect cube whether n is
2 so that P = (𝑛 − 1)𝑛(𝑛 + 1)(𝑛 + 2) = even on odd.
(𝑛2 + 𝑛 − 2)(𝑛2 + 𝑛) = (𝑛2 + 𝑛 − 1)2 − 1. If
P is a perfect square, then P and (𝑛2 + 𝑛 − 1)
would be two consecutive integers., both of Q4. Prove that if 𝒁𝟏 , 𝒁𝟐 represent two vertices
which one square, which is impossible. As such of an equilateral triangle in the Argand plane,
P cannot be a perfect square. then the third vertex is given by −𝒘𝒛𝟏 − 𝒘𝟐 𝒛𝟐
where w is a cube root of unity.
Now when n = 1, p = 0 and when n = 2, p = 24
which are not perfect cubes. So we take n > 2. Sol. : If 𝑧1 , 𝑧2 , 𝑧3 are the vertices of the
equilateral triangle, then since the sides are of
Again out of two integers n and n+1, only one equal length.
can be odd cubes, case (i) Let n be odd, then n-
1, n +1, are even and so n is relatively prime to ∣ 𝑧3 − 𝑧1 ∣= ∣ 𝑧2 − 𝑧1 ∣= ∣ 𝑧3 − 𝑧2
both n- 1 and n +1 . ∣ 𝑠𝑜 𝑡ℎ𝑎𝑡 𝑖𝑓 𝑧2 − 𝑧1 = 𝑎𝑒 𝑖𝜃
𝜋
Again it is also relatively prime to n +2. Since 𝑧3 − 𝑧1 = 𝑎𝑒 𝑖𝜃± 3 .
2m + 1 and 2m +3 cannot have a common
𝜋
𝑧 −𝑧
factor because if they had it would divide their So that 𝑧3 −𝑧1 = 𝑒 ±𝑖 3 = 𝑘. Where k is a root of
3 1
difference viz. 2. As such it is relatively prime to 𝜋
the equation 𝑘 2 − 𝑘 1 (2 cos 3 ) + 1 =
(n- 1) (n +1) (n +2).
0 𝑜𝑟, 𝑘 2 − 𝑘 + 1 = 0
Now, P = [n] [(n- 1) (n+ 1) (n+ 2)] since the two
factors in squares brackets are relatively prime.

241
Solving Mathematical Problems

So that k = -w where w is a imaginary cube root 𝑥


∑𝑘𝑛=1 3𝑛−1 . 𝑠𝑖𝑛3 ( 𝑛 ) =
3
of unity and 1+𝜔+ 𝜔2 = 0 3 𝑥 1 𝑥
∑𝑘𝑛=1 3𝑛−1 [ sin ( 𝑛 ) − sin ( )]. Then do
4 3 4 3𝑛−1
𝑧3 = 𝑘𝑧2 + 𝑧1 (1 − 𝑘) = 𝜔𝑧2 + 𝑧1 (1 + 𝜔) yourself.
= −𝜔𝑧2 − 𝜔2 𝑧1

Since w, 𝑤 2 are two imaginary cube roots of


unity, the third vertex is either−𝜔𝑧2 − Q7. Find all positive continuous functions f(x)
𝟏
𝜔2 𝑧, 𝑜𝑟 − 𝜔2 𝑧2 − 𝜔𝑧1 . defined in [0, 1] for which ∫𝟎 𝒇(𝒙)𝒅𝒆 =
𝟏 𝟏
𝟏, ∫𝟎 𝒙 𝒇(𝒙)𝒅𝒙 = 𝒂, ∫𝟎 𝒙𝟐 𝒇(𝒙)𝒅𝒙 = 𝒂𝟐 .

Q5. Find all pairs (m, n) of integer larger then > Sol. : Multiply these equations by
1 such that ∣ 𝒑𝒎 − 𝒒𝒏 ∣= 𝟏, where p and q are 𝑎2 , −2𝑎 𝑎𝑛𝑑 1 respectively and odd we get .
primes. 𝑎
∫0 (𝑎 − 𝑥)2 𝑓(𝑥) 𝑑𝑥 = 0 and this cannot be
Sol. : If is immediate that not both p and q are satisfied if f(x) is positive and continuous in [0,
odd, for this would imply that 𝑝𝑚 − 𝑞 𝑛 is even. 1].
So let that q =2. We will show, by using only
algebraic identities of this section, that the only As such there is no positive function satisfying
solution is that formal. Suppose m and n are the conditions of the problem.
larger than 1 and that ∣𝑝𝑚 − 2𝑛 ∣= 1. It cannot
Q8. Find all twice differentiable real valued
be the case that m and n are both even for if
functions with domain the set of all real
m = 2r and n = 2s, then
numbers and satisfying the functional
1 = ∣𝑝𝑚 − 2𝑛 ∣= ∣𝑝2𝑟 − 22𝑠 ∣ = ∣𝑝𝑟 − 2𝑠 ∣∣𝑝𝑟 + equation 𝒇𝟐 (𝒙) − 𝒇𝟐 (𝒚) = 𝒇(𝒙 + 𝒚)𝒇(𝒙 − 𝒚).
2𝑠 ∣ and this is impossible (∵𝑝𝑟 + 2𝑠 > 1).
Sol. : Putting x = 0, y = 0 we get f(0) = 0.
Then you do yourself.
Differentiating the given equation first partially
with respect to x and then with respect to y, we
𝒙 get
Q6. Evaluate ∑∞
𝒏=𝟏 𝟑
𝒏−𝟏
𝒔𝒊𝒏𝟑 (𝟑𝒏 )
2𝑓(𝑥)𝑓 ′ (𝑥) = 𝑓 ′ (𝑥 + 𝑦)𝑓(𝑥 − 𝑦) + 𝑓(𝑥 +
Sol. : Using the moivre’s theorem,
𝑦)𝑓 ′ (𝑥 − 𝑦)
3
sin 3𝜃 = 𝐼𝑚 (𝑒 3𝑖𝜃 ) = 𝐼𝑚 ((𝑒 𝑖𝜃 ) ) = 0 = 𝑓 ′′ (𝑥 + 𝑦)𝑓(𝑥 − 𝑦) − 𝑓 ′ (𝑥 + 𝑦)𝑓 ′ (𝑥 −
3
𝐼𝑚 [𝑐𝑜𝑠𝜃 + 𝑖𝑠𝑖𝑛𝜃] = 𝐼𝑚 [𝑐𝑜𝑠 3
𝜃+ 𝑦) + 𝑓 ′ (𝑥 + 𝑦)𝑓 ′ (𝑥 − 𝑦) − 𝑓(𝑥 − 𝑦)𝑓 ′′ (𝑥 − 𝑦)
3𝑐𝑜𝑠 𝜃𝑖𝑠𝑖𝑛𝜃 + 3𝑐𝑜𝑠𝜃𝑖 𝑠𝑖𝑛 𝜃 + 𝑖 3 𝑠𝑖𝑛3 𝜃]
2 2 2
= 𝑓 ′′ (𝑥 + 𝑦)𝑓(𝑥 − 𝑦) − 𝑓 ′′ (𝑥 − 𝑦)𝑓(𝑥 + 𝑦).
2 3
= 3𝑐𝑜𝑠 𝜃𝑠𝑖𝑛𝜃 − 𝑠𝑖𝑛 𝜃 = 3[(1 −
𝐼𝑓 𝑥 + 𝑦 = 𝜇, 𝑥 − 𝑦 = 𝑣 ∶ 𝑡ℎ𝑖𝑠 𝑔𝑖𝑣𝑒𝑛
𝑠𝑖𝑛2 𝜃)𝑠𝑖𝑛𝜃] − 𝑠𝑖𝑛3 𝜃
𝑓 ′′ (𝜇)𝑓(𝑣) − 𝑓 ′′ (𝑣)𝑓(𝜇) = 0
= 3𝑠𝑖𝑛𝜃 − 4𝑠𝑖𝑛3 𝜃 . It follows that 𝑠𝑖𝑛3 𝜃 =
3 1
𝑠𝑖𝑛𝜃 − 4 𝑠𝑖𝑛3𝜃. Thus, 𝑆𝑘 = 𝑓′′ (𝜇) 𝑓′′ (𝜇)
4 𝑜𝑟, 𝑓 ′′ (𝜇)/𝑓(𝜇) = 𝑓(𝑣)
𝑠𝑜 𝑡ℎ𝑎𝑡 𝑓(𝜇)
= 𝑐.

242
Solving Mathematical Problems

Which gives a using f(0) = 0 become imaginary ellipse. Every real ellipse
intersects every hyperbola at right angles at
f(𝜇) = A sin mu, C = 𝑚2 every point of intersection.
= A sin mu, C= −𝑚2

= Au C=0 Q10. Let p be a point in the interior of 𝛥ABC


There include the solution f(u) ≡ 0 when A ≡ and 𝒓𝟏 , 𝒓𝟐 , 𝒓𝟑 denote the distance from p to
0. the sides 𝒂𝟏 , 𝒂𝟐 , 𝒂𝟑 of the triangle respectively.
If R denote the circumradius of 𝛥ABC. Show
Q9. Find the orthogonal trajectories of the that
𝒙𝟐 𝒚𝟐
family of curves 𝒂𝟐+𝝀 + 𝒃𝟐+𝝀 − 𝟏 = 𝟎. 𝟏
𝟐 𝟏
𝟐 𝟐
Interprete your result. √𝒓𝟏 + √𝒓𝟐 + √𝒓𝟑 ≤ √𝟐𝑹 (𝒂𝟏 + 𝒂𝟐 + 𝒂𝟑 )𝟐
with equality if and only if 𝛥 ABC is equilateral
Sol. : Differentiating we get and p is the in centre.
𝑥2 𝑦𝑝 𝑑𝑦 Sol. : By the Cauchy-schwarz-inequality.
2
+ 2 = 0; 𝑝 = .
𝑎 +𝜆 𝑏 +𝜆 𝑑𝑥
1 1
𝑎2 + 𝜆 𝑏 2 + 𝜆 1 √𝑟1 + √𝑟2 + √𝑟3 = √𝑎1 𝑟1 √𝑎1 + √𝑎2 𝑟2 √𝑎2 +
𝑆𝑜 𝑡ℎ𝑎𝑡 + = −
1 1 𝑥𝑦(𝑥𝑝 − 𝑦) 1
𝑝𝑦 −𝑥 1 1 1
√𝑎3 𝑟3 √𝑎 ≤ (𝑎1 𝑟1 + 𝑎2 𝑟2 + 𝑎3 𝑟3 )2 (𝑎 + 𝑎 +
3 1 2
𝑎2 − 𝑏 2 1
= 1 𝑎1 𝑟1 𝑎2 𝑟2
1 1 with equality if and only if √ =√
2
+ 𝑎3
) 1 1
=
𝑝𝑦 𝑥 √𝑎 √𝑎
1 2

√𝑎3 𝑟3
So that the differentiable equation the family is 1
, or equivalently, if and only if 𝑎1 2 𝑟1 =
√𝑎
3
(𝑥𝑝 − 𝑦)(𝑥 + 𝑝𝑦) = (𝑎2 − 𝑏 2 )𝑝𝑦 𝑎2 2 𝑟2 = 𝑎3 2 𝑟3 .
To get the orthogonal trajectories, we replace p
1
√𝑟1 + √𝑟2 + √𝑟3
by − 𝑝, so that the differential equation of the 𝑎1 𝑎2 𝑎3 1 1 1
≤( )2( +
family of orthogonal trajectories is 2𝑅 𝑎1 𝑎2
1
𝑥 𝑦 𝑦 1 2
(− − 𝑦) (𝑥 − ) = (𝑎2 − 𝑏 2 ) (− ) + )
𝑝 𝑝 𝑝 𝑎3
𝑎1 𝑎2 𝑎3 1
= ( )2
𝑜𝑟, (𝑥 + 𝑝𝑦)(𝑥𝑝 − 𝑦) = (𝑎2 − 𝑏 2 )𝑝𝑦 2𝑅
1
𝑎2 𝑎3 + 𝑎3 𝑎1 + 𝑎1 𝑎2 2
Which is the same as the original family, so that ×( )
𝑎1 𝑎2 𝑎3
the family is self-orthogonal.
1
=
Let 𝑎2 > 𝑏 2 , then so long as 𝜆 > −𝑏 2, the √2𝑅
family represents ellipses. When 𝜆 lies between 1
−𝑏 2 𝑎𝑛𝑑 − 𝑎2 , it represents hyperbolas with (𝑎2 𝑎3 + 𝑎3 𝑎1 + 𝑎1 𝑎2 )2
the same face and when 𝜆 > −𝑎2 . Those Now, again by the Cauchy-Schwarz inequality,

243
Solving Mathematical Problems

𝑎2 𝑎3 + 𝑎3 𝑎1 + 𝑎1 𝑎2 ∵ (𝑥1 + 𝑥2 )3 = 𝑥1 3 + 3𝑥1 2 𝑥2 + 3𝑥1 𝑥2 2 +


1
𝑥2 3 , 𝑤𝑒 ℎ𝑎𝑣𝑒
≤ (𝑎2 2 + 𝑎3 2 + 𝑎1 2 )2 (𝑎3 2
1
+ 𝑎1 2 + 𝑎2 2 )2
1
𝑥1 3 + 𝑥2 3 = (𝑥1 + 𝑥2 )3 − 3𝑥1 2 𝑥2 − 3𝑥1 𝑥2 2
= (𝑎1 2 + 𝑎2 2 + 𝑎3 2 ) with equality if and only
2 = (𝑎 + 𝑑)3 − 3𝑥1 𝑥2 (𝑎1 + 𝑥2 )
𝑎 𝑎 𝑎 𝑎 +𝑎 +𝑎
of 𝑎2 = 𝑎3 = 𝑎1 = (𝑎2 +𝑎3 +𝑎1 ) = 1 or
3 1 2 3 1 2 = (𝑎 + 𝑑)3 − 3(𝑎𝑑 − 𝑏𝑐)(𝑎 + 𝑑)
equivalently if and only if 𝑎1 = 𝑎2 = 𝑎3 . = (𝑎 + 𝑑)(𝑎2 + 2𝑎𝑑 + 𝑑2
1
− 3𝑎𝑑 + 3𝑏𝑐)
Thus, we have √𝑟1 + √𝑟2 + √𝑟3 ≤ (𝑎1 2 +
√2𝑅
1 = (𝑎 + 𝑑)(𝑎2 − 𝑎𝑑 + 𝑑2 + 3𝑏𝑐)
𝑎2 2 + 𝑎3 2 ) with equality if and only if 𝑎1 2 𝑟1 =
2
= 𝑎3 + 𝑑3 + 3𝑎𝑏𝑐 + 3𝑏𝑐𝑑.
𝑎2 2 𝑟2 = 𝑎3 2 𝑟3 and 𝑎1 = 𝑎2 = 𝑎3 , i.e. 𝑎1 =
𝑎2 = 𝑎3 and 𝑟1 = 𝑟2 = 𝑟3 . Furthermore, 𝑥1 3 + 𝑥2 3 = (𝑎𝑑 − 𝑏𝑐)3, and
that the proof is complete.

MODEL TEST PAPER - 5


Q3. Let f is a function on the positive integers
Q1. Show that 𝒙𝒏 + 𝒚𝒏 = 𝒛𝒏 where n is an which satisfies f(2k) = 2f(k)- 1, f(k +1) = 2f(k) + 1
nteger >2 has no solution in integers x, y, z and also an arbitrary positive integer whose
with 0 ≤ x ≤ n; 0 ≤ y ≤ n. binary representation is
Sol. : We assume that without loss of generality 𝒂 = 𝒂𝒏 𝒂𝒏−𝟏 … 𝒂𝟐 𝒂𝟏 𝒂𝟎 = 𝒂𝒏 𝟐𝒏 +
x ≤ y, then z is an integer > y so that 𝒂𝒏−𝟏 𝟐𝒏−𝟏 + ⋯ + 𝒂𝟏 𝟐 +
𝑧 𝑛 > (𝑦 + 1)𝑛 = 𝑦 𝑛 + 𝑛𝑦 𝑛−1 + 𝑛𝑐2 𝑦 𝑛−2 + ⋯ 𝒂𝟎 . 𝑺𝒉𝒐𝒘 𝒕𝒉𝒂𝒕 𝒇(𝒂) = 𝒃𝒏 𝟐𝒏 + 𝒃𝒏−𝟏 𝟐𝒏−𝟏 +
+ ⋯ > 𝑦 𝑛 + 𝑛𝑦 𝑛−1 ⋯ 𝒃𝟏 𝟐 + 𝒃𝟎 , 𝒘𝒉𝒆𝒓𝒆 𝒃𝟏 = 𝟏, 𝒂𝒊 = 𝟏; =
> 𝑦 𝑛 + 𝑦 𝑛 𝑤ℎ𝑒𝑛 𝑛 > 𝑦. −𝟏, 𝒂𝒊 = 𝟎.

Sol. : We will induct on the number of digits in


> 𝑥 𝑛 + 𝑦 𝑛 . 𝑠𝑖𝑛𝑐𝑒 𝑦 > 𝑥.
the binary representation of a, The result is true
So that 𝑧 𝑛 > 𝑥 𝑛 + 𝑦 𝑛 𝑜𝑟, 𝑧 𝑛 ≠ 𝑥 𝑛 + 𝑦 𝑛 . for a = 1, so suppose it holds whenever a has
lower that k +1 digits. Now consider an integer
Q2. Suppose 𝒙𝟏 𝒂𝒏𝒅 𝒙𝟐 be the roots of the
a with k+ 1 digits (in base 2). Say
equation
𝒙𝟐 − (𝒂 + 𝒅)𝒙 + (𝒂𝒅 − 𝒃𝒄) = 𝑎 = 𝑎𝑘 𝑎𝑘 − 1 … . 𝑎2 𝑎1 𝑎0 . 𝐼𝑓 𝑎0 = 0, 𝑡ℎ𝑒𝑛
𝟎. 𝑺𝒉𝒐𝒘 𝒕𝒉𝒂𝒕 𝒙𝟏 𝟑 𝒂𝒏𝒅 𝒙𝟐 𝟑 are the roots of
𝑎 = 2(𝑎𝑘 𝑎𝑘 − 1 … . 𝑎1 ). 𝑓(𝑎) =
𝒚𝟐 − (𝒂𝟐 + 𝒅𝟐 + 𝟑𝒂𝒃𝒄 + 𝟑𝒃𝒄𝒅)𝒚 + (𝒂𝒅 − 2𝑓(𝑎𝑘 … . 𝑎1 ) − 1 = 2[𝑏𝑘 2𝑘−1 + ⋯ + 𝑏2 2 +
𝒃𝒄)𝟑 = 𝟎. 𝑏1 ] − 1 = 𝑏𝑘 2𝑘 + ⋯ + 𝑏2 22 + 𝑏1 2 + 𝑏0

Sol. : We know that 𝑥1 + 𝑥2 = 𝑎 + 𝑑, 𝑥1 𝑥2 = And the result holds. If 𝑎0 = 1, 𝑡ℎ𝑒𝑛


𝑎𝑑 − 𝑏𝑐
𝑎 = 2(𝑎𝑘 𝑎𝑘 − 1 … . 𝑎1 ) + 1. 𝑓(𝑎) =
2𝑓(𝑎𝑘 … . 𝑎1 ) + 1

244
Solving Mathematical Problems

2(𝑝𝑘 2𝑘−1 + ⋯ + 𝑏1 ) + 1 = 𝑏𝑘 2𝑘 + ⋯ + 𝑏1 2 +
𝑏0 and again the result holds.

Q4. Let 0 < 𝒙𝒊 < 𝜋, 𝑖 = 1, … , 𝑛 𝑎𝑛𝑑 𝑠𝑒𝑡 𝑥 =


𝟏
(𝒙𝟏 + 𝒙𝟐 + ⋯ + 𝒙𝒏 ).
𝒏
𝐴1 𝐶1 + 𝐶1 𝐵2 + 𝐵2 𝐶2 > 𝐴1 𝐴2 + 𝐴2 𝐶2
𝒏 𝐬𝐢𝐧 𝒙𝒊 𝐬𝐢𝐧 𝒙 𝒏
Prove that 𝜫
( 𝒙 ) ≤ ( 𝒙 )
𝒊
𝐴2 𝐶2 + 𝐶2 𝐵3 + 𝐵2 𝐵4 + 𝐵4 𝐶2 > 𝐴2 𝐴2 + 𝐴2 𝐶3
Sol. : The prob. Is equivalent to proving that
𝐴3 𝐶2 + 𝐶2 𝐵5 + 𝐵5 𝐶4 > 𝐴2 𝐴4 + 𝐴4 𝐶4
𝑛
sin 𝑥𝑖 sin 𝑥 𝐴4 𝐶4 + 𝐶4 𝐵6 + 𝐵6 𝐵7 + 𝐵7 𝐶5 > 𝐴4 𝐴5 + 𝐴5 𝐶5
∑ log ≤ 𝑛 log .
𝑥𝑖 𝑥
𝑖=1
𝐴5 𝐶5 + 𝐶5 𝐵1 + 𝐵1 𝐶5 > 𝐴5 𝐴1 + 𝐴1 𝐶1
𝑆 𝑖𝑛𝑡
Consider the function f(t) = log 𝑡
. It is Adding, we find that the perimeter of the outer
straight forward matter to show that f is polygon > perimeter of the interior polygon.
concave (𝑓 ′′ (𝑡) < 0)𝑜𝑛 𝑡ℎ𝑒 𝑖𝑛𝑡𝑒𝑟𝑣𝑎𝑙 (0, 𝜋). The Proof is quite general and holds in the
So, general care.
𝑥1 +𝑥2
𝑓( ) ≥ {𝑓(𝑘1 ) + 𝑓(𝑘2 )}. In a manner Q6. Let 𝑨𝒊 (𝒙𝒊 , 𝒚𝒊 )(𝒊 = 𝟏, 𝟐, … , 𝒌) be the k
2
completely analogous it follows that points in a plane and log P(x, y) and point in
𝑥1 +⋯ 𝑥𝑛 𝑓(𝑥1 )+⋯ 𝑓(𝑥𝑛 ) the plane satisfying 𝑷𝑨𝟏 + 𝑷𝑨𝟐 + ⋯ + 𝑷𝑨𝒌 =
𝑓( ) ≥ . Direct substitution
𝑛 𝑛 𝒌. Show that the equation of the locus of P is
into this inequality completes the proof. f(x, y) = k where f(x, y) is a convex function of x
sin 𝑥 1 sin 𝑥1 sin 𝑥𝑛 and y. Show also that if 𝑸𝟏 , 𝑸𝟐 , … . , 𝑸𝒌 , one the
log ≥ (log + ⋯ + log ). angles, which
𝑥 𝑛 𝑥1 𝑥𝑛
𝑷𝑨𝟏 , 𝑷𝑨𝟐 , … , 𝑷𝑨𝒌 make with the normal to
this locus at a point P, then ∑𝒌𝒊=𝟏 𝐬𝐢𝐧 𝑸𝒊 = 𝟎.
Q5. An m- sided polygon is inside an n-side
polygons (n > m ). Prove that perimeter of the Discuss the special cases when k = 1, 2,.
m- sided polygon is loss than the perimeter of
Sol. : The equation of the locus is given by f(x, y)
the n-sided polygon.
𝑘
Sol. : Consider the fig. where the pentagon is = ∑{(𝑥 − 𝑥𝑖 )2 + (𝑦 − 𝑦𝑖 )2 }
inside a heptagon. 𝑖=1

𝑘
𝑑𝑓
𝑆𝑜 𝑡ℎ𝑎𝑡 = ∑(𝑥 − 𝑥𝑖 )[(𝑥 − 𝑥𝑖 )2
𝑑𝑥
𝑖=1
+ (𝑦 − 𝑦𝑖 )2 ] − 𝑦2

245
Solving Mathematical Problems
𝑘 cos 𝜙1 + cos 𝜙2 + ⋯ + cos 𝜙𝑘 = 0
𝑑2 𝑓 2
1
2 ]−2
= ∑[(𝑥 − 𝑥𝑖 ) + (𝑦 − 𝑦𝑖 )
𝑑𝑥 2 Where 𝜙1 , 𝜙2 , … , 𝜙𝑘 are the angles which there
𝑖=1
− (𝑥 − 𝑥𝑖 )2 [(𝑥 − 𝑥𝑖 )2 radius vectors make with the tangent to the k-
3
+ (𝑦 − 𝑦𝑖 )2 ]2 ellipse at P. It 𝜃1 , 𝜃2 , … , 𝜃𝑘 are the angle which
there radius vectors make with the normal at P,
𝑘
3 then we get,
= ∑[(𝑥 − 𝑥𝑖 )2 + (𝑦 − 𝑦𝑖 )2 ]−2 [(𝑦 − 𝑦𝑖 )2 ]
𝑖=1 sin 𝜃1 + sin 𝜃2 + ⋯ + sin 𝜃𝑘 = 0.
𝑘
If k = 1, it gives sin 𝜃 = 0 i.e. the normal to a
= ∑{𝐴𝑖 (𝑦 − 𝑦𝑖 )}2 circle at every point coincides with the radius.
𝑖=1

𝑘 If k = 2, it gives sin 𝜃1 + sin 𝜃2 = 0 i.e. normal


𝑑2 𝑓 3
at any point of an ellipse makes equal angles
𝑎𝑛𝑑 = − ∑[(𝑥 − 𝑥𝑖 )2 + (𝑦 − 𝑦𝑖 )2 ]2 (𝑥
𝑑𝑥𝑑𝑦 with the lies the point to the foci.
𝑖=1
− 𝑥𝑖 )(𝑦 − 𝑦𝑖 )
𝑘

= − ∑ 𝐴𝑖 (𝑥 − 𝑥𝑖 )𝐴𝑖 (𝑦 − 𝑦𝑖 ). Q7. Find the point with in a triangle, the sum


𝑖=1 of which distances from the three vertices is
3 minimum. Find also the minimum distance in
𝑤ℎ𝑒𝑟𝑒 𝐴𝑖 = {(𝑥 − 𝑥𝑖 )2 + (𝑦 − 𝑦𝑖 )2 }4 terms of the lengths of the sides of the
triangle.
By using Cauchy-Schwarz is equality we find
2 Sol. : (Draw the figure yourself) Suppose we put
𝑑2 𝑓 𝑑2 𝑓 𝑑2 𝑓 𝑑2 𝑓 𝑑2 𝑓 three small pulleys O at A, B, C and a smooth
> 0, > 0, − − ( ) > 0.
𝑑𝑥 2 𝑑𝑦 2 𝑑𝑥 2 𝑑𝑦 2 𝑑𝑥𝑑𝑦 ring at P. A string passes through the ring and
over pulleys and equal weights are attached at
So that f(x, y) is a convex function and f(x, y) = k
the ends of the strings hanging over pulleys at
is closed convex curve.
A. B. C. in the position of equilibrium, the
When k = 1, it reduces to the ordinary circle. potential energy is minimum so that sum of the
lengths of the three strings within the triangle is
When k = 2, it reduces to the ordinary ellipse minimum.
which is defined as the locus of a point, the sum
of whose distances from two fixed points in a However if the string is smooth, the tensions in
plane (called foci is constant). the three string are equal and by lamis the
orem, sines of the three angles APB, BPC, CPA
When k is a positive integer > 2, its locus will be are equal, so that each of three angles is 120°.
called a k-ellipse.
Thus the point P which is chosen such that PA +
Since 𝑟1 + 𝑟2 + ⋯ + 𝑟𝑘 = 𝑘 PB + PC is minimum is such that AB, BC, CA
𝑑𝑟1 𝑑𝑟2 𝑑𝑟𝑘 subtend equal angles of 120° at point P. The
+ +⋯+ = 0. point P will be called steiner’s point of the
𝑑𝑠 𝑑𝑠 𝑑𝑠
triangle.

246
Solving Mathematical Problems

There are other methods of solving this


problem, but we have given this method to
illustrate Horos physical principles can also help Q8. If B(m, n) is a beta function with
us in solving purely mathematical problems. parameters m, n then Prove that

𝒅
If PA = x, PB = y, PC = z, then from the figure. 𝐥𝐨𝐠 𝑩(𝒎, 𝒏) ≤ (𝒎 − 𝟏) 𝒅𝒎 𝒊𝒏 𝑩(𝒎, 𝒏) +
(𝒏 −
𝑎2 = 𝑦 2 + 𝑧 2 + 𝑦 2 |⎸𝑏 2 = 𝑧 2 + 𝑥 2 + 𝑥𝑧|⎸𝑐 2 𝒅
𝟏) 𝒅𝒏 𝒊𝒏 𝑩(𝒎, 𝒏)𝒂𝒏𝒅 𝒊𝒇 𝒎, 𝒏 𝒂𝒓𝒆 𝒊𝒏𝒕𝒆𝒈𝒆𝒓𝒔.
= 𝑥 2 + 𝑦 2 + 𝑥𝑦

𝑧 2 3𝑧 2 𝑥 2 3𝑥 2
(𝒎 − 𝟏)! (𝒏 − 𝟏)!
= (𝑦 + 2) + | = (𝑧 + 2) + | = 𝐥𝐨𝐠 ≤ −(𝒎 − 𝟏)
4 4 (𝒎 + 𝒏 − 𝟏)!
𝑦 2 3𝑦 2
(𝑥 + 2 ) + 4 𝟏 𝟏
[ + + … 𝟏𝒎 ]
𝒎+𝒏−𝟏 𝒎+𝒏−𝟐
Also if area of the triangle is 𝛥, then
𝟏 𝟏 𝟏
1 1 −(𝒏 − 𝟏) [ + +⋯ ]
𝛥 = 𝑦𝑧 sin 120° + 𝑧𝑥 sin 120° 𝒎+𝒏−𝟏 𝒎+𝒏−𝟐 𝒏
2 2
1 Sol. : The entropy of any probability distribution
+ 𝑥𝑦 sin 120°
2 with density function f(x) over [0, 1] is given by
√3
= (𝑥𝑦 + 𝑦𝑧 + 𝑧𝑥) × 1
4 − ∫ 𝑓(𝑥) log 𝑓(𝑥) 𝑑𝑥
0
𝑎2 + 𝑏 2 + 𝑐 2 = 2(𝑥 2 + 𝑦 2 + 𝑧 2 ) + (𝑥𝑦 + 𝑦𝑧 +
𝑧𝑥) The entropy is maximum for the uniform
distribution. So that for any probability density
= 2 {(𝑥 + 𝑦 + 𝑧)2 − 2(𝑥𝑦 + 𝑦𝑧 + 𝑧𝑥)} +
function f(x). We have
(𝑥𝑦 + 𝑦𝑧 + 𝑧𝑥)
1 1
= 2(𝑥 + 𝑦 + 𝑧)2 − 3(𝑥𝑦 + 𝑦𝑧 + 𝑧𝑥) − ∫ 𝑓(𝑥) log 𝑓(𝑥) 𝑑𝑥 ≤ − ∫ 1. log 1 𝑑𝑥
0 0
4𝛥
= 2(𝑥 + 𝑦 + 𝑧)2 − 3 × 1
√3 𝑜𝑟, ∫ 𝑓(𝑥) log 𝑓(𝑥) 𝑑𝑥 ≥ 0
0
2 2 2 2
𝑜𝑟, 2(𝑥 + 𝑦 + 𝑧) = 𝑎 + 𝑏 + 𝑐 + 4√3𝛥
We can get any number of inequalities by taking
𝐿 =𝑥+𝑦+𝑧 = different density functions over the interval [0,
1). In particular if we take.
1
2 2 2 2
(𝑎 + 𝑏 + 𝑐 + 4√3𝛥 )√𝑠(𝑠 − 𝑎)(𝑠 − 𝑏)(𝑠 − 𝑐) 1
[ ] 𝑓(𝑥) = 𝑥 𝑚−1 (1 − 𝑥)𝑛−1
2 𝐵(𝑚, 𝑛)

𝑎+𝑏+𝑐 1
1
𝑤ℎ𝑒𝑟𝑒 𝑠 = 𝑤𝑒 𝑔𝑒𝑡, ∫ 𝑓(𝑥) [log + (𝑚 − 1) log 𝑥
2 0 𝐵(𝑚, 𝑛)
Which gives the minimum sum of distance from + (𝑛 − 1) log(1 − 𝑥)]
A, B, C.

247
Solving Mathematical Problems

𝑜𝑟, − log 𝐵(𝑚, 𝑛) 𝛤 ′ (𝑚 + 𝑛) 𝛤(𝑚)


1 −
𝑚−1 𝛤(𝑚 + 𝑛) 𝛤(𝑚)
+ ∫ 𝑥 𝑚−1 (1 1 1
𝐵(𝑚, 𝑛) 0 = +
− 𝑥)𝑛−1 log 𝑥 𝑑𝑥 𝑚+𝑚−1 𝑛+𝑚−2
1
𝑛 − 1 1 𝑚−1 +⋯+
+ ∫ 𝑥 (1 𝑚
𝐵(𝑚, 𝑛) 0
− 𝑥)𝑛−1 log(1 − 𝑥) 𝑑𝑥 ≥ 0. 𝛤 ′ (𝑚 + 𝑛) 𝛤(𝑛)

𝛤(𝑚 + 𝑛) 𝛤(𝑛)
1
𝑁𝑜𝑤, 𝐵(𝑚, 𝑛) = ∫ 𝑥 𝑚−1 (1 − 𝑥)𝑛−1 𝑑𝑥. 1 1
= +
0 𝑚+𝑚−1 𝑛+𝑚−2
1
Differentiating partially with respect to m, n we +⋯+
𝑛
get.
Substituting there values, we get the second
1
𝑑 inequality to be proved.
𝐵(𝑚, 𝑛) = ∫ 𝑥 𝑚−1 (1 − 𝑥)𝑛−1 log 𝑥 𝑑𝑥
𝑑𝑚 0

1
𝑑
𝐵(𝑚, 𝑛) = ∫ 𝑥 𝑚−1 (1 − 𝑥)𝑛−1 log(1 𝒙𝒙
𝑑𝑛 0
Q9. Does 𝒚 = 𝒙𝒙 represent a function of x?
− 𝑥) 𝑑𝑥 Can you find its derivative and area under this
curve?
So that the above inequality becomes
Sol. : 𝑦 = 𝑥 𝑥 is a function of x for all positive
𝑑 𝑥
− log 𝐵(𝑚, 𝑛) + (𝑚 − 1) log 𝐵(𝑚, 𝑛) values of x. However y =𝑥 𝑥 has two
𝑑𝑚 (𝑥)
𝑑 interpretations viz. =𝑥 (𝑥) or (𝑥 2 )𝑥 and these
+ (𝑛 − 1) log 𝐵 (𝑚, 𝑛) ≥
𝑑𝑛 are different. Similarly if x is raised to power
(2𝑛−2)!
Which was the inequality to be proved, so that which consists of n x’s, we can have (𝑛−1)
we
can restore uniqueness by defining the function
𝑇(𝑚)𝑇(𝑛) 𝑇 ′ (𝑚) recursively by 𝑢0 (𝑥) = 1, 𝑢1 (𝑥) =
𝑙𝑜𝑔 − (𝑚 − 1) {
𝑇(𝑚 + 𝑛) 𝑇(𝑚) 𝑥 𝑢0 (𝑥) , 𝑢2 (𝑥) = 𝑥 𝑢1 (𝑥) , … . 𝑢𝑛 (𝑥) =
′ (𝑚
𝑇 + 𝑛)
− } 𝑥 𝑢𝑛−1 (𝑥) … ..
𝑇(𝑚 + 𝑛)
𝑇 ′ (𝑛) 𝑇 ′ (𝑚 + 𝑛) 𝑆𝑜 𝑡ℎ𝑎𝑡 log 𝑢𝑛 (𝑥) =
− (𝑛 − 1) [ − ]
𝑇(𝑛) 𝑇(𝑚 + 𝑛) 𝑢𝑛−1 (𝑥) log 𝑥 ; log 𝑢𝑛−1 (𝑥) = 𝑢𝑛−1 (𝑥) log 𝑥.
≤0
𝑢𝑛 (𝑥)
𝑜𝑟 log = {𝑢𝑛−1 (𝑥) − 𝑢𝑛−2 (𝑥)} log 𝑥
𝑁𝑜𝑤 𝑇(𝑁) = (𝑁 − 1)𝑇(𝑛 − 1) 𝑢𝑛−1 (𝑥)

𝑇 ′ (𝑛) 1 𝑇 ′ (𝑁 − 1) 𝐼𝑓 𝑥 > 1, 𝑢1 (𝑥) > 𝑥, 𝑢2 (𝑥) > 𝑢, 𝑢3 (𝑥) >


𝑠𝑜 𝑡ℎ𝑎𝑡 = + 𝑢2 (𝑥) … 𝑢𝑛 (𝑥) > 𝑢𝑛−1 (𝑥) … and the sequence
𝑇(𝑛) 𝑁 − 1 𝑇(𝑛 − 1)
can be a divengent sequence. If 𝑥 < 1, 𝑢0 (𝑥) =
𝑇ℎ𝑢𝑠, 1, 𝑢1 (𝑥) = 𝑥 < 1,

248
Solving Mathematical Problems

𝑢2 (𝑥) Thus as y goes from 0 to 1 x goes from 0 to 1


log = {𝑢1 (2) − 𝑢0 (𝑥)} log 𝑥 1
𝑢1 (𝑥) and as y goes from 1 to∞ , x goes from 1 to 𝑐 𝑒
= (𝑥 − 1) log 𝑥 > 0 1
and then from 𝑐 𝑒 𝑡𝑜 1.
𝑆𝑜 𝑡ℎ𝑎𝑡 𝑢2 (𝑥) > 𝑢1 (𝑥).
For every value of y, there is a unique value of x,
𝑢3 (𝑥) 1
𝐴𝑔𝑎𝑖𝑛 log = {𝑢2 (𝑥) − 𝑢1 (𝑥)} log 𝑥 < 0. but for every value of x > 1, but <𝑐 𝑒 , there are
𝑢2 (𝑥)
two values of y at one of which the derivative is
𝑆𝑜 𝑡ℎ𝑎𝑡 𝑢3 (𝑥) < 𝑢2 (𝑥). positive and at the other the derivative is
1

In general 𝑢𝑛 (𝑥) ≷ 𝑢𝑛−1 (𝑥) according as negative. When x = <𝑐 𝑒 there is only one value
𝑢𝑛−1 (𝑥) ≷ 𝑢𝑛−1 (𝑥). of y i.e. c and at this point the tangent is parallel
to the axis of y.
Here we have made use of the fact that

𝑢𝑛 (𝑥)
log = {𝑢𝑛−1 (𝑥) − 𝑢𝑛−2 (𝑥)} log 𝑥
𝑢𝑛−2 (𝑥)

So that 𝑢𝑛 ≷ 𝑢𝑛−2 (𝑥) according as 𝑢𝑛−1 (𝑥) ≷


𝑢𝑛−1 (𝑥)

Thus the sequence {𝑢2𝑛 (𝑥)} is monotonic


decreasing and the sequence {𝑢2𝑛 − (𝑘)} is
monotonic increasing.

𝑢𝑛 (𝑥) However for every value of x, between 0 and 1,


Also 𝑢𝑛−2 (𝑥)
= 𝑥 𝑢𝑛−1 (𝑥) − 𝑢𝑛−3 (𝑥)
there is only one value of y lying between 0 and
1 and the derivative at all points is positive and
If the sequence {𝑢𝑛 (𝑥)} convenges to y, then
we are concerned with only the values of y
1 𝑑𝑦 when x lies between 0 and 1.
𝑦 = 𝑥 𝑥 ⟹ log 𝑦 = 𝑦 log 𝑥 ⟹
𝑦 𝑑𝑥
𝑑𝑦 𝑦 Now area under the curve up to ‘x’ < 1 is
= log 𝑥 +
𝑑𝑥 𝑥 𝑥 𝑦 𝑦 1
2 ∫ 𝑦 𝑑𝑥 = 𝑥𝑦 − ∫ 𝑥 𝑑𝑦 = 𝑥𝑦 − ∫ 𝑦 𝑑𝑦 𝑦
𝑦 𝑦 𝑦2 0 0 0
𝑑𝑦 𝑥 𝑥 2
⟹ = = = 𝑦
𝑑𝑥 1 − log 𝑥 1 − 𝑦 log 𝑥 1 − log 𝑦 1
𝑦 = 𝑦 4+1 − ∫ 𝑦1/𝑦 𝑑𝑦.
0
log 𝑦 1
Lt = −∞, Lt 𝑦 4 = 0. Which can be numerically evaluated.
𝑦→0 𝑦 𝑦→0

log 𝑦 1 1
Lt = Lt = 0 ⟹ 𝐿𝑡 𝑦 4 = 𝑒 0 = 1
𝑦→∞ 𝑦 𝑦→∞ 𝑦 Q10. Let E be the ellipse with centre at again O
whose major and minor axis are 2a and 2b
respectively. If 𝜃 be the acute angle of which E

249
Solving Mathematical Problems

is cut by a circle with centre at the origin (i.e. 𝜃 𝑎2 − 𝑏 2


∴ 𝜃 = tan−1 ( ).
is the acute angle of intersection). Prove that 2𝑎𝑏
the maximum possible value of 𝜃 is

𝒂𝟐 − 𝒃𝟐
𝐭𝐚𝐧 ( ). MODEL TEST PAPER – 6
𝟐𝒂𝒃
Q1. Find all solutions in integers of the
equations : 𝒙𝟖 + 𝒚𝟖 = 𝟏𝟗𝟖𝟒; 𝝁𝟑 + 𝒗𝟔 + 𝝎𝟖 =
Sol : 𝑡𝑎𝑛𝑔𝑒𝑛𝑡 ≡ 𝑥𝑥1 + 𝑦𝑦1 = 𝑟 2 𝟏𝟗𝟖𝟓 .
𝑑𝑦 𝑑𝑦 𝑥
𝑠𝑙𝑜𝑝𝑒 ≡ 𝑥1 + 𝑦1 𝑑𝑥 = 0, ⟹ = − 𝑦1 = 𝑚1 Hence on otherwise find solution of 𝒂𝟑 + 𝒃𝟑 =
𝑑𝑥 1
𝒄𝟑 + 𝒅𝟑 with a, b, c, d as different positive
𝑥𝑥1 𝑦𝑦1 𝑥1 𝑏1 𝑑𝑦 integers.
𝑡𝑎𝑛𝑔𝑒𝑛𝑡 𝐸 ≡ 2
+ 2 = 1, ⟹ + 2 .
𝑎 𝑏 𝑎1 𝑏 𝑑𝑥
=0 Sol. : 18 = 1, but 1983 is not a perfect cube.

𝑥1 28 = 256 𝑎𝑛𝑑 1984 − 256 = 1728 = 123 is a


𝑑𝑦 𝑎 2 𝑥1 𝑏 2
⟹ = − 𝑦 = − . = 𝑚2. perfect cube.
𝑑𝑥 1 𝑦1 𝑎1
𝑏2
∴Only solution in integers of the first equation
𝑥 𝑥 𝑏2 is x = 12, y = 2.
𝑚 −𝑚2 − 1+ 1 2
1 𝑦1 𝑦1 𝑎
Now, tan 𝜃 = ⎸ 1+𝑚 ⎸ = ⎸ 𝑥 ⎸
1 𝑚2 ( 1 )𝑏2
1+ 1
𝑦
𝑎
Next we try v= 1; 𝜔= 1; v = 1; 𝜔 = 2; v = 1; 𝜔= 3;
v = 2; 𝜔 = 2; v = 1; 𝜔 = 2; v = 2; 𝜔 = 3.
𝑎2 − 𝑏 2
𝑥𝑦 2 We find that 123 + 28 + 16 = 1985 𝑎𝑛𝑑 103 +
| 𝑎2 + ( 𝑦 ) 𝑏 2 |
= 1 28 + 36 = 1985
𝑎1
| − (𝑦 ) |
1 ∴ 123 + 28 + 16 = 103 + 36 + 28 = 1985
give the only solutions.

From the above 123 + 16 = 103 + 36

𝑜𝑟, 123 + 13 = 103 + 93 = 1729

Q2. For what real values of a does the


𝒆𝒒𝒙 . 𝒂𝒙𝟐 𝒕 𝒙 + 𝒂 − 𝟏 = 𝟎 have two distinct
𝑥 𝑎
Now, for max angle 𝑦1 = 𝑏 . real root 𝛼 and 𝛽. Satisfying the inequality
1
𝟏 𝟏
⎸ 𝜶 − 𝜷 ⎸ > 1 and ‘a’ belongs the domain of
𝑎2 −𝑏2
𝑎2 definition of the function
𝑎 + 2 .𝑏2
2
𝑎2 −𝑏 2 𝑎 2 −𝑏2
𝑏
∴tan 𝜃 = | 𝑎 | = |−𝑎𝑏−𝑎𝑏| = 2𝑎𝑏

𝑏

250
Solving Mathematical Problems

𝟏 𝟏𝟏 ∴ No. of member playing Cricket = 𝐶1 + 𝑡𝑐 +


𝒇(𝒂) = 𝟏 + √𝒂 − 𝟏𝟎 , ∣. ∣ being the greatest
[𝒂+ ]
𝟐
𝑡𝑏𝑐 + 𝑏 = 25 + 12 + 10 = 47.
integer.

Sol. : 𝐷 > 0; 1 − 4𝑎 (𝑎 − 1) > 0, ⟹ 1 − 4𝑎2 +


4𝑎 > 0.

1−√2 1+√2
⟹ 4𝑎2 − 4𝑎 − 1 < 0, 2
<𝛼< 2

1 1
⎸ |𝛼 − 𝛽 ⎸| > 1. ⟹ ∣ 𝛽 − 𝛼 ∣> ∣ 𝛼𝛽 ∣, ⟹
(𝛼 + 𝛽)2 − 4𝛼𝛽 > 𝛼 2 𝛽2

∴ 0 < a < 615 for f(a) to be defined 0 ≤ a+ ½ ≮1.


Q4. Let 𝑷𝒏 (𝒙) be the polynomial𝑷𝒏 (𝒙) = 𝟏 +
11 6
∴ reqd. value of a, a [ , ].
10 5
𝟐𝒙 + 𝟑𝒙𝟐 + ⋯ + (𝒙 + 𝟏)𝒙𝒏 . Show that 𝑷𝒏 (𝒙),
has no real zero if n is even and exactly one
real zero if m is odd and this zero lies between
-1 and 0.
Q3. In a club of 80 numbers, 10 members play
name of Tennis, Badminton and Cricket, 30 Sol. : When x > 0, 𝑃𝑛 (𝑥) > 0 𝑎𝑛𝑑 𝑠𝑜 𝑃𝑛 (𝑥) = 0
members play exactly one of these three can have no positive real root.
games and 30 members play exactly two of
these games. 45 members play at least one of Now, 𝑃𝑛 (𝑥) = 1 + 2𝑥 + 3𝑥 2 + ⋯ + (𝑥 + 1)𝑥 𝑛
the games among Tennis and Badminton,
whereas 18 members play both Tennis and 𝑥𝑃𝑛 (𝑥) = 𝑥 + 2𝑥 2 + 3𝑥 3 + ⋯ + 𝑛𝑥 𝑛 +
Badminton. Determine the number of Cricket (𝑛 + 2)𝑥 𝑛+1
playing members.
(1 − 𝑥)𝑃𝑛 (𝑥) = 1 + 𝑥 + 𝑥 2 + ⋯ + 𝑥 𝑛 − (𝑛 +
Sol. : Members playing either Tennis (T) or 1)𝑥 𝑛+1
Cricket (C) or Badminton (B). Winner , no. of
1 − (𝑛 + 2)𝑥 𝑛+1 + (𝑛 + 1)𝑥 𝑛+2
members playing exactly one of these three 𝑃𝑛 (𝑥) =
(1 − 𝑥)2
games = 𝑡1 + 𝑏1 + 𝑐1 = 30.No. of members
playing exactly two of there three games = tb For negative values of x, 𝑃𝑛 (𝑥) will vanish
+bc +ct = 30 and no. of members playing whenever
three games = tbc = 70 – 30- 30 = 10.
𝑓(𝑥) ≡ 1 − (𝑛 + 2)𝑥 𝑛+1 + (𝑛 + 1)𝑥 𝑛+2 = 0.
∵ 45 members are playing at least Tennis and
Badminton. 𝑓(𝑥) ≡ 1 − (𝑛 − 2)(−1)𝑛+1 𝑥 𝑛+1 + (𝑛 +
1)(−1)𝑛+2 𝑥 𝑛+2
𝐶1 = 70 − 45 = 25 𝑎𝑛𝑑 𝑏𝑐 + 𝑡𝑐 = 30 − 𝑡𝑏 =
30 − 18 = 12 If n is even, there is no changes of sing in this
expression and so there is no negative real root
[∵ 𝑡𝑏 = 18(𝑤𝑖𝑛𝑛𝑒𝑟)]
also. If n is odd , there is one change of sign
there can be one negative real root.

251
Solving Mathematical Problems

In this care f(-1) = 1 – (n +2) - (n+ 1) = -2n – 2 <0, Sol. : Using the functional equation again and
f(0) = 1 >0. As such when n is odd, the real root again
lies between 0 and -1.
𝑔(𝑥) = 𝑓(2𝑥 , 0) = 𝑓(2𝑥+1 , −2𝑥+1 ) =
𝑓(0, −2𝑥+3 )

Q5. Show that the set S= {. 𝟐, 𝟐𝟑 , 𝟐𝟓 , … … } is = 𝑓(−2𝑥+4 , −2𝑥+4 ) = 𝑓(−2𝑥+4 , 0) =


such that sum of any subset of element cannot 𝑓(−2𝑥+7 , 2𝑥+7 ) = 𝑓(0, 2𝑥+9 ) =
be a perfect square. 𝑓(2𝑥+10 , 2𝑥+10 )

Sol. : Let N = 2𝑠1 + 2𝑠2 + ⋯ + 2𝑠𝑘 = 2𝑠1 (1 + = 𝑓(2𝑥+12 , 0) = 𝑔(𝑥 + 12) . So that g(x) is a
2𝑠1 −𝑠2 + ⋯ + 2𝑠𝑘 −𝑠1 ) periodic function with period 12.

Without loss of generality, we can assume 𝑆1 < Q8. If a(x), b(x), c(x) and d(x) are polynomials is
𝑆2 < 𝑆3 … < 𝑆𝑘 , so that the expression within x, show that
breaketer is an odd number which may or may 𝒙 𝒙
not be a perfect square. ∫ 𝒂(𝒙) 𝒄(𝒙)𝒅𝒙 ∫ 𝒃(𝒙) 𝒅(𝒙)𝒅𝒙
𝟏 𝟏
𝒙 𝒙
− ∫ 𝒂(𝒙)𝒅(𝒙) 𝒅𝒙 ∫ 𝒃(𝒙) 𝒄(𝒙)𝒅𝒙 𝒊𝒔 𝒅𝒊𝒗𝒊𝒔𝒊𝒃𝒍𝒆 𝒃𝒚 (𝒙
𝟏 𝟏
Q6. Decide whether the following statements − 𝟏) . 𝟒

are true or false. There exist number


𝒂𝟏 , 𝒂𝟐 , … , 𝒂𝒏 such that 𝒇(𝒙) ≡ 𝒂, 𝐜𝐨𝐬 𝒙 + Sol. : Denote the expression in question by F(x).
𝒂𝟐 𝐜𝐨𝐬 𝟐𝒙 + … + 𝒂𝒏 𝐜𝐨𝐬 𝒏𝒙 > 0 for all x. Notice that F(x) is a polynomial in x. Also notice
that F(1) = 0 and therefore (x- 1) is a factor of
Sol. : Suppose the statement is true, then F(x).
2𝛱
Because F is a polynomial, we know that
∫ (𝑎1 cos 𝑥 + 𝑎2 cos 2𝑥 + ⋯ + 𝑎𝑛 cos 𝑛𝑥)𝑑𝑥
0 (𝑥 − 1)4 is a root of F(x) = 0 if and only if F(t)=
> 0. 0. We can compute fundamental theorem.
sin 2𝑥 𝑥 𝑥 𝑥
𝑜𝑟, [𝑎1 sin 𝑥 + 𝑎2 +⋯ 𝐹 ′ (𝑥) = 𝑎𝑐 ∫ 𝑏𝑑 + 𝑏𝑑 ∫ 𝑎𝑐 − 𝑎𝑑 ∫ 𝑏𝑐
2
1 1 1
sin 𝑛𝑥 2𝛱 𝑥
+ 𝑎𝑛 ] > 0.
𝑛 0 − 𝑏𝑐 ∫ 𝑎𝑑.
1
But L.H.S. = 0, so that there is a contradiction
(Note that F’(x) = 0 and hence (𝑥 − 1)2 is a root
and the given statement is false.
of F(x) = 0) the derivative F’’ and F’’’ are done in
a similar manner; it turns out that

Q7. Let f (x, y) be a function satisfying the F’’’ (1) = [(𝑎𝑐)1 𝑏𝑑 + (𝑏𝑑)1 𝑎𝑐 − (𝑎𝑑)1 𝑏𝑐 −
functional equation f(x, y) = f(2x+ 2y; 2- 2x).For (𝑏𝑐)1 𝑎𝑑]𝑥=1 = 0
all real numbers x, y. Define g(x) by g(x) =
This completes the proof.
f(𝟐𝒙 , 𝟎). Prove that g(X) is a periodic function
with period 12, i.e. show that g(x +12) = g(x).

252
Solving Mathematical Problems

Q9. The medians of two sides of a triangle concurrent. Into how many regions would the
meet at right angle. The two sides have lengths plane be divided, if n elliptic curves are drawn?
a and b units. Find condition on a and b for this
to be possible and express that length of the Sol. : One curve divides the plane into two
third side is terms of a and b. regions. Two curves divide the plane into 6
region and three curves into 14 region, so that if
Sol. : Let G be the centroid of the triangle since (R (n) is the number of regions, then R(1) = 2,
it trisects the medians. R(2) = 6, R(3) = 14.

AG = 2x, GD = x Now, suppose n curves have already drawn and


(x +1) curves is now drawn. It cuts each of the
BG = 2y, GE = y previous n curves in 4 distinct points and there
Then using Pythagorean theorem, we get from 4n points divide its perimeter into 4n across.
the figure.

4(𝑥 2 + 𝑦 2 ) = 𝑐 2 Each of these cuts across one of the earthier 𝑅𝑛


𝑏2 𝑎2 region resulting from the previous n curves
4𝑥 2 + 𝑦 2 = 4
, 4𝑦 2 + 𝑥2 = 4 dividing into two pieces gives rise to 4𝑛 new
regions so that.
Eliminating 𝑥 2 , 𝑦 2 we get,
𝑅𝑛+1 = 𝑅𝑛 + 4𝑛
2
𝑎2 + 𝑏 2 𝑎2 + 𝑏 2
𝑐 = 4 = .
20 5 𝑜𝑟, 𝑅𝑛 = 𝑅𝑛−1 + 4(𝑛 − 1) = 𝑅𝑛−2 + 4(𝑛 −
2) + 4(𝑛 − 1) = ⋯
It is also clear that 𝑐 2 < 𝑏 2 , 𝑐 2 < 𝑎2 so that c is
𝑎 2 +𝑏2 𝑎 2 +𝑏2
the smallest side and < 𝑎2 , < 𝑏2 , = 𝑅2 + 4(2 + 3 + ⋯ + ̅̅̅̅̅̅̅
𝑛 − 1 ) = 𝑅1 +
5 5
𝑜𝑟, 𝑏 2 < 4𝑎2 , 𝑎2 < 4𝑏 , 2 4(1 + 2 + ⋯ + 𝑛 − 1)

𝑏 𝑎 1 𝑏 (𝑛 − 1)𝑛
< 2, < 2, < < 2. = 2 + 4( ) = 2𝑛2 − 2𝑛 + 2.
𝑎 𝑏 2 𝑎 2

Q10. A number of elliptic curves are drawn in MODEL TEST PAPER - 7


the planes any two of them intersecting in 4
points and no three of the curves are Q1. The real number 𝜇, 𝜈, 𝜔 satisfy 0 <𝜇 <1; 0
<𝜈 < 1; 0 <𝜔 <1 prove that at least one of the

253
Solving Mathematical Problems

3 numbers 𝝁(𝟏 − 𝝂), 𝝂(𝟏 − 𝝎), 𝝎(𝟏 − 1 1 1


𝟏
= −
𝝁) 𝒊𝒔 ≤ . 𝑥𝑘 + 1 𝑥𝑘 𝑥𝑘+1
𝟒
1000 1000
Sol. : Let 𝑃 = 𝜇(1 − 𝜈), 𝑄 = 𝜈(1 − 𝜔), 𝑅 = 1 1 1 1 1
∑ = ∑( − )= −
𝜔(1 − 𝜇) 𝑥𝑘+1 𝑥𝑘 𝑥𝑘+1 𝑥1 𝑥1001
𝑘=1 𝑘=1

𝑇ℎ𝑒𝑛 𝑃𝑄𝑅 = 𝜇(1 − 𝜇)𝜈(1 − 𝜈)𝜔(1 − 𝜔) Now 𝑥𝑘+1 . > 𝑥𝑘 > 𝑥𝑘 (𝑥𝑘 ) is a positive
1
1 1 1 monotonic increasing sequence and {𝑥 } is a
𝑏𝑢𝑡𝜇(1 − 𝜇) ≤ 4 , 𝜈(1 − 𝜈) ≤ 4 , 𝜔(1 − 𝜔) ≤ 4 𝑘
positive monotonic decreasing sequence.
1 3
𝑆𝑜 𝑡ℎ𝑎𝑡 𝑃𝑄𝑅 ≤ ( ) . 1 3 21
4 𝑥1 = , 𝑥2 = , 𝑥3 = , 𝑥 > 1, 𝑤ℎ𝑒𝑛 𝑘 > 2
2 4 16 𝑘
1
And that one of P, Q, R ≤ 4. 1 1
∴0<𝑥 < 1, 𝑥 = 2.
1001 1

1 1
Integral part of = is unity.
𝑥1 𝑥1001
Q2. Find a six digits number which is multiplied
by the factor 6, if the final 3 digit are removed 1 1
In fact the integral part of 𝑥 = 𝑥 is unity for all
1 𝑘
and placed (without changing their order) at
k > 2.
the beginning.

So. : Let N be the number, A be the number


consisting of the first 3 digits and B be the Q4. Prove that in an acute angled
number consisting of last 3 digits. triangle𝐜𝐨𝐬 𝑨 𝐜𝐨𝐬 𝑩 𝐜𝐨𝐬 𝑪 ≤ 𝟖. Deduce that
𝟏

Then N = 1000A+ B and 6N = 1000B + A


𝟏 + 𝐜𝐨𝐬 𝑨 + 𝐜𝐨𝐬 𝑩 + 𝐜𝐨𝐬 𝑪
≥ 𝟏𝟎.
So that 1000B + A = 6(1000A+ B) 𝟐 𝐜𝐨𝐬 𝑨 𝐜𝐨𝐬 𝑩 𝐜𝐨𝐬 𝑪
𝛱
Or, 994B = 5999A Sol. : Let y = log 𝑐𝑜𝑠 𝑥 , 𝑜 < 𝑥 < 2

Or, 142B = 857A 𝑑𝑦 𝑑2 𝑦


= − tan 𝑥 , 2 = −𝑠𝑒𝑐 2 𝑥 < 0
𝑑𝑥 𝑑𝑥
We can chose B = 857, A = 142, since there are
co-prime integers N = 142, 857. ⟹ y is a concave function.

log cos 𝐴 cos 𝐵 cos 𝐶 is also a concvave function


and its maximum value subject to A + B + C = 𝜋
Q3. The numerical sequence 𝒙𝟏 , 𝒙𝟐 satisfies 𝜋
arises when 𝐴 = 𝐵 = 𝐶 = .
𝟏 3
𝒙𝟏 = 𝟐 , 𝒙𝒌+𝟏 = 𝒙𝒌 𝟐 + 𝒙𝒌 for all natural
number k. Find the integer part of the sum cos 𝐴 cos 𝐵 cos 𝐶 has its maximum value when
𝜋
𝟏 𝟏 𝟏 𝐴 = 𝐵 = 𝐶 = 3.
+ +⋯+
𝒙𝟏 +𝟏 𝒙𝟐 +𝟏 𝒙𝟏𝟎𝟎𝟎 +𝟏
1
1 1 1 1 ∴cos 𝐴 cos 𝐵 cos 𝐶 ≤ 8.
Sol. : 𝑥 = 𝑥𝑘 (𝑥𝑘+1 )
= 𝑥𝑘
−𝑥
𝑘+1 𝑘+1

254
Solving Mathematical Problems

And the equality sign holds only for the case


of the equilateral triangle.

1 + cos 𝐴 + cos 𝐵 + cos 𝐶


cos 𝐴 cos 𝐵 cos 𝐶
1
=
cos 𝐴 cos 𝐵 cos 𝐶
sec 𝐵 sec 𝐶 + sec 𝐶
+
sec 𝐴 + sec 𝐴 sec 𝐵

Using the above inequality and the arithmetic


geometric mean inequality.
∆𝐴𝐷𝐸 = 𝑎𝑟𝑒𝑎 𝑜𝑓 ∆𝐷𝑃𝐵
1 + cos 𝐴 + cos 𝐵 + cos 𝐶
cos 𝐴 cos 𝐵 cos 𝐶 If we add there three equalities and at 𝛥PQR
3 to both sides, we get 𝛥ACE = 𝛥BDF.
≥ 8 + 3 √𝑠𝑒𝑐 2 𝐵 𝑠𝑒𝑐 2 𝐶 𝑠𝑒𝑐 2 𝐴

1 2
=8+3 2 ≥ 8 + 3. (8)3
(cos 𝐴 cos 𝐵 cos 𝐶)3 Q6. Suppose f(x) be differentiable on [0, 1]
= 20. with f(0)= 0 and f(1)= 1. For each positive
integer n, so that there exist distinct points
1+cos 𝐴+cos 𝐵+cos 𝐶
2 cos 𝐴 cos 𝐵 cos 𝐶
≥0
𝒙𝟏 , 𝒙𝟐 , … , 𝒙𝒏 in [0, 1] such that
And the equality sign holds only for the 𝒏
equilateral triangle. 𝟏
∑ = 𝒏.
𝒇′ (𝒙𝟏 )
𝒊=𝟏

Sol. : To help generate ideas, consider the


Q5. Prove that if ABCDEF is a hexagon (not case n = 1, we wish to find 𝑥1 in [0, 1] such
necessarily convex) such that AB ∥DE, BC ∥ EF that
and CD∥ FA, then area of triangle ACE = area of
1
triangle BDF, where signed areas are used. 𝑓′ (𝑥1 )
= 1.

Sol. : BC ∥ EF⟹ area of 𝛥CER = area of 𝛥BRF This is possible by the mean value theorem
since on the interval [0, 1] there is a point 𝑥1
CD ∥ FA ⟹ area of 𝛥 CQA = area of 𝛥FQD
such that 𝑓 ′ (𝑥1 ).
AB ∥ DE ⟹ area of 𝛥 ADE = area of 𝛥 DPB
Consider the case n = 2. Consider the
subintervals [0, x] and [x, 1] where x is save
number between 0 and 1 yet to be determine.

By the mean value theorem, there is on 𝑥1 in


(0, x) and 𝑥2 𝑖𝑛 (𝑥, 1) such that

255
Solving Mathematical Problems

𝑓(𝑥) − 𝑓(0) 1 𝑖−1



𝑓 ′ (𝑥1 ) = 𝑎𝑛𝑑 𝑓 ′ (𝑥2 ) 𝑓 ′ (𝑥𝑖 ) = 𝑐𝑛−𝑐𝑛 =
𝑥−0 𝑖 𝑖−1
𝑓(1) − 𝑓(𝑥) 1 1
𝑠𝑜 𝑡ℎ𝑎𝑡 ∑𝑛𝑖=1 𝑓′ (𝑥 ) = ∑𝑛𝑖=1 𝑛(𝑐𝑖 −
= 𝑛(𝑐𝑖 −𝑐𝑖−1 ) 𝑖
1−𝑥
𝑐𝑖−1 ) = 𝑛.
1 1
𝑇ℎ𝑢𝑠 + =2
𝑓 ′ (𝑥1 ) 𝑓 ′ (𝑥2 )

𝑥 1−𝑥 Q7. Let f(x) be a polynomial with integer co-


𝑖𝑓 𝑎𝑛𝑑 𝑜𝑛𝑙𝑦 𝑖𝑓 + = 2, efficients it is known that f(b) – f(a) = 1 (whose
𝑓(𝑥) 1 − 𝑓(𝑥)
a and b are integers.) Prove that a and b differ
𝑥(1 − 𝑓(𝑥)) + (1 − 𝑥)𝑓(𝑥) = 2𝑓(𝑥) − by unity.
2
2(𝑓(𝑥)) ,
Sol. : 𝐿𝑒𝑡 𝑓(𝑥) = 𝑐0 + 𝑐1 𝑥 + 𝑐2 𝑥 2 + ⋯ + 𝑐𝑛 𝑥 𝑛
𝑥 − 𝑥𝑓(𝑥) + 𝑓(𝑥) − 𝑥𝑓(𝑥) − 2𝑓(𝑥) +
2 𝑇ℎ𝑒𝑛 𝑓(𝑏) − 𝑓(𝑎) = 𝑐1 (𝑏 − 𝑎) + 𝑐2 (𝑏2 −
2(𝑓(𝑥)) = 0 𝑎2 ) + ⋯ + 𝑐𝑛 (𝑏𝑛 − 𝑎𝑛 )
2
𝑥 − 2𝑥 𝑓(𝑥) − 𝑓(𝑥) + 2(𝑓(𝑥)) = 0 = (𝑏 − 𝑎)[𝑐1 + 𝑐2 (𝑏 + 𝑎) + ⋯ + 𝑐𝑛 (𝑏𝑛−1 +
𝑏 𝑛−2 𝑎 + ⋯ + 𝑎𝑛 )]
𝑥(1 − 2𝑓(𝑥)) − 𝑓(𝑥)(1 − 2𝑓(𝑥)) = 0
= (𝑏 − 𝑎)𝐼
[𝑥 − 𝑓(𝑥)][1 − 2𝑓(𝑥)] = 0.
Where I is an integer.
Now had we chosen x in (0, 1) so that 𝑓(𝑥) =
1 Since (b - a)I = 1, b - a can be ≠ 1. So that a and
(this could be done by intermediate value
2
b differ by unity.
theorem), the proof would be complete upon
reversing the previous steps. With this
background we can consider the care to an
arbitrary positive integer x, let 𝑐𝑖 be the 𝒙𝟐 𝒚𝟐
Q8. Prove that the ellipse 𝒂𝟐 + 𝒃𝟐 = 𝟏, the
𝑖
smallest number in [0, 1] such that f(𝑐𝑖 ) = 𝑛 distance between the centre and any normal
(the existence of this number is a does not exceed ∣ a- b∣. Find a point on the
consequence of the intermediate value ellipse normal at which is at a distance of ∣a -b∣
theorem together with the assumption of from center.
continuity). Then
Sol. : Let 𝑃(𝑎 cos 𝜃 , 𝑏 sin 𝜃) represents any pt.
0 < 𝑐1 < 𝑐2 < ⋯ < 𝑐𝑛−1 < 1. 𝐷𝑒𝑓𝑖𝑛𝑒 𝑐0 = at on the given ellipse and the slope of the
0 𝑎𝑛𝑑 𝑐𝑛 = 1 and for each internal (𝑐1 − 𝑑𝑦 𝑏2 𝑥 𝑏 cot 𝜃
tangent at this pt is 𝑑𝑥 = − = − at
1, 𝑐𝑖 )𝑖 = 1, 2, … , 𝑛. chose 𝑥𝑖 such that 𝑓 ′ (𝑥1 ) = 𝑎𝑦 𝑎
𝑓(𝑐𝑖 )−𝑓(𝑐𝑖 −1) P. So, the slope of the normal at the pt. P is
𝑐1 −𝑐𝑖−1 𝑎
tan 𝜃, so that the equation of the normal is
𝑏
(This can be done by the I mean-value
𝑎 tan 𝜃
theorem). Then 𝑦 − 𝑏 sin 𝜃 = (𝑛. 𝑎 cos 𝜃),
𝑏
𝑎 tan 𝜃
⟹ (− )
𝑏

256
Solving Mathematical Problems

𝑥 + 𝑦 + (−𝑏 sin 𝜃 + 𝑎2 sin 𝜃) = 0 ……………….(i) 𝑏 2 − 𝑎2


𝐷𝑚𝑎𝑥 = ⎸ ⎸
The distance from (0, 0) to the line (i) is 𝑎+𝑏
= ⎸𝑏 − 𝑎⎸ 𝑃𝑟𝑜𝑣𝑖𝑛𝑔 𝑡ℎ𝑎𝑡 𝐷
𝑎 sin 𝜃 ≤ ⎸𝑏 − 𝑎⎸
𝑏 sin 𝜃 −
𝐷 = ||⎸ 𝑏 ⎸|
2 + 𝑎2 𝜃
| 𝑏
√ 𝑎
+1 Equality occurs when +𝑎2 𝜃 = 𝑎 𝑜𝑛𝑒 𝑠𝑢𝑐ℎ 𝜃 =
𝑏2 √𝑏 √𝑏 √𝑎
(𝑏 2 − 𝑎) sin 𝜃 tan−1 ,⟹ sin 𝜃 = . cos 𝜃 = ,
√𝑎 𝑎+𝑏 𝑎+𝑏
= |⎸ ⎸|
√𝑏 2 + 𝑎2 𝑡𝑎𝑛2 𝜃
√𝑎 √𝑏
𝑏 2 − 𝑎2 ⟹ 𝑃 ≡ (𝑎 ,𝑏 ).
𝑎+𝑏 𝑎+𝑏
= ⎸| ⎸|
√𝑏 2 𝑐𝑜𝑠𝑒𝑐 2 𝜃 + 𝑎 𝑛 𝑠𝑒𝑐 2 𝜃

For fixed a and b, D is max. when S =


𝑏 2 𝑐𝑜𝑠𝑒𝑐 2 𝜃 + 𝑎2 𝑠𝑒𝑐 2 𝜃 is min. This happens Q9. Let y = f(x) = [x] + √𝒙 − [𝒙] be defined for
𝑑𝑠 all real members x where [x] denote the
when 𝑑𝜃 = −2𝑏2 × 𝑐𝑜𝑠𝑒𝑐 𝜃(𝑐𝑜𝑠𝑒𝑐𝜃 𝑐𝑜𝑡𝜃) +
greatest integer of x. Sketch f(x) is the range -5
2𝑎2 sec 𝜃 (𝑠𝑒𝑐𝜃 𝑡𝑎𝑛𝜃)
≤ x ≤ 5. Also show that given any real number
2𝑏 2 𝑐𝑜𝑠𝜃 2𝑎2 sin 𝜃 𝒚𝟎 there is a real number 𝒙𝟎 such that 𝒚𝟎 =
= − +
𝑠𝑖𝑛3 𝜃 𝑐𝑜𝑠 3 𝜃 𝒇(𝒙𝟎 ).
2(𝑏 2 𝑐𝑜𝑠 4 𝜃 − 𝑎2 𝑠𝑖𝑛4 𝜃)
= − Sol. :𝑦 = 𝑓(𝑥) = [𝑥] + √𝑛 − [𝑥] = [𝑥] + √{𝑥}
𝑠𝑖𝑛3 𝜃 𝑐𝑜𝑠 3 𝜃
= 0, where {x} = fractional part of x(i) we know, 0 ≤
𝑏
{x} < 1,
i.e., when 𝑡𝑎𝑛2 𝜃 = 𝑎.
⟹√{𝑥} ≥ {𝑥}, ⟹ [𝑥] + √{𝑥} ≥ [𝑥] + {𝑥}
𝑑2 𝑠 2𝑏 2 sin 𝜃 6𝑏 2 𝑐𝑜𝑠 2 𝜃 2𝑎2 𝑐𝑜𝑠𝜃
∴ = + +
𝑑𝜃 2 𝑠𝑖𝑛3 𝜃 𝑠𝑖𝑛4 𝜃 𝑐𝑜𝑠 3 𝜃 ⟹ 𝑓(𝑥) ≥
2 2
𝑏𝑎 𝑠𝑖𝑛 𝜃 𝑥 𝑒𝑞𝑢𝑎𝑙𝑖𝑡𝑦 ℎ𝑜𝑙𝑑𝑠 𝑤ℎ𝑒𝑛 𝑥 𝑡𝑎𝑥𝑒𝑠 𝑖𝑛𝑡𝑒𝑔𝑟𝑎𝑙 𝑣𝑎𝑙𝑢𝑒.
+
𝑐𝑜𝑠 4 𝜃

2𝑏 2 6𝑏 2 𝑐𝑜𝑠 2 𝜃 2𝑎2 6𝑎2 𝑠𝑖𝑛2 𝜃


= + + +
𝑠𝑖𝑛2 𝜃 𝑠𝑖𝑛4 𝜃 𝑐𝑜𝑠 2 𝜃 𝑐𝑜𝑠 4 𝜃
> 0. ∀ 𝜃.
𝑏
∴ 𝑡𝑎𝑛2 𝜃 = 𝑎 corresponds to the men of S given
by

S min = |⎸𝑏 2 (1 + 𝑐𝑜𝑠 2 𝜃) + 𝑎2 (1 + 𝑎2 𝜃)| ⎸ +


𝑏
𝑎2 𝜃 = 𝑎

𝑎 𝑏 ∴ The graph of f(x) will be: (ii) again,


= 𝑏 2 (1 + ) + 𝑎2 (1 + ) = (𝑎 + 𝑏)2
𝑏 𝑎
y = f(x) = [x] + √{𝑥}.
Hence the max. value of the reqd. distance is

257
Solving Mathematical Problems

As 0 ≤ {x} < 1, hence √{𝑥} is always real, MODEL TEST PAPER – 8

⟹ f(x) is always real, ⟹ there is 𝑎 𝑥0 , ∀ 𝑦0 ∊ Q1. Let 𝒂𝟏 , 𝒂𝟐 , … , 𝒂𝒏 are real (n > 1) and 𝑨 +
𝟏
𝑅 𝑎𝑛𝑑 𝑥0 𝑎𝑙𝑠𝑜 ∊ 𝑅, 𝑠𝑢𝑐ℎ 𝑡ℎ𝑎𝑡 𝑦0 = 𝑓(𝑥0 ). ∑𝒏𝒊=𝟏 𝒂𝒊 𝟐 < (∑𝒏𝒊=𝟏 𝒂𝒊 )𝟐 , 𝒔𝒉𝒐𝒘 𝒕𝒉𝒂𝒕 𝑨 <
𝒏−𝟏
2𝒂𝒊 𝒂𝒋 𝒇𝒐𝒓 𝟏 ≤ 𝒊 ≤ 𝒋 ≤ 𝒖

Sol. : By the Cauchy-Schwarz inequality


Q10. On [0, 1], let f have a continuous
derivative satisfying 0 < f1 (t) ≤ 1 also f(0) = 0, 𝑛 2

then show that (∑ 𝑎𝑖 ) = [(𝑎1 + 𝑎2 ) + ⋯ + 𝑎𝑛 ]2


𝑖=1
𝟐
𝒕
[∫𝟎 𝒇(𝒕) 𝒅𝒕] ≤
𝒕
∫𝟎 [𝒇(𝒕)]𝟑 𝒅𝒕. < (1 + ⋯ + 1)[(𝑎1 + 𝑎1 )2 + ⋯
+ 𝑎𝑛 2 ]
Sol. : For 0 ≤ 𝑥 ≤ 1, 𝑛

𝑥 2 𝑥 3 = (𝑛 − 1) [∑ 𝑎𝑖 2 + 2𝑎1 𝑎2 ]
Let F(x) ≡[∫0 𝑓(𝑡) 𝑑𝑡] − ∫0 (𝑓(𝑡)) 𝑑𝑡. 𝑖=1

𝑡ℎ𝑒𝑛 𝐹(0) = 0 𝑎𝑛𝑑 𝐹 ′ (𝑥) = 2 This together with the given inequality, implies
that
𝑥
[∫ 𝑓(𝑡) 𝑑𝑡] 𝑓(𝑥) − [𝑓(𝑥)]3 𝑛 𝑛 2
0 1
𝐴 < − (∑ 𝑎𝑖 2 ) − (∑ 𝑎𝑖 )
𝑥 𝑛−1
𝑖=1 𝑖=1
= 𝑓(𝑥) [2 ∫ 𝑓(𝑡) 𝑑𝑡 − [𝑓(𝑥)]2 ]
0 𝑛 𝑛
1 2
We do know that f(x) ≥ 0 for 0 < x < 1 (since < − (∑ 𝑎𝑖 ) − [(𝑛 − 1) [∑ 𝑎𝑖 2
𝑛−1
we are given f(0) = 0 and f(x) > 0) however, 𝑖=1 𝑖=1

it is not clear that the second factor in the last


expression for F’ is nonnegative therefore, let + 2𝑎1 𝑎2 ]]

𝑥
𝐺(𝑥) = 2 ∫0 𝑓(𝑡)𝑑𝑡 − [𝑓(𝑥)]2 , 0 ≤ 𝑥 ≤ 1
= 2𝑎1 𝑎2 .
𝑡ℎ𝑒𝑛 𝐺(0) = 0, 𝑎𝑛𝑑 𝐺 ′ (𝑥) = 2𝑓(𝑥) −
In a similar manner, 𝐴 < 2𝑎𝑖 𝑎𝑗 𝑓𝑜𝑟 1 ≤ 𝑖 ≤
2𝑓(𝑥)𝑓 ′ (𝑥)
𝑗 ≤ 𝑛.
= 2𝑓(𝑥)[1 − 𝑓 ′ (𝑥)] ≥ 0.

(The last inequality holds because f(x) ≥ 0


Q2. Let m be an integer >1 and define the
and by hypothesis 1- f’(x) ≥ 0). It follows
from their arguments that F(x) ≥ 0 for all x, numbers 𝒎𝟏 , 𝒎𝟐 … by postulating 𝒎𝟏 =
0≤ x ≤ 1; in particular, F(1) ≥ 0 and the proof 𝒎, 𝒎𝒊+𝟏 = 𝒎𝒊 𝟐 − 𝒎𝒊+𝟏 , 𝒊 = 𝟏, 𝟐, 𝟑, …. Show
is complete). that none of the numbers 𝒎𝟐 , 𝒎𝟑 , 𝒎𝟒 is
divisible by m.

258
Solving Mathematical Problems

Sol. : 𝑚2 = 𝑚2 − 𝑚 + 1 = 𝑚(𝑚 − 1) + 1 = random in there envelopes. Find the number


𝑚2 + 1 where 𝑞2 is an integer. As such 𝑚1 is of ways in which every latter goes in a wrong
not divisible by m. envelopes.

Now we assume 𝑚𝑘 = 𝑚𝑞𝑘 + 1, 𝑤ℎ𝑒𝑛 𝑞𝑘 is an Sol. : Let 𝑢𝑛 be the required number of ways
integer and prove that 𝑚𝑘 + 1 = 𝑚𝑞𝑘+1 + 1, consider two letters 𝑎1 , 𝑎2 and corresponding
when 𝑞𝑘+1 is also an integer. envelops 𝐴1 , 𝐴2 then for all the letters to be in
wrong envelops, two mutually exclusive
𝑚𝑘 + 1 = 𝑚𝑘 (𝑚𝑘−1 ) + 1 = (𝑚𝑞𝑘 + 1)𝑚𝑞𝑘 + possibilities arise.
1 = 𝑚𝑞𝑘+1 + 1
(i) 𝑎𝑖 goes into 𝐴2 𝑎𝑛𝑑 𝑎2 goes into 𝐴1
𝑤ℎ𝑜𝑠𝑒 𝑞𝑘+1 = 𝑞𝑘 (𝑚𝑞𝑘+1 ).
and the remaining (n -2)

letters are placed wrongly into the other (n-2)


Q3. Consider the Fibonacci sequence 0, 1, 2, 3, envelops. The number of ways for this is 𝑢𝑛+2 .
5, 8, 13, 21, 34, 55, 89, 144, ….. in which every
(ii) 𝑎𝑖 goes into A2 but a2 does not go
terms beginning from the third awards is the
into A1. In this case (n −1) letters go
sum of two proceeding terms. Show that there
wrongly into (n−1) enevlops. This
exists a number terminating with 4 zeroes in
can be done in 𝑢𝑛−1 ways.
the first 100, 000, 001 terms.
Thus the number of ways in which 𝑎1 goes into
Sol. : Suppose we divide by 7, the sequence of
𝐴2 𝑎𝑛𝑑 all the letters are in the wrong
remainders is 0, 1, 1, 2, 3, 5, 1, 6, 0, 6, 6, 5, 4, 2,
envelops is 𝑢𝑛−1 + 𝑢𝑛−2 . We get the same
6, 0, 1, 1……similarly if we divide by any positive
number of ways in which 𝑎1 goes into
integer n, we shall get a sequence of
𝐴3 , 𝐴4 , … , 𝐴𝑛 so that the total number of ways
remainders.
is (𝑛 − 1)(𝑢𝑛−1 + 𝑢𝑛−2 ), but those must be
Since, the original sequence is determined by same as 𝑢𝑛 , so that 𝑢𝑛 = (𝑛 − 1)(𝑢𝑛−1 +
two consecutive terms, this sequence of 𝑢𝑛−2 ).
remainders is also determined by two
This is a difference equation of the second
consecutive remainders. Since the sequence
order. To solve this, we write it as
starts with 0 and since there are only n possible
different remainder and 𝑛2 possible pairs of 𝑢𝑛 − 𝑛𝑢𝑛−1 = −(𝑢𝑛−1 − ̅̅̅̅̅̅̅
𝑛 − 1 𝑢𝑛−2 )
remainder i.e. pair (0, 1) will occur in every
block of 𝑛2 terms. If we choose n = 104 , there 𝑢𝑛 − ̅̅̅̅̅̅̅̅
𝑛 − 1 𝑢𝑛−1 = −(𝑢𝑛−2 − ̅̅̅̅̅̅̅
𝑛 − 2 𝑢𝑛−2 )
will be a remainder 0 in every block of 108
terms and the corresponding member in the ……………………………………………………………………….
original sequence will be divisible by 104 and so
will terminate in 4 zeroes. 𝑢3 − 3𝑢2 = −(𝑢2 − 2𝑢1 ).

Q4. n letters 𝒂𝟏 , 𝒂𝟐 , … , 𝒂𝒏 are written and Multiplying there equations, we get,


have to be placed in corresponding envelopes
𝑢𝑛 − 𝑛𝑢𝑛−1 = (−1)𝑛−2 (𝑢2 − 2𝑢1 ).
marked 𝑨𝟏 , 𝑨𝟐 , … , 𝑨𝒏 . The letters are placed at

259
Solving Mathematical Problems

Now 𝑢1 = 0, since if there is only one letter and Using similar results, we find that one root lies
𝜋
one envelope there is no way of the latter going between 0 𝑎𝑛𝑑 2 and two roots each lie
into the wrong envelope. Also 𝑢2 = 1, since if between
there are only two letters and two envelopes,
there is one way of putting letters in the wrong 3𝜋 5𝜋 7𝜋 9𝜋 11𝜋 13𝜋 15𝜋 17𝜋 19𝜋
, , , , , , , , ,
envelope and one way of putting there in the 2 2 2 2 2 2 2 2 2
right envelope, so that 21𝜋 23𝜋 25𝜋 27𝜋 29𝜋 31𝜋
, , , , ,− , 𝑡𝑜𝑡𝑎𝑙 16 𝑟𝑜𝑜𝑡𝑠.
2 2 2 2 2 2
𝑢𝑛 + 𝑛𝑢𝑛−1 = (−1)𝑛−2
Thus total number of real roots is 31.
Dividing this equation by n we get,

𝑢𝑛 𝑢𝑛−1 (−1)𝑛−2
− =
𝑛! (𝑛 − 1)! 𝑛! Q6. ABCD is a cyclic quadrilateral and M, N, P, Q
are the mid point of the sides CD, DA, AB, BC
𝑢𝑛−1 𝑢𝑛−2 (−1)𝑛−2
𝑠𝑜 𝑡ℎ𝑎𝑡 − = respectively. MS, NT, PN, QV are perpendicular
(𝑛 − 1) (𝑛 − 2)! (𝑛 − 1)!
to AB, BC, CD, DA respectively. Prove that they
𝑢2 𝑢1 (−1) MN and PQ are each parallel to AC and each
− = 𝟏
2! 1! 2! equal to 𝟐 AC so that MNPQ is a parallelogram.
Adding there together we get

𝑢𝑛 𝑢1 1 1 1 (−1)𝑛−2
− = − + ….+
𝑛! 1! 2! 3! 4! 𝑛!

1 1 (−1)𝑛
𝑢𝑛 = 𝑛! [ − + ⋯+ ]
2! 3! 𝑛!

Q5. Find the number of real roots of the


𝒙
equation 𝐜𝐨𝐬 𝒙 = 𝟓𝟎. Middle point of MP coincides with the middle
𝑥
point of NQ. Let 0 be the centre of the
Sol. : Let 𝑓(𝑥) = cos 𝑥 = 50. circumcircle of ABCD and let PV and intersect at
X.
We have to find the points of intersection 𝑦 =
𝑥
cos 𝑥 𝑎𝑛𝑑 𝑦 = 50 since −1 ≤ cos 𝑥 ≤ 1, the MP ⊥AB, MS ⊥ AB
roots lie between -50 and 50. We also find that
29𝜋
∴ OP ∥ MS
𝑓(14𝜋) > 0, 𝑓 ( ) < 0, 𝑓(15𝜋) < 0
2
Similarly since OM ⊥ CD, PV ⊥ CD
29𝜋 31𝜋
𝑓 (− ) > 0, 𝑓(−15𝜋) < 0, 𝑓 (− ) ∴ ON ∥ PX
2 2
> 0, 𝑓(16𝜋) > 0.
OPVXM is a parallelogram.

260
Solving Mathematical Problems

∴ They middle point of OX coincides with the


middle point of MP. Similarly y be the point of
intersecting of perpendiculars drawn from N
and Q of opposite sides, then by the same
argument the middle point of NQ coincides with
the middle point of OY.

However, the middle points MP and NQ


coincide. Therefore, the middle point of OX and
OY coincide, therefore X and Y coincide.

As such perpendiculars from P, Q, M, N on


opposite sides are concurrent.

Q7. Let P be a point outside a square ABCD.


Find the focus of P if the shortest angle
between two rays starting from P and
including the whole square between them is 𝜃.

Show that the locus is a closed curve consisting


of eight circular arcs. Find the perimeter of this
curve and also the area enclosed by it.

Sol. : The symmetries of the square will also be


Case A. P is a in region 1: The square is
the symmetries of the desired locus, so that the
obviously contained between the rays PC and
desired locus will be symmetrical about the two
PD and so ∠CPD = 𝜃 is the smallest angle
right bisectors of the two pairs of opposite
between the rays for which the rays can
parallel sides as well as about the two
include the square. Since ∠CPD is const., the
diagonals. It will also by unchanged if the locus of P is in region I is obviously the one of
square is rotated about the centre through a segment of a circle (fig. I).
right angle on two right angles or three rt.
Angles. Now three cases arise:
𝜋
(i) 𝜃 = . In this case the locus of P is
2
obviously the semicircle with CD as
diameter for convenience we shall
take the side of the square as of
unit length. The locus in this case is
given by four semi-circle areas (fig.
2). The total areas enclosed = 1 +
𝜋 1 2 𝜋
4. 2 (2) = 1 + 2 and the perimeter

261
Solving Mathematical Problems
1
of the fig (2) locus is 1 + 4 × 𝜋 2 = (4) and the area enclosed under it
in region 1 = area of segment +
2𝜋. 1
(ii)
𝜋
𝜃 > 2 .In this case we shall get fig (3) area of rectangle = 2 𝑟 2 2𝜃 +
1 1 4
and the centre of the circular are 2
1. 𝑥 = 𝑟 2 𝜃 + 2 cot 𝜃 = 4𝑠𝑖𝑛2 𝜃 +
will be written the square. If O is 1 cos 𝜃
.
2 sin 𝜃
the centre of the circle ∠COD = 2𝜋 -
𝜋
2𝜃. If r is the radius of the circle Which approaches 𝜋 as 𝜃 → 2 and →∞ 𝑎𝑠 𝜃 →
1
2𝑟 sin(𝜋 − 𝜃) = 1 𝑜𝑟 𝑟 = 2 sin 𝜃. 0. Also length of the circular arc = r.2.𝜃 =
2𝜋 𝜃
Area enclosed by arc CPD and the = sin 𝜃.
2 sin 𝜃
st. line CD = area of sec OCPD − area
1 Case B. P. lies in region II : The locus in this
of 𝛥 COD = 𝑟 2 (2𝜋 − 2𝜃) −
2
1 2 region on is still the arc QR of a circle whose
𝑟 sin(2𝜋− 2𝜃) = 𝑟 2 𝜋
2 centre will be within the square if 𝜃 > 4 and
1 1 𝜋
(𝜋 − 𝜃 − sin 2𝜃) = fig (5) and outside the square if 𝜃 > . The
2 85𝑖𝑟 2 𝜃 4
(2𝜋 − 2𝜃 − sin 2𝜃). Thus total area square is contained bet can be rays PA and PC
𝜋
enclosed by the locus when 𝜃 > is so that ∠ APC= 𝜃, ∠AQC = 𝜃, ∠ARC = 𝜃.
2
1 𝜋
given by 𝐴(𝜃) = (2𝜋 − 2𝜃 − ∴∠ QCD = 2 − 𝜃, ∠𝐴𝑂′ 𝐶 = 2𝜃, ∠𝑄𝑂′ 𝑅 =
25𝑖𝑛2 𝜃
𝜋
sin 2𝜃) + 1. Now Lt 𝐴(𝜃) = 1 + 2 2∠𝑄𝐶𝑅 = 2∠𝑄𝐶𝐷 = 𝜋 − 2𝜃
1
as expected. Also Lt (2𝜋 −
0→𝜋 2𝑠𝑖𝑛2 𝜃 ∴ ∠QO’C +∠RO’A
2𝜃 − sin 2𝜃)
−2−2 cos 2𝜃 = 2𝜋 - 2𝜃- (𝜋- 20) = 𝜋 but by symmetry there
= Lt =
0→𝜋 4 sin 𝜃 cos 𝜃 angles are equal, so that
4 sin 2𝜃
Lt = 0,
0→𝜋 4(𝑐𝑜𝑠2𝜃 −𝑠𝑖𝑛2 𝜃) 𝜋
∠QO’C = ∠RO’A = 2 . Area of RPQDR = Area
So that Lt [𝐴(𝜃)] = 1.
0→𝜋 of sec.
This is also obvious since in this case the locus 1
QO’R - 2 areas 4QDO’ = 2 𝑟′2
of P coincides with the four sides of the square
itself. Again if P(𝜃) is the length of the perimeter 1 𝜋
4(𝜋−𝜃) (𝜋 − 2𝜃) − 2. 𝑟 ′ 𝑥𝑠𝑖𝑛 (𝜃 − )
𝑃(𝜃) = 4𝑟 (2𝜋 − 2𝜃) = sin 𝜃
. 2 4
1 1
𝜋 = . (𝜋 − 2𝜃)
This approaches 2𝜋 as 𝜃→ as expected. If also 2 2𝑠𝑖𝑛2 𝜃
2 1 1
4(𝜋−𝜃) −4 − cot 𝜃 (sin 𝜃
approaches Lt = Lt = 4, again
0→𝜋 sin 𝜃 0→𝜋 cos 𝜃 √2 sin 𝜃 √2
as expected. Thus we get the results A𝜃 = 1 + 1 1
− cos 𝜃 )=
2
(2𝜋 − 2𝜃 − sin 2𝜃); 𝑃(𝜃) =
4(𝜋−𝜃)
; 𝜃≥ √2 4𝑠𝑖𝑛2 𝜃
2𝑠𝑖𝑛2 𝜃 sin 𝜃
𝜋
. 1 cos 𝜃
2 (𝜋 − 2𝜃) − (sin 𝜃 − cos 𝜃)
2 𝑠𝑖𝑛2 𝜃
1
(iii) When 𝜃 ≤ 2. In this case the locus
in region I is an arc of circle fig.

262
Solving Mathematical Problems

It can be shown that the same results holds 𝜋 𝜋


∴ 𝑓(𝑥) = cos [ 4 {1 − 𝑥 + [𝑥]}] = cos [ 4 {1 −
𝜋
when 𝜃 < , the locus is given in fig (4) and A(𝜃) 𝜋
2 (2𝑚 + 𝛿1 ) + 2𝑚}] = cos { (1 − 𝛿1 )}
4
= 4× 𝑎𝑟𝑒𝑎 𝐴1 + 4 area 𝐴2 + 𝐴3 𝑖 𝐴(𝜃) =
4 2 cos 𝜃 𝜋−2𝜃 2 cos 𝜃
+ + 𝑠𝑖𝑛2 𝜃 − (sin 𝜃 − cos 𝜃) + Let us examine the case when m = 1, [x] =
𝑠𝑖𝑛2 𝜃 𝑠𝑖𝑛𝜃 𝑠𝑖𝑛2 𝜃
10 ≤
𝜋
, then you do yourself. odd.
2
2 >x ≥ 1.

𝝅 ∴ x = 1+ 𝛿, where 0 ≤ 𝛿 <1. 𝛿 = 0, x = 4, 𝛿 →
Q8. Consider the function f(x) = 𝐬𝐢𝐧{ 𝟒 (𝒙 − 𝜋
1, x → 2. f(a) = sin 4 𝛿[𝑥] = 𝑒𝑣𝑒𝑛.
𝝅
[𝒙])} if [x] is odd, x ≥0 = 𝒄𝒐𝒔 { (𝟏 − 𝒙 +
𝟒
[𝒙]}𝒊𝒇 [𝒙] is even x ≥ 0, where [x] denote the 2 ≤ x < 3, x = 2 + 𝛿’ where 0 ≤ 𝛿’ < 1, 𝛿’ = 0,
greatest integer ≤ x, sketch the graph of the x = 2.
function f(x) determine the points of 𝜋
𝛿’ → 1, x → 3, f(x) = cos { 4 (1 − 𝛿1 )}. From the
discontinuities of f(x) and the points where
f(x) is not differentiable. curve we can safely conclude that f(x) is
discontinuous for every odd integral value of
Sol. : Let 2𝑥 > 𝑥 > 2𝑚 − 1, we can write, x = x, hence it is also not differentiable.
(2m- 1)+ 8, where 1 > 8 ≥ 0

∴ [x] = (2m- 1) which is odd.


Q9. Let {𝑪𝒏 } be an infinite sequence of circles
𝜋 𝜋
∴ f(x) = sin ( 4 (𝑥 − [𝑥]) ) = sin [ 4 {(2𝑚 − lying in the positive quadrant of the xy-plane,
𝜋 with strictly decreasing radii and satisfying the
1) + 𝛿 − (2𝑚 − 1)}] = sin 4 𝛿, 𝑤ℎ𝑒𝑟𝑒 1 > 𝛿 ≥
following conditions. Each 𝑪𝒏 touches both
0. the X-axis and Y-axis. Further for all n ≥ 1, the
circle 𝑪𝒏+𝟏 touches the circle 𝑪𝒏 externally. If
Let, 2m +1 > x ≥ 2m.
𝑪𝟏 has radius 10 cm, then show that sum of
𝟐𝟓𝝅
the areas of all these circle is 𝒄𝒎𝟐 .
𝟑√𝟐−𝟒

Sol. : O𝑂1 = 𝑅1 √2.

∴ 𝑂𝑃 = 𝑅1 √2 − 𝑅1

∴ 𝑂𝑄 = 𝑅1 √2 + 𝑅1 = 𝑅1 (√2 + 1).

𝑂𝑄
∴ 𝑅1 = .
√2+1

𝑂𝑃
∴ x = 2m + 𝛿’ 𝑁𝑜𝑤, 𝑂𝑃 = 𝑅2 (√2 + 1), 𝑅2 =
√2 + 1
Where 1 > 𝛿’ ≥ 0. √2 − 1
= 𝑅1 .
√2 + 1
[x] = 2m, which is even.

263
Solving Mathematical Problems

√2 − 1 √2 − 1 Sol. : Consider the function f(x) is tan x on [a,


∴ 𝑅3 = 𝑅2 . = 𝑅1 ( .) b]. According to the mean value theorem
√2 + 1 √2 + 1
there is a point c on (a, b) such that
𝑓(𝑏)− 𝑓(𝑎)
𝑏−𝑎
= 𝑓 ′ (𝑐)

In this case, this means that


tan 𝑏−tan 𝑎
𝑏−𝑎
= 𝑠𝑒𝑐 2 𝑐 - for same c in (a, b).

𝜋
𝑠𝑒𝑐 2 𝑎 < 𝑠𝑒𝑐 2 𝑐 < 𝑠𝑒𝑐 2 𝑒 𝑓𝑜𝑟 … 0 < 𝑎 < 𝑏 <
2

Let 𝑓 ′′ 𝑅 → 𝑅 𝑠𝑢𝑐ℎ 𝑡ℎ𝑎𝑡 𝑓 ′′ (𝑥) ≥ 0, then

𝑥 + 𝑦 𝑓(𝑥) + 𝑓(𝑦)
𝑓 ≤
∴ 𝐴𝑟𝑒𝑎 = 𝜋(𝑅1 2 + 𝑅2 2 + ⋯ + ∞) 2 2
𝑥+𝑦 𝑓(𝑥)+𝑓(𝑦)
4 And if 𝑓 ′′ (𝑥) ≤ 0, 𝑡ℎ𝑒𝑛 𝑓 ( ) >
√2 − 1 √2 − 1 2 2
= 𝜋 {𝑅1 2 + 𝑅2 2 ( ) + 𝑅1 2 ( )
√2 + 1 √2 + 1 For example, for real number a and b,

+ ⋯ + ∞} 𝑥 + 𝑦 2 𝑥2 + 𝑦2
𝑓( ) < .
2 2
2 4
2 √2 − 1 √2 − 1 Because f(x) = 𝑥 2 is a convex function. As
= 𝜋𝑅1 {1 + ( ) +( ) + ⋯ ∞}
√2 + 1 √2 + 1 another example if 0 < x, y < 𝜋

𝑥+𝑦 sin 𝑥 + sin 𝑦


sin ( )≥
1 3 + 2√2 2 2
= 𝜋𝑅1 2 2 = 𝜋𝑅1 2
√2 − 1 4√2 Because f(x) = sin x is a concave function on
1−( )
{ √2 + 1 } (0, 𝜋).

3√2 + 4 𝜋 18 − 16
= 𝜋𝑅1 2 = . 𝑅1 2
8 8 3√2 − 4 MODEL TEST PAPER 9
𝜋 1
= . 100.
4 3√2 − 9 Q1. If n arithmetic means 𝑨𝟏 , 𝑨𝟐 , … , 𝑨𝒏 and n
geometric means 𝑮𝟏 , 𝑮𝟐 , … , 𝑮𝒏 we inscribed
25𝜋
(∵ 𝑅1 = 10 𝑐𝑚) = 𝑠𝑞. 𝑐𝑚. between two positive numbers a and b, show
(3√2 − 4) 𝟏
that 𝒙 (𝑨𝟏 + 𝑨𝟐 + ⋯ 𝑨𝒏 ) ≥ 𝒏√𝑮𝟏 , 𝑮𝟐 , … 𝑮𝒏

Sol. : If d is the common difference of the A.P


𝝅 𝒃−𝒂
Q10. Prove that for 𝟎 ≤ 𝒂 < 𝑏 < , < and r is the common ratio of G.P., then
𝟐 𝒄𝒐𝒔𝟐 𝒂
𝒃−𝒂
𝐭𝐚𝐧 𝒃 − 𝐭𝐚𝐧 𝒂 < 𝒄𝒐𝒔𝟐 𝒃
. 𝑎 + (𝑛 + 1)𝑑 = 𝑏, 𝑎𝑟 𝑛+1 = 𝑏,

264
Solving Mathematical Problems

𝑏−𝑎 1 1
𝑠𝑜 𝑡ℎ𝑎𝑡 𝐴𝑖 + 𝑎 + 𝑖𝑑 = 𝑎 + 𝑖 = 63𝑘 + 7 𝑤ℎ𝑒𝑛 𝑘 + ≤𝑥<𝑘+
𝑛+1 8 4
𝑛+1−𝑖 𝑖
= 𝑎+ 𝑏 1 1
𝑛+1 𝑛+1 = 63𝑘 + 15 𝑤ℎ𝑒𝑛 𝑘 + ≤𝑥<𝑘+
4 2
𝑎 𝑖
𝑎𝑛𝑑 𝐺𝑖 = 𝑎𝑟 𝑖 = 𝑎 ( ) 1
𝑏 𝑛+1 = 63𝑘 + 31 𝑤ℎ𝑒𝑛 𝑘 + ≤𝑥 <𝑘+1
𝑛+1−𝑖 𝑖 2
=𝑎 . 𝑛 .
𝑛+1 𝑏 +1 So that f(x) can take only when six sets of
So that Ai and Gi are the weighted arithmetic values and f(x) = M has no real solution when
and geometric mean between the two M is different from these value.
numbers a and b, when the weights one
(𝑛+1−𝑖) 𝑖
(𝑛+1)
𝑎𝑛𝑑 (𝑛+1) but the weighted arithmetic
mean of two positive numbers ≥ weighted Q3. If a, b, c are the roots of 𝒙𝟑 − 𝒙𝟐 − 𝒙 − 𝟏 =
geometric means. If the weights are the same 𝟎, so that (i) a, b, c are distinct; (ii)
𝒂𝟏𝟗𝟗𝟐 −𝒃𝟏𝟗𝟗𝟐 𝒃𝟏𝟗𝟗𝟐 −𝒄𝟏𝟗𝟗𝟐 𝒄𝟏𝟗𝟗𝟐 −𝒂𝟏𝟗𝟗𝟐
and the equality sign holds only when the two 𝒂−𝒃
+ 𝒃−𝒄
+ 𝒄−𝒂
is an
numbers are equal so that. integer.
𝐴𝑖 ≥ 𝐺𝑖 𝑎𝑛𝑑 𝐴𝑖 = 𝐺𝑖 , 𝑖𝑓 𝑎 = 𝑏, 𝑖 = 1, 2, … 𝑛 Sol. : The polynomial f(x) =𝑥 3 − 𝑥 2 − 𝑥 − 1
then each arithmetic mean ≥ corresponding has only are change of sign but f(-x) =−𝑥 3 −
geometric mean. Again since, we get 𝑥 2 + 𝑥 − 1 has two change of sign. As such by
𝐴1 +𝐴2 +⋯+𝐴𝑛
𝑛
≥ 𝑛√𝐴1 , 𝐴2 , … 𝐴𝑛 ≥ 𝑛√𝐺1 , 𝐺2 , … 𝐺𝑛 Descartes rules of signs f(x) = 0 can have at
and the equality sign holds if a = b. the positive real root and at most two
negative real roots and all the roots can be
real. Now 𝑓(−∞) = −∞ < 0, 𝑓(0) =
−1, 𝑓(1) = −2 < 0, 𝑓(2) = 1 > 0, 𝑓(∞ =
Q2. Let [m] denote the largest integer ≤ m,
∞ > 0). Thus as x goes from −∞ to 0, f(x)
such that the equation [𝒙] + [𝟐𝒙] + [𝟒𝒙] +
goes −∞ 𝑡𝑜 − 1 and as x goes from 0 to ∞,
[𝟖𝒙] + [𝟏𝟔𝒙] + [𝟑𝟐𝒙]= M has no real so has
f(x) goes −1 𝑡𝑜 ∞.
no real solution unless M is of the form 63k or
63k +1 or 63k + 7 or 63k + 15 or 63k+ 31 Again, f’(x)= 3𝑥 2 − 2𝑥 − 1
where, k is any positive integer, negative or
zero. 1
𝑓 ′ (𝑥) = 0 ⟹ 𝑥 = 1 𝑜𝑟, 𝑥 = −
3
Sol. : Let 𝑓(𝑥) = [𝑥] + [2𝑥] + [4𝑥] + [8𝑥] +
1 1 1 1 32
[16𝑥] + [32𝑥] 𝑎𝑛𝑑 𝑓 ( ) = − − + − 1 = <0
3 27 9 3 27
1
Then f(x) = 63k when 𝑘 ≤ 𝑥 < 𝑘 + 32. 1
𝑓 ′′ (1) = 4 > 0. 𝑓 ′′ (− ) = −9 < 0.
2
1 1
= 63𝑘 + 1 𝑤ℎ𝑒𝑛 𝑘 + ≤𝑥<𝑘+ 1
32 16 Thus f(x) has a local maximum at x = − 3
22
1 1 when the local maximum value is − 27 and
= 63𝑘 + 3 𝑤ℎ𝑒𝑛 𝑘 + ≤𝑥<𝑘+
16 8 has a local minimum at 2 = 1

265
Solving Mathematical Problems

Where the local maximum value is -2. Q4. Given any 13 real numbers, show that
there are two of those 𝒂𝒊 , 𝒂𝒋 such that 𝟎 ≤
Then f(x) = 0 has three roots one real and 𝒂𝒊 −𝒂𝒋 √𝟑−𝟏
positive lying between 1 and 2 and the other 𝟏+𝒂𝒊 𝒂𝒋
≤ .
√𝟑+𝟏
two are complex in the negative real part. All
the three roots are distinct. Sol. : Consider the function y = tan x, then as x
𝜋 𝜋
goes from – 2 𝑡𝑜 2 y goes from −∞ 𝑡𝑜 ∞.
For the second part, 𝜋 𝜋
Divide the interval – 𝑡𝑜 into 12 equal parts
2 2
𝑎 𝑛 −𝑏𝑛 𝑏𝑛 −𝑐 𝑛 𝑐 𝑛 −𝑎𝑛 𝜋
Let 𝑥𝑛 = , 𝑦𝑛 , 𝑧𝑛 , each of length 12. If 𝐶1 , 𝐶2 , … . 𝐶12 are given
𝑎−𝑏 𝑏−𝑐 𝑐−𝑎
real numbers then
𝑎𝑛+3 − 𝑏 𝑛+3 tan−1 𝐶1 tan−1 𝐶2 … . tan−1 𝐶12 lie between
𝑠𝑜 𝑡ℎ𝑎𝑡 𝑥𝑛+3 =
𝑎−𝑏 𝜋 𝜋
– 2 , 2 but by the pigon hole principle all the
𝑛 (𝑎2
𝑎 + 𝑎 + 1) − 𝑏 𝑛 (𝑏2 + 𝑏 + 1)
= thirteen cannot be in 12 distinct intervals
𝑎−𝑏
unless at least two of there are in the same
𝑎𝑛+2 − 𝑏 𝑛+2 𝑎𝑛+1 − 𝑏 𝑛+1 𝑎𝑛 − 𝑏 𝑛 interval. Let tan−1 𝑒𝑖 and tan−1 𝑒𝑗 be in the
= + +
𝑎−𝑏 𝑎−𝑏 𝑎−𝑏 same interval, then
= 𝑥𝑛+2 + 𝑥𝑛+1 + 𝑥𝑛 . 0 ≤ 𝑐𝑖 − 𝑐𝑗 ≤ 15°
𝑠𝑖𝑚𝑖𝑙𝑎𝑟𝑙𝑦, 𝑦𝑛+3 = 𝑦𝑛+2 + 𝑦𝑛+1 + 𝑦𝑛
0 ≤ tan−1(𝑐𝑖 − 𝑐𝑗 ) ≤ tan−1 15°
𝑧𝑛+3 = 𝑧𝑛+2 + 𝑧𝑛+1 + 𝑧𝑛 .
tan−1 𝑐𝑖 − tan−1 𝑐𝑗 √3 − 1
0≤ ≤ .
So that (𝑥𝑛+3 + 𝑦𝑛+3 + 𝑧𝑛+3 ) = (𝑥𝑛+2 + −1 −1
1 + tan 𝑐𝑖 tan 𝑐𝑗 √3 + 1
𝑦𝑛+2 + 𝑧𝑛+2 ) + (𝑥𝑛+1 + 𝑦𝑛+1 + 𝑧𝑛+1 ) +
(𝑥𝑛 + 𝑦𝑛 + 𝑧𝑛 )
𝟏
𝑙𝑒𝑡 𝑢𝑛 = 𝑥𝑛 + 𝑦𝑛 + 𝑧𝑛 . Q5. If 𝒙𝟏 , 𝒙𝟐 are positive and 𝒙𝒏+𝟏 = 𝟐 (𝒙𝒏 +
𝒙𝒏−𝟏 ). Show that the sequences, 𝒙𝟏 , 𝒙𝟑 , 𝒙𝟓 ….
𝑠𝑜 𝑡ℎ𝑎𝑡 𝑢𝑛+3 = 𝑢𝑛+2 + 𝑢𝑛+1 + 𝑢𝑛
and 𝒙𝟐 , 𝒙𝟒 , 𝒙𝟔 … .. are the one increasing and
So that if 𝑢𝑛 , 𝑢𝑛+1 , 𝑢𝑛+2 one integers, then the other decreasing and both approach to a
𝟏
𝑢𝑛+1 ℎ𝑎𝑠 𝑎𝑙𝑠𝑜 𝑏𝑒 𝑐𝑎𝑛 𝑖𝑛𝑡𝑒𝑔𝑒𝑟. common limit 𝟐 (𝒙𝟏 + 𝟐𝒙𝟐 ).

Now 𝑢1 = 3, 𝑢2 = 2(𝑎 + 𝑏 + 𝑐) = 2, 𝑢3 = 1
Sol. : 𝑥𝑛+1 = 2 (𝑥𝑛 + 𝑥𝑛−1 )
2(𝑎2 + 𝑏 2 + 𝑐 2 ) + 𝑎𝑏 + 𝑏𝑐 + 𝑐𝑎 = 2[1 + 2] −
1= 5. 1 1
𝑥𝑛−1 − 𝑥𝑛 = (𝑥𝑛−1 − 𝑥𝑛 ) = − (𝑥𝑛 − 𝑥𝑛−1 )
2 2
So that 𝑢1 , 𝑢2 , 𝑢3 are integers, then applying
the result proved earlier in succession are we As such 𝑥𝑛 − 𝑥𝑛−1 and 𝑥𝑛+1 − 𝑥𝑛 have
get 𝑢4 , 𝑢5 , 𝑢6 are all integers. In fact we get opposite signs and if 𝑥𝑛 > 𝑥𝑛−1 𝑡ℎ𝑎𝑛 𝑥𝑛+1 <
the sequence 3, 2, 5, 10, 17, 32, 59, 108, 199, 𝑥𝑛 .
366, ….
1 1
Also 𝑥𝑛 = 2 (𝑥𝑛−1 + 𝑥𝑛−2 ), 𝑥𝑛−1 = 2 (𝑥𝑛−2 +
𝑥𝑛−3 )

266
Solving Mathematical Problems
1 1 1 Also Lt (𝑥2𝑛 + 2 − 𝑥2𝑛 ) = 0, Lt (𝑥2𝑛 +1 +
So that 𝑥𝑛+1 = 2 [2 𝑥𝑛+1 + 2 𝑥𝑛−2 + 𝑥𝑛−1 ] = 𝑛→∞ 𝑛→0
1 −3 1 2−𝑥 2𝑛 −1 ) = 0.
[ 𝑥 + (2𝑥𝑛−1 − 𝑥𝑛−2 )]
2 2 𝑛−1 2
So that both the odd and even numbered
1
= [5𝑥𝑛−1 − 𝑥𝑛−3 ]. sequences are convergent. If 𝑥2𝑛 → 𝑎, 𝑥2𝑛 −1 →
4 1
𝑏, 𝑡ℎ𝑒𝑛 𝑏 = 2 (𝑎 + 𝑏)𝑎, 𝑏 = 𝑎.
1
⟹ 𝑥𝑛+1 − 𝑥𝑛−1 = [𝑥𝑛−1 − 𝑥𝑛−3 ]
4 Thus both sequences approach a common
limit.
So that 𝑥𝑛−1 − 𝑥𝑛−1 has the same sign as
𝑥𝑛−1 − 𝑥𝑛−3 𝒙+𝒚
Q6. Let f; R→ R satisfies 𝒇 ( 𝟐
) <
1 2 𝒇(𝒙)+𝒇(𝒚)
∀𝒙, 𝒚 in an interval (a, b) x ≠ y, show
So that 𝑥𝑛+1 − 𝑥𝑛−1 = (4) (𝑥𝑛−3 − 𝑥𝑛−5 ) = 𝟐
𝟏 𝟏
1 3 that 𝒇 {𝒏 (𝒙𝟏 + 𝒙𝟐 + ⋯ + 𝒙𝒏 )} < 𝒏
(4) (𝑥𝑛−5 − 𝑥𝑛−7 )

{𝒇(𝒙𝟏 ) + 𝒇(𝒙𝟐 ) + ⋯ + 𝒇(𝒙𝒏 )} whenever the


1 2𝑛−2
𝑥2𝑛+1 − 𝑥2𝑛+1 = ( ) (𝑥3 − 𝑥1 ) 𝒙𝒊 ′𝒔 are in (a, b) with 𝒙𝒊 ≠ 𝒙𝒋 for at least are
2
1 2𝑛−1 pair (i, j).
= ( ) (𝑥2 − 𝑥1 )
2 Sol. : Assume the result holds for n = 𝑚𝑖 we
In the same way, will show if holds also for n = 2m. We have

𝑥1 + … . + 𝑥2𝑚
1 2𝑛−4 𝑓( )
𝑥2𝑛 − 𝑥2𝑛−2 = ( ) (𝑥1 − 𝑥2 ) 2𝑚
2
1 2𝑛−4 1 1 𝑥1 + ⋯ + 𝑥𝑚 𝑥𝑚+1 + ⋯ + 𝑥2𝑚
= ( ) (𝑥 − 𝑥2 ) = 𝑓{ ( + )}
2 2 3 2 𝑚 2𝑚
1 𝑥1 + ⋯ + 𝑥𝑚
1 2𝑛−2 ≤ [𝑓 ( )
=( ) (𝑥1 − 𝑥1 ) 2 𝑚
2 𝑥𝑚+1 + ⋯ + 𝑥2𝑚
+ 𝑓( )]
𝑚
Then if 𝑥1 > 𝑥2 , 𝑥2 < 𝑥4 < 𝑥6 < 𝑥8 … < 𝑥𝑛 < 1 (𝑥1 ) + ⋯ + 𝑓(𝑥𝑚 )
and the sequence {𝑥2𝑛 } is a monotonic < {𝑓
2 𝑚
decreasing sequence. Also, 𝑥1 > 𝑥3 > 𝑥5 > 𝑥𝑚+1 + ⋯ + 𝑥2𝑚
𝑥7 … > 𝑥2𝑛−1 > ⋯ and the sequence {𝑥2𝑛+1 } + 𝑓( )}
𝑚
is monotonic decreasing sequence.
𝑓(𝑥1 ) + 𝑓(𝑥2 ) + ⋯ + 𝑓(𝑥2𝑚 )
Similarly, if 𝑥1 < 𝑥2 , the sequence {𝑥2𝑛 } is =
2𝑚
monotonic decreasing and the sequence
{𝑥2𝑛+1 } is monotonic increasing. Thus, by induction, the result holds for 4 all
powers of 2. Now suppose that n >2 and n is
In either case, out of the sequences not a power of 2; i.e. let
{𝑥2𝑛−1 }𝑎𝑛𝑑{𝑥2𝑛 }, one is increasing and the
other is decreasing. 2𝑚−1 < 𝑛 < 2𝑚 for some integer m. Let k =
(𝑥1 +⋯+𝑥𝑛 )
2𝑚 − 𝑛 and set 𝑦𝑖 = 𝑛
for i = 1, 2, …,
k. Then 𝑥1 , 𝑥2 , … . , 𝑘𝑛 , 𝑦1 , … , 𝑦𝑘 are 2𝑚

267
Solving Mathematical Problems

numbers in the interval (a, b) and so our 𝐴 𝐵 𝐶


= 2𝑟 [cot + cot + cot ]
preceding argument implies that 2 2 2

(𝑥1 + ⋯ + 𝑥𝑛 + 𝑦1 + ⋯ + 𝑦𝑘 ) Since A and r are fixed, the perimeter is


𝑓 minimum, when
2𝑚
𝑓(𝑥1 ) + ⋯ + 𝑓(𝑦𝑘 )
< . 𝐵 𝐶
𝐵+𝐶
sin cos
𝐴
2𝑚 cot 2 + cot 2 = 2
= 2
is minimum
𝐵 𝐶 𝐵 𝐶
sin sin sin sin
2 2 2 2
(𝑥1 +⋯+𝑥𝑛 +𝑦1 +⋯+𝑦𝑘 ) 𝐵 𝐶
But note that 𝑓 2𝑚
when sin sin
2 2

𝑥1 + ⋯ + 𝑥𝑛 + 𝑘(𝑥1 + ⋯ + 𝑥𝑛 ) 1
= [cos
𝐵−𝐶
− cos
𝐵+2
]
1
= [cos
𝐵−𝐶 𝐴
− sin ] is
=𝑓 2 2 2 2 2 2
2𝑚 𝐵−𝐶
maximum i.e. when cos is maximum i.e.
2
𝑛(𝑥1 + ⋯ + 𝑥𝑛 ) + (2𝑚 − 𝑛)(𝑥1 + ⋯ + 𝑥𝑛 )
= 𝑓( ) when B = C i.e. when the triangle is isosceles.
𝑛 × 2𝑚

(𝑥1 + ⋯ + 𝑥𝑛 )
=𝑓 .
𝑛 Q8. Suppose f(x) is a real valued differentiable
function defined a [1, ∞) with f(1) = 1. Also
Making this substitution into the last
f(x) satisfies
inequality,
𝟏
(𝑥1 + ⋯ + 𝑥𝑛 ) 𝒇′ (𝒙) = .
𝑓 𝒙𝟐 + 𝒇𝟐 (𝒙)
𝑛
𝑓(𝑥1 ) + ⋯ + 𝑓(𝑥𝑛 ) + 𝑓(𝑦1 ) + ⋯ + 𝑓(𝑦𝑛 ) 𝑷𝒓𝒐𝒗𝒆 𝒕𝒉𝒂𝒕 𝐋𝐢𝐦 𝒇(𝒙) 𝒆𝒙𝒊𝒔𝒕𝒔 𝒂𝒏𝒅 𝒊𝒔 𝒍𝒆𝒔𝒔 𝒕𝒉𝒂𝒏 𝟏
< 𝒙→∞
2𝑚
𝟏
+ 𝝅.
𝑓(𝑥1 ) + ⋯ + 𝑓(𝑥1𝑛 ) + 𝑘𝑓(𝑥1 + ⋯ 𝑥𝑛 )/𝑛 𝟒
=
2𝑚
Sol. : By the fundamental theorem of calculus
𝑚 𝑚
Multiplying each side by 2 yields 2 𝑥
𝑥1 +⋯+𝑥𝑛
𝑓( ) < 𝑓(𝑎1 ) + ⋯ + 𝑓(𝑥𝑛 ) + 𝑓(𝑥) − 𝑓(1) = ∫ 𝑓 ′ (𝑥) 𝑑𝑥
𝑛 1
𝑥 +⋯+𝑥𝑛 𝑓(𝑥 )+⋯+𝑓(𝑥𝑛 )
(2𝑚 − 𝑛)𝑓 ( 1 )< 1 𝑛 .
𝑛
Observe that f(x) is increasing, moreover, f(x)
> 1 for all x > 1.

Q7. Among all triangular having a fixed LA and Since f(1) = 1 and f’(x)>0. Therefore
an inscribed circle of fixed radius r, determine 𝑥
𝑑𝑥 𝑥
𝑑𝑥
the triangle has the least perimeter. 𝑓(𝑥) − 𝑓(1) = ∫ 2 2
< ∫
1 𝑥 + 𝑓 (𝑥) 1 + 𝑥2
Sol. : Perimeter 2S = a + b+ c 𝑥 𝜋 𝜋 𝜋 𝜋
= [tan−1 𝑥] = tan−1 𝑥 − < − < .
𝐵 𝐶 𝐶 𝐴 1 4 2 4 4
= 𝑟 [cot + cot ] + 𝑟 [cot + cot ]
2 2 2 2 Thus f(x) is increasing and bounded above by
𝐴 𝐵 1
+ 𝑟 [cot + cot ] 1 + 4 𝜋, and consequently, Lim 𝑓(𝑥) exists
2 2 𝑥→∞
1
and is less than 1 + 4
𝜋.

268
Solving Mathematical Problems

Q9. A rabbit runs round a circular path with a


certain uniform speed v. A dog follows it from
the origin in such a way that the origin, dog,
rabbit are always on the same straight line and
the dog runs with the same speed v. Show that
the dog will catch the rabbit when it has run
round a quarter of the circle.

Sol. : Consider the semi-circle with centre at


𝑎 𝑎
(0, 2 ) 𝑎𝑛𝑑 𝑟𝑎𝑑𝑖𝑢𝑠 2 and let D be the position
of the dog on the semicircle and R be the
position of the rabbit on the circle, so that
ODR is a straight line .If ∠AOD = 𝜃, ∠ OCD=
2𝜃 and arc OD = arc AR, since the distance MODEL TEST PAPER - 10
moved by the dog = distance moved by the
Q1. Let 𝒙𝒊 >0 for i = 1, 2, …, n. For each non-
rabbit. The semicircle and the circle meet at B 𝟏
and the dog catches the rabbit here. negative integer k, Prove that 𝒏 (𝒙𝟏 𝟐 + ⋯ +
𝒙𝟏 𝒌+𝟏 +⋯+𝒙𝒏 𝒌+𝟏
𝒙𝒏 𝟐 ) ≤ 𝒙𝟏 +⋯+𝒙𝒏

Sol. : Let 𝑥1 + … + 𝑥𝑛 = 1, for if not, replace


𝑥𝑖
𝑥𝑖 by 𝑥𝑖 = .
(𝑥1 +⋯+𝑥𝑛 )

The results holds when k = 0. Let the result


holds for all non-negative integer < k. By
Cauchy Schwarz. Inequality.
𝑛
∑𝑛𝑖=1 𝑥𝑖 𝑘 𝑥𝑖 (𝑘−1)/2
= ∑ 𝑥𝑖 (𝑘+1)/2
𝑛 𝑛
𝑖=1
𝑛 1/2 𝑛 1/2
𝑘+1
𝑥𝑖 𝑘−1
Q10. Draw the graph of y = [sin x]+ sin x, ≤ (∑ 𝑥𝑖 ) (∑ )
𝑛
where [x] denotes the greatest integer 𝑖=1 𝑖=1

function. 𝑥𝑖 𝑘−1
By the inductive assumption, ∑𝑛𝑖=1 𝑛

Sol. : -1 ≤ sin x ≤ 1. ∴ 0 ≤ |sin x| ≤ 0 and [sin ∑𝑛𝑖=1 𝑥𝑖 𝑘
and so continuing from last in
x] = -1, 0, 1, then -1 ≤ [sin x] + |sin x|≤ 2. equation, we have
1 1
𝑛 2 𝑛 𝑘−1 2
𝑥𝑖
(∑ 𝑥𝑖 𝑘+1 ) (∑ )
𝑛2
𝑖=1 𝑖=1
1 1
𝑛 2 𝑛
𝑥𝑖 𝑘 2
≤ (∑ 𝑥𝑖 𝑘+1 ) (∑ )
𝑛
𝑖=1 𝑖=1

269
Solving Mathematical Problems
1 1 there are (n- 1) 𝑓𝑛−1 permutations of this
𝑛 𝑛 2 𝑛
𝑥𝑖 𝑘 𝑥𝑖 𝑘 2 type. Thus the total number of permutations
𝑡ℎ𝑢𝑠 ∑ ≤ (∑ 𝑥𝑖 𝑘+1 ) (∑ ) ,
𝑛 𝑛 in which no elements changes is (n -1)
𝑖=1 𝑖=1 𝑖=1
(𝑓𝑛−2 + 𝑓𝑛−1 ) and this would be sane as 𝑓𝑛 so
1 1
𝑛 𝑛 that
𝑥𝑖 𝑘 2 2
⟹ (∑ ) ≤ (∑ 𝑥𝑖 𝑘+1 ) ,
𝑛 𝑓𝑛 = (𝑛 − 1)(𝑓𝑛−1 + 𝑓𝑛−2 ) Also be definition
𝑖=1 𝑖=1
𝑔𝑛 = 𝑛𝑓𝑛−1 . 𝑠𝑜 𝑡ℎ𝑎𝑡 𝑓𝑛−1 = 𝑛𝑓𝑛 +
Then you do yourself. 𝑛𝑓𝑛−1 , 𝑔𝑛−1 = (𝑛 + 1)𝑓𝑛

∴ 𝑓𝑛+1 − 𝑔𝑛+1 = 𝑛𝑓𝑛−1 − 𝑓𝑛 = 𝑔𝑛 − 𝑓𝑛 =


−(𝑓𝑛 − 𝑔𝑛 )
Q2. Find all positive integer x, y, z, 𝜔 which
satisfy 𝜔! = x!+ y! + z!. = (−1)2 (𝑓𝑛−1 − 𝑔𝑛−1 ) = (−1)3 (𝑓𝑛−2 −
𝑔𝑛−2 )
Sol. : Without less of generality. We can
assume x ≤ y ≤ z. = (−1)𝑛 (𝑓1 − 𝑔1 ) = (−1)𝑛+1 , 𝑠𝑖𝑛𝑐𝑒 𝑓1 =
0, 𝑔1 = 1
It is obvious that 𝜔 ≥ z +1, so that
∴|𝑓𝑛 − 𝑔𝑛 | = 1 or, 𝑓𝑛 − 𝑔𝑛 = 1.
(z+ 1)! 𝜔! = x! + y! +z! ≤ 3z!

So that z +1 ≤3 a, z ≤ 2 and x ≤ 2, y ≤ 2
calculations show that x = y = z = 2 and 𝜔 = Q4. Let 𝒙𝟏 , 𝒙𝟐 , … , 𝒙𝒏 be n real numbers
3 give the only solution. between o and 1. Find the greatest and
smallest values of
Q3. n elements 𝒂𝟏 , 𝒂𝟐 , … , 𝒂𝒏 are permuted
among themselves so that the total number of 𝛿 = ∑𝟏≤𝒊𝒋≤𝒏|𝒙𝒊 − 𝒙𝒋 |
permutations is n! Let 𝒇𝒏 denotes the number
of permutations in which no element remain Sol. : Since we can choose 𝑥1 , 𝑥2 , … , 𝑥𝑛 all
fixed i.e. every element changes its position equal, the minimum value of S is zero. For
and let 𝒈𝒏 denote the number of permutations finding the largest value, we assume without
where only one element remains fixed and the loss of generality that
(n−1) remaining elements change their
position. Show that 𝒇𝒏 𝟐 𝒈𝒏 = 𝟏. 0 ≤ 𝑥1 ≤ 𝑥2 ≤ 𝑥3 ≤ ⋯ ≤ 𝑥𝑛−1 ≤ 𝑥𝑛 ≤ 1

Sol. : By a transformation let 𝑎𝑖 go into 𝑎𝑗 . Then S = (𝑥2 − 𝑥1 ) + [(𝑥3 − 𝑥1 ) + (𝑥3 −


𝑥2 )] + [(𝑥4 − 𝑥1 ) + (𝑥4 − 𝑥2 ) + (𝑥4 − 𝑥3 )] +
Now there are two posibilities either 𝑎𝑗 goes
[(𝑥𝑛 − 𝑥1 ) + ⋯ + (𝑥𝑛 − 𝑥𝑛−1 )]
into a, or 𝑎𝑗 does not go into 𝑎𝑖 .
On the R.H.S., 𝑥1 occurs (n -1) lines with a
In the first case n -2 elements change their negative sign giving a total of – (n- 1) x. Also
positions and there are 𝑓𝑛−2 permutations of
𝑥2 occurs (n-2) lines with a negative sign and
this type corresponding to each j = 1, 2, 3, …, once with a positive sign giving a total of – (n-
n so that this give (n -1) 𝑓𝑛−2 permutations.
3) 𝑥2 .
In the second case there are 𝑓𝑛−1
permutations corresponding to each f so that

270
Solving Mathematical Problems

Proceeding in this way we get 𝑥𝑘 occurs (n - Sol: f(x) ≡ (𝑥 − 𝑎)(𝑥 − 𝑏)(𝑥 − 𝑐)(𝑥 −
k) lines with a negative sign and k -1 line with 𝑑)𝜙(𝑥) + 5 𝑤ℎ𝑒𝑟𝑒 𝜙(x) is a polynomial with
a positive sign. integral coefficients of (n−4)th degree. If f(k) =7,
then
So that S = ∑𝑛𝑘=1[(𝑘 − 1) − (𝑛 − 𝑘)]𝑥𝑘 =
∑𝑛𝑘=1(2𝑘 − 𝑛 − 1)𝑥𝑘 (k -a) (k -b) (k -c) (k -d) 𝜙(k) = 2

Case (i) n is even say n = 2m, then S = Now, four distinct integral factors of 2 are 1, -1,
∑2𝑚
𝑘=1(2𝑘 − 2𝑚 − 1)𝑥𝑘 2, -2 since (k -a), (k -b), (k -c), (k -d) are distinct,
their product cannot be 2.
If k = 1, 2, …, m the coefficients are negative
and since we want to maximize S, we choose
𝑥1 = 𝑥2 = ⋯ = 𝑥2𝑚 = 1
Q6. Let 𝒙𝟏 = 𝐭𝐚𝐧−𝟏 𝟐 > 𝒙𝟐 > 𝒙𝟑 > ⋯ are
2 𝑛2
So that S = 1+ 3+ …+ (2𝑚−1 )= 𝑚 = positive real numbers satisfying 𝐬𝐢𝐧(𝒙𝒏−𝟏 −
4
𝒙𝒏 ) + 𝟐−𝒏+𝟏 𝐬𝐢𝐧 𝒙𝒏+𝟏 = 𝟎 for n ≥ 1. Find
Case (ii) n is odd say n = 2m +1, then 𝝅
𝐜𝐨𝐭 𝒙𝒏 and also show that 𝐥𝐢𝐦 .
𝒙→∞ 𝟒
2
𝑚+1
S= ∑𝑘=1 (2𝑘 − 2𝑚 − 2) 𝑥𝑘
Sol. : By the problem sin 𝑥𝑛+1 cos 𝑥𝑛 −
If k = 1, 2, …, m, the coefficient are negative cos 𝑥𝑛+1 sin 𝑥𝑛 + 2−(𝑛+1) sin 𝑥𝑛 sin 𝑥𝑛+1 =
and in order to maximize S, we choose 0, ⟹ sin 𝑥𝑛+1

𝑥1 = 𝑥2 … = 𝑥𝑚 = 0, {cos 𝑥𝑛+2 − (𝑛 − 1) × sin 𝑥𝑛 }


= cos 𝑥𝑛+1 sin 𝑥,
If k = m +1, co-efficient of 𝑥𝑚 + 1is zero and
we can give any value to 𝑥𝑚 + 1. ⟹ cot 𝑥𝑛+1 = cot 𝑥𝑛+2
− (𝑛
If k=m +2, …, 2𝑚 + 1 the coefficients are + 1)(𝑑𝑖𝑣𝑖𝑑𝑖𝑛𝑔 sin 𝑥𝑛+1 sin 𝑥𝑛 )
positive and in order to maximize S, we
choose cot 𝑥𝑛 = cot 𝑥𝑛−1 + 2−𝑛
= cot 𝑥𝑛−2 + 2−(𝑛+1) + 2−𝑥
𝑥𝑚+2 = 𝑥𝑚+3 = ⋯ 𝑥2𝑚+1 = 1
= cot 𝑥𝑛−3 + 2−(𝑛−2) + 2−(𝑛−1) + 2−𝑛 = ⋯ =
So that S = (2+ 4+ …+ 2m)= m(m+ 1) =
cot 𝑥1 + 2−2 + 2−3 + ⋯ + 2𝑛 = 2−1 + 2 +
𝑛−1 𝒏+𝟏 𝒏𝟐 −1 𝑛2
𝟐 𝟐
= 4
. Thus when n is even S= 4 2 + ⋯ + 2−𝑛 (∵ tan−1 2 = 𝑥1 )
(𝑛2 −1)
and when n is odd S = so that we can say 1 𝑛
4
𝑛2 1 1 − (2) 1 𝑛
that S= [ 4 ]. = [ ] = 1 − ( ) ∴ cot 𝑥𝑛
2 1 − (1) 2
2
1 𝑛
=1−( ) .
2
Q5. Given that the polynomial f(x) = 𝒙𝒏 +
𝒂𝟏 𝒙𝒏−𝟏 + ⋯ + 𝒂𝒏 with integral coefficients is 1 𝑛
𝑁𝑜𝑤, Lt cot 𝑥𝑛 = Lt 1 − ( ) = 1,
equal to 5 for 4 distinct integers a, b, c, d. Show 𝑛→𝑥 𝑛→𝑥 2
that there is no integer k such that f(k) = 7. ⟹ cot ( Lt 𝑥𝑛 ) = 1,
𝑛→𝑥

271
Solving Mathematical Problems

⟹ Lt 𝑥𝑛 tan 4𝛽 − tan 𝛼
𝑛→𝑥 tan(4𝛽 − 𝛼) =
𝜋 𝜋 1 + tan 4𝛽 tan 𝛼
= [cot 𝑥 𝑖𝑠 𝑎 𝑐𝑜𝑛𝑡𝑖𝑛𝑢𝑜𝑢𝑠 𝑓𝑢𝑛𝑐𝑡𝑖𝑜𝑛 𝑜𝑓 ]. 120 1
4 4 − 239
= 119
120 1
1+ ×
119 239
Q7. A hexagon is inscribed in a circle. Show 120 × 239 − 119 119 × 239 + 120
that its shortest side cannot be less than the = = =1
119 × 239 + 120 119 × 239 + 120
radius. What is the length of the largest side? = tan ∞

Sol. : If n is the radius of the circle, the length of ∴ 4𝛽 − 𝛼 = 𝛾


the circumference is 2𝜋r. This is divided into six
area by the six vertices of the hexagon. All there
2𝜋𝑟
cannot be greater than 6
. As such the length Q9. If 𝜶𝟏 , 𝜶𝟐 , 𝜶𝟑 , … , 𝜶𝒏 are the roots of the
𝜋𝑟
of shortest one has to ≤ 3 and angle 𝜃 equation 𝒙𝒏 − 𝒏𝒂𝒙 − 𝒃 = 𝟎 𝒂𝒏𝒅(𝜶𝟏 −
𝜋
subtended by it at the centre ≤ radius. The 𝜶𝟐 ). (𝜶𝟏 − 𝜶𝟑 ) … (𝜶𝟏 − 𝜶𝒏 ) = 𝑨, then find the
3
𝜋 value of 𝑨 − 𝒏𝜶𝟏 𝒏−𝟏 .
length of the shortest side ≤ 2𝑟 sin = 𝑟 ≤
6
radius of the circle and it will actually by equal Sol. : 𝑥 𝑛 − 𝑛𝑎𝑥 − 𝑏 = (𝑥 − 𝛼1 )(𝑥 − 𝛼2 )(𝑥 −
𝑥 𝑛 −𝑛𝑎𝑥−𝑏
to r if the hexagon is regular. 𝛼3 ) … (𝑥 − 𝛼𝑛 ) ⟹ = (𝑥 − 𝛼2 )(𝑥 −
(𝑥−𝛼1 )

The length of the largest side can of course 𝛼3 ) … (𝑥 − 𝛼𝑛 ),


equal to 2r. 𝑥 𝑛 − 𝑛𝑎𝑥 − 𝑏
Lt
𝑥→𝑎 (𝑥 − 𝛼1 )
= Lt (𝑥 − 𝛼2 )(𝑥 − 𝛼3 ) … (𝑥
𝑥→𝛼1
Q8. In a rectangle ABCD, AB = 240 a, BC = a. − 𝛼𝑛 ),
The side AB is divided into 240 parts by 239
points 𝑶𝟏 , 𝑶𝟐 … . 𝑶𝟐𝟑𝟗 . The point C is joined to 𝑛𝑥 𝑛−1 − 𝑛𝑎
⟹ Lt
𝑶𝟏 , 𝑶𝟐𝟑𝟓 and 𝑶𝟐𝟑𝟗 to give angles 𝛼, 𝛽, 𝛾. Show 𝑥→𝛼1 1
that 4𝛽 – 𝛼 = 𝛾. = Lt (𝑥 − 𝛼2 )(𝑥 − 𝛼3 ) … (𝑥
𝑥→𝛼1

𝑎 1 𝑎 − 𝛼𝑛 ),
Sol. : tan 𝛼 = 239𝑎 = 239 tan 𝛽 = 5𝑎 =
1
, tan 𝛾
𝑎
=𝑎=1 ⟹ 𝑥𝛼1 𝑛−1 − 𝑛𝑎
5
= (𝛼1 − 𝛼2 )(𝛼1 − 𝛼3 )(𝛼1
2 − 𝛼4 ) … (𝛼1 − 𝛼𝑛 ),
2 tan 𝛽 5 5
tan 2𝛽 = = = tan 4𝛽
1 − 𝑡𝑎𝑛 𝛽 1 − 1
2 12 ⟹ 𝑛𝛼1 𝑛−1 − 𝑛𝑎 = 𝐴. ∴ 𝐴 − 𝑛𝛼1 𝑛−1 = −𝑛𝑎.
25
2 tan 2𝛽
=
1 − 𝑡𝑎𝑛2 2𝛽
𝟏
Q10. Draw the graph of 𝒇(𝒙) = 𝒙𝟐 + 𝒙 + 𝒙 +
5
120 𝟏
= 6 = for x ≠0. Show that the function f(x) defined
𝒙𝟐
25 119
1 − 144 for positive real numbers attains a unique

272
Solving Mathematical Problems

minimum. What is the minimum value of the MODEL TEST PAPER - 11


function? What is the value of x of which the
minimum is attained? Q1. Given that m and n are relatively prime
positive integers greater than one, show that
1 1 𝒍𝒐𝒈𝟏𝟎𝒎
Sol. : 𝑦 = 𝑥 2 + 𝑥 + 𝑥 + 𝑥 2 ; 𝑥 ≠ 0 is not a rational number.
𝒍𝒐𝒈𝟏𝟎𝒏

𝑑𝑦 1 2 𝑙𝑜𝑔10𝑚 𝑎
∴ = 2𝑥 + 1 − 2 − 3 = 0, ⟹ 𝑥 ≠ 1 Sol. : If possible, let 𝑙𝑜𝑔10𝑛
= 𝑏 where a, b are
𝑑𝑥 𝑥 𝑥
positives integers with no common factor so
𝑑2 𝑦 2 6 that 𝑎 𝑙𝑜𝑔10𝑛 = 𝑎 𝑙𝑜𝑔10𝑚 𝑜𝑟 log 𝑛10 𝑎 =
2
= 2 + 3 + 4.
𝑑𝑥 𝑥 𝑥 log 𝑚10 𝑏 , 𝑜𝑟 𝑛𝑎 = 𝑚𝑏 , but this cannot be true
𝑑𝑦 since a and b are integers and n and m are
∴[ ] =2≠2+6
𝑑𝑥 2 𝑥=≠1 relatively prime i.e. here no common factor
accept 1.
= 𝑦 ≠ 2 > 0(min 𝑝𝑡).

𝑥 = 1, 𝑦 = 4
Q2. Prove that if the coefficients of the
𝑥 = −1, 𝑦 = 0
quadratic equation 𝒂𝒙𝟐 + 𝒃𝒙 + 𝒄 = 𝟎 are odd
𝑥 → +∞, 𝑦 → +∞ integers, then the roots of the equations
cannot be rational numbers.
𝑥 → −∞, 𝑦 → +∞
Sol. : Let 𝑎 = 2𝑚 + 1, 𝑏 = 2𝑛 + 1, 𝑐 = 2𝑘 + 1,
𝑏𝑢𝑡 𝑥 > 0, 𝑠𝑜 𝑚𝑖𝑛. 𝑝𝑡. → (1, 4)(𝑢𝑛𝑖𝑞𝑢𝑒) then the roots will be rational if (2𝑛 + 1)2 −
4(2𝑚 + 1)(2𝑘 + 1) is a perfect square.

Since the number is odd, if can be only the


square of an odd number, so that(2𝑛 + 1)2 −
4(2𝑚 + 1)(2𝑘 + 1) = (21 + 1)2 ,

𝑜𝑟 (2𝑛 + 1)2 − (21 + 1)2 = 4(2𝑚 + 1)(2𝑘 +


1),

𝑜𝑟 (2𝑛 + 21 + 2)(2𝑛 − 21) = 4(2𝑚 +


1)(2𝑘 + 1)𝑜𝑟 (𝑛 + 𝑙 + 1)(𝑛 − 𝑙)

= (2𝑚 + 1)(2𝑘 + 1)

Now n and l cannot be both even or odd, since


then L.H.S will be even and the R. H. S. would be
odd. If one of them is even and the other is odd,
the first factor is again even and this again gives
a contradiction.

273
Solving Mathematical Problems

Q3. Suppose 𝒏(𝒏 + 𝟏)𝒂𝒏+𝟏 = 𝒏(𝒏 − 𝟏)𝒂𝒏 − 1


∴ 𝑎𝑘+1 =
(𝒏 − 𝟐)𝒂𝒏−𝟏 for every positive n ≥1. Given (𝑘 + 1)!
𝒂 𝒂 𝒂
that 𝒂𝟎 = 𝟏, 𝒂𝟏 = 𝟐 . Find 𝒂𝟎 + 𝒂𝟏 + 𝒂𝟐 + ⋯ +
𝟏 𝟐 𝟑 This shows by mathematical induction that
𝒂𝟏𝟎𝟎 1
𝒂𝟏𝟎𝟏
. 𝑎𝑘 = 𝑘 for all positive values of k.
𝑖

Sol. : If 𝑎0 = 1, 𝑎1 = 2, the given equation gives 𝑎 𝑎 𝑎 𝑎 1 2


1
!
Now 𝑎0 + 𝑎1 + 𝑎2 + ⋯ + 𝑎110 = 2 + 1 + 21 +
1 2 3 101 ! !
1 1 2 3
2𝑎2 = 𝑎0 = 1, ⟹ 𝑎2 = = 1
! 1
2 21 ⋯+ 100
1 = 2 + 4 + 3 + 4 + 5 + ⋯ + 101
!
101
1 1
6𝑎3 = 2𝑎2 − 0𝑎1 = 1, ⟹ 𝑎3 = = 3 3 101 − 102
6 31
= + 1 + 2 + 3 + ⋯ + 101 = +
2 2 2
1 1
a b c d = 5151 + 1 = 5152 .
2 2
d c b a Q4. Show that it is possible to put pair wise
distinct positive integers less than 100 in the
b a d c
cells of a 4× 𝟒 table so that the product of all
c d a b the numbers in every row and every column
are row equal to each other.
1 1 1
12𝑎4 = 6𝑎3 − 2 = 1 − , ⟹ 𝑎4 = = . Sol. : This can be achieved by using orthogonal
2 24 41
latin squares.
1
The suggests 𝑎𝑘 = 𝑘 . Suppose this is true for
1
𝑎1 , 𝑎2 , … , 𝑎𝑘 , then
A B C D
𝑘(𝑘 + 1)𝑎𝑘+1 = 𝑘(𝑘 − 1)𝑎𝑘 − (𝑘 − 2)𝑎𝑘−1
B A D C
Aa Bb Cc Dd
C D A B
Bd Ac Db Ca
D C B A
Cb Da Ad Bc
(1)
Dc Cd Ba Ab
(2)
𝑘(𝑘 − 1) 𝑘−2
= −
𝑘! (𝑘 − 1)!
0 1
= −
(𝑘 − 2)! (𝑘 − 2)!
1 (3)
+
(𝑘 − 1)!
Fig (1) gives a Latin square since in it each of the
letters A, B, C, D occurs once in every row and

274
Solving Mathematical Problems

once in every column. Similarly the (2) fig. 𝑛 4 𝑛 3


𝑎𝑛 + 1 = 3( 𝑏𝑛 103 − 1) + 4(𝑏𝑛 102 − 1)
represents another latin square in which each 𝑛 3 𝑛
= (102 𝑏𝑛 − 1) (3 × 102 𝑏𝑛
of the letters a, b, c, d occurs once in every row
and once in every column. The 3rd fig. is + 1)
obtained by superposing there so that each of = (103 × 2𝑛 𝑏𝑛 3 − 3
𝑛 𝑛
the four letters A, B, C, d are each of the four × 102.2 𝑏𝑛 2 + 3. 102 𝑏𝑛
𝑛
letter a, b, c, d occurs in every row and column − 1)(3. 102 𝑏𝑛 + 1)
and each of the sixteen pairs of these elements 𝑛 𝑛
occurs once only. If A, B, C, D, a, b, c, d = (3. 102 𝑏𝑛 − 1)(3 × 102
represents numbers and Aa stands for A× 𝑎 etc. − 1)(𝑚𝑜𝑑 102𝑛+1 )
then it is easily seen that the product of
= −1(𝑚𝑜𝑑 102𝑛+1 ), so that by induction our
elements is every row and every column is
result follows.
ABCD abcd and so is the same for every row and
column. We choose there as prime numbers i.e.
we take A = 1, B = 2, C = 3, D = 5, a = 7, b = 11, c
= 13, d = 17 to get the result for fig (3). Q6. Prove that (𝒂𝟏 𝒃𝟏 + 𝒂𝟐 𝒃𝟐 + ⋯ + 𝒂𝒏 𝒃𝒏 )𝟐 ≤
(𝒂𝟏 𝟐 + 𝒂𝟐 𝟐 + ⋯ + 𝒂𝒏 𝟐 )(𝒃𝟏 𝟐 + 𝒃𝟐 𝟐 + ⋯ +
7 22 39 85
𝒃𝒏 𝟐 ) and the equality sign holds, only when
𝒂𝟏 𝒂 𝒂
34 13 15 21 𝒃𝟏
= 𝒃𝟐 = ⋯ = 𝒃𝒏 .
𝟐 𝒏

33 35 17 26 Deduce that the root mean square ≥


arithmetic mean.
65 51 14 11
Sol. : Consider the quadratic equation.

(𝑎1 𝑥 − 𝑏1 )2 + (𝑎2 𝑥 − 𝑏2 )2 + ⋯ + (𝑎𝑛 𝑥 −


The const. is 1 × 2 × 3 × 5 × 7 × 11 × 13 × 𝑎1
𝑏𝑛 )2 = 0. This can have real roots only if =
17 = 510510 𝑏1
𝑎2 𝑎𝑛 𝑎
=⋯= = (say) and in this case the two
𝑏2 𝑏𝑛 𝑏
𝑎
real roots councicle and each is equal to .
𝑏
Q5. A sequence [𝒂𝒏 ] is determine by the rule
𝒂𝟎 = 𝟗 and for any positive r, 𝒂𝒏+𝟏 = 𝟑𝒂𝟒 𝒏 + In all other cases, the roots are complex and the
𝟒𝒂𝟑 𝒏. Prove that 𝒂𝒏 = −𝟏(𝒎𝒐𝒅 𝟏𝟎 ). 𝟐𝒏 discriminate of the quadratic equation is
negative. The quadratic equation is
Sol. : 𝑎0 = −1(𝑚𝑜𝑑 10). Suppose the result is
𝑥 2 (𝑎1 2 + 𝑎2 2 + ⋯ + 𝑎𝑛 2 ) − 2𝑥
true for some n, then 𝑎𝑛 = 𝑏𝑛 102𝑛−1 , 𝑡ℎ𝑒𝑛
(𝑎1 𝑏1 + 𝑎2 𝑏2 + ⋯ + 𝑎𝑛 𝑏𝑛 ) − (𝑏1 2 + 𝑏2 2 +
⋯ + 𝑏𝑛 2 ) = 0

Since its discriminant ≤ 0, we get

275
Solving Mathematical Problems

(𝑎1 𝑏1 + 𝑎2 𝑏2 + ⋯ + 𝑎𝑛 𝑏𝑛 )2 ≤ (𝑎1 2 + 𝑎2 2 +
⋯ + 𝑎𝑛 2 )(𝑏1 2 + 𝑏2 2 + ⋯ + 𝑏𝑛 2 ) and the
𝑎1 𝑎2 𝑎𝑛 Q8. Find the max. and min. values of sin x sin
equality sign holds only if = =⋯= .
𝑏1 𝑏2 𝑏𝑛 2x and draw its graph.
Putting 𝑏1 = 𝑏2 = ⋯ 𝑏𝑛 = 1, we get
1
Sol. : Let 𝑧 = sin 𝑥 sin 2𝑥 = [cos 𝑥 −
2
(𝑎1 + 𝑎2 + ⋯ + 𝑎𝑛 )2 ≤ 𝑛(𝑎1 2 + 𝑎2 2 + ⋯ + 1
𝑎𝑛 2 ). cos 3𝑥] = 2

𝑎1 + 𝑎2 + ⋯ + 𝑎𝑛 2 [cos 𝑥 − 4𝑐𝑜𝑠 3 𝑥 + 3 cos 𝑥]


⟹( ) = 2 cos 𝑥 (1 − 𝑐𝑜𝑠 2 𝑥)
𝑛
𝑎1 2 + 𝑎2 2 + ⋯ + 𝑎𝑛 2 = 2(𝑦 − 𝑦 3 ),

𝑛
𝑑𝑧
𝑦 = cos 𝑥 , 𝑠𝑜 𝑡ℎ𝑎𝑡
𝑜𝑟, 𝐴. 𝑀. ≤ 𝑅. 𝑀. 𝑆. 𝑑𝑦
𝑑2 𝑧 𝑑𝑧
= 2(1 − 3𝑦𝑒), 2
− 12𝑦,
𝑑𝑦 𝑑𝑦
2
Q7. If A and B are fixed points on a given circle 1 𝑑 𝑦
= 0, ⟹ 𝑦 =≠ ,
not collinear with centre O of the circle and if √3 𝑑𝑧 2
XY is a variable diameter, find the locus of P < 0, 𝑤ℎ𝑒𝑛
(the intersection of the line through A and X 1 1
𝑦= and it is > 0 when 𝑦 = − there is a
and the line through B and Y). √3 √3
1
max. value when cos 𝑥 = and a min. value
√3
Sol. : 1 1
when cos 𝑥 = − . The max. value is 2 (1 −
√3 √3
1 4 𝜋
3
) =3 = .7698, 𝑥 = .3041𝜋 > 4 .
√3

1 1 4
The main value is −2. (1 − 3) = − 3+3 =
√3
3𝜋
−7698, 𝑥 = .6959𝜋 < 4.
∠XBY =90°(angle in a semi-circle) ∠ A× 𝐵 =
𝑑𝑧 𝑑𝑧 𝑑𝑦 𝑑𝑧 𝑑𝑧
𝛼(const. angle subtended by given are AB at Now 𝑑𝑥 = 𝑑𝑦 . 𝑑𝑥 = − 𝑑𝑦 sin 𝑥 , 𝑠𝑜 𝑡ℎ𝑎𝑡 𝑑𝑥 also
any point on the circle) vanishes when sin 𝑥 = 0. cos 𝑥 =≠ 1, 𝑧 = 0

∠XPB = 90° − 𝛼 and ∠APB = 90° + 𝛼 = const. so 𝐿𝑒𝑡 cos 𝛼 =


1
,0
𝜋
< 𝛼 < , then the given
√3 2
that the arc of AB subtends a const.
function has max value of .7698 at ≠ 𝛼, ≠ 𝛼≠
angle 90° + 𝛼 at P. 2𝜋, ≠𝛼≠ 4𝜋, …, ≠

∴ Locus of P is the arc of a circle passing (𝜋 − 𝛼) ≠ 2𝑘𝜋, 𝑘 = 0, 1, 2, 3, … ..


through A and B and subtending a const. angle
It has also min. and min. value at x = x𝜋, when n
90° + 𝛼 at all points on this arc in fig.(2), the
is an integer, positive, negative on zero.
locus is again the arc of a circle passing through
A and B with angle 90°𝑋. The graph is, therefore, as shown below.

276
Solving Mathematical Problems

Q10. M is an interior pt. of a rectangle ABCD


and S is its area prove that S ≤AM. CM + BN
.DM.

Sol. : Let G, F, GH be the middle points of the


AB, BC, CD, DA.

𝑆1 = area of rectangle AHME + area of rectangle


BFME + area of rectangle CGMF + area of
rectangle GDHM = 𝑏𝑑 cos 𝛽 sin 𝛾 𝑠 +
𝑎𝑐 cos 𝛼 sin 𝛾 + 𝑏𝑑 cos 𝛿 sin 𝛽 +
𝑎𝑐 cos 𝛾 sin 𝛼 = 𝑏𝑑 (cos 𝛽 sin 𝛿 +
Q9. Let f be a real function with a continuous cos 𝛿 sin 𝛽) + 𝑎𝑐(cos 𝛼 sin 𝛾 + sin 𝛼 sin 𝛾) =
third derivative such that 𝒇(𝒙), 𝒇𝒏 (𝒙), 𝒇𝒎 (𝒙) 𝑏𝑑 cos(𝛽 + 𝛿) + 𝑎𝑐 sin(𝛼 + 𝛾) ≤ 𝑏𝑑 + 𝑎𝑐 =
are positive for all real x suppose that 𝒇𝒎 (𝒙) ≤ 𝐵𝑀. 𝑀𝐷 + 𝐴𝑀. 𝐴𝐶.
𝒇(𝒙)∀𝒙.

Show that 𝒇′ (𝒙) < 2𝒇(𝒙)∀ 𝒙.

Sol. : If f is a differentiable function on all of R


lim 𝑓(𝑥) ≥ 0 𝑎𝑛𝑑 𝑓 ′ (𝑥) > 0∀ 𝑥 ∈ 𝑅, then
𝑛→∞

𝑓(𝑥) > 0∀𝑥 ∈ 𝑅. If f(y) <0 for some x, then f(x)


< f(y) ∀ x< y.

∵ f’ > 0but then lt 𝑓(𝑥) ≤ 𝑓(𝑦) < 0 from


𝑛→∞
the inequality 𝑓 ′′′ (𝑥) ≤ 𝑓(𝑥). We obtain

𝑓 ′′ 𝑓 ′′ (𝑥) ≤ 𝑓 ′′ (𝑥). 𝑓(𝑥) MODEL TEST PAPER - 12


< 𝑓 ′′ (𝛼). 𝑓(𝑥) + 𝑓 ′ 2(𝑥).
Q1. Show that the equation 𝒙𝒏 − 𝒙𝒏−𝟏 −

∴ 𝑓 (𝑥) is positive.
𝒙 … . 𝟏 = 𝟎 has only one positive real root
1 which lines between 1 and 2 and this root
∴ 2 (𝑓′′(𝑥))2 < 𝑓(𝑥). 𝑓 ′ (𝑥) …………. (1)
aproaches 2 as n→∞.
∵f(x) and f’’’(x) are both positive for all x we
Sol. : Let 𝑓(𝑥) = 𝑥 𝑛 − 𝑥 𝑛−1 − 𝑥 𝑛−2 … . −1.
have 2𝑓 ′ (𝑥). 𝑓 ′′ (𝑥) < 2𝑓(𝑥). 𝑓 ′′′ (𝑥)
……………..(2) Since there is only one change of sign in f(x)
2
by Descartes rule of signs, these cannot be
1 𝑓′(𝑥) 1 2
From (1) and (2), 2 {2𝑓(𝑥)} < 2 (𝑓 ′′ (𝑥)) < more than one positive real root of f(x) = 0.
3 3 Also f(0) = -1, f(1) = -(x- 1)<0, f(2) =
𝑓(𝑥). 𝑓 ′ (𝑥), ⟹ (𝑓 ′ (𝑥)) < (𝑓(𝑥)) . 2𝑛 −1
2𝑛 = 1 > 0.
2−1
′ (𝑥)
∴𝑓 < 2𝑓(𝑥).

277
Solving Mathematical Problems

So that f(x) changes sign as x goes from 1 to 2. 𝑟 𝑟


Reqd. sum = 2 ∑3𝑁−1
𝑟=1 min ({3𝑁} , {6𝑁 }) =
As such f(x) = 0 has exactly one root lying 𝑟 𝑟
between 1 and 2. 2 ∑2𝑁 3𝑁−1
𝑟=1 {6𝑤} + 2 ∑2𝑁+1 {3𝑁 }

2𝑁 𝑁−1
Let this root by 2- y, then (2 − 𝑦)𝑛+1 − 𝑟 (𝑁 − 1)𝑁
2(2 − 𝑦)𝑛 + 1 = 0, = 2∑ +2∑ = 𝑁.
6𝑁 2
𝑟=1 𝑟=1
𝑦 (𝑥 + 1)𝑛 𝑦 2
⟹ 2𝑛+1 (1 − 𝑛 + 1 + . …..)
2 2 4
𝑦 Q4. Show that if there are 𝒏𝒌 + 𝟏 pigeons and
− 2𝑛+1 (1 − 𝑛.
2 only n holds for them, then one of the holes
2
𝑥(𝑛 − 1) 𝑦 has to have k+1 on more pigeons.
+ . … . . ) + 1 = 0,
2 4
Sol. : This is obvious since if all the holes have k
𝑛 𝑛−1 2 or fewer pigeons. Thus the total number of
⟹1−2 𝑦+2 𝑦 𝑛+⋯=
0 𝑠𝑜 𝑡ℎ𝑎𝑡 𝑎𝑠 ℎ → ∞, y → 0 and the root of the pigeons would be less than or equal to 𝑛𝑘 and
equation approaches 2. cannot be 𝑛𝑘 + 1. This principle is known as the
pigeon-hole principle.

Q5. Prove that if 𝒏𝟏 , 𝒏𝟐 , … , 𝒏𝒌 are any integers


Q2. Let n be a number consisting of 1991 one’s
and a is any number, they
i.e. 𝒏 = ⏟
𝟏𝟏𝟏 … . 𝟏 … 𝟏 show that n is not a
𝟏𝟗𝟗𝟏 𝒏𝟏 +𝒏𝟐 +⋯+𝒏𝑲 𝒏 𝒏 𝒏
[ ] ≥ [ 𝟏] + [ 𝟐] + ⋯ + [ 𝒌 ], where
prime number. 𝒂 𝒂 𝒂 𝒂
[x] denotes the integral part of x. Deduce that
Sol. : Since 1991 = 11× 181; we can write the high e = t power of a prime p contained in
𝒏 𝒏 𝒏
𝑛 = 11
⏟ …1⏟111 … 1 … ⏟
111 … .1 so that 𝑛 = n! is [ ] + [ 𝟐] + [ 𝟑] + ⋯
𝒑 𝒑 𝒑
⏟ 11 11 11
181 Sol. : Let 𝑛1 = 𝑎𝑞1 + 𝑟1 , 𝑛2 = 𝑎𝑞2 + 𝑟2 , … 𝑛𝑘 =
1980 1959
𝑎(10 + 10 + 101958 + ⋯ + 1011 + 1) 𝑛1 +𝑛2 +⋯+𝑛𝑘 𝑟1 +𝑟2 +⋯+𝑟𝑘
𝑎𝑞𝑘 + 𝑟𝑘 𝑠𝑜 𝑡ℎ𝑎𝑡 𝑎
= 𝑎
,
𝑎 = 11
⏟ … 1 so that n is not a prime number.
𝑛 𝑛1 + 𝑛2 + ⋯ + 𝑛𝑘
⟹[ ] ≥ 𝑞1 + 𝑞2 + ⋯ + 𝑞𝑘
𝑎
𝑛1 𝑛2 𝑛𝑘
= [ ] + [ ] + ⋯ + [ ].
Q3. Find 𝑎 𝑎 𝑎
𝐫 𝐫
∑𝟔𝑵−𝟏
𝒓=𝟏 𝐦𝐢𝐧 ({𝟑𝐍} , {𝟔𝐍}) , 𝒘𝒉𝒆𝒓𝒆 {𝒂} = 𝑛
Now out of the numbers 1, 2, …., n. [𝑝 ] are
𝐦𝐢𝐧(𝒂 − [𝒂], [𝒂] − 𝒂 + 𝟏) the distance to the 𝑛 𝑛
nearest integer ([a] represents G.I.F). divisible by r, [𝑝2 ] are divisible by 𝑝2 [𝑝3 ] are
divisible by 𝑝3 and so on, so that the highest
3𝑁
Sol. : Middle term of the sequence is {3𝑁} = 0 power of the prime r contained in n! is [ ] +
𝑛
𝑝
6𝑁−𝑟 𝑟 6𝑁−𝑟 𝑟
and { } = {3𝑁} , { } = {6𝑁} 𝑛 𝑛
3𝑁 6𝑁 [𝑝 2 ] + [𝑝 3 ] + ⋯

278
Solving Mathematical Problems

Q6. For a cyclic quadrilateral ABCD inscribed in 𝐴𝑅2 1


Area of shaded strip = 6
− 2 𝑅. 𝑅 cos 30° =
a circle, we have BC = CD. Prove that area of
𝟏
𝑅2
the quadrilateral 𝟐
(𝑨𝑪)𝟐 𝐬𝐢𝐧 𝑨.
𝜋 √3
Sol. : In the quadrilateral ∠DAC = ∠DBC (angles ( − ) . 𝐴𝑟𝑒𝑎 𝑜𝑓 ℎ𝑎𝑙𝑓 𝑡ℎ𝑖𝑠 𝑠𝑡𝑟𝑖𝑝
6 4
in the same segment), ∠DBC = ∠BDC (∵ BD = 𝑅 2 𝜋 √3
CD), = ( − ) = 𝑎𝑟𝑒𝑎 𝐼.
2 6 4
∠BCD = ∠CAB (angles in the same segment), 𝜋 √3
∠DCA = ∠DBA (angles in the same segment) . Area of II = 𝑅 2 ( 6 − 4
).

∴𝛥’s ABE and ACD are similar, so that 𝜋𝑅2


Area of sector ODF = 12
.
𝐵𝐸 𝐴𝐸 𝐴𝐵
𝐴𝐶
= 𝐴𝐷 = 𝐴𝐶 . Similarly since 𝛥’s CED and CAD 1 𝜋𝑅 2 3𝑅2 𝜋 √3
𝑜𝑓 𝑟𝑒𝑞𝑑. 𝑎𝑟𝑒𝑎 = − ( − ).
are similar, we get 8 12 2 6 4
𝐶𝐸 𝐷𝐸 𝐶𝐷
𝐶𝐷
= 𝐷𝐴 = 𝐴𝐶 . 𝜋
∴ Reqd. area =8𝑅2 (12 − 12 +
3𝜋 3√3
)−
4
4𝜋
Now area of ABCD = area of 𝛥ABD + Area of 𝛥 𝑅 2 (6√3 − 3
).
1 1
BCD = 2 𝐴𝐵 × 𝐴𝐷 𝑆𝑖𝑛 𝐴 + 2 𝐵𝐶 × 𝐶𝐷 sin(𝜋 −
1
𝐴) = 2 sin 𝐴 [𝐴𝐵 × 𝐴𝐷 + 𝐵𝐶 × 𝐵𝐷] =
1
2
sin 𝐴 [𝐴𝐶 × 𝐴𝐸 + 𝐴𝐶 × 𝐶𝐸]

1 1
= 2 sin 𝐴 . 𝐴𝐶(𝐴𝐸 + 𝐸𝐶) = 2 sin 𝐴 (𝐴𝐶)2 .

Q7. Is it possible to divide the plane into


polygons so that each polygon is transformed
𝟐𝝅
into itself under same rotation by 𝟕
about Q9. Find all solutions (𝒙𝟏 , 𝒙𝟐 , 𝒙𝟑 , 𝒙𝟒 , 𝒙𝟓 ) of the
same pt.? All sides of there polygon must be system of inequalities
greater than 1 cm. (A polygon is the part of a
plane bounded by one non-self intersecting (𝒙𝟏 𝟐 − 𝒙𝟑 𝒙𝟓 )(𝒙𝟐 𝟐 − 𝒙𝟑 𝒙𝟓 ) ≤ 𝟎;
closed broken line, not necessary convex)?
(𝒙𝟐 𝟐 − 𝒙𝟒 𝒙𝟏 )(𝒙𝟑 𝟐 − 𝒙𝟒 𝒙𝟏 ) ≤ 𝟎;
Sol. : Try yourself.
(𝒙𝟑 𝟐 − 𝒙𝟓 𝒙𝟐 )(𝒙𝟒 𝟐 − 𝒙𝟓 𝒙𝟐 ) ≤ 𝟎;
Q8. Compute the area of the shaded fig.
(𝒙𝟒 𝟐 − 𝒙𝟏 𝒙𝟑 )(𝒙𝟓 𝟐 − 𝒙𝟏 𝒙𝟑 ) ≤ 𝟎 𝒂𝒏𝒅
knowing that all areas come from circles with
radius R and knowing that A and B are (𝒙𝟓 𝟐 − 𝒙𝟐 𝒙𝟒 )(𝒙𝟏 𝟏 − 𝒙𝟐 𝒙𝟒 ) ≤
mutually perpendicular axes of symmetry. 𝟎, 𝒘𝒉𝒆𝒓𝒆 𝒙𝟏 , 𝒙𝟐 , 𝒙𝟑 , 𝒙𝟒 , 𝒙𝟓 are positive real
numbers.
Sol. : Since OD = R, OE =R, ED = R, 𝛥OED is an
equilateral triangle and ∠DOE= 60°, ∠DOF = Sol. : Each inequality is of the form (𝑥𝑖 2 −
30°. 𝑥𝑖+2 𝑥𝑖 + 4)(𝑥𝑖+1 2 − 𝑥𝑖+2 𝑥𝑖 + 4) ≤ 0, where

279
Solving Mathematical Problems

the idices are read modulo 5, i.e. 𝑥𝑗 + 5 = 𝑥𝑗 . If


we multiply out each expression on the left and
then odd all the inequalities, we find that all (5)
= 10 terms of the from 𝑥𝑖 2 𝑥𝑗 2 (𝑖 ≠ 𝑗) appear
the sum, as well as ten cross terms, five of the
form −𝑥1 2 𝑥1 + 1𝑥1 + 3 and five of the form
−𝑥1 2 𝑥1 + 2𝑥1 + 4. This suggests a sum of
1
squares of the form 2 (𝑦1 2 + 𝑦2 2 + ⋯ + 𝑦10 2 ), It is clear y is not
defined at x = 0, ≠ 1.
where to each cross terms, we associate ay, for
example, to the cross term 𝑥2 2 − 𝑥3 𝑥5 , we
associate 𝑦1 = 𝑥2 𝑥3 − 𝑥2 𝑥5 , knowing that the
terms 𝑥2 2 𝑥3 2 , 𝑥2 2 𝑥5 2 appearing in 𝑦1 2 also MODEL TEST PAPER – 13
appear in our sum. Thus we arrive at the
Q1. If p(x), Q(x), R(x), S(x) are polynomials so
following representation of the sum of the
that
given inequalities.
5
𝑷(𝒙𝟓 ) + 𝒙𝑸(𝒙𝟓 ) + 𝒙𝟐 𝑲(𝒙𝟓 )(𝒙𝟒 + 𝒙𝟑 + 𝒙𝟐 +
0 ≥ ∑(𝑥𝑖 2 − 𝑥𝑖+2 𝑥𝑖 + 4)(𝑥𝑖+1 2 𝒙 + 𝟏)S(x) …… (i), then show that x- 1 is a
𝑖=1 factor of P(x).
− 𝑥𝑖+2 𝑥𝑖 𝑥𝑖+4 )
5 Sol. : 𝜆 Let 𝜔 = 𝑒 2𝜋𝑖/5 , so that 𝜔5 = 1. We see
1
= ∑{(𝑥𝑖 𝑥𝑖+1 − 𝑥𝑖 𝑥𝑖+3 )2 for x in (1), 𝜔, 𝜔2 , 𝜔3 , 𝜔4 successively and get
2
𝑖=1 the following equations.
+ (𝑥𝑖−1 𝑥𝑖+1 − 𝑥𝑖−1 𝑥𝑖+3 )2 }
𝑃(1) + 𝜔𝑄(1) + 𝜔2 . 𝑅(1) = 0.
Since this sum of squares cannot be negative.
We conclude that it is zero, which means that 𝑃(1) + 𝜔2 𝑄(1) + 𝜔4 𝑅(1) = 0.
each term vanishes. This implies that 𝑥1 = 𝑓(𝑏) 1
𝑥2 = 𝑥3 = 𝑥4 = 𝑥5 . Every set of five equal = 2∫ 𝑥 [𝑏 − ∫ (𝑥)] 𝑑𝑥,
𝑓(𝑎)
positive numbers is a solution of the given
system of inequalities. 𝑃(1) + 𝜔4 𝑄(1) + 𝜔3 𝑅(1) = 0,
(𝒙−𝟏) ⎸|𝒙+𝟏|⎸ 𝟏
Q10. Draw the graph of y = ⎸|𝒙|⎸−𝟏 + + 𝒙. −𝜔 𝑃(1) − 𝜔2 𝑄(1) − 𝜔3 𝑅(1) = 0,
(𝒙+𝟏)

−𝜔2 𝑃(1) − 𝜔4 𝑄(1) − 𝜔𝑅(1) = 0,


Sol. :
−𝜔3 𝑃(1) − 𝜔𝑄(1) − 𝜔4 𝑅(1) = 0,

−𝜔4 𝑃(1) − 𝜔3 𝑄(1) − 𝜔2 𝑅(1) = 0.

Using 1+𝜔 +𝜔2 + 𝜔3 + 𝜔4 = 0, we get the


(𝑥−1)
see m 5P(1)= 0, i.e. 𝑃(𝑥)
.

280
Solving Mathematical Problems

Q2. Find x in 88 ….. 8x 999….9 (there are 50 Q4. Show that if in a party there are 6 persons
eights and 50 mines) so that the number is then there must be of least 3 persons who are
divisible by 7. either mutual acquaintances or mutual
strangers.
Sol. : If the number is divisible by 7, then so is
11….1x 22……2. Sol. : We represents the 6 persons by the 6
vertices 𝐴1 , 𝐴2 , 𝐴3 , 𝐴4 , 𝐴5 , 𝐴6 of a hexagon so
No 111111 is divisible by 7, so that we can
that no 3 of those points are collinear and we
remove 48 one’s from the left hand side and
can draw 6𝑐2 = 15 lines joining pair of those 6
48 two’s from the R.H.s. if the number
without affecting divisibility, so that the given points. This gives us 6 edges of a hexagon
number is divisible by 7 if 11x 22 is divisible 𝐴1 𝐴2 𝐴3 𝐴4 𝐴5 𝐴6 and 9 diagonals of this
by 7. hexagon. We draw is of these 15 lines either in
red or blue. We draw a red or blue line joining
The requires 2 + 3 × 2 + 2 × 𝑥 + 6 × 1 + 4 × pts. 𝐴𝑖 , 𝐴𝑗 according as 𝐴𝑖 , 𝐴𝑗 are acquaintances
1 ≡ 0(𝑚𝑜𝑑 7), or strangers. Some of the 15 lines will be red
and some will be blue. If we get a triangle all of
⟹ 2𝑥 + 4 ≡ 0 (𝑚𝑜𝑑 7), ⟹ 𝑥 = 5.
whose sides are red, we get 3 mutual
Q3. Prove that the equation 𝒙𝟑 + 𝟏𝟏𝟑 = 𝒚𝟑 acquaintances and if we get a triangle all of
has no solution for positive integers x and y. whose sides are blue, we shall have 3 persons
who are mutual strangers. Thus we have to
Sol. : 113 = 𝑦 3 − 𝑥 3 = (𝑦 − 𝑥)(𝑦 2 + 𝑥 2 + 𝑥𝑦) show that in the complete graph of 6 vertices
∵ 𝑦 − 𝑥 𝑑𝑖𝑣𝑖𝑑𝑒𝑠 113 , we have the following and 15 lines, these must be either one blue
possibilities. triangle or one red, i.e. there must be a
monochromatic triangle, whatever be the way
𝑦 − 𝑥 = 1, 𝑦 2 + 𝑥 2 = 𝑥𝑦 = 113 in which we colour 15 lines as red or blue.

𝑦 − 𝑥 = 11, 𝑦 2 + 𝑥𝑦 + 𝑥 2 = 112 From the vertex, 𝐴𝑖 , 5 edges emerge, some of


which are red and some of which blue, by
𝑦 − 𝑥 = 112 , 𝑦 2 + 𝑥𝑦 + 𝑥 2 = 11
pigeon hole principle at least 3 of these must be
𝑦 − 𝑥 = 113 , 𝑦 2 + 𝑥𝑦 + 𝑥 2 = 1 of the same colour. For definiteness, let us take
there as red. Then the 3 other vertices of their 3
In the last three cases y >11 and so 𝑦 2 + 𝑥𝑦 + edges will from a triangle with 3 edges of its
𝑥 2 > 112 and so no solution is possible. In the own. If one of their 3 edges is red, we get a red
first case, we get triangle with 3 red edges and out result is
proved.
(𝑥 + 1)2 + 𝑥 2 + 𝑥(𝑥 + 1) = 1331, ⟹ 3𝑥 2 +
3𝑥 − 1330 = 0, but 1330 is not divisible by 3, However if none of their 3 edges is red, we get
so no solution in integers is possible in this case a blue triangle will all its sides blue. In either
also. case, we get either a red triangle or a blue
triangle and our result is proved. The result
does not hold if there are only 5 persons in the
party since then we shall have a complete graph

281
Solving Mathematical Problems

of 5 edges and 5 diagonals of a pentagon and Sol. : If possible let PQ be such a line, then since
we can have a graph of the type shown in the 1
it bisects the perimeter 𝑥 + 𝑦 = 2 (𝑎 + 𝑏 + 𝑐)
following fig 1 where all the sides have been 1 1 1
and since the area 𝑥𝑦 sin 𝐴 = ( 𝑏𝑐. sin 𝐴),
coloured blue. 2 2 2

1
⟹ 𝑥𝑦 = 𝑏𝑐, 𝑠𝑜 𝑡ℎ𝑎𝑡 (𝑥 − 𝑦)2
2
𝑎+𝑏+𝑐 2
= ( ) − 2𝑏𝑐.
2

1 𝑎+𝑏+𝑐 𝑎+𝑏+𝑐 2
𝑥, 𝑦 = [ ≠ √( ) − 2𝑏𝑐].
2 2 2
Q5. Prove that 𝑨𝟏 , 𝑨𝟐 , 𝑨𝟑 , 𝑨𝟒 are the angles of
𝑨𝟏 𝑨𝟐 For the line PQ with the reqd. properties to
a convex quadrilateral, then 𝐬𝐢𝐧 𝟐
+ 𝐬𝐢𝐧 𝟐
+
exist, if is necessary that
𝑨𝟑 𝑨𝟒 𝝅
𝐬𝐢𝐧 𝟐
+ 𝐬𝐢𝐧 𝟐
≤ 𝟒𝐬𝐢𝐧 𝟒 .
(i) (𝑎 + 𝑏 + 𝑐)2 ≥ 8𝑏𝑐,
Sol. : Let 𝐴1 , 𝐴2 , … . , 𝐴𝑛 be the angles of n-sided (ii) 𝑥 ≤ 𝑐, 𝑦 = 𝑏 𝑜𝑟 𝑥 ≤ 𝑏, 𝑦 ≤ 𝑐.
convex polygon, then 𝐴1 + 𝐴2 + ⋯ 𝐴𝑛 =
𝐴1 𝐴 𝐴
(𝑛 − 2)𝜋, + 2 + ⋯ + 𝑛 = 𝜋, so that
𝑛−2 𝑛−2 𝑛−2
𝐴1 𝐴2 𝐴𝑛
each of the angles 𝑛−2 , 𝑛−2 , … , 𝑛−2 is less than
′′ (𝑥)
𝜋. Now if f(x) = sin x, f’(x) = cos 𝑥 , 𝑓 =
− sin 𝑥,

So that if x is between 0 and 𝜋, f(a) is a concave


function. Similarly, the condition that a line exists
1 𝐴 2 𝐴 𝑛 𝐴 intersecting AB and BC and also bisecting
Thus sin 𝑛−2 , sin 𝑛−2 , … , sin 𝑛−2 are all concave
perimeter and area simultaneously are (i) (𝑎 +
functions and therefore their sum is also a 𝑏 + 𝑐)2 ≥ 8𝑏𝑐,
concave function of max. value subjected to
𝐴1 𝐴2 𝐴𝑛 1 𝑎+𝑏+𝑐
𝑛−2
+ 𝑛−2 + ⋯ + 𝑛−2 = 𝜋 arise when A : = (ii)It is possible to cut off lengths [ 2 ±
2
𝜋(𝑛−2)
𝑛
for each i, so that the max. value of 2
𝐴1 𝐴2 𝐴𝑛 √(𝑎+𝑏+𝑐) − 2𝑏𝑐] from the sides AB and BC.
sin 𝑛−2 + sin 𝑛−2 + ⋯ + sin 𝑛−2 = 2
𝜋 𝐴1 𝐴2
𝑛. sin 𝑛 , 𝑠𝑜 𝑡ℎ𝑎𝑡 sin 𝑛−2 + sin 𝑛−2 + ⋯ +
A similar set of conditions can be written down
𝐴𝑛 𝜋
sin 𝑛−2 ≤ 𝑛 sin 𝑛. for a line to exist intersecting AC and BC and
simultaneously bisecting the perimeter and
Q6. Find the condition that in a triangle of area of the triangles.
lengths of sides a, b, c you can draw one or
more st. lines each of which simultaneously
bisects the perimeter and the area of the
triangle.

282
Solving Mathematical Problems

Q7. Check whether the function defined by 𝑓(𝑥 + 𝑛) − 𝑓(𝑥 − 𝑛)


𝑓 ′ (𝑥) = Lt
𝒇(𝒙 + 𝝀) = 𝝀 + √𝟐𝒇(𝒙) − 𝒇𝟐 (𝒙) ∀𝒙
∊ 𝑹, is 𝑛→0 2𝑛
𝑓(𝑥)[𝑓(𝑛) − 𝑓(−𝑛)]
periodic on not, if periodic, then find its = Lt
𝑛→0 2𝑛
period.
𝑓(𝑥) 𝑓(𝑛) − 1 𝑓(−𝑛) − 1
Sol. : The given function is true if 2𝑓(𝑥) − ⟹ 𝑓 ′ (𝑥) = Lt [ + ]
2 𝑛→0 𝑛 −𝑛
𝑓 2 (𝑥) ≥ 0,
𝑓(𝑥) 𝑓 ′ (𝑥)
⟹ 𝑓(𝑥)[𝑓(𝑥) − 2] ≤ 0, ⟹ 0 ≤ 𝑓(𝑥) ≤ = 2 log 𝑎 , ⟹ = log 𝑎 , ⟹ log 𝑓(𝑎)
2 𝑓(𝑥)
2 … . (𝑖).
= 𝑥 log 𝑎 + log 𝑐 (𝑤ℎ𝑒𝑟𝑒 𝑐 𝑖𝑠 𝑐𝑜𝑛𝑠𝑡. ), ⟹
Also from the given function, it is clear that
log 𝑓(𝑥)
𝑓(𝑥 + 𝜆) ≥ 1, ⟹ 𝑓(𝑥) ≥ 1 … … . (𝑖𝑖)
= log(𝑎 𝑥 𝑐) , ⟹ 𝑓(𝑥) = 𝑎 𝑥 𝑐 … … . . (𝑖𝑖)
From (i) and (ii), we conclude that 1 ≤ 𝑓(𝑥) ≤
2. Putting x = 0 in (ii), we get f(0) = c, ⟹ c = 1.

Again, we have {𝑓(𝑥 + 𝜆) − 1}2 = 2𝑓(𝑥) − ∴ f(x) = 𝑎 𝑥 … . (𝑖𝑖𝑖)


𝑓 2 (𝑎)1
𝑑𝑦
∴ Given differential equation =
𝑑𝑥
⟹ {𝑓(𝑥 + 𝜆) − 1}2 = −{𝑓(𝑥) − 1}2 + {log 𝑎(𝑓(𝑥)𝑓(𝑦)}2 𝑑𝑦
1 … … … (𝑖𝑖𝑖) 𝑏𝑒𝑐𝑜𝑚𝑒𝑠 =
(log 𝑎 𝑓(𝑥)+2)(log 𝑎 𝑓(𝑦)−2) 𝑑𝑥
(𝑥+𝑦)2
Replacing x by x + 𝜆 in above equation, we get (𝑎+2)(𝑦−2)
(∵ 𝑓(𝑥) = 𝑎 𝑥 , ⟹ log 𝑎 𝑓(𝑥) = 𝑥),

{𝑓(𝑥 + 𝜆) − 1}2 − {𝑓(𝑥) − 1}2 + 1 … … … (𝑖𝑣) 𝑑𝑦 (𝑥 + 2 + 𝑦 − 2)2 𝑑𝑦 (𝑥 + 𝑦)2


⟹ = ,⟹ =
𝑑𝑥 (𝑥 + 2)(𝑦 − 2) 𝑑𝑥 𝑥𝑦
From (iv) –(iii), we get
(Putting x + y = x, y – 2 = y) …….. (iv)
{𝑓(𝑥 + 2𝜆) − 1}2 = {𝑓(𝑥 + 𝜆) − 1}2 , ⟹
𝑓(𝑥 + 2𝜆) = 𝑓(𝑥), ⟹ 𝑓 is a periodic function 𝑑𝑦 𝑑𝑡
Putting y = +x, so that 𝑑𝑥 = 𝑡 + 𝑥 𝑑𝑥
with period 2𝜆.
𝑑𝑦
∴ Equation (iv) becomes ⟹ 𝑡 + 𝑥 𝑑𝑥 =
(1+𝑡)2 𝑑𝑡 1 𝑑𝑡
𝑡
⟹ 𝑡 + 𝑥 𝑑𝑥 = 𝑡 + 𝑡 + 2, ⟹ 𝑥 𝑑𝑥 =
Q8. f : R→ [0, ∞) be a function satisfying 1+2𝑡 𝑡𝑑𝑡 𝑑𝑥
𝒇(𝒙 + 𝒚) − 𝒇(𝒙 − 𝒚) = 𝒇(𝒙)[𝒇(𝒚) − 𝑡
, ⟹ 1+2𝑡 = 𝑥
,
𝒇(−𝒚)], 𝒇′ (𝟎) = 𝐥𝐨𝐠 𝒂 , 𝒇(𝟎) = 𝟏 (for all
1 (2𝑡 + 1 − 1) 𝑑𝑥
values of a except 1), then solve the ⟹ ∫ = ∫ ,
𝒅𝒚 {𝐥𝐨𝐠 𝒂(𝒇(𝒙)𝒇(𝒚)}𝟐
2 (2𝑡 + 1) 𝑥
differential 𝒅𝒙 = (𝐥𝐨𝐠 𝒂 𝒇(𝒙)+𝟐)(𝐥𝐨𝐠 𝒂 𝒇(𝒚)−𝟐). 1
⟹ 𝑡 − log| ⎸(1 + 2𝑡)⎸|
2
Sol. : Given that 𝑓(𝑥 + 𝑦) − 𝑓(𝑥 − 𝑦) =
𝑓(𝑛)−𝑓(0)
𝑦−2
𝑓(𝑥)[𝑓(𝑦) − 𝑓(−𝑦)] ∴ 𝑓 ′ (0) = Lt = 2 log 𝑥 + 𝑐, ⟹ ( )
𝑛→0 𝑛 𝑥+2
1 𝑦−2
𝑓(𝑛) − 1 − log ⎸1 + 2 ( )⎸
= Lt = log 𝑎 … . . (𝑖)𝑎𝑛𝑑 2 𝑥+2
𝑛→0 𝑛

283
Solving Mathematical Problems

= 2 log(𝑥 + 2) + 𝑐. 𝑏
= ∫ [𝑓 2 (𝑦) − 𝑓 2 (𝑎)] 𝑑𝑦
𝑎
𝑏
= ∫ [𝑓 2 (𝑥) − 𝑓 2 (𝑎)] 𝑑𝑥
Q9. If f(x) is monotonic and differentiable to 𝑎
real valued f function and a, b are two real 𝑏
numbers, show that = ∫ [𝑓(𝑥) + 𝑓(𝑎)] [𝑓(𝑎) − 𝑓(𝑎)]𝑑𝑥
𝑎
𝒃
∫ [𝒇(𝒙) + 𝒇(𝒂)] [𝒇(𝒙) − 𝒇(𝒂)]𝒅𝒙 = 𝑅. 𝐻. 𝑆. 𝑜𝑓 (𝑖)𝑝𝑟𝑜𝑣𝑒𝑑.
𝒂
𝒇(𝒃) 𝟏 𝟏
=𝟐 ∫ 𝒙 [𝒃 − ∫ (𝒙)] 𝒅𝒙. Q10. Draw the graph of y = (𝐭𝐚𝐧 𝒙 + 𝐜𝐨𝐭 𝒙) +
𝟐
𝒇(𝒂) 𝟏
|⎸ 𝟐 (𝐭𝐚𝐧 𝒙 − 𝐜𝐨𝐭 𝒙)⎸|.
Sol. : To prove that
𝑏 Sol. : If tan 𝑥 ≥ cot 𝑥 , 𝑡ℎ𝑒𝑛 𝑦 =
∫𝑎 𝑥[𝑓(𝑥) + 𝑓(𝑎)] [𝑓(𝑥) − 𝑓(𝑎)]𝑑𝑥 =
tan 𝑥 𝑎𝑛𝑑 𝑖𝑓 tan 𝑥 ≤ cot 𝑥 , 𝑡ℎ𝑒𝑛 𝑦 = cot 𝑥.
𝑓(𝑏)
2 ∫𝑓(𝑎) 𝑥 [𝑏 − 𝑓 −1 (𝑥)] 𝑑𝑥….. (i)

We know that a differentiable function is also


continuous, so if f is differentiable if will be
continuous, f is also monotonous (given). So f is
bijective and its inverse also exists. Now, let
𝑓 −1 (𝑥) = 𝑦; ⟹ 𝑥 = 𝑓(𝑥), ⟹ 𝑑𝑥 =
𝑓(𝑦)𝑓 ′ (𝑦)𝑑𝑦,
𝑓(𝑏)
2∫ 𝑥 [𝑏 − 𝑓 −1 (𝑥)] 𝑑𝑥
𝑓(𝑎)
𝑏
= ∫ 2𝑓(𝑦)(𝑏
𝑎
− 𝑦) 𝑓 ′ (𝑦)𝑑𝑦 (𝐿. 𝐻. 𝑆. 𝑜𝑓 (𝑖))
MODEL TEST PAPER - 14
𝑏
∫ 2𝑥𝑓(𝑦) (𝑏 − 𝑦)𝑑𝑦 Q1. Show that A = 10101…… 101 is not a prime
𝑎 number, unless A = 101.
𝑏
= [(𝑏 − 𝑦)𝑓 2 (𝑦)]
𝑎 Sol. : 𝐴 = 102𝑛 + 102𝑛+2 + ⋯ + 102 + 1
𝑏
+ ∫ 𝑓 2 (𝑦) 𝑑𝑦
𝑎 ⟹ 100𝐴 = 102𝑛+2 + 102𝑛 + 106 + 104 +
= −(𝑏 − 𝑎)𝑓 2 (𝑎) 102
𝑏
+ ∫ 𝑓 2 (𝑦) 𝑑𝑦 ∴→ 9𝐴 = 102𝑛+2 − 1 = (10𝑛+1 + 1)(10𝑛+1 −
𝑎
1)

When n >1, 10𝑛+1 − 1 = 9999 or 999999, or ….


And the 2nd factor is divisible by 99. Except
when n = 1.

284
Solving Mathematical Problems

The quotient is x1 and A has two proper factors. Q3. The real numbers 𝒂𝟎 , 𝒂𝟏 , … , 𝒂𝒏 , … satisfy
If n is even 10𝑛+1 + 1 = 1001, 100001, …. and the condition: 1 = 𝒂𝟎 ≤ 𝒂𝟏 ≤ 𝒂𝟐 ≤ ⋯ ≤ 𝒂𝒏 ≤
all there are divisible by 11. Thus A is nonprime ⋯ the numbers 𝒃𝟏 , 𝒃𝟐 , … , 𝒃𝒏 , …. are defined by
except when it is 101. 𝒏
𝒂𝒌 − 𝟏 𝟏
𝒃𝒏 = ∑ (𝟏 − ) .
𝒂𝒌 √𝒂𝒌
𝒌=𝟏
Q2. Let a, b, c, d be any four positive integers.
(a) Prove that 𝟎 ≤ 𝒃𝒏 < 2 ∀ 𝑛.
Let 𝒂𝟏 , 𝒃𝟏 , 𝒄𝟏 , 𝒅𝟏 be the differences |⎸𝒂 −
(b) Given C with 𝟎 ≤ 𝒄 < 2, prove that
𝒃|⎸, | ⎸𝒃 − 𝒄| ⎸, | ⎸𝒄 − 𝒅| ⎸, |⎸𝒅 − 𝒂| ⎸. In the
there exist numbers 𝒂𝟎 , 𝒂𝟏 , … with the
same way define 𝒂𝒏+𝟏 = ⎸|𝒂𝒏 −
above properties such that 𝒃𝒏 > 𝑐 for
𝒃𝒏 | ⎸, | ⎸𝒃𝒏 + 𝟏| ⎸, |⎸𝒃𝒏 − 𝒄𝒏 |⎸, 𝒄𝒏 + 𝟏= |𝒄𝒏 −
large enough n.
𝒅𝒏 | , 𝒅𝒏 + 𝟏 = | ⎸𝒅𝒏 − 𝒂𝒏 | ⎸, show that
whatever be the four numbers we start with Sol. :
ultimately four zerox must be obtained.
𝑎𝑘−1
(a) We note that 𝑎𝑘
≤ 1, 𝑠𝑜 𝑡ℎ𝑎𝑡 𝑏𝑛 ≥ 0
Sol. : We easily note the following properties:
for all n denote √𝑎𝑘 𝑏𝑦 𝑎𝑘 . Then K th
(i) max (𝑎𝑛 , 𝑏𝑛 , 𝑐𝑛 , 𝑑𝑛 ] ≤ term of the sum 𝑏𝑛 is
max(𝑎𝑛−1 , 𝑏𝑛−1 𝑐𝑛−1. 𝑑𝑛−1 ) for all
𝑛 ≥ 1. 𝑎𝑘 2 − 1 1
(1 − )
(ii) If ultimately zeros are obtained 𝑎𝑘 2 𝑎𝑘
when we start with kn, kb, kc, kd 𝑎𝑘 2 − 1 1 1
= ( 2
− 2
we shall also obtain zeros when we 𝑎𝑘 𝛼𝑘−1 𝑎𝑘
2
start with a, b, c, d where k is any 𝑎𝑘 − 1
= )
positive integers. 𝛼𝑘
(iii) After at most 4 stages, 4 even 1 1 1 1
×( + )( − )
number are obtained. If we denote 𝛼𝑘−1 𝛼𝑘 𝛼𝑘−1 𝛼𝑘
by 𝐴𝑛 , 𝐵𝑛 , 𝐶𝑛 , 𝐷𝑛 the four numbers 𝛼𝑘−1 𝛼𝑘−1
= (1 + )
after n stages of the process and 𝛼𝑘 𝛼𝑘
1 1
after cancelling out any common ×( − )
𝛼𝑘−1 𝛼𝑘
factors we
1 1
(iv) max(𝐴𝑛 + 4, 𝐵𝑛 + 4, 𝐶𝑛 + 4, 𝐷𝑛 + ≤ 2( − ).
𝛼𝑘−1 𝛼𝑘
4) ≤ max(𝐴𝑛 , 𝐵𝑛 , 𝐶𝑛 , 𝐷𝑛 )
(v) max(𝐴4𝑘 , 𝐵4𝑘 , 𝐶4𝑘 , 𝐷4𝑘 ) ≤ Adding there inequalities for k = 1, 2, …, n. We
1
max(𝐴0 , 𝐵0 , 𝐶0 , 𝐷0 ) observe that the right side from a telescoping
2𝑘
1 1
sum, and we get 0 ≤ 𝑏𝑛 < 2 (𝛼 − 𝛼 ) =
0 𝑛
Hence we must have
1 1 1
max(𝐴4𝑘 , 𝐵4𝑘 , 𝐶4𝑘 , 𝐷4𝑘 ) = 0 for sufficiently 2( 𝛼 − ) = 2 (1 − ) < 2 𝑓𝑜𝑟 𝑎𝑙𝑙 𝑛.
√ 0 √𝛼𝑛 √𝛼 𝑛
large k and the destined result is established.
(b) Winners, 0 ≤ 𝑐 < 2, we shall establish
the existence of the appropriate 𝑎𝑖 by

285
Solving Mathematical Problems

constructing them as terms of a 𝐶𝑒 4𝜋𝑖


and C goes to Q represented by 3
. Also since
geometric series.
G is the origin a + b + c = 0, so that points A, P, Q
1 𝑏𝑒 2𝜋𝑖 𝐶𝑒 4𝜋𝑖
Set = 𝑑𝑘 , 𝑡ℎ𝑒𝑛 𝑡ℎ𝑒 𝑘 − are –(b + c), , 3 , 𝑜𝑟 − (𝑏 +
√ 𝑎𝑘 3
𝑑 −2(𝑘−1) 𝑒 2𝜋𝑖
𝑡ℎ 𝑡𝑒𝑟𝑚 𝑜𝑓 𝑡ℎ𝑒 𝑠𝑢𝑚 𝑏𝑛 𝑖𝑠 (1 − ) 𝑑𝑘 = 𝑐), 𝑏𝑤 2 , 𝐶𝑤 4 , when w = , so that 𝑤 2 −𝑤+
𝑑 −2𝑘 6
3
(1 − 𝑑2 )𝑑𝑘 . 1 = 0, 𝑤 = −1.

Hence 𝑏𝑛 = ∑𝑛𝑘=1(1 − 𝑑2 )𝑑𝑘 = (1 − Then A, P, Q are –(b + c), b𝑤 2 , −𝑐𝑤. If A and P


𝑑−𝑑 𝑛+1 coincide the 𝑛 − 𝑏 − 𝑐 − 𝑏𝑤 2 = 0 𝑜𝑛 − 𝑐 −
𝑑2 ) ∑𝑛𝑘=1 𝑑𝑘 = (1 − 𝑑2 ) × = 𝑑(1 +
1−𝑑 𝑏𝑤 = 0, 𝑜𝑟 𝑐 + 𝑏𝑤 = 0, 𝑜𝑟 𝑏𝑤 2 = −𝑐𝑤 =
𝑑)(1 − 𝑑𝑛 ) we must pick d between 0 and 1 so 𝑐𝑤 4 . So that P and Q also coincide. If A and P do
that 𝑏𝑛 = 𝑑(1 + 𝑑)(1 − 𝑑𝑛 ) > 𝑐 for large
not coincide, then 𝑃 − 𝑎 = 𝑏𝑛2 − 𝑎 = (𝑏 +
enough n. We containly need to have d(1 +d)
𝑎)𝜔2 − 𝑎(1 + 𝜔2 ) = −𝑐𝜔2 − 𝑎𝜔 = 𝑐𝜔5 −
>c.
𝑎𝜔 = 𝑞𝜔 − 𝑎𝜔 = (𝑞 − 𝑎)𝜔 = (𝑞 − 𝑎)𝑒 𝑖𝜋/6
This can be achieved for any c <2. Since d (1+ d) 𝑏𝑒 2𝜋𝑖 𝐶𝑒 4𝜋𝑖
approaches 2 as d tends to 1. ∴ ∠PAQ = 60°. Also 𝑝 − 𝑞 = 3
− 3
=
𝑏𝜔2 − 𝑐𝜔4 = 𝜔2 − (−𝑎 − 𝑏)𝜔 4

(In fact the reader can easily verify that d(1+ d)


>c if d =
2
). Now since d <1, we see that 1 − 𝑑𝑛 = 𝑏𝜔2 + (𝑎 + 𝑏)𝜔4 = 𝑏𝜔2 − (𝑎 + 𝑏)𝜔
𝑐
√2 = −𝑏 − 𝑎𝜔 = 𝑏𝜔3 − 𝑎𝜔
is as close to 1 as we please for all sufficiently = (𝑏𝜔2 − 𝑎)𝜔 = (𝑏 − 𝑎)𝜔.
𝑐
larger. In particular, 1 − 𝑑ℎ > (1 + ∴ ∠QPA = 60°, so that the 𝛥APQ is equilateral.
𝑑
𝑑), 𝑖. 𝑒. 𝑑(1 + 𝑑)(1 − 𝑑ℎ ) > 𝑐 for all sufficiently
large n. we suggest that the reader find a
number N depending on the given C such that Q5. Let f(x) be a continuous function is [-1, 1]
𝑏𝑛 > 𝑐 for all n > N.) and satisfies 𝒇(𝟐𝒙𝟐 − 𝟏) = 𝟐𝒙𝒇(𝒙)∀ 𝒙 ∊
[−𝟏, 𝟏].Prove that f(x) is identically zero for all
x ∊ [-1, 1].
Q4. Let G be the centroid of the 𝚫 ABC. Under
rotation by 𝟏𝟐𝟎° about the point G, the pt. B is Sol. : We have 𝑓(2𝑥 2 − 1) = 2𝑥𝑓(𝑥)∀𝑥 ∊
taken to the pt. P and under n rotation by [−1, 1] ……. (i)
𝟐𝟒𝟎° about G, the pt. C is taken to the pt. Q.
Replacing x → -x, we get 𝑓(2𝑥 2 − 1) =
Prove that either APQ is an equilattienal
−2𝑥𝑓(𝑥) ……. (ii)
triangle on the points A, P, Q. Coincide.
From equation (i) and (ii), we get 2xf(x) = -2xf(-
Sol. :
x), on f(x) = -f (−x) ∀ 𝑥 ∊ [−1, 1] ……. (iii)
Let G be the original and let A, B, C. be
Hence f(x) is an odd function. Now putting x = 0
represented by complex number a, b, c then
in equation (iii), we have f(0) = -f(0), ⟹f(0) = 0
𝑏𝑒 2𝜋𝑖
after rotation B goes to P is represented by 3 …..(iv).

286
Solving Mathematical Problems

Putting 𝑥 = cos 𝜃 in equation (i), we get Sol: Given


𝑓(cos 2𝜃) = 2 cos 𝜃 . 𝑓(cos 2𝜃) =
𝜃
𝑥1 + 𝑥2 + ⋯ + 𝑥𝑛
2 cos 𝜃 . 𝑓 (2𝑐𝑜𝑠 2 2 − 1) 𝑓( )
𝑛
𝑓(𝑥1 ) + 𝑓(𝑥2 ) + ⋯ + 𝑓(𝑥𝑛 )
𝜃 𝜃 =
= 2 cos 𝜃 . 2 cos 𝑓 (cos ) 𝑛
2 2
𝜃 Taking n = 2, we have 𝑓 (
𝑥1 +𝑥2
)
1
= 2 {𝑓(𝑥1 ) +
= 2𝑛+1 . cos 𝜃 . cos 2
2
𝑓(𝑥2 )}
𝜃 𝜃 𝜃
cos . cos 𝑛 𝑓 (cos 𝑛 (Where 𝑥1 𝑎𝑛𝑑 𝑥2 are independent variables)
2𝑛 −1 2 2
…. (iii). Now differentiating (ii) w. r. t. 𝑥1 , we get
𝜃 𝜃
= sin 2𝜃 . 𝑓 (cos 𝑛
)) / sin 𝑛 𝑓′ (
𝑥1 +𝑥2 1 𝑑𝑥 1
) (1 + 2 ) = {𝑓 ′ (𝑥1 ) +
2 2 2 2 𝑑𝑥1 2
′ (𝑥 ) 𝑑𝑥2
𝑓 2 }….. (iii) . Since 𝑥2 is independent f𝑥1
Taking limit on the both sides as n → ∞, we get 𝑑𝑥1
𝑑𝑥
so 𝑑𝑥2 = 0
𝜃 1
𝑓 {cos ( 𝑛 )}
Lt 𝑓(cos 2𝜃) = sin 2𝜃 Lt 2 , 1 𝑥1 +𝑥2 1
𝑛→∞ 𝑛→∞ 𝜃 ∴ (iii) ⟹ 2 𝑓 ′ ( ) = 2 𝑓 ′ (𝑥1 ).
sin ( 𝑛 ) 2
2

⟹ 𝑓(cos 2𝜃) Putting 𝑥1 = 0 𝑎𝑛𝑑 𝑥2 = 𝑥 ′ , we get


𝜃
𝑓 (2𝑐𝑜𝑠 2 − 1) 1 ′ 0 + 𝑥′ 1 𝑥′
= sin 2𝜃 , lt 2𝑛+1 = sin 2𝜃. 𝑓 ( ) = 𝑓 ′ (0), ⟹ 𝑓 ′ ( ) = 𝑎,
𝑛→∞ 𝜃 𝜃 2 2 2 2
2 sin 𝑛+1 cos
2 2𝑛+1 ⟹𝑓 ′ (𝑥)
=𝑎
𝜃
𝑓 (2𝑠𝑖𝑛2 𝑛+1 − 1) (Putting x’/2 = x). Integrating above equation.
lt 2 (∵ 𝑓(𝑥)𝑖𝑠 𝑎𝑛 𝑜𝑑𝑑 𝑓𝑢𝑛𝑐𝑡𝑖𝑜𝑛) W. r. t. x, we get f(x) = ax + c.
𝑛→∞ 𝜃 𝜃
2 sin 𝑛+1 cos 𝑛+1
2 2
Putting x = 0, we get f(0) = 0+ c, ⟹ c = b
𝜃 𝜃
2 sin 𝑛 𝑓 (sin 𝑛 )
= − sin 2𝜃 { lt 2 +1 2 +1 } ∴ f(x) = ax + b. ∴ 𝑓 ′ (𝑥) = 𝑎, 𝑓 ′′ (𝑥) =
𝑛→∞ 𝜃 𝜃
2 sin 2𝑛 + 1 cos 2𝑛 + 1 𝑑 𝑘 𝑓(𝑥)
0, 𝑓 ′′′ (𝑥) = 0, 𝑑𝑥 𝑘 = 0.
= 0
Q7. If f(x) is an increasing function from R →R
𝑓(0)
= sin 2𝜃 . { } = 0, ⟹ 𝑓(cos 2𝜃) = 0 ∀ 0 such that 𝒇′′ (𝒙) > 0, 𝑓(𝒙) ≠ 𝟎 𝒂𝒏𝒅 𝒇−𝟏 exists,
1 𝒅𝟐 {𝒇−𝟏 (𝒙)}
∊ 𝑅, ⟹ 𝑓(𝑥) = 0∀ 𝑥 ∊ [−1, 1]. then show that < 0.
𝒅𝒙𝟐

Sol. : Let f be an increasing function ⟹ 𝑓 ′ (𝑎) >


0′ 𝑎𝑛𝑑 𝑓 ′′ (𝑥) > 0 …..(i) (given).
𝒙𝟏 +𝒙𝟐 +⋯+𝒙𝒏 𝒇(𝒙𝟏 )+𝒇(𝒙𝟐 )+⋯+𝒇(𝒙𝒏 )
Q6. Let 𝒇 ( 𝒏
) = 𝒏
Let 𝑔(𝑥) = 𝑓 −1 (𝑥) …… (ii), then f{g(x)} = x,
where 𝒙𝒊 𝝐𝑹 𝒂𝒏𝒅 𝒏𝝐𝑵. If f(x) is differentiable 1
𝒅𝒌 𝒇(𝒙) ⟹𝑓 ′ {𝑔(𝑥)}. 𝑔′ (𝑥) ≡ 1, ⟹ 𝑔′ (𝑥) = ……
and f’(0)=a, f(0) = 𝒃. Evaluate (𝒌 ≥ 𝟒). 𝑓′ {𝑔(𝑥)}
𝒅(𝒙𝒌 )
(iii).

287
Solving Mathematical Problems

Again differentiating both sides w. r. t. x, we get 𝒔𝒊𝒏𝟐 𝒙


1 ∫ (𝐬𝐢𝐧−𝟏 √𝒕) 𝒅𝒕
𝑔′′ (𝑥) = −1 {𝑓(𝑔(𝑥))} 𝑓 ′′ {𝑔(𝑥)}𝑔′ (𝑥) …. (iv). 𝟏/𝟖
𝒄𝒐𝒔𝟐 𝒙
Let g(x) = 𝑓 −1 (𝑥) = 𝑦, ⟹ 𝑥 = 𝑓(𝑦), ⟹ 1 + ∫ (𝐜𝐨𝐬 −𝟏 √𝒕) 𝒅𝒕 (𝟎 ≤ 𝒙
𝟏/𝟖
𝑑𝑦 𝑑𝑦 1 ≤ 𝝅 𝟏𝟐)
= 𝑓 ′ (𝑦) ,⟹ = ′ > 0, ⟹ 𝑔′ (𝑥) > 0
𝑑𝑥 𝑑𝑥 𝑓 (𝑦)
And the curve satisfying the differential
𝑑2 equation 𝒚(𝒙 + 𝒚𝟑 )𝒅𝒙 = 𝒙(𝒚𝟑 − 𝒙)𝒅𝒚
∴ From equation (iv), we have 𝑑𝑥 2 {𝑔(𝑥)} < 0
passing through (𝟒, −𝟏𝟐).
𝑑 2 {𝑓−1 (𝑥)}
(∵ f’(g(x))>0), ⟹ 𝑑𝑥 2
<0 Sol. : Given differential equation is 𝑥(𝑦𝑑𝑥 +
𝑥𝑑𝑦) = 𝑦 3 (𝑥𝑑𝑦 − 𝑦𝑑𝑥). ⟹
[∵ 𝑔(𝑥) = 𝑓 −1 (𝑥)] . 𝑥𝑑𝑦−𝑦𝑑𝑥
𝑥𝑑(𝑥𝑦)𝑦 3 𝑥 2 ( ).
𝑥2

𝑦 𝑑(𝑥𝑦)
⟹ 𝑥𝑑(𝑥𝑦) = 𝑥 2 𝑦 3 𝑑 ( ) , ⟹
Q8. Let 𝒂𝟏 , 𝒂𝟐 … be a non-decreasing sequence 𝑥 (𝑥𝑦)2
of positive integers. For m ≥1, define 𝒃𝒎 = 𝑦 𝑦
= . 𝑑 ( ).
𝐦𝐢𝐧{𝒏: 𝒂𝒏 ≥ 𝒎} , 𝒊. 𝒆. 𝒃𝒎 is the minimum 𝑥 𝑥
value of n such that 𝒂𝟏 ≥ 𝒎. If 𝒂𝟏𝟗 = 𝟖𝟓, On integrating,
determine the maximum value of 𝒂𝟏 + 𝒂𝟐 +
⋯ + 𝒂𝟏𝟗 + 𝒃𝟏 + 𝒃𝟐 + ⋯ + 𝒃𝟖𝟓 . 1 1 𝑦 2
− 𝑥𝑦 = 2 (𝑥 ) + 𝑐…….. (i)
Sol. : We will show that if 𝑎𝑞 = 𝑝, then 𝑆𝑝𝑞 =
𝑎1 + 𝑎2 + ⋯ + 𝑎𝑞 + 𝑏1 + 𝑏2 + 𝑏𝑝 = 𝑝(𝑞 + 1).
In particular, for the case q = 19, p = 85, we
have the sum 𝑝(𝑞 + 1) = 1700.

If 𝑎𝑖 = 𝑝 𝑓𝑜𝑟 𝑎𝑙𝑙 1 ≤ 𝑖 ≤ 𝑞, 𝑡ℎ𝑒𝑛 𝑏𝑗 =


1 𝑓𝑜𝑟 𝑎𝑙𝑙 1 ≤ 𝑗 ≤ 𝑝.

Hence, 𝑠𝑝𝑞 = 𝑝𝑞 + 𝑝 as reqd. If not, let t be the


largest index such that 𝑎𝑖 < 𝑝. 𝐿𝑒𝑡 𝑎𝑖 = 𝑢. 𝐼𝑓 𝑎𝑖
is increased by 1, then all the 𝑏𝑗 remain
unchanged except for 𝑏𝑛+1 which decreases by
1. Hence the value of the desired sum is
unchanged. By repeating this increment process
(in decreasing order of the subscript so as to Curve (i) passes through
maintain a non-decreasing sequence) as long as 1 1
necessary, we will eventually arrive at the (4, −2), ⟹ = + 𝑐 ⟹ 𝑐 = 0.
8 8
const. sequence which gives the desired result.
Hence curve (i) becomes
Q9. Find out the area bounded by the curve y =

288
Solving Mathematical Problems

𝑦 3 + 2𝑥 = 0, ⟹ 𝑦 = (−2𝑥)1/3 , ⟹ 𝑓(𝑥) = to the diameter CP then its extremities will be.


(−2𝑥)1/3 (– 𝑎 𝑠𝑖𝑛 𝛼, 𝑏 cos 𝛼).

The second equation is ∴ 𝑑1 2 = 𝑎2 𝑠𝑖𝑛2 𝛼 + 𝑏 2 𝑐𝑜𝑠 2 𝛼.

𝑦 = ∫1/8
𝑠𝑖𝑛2 𝑥
sin−1 √𝑡 𝑑𝑡 + ∫1/8
𝑐𝑜𝑠2 𝑥
cos −1 √𝑡 𝑑𝑡 𝑆𝑖𝑚𝑖𝑙𝑎𝑟𝑙𝑦, 𝑑2 2 = 𝑎2 𝑠𝑖𝑛2 𝛽 + 𝑏 2 𝑐𝑜𝑠 2 𝛽, ⟹
𝑑1 2 𝑑2 2
………. (ii)
= (𝑎2 𝑠𝑖𝑛2 𝛼 + 𝑏 2 𝑐𝑜𝑠 2 𝛼)(𝑎2 𝑠𝑖𝑛2 𝛽 +
⟹ 𝑦 ′ = 𝑥. 2 sin 𝑥 cos 𝑥 + 𝑥. 2 cos 𝑥 (− sin 𝑥) =
𝑏 2 𝑐𝑜𝑠 2 𝛽)
0

⟹ 𝑦 = 𝑐1 (𝑐𝑜𝑛𝑠𝑡. ). 𝑁𝑜𝑤 𝑝𝑢𝑡𝑡𝑖𝑛𝑔 sin 𝑥 = = (𝑎2 𝑠𝑖𝑛𝛼 sin 𝛽 + 𝑏 2 cos 𝛼 cos 𝛽)2 +
1 𝑎2 𝑏 2 (𝑠𝑖𝑛𝛼 𝑐𝑜𝑠𝛽 − 𝑐𝑜𝑠𝛼 𝑠𝑖𝑛𝛽)2 = 0 +
cos 𝑥 = is equation (ii), we get y =
√2 𝑎2 𝑏 2 𝑠𝑖𝑛2 (𝛼 − 𝛽)
1/2 −1 1/2 𝜋
∫1/8 (sin √𝑡 + cos−1 √𝑡) 𝑑𝑡 = ∫1/8 ( 2 ) 𝑑𝑡 =
3𝜋
[𝑓𝑟𝑜𝑚 (𝛽)] = 𝑎2 𝑏 2 𝑠𝑖𝑛2 𝛹 [𝑓𝑟𝑜𝑚 (2)]. ∴
16
……….. (iii) sin 𝛹 = 𝑑1 𝑑2 .

Now reqd. area

3𝜋 MODEL TEST PAPER - 15


3𝜋/16
16 𝑦3
=∫ 𝑥𝑑𝑦 = |⎸ (∫ (− ) 𝑑𝑦)| ⎸
0 0 2 Q1. Let 𝑮𝒏 = 𝒙𝒏 𝐬𝐢𝐧 𝒏𝑨 + 𝒚𝒏 𝐬𝐢𝐧 𝒏𝑩 +
1 3𝜋 4 𝒛𝒏 𝐬𝐢𝐧 𝒏𝑪, where x, y, z, A, B, C are real and A +
= ( ) . B+ C is an integral multiple of 𝜋. Prove that if
8 16
𝑮𝟏 = 𝑮𝟐 = 𝟎, 𝒕𝒉𝒆𝒏 𝑮𝒏 = 𝟎 for all positive
integral n.

Q10. If 𝛹is the difference of eccentric angles of Sol. : A standard trick is to recognize that 𝐺𝑛
two points on an ellipse, the tangents of which is the imaginary part of the expression 𝐻𝑛 =
are at right angles. Prove that 𝒂𝒃 𝐬𝐢𝐧 𝜳 = 𝑥 𝑛 𝑒 𝑖𝑛𝐴 + 𝑦 𝑛 𝑒 𝑖𝑛𝐵 + 𝑧 𝑛 𝑒 𝑖𝑛𝐶 .
𝒅𝟏 𝒅𝟐 , where 𝒅𝟏 , 𝒅𝟐 , are the semi-diameters
parallel to the tangents at the points and a, b, Suppose that 𝐻𝑛 is real for n = 0, 1 , …. , k and
are semi-axes of the ellipse. consider 𝐻𝑘+1 . We have

𝑥2 𝑦2 𝐻1 𝐻𝑘 = 𝐻𝑘+1 + 𝐻, 𝑤ℎ𝑒𝑟𝑒 𝐻 = 𝑥𝑒 𝑖𝐴 𝑦 𝑘 𝑒 𝑖𝑘𝐵 +


Sol. : Let the given ellipse be 𝑎2 + 𝑏2 = 1 …… (i).
𝑥𝑒 𝑖𝐴 𝑧 𝑘 𝑒 𝑖𝑘𝐶 + 𝑦𝑒 𝑖𝐵 𝑥 𝑘 𝑒 𝑖𝑘𝐴 + 𝑦𝑒 𝑖𝑘𝐵 = 𝑒 𝑖𝑘𝐶 +
Let P(𝛼) and Q(𝛽) be two pts. On (i) such that 𝑧𝑒 𝑖𝐶 𝑥 𝑘 𝑒 𝑖𝑘𝐴 + 𝑧 𝑒 𝑖𝐶 𝑦 𝑘 𝑒 𝑖𝑘𝐵 =
𝛹= 𝛼 –𝛽…… (iii). Given that tangents at P and 𝑥𝑦𝑒 𝑖(𝐴+𝐵) [𝑦 𝑘−1 𝑒 𝑖(𝑘−1)𝐵 + 𝑥 𝑘−1 𝑒 𝑖(𝑘−1)𝐴 ] +
Q are at right angles.
𝑥𝑧𝑒 𝑖(𝐴+𝐶) [𝑧 𝑘−1 𝑒 𝑖(𝑘−1)𝐶 + 𝑥 𝑘−1 𝑒 𝑖(𝑘−1)𝐴 ] +
𝑏 𝑏
∴(− 𝑎 cot 𝛼) (− 𝑎 cot 𝛽) = −1, ⟹ 𝑦𝑧𝑒 𝑖(𝐵+𝐶) [𝑦 𝑘−1 𝑒 𝑖(𝑘−1)𝐵 + 𝑧 𝑘−1 𝑒 𝑖(𝑘−1)𝐶 ] =
𝑥𝑦 𝑒 𝑖(𝐴+𝐵) × [𝐻𝑘−1 − 𝑧 𝑘−1 𝑒 𝑖(𝑘−1)𝐶 ] +
𝑎2 sin 𝛼 sin 𝛽 + 𝑏 2
𝑥𝑧𝑒 𝑖(𝐴+𝐶) [𝐻𝑘−1 − 𝑦 𝑘−1 𝑒 𝑖(𝑘−1)𝐵 ] +
cos 𝛼 cos 𝛽 = 0 …… (iii). But the diameter 𝑦𝑧𝑒 𝑖(𝐵+𝐶) [𝐻𝑘−1 − 𝑥 𝑘−1 𝑒 𝑖(𝑘−1)𝐴 ] =
parallel to the tangent at P(x) will be conjugate

289
Solving Mathematical Problems

𝐻𝑘−1 [𝑥𝑦𝑒 𝑖(𝐴+𝐵) + 𝑥𝑧𝑒 𝑖(𝐴+𝐶) + 𝑦𝑧𝑒 𝑖(𝐵+𝐶) ] − determined recursively by means of 𝑥𝑛+1 =
𝑥𝑦𝑧𝑒 𝑖(𝐴+𝐵+𝐶)
𝐻𝑘−2 𝑎𝑥𝑛 + 𝑏𝑥𝑛−1 , 𝑛 = 1, 2, 3, … and if 𝑎2 + 4𝑏 ≠ 0,
then 𝑥𝑛 can be expressed in terms of a, b, 𝑥0 , 𝑥1
= 𝐻𝑘−1 𝑘 − 𝑥𝑦𝑧𝑒 𝑖(𝐴+𝐵+𝐶) 𝐻𝑘−2 , 𝑤ℎ𝑒𝑟𝑒 𝑘 = by the formula
𝑥𝑦𝑒 𝑖(𝐴+𝐵) + 𝑥𝑧𝑒 𝑖(𝐴+𝐶) + 𝑦𝑧𝑒 𝑖(𝐵+𝐶)
(𝑥1 −𝑘1 𝑥1 )𝑘2 𝑛 −(𝑥1 −𝑘2 𝑥1 )𝑘1 𝑛
𝑥𝑛 = where 𝑘1 , 𝑘2 are
Observe that 𝐻2 = 𝐻1 2 + 2𝑘 and since 𝑘2 −𝑘1

𝐻1 𝑎𝑛𝑑 𝐻2 are real, by hypothesis, if must be the roots of 𝑘 2 − 𝑎𝑘 − 𝑏 = 0(𝑖𝑓 𝑎2 + 4𝑏 =


the case that k is real also, by the inductive 0, 𝑘1 =
𝑘2 𝑎𝑛𝑑 𝑡ℎ𝑒 𝑓𝑜𝑟𝑚𝑢𝑙𝑎 𝑓𝑜𝑟 𝑥𝑛 𝑖𝑠 𝑑𝑖𝑓𝑓𝑒𝑟𝑒𝑛𝑡).
assumption, 𝐻𝑘−1 𝑎𝑛𝑑 𝐻𝑘−2 are real.

Because A + B + C is a multiple of 𝜋, 𝑒 𝑖 (𝐴 + 𝐵 +
𝐶) is real. Putting there facts together, the 1st case: We have 𝑥0 = 1, 𝑥1 = 1, 𝑎 = 1, 𝑏 =
formula of the last paragraph show that it is 2 and find 𝑘2 = 2, 𝑘1 = −1(or vice-ressa), we
real. Now since 𝐻𝑘 is real, by the inductive 1
get 𝑥𝑛 = [2𝑛+1 + (−1)𝑛 ] for the 2nd
3
assumption and since 𝐻𝑘+1 = 𝐻1 𝐻𝑘−1 , it
sequence, we find 𝑦𝑛 = 2. 3𝑛 −
follows that 𝐻𝑘+1 is real. Thus, the result of the
(−1)𝑛 . 𝑇𝑜 𝑔𝑒𝑡 𝑥𝑛 = 𝑦𝑚 , we must have 3𝑚+1 −
prob. follows by mathematical induction. 1
2𝑛 = 2 [3 − (−1)𝑚 + (−1)𝑛 ]. If n = 0 or 1, we
see that m = 0 is the only solution. Hence forth,
take n ≥ 2. If m and n are both even or odd, the
Q2. Let {𝒙𝒏 }𝒂𝒏𝒅 {𝒚𝒏 } denote two sequences
right member of take this equation is even, but
of integers defined as follows:
the left member is odd. If m and n are of
𝒙𝟎 = 𝟏, 𝒙𝟏 = 𝟏, 𝒙𝒏−𝟏 = 𝒙𝒏 + 𝟐𝒙𝒏−𝟏 (𝒏 = opposite parity, the equation is invalid mod 4.
𝟏, 𝟐, 𝟑, … . )

𝒚𝟎 = 𝟏, 𝒚𝟏 = 𝟕, 𝒚𝒏+𝟏 = 𝟐𝒚𝒏 + 𝟑𝒚𝒏−𝟏 (𝒏 =


𝟏, 𝟐, 𝟑, … . ) Q3. An integer n will be called good if we can
write 𝒏 = 𝒂𝟏 + 𝒂𝟐 + ⋯ + 𝒂𝒌 , where
Thus the first few terms of the sequences are: 𝒂𝟏 , 𝒂𝟐 , 𝒂𝒌 are positive integers (not necessarily
𝟏 𝟏 𝟏
X: 1, 1, 3, 5, 11, 21, …. distinct) satisfying 𝒂 + 𝒂 + ⋯ + 𝒂 = 𝟏. Given
𝟏 𝟐 𝒌
the information that the integers 33 through
Y: 1, 7, 17, 55, 161, 487 ……. 73 are good, prove that every integer ≥ 33 is
Prove that, except for the ‘1’, there is no term good.
which occurs both sequences. Sol. : From n good integer n, we produce the
two larger good integers 2n + 8 and 2n + 9 as
Sol. : Mod 8, the first few terms of the two
follows: let (𝑎1 , 𝑎2 , … , 𝑎𝑘 ) be a partition of n
sequences are X: 1, 2, 3, 5, 3, 5 and Y: 1, 7, 1, 7,
1 1 1
1, 7, …… An easy induction shows that this which is good, then 2𝑎 + 2𝑎 + ⋯ + 2𝑎 =
1 2 𝑘
alternate periodic behavior persists. Thus 1 is 1 1 1 1 1 1
2
. 𝑆𝑖𝑛𝑐𝑒 2 = + = +
4 4 3 6
, it follows that the
the only common term of the two sequences. If
two partitions
a, b, 𝑥0 , 𝑥1 are given numbers and 𝑥2 , 𝑥3 , … are

290
Solving Mathematical Problems

(4, 4, 2𝑎1 , 2𝑎2 , … , 2𝑎𝑘 )𝑎𝑛𝑑 (3, 6, 2𝑎1 , 2𝑎2 , … , 2𝑎𝑘 ) 1 1−(1−𝑥)𝑛 1 1−𝑦 𝑛 1
∫0 𝑥
𝑑𝑥 = ∫0 1−𝑦
𝑑𝑦 = ∫0 (1 + 𝑦 +
also have the prop that the sum of the 1 1
reciprocals is 1. 𝑦 2 + ⋯ + 𝑦 𝑛−1 ) 𝑑𝑦 = [𝑦 + 2 𝑦 + ⋯ + 𝑛 𝑦 𝑛 ] 10 =
1 1
1 + 2 + ⋯ + 𝑛.
There are partitions of the integers 2n + 8 and
2n + 9 respectively. So (i) if n is good, so also 2n
+8 and 2n +9. So “33 is good” implies that 74,
75 are good. We use the hypothesis to fill the Q5. 9 Mathematicians meet at an international
gate between n = 33 and 2n +8 = 74: let 𝑠𝑛 conference and discover that among any 3 of
denote the statement “all the integers n, n+ 1, them, at least 2 speak a common language. If
…., 2n +7 are good”. We begin an induction with each of the mathematicians can speak at most
the given information that 𝑆33 is valid. By (i) we 3 languages. Prove that there are at least 3 of
conclude that 𝑆𝑛 → 𝑆𝑛−1 . Hence by induction, mathematicians who can speak the same
𝑆𝑛 is valid for all n ≥ 33, giving the desired language.
result.
Sol. : We assume that at most 2 mathematicians
𝟏 𝟏 speak a common language. Each mathematician
Q4. Show that (𝒏𝟏) − (𝒏𝟐) + (𝒏𝟑) … +
𝟐 𝟑
𝟏 𝟏 𝟏 can speak to at most 3 others, one for each
(−𝟏)𝒏+𝟏 . (𝒏𝒏) = 𝟏 + + ⋯ + .
𝒏 𝟐 𝒏 language he or she knows. Suppose
mathematician M, can only speak with
Sol. : The left side of the identity looks like the
𝑀2 , 𝑀3 , 𝑀4 . Now mathematicians 𝑀5 can speak
definite integral of a binomial series and this
with at most three of 𝑀2 , 𝑀3 , 𝑀4 or at most 3 of
provides the idea for the following argument.
𝑀6 , 𝑀7 , 𝑀8 , 𝑀9 . This leaves one of the last 4
𝑛 𝑛 𝑛 who cannot speak with 𝑀1 𝑜𝑟 𝑀5 giving the
(1 − 𝑥)𝑛 = ( ) − ( ) 𝑥 + ( ) 𝑥 2 …
0 1 2 desired contradiction.
𝑛 𝑛 𝑛
1 − (1 − 𝑥)𝑛 ( ) 𝑥 − ( ) 𝑥 2 + ( ) 𝑥 3 …,
1 2 3
1 − (1 − 𝑥)𝑛 𝑛 𝑛 𝑛 Q6. Two pts. P and Q lie in the interior of a
= ( ) − ( ) 𝑥 + ( ) 𝑥2 … regular tetrahedron ABCD. Prove that ∠APQ =
𝑥 1 2 3
∠60°.
We are now set up to integrate each side from
0 to 1 and we get; Sol. : We can assume without loss of generally
that each edge of ABCD= 1, that P an Q lie in the
1
1 − (1 − 𝑥)𝑛 𝑛 1 𝑛 1 𝑛 interior of 𝛥BCD and that line PQ intersects BC
∫ 𝑑𝑥 = ( ) − ( ) + ( ) …
0 𝑥 1 2 2 3 3 in R and CD in S as in fig. Then ∠PAQ ∠RAS. We
now show that RS is the shortest side of 𝛥ARS
To finish the prob., we must show the integral
1 1 1 and this implies that ∠RDS ∠60° . Thus RD > RS.
on the left is equal to 1 + 2 + 3 + ⋯ + 𝑛. Let y =
Since AR = RD (from congruent triangles BDR
1 –x , then and BAR), AR >RS. Similarly, AS > RS. Hence RS is
the shortest side of 𝛥ARS.

291
Solving Mathematical Problems
𝜋 2𝜋
⟹ ∫0 |sin 𝑡 + cos 𝑡| 𝑑𝑡 = ∫𝜋 |sin 𝑡 +
cos 𝑡| 𝑑𝑡 …
𝑛𝜋
= ∫ |sin 𝑡 + cos 𝑡| 𝑑𝑡
(𝑛−1)𝜋

𝜋 2𝜋/4
Now, ∫0 │ sin 𝑡 + cos 𝑡 │ 𝑑𝑡 = ∫𝜋 │ sin 𝑡 +
3𝜋/4
cos 𝑡 │ 𝑑𝑡 − ∫3𝜋/4 │ sin 𝑡 + cos 𝑡 │ 𝑑𝑡 = 0
Q7. Let 𝑨𝟎 denotes the area bounded by
1 2√2 2√2 2√2
𝐬𝐢𝐧 𝟖𝒏𝒙+𝐜𝐨𝐬 𝟖𝒏𝒙 ∴𝐴𝑛 > 𝑛 [2√2 + + 3 + ⋯+ ]
𝒇𝒏 (𝒙) = | 𝒙
| , 𝒙 − 𝒂𝒙𝒊𝒔, 𝒚 − 2 𝑛
𝝅
𝒂𝒙𝒊𝒔 𝒂𝒏𝒅 𝒍𝒊𝒏𝒆 𝒙 = 𝟖 . Then prove that 𝑨𝒏 > 2√2 1 1 1
𝟐√𝟐 𝟏 𝟏
∴ 𝐴𝑛 > [1 + + + ⋯ + ].
[𝟏 + 𝟐 + ⋯ + 𝒏] (𝒏 ∊ 𝑵). 𝜋 2 3 𝑛
𝝅

Sol. : 𝑓𝑛 (𝑥) ≥ 0, so reqd. area,


𝜋
Q8. Two given circles intersect in two pts. P
8 sin 8𝑛𝑥 + cos 8𝑛𝑥 and Q. Show how to construct a segment AB
𝐴𝑛 = ∫ | | 𝑑𝑥. 𝑃𝑢𝑡 8𝑛𝑥 = 𝑡,
0 𝑥 passing through P and terminating on the two
𝑑𝑡 circles such that AP.PB is a maximum.
⟹ 𝑑𝑥 = .
8𝑛
𝑛𝜋
Sol. : Since AP = 2 sin 𝛼 𝑎𝑛𝑑 𝐵𝑃 = 2 sin 𝛽, we
sin 𝑡 + cos 𝑡
⟹ 𝐴𝑛 = ∫ | | 𝑑𝑡 want to maximize sin 𝛼 sin 𝛽. We note that
0 𝑡
𝜋
since ∠𝑂1 𝑃𝑂2 is fixed, so also is the sum 𝛼 +
sin 𝑡 + cos 𝑡
=∫ | | 𝑑𝑡 𝛽. Now 2 sin 𝛼 sin 𝛽 = cos(𝛼 − 𝛽) − cos(𝛼 +
(𝑛−1)𝜋 𝑡
𝛽), and since cosine is a decreasing function,
𝜋 sin 𝑡+cos 𝑡 𝜋 sin 𝑡+cos 𝑡 the max. occurs when 𝛼= 𝛽. This implies that
Now, 𝑡 ∊ (0, 𝜋) ∫0 | 𝜋
| < ∫0 | 𝜋
| 𝐴𝑂1 ∥ 𝑃𝑂2 𝑎𝑛𝑑 𝐵𝑂2 ∥ 𝑃𝑂2 and the rest
2𝜋
follows as before.
sin 𝑡 + cos 𝑡
𝑡 ∊ (𝜋, 2𝜋) ∫ | |
0 𝑡
2𝜋
sin 𝑡 + cos 𝑡
<∫ | |
𝜋 2𝜋

𝑡
∊ {(𝑛
𝑛𝜋
sin 𝑡 + cos 𝑡 𝑛𝜋 sin 𝑡 + cos 𝑡
− 1)𝜋, 𝑛𝜋} ∫ | |∫ | |
(𝑛−1)𝜋 𝑡 (𝑛−1)𝜋 𝑛𝜋

1 𝜋
So, 𝐴𝑛 => 𝜋 [∫0 |sin 𝑡 + cos 𝑡| 𝑑𝑡 +
1 2𝜋
∫ |sin 𝑡
2 𝜋
+ cos 𝑡| 𝑑𝑡 … ] Q9. Find the area enclosed by the curve 𝒚 =
𝟐𝒙 𝒂𝒏𝒅 𝐦𝐚𝐱{│𝒙│, │𝒚│} = 𝟏.
∵|sin 𝑡 + cos 𝑡| is periodic with period 𝜋.

292
Solving Mathematical Problems

Sol. : Given max{│𝑥│, │𝑦│} = 1 …. (i) This ⟹I


curve represents four lines. 𝑠𝑖𝑛4 𝑧 √𝑓(cos 2𝑧−𝑚)
=∫𝑐𝑜𝑠4 𝑧 𝑑𝑚 (𝑢𝑠𝑖𝑛𝑔 𝑝𝑟𝑜𝑝. )
√𝑓(𝑚)+ √𝑓(cos 2𝑧−𝑚)

Case I: If │𝑥│ > │𝑦│, ⟹ │𝑥│ = 1 ∴ 𝑥 =≠ 1. ……. (iii)

Case II: If │𝑥│ < │𝑦│, ⟹ │𝑦│ = 1, ∴ 𝑦 =≠ 1. Adding (ii) and (iii), we get

𝑠𝑖𝑛4 𝑧 √𝑓(𝑚)+√𝑓(cos 2𝑧−𝑚)


Hence the curve (i) represents a sequence. 2I = ∫𝑐𝑜𝑠4 𝑧 𝑑𝑚 =
√𝑓(𝑚)+ √𝑓(cos 2𝑧−𝑚)
The shaded area represents the reqd. area
𝑠𝑖𝑛4 𝑧
enclosed by the given curves. ∫𝑐𝑜𝑠4 𝑧 𝑑𝑚 = −[𝑠𝑖𝑛4 𝑧 + 𝑐𝑜𝑠 4 𝑧],

∴ Reqd. area A= 3 × area of (OTMP + OPQS) 1 1 1


∴ 1 = − [𝑠𝑖𝑛4 𝑧 + 𝑐𝑜𝑠 4 𝑧] = − + 𝑠𝑖𝑛2 2𝑧
2 2 4
0
1
= 3 × (1 × 1) + ∫ 2𝑥 𝑑𝑥 = 3 + log 𝑒 2. Hence, differential equation (i) becomes
−1 2

𝑑𝑦 1 1 1
= √ − + 𝑠𝑖𝑛2 2𝑧
𝑑𝑥 2 2 4
1 1
= |sin 2𝑧|, | 𝑠𝑖𝑛2 (𝑥 + 𝑦)|
2 2
𝑑𝑦 1 𝜋
⟹ = − 𝑠𝑖𝑛2 (𝑥 + 𝑦) … . . (𝑖𝑣) [ < 𝑥 + 𝑦
𝑑𝑥 2 2
3𝜋
< ]
4
𝑑𝑦 𝑑𝑧
Given that 𝑥 + 𝑦 = 𝑧, ⟹ 1 + 𝑑𝑥 = 𝑑𝑥
𝒅𝒚
Q10. Sol. : 𝒅𝒙 =
𝑑𝑧 1
So, equation (iv) becomes 𝑑𝑥 = 1 − 2 sin 2𝑧,
𝟏 𝒔𝒊𝒏𝟒 𝒛 √𝒇(𝒎)
√ + ∫𝒄𝒐𝒔𝟒𝒛 𝒅𝒎,
𝟐 √𝒇(𝐜𝐨𝐬 𝟐𝒛−𝒎)+√𝒇(𝒎) 𝑑𝑧 𝑑𝑧
⟹ = 𝑑𝑥, ⟹ ∫
1 − sin 𝑧 cos 𝑧 1 − sin 𝑧 cos 𝑧
𝝅 𝟑𝝅
𝒘𝒉𝒆𝒓𝒆 𝒛 = 𝒙 + 𝒚 𝒂𝒏𝒅 < (𝒙 + 𝒚) < . = ∫ 𝑑𝑥
𝟐 𝟒

Sol.: Given that sec 2 𝑧𝑑𝑡 𝑑𝑟


⟹∫ 2
= ∫ 𝑑𝑥, ⟹ ∫
sec 𝑧 − tan 𝑧 1 + 𝑟2 − 𝑟
𝑑𝑦
𝑑𝑥 = ∫ 𝑑𝑥
𝑠𝑖𝑛4 𝑧
1 √𝑓(𝑚)
= √ +∫ 𝑑𝑚 … . . (𝑖) (Putting tan z = r, so thatsec 2 𝑧𝑑𝑡 = 𝑑𝑟 ),
2 𝑐𝑜𝑠 𝑧 √𝑓(cos 2𝑧 − 𝑚) + √𝑓(𝑚)
4

𝑑𝑟 2 2𝑟−1
⟹∫ 2 = ∫ 𝑑𝑥, ⟹ tan−1 ( ) =
𝑠𝑖𝑛4 𝑧 √𝑓(𝑚) 1 √3 √3 √3
Let I = ∫𝑐𝑜𝑠4 𝑧 𝑑𝑚 …… (ii) (𝑟− )+( )
2 2
√𝑓(cos 2𝑧−𝑚)+√𝑓(𝑚)
𝑐+𝑥

293
Solving Mathematical Problems

Show that M ≥ 1 and find all cases where


2𝑟 − 1 √3
⟹ = tan 𝑥 ( (𝑥 + 𝑐)), equality occurs.
√3 2
Sol.: a = 0, b = -3, c = 0, where M = 1, with the
2 tan(𝑥 + 𝑦) − 1 √3 maximum achieved at -1, -1/2, ½ , 1. On the
⟹ = tan { (𝑥 + 𝑐)}
√3 2 other hand, if M < 1 for some choice of a, b, c,
then
√3
∴ 2 tan(𝑥 + 𝑦) = √3 tan { (𝑥 + 𝑐)} + 1. (4𝑥 3 + 𝑎𝑥 2 + 𝑏𝑥 + 𝑐) − (4𝑥 3 + 3𝑥)
2
Must be positive at -1, negative -1/2, positive at
½, and negative at 1, which is impossible for a
quadratic function. Thus M ≥ 1, and the same
argument shows that equality only occurs for
PROBLEMS WITH SOLUTIONS FOR (a, b, c) = (0, -3, 0). (Note: this is a particular
I.S.I. MSQE. MSQMS, MTECH case of the minimum deviation property of
Chebyshev polynomials).
ENTRANCE TESTS

3. The real numbers 𝒂𝟏 , 𝒂𝟐 , … , 𝒂𝒏 (𝒏 ≥ 𝟑)


1. Prove that for all natural numbers n ≥ 3 from an arithmetic progression. There
there exist odd natural numbers 𝒙𝒏 , 𝒚𝒏 exists a permutation
such that 𝟕𝒙𝒏 𝟐 + 𝒚𝒏 𝟐 = 𝟐𝒏 . 𝒂𝒊𝟏 , 𝒂𝒊𝟐 , … , 𝒂𝒊𝒏 𝒐𝒇 𝒂𝟏 , 𝒂𝟐 , … , 𝒂𝒏 which is a
geometric progression. Find the numbers
Sol: For n = 3, we have 𝑥3 = 𝑦3 = 1. Now
𝒂𝟏 , 𝒂𝟐 , … 𝒂𝒏 if they are all different and the
suppose that for a given natural number n we
largest of them as equal to 1996.
have odd natural numbers 𝑥𝑛 , 𝑦𝑛 such that
7𝑥𝑛2 + 𝑦𝑛2 = 2𝑛 we shall exhibit a pair (X, Y) Sol.: Let 𝑎1 < 𝑎2 < −< 𝑎𝑛 = 1996 and let q
such that 7𝑋 2 + 𝑌 2 = 2𝑛 we shall exhibit a pair be the ratio of the geometric progression
(X, Y) such that 7𝑥𝑛 2 + 𝑦𝑛 2 = 2𝑛+1 . In fact, 𝑎𝑖1 … … … 𝑎𝑖𝑛 ; clearly q≠ 0 ± 1. By reversing
the geometric progression if needed, we may
𝑥𝑛 ± 𝑦𝑛 2 7𝑥𝑛 ± 𝑦𝑛 2
7( ) +( ) = 2(7𝑥𝑛 2 + 𝑦𝑛 2 ) assume |q| > 1, and so |𝑎𝑖1 |< |𝑎𝑖2 | < −|𝑎𝑖𝑛 |.
2 2
Note that either all of the terms are positive, or
= 2𝑛+1
they alternate in sign; in the latter case, the
(𝑥𝑛 + 𝑦𝑛 ) │𝑥𝑛 −𝑦𝑛 │ terms of either sign form a geometric
One of 2
𝑎𝑛𝑑 2
is odd (as their
progression by themselves.
sum is the larger of 𝑥𝑛 𝑎𝑛𝑑 𝑦𝑛 which is odd),
giving the desired pair. There cannot be three positive terms, or else
we would have a three term geometric
2. Let a, b, c be real numbers and let M be
progression a, b, c which is also an arithmetic
the maximum of the function 𝒚 = │𝟒𝒙𝟑 + progression, violating the AM –GM inequality.
𝒂𝒙𝟐 + 𝒃𝒙 + 𝒄│ in the interval │-1, 1│.

294
Solving Mathematical Problems

Similarly, there cannot be three negative terms, 5. If 𝛼, 𝛽, 𝛾 are the roots of 𝒙𝟑 − 𝒙 − 𝟏 = 𝟎,


so there are at most two terms of each sign and 𝟏−𝜶 𝟏−𝜷 𝟏−𝜸
compute 𝟏+𝜶 + 𝟏+𝜷 + 𝟏+𝜸.
n ≤4.
Sol.: The given quantity equals
If n = 4, we have 𝑎1 < 𝑎2 < 0 < 𝑎3 <
𝑎4 𝑎𝑛𝑑 2𝑎2 = 𝑎2 + 𝑎3 , 2𝑎3 = 𝑎2 + 𝑎4 . In this 1 1 1
2( + + ) − 3.
case, q < -1 and the geometric progression is 𝛼+1 𝛽+1 𝛾+1
either 𝑎3 , 𝑎2 , 𝑎4 , 𝑎1 𝑜𝑟 𝑎2 , 𝑎3 , 𝑎1 , 𝑎4 . Suppose
the former occurs (the argument in similar in Since 𝑃(𝑥) = 𝑥 3 − 𝑥 − 1 has roots 𝛼, 𝛽, 𝛾, the
the latter case): then polynomial 𝑃(𝑥 − 1) = 𝑥 3 − 3𝑥 2 + 2𝑥 − 1
has roots 𝛼+1, 𝛽+1, 𝛾+1.
2𝑎3 𝑞 = 𝑎3 𝑞 3 + 𝑎3 𝑎𝑛𝑑 2𝑎3 + 𝑎3 𝑞 3 + 𝑎3 𝑞 2,
By a standard formula, the sum of the
giving q =1, a contradiction.
reciprocals of the roots of 𝑥 3 + 𝑐2 𝑥 2 + 𝑐1 𝑥 +
We deduce n = 3 and consider two possibilities. 𝑐0 𝑖𝑠 − 𝑐1 /𝑐0, so the given expression equals
If 𝑎1 < 𝑎2 < 0 < 𝑎3 = 1996, 𝑡ℎ𝑒𝑛 2𝑎2 = 2(2)-3= 1.
𝑎2 𝑞 2 + 𝑎2 𝑞, so 𝑞 2 + 𝑞 − 2 = 0 𝑎𝑛𝑑 𝑞 = −2,
6. Find all real solution to the following
yielding (𝑎1 , 𝑎2 , 𝑎3 ) = (−3992, −998, 1996). If
system of equations:
𝑎1 < 0 < 𝑎2 < 𝑎3 = 1996, then
𝟒𝒙𝟐
=𝒚
2𝑎2 = 𝑎2 𝑞 + 𝑎2 𝑞2 , so again q = -2, yielding 𝟏 + 𝟒𝒙𝟐
𝟒𝒚𝟐
(𝑎1 , 𝑎2 , 𝑎3 ) = (−998, 499, 1996). =𝒛
𝟏 + 𝟒𝒚𝟐
𝟒𝒛𝟐
= 𝒙.
𝟏 + 𝟒𝒛𝟐
4. Find all prime numbers p, q for which pq 4𝑥 2
Sol.: Define 𝑓(𝑥) = (1+4𝑥2 ) ; the range of f is [0,
divides (𝟓𝒑 − 𝟐𝒑 )(𝟓𝒒 − 𝟐𝒒 ).
1), so x, y, z must lie in that interval. If one of x,
Sol.: If p│5𝑝 − 2𝑝 , 𝑡ℎ𝑒𝑛𝑝│5 -2 by Fermat’s y, z is zero, then all three are, so assume they
theorem, 𝑓(𝑥) 4𝑥
are nonzero. Then 𝑥
= (1+4𝑥2 ) is at least 1
So p = 3, suppose p, q ≠3; then p│5𝑞 − 2𝑞 but the AM –GM inequality, with equality for x
and q│5𝑝 − 2𝑝 . Without lose of generality = ½ . Therefore x ≤y ≤ z ≤ x, and so equality
assume p >q, so that (p, q -1) = 1. Then if a is holds everywhere, implying x = y = z = ½ . Thus
an integer such that 2a ≡5 (mod q), then the the solutions are (x, y, z) = (0, 0, 0), (½ , ½ , ½ ).
order of a mod q divides p as well as q -1, a
contradiction.

Hence one of p, q is equal to 3. If q ≠ 3, then 7. Let f(n) be the number of permutations


q│53 − 23 = 9.13. so q = 13, and similarly p 𝒂𝟏 , … , 𝒂𝒏 of the integers 1, …, n such that
∊(3, 13). (i) 𝒂𝟏 = 𝟏;
(ii) │𝒂𝒊 − 𝒂𝒊+𝟏 │ ≤ 𝟐, 𝒊 = 𝟏, … , 𝒏 − 𝟏.
Thus the solutions are (p, q) = (3, 3), (3, 13),
(13, 3). Determine whether f(1996) is divisible by 3.

295
Solving Mathematical Problems

Sol.: Let g(n) be the number of permutations of Cancelling cos 𝛽, we have cos 3𝛽 − cos 7𝛽 =
the desired form with 𝑎𝑛 = 𝑛. Then either cos 2𝛽 − cos 6𝛽, which implies
𝑎𝑛−1 = 𝑛 − 1 𝑜𝑟 𝑎𝑛−1 = 𝑛 − 2; in the latter
case we must have 𝑎𝑛−2 = 𝑛 − 1 𝑎𝑛𝑑 𝑎𝑛−3 = sin 2𝛽 sin 5𝛽 = sin 2𝛽 sin 4𝛽.
𝑛 − 3. Hence g(n) = g(n-1) + g(n -3) for n ≥4. In Now sin 5𝛽 = sin 4𝛽 , 𝑠𝑜 9𝛽 = 𝜋 𝑎𝑛𝑑 𝛽 = 9 .
𝜋

particular, the values of g(n) modulo 3 are g(1) =


1, 1, 1, 2, 0, 1, 0, 0….. repeating with period 8.

Now let h(n) = f(n) – g(n); h(n) counts 9. Let 𝒓𝟏 , 𝒓𝟐 , … , 𝒓𝒎 be a given set of positive
permutations of the desired from where n rational numbers whose sum is 1. Define
occurs in the middle, sandwiched between n-1 the function f by 𝒇(𝒏) = 𝒏 −
and n -2. Removing n leaves an acceptable ∑𝒎𝒌=𝟏 ⎿⌊𝒓𝒌 𝒏⌋⏌ for each positive integer n.
permutation, and any acceptable permutation Determine the minimum and
on n -1 symbols can be so produced except maximum values of f(n).
those ending in n -4, n -2, n -3, n -1. Hence h(n)
= h(n -1)+ g(n -1) –g(n -4) = h(n -1)+ g(n -2); one Sol.: Of course ⎿⌊𝑟𝑘 𝑛⌋⏌ ≤ 𝑟𝑘 𝑛, 𝑠𝑜 𝑓(𝑛) ≥ 0,
checks that h(n) modulo 3 repeats with period with equality for n = 0, so 0 is the minimum
24. value. On the other hand, we have 𝑟𝑘 𝑛 −
⎿⌊𝑟𝑘 𝑛⌋⏌ < 1, 𝑠𝑜 𝑓(𝑛) ≤ 𝑚 − 1.
Since 1996 ≡ 4 (mod 24), we have f(1996) ≡
f(4) = 4(mod 3), so f(1996) is not divisible by 3. Here equality holds for n = t- 1 if t is the least
common denominator of the 𝑟𝑘 .

8. Let ∆ABC be an isosceles triangles with AB


= AC. Suppose that the angle bisector of 10. Find the smallest positive integer K such
∠B meets AC at D and that BC = BD + AD. that every K-element subset of (1, 2, …, 50)
Determine ∠A. contains two distinct elements a, b such
that a+b divides ab.
(𝜋−𝛼)
Sol.: Let 𝛼 =∠A, 𝛽= 4
and assume AB = 1.
Sol.: The minimal value is k = 39. Suppose a, b∊S
Then by the Law of Sines, are such that a + b divides ab. Let c = gcd (a, b)
and put a = c𝑎1 , 𝑏 = 𝑐𝑏1, so that 𝑎1 𝑎𝑛𝑑 𝑏1are
sin 𝛼 sin 𝛼 sin 𝛽 relatively prime. Then c(𝑎1 +
𝐵𝐶 = , 𝐵𝐷 = , 𝐴𝐷 = .
sin 2𝛽 sin 3𝛽 sin 3𝛽 𝑏1 )𝑑𝑖𝑣𝑖𝑑𝑒𝑠 𝑐 2 𝑎1 𝑏1 , 𝑠𝑜 𝑎1 + 𝑏1 𝑑𝑖𝑣𝑖𝑑𝑒𝑠 𝑐𝑎1 𝑏1 .
Thus we are seeking a solution to the equation Since 𝑎1 𝑎𝑛𝑑 𝑏1 have no common factor,
sin(𝜋 − 4𝛽) sin 3𝛽 = (sin(𝜋 − 4𝛽) + neither do 𝑎1 𝑎𝑛𝑑 𝑎1 + 𝑏1 , 𝑜𝑟 𝑏1 𝑎𝑛𝑑 𝑎1 + 𝑏1 .In
sin 𝛽) sin 2𝛽.
short, 𝑎1 + 𝑏1 divides c.
Using the sum-to-product formula, we rewrite
Since S ⊆ {1, … , 50}, we have a +b ≤99, so
this as
c(𝑎1 + 𝑏1 ) ≤ 99, which implies 𝑎1 + 𝑏1 ≤ 9, on
cos 𝛽 − cos 7𝛽 = cos 2𝛽 − cos 6𝛽 + cos 𝛽 the other hand, of course 𝑎1 + 𝑏1 ≥ 3. An
− cos 3𝛽.

296
Solving Mathematical Problems

exhaustive search produces 23 pairs, a, b 4 8


𝑚( ) = 𝑟( )
satisfying the conditions. 2 2
14𝑟
𝑎1 + 𝑏1 = 3 (6, 3), (12, 6), (18, 9), (24, 12), (30, And so m = 3
; in particular, m ≥14. However,
15), (36, 18), (42, 21), (48, 24) m = 14 is indeed possible, using the
arrangement
𝑎1 + 𝑏1 = 4 (12, 4), (24, 8), (36, 12), (48, 16)
{1, 2, 3, 4} {5, 6, 7, 8} {1, 2, 5, 6} {3, 4, 7, 8}
𝑎1 + 𝑏1 = 5 (20, 5), (40, 10), (15, 10), (30, 20),
(45, 30) {3, 4, 5, 6} {1, 3, 5, 7} {2, 4, 6, 8} {1, 3, 6, 8}

𝑎1 + 𝑏1 = 6 (30, 6) {2, 4, 5, 7} {1, 4, 5, 8} {2, 3, 6, 7} {1, 4, 6, 7}

𝑎1 + 𝑏1 = 7 (42, 7), (35, 14), (28, 21) {1, 2, 7, 8} {2, 3, 5, 8}

𝑎1 + 𝑏1 = 8 (40, 24) 12. In triangle ABC, ∠𝑪 = 𝟗𝟎°, ∠𝑨 =


𝟑𝟎° 𝒂𝒏𝒅 𝑩𝑪 = 𝟏.Find the minimum of the
𝑎1 + 𝑏1 = 9 (45, 36)
length of the longest side of a triangle
Let M = {6, 12, 15, 18, 20, 21, 24, 35, 40, 42, 45, inscribed in ABC (that is, one such that
48} and T = {1, …, 50} –M. Since each pair listed each side of ABC contains a different
above contains an element of M, T does not vertex of the triangle).
have the desire property. Hence we must take k
Sol.: We first find the minimum side length of
≥│T│+1 = 39. On the other hand, from the
an equilateral triangle inscribed in ABC. Let D be
23 pairs mentioned above we can select 12
a point on BC and put x = BD. Then take points
pairs which are mutually disjoint:
E, F on CA, AB, respectively, such that CE =
(6, 3), (12, 4), (20, 5), (42, 7), (24, 8), (18, 9), √3𝑥 𝑥
2
𝑎𝑛𝑑 𝐵𝐹 = 1 − 2. A calculation using the
(40, 10), (35, 14), (30, 15), (48, 16), (28, 21),
Law of Cosines shows that
(45, 36).
7
Any 39-element subset must contain both 𝐷𝐹 2 = 𝐷𝐸 2 = 𝐸𝐹 2 = 𝑥 2 − 2𝑥 + 1
4
elements of one of these pairs. We conclude 7 4 2 3
the desired minimal number is k = 39. = (𝑥 − ) +
4 7 7

Hence the triangle DEF is equilateral, and its


3
11. Eight singers participate in an art festival minimum possible side length is √ .
7
where m songs are performed. Each song is
performed by 4 singers, and each pair of We know argue that the minimum possible
singers performs together in the same longest side must occur for some equilateral
number of songs. Find the smallest m for triangle. Starting with an arbitrary triangle, first
which this is possible. suppose it is not isosceles. Then we can side
one of the endpoints of the longest side so as to
Sol.: Let r be the number of songs each pair of
decrease its length; we do so until there are two
singers performs together, so that
longest sides, say DE and EF. We now fix D,

297
Solving Mathematical Problems

move E so as to decrease DE and move F at the 14. For which integers k does there exist a
same time so as to decrease EF; we do so until function f : N →Z such that
all three sides become equal in length. (It is fine (a) f(1995) = 1996, and
if the vertices move onto the extensions of the (b) f(xy) = f(x) + f(y) + kf(gcd(x, y))for
sides, since the bound above applies in that all x, y ∊ N?
case as well.)
Sol.: Such f exists for k = 0 and k = -1. First
3 take x = y in (b) to get 𝑓(𝑥 2 ) = (𝑘 + 2)𝑓(𝑥).
Hence the minimum is indeed √7, as desired.
Applying this twice, we get

13. Prove that if a sequence {𝑮(𝒏)}∞ 𝒏=𝟎 of 𝑓(𝑥 4 ) = (𝑘 + 2)𝑓(𝑥 2 ) = (𝑘 + 2)2 𝑓(𝑥).
integers satisfies
G(0) = 0, On the other hand,
G(n) = 𝒏 − 𝑮{𝑮(𝒏)} (n= 1, 2, 3, 𝑓(𝑥 4 ) = 𝑓(𝑥) + 𝑓(𝑥 3 ) + 𝑘𝑓(𝑥)
….) = (𝑘 + 1)𝑓(𝑥) + 𝑓(𝑥 3 )
then
(a) 𝑮(𝒌) ≥ 𝑮(𝒌 − 𝟏) for any positive = (𝑘 + 1)𝑓(𝑥) + 𝑓(𝑥) + 𝑓(𝑥 2 ) + 𝑘𝑓(𝑥)
integer k;
= (2𝑘 + 2)𝑓(𝑥) + 𝑓(𝑥 2 ) = (3𝑘 + 4)𝑓(𝑥).
(b) No integer k exists such that G(k -
1) = G(k) = G(k +1). Setting x = 1995 so that f(x) ≠ 0, we deduce
(𝑘 + 2)2 = 3𝑘 + 4, which has roots k = 0, -1.
Sol.:
For k = 0, an example is given by
(a) We show by induction that 𝐺(𝑛) −
𝑓(𝑝1 𝑒1 … 𝑝𝑛 𝑒𝑛 ) = 𝑒1 𝑔(𝑝1 ) + ⋯ + 𝑒𝑛 𝑔(𝑝𝑛 ).
𝐺(𝑛 − 1) ∊ {0, 1} for all n. If this holds
up to n, then Where g(5) = 1996 and g(p) = 0 for all
𝐺(𝑛 + 1) − 𝐺(𝑛) primes p ≠5 for k = 1, as example is given by
= 1 + 𝐺(𝐺(𝑛 − 1))
𝑓(𝑝1 𝑒1 … 𝑝𝑛 𝑒𝑛 ) = 𝑔(𝑝1 ) + ⋯ + 𝑔(𝑝𝑛 )
− 𝐺(𝐺(𝑛)).
𝐼𝑓 𝐺(𝑛 − 1) = 𝐺(𝑛), 𝑡ℎ𝑒𝑛 𝐺(𝑛 + 1) −
𝐺(𝑛) = 1; otherwise, 𝐺(𝑛 −
1)𝑎𝑛𝑑 𝐺(𝑛) are consecutive integers 15. A triangle ABC and points K, L, M on the
sides AB, BC, CA respectively, are given
not greater than n, so 𝐺(𝐺(𝑛)) −
such that
𝐺(𝐺(𝑛 − 1)) ∊ {0, 1}, again completing 𝑨𝑲 𝑩𝑳 𝑪𝑴 𝟏
the induction. = = =
𝑨𝑩 𝑩𝑪 𝑪𝑨 𝟑
(b) Suppose that G(k -1)= G(k) = G(k+1)+A Show that if the circumcircles of the
for some k, A. Then triangles of the triangles AKM, BLK,
A= G(k+ 1)= k +1 –G(g(k))= k+1-G(A) CML are congruent, then so are the in
And similarly A = k –G(A) (replacing k +1 circles of these triangles.
with k above), a contradiction.
Sol.: We will show that ABC is equilateral, so
Note: It can be shown that G(n) = ⌊𝑛𝜔⌋
that AKM, BLK, CML are congruent and hence
(√5−1)
for 𝜔 = . have the same in radius.
2

298
Solving Mathematical Problems

Let R be the common circumradius; then Now note that

KL = 2R sin A, LM = 2R sin B, MK = 2R sin C, 2(𝑠𝑖𝑛2 𝜃 + sin 𝜃 cos 𝜃) = 1 − cos 2𝜃 + sin 𝜃

So the triangles KLM and ABC are similar. 𝜋


= 1 + √2 sin (2𝜃 − ).
Now we compare areas: 4
𝜋 𝜋
2 Thus we either have B = C or 2𝐵 − 4 + 2𝐶 − 4
[AKM] = [BLK] = [CLM] = 9[ABC],

1 3𝜋
So, [KLM] = 3[ABC] and the coefficient of = 𝜋, 𝑜𝑟 𝐵 + 𝐶 = .
4
1
similarity between KLM and ABC must be √3. In particular, two of the angles must be equal,
By the law of cosines applied to ABC and say A and B, and we either have A = B = C, so
AKM. the triangle is equilateral, or 𝐵 + (𝜋 − 2𝐵) =
3𝜋 𝜋
4
, in which case A = B = 4 and the triangle is
𝑎2 = 𝑏 2 + 𝑐 2 − 2𝑏𝑐 cos 𝐴
isosceles right.
2
1 2 2𝑝 𝑐 2 2𝑏 𝑐 17. Let a, b be positive integers with a odd.
𝑎 = ( ) + ( ) −2 cos 𝐴.
3 3 3 3 3 Define the sequence {𝒖𝒏 } as follows: 𝒖𝟎 =
From these we deduce 𝑎2 = 2𝑏 2 − 𝑐 2 , and 𝒃 & n ∊ ℕ.
𝟏
similarly 𝑏 2 = 2𝑐 2 − 𝑎2 , 𝑎2 = 2𝑎2 − 𝑏 2 . 𝒖 𝒊𝒇 𝒖𝒏 𝒊𝒔 𝒆𝒗𝒆𝒏
𝒖𝒏+𝟏 = { 𝟐 𝒏
Combining these gives 𝑎2 = 𝑏 2 = 𝑐 2 , so ABC
𝒖𝒏 + 𝒂 𝒐𝒕𝒉𝒆𝒓𝒘𝒊𝒔𝒆
is equilateral, as desired.
(a) Show that 𝒖𝒏 ≤ 𝒂 for some n ∊ℕ.
(b) Show that the sequence {𝒖𝒏 } is
periodic from some point onwards.
16. Let ABC be a triangle and construct
squares ABED, BCGF, ACHI externally on Sol:
the sides of ABC. Show that the points D, E,
(a) Suppose 𝑢𝑛 > 𝑎, if 𝑢𝑛 is even, 𝑢𝑛+1 =
F, G, H, I are concyclic if and only if ABC is 𝑢𝑛 (𝑢𝑛 +𝑎)
< 𝑢𝑛 ; if 𝑢𝑛 is odd, 𝑢𝑛+2 = <
equilateral or isosceles right. 2 2
𝑢𝑛 . Hence for each term greater than
Sol.: Suppose D, E, F, G, H, I are concyclic; the a, there is a smaller subsequent term.
perpendicular bisectors of DE, FG, HI coincide These form a decreasing subsequence
with those of AB, BC, CA respectively, so the which must eventually terminate,
center of the circle must be the circumcenter which only occurs once 𝑢𝑛 ≤ 𝑎.
O of ABC. By equating the distances OD and (b) If 𝑢𝑚 ≤ 𝑎, then for all n ≥ m, either
OF, we find 𝑢𝑛 ≤ 𝑎, 𝑜𝑟, 𝑢𝑛 is even and 𝑢𝑛 ≤ 2𝑎, by
induction on n. In particular, 𝑢𝑛 ≤
(cos 𝐵 + 2 sin 𝐵)2 + 𝑠𝑖𝑛2 𝐵
2𝑎 𝑓𝑜𝑟 𝑎𝑙𝑙 𝑚 ≥ 𝑛, and so some value
= (cos 𝐶 + 2 sin 𝐶)2 = 𝑠𝑖𝑛2 𝐶
of 𝑢𝑛 eventually repeats, leading to a
Expanding this end cancelling like terms, we periodic sequence.
determine 𝑠𝑖𝑛2 𝐵 + sin 𝐵 cos 𝐵 = 𝑠𝑖𝑛2 𝐶 +
sin 𝐶 cos 𝐶.

299
Solving Mathematical Problems

18. (a) Find the minimum value of 𝒙𝒙 for x a Sol.: It is necessary and sufficient that gcd(x,
positive real number. y) = 2𝑥 for some nonnegative integer s. We
show necessity by nothing that gcd(p, q) =
(b) If x and y are positive real numbers, gcd(p, q –p), so an odd common divisor can
show that 𝒙𝒙 + 𝒚𝒙 > 1. never be introduced, and nothing that initially
Sol.: gcd(1, 1)= 1.

As for sufficiency, suppose gcd(x, y) = 2𝑥 . Of


(a) Since 𝑥 𝑥 = 𝑒 𝑥𝑙𝑜𝑔 𝑥 𝑎𝑛𝑑 𝑒 𝑥 is an
those pairs (p, q) from which (x, y) can be
increasing function of x, it suffices to
reached, choose one to minimize p +q.
determine the minimum of x log x.
Neither p and q can be even, else one of
This is easily done by setting its
𝑝 𝑞
derivative 1+ log x to zero, yielding (2 , 𝑞) 𝑜𝑟 (𝑝, 2) is an admissible pair. If p > q,
1 1 (𝑝+𝑞)
𝑥 = . The second derivative is then (p, q) is reachable from ( , 𝑞), a
𝑒 𝑥 2
positive for x > 0, so the function is contradiction; similarly p < q is impossible.
everywhere convex, and the unique Hence p = q, but gcd(p, q) is a power of 2 and
extremum is needed a global neither p nor q is even. We conclude p =q =
minimum. Hence 𝑥 𝑥 has minimum 1, and so (x, y) is indeed reachable.
value 𝑒 −1/𝑒 .
(b) If x ≥ 1, then 𝑥 𝑦 ≥ 1 for y > 0, so we
may assume 0< x, y<1. Without loss
20. Prove that every integer k > 1 has a
of generality, assume x ≤y; now note
multiple less than 𝒌𝟒 whose decimal
that the function 𝑓(𝑥) = 𝑥 𝑥 + 𝑦 𝑥 has
expansion has at most four distinct digits.
derivative 𝑓 ′ (𝑥) = 𝑥 𝑥 log 𝑥 + 𝑦 𝑥−1 .
Since 𝑦 𝑥 ≥ 𝑥 𝑥 ≥ 𝑥 𝑦 𝑓𝑜𝑟 𝑥 ≤ Sol.: Let n be the integer such that 2𝑛−1 ≤ 𝑘 ≤
1
𝑦 𝑎𝑛𝑑 𝑥 ≥ − log 𝑥, we see that 2𝑛 . For n ≤6 the result is immediate, so
𝑓′ (𝑥)
> 0 𝑓𝑜𝑟 0 ≤ 𝑥 ≤ 𝑦 and so the assume n > 6.
minimum of f occurs with x = 0, in
Let S be the set of nonnegative integers less
which case f(x) = 1; since x > 0, we
than 10𝑛 whose decimal digits are all 0s or 1s.
have strict inequality.
Since │S│ = 2𝑛 > 𝑘, we can find two
elements a < b of S which are congruent
modulo, k, and b − a only has the digits 8, 9, 0,
19. Starting at (1, 1), a stone is moved in the 1 in its decimal representation. On the other
coordinate plane according to the hand,
following rules:
(i) From any point (a, b), the stone 𝑏 − 𝑎 ≤ 𝑏 ≤ 1 + 10 + ⋯ + 10𝑛−1 < 10𝑛
can move to (2a, b) or (a, 2b). < 16𝑛−1 ≤ 𝑘 4 ,
(ii) From any point (a, b), the stone
Hence b – a is the desired multiple.
can move to (a –b, b) if a > b, or to
(a, b –a) if a < b.
For which positive integers x, y can
the stone be moved to (x, y)?

300
Solving Mathematical Problems

21. Given 81 natural numbers whose prime ∑ 𝑥 4 𝑦𝑧 − 2𝑥 3 𝑦 2 𝑧 + 𝑥 2 𝑦 2 𝑧 2 ≥ 0


divisors belong to the set {2, 3, 5}, prove 𝑠𝑦𝑚
there exist 4 numbers whose product is
the fourth power of an integer. On the other hand,

Sol.: It suffices to take 25 such numbers. To ∑(𝑥 5 𝑦 − 𝑥 4 𝑦 2 ) + 3(𝑥 5 − 𝑥 3 𝑦 3 ) ≥ 0


each number, associate the triple (𝑥2 , 𝑥3 , 𝑥5 ) 𝑠𝑦𝑚
recording the parity of the exponents of 2, 3
and 5 in its prime factorization. Two numbers By two applications of AM-GM; combining the
have the same triple if and only if their last two displayed inequalities gives the
product is a perfect square. As long as there desired result.
are 9 numbers left, we can select two whose
product is a square, in so doing, we obtain 9
such pairs. Repeating the process with the 23. Prove that for every pair m, k of natural
square roots of the products of the pairs, we numbers, m has a unique representation in
obtain four numbers whose product is a the from
fourth power. 𝒂𝒌 𝒂𝒌−𝟏 𝒂𝒕
𝒎 = ( )+( ) + ⋯+ ( )
𝒌 𝒌−𝟏 𝒕
22. Prove the following inequality for positive
real numbers x, y, z: where 𝒂𝒌 > 𝒂𝒌−𝟏 > ⋯ > 𝒂𝒕 ≥ 𝒕 ≥ 𝟏.
𝟏 𝟏
(𝒙𝒚 + 𝒚𝒛 + 𝒛𝒙) ( 𝟐
+ Sol.: We first show uniqueness. Suppose m is
(𝒙 + 𝒚) (𝒚 + 𝒛)𝟐
𝟏 𝟗 represented by two sequences 𝑎𝑘 , … , 𝑎𝑡 and
+ 𝟐
)≥ . 𝑏𝑘 , … . , 𝑏𝑡 . Find the first position in which they
(𝒛 + 𝒙) 𝟒
differ, without loss of generally, assume this
Sol.: After clearing denominators, the given position is k and that 𝑎𝑘 > 𝑏𝑘 . Then
inequality becomes
𝑏 𝑏 𝑏 −𝑘+1
𝑚 ≤ ( 𝑘 ) + ( 𝑘−1 ) + ⋯ + ( 𝑘 )<
5 4 2 3 3
∑ 4𝑥 𝑦 − 𝑥 𝑦 − 3𝑥 𝑦 + 𝑥 𝑦𝑧 − 2𝑥 𝑦 𝑧4 3 2 𝑘 𝑘−1 1
𝑏 +1
𝑠𝑦𝑚 ( 𝑘 ) ≤ 𝑚, a contradiction.
1
+ 𝑥 2 𝑦 2𝑧 2 ≥ 0
To show existence, apply the greedy
Where the symmetric sum runs over all six algorithm: find the largest 𝑎𝑘 such that
permutations of x, y, z. (In particular, this 𝑎
( 𝑘 ) ≤ 𝑚, and apply the same algorithm with
means the coefficient of 𝑥 3 𝑦 3 in the final 𝑚
expression is -6, and that 𝑥 2 𝑦 2 𝑧 2 is 6.) m and k replaced by 𝑚 − (𝑎𝑘𝑘 ) 𝑎𝑛𝑑 𝑘 − 1.

Recall the inequality: We need only make sure that the sequence
obtained is indeed decreasing, but this
𝑥(𝑥 − 𝑦)(𝑥 − 𝑧) + 𝑦(𝑦 − 𝑧)(𝑦 − 𝑥) follows because by assumption, 𝑚 <
+ 𝑧(𝑧 − 𝑥)(𝑧 − 𝑦) ≥ 0 (𝑎𝑘𝑚+1), 𝑎𝑛𝑑 𝑠𝑜 𝑚 − (𝑎𝑘𝑘 ) < (𝑘−1
𝑎𝑘
).
Multiplying by 2xyz and collecting symmetric
terms, we get

301
Solving Mathematical Problems

24. The top and bottom edges of a chessboard which also divides n! and so does not divide
are identified together, as are the left and n! +1. Hence f(n) = 1. If n +1 is prime, the
right edges, yielding a torus. Find the same argument shows that f(n) is a power of
maximum number of knights which can be n +1, and in fact n +1 │n! +1 by Wilson’s
placed so that no two attack each other. theorem. However, (𝑛 + 1)2 does not divide
(n +1)!, and thus f(n) = n +1.
Sol.: The maximum is 32 knights; if the
chessboard is alternately colored black and
white in the usual fashion, an optimal
arrangement puts a knight on each black 27. For each positive integer n, let S(n) be the
square. To see that this cannot be improved, sum of the digits in the decimal expansion
suppose that k knights are placed. Each of n. Prove that for all n,
knight attack 8 squares, but no unoccupied 𝑺(𝟐𝒏) ≤ 𝟐𝑺(𝒏) ≤ 𝟏𝟎𝑺(𝟐𝒏) & show
square can be attacked by more than 8 that there exists n such that S(n) =
knights. Therefore 8k ≤ 8(64 –k), where k ≤ 1996S(3n).
32. Solution: It is clear that S(a +b) ≤ S(a) +
S(b), with equality if and only if there are no
carries in the addition of a and b. Therefore
25. Let P(x) be a polynomial with rational S(2n) ≤ 2S(n). Similarly S(2n) ≤ 5S(10n) =
coefficients such that 𝑷−𝟏 (𝑸) ⊆ 𝑸. Show 5S(n). An example with S(n) = 1996S(3n) is
that P is linear. 133 … 35 (with 5968 threes).

Sol: By a suitable variable substitution and 28. Let 𝑭𝒏 denote the Fibonacci sequence, so
constant factor, we may assume P(x) is monic that 𝑭𝟎 = 𝑭𝟏 = 𝟏 and 𝑭𝒏+𝟐 = 𝑭𝒏+𝟏 + 𝑭𝒏
and has integer coefficients; let P(0)= 𝑐0 . If p for n ≥0. Prove that
is a sufficiently large prime, the equation (i) The statement “𝑭𝒏+𝒌 − 𝑭𝒏 is
P(x)= p +𝑐0 has a single real root, which by divisible by 10 for all positive
assumption is rational and which we may also integers n” is true if k = 60 and
assume is positive (since P has positive false or any positive integer k
leading coefficient). However, by the rational < 60;
root theorem, the only rational roots of P(x) – (ii) The statement “𝑭𝒏+𝒕 − 𝑭𝒏 is
p - 𝑐0 can be ±1 𝑎𝑛𝑑 ± 𝑝. Since the root must divisible by 100 for all positive
be positive and cannot be 1 for large p, we integers n” is true if t = 300
have P(p) –p -𝑐0 = 0 for infinitely many p, so and false or any positive
P(x) = x +𝑐0 is linear. integer t<300.

Solution: A direct computation shows that the


Fibonacci sequence has period 3 modulo 2
26. For each positive integer n, find the and 20 modulo 5(compute terms until the
greatest common divisor of n! +1 and initial terms 0, 1 repeat, at which time the
(n+1)!. entire sequence repeats), yielding (a). As for
Sol: If n + 1 is composite, then each prime (b), one computes that the period mod 4 is 6.
divisor of (n+ 1)! is a prime less than n, The period mod 25 turns out to be 100, which
is awfully many terms to compute by hand,

302
Solving Mathematical Problems

but knowing that the period must be a 𝑐 2 − 𝑏 2 = (𝑐 + 𝑏)(𝑐 − 𝑏) = 𝑎2 − 3.


multiple of 20 helps, and verifying the
recurrence 𝐹𝑛+8 = 𝑡𝐹𝑛+4 + 𝐹𝑛 , where t is an
integer congruent to 2 modulo 5, shows that 32. Let A and B be opposite vertices of a cube
the period divides 100, finally, an explicit of edge length 1. Find the radius of the
computation shows that the period is not 20.
sphere with center interior to the cube,
tangent to the three faces meeting at A and
tangent to the three edges meeting at B.
29. Prove that for all positive integers n,
𝒏 Solution: Introduce coordinates so that A =
𝟐𝟏/𝟐 . 𝟒𝟏/𝟒 … . (𝟐𝒏 )𝟏/𝟐 < 4.
(0, 0, 0), B = (1, 1, 1) and the edges are
Solution: It is sufficient to show parallel to the coordinate axes. If r is the
radius of the sphere, then (r, r, r) is its center,
𝑥
𝑛 and (r, 1, 1) is the point of tangency of one of
∑ = 2;
2𝑛 the edges at B. Therefore 𝑟 2 = 2(1 −
𝑛=1
𝑟)2 , 𝑔𝑖𝑣𝑖𝑛𝑔 𝑟 2 − 4𝑟 + 2 = 0 and so r = 2 − √2
𝑥 𝑥 𝑥 𝑥
𝑛 1 1 (the other root puts the center outside of the
∑ 𝑛 = ∑ ∑ 𝑘 = ∑ 𝑛−1 = 2. cube).
2 2 2
𝑛=1 𝑛=1 𝑛=1 𝑛=1

33. Given an alphabet with three letters a, b, c


30. Let p be a prime number and a, n positive
find the number of words of n letters
integers.
which contain an even number of a’s.
Prove that if 𝟐𝒑 + 𝟑𝒑 = 𝒂𝒏 , then n =
1. Solution: If there are 2k occurrences of a,
𝑛
these can occur in (2𝑘 ) places, and the
Solution: If p = 2, we have 22 + 32 = 13 and
n = 1. If p > 2, then p is odd, so 5 divides 2𝑝 + remaining positions can be filled in 2𝑛−2𝑘
3𝑝 and so 5 divides a. Now if n > 1, then 25 ways. So the answer is
2𝑝 +3𝑝
divides 𝑎𝑛 and 5 divides = 2𝑝−1 − 𝑛
2+3 ∑ ( ) 2𝑛−2𝑘 .
𝑝−2
2 .3 + ⋯+ 3 𝑝−1
≡ 𝑝2 𝑝−1
(mod 5), a 2𝑘
𝑘
contradiction if p ≠ 5. Finally, if p = 5, then
25 + 35 = 753 is not a perfect power, so n = 1 To compute this, note that
again. 𝑛
(1 + 𝑥)𝑛 + (1 − 𝑥)𝑛 = 2 ∑ ( ) 𝑥 2𝑘 .
2𝑘
𝑘

31. Prove that the equation 𝒂𝟐 + 𝒃𝟐 = 𝒄𝟐 + 𝟑 So the answer is


has infinitely many integer solutions (a, b,
1 𝑛 1 𝑛 1 𝑛 1
c). 2 [(1 + ) + (1 − ) ] = (3𝑛 + 1).
2 2 2 2
Sol.: let a be any odd number, let b =
(𝑎 2 −5) (𝑎 2 −1)
𝑎𝑛𝑑 𝑐 = . Then
2 2

303
Solving Mathematical Problems

34. What is the minimum number of squares Sol.: We may assume 𝜃 = ∠A. The case where
that one needs to draw on a white sheet in ABC belongs to the triangulation is easy, so
order to obtain a complete grid with n assume this is not the case. If BC is an edge of
squares on a side? the triangulation, one of the two triangles
bounded by BC has common interior points
Solution: It suffices to draw 2n -1 squares: with ABC, and this triangle satisfies the
in terms of coordinates, we draw a square desired condition. Otherwise, there is a
with opposite corners (0, 0) and (i, i) for triangle BEF in the triangulation whose
1 ≤ i ≤ n and a square with opposite interior intersects BC. Since EF crosses BC at
corners (i, i) and (n, n) for 1 ≤ i≤ n -1. an interior point, ∠BEF < ∠BAF < ∠BAC, so
To show this many squares are necessary, triangle BEF satisfies the desired condition.
note that the segments from (0, i) to (1, i) 36. Let m and n be positive integers with
and from (n -1, i) to (n, i) for 0 < i < n all gcd(m, n) = 1. Compute gcd(𝟓𝒎 +
must lie on different squares, so surely 2n 𝟕𝒎 , 𝟓𝒏 + 𝟕𝒏).
-2 squares are needed. If it were possible
to obtain the complete grid with 2n -2 Sol.: Let 𝑠𝑛 = 5𝑛 + 7𝑛 . 𝐼𝑓 𝑛 ≥ 2𝑚 , note that
squares, each of these segments would lie
on one of the squares, and the same 𝑠𝑛 = 𝑠𝑚 𝑠𝑛−𝑚 − 5𝑚 7𝑚 𝑠𝑛−2𝑚 ,
would hold for the segments from (i, 0) to So gcd(𝑠𝑚 , 𝑠𝑛 ) = gcd(𝑠𝑚 , 𝑠𝑛−2𝑚 ) … similarly,
(i, 1) and from (i, n-1) to (i, n) for 0 < I < if m < n < 2m, we have gcd(𝑠𝑚 , 𝑠𝑛 )=
n. Each of the aforementioned horizontal gcd(𝑠𝑚 , 𝑠𝑛−2𝑚 ). Thus by the Euclidean
segments shares a square with only two algorithm, we conclude that if m + n is even,
of the vertical segments, so the only then gcd(𝑠𝑚 , 𝑠𝑛 ) = gcd(𝑠1 , 𝑠2 ) = 12, and if m
possible arrangements are the one we + n is odd, then gcd(𝑠𝑚 , 𝑠𝑛 ) = gcd(𝑠0 , 𝑠1 ) = 2.
gave above without the square with
corners (0, 0) and (n, n), and the 90°
rotation of this arrangement, both of
which are insufficient. Hence 2n -1 37. Let x > 1 be a real number which is not an
squares are necessary. integer. For n = 1, 2, 3, …., let 𝒂𝒏 =
⎿⌊𝒙𝒏+𝟏 ⌋⏌ − 𝒙⎿⌊𝒙𝒏 ⌋. Prove that the
35. Consider a triangulation of the plane, i.e. a sequence {𝒂𝒏 } is not periodic.
covering of the plane with triangles such
that no two triangles have overlapping Solution: Assume, on the contrary, that there
interiors, and no vertex lies in the interior exist p > 0 such that 𝑎𝑝+𝑛 = 𝑎𝑛 for every n.
of an edge of another triangle. Let A, B, C Since ⌊𝑥𝑛 ⌋ ⟶ ∞ 𝑎𝑠 𝑛 → ∞,we have
be three vertices of the triangulation and ⎿⌊𝑥 𝑛+𝑝 ⌋⏌ − ⎿⌊𝑥 𝑛 ⌋⏌ > 0 for some n; then
let 𝜽 be the smallest angle of the triangle setting 𝑎𝑛+𝑝 = 𝑎𝑛 and solving for x, we get
∆ABC. Suppose no vertices of the
triangulation lie inside the circumcircle of ⌊𝑥 𝑛+𝑝+1 ⌋ − ⌊𝑥 𝑛+1 ⌋
𝑥=
∆ABC. Prove there is a triangle 𝜎 in the ⌊𝑥 𝑛+𝑝 ⌋ − ⌊𝑥 𝑛 ⌋
triangulation such that 𝜎 ∩ ∆ABC ≠ 𝜃 and
And so x is rational.
every angle of 𝜎 is greater than 𝜃.
Put y = 𝑥 𝑝 and

304
Solving Mathematical Problems
𝑝−1 (−1+𝑖)
Sol.: Let t = 2
be one of the roots of 2𝑥 2 +
𝑝−𝑘−1
𝑏𝑚 = ∑ 𝑥 𝑎𝑚𝑝+𝑘
2𝑥 + 1; then (𝑥 + 1)𝑛 − 𝑟 is divisible by
𝑘=0
= ⎿⌊𝑥 𝑚+𝑝 ⌋ − 𝑥 𝑝 ⎿⌊𝑥 𝑚 𝑟⌋⏌ 2𝑥 2 + 2𝑥 + 1 for r real if and only if (𝑡 +
𝜋
= ⎿⌊𝑦 𝑚+1 ⌋ − 𝑦⎿⌊𝑦 𝑚 ⌋⏌. 1)𝑛 = 𝑟. Since the argument of t + 1 is 4 , this
is possible if and only if n = 4m, in which case
Since 𝑎𝑝+𝑛 = 𝑎𝑝 , we have 𝑏𝑚+1 = 𝑏𝑚 , and y (𝑡 + 1)4 𝑚 = (−4)4 . Hence (4𝑚, (−4)𝑚 ) are
is also rational number which is not an the only solutions.
integer. Now put 𝑐𝑚 = ⎿⌊𝑦 𝑚+1 −
𝑦 𝑚 ⌋⏌; 𝑡ℎ𝑒𝑛 𝑐𝑚+1 = 𝑦𝑐𝑚 = 𝑦 𝑚 𝑐1 . This means
𝑐𝑚 cannot be an integer for large m, a
40. For a natural number k, let p(k) denote the
contradiction.
smallest prime number which does not
38. Let 𝜃 be the maximum of the six angles divide k. If p(k) > 2, define q(k) to be the
between the edges of a regular product of all primes less than p(k),
tetrahedron and a given plane. Find the otherwise let q(k)= 1. Consider the
𝒙𝒏 𝒑(𝒙𝒏 )
minimum value of 𝜃 over all positions of sequence. 𝒙𝟎 = 𝟏, 𝒙𝒏+𝟏 = ; 𝒏=
𝒒(𝒙𝒏 )
the plane.
𝟎, 𝟏, 𝟐, …
Sol.: Assume the edges of the tetrahedron 𝛤
Determine all natural numbers n such that
= ABCD have length l. If we place the
𝒙𝒏 = 𝟏𝟏𝟏𝟏𝟏𝟏.
tetrahedron so that AC and BC are parallel to
the horizontal plane H. We obtain 𝜃 = 45°, Sol.: An easy induction shows that if
and we shall show this is the minimum angle. 𝑝0 , 𝑝1 , …, are the primes in increasing order
Let a, b, c, d be the projections of A, B, C, D to an n has base 2 representations 𝑐0 + 2𝑐1 +
the horizontal plane H, and 𝑙1 , … , 𝑙6 the 4𝑐2 + ⋯, then 𝑥𝑛 = 𝑝0 𝑐0 𝑝1 𝑐1 … in particular,
projection of the edges 𝐿1 , … , 𝐿6 . Since the 111111 = 3.7.11.13.37 =
angle between 𝐿1 and H has cosine l, it 𝑝1 𝑝3 𝑝4 𝑝5 𝑝10 , 𝑠𝑜 𝑥𝑛 = 111111 if and only if n
suffices to consider the shortest 𝑙𝑖 . = 210 + 25 + 24 + 23 + 21 = 1082.

If a, b, c, d from a convex quadrilateral with


largest angle at a, then one of ab or ad is at
most
1
since bd ≤ 1. Otherwise, it is easily 41. Find the greatest positive integer n for
√2 which there exist n nonnegative integers
shown that one of the 𝑙1 originating from the
𝒙𝟏 , 𝒙𝟐 , … , 𝒙𝒏, not all zero, such that for any
vertex inside the convex hull has length at
1
sequence 𝝐𝟏 , 𝝐𝟐 , … , 𝝐𝒏 , of elements of {-1,
most . 0, 1}, not all zero, 𝒏𝟑 does not divide
√3
𝝐𝟏 𝒙𝟏 + 𝝐𝟐 𝒙𝟐 + ⋯ + 𝝐𝒏 𝒙𝒏 .

Solution: The statement holds for n = 9 by


39. Find all pairs (n, r), with n a positive choosing 1, 2, 22 , … , 28 , since in that case
integer and r a real number, for which the
polynomial (𝒙 + 𝟏)𝒏 − 𝒓 is divisible by │𝜖1 + ⋯ + 𝜖𝑔28 │ ≤ 1 + 2 + ⋯ + 28 < 93 .
𝟐𝒙𝟐 + 𝟐𝒙 + 𝟏.
However, if n = 10, then 210 > 103 , so by the
pigeonhole principle, there are two subsets A

305
Solving Mathematical Problems

and B of {𝑥1 , … , 𝑥10 } whose sums are 𝑛−1

congruent modulo 103 . Let 𝜖1 = 1 if 𝑥𝑖 occurs 2𝑛(2𝑛 − 2) − ∑ 𝑘 2 𝑥𝑘 .


in A but not in B, -1 if 𝑥𝑖 occurs in B but not in 𝑘=1

A, and 0 otherwise; then ∑ 𝜖𝑖 𝑥𝑖 is divisible by Now note


𝑛3 .
𝑛−1 𝑛−1
2
∑ 𝑘 𝑥𝑘 = ∑ 𝑥𝑘 + (𝑘 − 1)(𝑘 + 1)𝑥𝑘
𝑘=1 𝑘=1
42. Let x, y be real numbers. Show that if the ≤𝑛+𝑛
set
{𝐜𝐨𝐬(𝒏𝝅𝒙) + 𝐜𝐨𝐬(𝒏𝝅𝒚) │𝒏 ∈ 𝑵} ∑ 𝑛 − 1 (𝑘 − 1)𝑥𝑘 = 𝑛 + 𝑛(2𝑛 − 2 − 𝑛)
Is finite, then x, y ∈ Q. 𝑘=1
= 𝑛2 − 𝑛.
Sol.: Let 𝑎𝑛 = cos 𝑛𝜋𝑥 𝑎𝑛𝑑 𝑏𝑛 = sin 𝑛𝜋𝑥.
Then Hence the quantity in question is at most

(𝑎𝑛 + 𝑏𝑛 )2 + (𝑎𝑛 − 𝑏𝑛 )2 = 2(𝑎𝑛 2 + 𝑏𝑛 2 ) 2𝑛(2𝑛 − 2) − (𝑛2 − 𝑛) = 3𝑛2 − 3𝑛, with


= 2 + (𝑎2𝑛 + 𝑏2𝑛 ). equality for 𝑥1 = 𝑛 − 1, 𝑥2 = ⋯ = 𝑥𝑛−2 =
0, 𝑥𝑛−1 = 1.
If {𝑎𝑛 + 𝑏𝑛 } is finite, it follows that {𝑎𝑛 − 𝑏𝑛 }
is also a finite set, and hence that {𝑎𝑛 } is
finite, since

1
𝑎𝑛 = [(𝑎𝑛 + 𝑏𝑛 )(𝑎𝑛 − 𝑏𝑛 )]. 44. Find all prime numbers p, q for which the
2
congruence 𝜶𝟑𝒑𝒒 ≡ 𝜶(𝒎𝒐𝒅 𝟑𝒑𝒒) holds
And similarly {𝑏𝑛 } is finite. In particular, for all integers 𝛼.
𝑎𝑚 = 𝑎𝑛 for some m < n, and so (n –m)𝜋x is
an integral multiple of 𝜋. We conclude x and y Sol.: Without loss of generality assume p ≤ q;
are both rational. the unique solution will be (11, 17), for which
one many check the congruence using the
Chinese Remainder Theorem.

43. Let n ≥ 3 be an integer and 𝒙𝟏 , 𝒙𝟐 , … , 𝒙𝒏−𝟏 We first have 23𝑝𝑞 ≡ 2(𝑚𝑜𝑑 3), which means
nonnegative integers such that p and q are odd. In addition, if 𝛼 is a primitive
𝒙𝟏 + 𝒙𝟐 + ⋯ + 𝒙𝒏−𝟏 = 𝒏 root mod p, then 𝛼 3𝑝𝑞−1 ≡ 1 (𝑚𝑜𝑑 𝑝) implies
𝒙𝟏 + 𝟐𝒙𝟐 + ⋯ + (𝒏 − 𝟏)𝒙𝒏−𝟏 that p -1 divides 3pq -1 as well as 3pq -1-
= 𝟐𝒏 − 𝟐. 3q(p -1) = 3q -1, and conversely that q -1
Find the minimum of the sum divides 3p -1. If p = q, we now deduce p = q =
𝒏−𝟏 3, but 427 ≡ 1(mod 27), so this fails. Hence p
𝑭(𝒙𝟏 , … , 𝒙𝒏−𝟏 ) = ∑ 𝒌𝒙𝒌 (𝟐𝒏 − 𝒌). < q.
𝒌=𝟏
Since p and q are odd primes, q ≥ p +2, so
Sol.: The desired sum can be written as (3𝑝−1)
(𝑞−1)
< 3. Since this quantity is an integer,
and it is clearly greater than 1, it must be 2.
That is, 2q = 3p +1. On the other hand, p -1

306
Solving Mathematical Problems
(9𝑝+1) X and A, we get S + (a -1) i ≡ 0 (mod p), and
divides 3q -1= 2
as well as (9p +1) –(9p
similarly, S + (b -1) (p –i) ≡ 0 (mod p).
-9) = 10. Hence p = 11, q = 17.
Therefore a + b -2 ≡ 0 (mod p) ; since a + b
≤2n +4 < p, we have a + b = 2 and so a = b =
1, contradicting the assumption that the points
45. Let n ≥ 3 be an integer and p ≥ 2n -3 a do not all lie on a circle.
prime. Let M be a set of n points in the
plane, no three collinear, and let f: M ⟶{0,
1, …, p -1} be a function such that:
46. Let x, y, z be real numbers. Prove that the
(i) Only one point of M maps to 0,
following conditions are equivalent.
and 𝟏 𝟏 𝟏
(ii) If A, B, C are distinct points in (i) x, y, z > 0 and + + ≤ 𝟏.
𝒙 𝒚 𝒛
M and k is the circumcircle of (ii) For every quadrilateral with
the triangle ABC, then sides a, b, c, d, 𝒂𝟐 𝒙 + 𝒃𝟐 𝒚 +
𝒄𝟐 𝒛 > 𝒅𝟐 .
∑ 𝒇(𝑷) ≡ 𝟎(𝒎𝒐𝒅 𝑷).
𝑷 ∈𝑴∩𝒌 Sol.: To show (i) implies (ii), note that

Show that all of the points of M lie on a 𝑎2 𝑥 + 𝑏 2 𝑦 + 𝑐 2 𝑧


circle. 1 1
≥ (𝑎2 𝑥 + 𝑏 2 𝑦 + 𝑐 2 𝑧) ( +
𝑥 𝑦
Solution: Let X be the point mapping to 0. We 1
first show that if every circle through X and + ) ≥ (𝑎 + 𝑏 + 𝑐)2 > 𝑑2 .
𝑧
two points of M contains a third point of M,
then all of the points of M lie on a circle. Using Cauchy-Schwarz after the first
Indeed, consider an inversion with center at X. inequality.
Then the image of M – {X} has the property
To show (i) implies (ii), first note that if x ≤ 0,
that the line through any two of its points
we may take a quadrilateral of sides a = n, b
contains a third point; it is a standard result
= 1, c = 1, d = n and get y + z >𝑛2 (1 − 𝑥), a
that this means the points are collinear.
contradiction for large n. Thus x > 0 and
(Otherwise, find a triangle ABC minimizing the
similarly y > 0, z > 0. Now use a quadrilateral
length of the altitude AH; there is another 1 1 1 1 1 1 1
point N on BC, but then either ABN OR CAN has of sides 𝑥 , 𝑦 , 𝑧 𝑎𝑛𝑑 𝑥 + 𝑦 + 𝑧 − 𝑛, where n is
a shorter altitude than AH, contradiction). 𝑥 𝑦 𝑧 1 1 1
large. We then get 𝑥 2 + 𝑦2 + 𝑧2 > (𝑥 + 𝑦 + 𝑧 −
Now suppose the points of M do not lie on a 1 2
)
𝑛
circle. By the above, there exists a circle
passing through M and only two points A, B of Since this holds for all n, we may take the
M. Let f(A) = i, so that by the hypothesis, f(B) limit as n⟶∞ and get
= p –i. Let a be the number so circles passing
through X, A and at least one other point of M, 1 1 1 1 1 1 1 2
+ + ≥( + + − )
let b be the number of circles passing through 𝑥 𝑦 𝑧 𝑥 𝑦 𝑧 𝑛
X, B and at least one other point of M, and let S
1 1 1
be the sum of f(P) over all P in M. By adding And hence 𝑥 + 𝑦 + 𝑧 ≤ 1.
the relations obtained from the circles through

307
Solving Mathematical Problems

𝒂𝟏 𝒙 + 𝒂𝟐 𝒚 + 𝒂𝟑 𝒛 = 𝟎
𝒂𝟒 𝒙 + 𝒂𝟓 𝒚 + 𝒂𝟔 𝒛 = 𝟎
47. Let n be a positive integer and D a set of n 𝒂𝟕 𝒙 + 𝒂𝟖 𝒚 + 𝒂𝟗 𝒛 = 𝟎
concentric circles in the plane. Prove that Has a solution (𝒙𝟎 , 𝒚𝟎 , 𝒛𝟎 ) in nonzero
if the function f : D ⟶D satisfies integers.
d(f(A), f(B)) ≥ d(A, B) for all A, B, 𝜖, D,
then d(f(A), f(B)) = d(A, B) for every Sol.: Label the elements of X in increasing
A, B, 𝜖D. order 𝑥1 < ⋯ < 𝑎3𝑛2 , and put

Sol.: Label the circles 𝐷1 , … , 𝐷𝑛 in increasing 𝑋1 = {𝑥1 , … , 𝑥𝑛2 }, 𝑋2 = {𝑥𝑛2 +1 , … . , 𝑥2𝑛2 }, 𝑋3


order of radius, and let 𝑟1 denote the radius = {𝑥𝑛2 +1 , … , 𝑥3𝑛2 },
𝐷𝑖 . Clearly the maximum of d(A, B) occurs
when A and B are antipodal points on D. Let Define the function f: 𝑋1 × 𝑋2 × 𝑋3 → 𝑋 × 𝑋
ABCD be the vertices of a square inscribed in as follows: f(a, b, c) = (b –a, c –b).
𝐷𝑛 ; then f(A) and f(C) are antipodal, as are
The domain of f contains 𝑛6 elements. The
f(B) and f(D). In addition, each of the minor
range of f, on the other hand, is contained in
arcs f(A) f(B) and f(B) f(C) must be at least a
the subset of X × 𝑋 of pairs whose sum is at
quarter arc, thus f(B) bisects one of the
most 𝑛3 , a set of cardinality.
semicircles bounded by f(A) and f(C), and
f(D) bisects the other. Now if P is any point on 𝑛3 −1
𝑛3 (𝑛3 − 1) 𝑛6
the minor arc AB, then the arcs f(P) f(A) and ∑ 𝑘= < .
2 2
f(P) f(B), which are at least as long as the arc 𝑘=1
PA and PB, and up to the quarter arc f(P) f(B).
By the pigeonhole principle, some three
We conclude f is isometric on 𝐷𝑛 .
triples (𝑎𝑖 , 𝑏𝑖 , 𝑐𝑖 ) (i = 1, 2, 3) map to the same
Since f is clearly injective and is now bijective pair, in which case x =𝑏1 − 𝑐1 , 𝑦 = 𝑐1 −
on 𝐷𝑛 𝑓 maps 𝐷1 𝑈 … . 𝑈𝐷𝑛−1 into itself. Thus 𝑎1 , 𝑧 = 𝑎1 − 𝑏1 is a solution in nonzero
we many repeat the argument to show that f integers. Note that 𝑎𝑖 , cannot equal 𝑏𝑗 since 𝑋1
is isometric on each 𝐷𝑖 . To conclude, it and 𝑋2 and so on, and that 𝑎1 = 𝑎2 implies
suffices to show that distances between that the triple (𝑎1 , 𝑏1 , 𝑐1) and (𝑎2 , 𝑏2 , 𝑐2 ) are
adjacent circles, say 𝐷1 and 𝐷2, are preserved. identical, a contradiction. Hence the nine
This is easy; choose a square ABCD on 𝐷1 and numbers chosen are indeed distinct.
A’, B’, C’, D’ be the points on 𝐷2 closet to A, B,
C, D, respectively. Then A’B’C’D’ also from a
square, and the distance from A to C’ is the 49. Which are there more of among the
maximum between any point on 𝐷1 and any natural numbers from 1 to 1000000,
point on 𝐷3. Hence the eight points maintain inclusive: numbers that can be
their relative position under f, which suffices represented as the sum of a perfect square
to prove isometry. and a (positive) perfect cube,
48. Let n ≥ 3 be an integer and X ⊆ {1, 2, …, or numbers that cannot be?
𝒏𝟑 } a set of 3𝒏𝟐 elements. Prove that one Sol.: There are more numbers that not of this
can find the distinct numbers 𝒂𝟏 , … , 𝒂𝟗 in form. Let n = 𝑘 2 + 𝑚3 , where k, m, n ∈ N and
X such that the system n ≤ 1000000. Clearly k ≤ 1000 and m ≤ 100.

308
Solving Mathematical Problems

Therefore there cannot be more numbers in Let n = pq. Then 𝑥 𝑝𝑞 + 𝑦 𝑝𝑞 = 𝑝𝑘 𝑜𝑟 (𝑥 𝑝 )𝑞 +


the desired from than the 1000000 pairs (k, (𝑦 𝑝 )𝑞 = 𝑝𝑘 . If q > 1, then by the same
m). argument, p divides q. If q =1, then n = p.
Repeating this argument, we deduce that n =
𝑝𝑙 for some natural number l.
50. Let x, y, p, n, k be natural numbers such
that
𝒙𝒏 + 𝒚𝒏 = 𝒑𝒌 . 51. In the Duma there are 1600 delegates, who
Prove that if n > 1 is odd, and p is an odd have formed 16000 committees of 80
prime, then n is a power of p. persons each. Prove that one can find two
committees having no fewer than four
Sol.: Let m = gcd(x, y). Then x = m𝑥1 , 𝑦 = common members.
𝑚𝑦1 and by virtue of the given equation,
𝑚𝑛 (𝑥1 𝑛 + 𝑦1 𝑛 ) = 𝑝𝑘 , and so m = 𝑝𝛼 for some Solution: Suppose any two committees have
nonnegative integer 𝛼. It follows that 𝑥1 𝑛 + at most three common members. Have two
𝛼 deputies count the possible ways to choose a
𝑦1 𝑛 = 𝑝𝑘−𝑛 . (1)
chairman for each of three sessions of the
Since n is odd, Duma. The first deputy assumes that any
deputy can chair any session, and so gets
𝑥1 𝑛 + 𝑦1 𝑛 16003 possible choices. The second deputy
= 𝑥1 𝑛−1 − 𝑥1 𝑛−2 𝑦1
𝑥1 + 𝑦1 makes the additional restriction that all of the
+ 𝑥1 𝑛−3 𝑦1 2 − ⋯ chairmen belong to a single committee. Each
− 𝑥1 𝑦1 𝑛−2 + 𝑦1 𝑛−1 , of the 16000 committees yields 803 choices,
but this is an over count; each of the 16000
Let A denote the right side of the equation. By (16000−1)
the condition p > 2, it follows that at least one 2
pairs of committees give at most 33
of 𝑥1 , 𝑦1 is greater than 1, so since n > 1. A > overlapping choices. Since the first deputy
1. counts no fewer possibilities than the second,
𝛼
we have the inequality
From (1) it follows that A(𝑥1 + 𝑦1 ) = 𝑝𝑘−𝑛 ,
so since 𝑥1 + 𝑦1 > 1, A >1, both of these 16000.15999 3
16003 ≥ 16000. 803 − 3 .
numbers are divisible by p, moreover, 𝑥1 + 2
𝑦1 = 𝑝𝛽 for some natural number 𝛽. However,
Thus 16000.15999 3
1600. 803 − 3
2
𝐴 = 𝑥1 𝑛−1 − 𝑥1 𝑛−2 (𝑝𝛽 − 𝑥1 ) + ⋯
> 16000. 803
− 𝑥1 (𝑝𝛽 − 𝑥1 )𝑛−2 16000.15999 42
+ (𝑝𝛽 − 𝑥1 )𝑛−1 −
2 2
= 𝑛𝑥1 𝑛−1 + 𝐵𝑝.
16000. 43
Since A is divisible by p and 𝑥1 is relatively = + 213 . 106 − 212 . 106 > 212 . 106
4
prime to p, it follows that n is divisible by p. = 16003 .

309
Solving Mathematical Problems

We have a contradiction. 𝑦1 ≥ ⋯ ≥ 𝑦𝑡 ≥ 𝑥 ≥ 𝑦𝑡+1 ≥ ⋯ ≥ 𝑦𝑚 . (If there


are no coins of weight greater than x, the
result is clear). We need to show that 𝑥𝑠 +
52. Show that in the arithmetic progression 𝑥𝑠+1 + ⋯ + 𝑥𝑛 ≥ 𝑥𝑡 + 𝑦𝑡+1 + ⋯ + 𝑦𝑚 . Since
with first term 1 and ratio 729, there are 𝑥1 + ⋯ + 𝑥𝑛 = 𝑦1 + ⋯ + 𝑦𝑚 = 𝐴, this
infinitely many powers of 10. inequality can be equivalently written 𝑥𝑠 +
(𝐴 − 𝑥1 − ⋯ − 𝑥𝑚 ) ≥ 𝑥𝑡 + (𝐴 − 𝑦1 − ⋯ − 𝑦𝑡 ),
Sol.: We will show that for all natural which in turn can be rewritten
numbers n, 1081𝑛 − 1 is divisible by 729. In
𝑥1 + ⋯ + 𝑥𝑠 + 𝑥(𝑡 − 𝑠) ≤ 𝑦1 + ⋯ + 𝑦𝑡 ,
fact, 1081𝑛 − 1 = (1081 )𝑛 − 1𝑛 = (1081 −
1). 𝐴, 𝑎𝑛𝑑 This is what we will prove,
1081𝑛 − 1 = ⏟
9…9 If t ≥ s, then
81

=⏟
9…9… ⏟
10 … 01 ⏟
10 … 01 … ⏟
10 … 01 𝑥1 + ⋯ + 𝑥𝑠 + 𝑥(𝑡 − 𝑠)
9 8 8 8 = (𝑥1 + ⋯ + 𝑥𝑠 )
+⏟ (𝑥 + ⋯ + 𝑥)
= 9⏟
1…9…⏟
10 … 01 ⏟
10 … 01 … ⏟
10 … 01 𝑡−𝑠
9 8 8 8 ≤ (𝑦1 + ⋯ + 𝑦𝑠 )
The second and third factors are composed of + (𝑦𝑠+1 + ⋯ + 𝑦𝑡 ),
9 units, so the sum of their digits is divisible
Since 𝑥1 + ⋯ + 𝑥𝑠 ≤ 𝑦1 + ⋯ + 𝑦𝑠 (from the
by 9, that is, each is a multiple of 9. Hence
given condition) and 𝑦𝑠+1 ≥ ⋯ ≥ 𝑦𝑡 ≥ 𝑥.
1081𝑛 − 1 is divisible by 93 = 729, as is
1081𝑛 − 1 for any n. If t < s, then 𝑥1 + … + 𝑥𝑠 + 𝑥(𝑡 − 𝑠) ≤ 𝑦1 +
⋯ + 𝑦𝑡 is equivalent to

𝑥1 + ⋯ + 𝑥𝑠 ≤ 𝑦1 + ⋯ + 𝑦𝑡 + ⏟
(𝑥 + ⋯ + 𝑥)
53. Two piles of coins lie on a table. It is 𝑡−𝑠
known that the sum of the weights of the
coins in the two piles are equal, and for The latter inequality follows from the fact
any natural number k, not exceeding the that
number of coins in either pile, the sum of
𝑥1 + ⋯ + 𝑥𝑠 ≤ 𝑦1 + ⋯ + 𝑦𝑠
the weights of the k heaviest coins in the
= (𝑦1 + ⋯ + 𝑦𝑡 )
first pile is not more than that of the
+ (𝑦𝑡+1 + ⋯ + 𝑦𝑠 )𝑎𝑛𝑑 𝑦𝑠 ≤ ⋯
second pile. Show that for any natural
≤ 𝑦𝑡+1 ≤ 𝑥.
number x, if each coin (in either pile) of
weight not less than x is replaced by a coin
of weight x, the first pile will not be lighter
than the second.

Sol.: Let the first pile have n coins of weights

𝑥1 ≥ 𝑥2 ≥ ⋯ ≥ 𝑥𝑛 , and let the second pile 54. Can a 𝟓 × 𝟕 checkerboard be covered by


have m coins of weights 𝑦1 ≥ 𝑦2 ≥ ⋯ ≥ L’s (figures formed from a 𝟐 × 𝟐 square by
𝑦𝑚 , 𝑤ℎ𝑒𝑟𝑒 𝑥1 ≥ ⋯ ≥ 𝑥𝑠 ≥ 𝑥 ≥ 𝑥𝑠+1 k and removing one of its four 𝟏 × 𝟏 corners),

310
Solving Mathematical Problems

not crossing its borders, in several layers Then 𝑥 3 + 𝑦 3 = 3𝑚 and x + y = 3𝑛 . To prove


so that each square of the board is covered the inequality n ≥ 2m, it suffices to show that
by the same number of L’s? 𝑥 3 + 𝑦 3 ≥ (𝑥 + 𝑦)2 , or 𝑥 2 − 𝑥𝑦 + 𝑦 2 ≥ 𝑥 +
𝑦. Since x ≥ y +1, 𝑥 2 − 𝑥 = 𝑥(𝑥 − 1) ≥
Solution: Color the cells of the checkerboard
𝑥𝑦, 𝑎𝑛𝑑 (𝑥 2 − 𝑥 + 𝑥𝑦) + (𝑦 2 − 𝑦) ≥ 𝑦(𝑦 −
alternately black and white, so that the
1) ≥ 0, and the inequality n ≥ 2m follows.
corners are all black. In each black square we
write the number -2, and in each white From the identity (𝑥 + 𝑦)3 − (𝑥 3 + 𝑦 3 ) =
square 1. Note that the sum of the numbers in 3𝑥𝑦(𝑥 + 𝑦) it follows that 32𝑚−1 − 3𝑛−𝑚−1 =
the cells covered by each L is non-negative, 𝑥𝑦.
and consequently if we are given a covering
of the board in k layers, the sum over each L But 2m -1 ≥ 1, and n –m -1 ≥n -2m ≥0. If
of the numbers covered by that L is strict inequality occurs in either place in the
nonnegative. But if this number is S and s is last inequality, then 32𝑚−1 − 3𝑛−𝑚−1 is
the sum of the numbers on the board, then divisible by 3 while xy is not. Hence n –m -1 =
n -2m = 0, and so m = 1, n = 2 and 32 = 23 +
S = ks = k (-2.12+23.1)= -k <0. We have a 13 .
contradiction.
Note: The inequality 𝑥 2 − 𝑥𝑦 + 𝑦 2 ≥ 𝑥 + 𝑦
can alternatively be shown by nothing that

55. Find all natural numbers n, such that there 𝑥 2 − 𝑥𝑦 + 𝑦 2 − 𝑥 − 𝑦 = (𝑥 − 𝑦)2 + (𝑥 −


exist relatively prime integers x and y and 1)(𝑦 − 1) − 1 ≥ 0,
an integer k >1 satisfying the equation
𝟑𝒏 = 𝒙𝒌 + 𝒚𝒌 . Since (𝑥 − 𝑦)2 ≥ 1.

Sol.: The only solution is n = 2.


56. Show that if the integers 𝒂𝟏 , … , 𝒂𝒎 are
𝑛 𝑘 𝑘
Let 3 = 𝑥 + 𝑦 , where x, y are relatively nonzero and for each k = 0, 1, …, m(n < m
prime integers with x > y, k > 1, and n a -1),
natural number. Clearly neither x nor y is a 𝒂𝟏 + 𝒂𝟐 𝟐𝒌 + 𝒂𝟑 𝟑𝒌 + ⋯ + 𝒂𝒎 𝒎𝒌 = 𝟎,
multiple of 3. Therefore, if k is even𝑥 𝑘 𝑎𝑛𝑑 𝑦 𝑘
Then the sequences 𝒂𝟏 , … , 𝒂𝒎 contains at least
are congruent to 1 mod 3, so their sum is
n + 1 pairs of consecutive terms having
congruent to 2 mod 3, and so is not a power
opposite sings.
of 3.
Solution: We many assume 𝑎𝑚 > 0, since
If k is odd and k > 1, then 3𝑛 = (𝑥 +
otherwise we may multiply each of the
𝑦)(𝑥 𝑘−1 − ⋯ + 𝑦 𝑘−1 ). Thus x + y = 3𝑚 for
numbers by -1. Consider the sequence
some m ≥ 1. We will show that n ≥ 2m. Since
3
𝑏1 , … , 𝑏𝑚 , where 𝑏𝑖 = ∑𝑛𝑗=0 𝑐𝑗 𝑖 𝑗 for an
𝑘
(see the solution to Russia 3), by putting arbitrary sequence of real numbers 𝑐0 , … , 𝑐𝑛 .
𝑘/3 𝑘/3
𝑥1 = 𝑥 𝑎𝑛𝑑 𝑦1 = 𝑦 we may assume k = From the given condition,
3.

311
Solving Mathematical Problems
𝑚 𝑚 𝑛 𝑛 𝑛
these is either half of or twice a. If at least one
∑ 𝑎𝑖 𝑏𝑖 = ∑ 𝑎𝑖 ∑ 𝑐𝑗 𝑖 𝑗 = ∑ 𝑐𝑖 ∑ 𝑎𝑖 𝑖 𝑗 = 0. is less a, it equals b; otherwise, both are equal.
𝑖=1 𝑖=1 𝑗=0 𝑗=0 𝑗=1
Either option contradicts the assumption that
Suppose now that the sequence 𝑎1 , … , 𝑎𝑚 has the numbers are distinct.
k pairs of neighbors that differ in sign, where
k < n +1, and let 𝑖1 , … , 𝑖𝑘 be the indices of the
first members of these pairs. 58. Can the number obtained by written the
numbers from 1 to n in order (n >1) be
Let 𝑏𝑖 = 𝑓(𝑖) = (𝑖 − 𝑥1 )(𝑖 − 𝑥2 ) … (𝑖 − 𝑥𝑘 ), the same when read left-to-right and right-
1
where 𝑥𝑘 = 𝑖𝑘 + 2 (𝑖 = 1, 2, … , 𝑘). The to left?
function f changes sign only at the points
Sol.: This is not possible. Suppose N = 123
𝑥1 , … , 𝑥𝑘 , and so 𝑏1 𝑎𝑛𝑑 𝑏𝑖+1 have different
…321 is an m digit symmetric number,
signs if and only one of the 𝑥𝑙 falls between
them, which means i = 𝑖𝑙 . We deduce that the formed by writing the numbers from 1 to n in
sequences 𝑎1 , … , 𝑎𝑚 𝑎𝑛𝑑 𝑏1 , … , 𝑏𝑚 have the succession. Clearly m >18. Also let A and B be
the numbers formed from the first and last k
same pairs of neighbors of opposite sign.
Since 𝑎𝑚 𝑎𝑛𝑑 𝑏𝑚 are positive, we have that digits, respectively, of N, where k = ⌊𝑚/2⌋⎿ .
Then if 10𝑝 is the largest power of 10 dividing
𝑎𝑖 𝑎𝑛𝑑 𝑏𝑖 have the same sign for i = 1, …., m,
so ∑𝑚 A, then n> 2. 10𝑝+1 , that is, n has at most p
𝑖=1 𝑎𝑖 𝑏𝑖 >0, a contradiction.
+2 digits. Moreover, A and B must contain the
fragments

57. At the vertices of a cube are written eight 99


⏟ …9⏟
100 … 01 𝑎𝑛𝑑 ⏟
100 … 0 ⏟
199 … 9
pair wise district natural numbers, and on 𝑝 𝑝 𝑝 𝑝

each of its edges is written the greatest Respectively, which is impossible.


common divisor of the numbers at the end
points of the edge. Can the sum of the
numbers written at the vertices be the
same as the sum of the numbers written at 59. Do there exist three natural numbers
the edges? greater than 1, such that the square of
each, minus one, is divisible by each of the
Sol.: This is not possible. Note that if a and b others?
are natural numbers with a > b, then gcd(a,
𝑎 Sol.: Such integers do not exist. Suppose a ≥ b
b) ≤ b and gcd(a, b) ≤2 .It follows that if a ≠ b,
(𝑎+𝑏)
≥ c satisfy the desired condition. Since 𝑎2 −
then gcd(a, b)≤ . Adding 12 such 1is divisible by b, the numbers a and b are
3
inequalities, corresponding to the 12 edges, relatively prime. Hence the number 𝑐 2 − 1,
we find that the desired condition is only which is divisible by a and b, must be a
possible if gcd(a, b) =
(𝑎+𝑏)
in each case. But multiple of ab, so in particular 𝑐 2 − 1 ≥
3
𝑎𝑏. 𝐵𝑢𝑡 𝑎 ≥ 𝑐 𝑎𝑛𝑑 𝑏 ≥ 𝑐, 𝑠𝑜 𝑎𝑏 ≥ 𝑐 2 , a
in this case the larger of a and b is twice the
contradiction.
smaller; suppose a = 2b. Consider the
numbers c and d assigned to the vertices of
the other end points of the other two edges
coming out of the vertex labeled a. Each of

312
Solving Mathematical Problems

60. Does there exist a finite set M of nonzero Solution: Let d = gcd(a, b) and put a =
real numbers, such that for any natural (𝑚𝑑+1)
md and b = nd. Then we have 𝑛𝑑
+
number n a polynomial of degree no less (𝑛𝑑+1) (𝑚2 𝑑+𝑚+𝑛2 𝑑+𝑛)
than n with coefficients in M, all of whose = is an integer, so
𝑚𝑑 𝑚𝑛𝑑
roots are real and belong M? that in particular, d divides 𝑚2 𝑑 + 𝑚 +
𝑛2 𝑑 + 𝑛 and also m + n. However, this
Solution: Such a set does not exist. Suppose means d ≤ m + n, and so d ≤
on the contrary that M = {𝑎1 , 𝑎1 , … , 𝑎𝑛 }
√𝑑(𝑚 + 𝑛) = √𝑎 + 𝑏 .
satisfies the desired property. Let m = min
{│𝑎1 │, …, │𝑎𝑛 │} and M = max {│𝑎1 │, …,
│𝑎𝑛 │}; the condition implies M ≥ m > 0.
62. Let G be the centroid of the triangle ABC.
Consider the polynomial P(x) = 𝑏𝑘 𝑥 𝑘 + ⋯ + Prove that if AB + GC = AC + GB, then ABC
𝑏1 𝑥 + b0 all of whose coefficients b0 , … , bk are is isosceles.
roots x1 , … . , xk lie in M. By Vieta’s theorem.
Sol.: Let a, b, c, be the lengths of sides BC, CA,
𝑏𝑘−1 AB, respectively. By Stewart’s theorem and
− = 𝑥1 + ⋯ 𝑥𝑘
bk the fact that G trisects each median (on the
side further from the vertex), we deduce
𝑏𝑘−2
𝑥1 𝑥2 + 𝑥1 𝑥3 + ⋯ + 𝑥𝑘−1 𝑥𝑘 =
𝑏𝑘 9𝐺𝐵2 = 2𝑎2 + 2𝑐 2 − 𝑏 2 , 9𝐺𝐶 2
= 2𝑎2 + 2𝑏 2 − 𝑐 2 .
And so
Now assume b > c. Assuming AB + GC = AC +
2 2
𝑏𝑘−1 2 𝑏𝑘−2
𝑥1 + ⋯ + 𝑥𝑘 = −2 . GB, we have
𝑏𝑘 2 𝑏𝑘
3(𝑏 − 𝑐) = √2𝑎2 + 2𝑏 2 − 𝑐 2
It follows that
− √2𝑎2 + 2𝑐 2 − 𝑏 2
2
𝑏𝑘−1 𝑏𝑘−2
𝑘𝑚2 ≤ 𝑥1 2 + ⋯ + 𝑥𝑘 2 = 2 −2 3(𝑏 2 − 𝑐 2 )
𝑏𝑘 𝑏𝑘 =
𝑀 2
𝑀 √2𝑎2 + 2𝑏 2 − 𝑐 2 + √2𝑎2 + 2𝑐 2 − 𝑏 2
≤ 2+2 .
𝑚 𝑚 3(𝑏 2 − 𝑐 2 )
<
Hence 𝑘 ≤
𝑀2
+
2𝑀
, contradiction the fact that √2 (𝑏 − 𝑐)2 + 2𝑏 2 − 𝑐 2 + √2 (𝑏 − 𝑐)2 + 2𝑐 2 − 𝑏 2
𝑚4 𝑚3
P may have arbitrarily large degree. Since 𝑎2 > (𝑏 − 𝑐)2 by the triangle inequality.
However,

2 (𝑏 − 𝑐)2 + 2𝑏 2 − 𝑐 2 = (2𝑏 − 𝑐)2 , so we


61. The natural numbers a and b are such that
𝒂+𝟏 𝒃+𝟏 have
𝒃
+ 𝒂
3(𝑏2 −𝑐 2 )
3(𝑏 − 𝑐) < .
is an integer. Show that the greatest common 2𝑏−𝑐+│2𝑐−𝑏│

divisor of a and b is not greater than √𝒂 + 𝒃. If b ≤2c then the two sides are equal, a
contradiction. If b ≤ 2c we get 9(𝑏 − 𝑐)2 <

313
Solving Mathematical Problems

3(𝑏 2 − 𝑐 2 ); upon dividing off 3(b –c) and Sol.: Note that 𝑘𝑖 is the number obtained by
rearranging, we get 2b < 4c, again a writing i in base 2 and reading the result as a
contradiction. Thus we cannot have b > c or number in base 3, and 𝑎𝑖 is the sum of the
similarly b < c, so b = c. exponents of the powers of 3 used. In
particular, 1996 = 210 + 29 + 28 + 27 + 26 +
23 + 22 , 𝑠𝑜 𝑎1996 = 10 + 9 + 8 + 6 + 6 + 3 +
63. Find all real solutions of the equation 2 = 45.

√𝒙𝟐 − 𝒑 + 𝟐√𝒙𝟐 − 𝟏 = 𝒙
65. In a parallelogram ABCD with ∠A <𝟗𝟎°,
For each real value of p.
the circle with diameter AC meets the lines
Sol.: Squaring both sides, we get CB and CD again at E and F, respectively,
and the tangent to this circle at A meets BD
𝑥 2 = 5𝑥 2 − 4 − 𝑝 + 4√(𝑥 2 − 𝑝)(𝑥 2 − 1 at P. Show that P, F, E are collinear.

Isolating the radical and squaring again, we Sol.: Without loss of generality, suppose B, D,
get P occur in that order along BD. Let G and H be
the second intersection of AD and AB with the
16(𝑥 2 − 𝑝)(𝑥 2 − 1) = (4𝑥 2 − 𝑝 − 4)2 , circle. By Menelaus' theorem, it suffices to
show that
Which reduces to (16 − 8𝑝)𝑥 2 = 𝑝2 − 8𝑝 +
16. Since x ≥ 0(it is the sum of two square 𝐶𝐸. 𝐵𝑃. 𝐷𝐹
│𝑝−4│
=1
roots), we have x = 𝐸𝐵. 𝑃𝐷. 𝐹𝐶
√16−8𝑝
Find note that
If a solution exists. We need only determine
𝐵𝑃 𝐴𝐷 sin ∠𝐵𝐴𝑃 sin ∠𝐴𝑃𝐷 sin ∠𝐵𝐴𝑃
when this value actually satisfies. Certainly = sin ∠𝐴𝑃𝐵 sin ∠𝐷𝐴𝑃 = sin ∠𝐷𝐴𝑃
𝐴𝐵 𝐷𝑃
we need p ≤ 2. In that case plugging in our
claimed value of x and multiplying through by Since AP is tangent to the circle, ∠BAP =
√16 − 8𝑝 gives │3p -4│+2│p│ = 4 –p. ∠HAP

4 4 = 𝜋 - ∠HCA = 𝜋 -∠FCA; similarly, ∠DAP =


If p ≥ 3 this becomes 6p = 8, or p = 3; if 0 ≤ p
∠GCA = ∠EAC. We conclude
4
≤ 3 this holds identically; if p ≤ 0 this
𝐵𝑃 𝐴𝐷 sin ∠𝐹𝐴𝐶 𝐹𝐶
becomes 4p = 0, or p = 0. We conclude there 𝐴𝐵 𝐷𝑃
= sin ∠𝐸𝐴𝐶 = 𝐸𝐶
exists a solution if and only if 0≤ 𝑝 ≤ 4/3, in
𝐷𝐹 𝐷𝐴
which case it is the solution given above. Finally we note that = because the right
𝐵𝐸 𝐴𝐵
triangles AFD and AED have the same angles
at B and D and are thus similar. This prove
64. Let ∏𝟏𝟗𝟗𝟔 𝟑 𝒏 𝒌𝟏 the claim.
𝒏=𝟏 (𝟏 + 𝒏𝒙 ) = 𝟏 + 𝒂𝟏 𝒙 +
𝒂𝟐 𝒙𝒌𝟐 + ⋯ + 𝒂𝒎 𝒙𝒌𝒎 , 66. Given real numbers 0 = 𝒙𝟏 < 𝒙𝟐 < ⋯ <
𝒙𝟐𝒏 < 𝒙𝟐𝒏+𝟏 = 𝟏 with 𝒙𝒊+𝟏 − 𝒙𝒊 ≤ 𝒉 for 1
where 𝒂𝟏 , 𝒂𝟐 , … , 𝒂𝒎 are nonzero and 𝒌𝟏 <
𝒌𝟐 < ⋯ < 𝒌𝒎 . Find 𝒂𝟏𝟗𝟗𝟔 .

314
Solving Mathematical Problems
𝟏−𝒉 Sol.: The holds if and only if b < a, if b > a, the
≤ i ≤ 2n, show that 𝟐
<
𝑏 𝑛
∑𝒏𝒊=𝟏 𝒙𝟐𝒊 (𝒙𝟐𝒊+𝟏 − 𝒙𝟐𝒊−𝟏 ) <
𝟏+𝒉
. sequence 𝑥𝑛 = (𝑎) satisfies the condition
𝟐
but does not go to zero, if b = a, the sequence
Sol.: The different between the middle 1 1
𝑥𝑛 = 1 + 2 + ⋯ + 𝑛 does likewise. Now
1
quantity and is the difference between the
2 suppose b < a. If L and M are the limit inferior
sum of the areas of the rectangles bounded by and limit superior of the given sequence, the
the lines x = 𝑥2𝑖−1 , 𝑥 = 𝑥2𝑖+1 , 𝑦 = 0, 𝑦 = 𝑥2𝑖 𝑏
condition implies M ≤ (𝑎) 𝐿; since L ≤ M, we
and the triangle bounded by the lines y = 0, x 𝑏
= 1, x = y. The area contained in the have M ≤ ( ) 𝑀, and so L, M ≥ 0. Similarly,
𝑎
rectangles but not the triangle is a union of 𝑏
the condition implies L ≥ (𝑎) 𝑀, and since M
triangles of total base less than 1 and height 𝑏
at most h, as is the area contained in the ≥ L, we have L ≥ (𝑎) 𝐿, so L, M ≤ 0; therefore
triangle but not the rectangles. Hence the sum L = M = 0 and the sequence converges to 0.
1 ℎ
differs from but at most , as desired.
2 2 69. Consider the pair of four –digit positive
integers (M, N) = (3600, 2500). Notice
67. Find the maximum number of pair wise
that M and N are both perfect squares,
disjoint sets of the from 𝑺𝒂,𝒃 = {𝒏𝟐 +
with equal digits in two places, and
𝒂𝒏 + 𝒃: 𝒏 ∈ 𝒁}𝒘𝒊𝒕𝒉 𝒂, 𝒃 ∈ 𝒁. differing digits in the remaining two
places. Moreover, when the digits differ,
Solution: Only two such sets are possible, for
the digit in M is exactly one greater than
example, with (a, b) = (0, 0) and (0, 2) (since
the corresponding digit in N. Find all pairs
2 is not a difference of squares). There is no
of four –digit positive integers (M, N) with
loss of generality in assuming a ∈ {0, 1} by a
these properties.
suitable shift of n, and the sets generated by
(0, a) and (1, b) have the common value Sol.: If M = 𝑚2 𝑎𝑛𝑑 𝑁 = 𝑛2, then (m+ n)
(𝑎 − 𝑏)2 + 𝑎 = (𝑎 − 𝑏)2 + (𝑎 − 𝑏) + 𝑏.Thus (m –n) ∈{11, 101, 110, 1001, 1010, 1100}.
we have a = 0 or a = 1 universally.
Since M and N are four-digit numbers, we
First suppose a = 0. If b –c ≠ 2 (mod 4), then must have 32 ≤ n <m ≤ 99, and so 65 ≤
(0, b) and (0, c) gives a common value m +n ≤ 197. Moreover, m +n and m –n
because b –c is a difference of squares, clearly are both odd or both even, so 11, 110 and
this precludes having three disjoint sets. Now 1010 lead to no solutions. From this we
suppose a = 1. If b –c is even, we can find x, y get exactly five acceptable factorizations.
such that b –c = (x +y +1) (x –y), and so 𝑥 2 +
𝑥 + 𝑏 = 𝑦 2 + 𝑦 + 𝑐, again, this precludes 101 = (m+ n) (m –n) = 101 × 1
having three disjoint sets.
1001 = (m +n) (m –n) = 143 × 7

1001 = (m +n) (m –n) = 91 × 11


68. For which ordered pairs of positive real
1001 = (m+ n) (m –n) = 77 × 13
numbers (a, b) is the limit of every
sequence {𝒙𝒏 } satisfying the condition. 1100 = (m+ n) (m –n) = 110 × 10
𝐥𝐢𝐦 (𝒂𝒙𝒏+𝟏 − 𝒃𝒙𝒏 ) = 𝟎
𝒏 →𝜶

315
Solving Mathematical Problems

Giving the solutions (M, N) = (2601, Which is non-decreasing.


2500), (5625, 4624), (2601, 1600),
(2025, 1024), (3600, 2500).
72. Let a, b, c be positive real numbers.
(a) Prove that 𝟒(𝒂𝟑 + 𝒃𝟑 ) ≥ (𝒂 + 𝒃)𝟑
70. A function f defined on the positive (b) Prove that 𝟗(𝒂𝟑 + 𝒃𝟑 + 𝒄𝟑 ) ≥
integers satisfies f(1) = 1996 and (𝒂 + 𝒃 + 𝒄)𝟑
𝒇(𝟏) + 𝒇(𝟐) + ⋯ + 𝒇(𝒏) =
𝒏𝟐 𝒇(𝒏)(𝒏 > 1). Sol.: Both parts follow from the Power Mean
inequality: for r > 1 and 𝑥1 , … , 𝑥𝑛 positive,
Sol.: An easy induction will show that
1/𝑟
𝑥1 𝑟 + ⋯ + 𝑥𝑛 𝑟 𝑥1 + ⋯ + 𝑥𝑛
2 × 1996 ( ) ≥ ,
𝑓(𝑛) = 𝑛 𝑛
𝑛(𝑛 + 1)

Namely, Which in turn follows from Jensen’s


inequality applied to convex function 𝑥 𝑟 .
1 3992 3992
𝑓(𝑛) = 𝑛2 −1 ( 1.2
+ ⋯ + (𝑛−1)𝑛)

3992 1 1 1 1 1
= 𝑛2 −1 (1 − 2 + 2 − 3 + ⋯ + 𝑛−1 − 𝑛) 73. Find all Solutions in non-negative integers
x, y, z of the equation.
3992
= (𝑛+1)(𝑛−1) (1 − )
1 𝟐𝒙 + 𝟑𝒚 = 𝒛𝟐
𝑛

3992 𝑛−1 3992 Sol.: If y = 0, then 2𝑥 = 𝑧 2 − 1 = (𝑧 + 1)(𝑧 −


= (𝑛+1)(𝑛−1) 𝑛
= 𝑛(𝑛+1) 1), so z +1 and z -1 are powers of 2. The only
powers of 2 which differ by 2 are 4 and 2, so
2
In particular, f(1996) = (x, y, z) = (3, 0, 3).
1997

If y > 0, then 2𝑥 is a quadratic residue modulo


3, hence x is even. Now we have 3𝑦 = 𝑧 2 −
𝒏 𝑥 𝑥
71. Define 𝒒(𝒏) = ⎿ ⌊
⌊√𝒏⌋
⌋ (𝒏 = 𝟏, 𝟐, … ). 2𝑥 = (𝑧 + 22 ) (𝑧 − 22 ). The factors are
powers of 3, say z+2𝑥/2 = 3𝑚 and z −2𝑥/2 =
Determine all positive integers n for which 𝑥

q(n) > q(n +1). 3𝑛 , but then 3𝑚 − 3𝑛 = 22+1 . Since the right
side is not divisible by 3, we must have n = 0
𝑥
Sol.: We have q(n) > q(n+1) if and only if n
and 3𝑚 − 1 = 22+1 .
+1 is a perfect square. Indeed, if n +1 = 𝑚2 ,
then If x = 0, we have m = 1, yielding (x, y, z) = (0,
1, 2). Otherwise, 3𝑚 − 1 is divisible by 4, so m
𝑚2 − 1 𝑚2 𝑥 𝑚 𝑚
𝑞(𝑛) = ⎣ ⎦ = 𝑚 + 1, 𝑞(𝑛 + 1) = ⎣ ⎦ is even and 22+1 = (3 2 + 1) (3 2 − 1). The
𝑚−1 𝑚
=𝑚 two factors on the right are powers of 2
differing by 2, so they are 2 and 4, giving x =
On the other hand, for n = 𝑚2 + 𝑑 with 0 ≤ 4 and (x, y, z) = (4, 2, 5).
𝑚2 +𝑑 𝑑
d≤ 2m, 𝑞(𝑛) = ⎣ 𝑚
⎦ = 𝑚 + ⎣𝑚⎦

316
Solving Mathematical Problems

74. The sides a, b, c and u, v, w of two triangles However, 𝑥 2 − 6𝑥 − 27 ≥ 0 𝑓𝑜𝑟 𝑥 ≤


ABC and UVW are related by the −3 𝑜𝑟 𝑥 ≥ 9, so (𝑎𝑏𝑐𝑑)1/2 ≥ 9, hence abcd ≥
equations. 81. We conclude abcd = 81, and hence a = b=
c= d= 3.
𝒖(𝒗 + 𝒘 − 𝒖) = 𝒂𝟐 ,

𝒗(𝒘 + 𝒖 − 𝒗) = 𝒃𝟐 ,
76. Prove that the average of numbers n
𝒘(𝒖 + 𝒗 − 𝒘) = 𝒄𝟐 . 𝐬𝐢𝐧 𝒏° (𝒏 = 𝟐, 𝟒, 𝟔, … , 𝟏𝟖𝟎)𝒊𝒔 𝐜𝐨𝐭 𝟏°.
Prove that ABC is acute, and express the angles
Solution: All arguments of trigonometric
U, V, W in terms of A, B, C.
functions will be in degrees. We need to prove
Sol.: Note that 𝑎2 + 𝑏 2 − 𝑐 2 = 𝑤 2 − 𝑢2 −
2 sin 2 + 4 sin 4 + ⋯ + 178 sin 178
𝑣 2 + 2𝑢𝑣 = (𝑤 + 𝑢 − 𝑣)(𝑤 − 𝑢 + 𝑣) > 0 by
= 90 cot 1 (2)
the triangle inequality, so cos 𝐶 > 0. By this
reasoning, all of the angles of triangle ABC are Which is equivalent to
acute. Moreover,
2 sin 2 sin 1 + 2(2 sin 4 . sin 1) + … + 89
2 2 2
𝑎 +𝑏 −𝑐
cos 𝐶 = (2 sin 178 . sin 1) = 90 cos 1. (3)
2𝑎𝑏
Using the identity 2 sin 𝑎 . sin 𝑏 = cos(𝑎 − 𝑏) −
(𝑤 + 𝑢 − 𝑣)(𝑤 − 𝑢 + 𝑣)
=√ cos(𝑎 + 𝑏), we find
4𝑢𝑣
2 sin 2 . sin 1 + 2(2 sin 4 . sin 1) + ⋯ + 89
𝑤 2 − 𝑢2 − 𝑣 2 + 2𝑢𝑣 1
=√ = √1 − cos 𝑈 (2 sin 178 . sin 1)
4𝑢𝑣 √2
= (cos 1 − cos 3) + 2(cos 3 − cos 5) + ⋯ +
2
From which we deduce U = 1 − 2𝑐𝑜𝑠 𝐴 = 89(cos 177 − cos 179)
cos(𝜋 − 2𝐴).
= cos 1 + cos 3 + cos 5 + ⋯ + cos 175
Therefore U = 𝜋 -2A, and similarly V = 𝜋- 2B,
W = 𝜋- 2C. cos 177 − 89 cos 179

75. Find all solutions in positive real numbers = cos 1 + (cos 3 + cos 177) + ⋯
a, b, c, d to the following system of + (cos 89 + cos 91)
equations: − 89 cos 179
a + b+ c + d = 12
= cos 1 + 89 cos 1 = 90 cos 1,
abcd = 27 +ab +ac +ad +bc +bd +cd.
So (1) is true.
Sol.: The first equation implies abcd = ≤ 81 by
the arithmetic geometric mean inequality, Note: An alternate solution involves complex
with equality holding for a = b = c = d = 3. numbers. One expresses sin n as
Again by AM-GM,

abcd ≥ 27 +6 (𝑎𝑏𝑐𝑑)1/2

317
Solving Mathematical Problems
𝜋𝑖𝑛 −𝜋𝑖𝑛
(𝑒 180 −𝑒 180 )
contains at least one addend 𝑎𝑘 𝑤𝑖𝑡ℎ 𝑘 ≥ 𝑖.
𝑎𝑛𝑑 𝑢𝑠𝑒𝑠 𝑡ℎ𝑒 𝑓𝑎𝑐𝑡 𝑡ℎ𝑎𝑡 Then since then 𝑎𝑘 ≥ 𝑎𝑖 , we have
(2𝑖)

𝑥 + 2𝑥 2 + ⋯ + 𝑛𝑥 𝑛 = (𝑥 + ⋯ + 𝑥 𝑛 ) + 𝑠𝑖 − 𝜎 < 𝑠𝑖 − 𝑠𝑖−1 = 𝑎𝑖 ≤ 𝑎𝑘 ≤ 𝜎, which


(𝑥 2 + ⋯ + 𝑥 𝑛 ) + ⋯ + 𝑥 𝑛 together with 𝜎≤ 𝑠𝑖 implies (2).

1 Note: The result does not hold if 2 is replaced


= 𝑥−1 [(𝑥 𝑛+1 − 𝑥) + (𝑥 𝑛+1 − 𝑥 2 ) + ⋯ +
by any smaller constant c. To see this, choose n
(𝑥 𝑛−1 − 𝑥 𝑛 )]
such that 𝑐 < 2 − 2−(𝑛−1) and consider the set
𝑛𝑥 𝑛+1 𝑥 𝑛+1 −𝑥 {1, … , 2𝑛−1 }. If this set is divided into n subsets,
= − (𝑥−1)2
.
𝑥−1 two of 1,……, 2𝑛−1 , 1 + …….+ 2𝑛−1 must lie in
the subset, and their ratio is at least (1+…..+
2𝑛−1 )/(2𝑛−1 ) = 2 − 2(𝑛−1) > c.
77. For any nonempty set S of real numbers,
let 𝜎(S) denote the sum of the elements of
S. Given a set A of n positive integers,
78. An n –term sequence {𝒙𝟏 , 𝒙𝟐 , … , 𝒙𝒏 } in
consider the collection of all distinct sums
which each term is either 0 or 1 is called a
𝜎(S) as S ranges over the nonempty
binary sequence of length n. Let an be the
subsets of A. Prove that this collection of
number of binary sequences of length n
sums can be partitioned into n classes so
containing no three consecutive terms
that in each classes, the ratio of the largest
equal to 0, 1, 0 in that order. Let 𝒃𝒏 be the
sum of the smallest sum does not exceed
number of binary sequences of length n
2.
that contain no four consecutive terms
Sol.: Let A = {𝑎1 , 𝑎2 , … , 𝑎𝑛 } where 𝑎1 < 𝑎2 < equal to 0, 0, 1, 1 or 1, 1, 0, 0 in that order.
⋯ < 𝑎𝑛 . For i = 1, 2, …, n let 𝑠𝑖 = 𝑎1 + 𝑎2 + Prove that 𝒃𝒏+𝟏 = 𝟐𝒂𝒏 for all positive
⋯ + 𝑎𝑖 𝑎𝑛𝑑 𝑡𝑎𝑘𝑒 𝑠0 = 0. All the sums is integers n.
question are less than or equal to 𝑠𝑛 , and if 𝜎 is
Sol.: We refer to the binary sequences counted
one of them, we have by (𝑎𝑛 ) and (𝑏𝑛 ) as “type A” and “type B”,
𝑠𝑖−1 < 𝜎 < 𝑠𝑖 (1) respectively. For each binary sequence
(𝑥1 , 𝑥2 , … , 𝑥𝑛 ) there is a corresponding binary
For an appropriate i. Divide the sums into n sequence (𝑦0 , 𝑦1 , … , 𝑦𝑛 ) obtained by setting
classes by letting 𝐶𝑖 denote the class of sums 𝑦0 = 0 𝑎𝑛𝑑 𝑦1 = 𝑥1 + 𝑥2 + ⋯ + 𝑥𝑖 mod 2, i =
satisfying (1). We claim that these classes have 1, 2, …, n. (2)
the desired property. To establish this, it
suffices to show that (1) implies. (Addition mod 2 is defined as follows: 0 +0 = 1
+1 = 0 and 0 + 1 = 1 +0 = 1.) Then
1
𝑠 < 𝜎 < 𝑠𝑖 (2)
2 𝑖 𝑥𝑖 = 𝑦𝑖 + 𝑦𝑖−1 𝑚𝑜𝑑 2, 𝑖 = 1, 2, … , 𝑛,

Suppose (1) holds. The inequality 𝑎1 + 𝑎2 + And it is easily seen that (1) provides a one-to –
⋯ + 𝑎𝑖−1 = 𝑠𝑖−1 < 𝜎 shows that the sum 𝜎 one correspondence between the set of all
binary sequences of length n and the set of

318
Solving Mathematical Problems

binary sequences of length n +1 in which the 2 2


1 2 2√2
first term is 0. Moreover, the binary sequence 𝑢= ( ± ) ,𝑣 = ( ± √2) .
√3 √21 √7
(𝑥1 , 𝑥2 , … , 𝑥𝑛 ) has three consecutive terms
equal 0, 1, 0 in that order if and only if the
corresponding sequence (𝑦0 , 𝑦1 , … , 𝑦𝑛 ) has four
80. Determine, as a function of n, the number
consecutive terms equal to 0, 0, 1, 1 or 1, 1, 0, 0
of permutations of the set
in that order, so the first is of type A if and only
{1, 2, …, n} such that no three of 1, 2, 3, 4
if the second is of type B. The set of Type B
appear consecutively.
sequences of length n +1 in which the first term
is 0 is exactly half the total number of such Sol.: There are n! permutations in all. Of those,
sequences, as can be seen by means of the we exclude (n -2)! Permutations for each
mapping in which 0’s and 1’s are interchanged. arrangement of 1, 2, 3, 4 into an ordered triple
and one remaining element, or 24(n -2)! in all.
However, we have twice excluded each of the
79. Solve the system of equations: 24(n-3)! Permutations in which all four of 1, 2,
𝟏 3, 4 occur in a block. Thus the number of
√𝟑𝒙 (𝟏 + 𝒙+𝒚) = 𝟐
𝟏
permutations of the desired from is n! -24 (n -
√𝟕𝒚 (𝟏 − 𝒙+𝒚) = 𝟒√𝟐 2)! + 24(n -3)!

Sol.: Let u =√𝑥, 𝑦 = √𝑦, so the system


becomes
81. Determine all function f: ℕ ⟶ℕ satisfying
𝑢 2 (for all n ∈ℕ)
𝑢+ =
𝑢2 +𝑣 2
√3
𝒇(𝒏) + 𝒇(𝒏 + 𝟏)
𝑣 4√2 = 𝒇(𝒏 + 𝟐)𝒇(𝒏 + 𝟑)
𝑣− = . − 𝟏𝟗𝟗𝟔.
𝑢2 +𝑣 2
√7

Now let z = u +vi; the system then reduces to Sol.: From the given equation, we deduce
the single equation 𝑓(𝑛) − 𝑓(𝑛 + 2)
= 𝑓(𝑛 + 3)[𝑓(𝑛 + 2)
1 1 2√2
𝑧+ = 2( + 𝑖). − 𝑓(𝑛 + 4)]
𝑧 √3 √7
If f(1) > f(3), then by induction, f(2m -1) > f(2m
Let t denote the quantity inside the
+1) for all m > 0, giving an infinite decreasing
parentheses; then
sequence f(1), f(3), …. Of positive integers, a
contradiction. Hence f(1) ≤ f(3), and similarly
𝑧 = 𝑡 ± √𝑡 2 − 1
f(n) ≤ f(n +2) for all n.
1 2√2 2
= + 𝑖 ±( + √21) Now note that
√3 √7 √21

From which we deduce

319
Solving Mathematical Problems

0 = 1996 + 𝑓(𝑛) + 𝑓(𝑛 + 1) − 𝑓(𝑛 3𝑠2 + 𝑠3 = 4 ⇒ 𝑠1 ≥ 𝑠2 .


+ 2)𝑓(𝑛 + 3)
≤ 1996 + 𝑓(𝑛 + 2) It suffices to prove the (unconstrained)
+ 𝑓(𝑛 + 3) homogeneous inequality
− 𝑓(𝑛 + 2)𝑓(𝑛 + 3)
3𝑠2 2 𝑠1 2 + 𝑠3 𝑠1 3 ≥ 4𝑠2 3 ,
= 1997 − [𝑓(𝑛 + 2) − 1][𝑓(𝑛 + 3) − 1].
As then 3𝑠2 + 𝑠3 = 4 will imply
In particular, either f(n +2) = 1 or f(n +3) ≤
(𝑠1 − 𝑠2 )3 + 3(𝑠1 3 − 𝑠2 3 ) ≥ 0.
1997, and vice versa. The numbers f(2m+1) –
f(2m-1) are either all zero or all positive, and We now recall two basic inequalities about
similarly for the numbers f(2m+ 2) –f(2m). If symmetric means of nonnegative real numbers.
they are both positive, eventually f(n +2) and The first is Schur’s inequality:
f(n+3) both exceed 1997, a contradiction.
3𝑠1 3 + 𝑠3 ≥ 4𝑠1 𝑠2 .
We now split into three cases. If f(2m) and f(2m
+1) are both constant, we have [f(2m) - While the second,
1][f(2m+1)-1] = 1997 and so either f(2m) = 1
𝑠1 2 ≥ 𝑠2
and f(2m +1) = 1997 or vice versa. If f(2m +1) is
constant but f(2m) is not, then 𝑓(2𝑚 + 1) = 1 Is a case of Maclaurin's inequality 𝑠𝑖 𝑖+1 ≥
for all m and 𝑓(2𝑚 + 2) = 𝑓(2𝑚) + 𝑠𝑖+1 𝑖 .These combine to prove the claim:
1997, 𝑠𝑜 𝑓(2𝑚) = 1997(𝑚 − 1) + 𝑓(2).
𝑠2 2 𝑠3
Similarly, if f(2m) is not constant, then f(2m)= 1 3𝑠2 2 𝑠1 2 + 𝑠3 𝑠1 3 ≥ 3𝑠2 2 𝑠1 2 + ≥ 4𝑠2 3 .
𝑠1
and f(2m +1) = 1997m + f(1).
Finally, for those who have only seen Schur’s
inequality in three variables, note that in
general any inequality involving 𝑠1 , … , 𝑠𝑘 which
82. Let a, b, c, d be four nonnegative real
holds for n ≥ k variables also holds for n +1
numbers satisfying the condition
variables, by replacing the variables 𝑥1 , … , 𝑥𝑛+1
𝟐(𝒂𝒃 + 𝒂𝒄 + 𝒂𝒅 + 𝒃𝒄 + 𝒃𝒅 + 𝒄𝒅)
by the roots of the derivative of the polynomial
+ 𝒂𝒃𝒄 + 𝒂𝒃𝒅 + 𝒂𝒄𝒅
(𝑥 − 𝑥1 ) … (𝑥 − 𝑥𝑛−1 ).
+ 𝒃𝒄𝒅 = 𝟏𝟔

Prove that

𝟐
𝒂+𝒃+𝒄+𝒅≥ (𝒂𝒃 + 𝒂𝒄 + 𝒂𝒅
𝟑 83. Let m and n be positive integers such that
+ 𝒃𝒄 + 𝒃𝒅 + 𝒄𝒅)
n ≤ m. Prove that
And determine when equality occurs. (𝒎 + 𝒏)!
𝟐𝒏 𝒏! ≤ ≤ (𝒎𝟐 + 𝒎)𝒏 .
(𝒎 − 𝒏)!
Sol.: For i = 1, 2, 3, define 𝑠𝑖 as the average of
the products of the i-element subsets of {a, b, c, Sol.: The quantity in the middle is (𝑚 + 𝑛)(𝑚 +
d}. Then we must show 𝑛 − 1) … (𝑚 − 𝑛 + 1). If we pair off terms of
the form (𝑚 + 𝑥)𝑎𝑛𝑑 (𝑚 + 1 − 𝑥), we get

320
Solving Mathematical Problems
𝒏
products which do not exceed 𝑚(𝑚 + 1), since (d) The number m = 𝟓 has k odd
the function 𝑓(𝑥) = (𝑚 + 𝑥)(𝑚 + 1 − 𝑥) is a (decimal) digits.
1
concave parabola with maximum at 𝑥 = . From
2
Sol.: The multiplication in each place must
this the right inequality follows. For the left, we
produce an even number of carries, since these
need only show (𝑚 + 𝑥)(𝑚 + 1 − 𝑥) ≥ 2𝑥 for
will be added to 5 in the next place and an odd
x ≤n; this rearranges to (𝑚 − 𝑥)(𝑚 + 1 + 𝑥) ≥
digit must result. Hence all of the digits of m
0, which holds because 𝑚 ≥ 𝑛 ≥ 𝑥.
must be 1, 5 or 9 and the first digit must be1,
since m and n have the same number of
decimal digits. Hence there are 3𝑘−1 choices for
84. Let a, b and c be the lengths of the sides of
m and hence for n.
a triangle. Prove that
√𝒂 + 𝒃 − 𝒄 + √𝒃 + 𝒄 − 𝒂
+ √𝒄 + 𝒂 − 𝒃
86. The polynomials 𝑷𝒏 (𝒙) are defined by
≤ √𝒂 + √𝒃 + √𝒄 𝑷𝟎 (𝒙) = 𝟎, 𝑷𝟏 (𝒙) = 𝒙 and 𝑷𝒏 (𝒙) =
And determine when equality occurs. 𝒙𝑷𝒏−𝟏 (𝒙) + (𝟏 − 𝒙)𝑷𝒏−𝟐 (𝒙) 𝒏 ≥ 𝟐.
For every natural number n ≥ 1, find
Sol.: By the triangle inequality, 𝑏 + 𝑐 − all real numbers x satisfying the
𝑎 𝑎𝑛𝑑 𝑐 + 𝑎 − 𝑏 𝑎𝑏𝑐 positive. For any positive equation 𝑷𝒏 (𝒙) = 𝟎.
x, y, we have
Sol.: One shows by induction that
2
2(𝑥 + 𝑦) ≥ 𝑥 + 𝑦 + 2√𝑥𝑦 = (√𝑥 + √𝑦) 𝑥
𝑃𝑛 (𝑥) = [(𝑥 − 1)𝑛 − 1]
𝑥−2
By the AM-GM inequality, with equality for x =
y. Substituting 𝑥 = 𝑎 + 𝑏 − 𝑐, 𝑦 = 𝑏 + 𝑐 − 𝑎 we Hence 𝑃𝑛 (𝑥) = 0 if and only if x = 0 or 𝑥 = 1 +
get 𝑒 2𝜋𝑖𝑘/𝑛 for some k ∈ {1, …, n -1}.

√𝑎 + 𝑏 − 𝑐 + √𝑏 + 𝑐 − 𝑎 ≤ 2√𝑎,

Which added to the two analogous inequalities 87. The real numbers x, y, z, t satisfy the
yields the desired result. Inequality holds for equalities 𝒙 + 𝒚 + 𝒛 + 𝒕 = 𝟎 𝒂𝒏𝒅 𝒙𝟐 +
𝑎 + 𝑏 − 𝑐 = 𝑏 + 𝑐 − 𝑎 = 𝑐 + 𝑎 − 𝑏, 𝑖. 𝑒. 𝑎 = 𝒚𝟐 + 𝒛𝟐 + 𝒕𝟐 = 𝟏. Prove that
𝑏 = 𝑐. −𝟏 ≤ 𝒙𝒚 + 𝒚𝒛 + 𝒛𝒕 + 𝒕𝒙 ≤ 𝟎.

Sol.: The inner expression is (𝑥 + 𝑧)(𝑦 + 𝑡) =


−(𝑥 + 𝑧)2 , so the second inequality is obvious.
85. Let k ≥1 be an integer. Show that there are
As for the
exactly 𝟑𝒌−𝟏 positive integers n with the
following properties: first, note that
(a) The decimal representation of n 1
consists of exactly k digits. 1 = (𝑥 2 + 𝑧 2 ) + (𝑦 2 + 𝑡 2 ) ≥ 2 [(𝑥 + 𝑧)2 +
(b) All digits of k are odd. (𝑦 + 𝑡)2 ] ≥ [(𝑥 + 𝑧)(𝑦 + 𝑡)]
(c) The number n is divisible by 5.

321
Solving Mathematical Problems

By two applications of the power mean If k +1 is not prime, it has a prime divisor
inequality. greater than 3, but this prime divides k! and not
k! +48. Hence k +1 is prime, and by Wilson’s
theorem k! +1 is a multiple of k +1. Since k! +48
88. Natural numbers k, n are given such that 1 is as well, we find k +1 = 47, and we need only
46!
< k < n. Solve the system of n equations. check that is not a power of 47. We check
48+1
𝒙𝒊 𝟑 (𝒙𝒊 𝟐 + ⋯ + 𝒙𝒊+𝒌−𝟏 𝟐 ) = 𝒙𝒊−𝟏 𝟐 𝟏 ≤ that
46!
= 29(𝑚𝑜𝑑 53) (by cancelling as many
48+1
𝒊≤𝒏 terms as possible in 46! Before multiplying), but
in n real unknowns 𝒙𝟏 , … , 𝒙𝒏 . (Note:
that 47 has order 13 modulo 53 and that none
𝒙𝟎 = 𝒙𝒏 , 𝒙𝟏 = 𝒙𝒏+𝟏 , 𝒆𝒕𝒄.)
of its powers is congruent to 29 modulo 53.
Sol.: The only solution is 𝑥1 = ⋯ 𝑥𝑛 = 𝑘 −1/3 .
Let L and M be the smallest and largest of the
𝑥𝑖 , respectively. If M = 𝑥𝑖 , then

𝑘𝑀3 𝐿2 ≤ 𝑥𝑖 3 (𝑥𝑖 2 + ⋯ + 𝑥𝑖+𝑘−1 2 ) = 𝑥𝑖−1 2 90. Let O and G be the circumcenter and
≤ 𝑀2 centroid, respectively, of triangle ABC, If R
is the circumradius and r the inradius of
1
And so M ≤ (𝑘𝐿2 ). Similarly, if L = 𝑥𝑗 , then ABC, show that
𝑶𝑮 ≤ √𝑹(𝑹 − 𝟐𝒓).
𝑘𝐿3 𝑀3 ≥ 𝑥1 3 (𝑥𝑖 2 + ⋯ + 𝑥𝑖−𝑘+1 2 ) = 𝑥𝑖−1 2
≥ 𝐿2 Sol.: Using vectors with original at O, we note
1 1
1 that 𝑂𝐺 2 = (𝐴 + 𝐵 + 𝐶)2 = 𝑅 2 +
and so 𝐿 ≥ (𝑘𝑀2 ). Putting this together, we get 9 3
2 2
9
𝑅 (cos 2𝐴 + cos 2𝐵 + cos 2𝐶).
1
𝐿≥ ≥ 𝑘𝐿4
𝑘𝑀2 (𝑎 2 +𝑏2 +𝑐 2 )
Hence 𝑅 2 − 𝑂𝐺 2 = 9
. On the other
And so 𝐿 ≥ 𝑘 −1/3 ; 𝑠𝑖𝑚𝑖𝑙𝑎𝑟𝑙𝑦, 𝑀 ≥ 𝑘 −1/3 . hand, by the standard area formula K = 𝑟𝑠 =
Obviously L ≤ M, so we have L = M = 𝑎𝑏𝑐
,
𝑎𝑏𝑐
we have 2𝑟𝑅 = (𝑎+𝑏+𝑐). We now note that
4𝑅
𝑘 −1/3 𝑎𝑛𝑑 𝑥1 = ⋯ = 𝑥𝑛 = 𝑘 −1/3 .
(𝑎2 + 𝑏 2 + 𝑐 2 )(𝑎 + 𝑏 + 𝑐) ≥ 9𝑎𝑏𝑐

By two applications of the AM-GM inequality,


89. Shows that there do not exist nonnegative
so 2𝑟𝑅 ≤ 𝑅 2 − 𝑂𝐺 2 , proving the claim.
integers k and m such that k! +48 = 48
(𝒌 + 𝟏)𝒎 .

Sol.: Suppose such k, m exist. We must have


48│k!, so k ≥ 6; one checks that k = 6 does not
yield a solution, so k ≥ 7. In that case k! is 91. Show that there exists a subset A of the set
(𝑘!+48) {1, 2, …, 1996} having the following
divisible by 32 and by 9, so that 48
is
properties:
relatively prime to 6, as then is k +1.
(a) 1, 𝟐𝟏𝟗𝟗𝟔 − 𝟏 ∈ 𝑨;

322
Solving Mathematical Problems

(b) Every element of A, except 1, is the suppose that p is composite. If p ≡ 0, 2, 3 or 4


sum of two (not necessarily (mod 6), then 𝑁1 𝑎𝑛𝑑 𝑁2 are not integers.
distinct) elements of A;
Otherwise, all divisors of p are congruent to ±1
(c) A contains at most 2012 elements.
(mod 6). So there exist natural numbers c, d
Sol.: We state the problem a bit differently: we such that
want to write down at most 2012 numbers,
starting with 1 and ending with 21996 − 1, such 𝑝 = (6𝑐 + 1)(6𝑑 + 1)𝑜𝑟(6𝑐 − 1)(6𝑑
− 1)𝑜𝑟(6𝑐 + 1)(6𝑑 − 1).
that every number written is the sum of two
numbers previously written. If 2𝑛 − 1 has been In the first case, 𝑁2 is not an integer and 𝑁1 = 0
written, then 2𝑛 (2𝑛−1 ) can be obtained by n for 𝑎 = −𝑐, 𝑏 = −𝑑.
doublings, and 2𝑛 − 1 can be obtained in one
more step. In the second case, 𝑁2 is not an integer and
𝑁1 = 0 for 𝑎 = 𝑐, 𝑏 = 𝑑.
Hence we can obtain 22 − 1, 24 − 1, … , 2256 −
1 𝑖𝑛 (1 + 1) + (2 + 1) + ⋯ + (128 + 1) = In the third case, 𝑁1 is not an integer and 𝑁2 = 0
263 steps. In 243 steps, we turn 2256 − for 𝑎 = 𝑐, 𝑏 = −𝑑.
1 𝑖𝑛𝑡𝑜 2499 − 2243 . Now notice that the
93. Let M be a nonempty set and *a binary
numbers 2243 − 2115 , 2115 − 251 , 251 −
operation on M. That is, to each pair (a, b)
219 , 219 − 23 , 23 − 21 , 21 − 1 have all be
∈𝑴 × 𝑴 one assigns an element a * b.
written down; in 6 steps, we now obtain 2499 −
Suppose further that for any a, b ∈M,
1. We make this into 2998 − 1 in 500 steps, and (a* b)*b = a and a*(a* b) = b.
make 21996 − 1 in 999 steps. Adding 1 for the (a) Show that a *b = b *a for all a, b
initial 1, we count ∈M.
(b) For which finite sets M does such a
1 + 263 + 243 + 6 + 500 + 999 = 2012
binary operation exist?
Numbers written down, as desired.
Sol.:

(a) First note that [a* (a* b)] *(a* b) = a by


92. Let ℤ+ denote the set of nonzero integers. the first rule. By the second rule, we
Show that an integer p > 3 is prime if an may rewrite the left side as b* (a *b), so
only if for any a, b ∈ ℤ+ , exactly one of the b* (a *b) = a and so b *a = b*[b* (a* b)].
numbers b* a = b*[b*(a* b)]. By the second rule
𝒑−𝟏 this equals a *b, so a* b = b* a.
𝑵𝟏 = 𝒂 + 𝒃 − 𝟔𝒂𝒃 + ,
𝟔 (b) Such sets exist for all finite sets M.
𝒑+𝟏 Identify M with {1, …, n} and define
𝑵𝟐 = 𝒂 + 𝒃 + 𝟔𝒂𝒃 +
𝟔 a* b = c ⟺a + b+ c = 0 (mod n).
belongs to ℤ+ .
It is immediate that the axioms are
Sol.: If 𝑁1 = 0, then 𝑝 = (6𝑎 − 1)(6𝑏 − 1) is satisfied.
composite; similarly, 𝑁2 = 0 𝑖𝑚𝑝𝑙𝑖𝑒𝑠 𝑝 =
−(6𝑎 + 1)(6𝑏 + 1) is composite. Conversely, 94. Determine whether there exist a function
f: ℤ⟶ℤ such that for each k = 0, 1, …, 1996

323
Solving Mathematical Problems

and for each m ∈ℤ the equation 𝒇(𝒙) + whose side lengths are also natural
𝒃𝒙 = 𝒎 has at least one solution x ∈ℤ. numbers. Determine the smallest possible
value of n.
Sol.: Each integer y can written uniquely as
1997m +k with m ∈ℤ and k ∈ {0, …, 1996}. Sol.: Since 1996 >123 , we must have n ≥ 13,
Define the function f by 𝑓(𝑦) = 𝑚 − 𝑘𝑦; then and we now show n = 13 suffices, Inside a cube
𝑓(𝑥) + 𝑘𝑥 = 𝑚 has the solution 𝑥 = 1997𝑚 + of edge 13, we place one cube of edge 5, one
𝑘, so the condition satisfied. cube length 4, and 2 of length 2, and fill the
remainder with cubes of edge 1. The number of
cubes used is
95. Two sets of intervals A, B on a line are
133 − (53 − 1) − (43 − 1) − 2(23 − 1) =
given. The set A contains 𝟐𝒎 − 𝟏 intervals, 2197 − 124 − 63 − 2(7) = 1996, as desired.
every two of which have a common
interior point. Moreover, each interval in A
contains at least disjoint intervals of B.
Show that there exists an interval in B 97. Find all positive integers n such that
which belongs to at least m intervals from 𝟑𝒏−𝟏 + 𝟓𝒏−𝟏 𝒅𝒊𝒗𝒊𝒅𝒆𝒔 𝟑𝒏 + 𝟓𝒏 .
A. Sol.: This only occurs for n = 1. Let 𝑠𝑛 = 3𝑛 + 5𝑛
Sol.: Let 𝛼1 = [𝑎𝑖 , 𝑏𝑖 ](𝑖 = 1, … , 2𝑚 − 1) be the and note that
intervals, indexed so that 𝑎1 ≤ 𝑎2 ≤ ⋯ ≤ 𝑠𝑛 = (3 + 5)𝑠𝑛−1 − 3.5. 𝑠𝑛−2
𝑎2𝑚−1 . Choose k ∈{𝑚, … , 2𝑚 − 1} to minimize
𝑏𝑘 . By assumption, the interval 𝛼𝑘 contains two So 𝑠𝑛−1 must also divide 3.5. 𝑠𝑛−2 . If n >1, then
disjoint intervals from B, say 𝛽1 = 𝑠𝑛−1 is coprime to 3 and 5, so 𝑠𝑛−1 must divide
[𝑐1 , 𝑑1 ] 𝑎𝑛𝑑 𝛽2 = [𝑐2 , 𝑑2 ]. Without loss of 𝑠𝑛−2, which is impossible since 𝑠𝑛−1 > 𝑠𝑛−2 .
generality, assume

𝑎𝑘 ≤ 𝑐1 < 𝑑1 < 𝑐2 < 𝑑2 ≤ 𝑏𝑘 .


98. Prove that for every polynomial 𝒙𝟐 + 𝒑𝒙 +
If 𝑑1 ≤ 𝑏𝑖 for i = 1, 2, …, m, then 𝛽1 ⊂ 𝛼1 for i = 𝒒 with integer coefficients, there exists a
1, 2, …, m, so 𝛽1 satisfies the desired property. polynomial 𝟐𝒙𝟐 + 𝒓𝒙 + 𝒔 with integer
Otherwise, 𝑑1 > 𝑏𝑥 for some s ∈ {1, 2, …, m}. By coefficients such that the sets of values of
assumption, 𝑐2 > 𝑑1 > 𝑏8 . Since no two of the the two polynomials on the integers are
𝛼 are disjoint, we have 𝑏8 ≥ 𝑎𝑖 for all i, so 𝑐2 > disjoint.
𝑎𝑖 . On the other hand, by the choice of 𝑘, 𝑏𝑘 <
𝑏1 for i = m, …, 2𝑚1 . Therefore 𝑎𝑖 < 𝑐2 < 𝑑2 ≤ Sol.: If p is odd, then 𝑥 2 + 𝑝𝑥 + 𝑞 has the same
𝑏𝑘 ≤ 𝑏𝑖 for each 𝑖 ∈ {𝑚, 𝑚 + 1, … , 2𝑚 − parity as q for all integers x, and it suffices to
1}, 𝑎𝑛𝑑 𝑠𝑜 𝛽2 has the desired property. choose r even and s of the opposite parity as q.
If p = 2m is even, then 𝑥 2 + 𝑝𝑥 + 𝑞 = (𝑥 +
𝑚)2 + (𝑞 − 𝑚)2 which is congruent to 𝑞 −
𝑚2 𝑜𝑟 𝑞 − 𝑚2 + 1 modulo 4. Now it suffices to
96. Let n be a natural number. A cube of side
choose r even and s congruent to 𝑞 − 𝑚2 + 2
length n can be divided into 1996 cubes
modulo 4.

324
Solving Mathematical Problems
𝑛−𝑖
5 1
∏ (1 − 𝑖−1 ).
99. Sergey found 11 different solutions to the 8 2
𝑖=6
𝟗𝟔
equation 𝒇 (𝟏𝟗𝒙 − ) = 𝟎. Prove that if We show this by induction on a good
𝒙
he had tried harder, he could have found assignment or r h -1 vertices can be extended to
at least one more solution. a good assignment on vertices simply by
96 avoiding having all edges from the last vertex
Sol.: The equations 19𝑥 − 𝑥
= 𝑡 can be pointing in the same direction, which occurs in
rewritten 19𝑥 2 − 𝑡𝑥 − 96 = 0; 𝑠𝑖𝑛𝑐𝑒 𝑡 2 + 2 cases out of 2𝑛−1 .
19.96 > 0, it always has two real roots.
Therefore the number of zeroes of f (if finite) is Now it suffices to show that the above
1
an even integer, so Sergey can find at least one expression is more than 2.
more zero.
In fact,
∞ ∞
1 −1 𝑖−4
100. There are 2000 towns in a country, ∏ (1 − ) ≤ 1 + ∑ 𝑖
2 2
each pair of which is linked by a road. The 𝑖=5 𝑖=5

Ministry of Reconstruction proposed all of ∞


1 𝑖+1
the possible assignments of one way traffic = 1+ 5∑ 𝑖
2 2
to each road. The ministry of 𝑖=0

Transportation rejected each assignment ∞ ∞


1 1
that did not allow travel from any town to = 1 + 5∑∑ 𝑖
2 2
any other town. Prove that more of half of 𝑖=0 𝑘=𝑖
the assignments remained. ∞
1 1
= 1+ 5∑ 𝑖
Sol.: We will prove the same statement for n ≥ 2 2 −1
𝑖=0
6 towns. First suppose n = 6. In this case there
are 215 assignments, and an assignment is 4 9
=1+ 5
=
rejected only if either one town has road to all 2 8
of the others in the same direction, or if there Thus the fraction of good assignments is at least
are two sets of three towns, such that within 5 8 5 1
(8 ) ( 9 ) = 9 > 2 .
each town the roads point in a circle, but all of
the roads from one set to the other point in the
same direction. There are 5.211 had
assignments of the first kind and 20.8 of the
second kind, so the fraction of good
5
assignments is at least 8.

For n ≥ 6, we claim that the fraction of good


assignments is at least

325

You might also like